.
.
Math Central - mathcentral.uregina.ca
Quandaries & Queries
Q & Q
. .
topic card  

Topic:

f

list of
topics
. .
start over

4251 items are filed under this topic.
 
Page
1/1
Foiled by foil 2009-07-20
From RAY:
how u do the foil ?

the question (x+4)(x+5)

Answered by Robert Dawson.
16 golfers 2009-03-18
From Bill:
We have 16 golfers. We golf for 5 days, in foursomes. It should work out that each player plays with each other player 1 time exactly.
I just can't figure it out.

Thank you in advance!

Bill

Answered by Claude Tardif and Victoria West.
How to convert fractions to percentages 2007-07-05
From Anita:
How do I turn 1 3/20 to a percent?
Answered by Stephen La Rocque and Melanie Tyrer.
Finding the area of a regular polygon 2007-05-04
From Dana:
We are trying to figure the square footage of a tetradecagon....sort of a round house with 14 sides that are 8 ft' in length. It has a height of 9 ft. How do we figure the square footage of this?
Answered by Stephen La Rocque.
Strings of letters 2022-04-25
From Javar:
How many different combinations can I make with A 12 selection list but only using A&B

Example:

A,B,A,B,A,B,A,B,A,B,A,B
A,B,B,A,A,B,B,A,A,B,B,A
B,B,A,A,B,A,B,A,B,A,A,B

Answered by penny Nom.
The next rational number 2022-01-14
From Sophia:
what comes after 3 1/2?
Answered by Penny Nom.
Roll of film 2021-11-20
From Brian:
Trying to create a formula to calculate the build up on a roll of film is calculated. Except instead of of a roll, it's essentially a rectangle with two semicircles on each at the end. Example: I start with a 6" diameter core on each side, and I start winding .005" thick film on the core. The distance between the center of those cores, length of the inner rectangle is 7.125. If I continue to do this for a total length of film of 3000 feet, what will the roll length and width be. So what I need is the formula to perform this type of calculation. Can you help me? I want to be able to plug the formula in a spread sheet and to be able to input a core diameter, a film thickness and a total length and the length between the center of the two cores(which is the inner rectangle's length I'm assuming). Thanks
Answered by Harley Weston.
The sides of a 5-pointed star inscribed in a circle 2021-09-18
From Scott:
Is there a calculation for the length of each side of a five-pointed star? I am not worried about area but if I have a circle with a 7.5 foot radius and place a star inside it, how can I measure the lines creating the star?
Answered by Harley Weston.
30 % off 2021-09-07
From Corrine:
When I am shopping for clothing that is for sale, the advertisement may say the item is 30% off. The original price was $70. How much would it now be? (how do I figure that out?) Thank you!
Answered by Penny Nom.
The units digit of a square 2021-09-04
From Aarzu:
Sorry this may be a dumb question but how do you find possible unit digits of a perfect square
Answered by Penny Nom.
Fibonacci numbers 2021-08-25
From John:
Make a single column of numbers. Start with two numbers of your choice. The third number is the sum of the previous two, the fourth number is the sum of numbers two and three, and so on until you have ten numbers in the column. Add up all ten numbers. Now, take the seventh number and multiply it by eleven. This product will equal the sum of the ten numbers. The same result will occur regardless of the first two numbers chosen. The question is why does the 7th number multiplied by 11 always equal the sum of the ten numbers?
Answered by Penny Nom.
The diameter of an octagon 2021-05-22
From Sue:
What is the diameter of an octagon with 8 foot sides
Answered by Penny Nom.
The height of a cone 2021-05-13
From Carmen:
If a cone has a slant height of 25 inches and a radius of 7 inches what is the height in inches of the cone?
Answered by Harley Weston.
More on a roll of paper 2021-05-07
From Stephen:
Can you help with the equations to calculate the length of paper required to achieve a target outside diameter when wrapped around a core please?

The inside diameter of the cardboard core is 76mm
The thickness of the wall of the cardboard core is 5mm
The thickness of the paper is 138microns
The desired outside diameter of the finished roll is 320mm

I hope you can help me with a solution, Thank you

Answered by Harley Weston.
The maximum area of a garden 2021-04-28
From Lexie:
suppose you want to make a rectangular garden with the perimeter of 24 meters. What's the greatest the area could be and what are the dimensions?
Answered by Penny Nom.
The volume of a spherical bowl 2021-04-23
From Danny:
How many gallons of water in a glass bowl with the dimensions of 10 inches high and 13.5 inches wide?
Answered by Penny Nom.
Scale drawing 2021-02-10
From Elias:
a football field is 160m long and 120m wide . Find the distance between its opposite corner flag posts . use the scale of 1cm:20m
Answered by Penny Nom.
Division in different bases 2021-02-06
From Promise:
Simplify the following 302 in base6 divided by 5 in base6
Answered by Penny Nom.
Simplify 2021-02-05
From Leslie:
To simplify improper fractions, do you convert it to a mixed number or leave as an improper fraction?

My daughter had a quiz in which she was supposed to simplify her answers. So, for example, on one question the answer she got was 11/10 and on another question she came up with 17/12. The teacher marked these as incorrect because they were mixed numbers. Her teacher wrote on her paper, "**name**, I even gave this back to you and told you to go through and simplify your answers!" We (her father and I) feel that the teacher wanted her to convert her answers, not simplify them but I haven't been able to find a definitive answer online. So, does simplifying improper fractions mean converting them to mixed numbers? Thank you for your help!


Answered by Penny Nom.
A scale factor 2021-02-02
From Layla:
Beth planned to travel from Battery Park to Times Square. The distance between the two places is 2.48 inches on the map of New York City. If the scale factor used for the map was 1 mile = 0.4 inch, find the actual distance between the two tourist attractions.
Answered by Penny Nom.
A 2.5 acre square 2021-01-09
From hannah:
how many feet of fence needed to fence a square 2.5 acres??
Answered by Penny Nom.
The distance between opposite vertices in an octagon 2020-11-14
From joe:
If the dimension from one flat part of an octagon to opposite flat is 500 mm what is the largest diameter (point to point) of the octagon?
Answered by Harley Weston.
Golf for 28 golfers over 4 days 2020-11-14
From David:
hi, I was wondering if you could help me with a golf draw, I have 28 players and wish to play a four round foursomes over 4 days
I would like to minimize the amount of games that they play together.

Answered by Harley Weston.
Equivalent fractions 2020-10-31
From Lexi:
Which of the following fractions is equivalent to -84 / -90?
A. -14 / 15
B. -42 / 45
C. 14 / 15
D. 42 / 45

Answered by Penny Nom.
An analytic proof that a quadrilateral is a parallelogram 2020-10-26
From Apollo:
Prove analytically that if ABCD is a parallelogram in which points P and Q trisects the diagonal AC, then BPDQ is a parallelogram.
Answered by Penny Nom.
Two equations with fractions 2020-10-22
From mia:
I am supposed to look for x and y. my teacher didn't explain very well.
please help : ((
2x - y = 5
x/4 + y/3= 2

Answered by Penny Nom.
A confusing problem 2020-10-01
From Kgaugelo:
a triangle has the vertices A(-4;-3),B(x;y)and C(2;1). Determine the coordinates of B if the equation of the line BC is y=5x-9
Answered by Harley Weston.
Factoring an interest formula 2020-09-29
From Kenneth:
Hello

P + Pr has a common factor of P so it can be expressed as P(1 + r) after the P is factored out.. How does the "P" get in front of (1 + r)?

P/P + Pr/P = 1 + r What step is used to show that P is added in front of (1 + r)?

I thank you for your reply.

Answered by Harley Weston.
The height of a tree 2020-09-27
From Kerissa:
A man height 1.2 m standing on top of a building 34.6m high views a tree some distance away he observes that the angle of depression of the base of the tree is 35 and the angle of depression of the top of the tree is 29, assuming that the tree stands of level ground. Determine the height of the tree.
Answered by Penny Nom.
my prcentage of years on earth is 2020-09-21
From Bill:
This Tuesday is my birthday and I will be 69. and I just wanted to know life ear is say earth is 6 billion years old I'm just curious to what percentage does my existence on earth reptresent I do realize that it's a flash in the pan
Answered by Penny Nom.
The surface area of a dome 2020-09-13
From Sue:
i need to know the outside square footage of a dome house 24ft across at the bottom and 16ft high so i know how much paint it will take to cover it.
Answered by Harley Weston.
The angular speed of a wheel 2020-09-03
From Catrina:
A car is moving at a rate of 75 miles per hour, and the diameter of its wheels is 2.6 in. Find the angular speed of the wheels in radians per minute.
Answered by Penny Nom.
A diameter of a circle 2020-08-19
From Apple:
One end of the the diameter of the circle with equation x^2+y^2-4x+4y=2 is (3,1). Find the coordinates of the other end of the diameter.
Answered by Penny Nom.
More on the last 2 digits of a^k = N 2020-08-07
From Jay:
I've seen solution methods for finding the last 2 digits of a^k = N, where a and k are known. My question is:

What if the number N is known and the last 2 digits are, say, ...62. Is there a way to find, for any given a, the value of k which will result in a number where the last 2 digits are ...62? and if that is possible, then do the same for a number with the last n digits (say 7 or 53 or something)?

Answered by Harley Weston.
Filling holes with stone dust 2020-07-30
From Zach:
I’m looking for amount of stone dust needed for thirteen holes with a 3ft depth and 12in diameter with a 4x4 post. I would need the measurement in KG.
Answered by Harley Weston.
A tank, 5/9 full of water 2020-07-28
From Nat:
A rectangular tank contains 1200cmsquare of water when it is 5 out of 9 full . Its base area is 80cmsquare . Find the height of the tank.
Answered by Penny Nom.
Choosing 4 balls from 9 2020-07-22
From sandiso:
What is the probability associated with each possible random sample of size 4 from the population consisting of these nine balls
Answered by Penny Nom.
4sin(2x)cos(2x)+1=0 2020-07-02
From Sheila:
4sin(2x)cos(2x)+1=0
Answered by Penny Nom.
Some strings of 4 digits 2020-06-26
From Judy:
I wrote previously and need more help. I need to generate a list of 4 digit. Numbers using 0-9 with no repetitions
I know that there is a 6 followed by a 2. Order does matter
Thank you in advance for your help.

Answered by Penny Nom.
Scale factor 2020-06-26
From Kinza:
Hi, my question is The perimeter of a triangle is reduced from 75cm to 20cm. What is the linear scale factor?
Answered by Penny Nom.
A word problem with fractions 2020-06-21
From Sthantsa:
I'm stuck with this question, I don't know where to begin because i really don't Know where to start.

Jessie and Thulani each has a sum of money. Jessie’s amount is 2/5 that of Thulani’s. If Thulani were to give Jessie R198, then his remaining amount will be 6/8 that of Jessie’s amount.
How much does Jessie have originally??

Answered by Penny Nom.
1 square cm is 100 square mm 2020-06-19
From macie:
explain why the conversion factor for cm squared to mm squared is 100 ,and not 10
Answered by Penny Nom.
390 as a percentage of 800 2020-06-18
From Yug:
What percentage is 390 grams of 800 grams?
Answered by Penny Nom.
Percentage profit 2020-05-30
From jaheem:
A shopkeeper buys an article for 80cents and sold it for $1. Calculate the percentage profit.
Answered by Penny Nom.
10 of 24 hours as a percentage 2020-05-27
From Larzy:
what is 10 of 24 hours in percentages
Answered by Harley Weston.
A triangular garden 2020-05-24
From yoonji:
the 3 sides of a triangular garden measure 200 ft., 250 ft., and 300 ft., respectively. the garden is to be divided by a line bisecting the longest side and drawn from the opposite vertex. what is the length of the bisecting line?
Answered by Penny Nom.
Drawing a counter from 2 bags 2020-05-21
From Sephy:
Question 4: Two bags, 1 and 2, each contain equal size counters.
Bag 1 contains a pink counter, yellow counter and white counter. Bag 2 contains counters labelled 1, 4, 5 and 7.
A counter is drawn at random from bag 1 and a counter is drawn at random from bag 2.
If the counter from bag 1 is pink, the number on the counter from bag 2 is increased by 1
If the counter from bag 1 is yellow, the number on the counter from bag 2 is decreased by 5
If the counter from bag 1 is white, the number is halved.
Find the probability of scoring a number below 3

Answered by Penny Nom.
Forming the largest cylinder 2020-05-20
From Amanda:
How do I find the maximum surface area and volume of a cylinder made up of ONE 8.5x11 piece of paper?
Answered by Penny Nom.
Maximizing the volume of a cone 2020-05-18
From Ella:
Hello, this is question - 'If you take a circle with a radius of 42cm and cut a sector from it, the remaining shape can be curled around to form a cone. Find the sector angle that produces the maximum volume for the cone made from your circle.'
Answered by Penny Nom.
An angle i a triangle 2020-05-16
From Ogunjobi:
Two goal post are 8m apart a footballer is 34 m from one post and 38m from the other within what angle must he kick the ball if he is to score
Answered by Penny Nom.
Three numbers have a sum of 45 2020-05-06
From Jayden:
Three numbers have a sum of 45. The greatest of the 3 numbers is 2 greater than the least number. What are the numbers? *
Answered by Penny Nom.
The flight of a football 2020-05-06
From Michael:
Football is thrown from a 10-yard line. It reaches its highest height of 20 yards. It lands on the 50-yard line after 2 seconds. What is the equation of the parabola that models this throw? I really need help as I've been on this for the longest amount of time.
Answered by Penny Nom.
Signs at 1/3 mile intervals 2020-05-03
From Heather:
The butterfly garden in St. Thomas gardens is a three mile loop at every 1/3 there is a sign along the loop to mark each 3rd of a mile along the loop if Melanie is standing next to the 7th sign at what mile is she?
Answered by Penny Nom.
The volume of a stone 2020-05-02
From KHOSI:
A rectangular container is 9cm wide,17cm long and contains water to a depth of 11cm. A stone is placed in the water and the water rises 2,2cm .Find the volume of the stone
Answered by Penny Nom.
The volume under an an uneven surface 2020-05-01
From Solon:
I am trying to fill an area of floor which has an uneven surface. I need to know the volume of fill. If you imagine a rectangular room (4 corners) that measures 189 inches by 240 inches and the depth at each corner is different.
Let us say corner #1 has a depth of 1", corner #2 has a depth of 0", corner #3 has a depth of 2" and corner #4 has a depth of 4".
How would i find the volume of said floor area?

Any assistance would be greatly appreciated!

Answered by Harley Weston.
A system of equations. 2020-04-27
From solomon:
xy + x =28
x + y +4

Answered by Penny Nom.
The width of a rectangle of land 2020-04-26
From Teresa:
If a rectangle shape piece of land is 813 feet long. How many feet width would be 2 acres?
Answered by Penny Nom.
Gravel for a French Drain 2020-04-17
From Ray:
Hello,

** I am wanting to add 2 French Drain's in my yard. The 1st drain will be 25 ft. long, 8 in wide and 18 in deep. The pipe I'm using is 4in. in diameter. I'm trying to figure out how many yards of clean gravel I will need for this project?

** The 2nd French Drain will be 100 ft in Length, 10in wide and 18 in deep. I will also be using the 4in in diameter pipe for this drain. I am needing to see how many yards of clean gravel, I'll need for this drain? Any assistance would be greatly appreciated!! Thank you!!

Answered by Harley Weston.
Two chords 2020-04-14
From Frank:
How to find the length of the radius of a circle if you know the lengths of two chords
Answered by Penny Nom.
A hexagon constructed from two triangles 2020-04-13
From sunny:
Triangles AEC and FDB are equilateral triangles and EA= DF= 12cm. The polygon at the center of the star is a regular hexagon. What is the area of the hexagon?
Answered by Penny Nom.
A pond in a garden 2020-04-09
From Jin:
c) I have a large square pond set inside a square garden: both the pond and the garden have sides which are a whole number of metres, and outside the pond, the garden is grassed over. The area covered by grass is 188 square metres. Find the area of the pond. (5 marks)
Answered by Penny Nom.
An arithmetic sequence 2020-04-04
From ally:
-10 ... last term 35 common difference is 3
(a) find the number of term in the sequence.
i have lost the formula.

Answered by Penny Nom.
A stem-and-leaf plot with decimals 2020-04-02
From Renee:
I am an AP Statistics student who is reviewing for the exam.

On one of my review worksheets, we have to create a stem-and-leaf plot for values with 2 decimal places (i.e. 9.14, 8.14, 8.74, 8.77, 9.26, 8.1, 6.13, 3.1, 9.13, 7.26, and 4.74).

How would I make the stem-and-leaf plot for this? I've already searched your database and nothing popped up.

If you could help me, that would be great. Thank you!

Answered by Penny Nom.
A circle in a square 2020-04-02
From Sabra:
Hello, I have twins in the 7th grade that are learning how to find circumference of circles. I am struggling with figuring out how to help my children to find the answer to this question:

Tye has a square piece of yellow felt that has an area of 81 square inches. She wants to cut the largest circle possible from the material to create a sun for her art project. What is the area of the felt circle? Use 3.14 for pi. Round to the nearest hundredth if necessary.

I wasn't sure if I am to find the area of the circle through finding the area of the square or what steps we needed to follow to find the answer. Once I can see how one is worked out step-by-step, I usually can help them along with questions that are similar.

Answered by Penny Nom.
Four digit combinations from 1-3-4-8-0 2020-03-26
From nancy:
Good evening. I'm in a quandry. Can you help? I would like to know what 4-digit combinations can be derived from 1-3-4-8-0 no repetitions
Answered by Penny Nom.
The center and radius of a circle 2020-03-25
From Riley:
so it says find the center and the radius of the below circle
x^2+(y-6)^2=121

i need help learning how to solve this

Answered by Penny Nom.
Is there an equation to find what percent of 0 is 1? 2020-03-09
From Asher:
Question: Is there an equation to find what percent of 0 is 1.

I remember learning quite a long time ago that the answer inst 0 and it isn't infinity. I'm pretty sure it was something like %=0 approaching infinity or %=1 approaching infinity. And I know it depends what value you assign the numbers i.e. dollars or temperature. Furthermore, is asking "what percent 1 dollar is of 0 dollars" the same question as "what percent profit do you make from selling something worth 0 dollars for 1 dollar."

Thank you for your time and consideration -Asher

Answered by Penny Nom.
The volume of a berm 2020-02-28
From jason:
how much dirt is in an 20'w 12't 1350'l berm
Answered by Penny Nom.
A straight line graph 2020-02-26
From Penehafo:
I have a problem of drawing a straight line graph by the equation y=2x-1
Answered by Penny Nom.
The height of an isosceles triangle 2020-02-23
From Reagan:
I need to find the height of an isosceles triangle with a base of 6 and sides of 4 units. How do I find it?
Answered by Penny Nom.
30% profit 2020-02-20
From Sidra:
A car dealer bought a car for £7500 and sold it for 30% profit.

For how much did he sell the car?

Answered by Penny Nom.
The volume of a cylinder 2020-02-12
From Mack:
I need to find how close the actual volume was of a cylinder (with both ends closed) to being optimized. Can you please send step by step response?

Height = 11cm
Circumference = 24.4cm

Thanks,
Mack

Answered by Penny Nom.
Sipping juice from a Tetra Pak 2020-02-11
From Anjhelic:
Karen is sipping juice from a 1 in. by 3 in. by 6 in tetra pack at the rate of 0.5in³/sec. How fast is the height of juice in the pack decreasing?
Answered by Harley Weston.
The area of a triangle from two angles and a side 2020-02-10
From Chinmoy:
How to measure the area of a triangle with two angles and length of the included side known?
Answered by Harley Weston.
A system of inequalities 2020-02-01
From Isaac:
Austin has x nickels and y dimes, having a maximum of 15 coins worth a minimum of $1 combined. No more than 4 of the coins are nickels and no less than 11 of the coins are dimes. Solve this system of inequalities graphically and determine one possible solution.
Answered by Penny Nom.
A question about fractions 2020-01-26
From Awotile:
A fraction whose denominator is more than the numerator is double when the numerator is increased by 6 and the denominator is increased by 5 find the original fraction?
Answered by Penny Nom.
The volume of a coffee cup 2020-01-25
From Amirah:
What is the volume of a polystyrene cup with a height of 8cm, diameter of 5cm at the top and 4cm at the bottom when it is filled with water?
Answered by Penny Nom.
Filling a 48 cubic meter tank 2020-01-19
From kamohelo:
An empty tank for storing water from a borehole has a volume of 480m3 if it is filled by a pump that pump water at a rate of 16L/s how many hours will it take the pump to fill this tank
Answered by Penny Nom.
What is the smallest 4 digit number? 2020-01-10
From Ullas:
What is the smallest 4 digit number?
Answered by Penny Nom.
The difference between two number is 7 2020-01-07
From Jaypercey:
the difference between two number is 7 find the two number if the larger 3times the smaller
Answered by Penny Nom.
The equation of a line 2020-01-05
From coco:
Good afternoon If relationship between total cost and the number of units made it linear ,and if costs increases by 7$ for each additional unit made ,and if the total cost of 10 units is 180$ .
Find the equation of the relationship b/n total cost (Y) & number of unit made (x).

Answered by Penny Nom.
Dividing the tips 2020-01-02
From Pat:
trying to figure out division of tips I have 3 full time employees 40 hrs a week and 1 part time worker 12 hours a week what percentages do i give them to give them appropriate tips :-)
Answered by Harley Weston.
A circle, a point, and a tangent line 2019-12-31
From Faisal:
A circle has radius 10 units and passes through the point (5,-16). The x-axis is a tangent to the circle. Find the possible equations of circle?
Answered by Penny Nom.
Volume of a tree 2019-12-19
From Maria:
I have trees for sale. I need to calculate the volume of a tree from base which is wider than the stem.The base will be used for the making of hurleys, similar to hockey sticks so the wider base is crucial.
Answered by Penny Nom.
Odds 2019-12-16
From Craig:
I’d like to know what are the odds of a lotto machine giving out a specific 4 digit number as a quick pick?
I know there are 10,000 possible 4 digit combinations. So let’s say the number is 3587 what are the odds the lotto machine spits it out EXACTLY that way on a quick pick?

Answered by Penny Nom.
Input and output of a function 2019-12-16
From Destanie:
What is the output if the input is -1???
Answered by Penny Nom.
A function problem 2019-12-12
From akash:
if (2,1) is on the f(x), what is the coordinate of the corresponding point of -2(f(2x-6)+5?
Answered by Penny Nom.
The fencing needed for a square pasture 2019-12-10
From Penny:
I'm trying to calculate how many feet of fence I will need to fence a 4 acre square pasture.
Answered by Harley Weston.
Strings made from three characters 2019-12-10
From Kelley:
Hi,

I vaguely remember some formula for determining all the possible combinations of a set of numbers (or whatever).

If I have 3 numbers (0 - 2), then I have 0, 1, 2, 00, 01, 02, 10, 11, 12, 20, 21, 22, 000, 001, 002, 010, 011, 012, 020, 021, 022, 100, 101, 102 and so on.

What is the formula for determining all potential results?

Thank you!!

Kelley

Answered by Penny Nom.
The equation of a line 2019-12-10
From Jourdan:
Suppose a mining company will supply 96000 tons of ore per month if the price is 80 dollars per ton but will supply 70500 tons per month if the price is 20 dollars per ton. Assuming the supply function is of the form y = m x + b , find the slope, m and y-intercept, b
Answered by Penny Nom.
Roll two dice and record the difference 2019-12-09
From Barbara:
Suppose you roll two dice 100 times. Each time you record their difference (always subtracting the smaller one from the bigger one to get a positive difference). The possible values you get are 0,1,2,3,4 and 5. You record the frequency of each value in the following table:
Difference of two dice 0 1 2 3 4 5
Observed frequency 12 31 26 13 10 8
Let your null hypothesis be that the dice are fair, and the alternative hypothesis be that they are not fair. Using a confidence level of α = 0.10, test the null hypothesis by a goodness-of-fit test.

Hint: begin by
completing table:
x 0 1 2 3 4 5
f(x)

Answered by Penny Nom.
3/8 and 0.375 2019-12-05
From Zamyiah:
how does 3/8 turn into 0.375 i just don't understand it the question is hard. please answer in time thank you for your time.
Answered by Penny Nom.
The area and volume of a wedge 2019-11-30
From Jasmine:
what is the volume and surface area of a wedge with definition
Answered by Penny Nom.
cos (3pi/2 -x) 2019-11-20
From sam:
1. express the following as a trigonometric function of angle x for cos (3pi/2 -x) the answer in the textbook is -sin x but i am unsure of the process to get the answer.
Answered by Penny Nom.
The product of 3 integers is -24 2019-11-06
From Rick:
This is a question on my sons pre-ap practice quiz. I think that there is information missing (?)

The product of 3 integers is -24
The sum of 3 integers is -12
What are the 3 integers ?

This is exactly how it was written on his quiz paper. I have wasted to much time on the internet, trying to find a formula(s) to help him.
Please help me.

Answered by Penny Nom.
The sum of the ages of Ruth and her mother is 77 years. 2019-11-06
From Lydia:
The sum of the ages of Ruth and her mother is 77 years. The difference in their ages is 27 years. How old is each?
Answered by Penny Nom.
Algebra 2019-11-06
From Krizhia:
How is y= 1/2 * (x-1)² be x= √2y + 1. Please show me the steps
Answered by Penny Nom.
The divisors (aliquot parts) of a positive integer 2019-10-31
From Kenneth:
Is there a fast and simple method that can be used to determine the aliquot parts, whole, positive numbers that can divide into another whole, positive number?
Answered by Harley Weston.
The largest integer k so that that 3335^k is a divisor of 3336! 2019-10-29
From Brandon:
Hello my name is Brandon and I having difficulties in solving the following question: Given the number 3336! = 1.2.3.4........................3336 . What's the greater positive integer number k with the condition that 3335expk is a divisor of 3336!
Thanks for your help.

Answered by Penny Nom.
The center of a circle 2019-10-24
From Patrick:
I'm trying to find a quick calculation to find the center of the circle that will touch the 3 points of an isosceles triangle.
Known dimensions are length of the triangle base and the height from the top point to the middle of the base.
Thanks in advance,
Kind regards,
Patrick

Answered by Penny Nom.
How many numbers from the set have the sum of its digits equal to 2? 2019-10-24
From Marcel:
Hello. My question is the following: Given the set {1,2...........10exp2019}, how many numbers from the set have the sum of its digits equal to 2?
I got some of them : 2 20 200 2000 20000 .......2. 10exp2018 11 110 1100 11000 110000....101 1010 10100 101000 1010000....10011 10010 100100 1001000 1010000.....10001 100010 1000100 10001000,,,,,,100001 1000010 10000100....... My problem is that I have difficulty in finding a strategy to obtain the exact quantity of numbers.
Please help me, thanks

Answered by Penny Nom.
The perimeter of a piece of a circle 2019-10-18
From Arwin:
The radius of a circle is 17 cm. A chord XY lies 9 cm from the centre and divides the circle into two segments. Find the perimeter of the minor segment.
Answered by Penny Nom.
The domain of f + g 2019-10-16
From Angel:
The domain of f+g when f(x)=8x+3 and g(x)=1-5x
Answered by Penny Nom.
How do you find the length and the width if all you had was area? 2019-10-16
From Ali:
So, How do you find the length and the width if all you had was area?
Answered by Penny Nom.
Simultaneous equations 2019-10-16
From deepak:
This is too complicated please help me
8/x - 10/y = 1 and x+y=9

Answered by Penny Nom.
Four fours 2019-10-03
From Irene:
Using number 4, four times with any equation and the answer will be -2 and -5
Answered by Penny Nom.
Fractions and negative powers 2019-09-22
From Will:
-2/3 to the -6 power
Answered by Penny Nom.
The volume of a fire hose 2019-09-18
From Carlou:
How many liters of water in a 2.5 inches diameter of fire hose with a 20 meters in length
Answered by Penny Nom.
4" x 4" square tiles 2019-09-18
From Jill:
how many 4" x 4" square tiles would I need for 50 square feet?
Answered by Penny Nom.
The amount of concrete in a foundation 2019-09-11
From Lorinda:
How do you calculate ( in cubic yards) how much concrete is needed for the foundation of a rectangular building 40 ft by 85 ft if the foundation walls are 8 inches thick and 7 ft 6 inches high?
Answered by Penny Nom.
What is -5 squared? 2019-09-10
From Pori:
What is -5 squared?
Answered by Penny Nom.
Vectors 2019-09-06
From Kayla:
Hi! Due to time constraint issues, we skipped vectors in Pre-Calc last year. However, I'm attempting to complete my AP Physics summer homework and there are 3 problems with vectors. As I explained, I have no idea even where to begin and as such I need some help.

Problem 1:
A plane flying at 90° at 45 m/s is blown toward 0° at 62 m/s by a strong wind. Find the plane's resultant velocity. I have tried all sorts of online calculators but none are providing me with a formula I can use to solve this problem on my own.

Problem 2:
If you walk 367 m North and 785 m West what is your total displacement from your original location?
What is the problem asking for when it says displacement?

Problem 3:
A plane travels on a heading of 127.0° at a velocity of 25 km/hr. What are the horizontal and vertical components of the plane's velocity? As you know already, I have never done vectors so I don't know what horizontal and vertical components of velocity are.

Thank you So Much for your help!! I am struggling!!

- Kayla

Answered by Penny Nom.
Using the Laws of Exponents 2019-09-06
From Samiya:
i have been struggling with the laws of exponents so im having trouble with this question. the question is to write (b*exponent2*c)(b*exponent5*c*exponent4*)
Answered by Penny Nom.
The volume of a berm 2019-09-06
From John:
I need to construct a berm:
8' tall
5' across the top
20' at the base
125' long
How much soil will I need

Answered by Harley Weston.
Can one equation with two variables be solved algebraically? 2019-08-30
From Don:
Can one equation with two variables be solved algebraically?
Answered by Penny Nom.
400 in 7.5 billion 2019-08-28
From Emma:
What is a better way to express 400 in 7.5 billion? My daughter has a rare condition, I want to know the answer to say “she is 1 in a ...”
Answered by Penny Nom.
A flat top cone 2019-08-25
From Mary:
I want to make a paper model of a flat top cone with the radius of the base is 12.5 cm and the radius of the apex is 7 cm.The height is 15 cm and the radius of the apex is 5 cm. The height is 15 cm.
Can you help me with the angle and construction, please?
Thanks.

Answered by Penny Nom.
The length of half an acre 2019-08-23
From Angeles:
How many Feet Long In Half An Acre (Rectangle)
Answered by Penny Nom.
The area of a shape 2019-08-14
From Lymp:
Hi, for math we are calculating the area of shapes and my teacher wanted me to research how to calculate the area of shapes so i was wondering if you could tell me how t calculate the area of any shapes. Please tell me how to calculate a cross, parallelogram and a polygon
Answered by Penny Nom.
A cone of maximum volume 2019-08-14
From Refilwe:
The slant height of a cone is 10cm. Determine the radius of the base so that the volume of the cone is a maximum
Answered by Penny Nom.
The radius of a circle of given area 2019-08-14
From shelby:
What's the radius of a circle that has an area of 803.84cm2? I also need examples of how you got the answer.
Answered by Penny Nom.
Conversing from tons to cubic yards 2019-08-03
From Kathleen:
Is there a simple formula to convert the following from tons to cubic yards:
Topsoil
Sand base
Fill dirt

Answered by Penny Nom.
Filling a hole 2019-08-01
From Jackie:
Filling a hole 25ft round 3ft deep how much dirt is needed?
Answered by Penny Nom.
The volume of a well 2019-07-30
From Jordan:
How much water could my well hold if the depth of the well is 30 feet feet and the diameter of the well is 24 feet ?
Answered by Penny Nom.
The maximum volume of a cone 2019-07-14
From A student:
find the maximum volume of a cone if the sum of it height and volume is 10 cm.
Answered by Penny Nom.
Volume of a spherical cap 2019-07-12
From Shanto:
In a jar of water, we kept a sphere with radius r. Then 75% of its volume went under water. Find out the distance of the top of the sphere from the surface of the water.
Answered by Penny Nom.
Order of operations 2019-07-06
From Carri:
What is the answer (( 5 × 12) / 3 ) + 30 - 50
Answered by Penny Nom.
Distance traveled 2019-07-03
From J:
If a fly flies at 15 feet per a second between two walls closing in upon themselves. Wall A is moving to the right at 3 feet a second and wall B is moving to the left at 15 feet a minute, how far will the fly fly before the walls touch?
Answered by Penny Nom.
Forty foot and 9 spacers 2019-06-28
From Christopher:
Ok I've got forty foot and 9 spacers one each end and 7 between what is the distance between the seven in the middle
Answered by Penny Nom.
The volume of a frustum 2019-06-24
From Abdulganiy:
A right pyramid on a base 10cm square is 15m high
a)find the volume of the pyramid
b)if the top 6m of the pyramid is removed what is the volume of the remaining frustum?

Answered by Penny Nom.
The area of a triangle in acres 2019-06-23
From Karen:
if I have a triangle that measures 78 ft x 54 ft x 50 ft, how do I figure out what part of an acre that is??
Answered by Harley Weston.
The perimeter of 200 acres 2019-06-14
From AARON:
How many feet is the perimeter of 200 acres?
Answered by Penny Nom.
Yards, feet and gallons 2019-06-12
From Ralph:
How many gallons of water in a 10 yard by 10 yard container 2 1/2 feet deep?
Answered by Penny Nom.
150 litres has been drawn from a tank 2019-06-12
From Eke:
When 150 litres has been drawn from a tank, it is 3/8 full, how many litres will the tank hold?
Answered by Penny Nom.
Conversions, feet, kilometers and acres 2019-06-10
From Erika:
A lake has 113 feet deep and an area of 115 acres, what is the volume of the lake in cubic kilometer?
Answered by Harley Weston.
Finite and infinite sets 2019-06-10
From Pretzie:
What is Finite Set & Infinite Set?
Answered by Penny Nom.
Two power generators 2019-06-07
From A student:
At 100% efficiency two generators would produce 750 MW of power. At efficiencies of 65% and 75%, they produce 530 MW. At 100% efficiency, what power would each produce?
Answered by Harley Weston.
How many lines of symmetry does a circle have? 2019-05-28
From raji:
how many lines of symmetry does a circle have???
Answered by Penny Nom.
Subdividing land 2019-05-09
From Reuben:
This is the measurements of my plot, A-B 46.7M, B-C 193.1, C-D 198.5 & D-A 208.25 (Clockwise naming of sides) angle A at 90 degrees. My questions is how do i subdivide this plot from the bottom having lines running parallel to C-D, eg two 2acre plots. the the remaining part becomes my compound (Uper part at line A-B)
Answered by Harley Weston.
The volume of a wall 2019-05-08
From Peter:
I’m building a dry stack stone wall which is 36” wide at the base, 42” tall and 24” wide at the top. What would be the volume for a 36” long segment ?
Thanks, Peter

Answered by Penny Nom.
Volume and Surface area of a sphere 2019-05-03
From Caitlin:
Why does the surface area formula of a sphere have a squared radius while the volume of a sphere has a cubed radius?
Answered by Penny Nom.
A dot on a child's car wheel 2019-04-28
From julia:
A wheel of a child’s car revolves at 10 rpm. A painted dot on the circumference of the wheel is 12 cm from the centre of rotation of the wheel. How far does the dot travel in 5 s, to one decimal place?
Answered by Penny Nom.
The length of an arc 2019-04-28
From Patrick:
If an arch is 48 inches wide at the base and 30 inches tall at its apex, what is the length of the arch?
Answered by Penny Nom.
A thread around a cylinder 2019-04-28
From Ally:
A cylinder has a diameter of 3.5 cm. How many times will a thread 143mm go round the cylinder?
Answered by Penny Nom.
Constructing a hexagonal pyramid 2019-04-25
From REBECCA:
I'm doing a project for my Honors Geometry class and I have to build a regular hexagonal pyramid. I was given the volume of .75 gallons, but I don't know how to figure everything else out. Please help?
Thanks,
Rebecca

Answered by Harley Weston.
The area of a triangle 2019-04-25
From troy:
The base and the height of a right triangle are represented by the expressions below. The area of the triangle is 28 square units. Find the lengths of the base and the height of the triangle

base= x+4
height=x-6

Answered by Penny Nom.
The area of a lot 2019-04-06
From Robert:
Can you help me determine the rough square footage of this lot please(attached)? I think it is nearly 7000 square feet. I am sure I need the degrees of of the corners but I don’t have that. An estimate would be all I need. Thank you for you help. Rob
Answered by Harley Weston.
The height of a building 2019-04-05
From Hayley:
A young man is 36 meters away from a building and looking at a bird sitting on top of the building. The man knows he is 85 meters away from the bird. How tall is the building?
Answered by Penny Nom.
Percentage change in refractive error 2019-04-04
From Ammara:
Hi, I'm trying to figure out how a paper I'm looking at has found a percentage from the numbers below. its a paper comparing two lenses. it says that the new lenses 'slowed the progression of refractive error by approximately 50%'.

the refractive error with the old lenses was -2.26 after two years it was -3.28

the refractive error with the new lenses was -2.24 after two years the refractive error is - 2.75
Thank you so much for your help it is most appreciated!

Answered by Penny Nom.
Internal acute angles 2019-04-02
From karan:
Prove that a convex polygon cannot have more than three acute internal angles.
Answered by Penny Nom.
The volume of a pile of dirt 2019-04-01
From Genevieve:
We have a pile of dirt approximately 6 feet high by 15 feet wide by 80 feet long.
We need to know how many cubic yards of dirt are in this pile. Thank you

Answered by Harley Weston.
The volume of water in a pipe 2019-03-21
From Abati:
How can I calculate the volume of water in a pipe with below details;

Pipe diameter - 6 inches
Length of pipe - 127meters

Answered by Penny Nom.
A scale factor 2019-03-19
From Miracle:
A man in a photograph is 2cm tall. His actual height is 1.8m. Write a scale statement and determine the scale factor.
Answered by Penny Nom.
The area of a triangle 2019-03-19
From Dylan:
The area of a triangle is 108in squared. A base and corresponding height are in the ratio 3:2. Find the length of the base and the corresponding height
Answered by Penny Nom.
Four consecutive positive integers 2019-03-12
From Nabila:
The product of four consecutive positive integers in 24024
What is their sum?

Answered by Penny Nom.
A triangle of maximum area 2019-03-07
From Tom:
Triangle ABC is such that AB=3cm and AC=4cm.
What is the maximum possible area of triangle ABC?

Answered by Penny Nom.
Misuse of greater than 2019-03-07
From Kenneth:
I have an old business mathematics textbook. The authors have indicated that the following expressions indicate multiplication:

? is 2/3 greater than 90; ? is 2/3 smaller than 90. They also indicated that the following expression would indicate division: 30 is 2/3 greater than ? and 30 is 2/3 smaller than ?.

How can these phrases indicate multiplication and division? How can 60 be 2/3 greater than 90 and also smaller than 90 as indicated above. What were the authors thinking? I have added the page from the book that indicates what I have explained in my message Kenneth

Answered by Harley Weston.
3 months loss and then a profit 2019-03-02
From Ida:
Let say, I have 10k of capital and I ran business for 4 months. The first months i loss 50%, the second month i loss 50% and so do the third month. The fourth month my profit is 50%

So can I calculate the balance of money that i have out of my capital?

Answered by Penny Nom.
The top half of a cone 2019-02-27
From danica:
What portion of the height from top that the cone be cut into two equal volume?
Answered by Penny Nom.
A quadratic equation problem 2019-02-26
From Em:
One root of the equation 2x^2 + bx - 24 = 0 is -8. What are the possible values of b and the other root?
Answered by Penny Nom.
f(x + 2) 2019-02-26
From a student:
How do you use f(x) = 3x+1 to determine the transformed function of f(x + 2)?
Answered by Penny Nom.
Composition of functions 2019-02-24
From Joshua:
Let g(x)=(x)^(2)+x-1 Find such a function f such that (fog)(x)=(x)^(4)+2(x)^(3)-3(x)^(2)-4x+6
Answered by p.
Loan payment formula 2019-02-24
From Kenneth:

I have a question regarding the loan payment formula shown below.

Calculating the Payment Amount per Period
The formula for calculating the payment amount is shown below.

Simple Amortization Calculation Formula

A = P X r(1 + r)n over (1 + r)n - 1

where

A = payment Amount per period

P = initial Principal (loan amount)
r = interest rate per period

n = total number of payments or periods

Is this formula/calculation a condensed version of a longer calculation? I am curious to know how the (1 +r)n - 1 was developed from the longer calculation. For example, r(1 + r)n may have been (r + rn)n. The n's are exponents.

I thank you for whatever helpful explanation that may be provided.

Kenneth


Answered by Harley Weston.
The Polynomial Remainder Theorem 2019-02-23
From pasandi:
f(x) is a quadratic polynomial. when f(x) is divided by (x-1),(x-2) & (x+2) the remainders respectively are -1, 4 and 2 how to find the f(x) in a question like this?
Answered by Penny Nom.
An arithmetic sequence 2019-02-23
From Dalal:
If x+1 and -x+17 are the second and sixth term of a sequence with a common difference of 5, what's the value of x.
Answered by Penny Nom.
Margin with commission as part of selling price 2019-02-13
From Anni:
I need to find selling price to put us at 24% margin.
15% commission is based on selling price and is included in our cost.
I'm trying to do it excel and keep getting a circular reference.
Total cost= cost + commission (15% of selling price)
Selling price=total cost + 24% margin

Answered by Penny Nom.
Factoring n^2+3n-270 2019-02-11
From Ashley:
n^2+3n-270
Answered by Penny Nom.
A cylindrical drum 2019-02-10
From Ronald:
I have a cylinder that is 32 inches round by 25 inches deep how much water can it hold
Answered by Penny Nom.
The exponential form of sqrt(13) 2019-02-10
From isaebl:
what is the exponential form of sqrt(13)
Answered by Penny Nom.
Function notation 2019-02-10
From Paige:
What is the difference between f(x+2) and f(x)+2?
Answered by Penny Nom.
Fractions with roots 2019-02-06
From DEBMALYA:
5/√2+√3 –1/√2–√3
Answered by Penny Nom.
A fraction of a cent 2019-02-01
From michael:
$ 300 divided by 1,000,000 is 00.0003 ... so, is that three hundredths of one ($ 00.0003) cent or what ... thanks
Answered by Penny Nom.
An arithmetic sequence 2019-01-31
From sara:
if the 6th term of an arithmetic sequence is 8 and the 11th term is -2, what is the first term?
Answered by Penny Nom.
One quarter of a quarter cake 2019-01-30
From Tre:
1/4 of a birthday cake was left over from a party. The next day, it is shared among 4 people. How big a piece of the original cake did each person get?
Answered by Penny Nom.
y=lnx+(1+ln2)/2 and y=x^2 2019-01-28
From Mike:
Prove that y=lnx+(1+ln2)/2 and y=x^2 touch each other.

The course is about logarithm and root functions... how should I solve this problem?

Answered by Penny Nom.
A scale factor 2019-01-27
From Hannah:
What is the scale factor of 36 meters and 1.5 centimeters?
Answered by Penny Nom.
A median of a triangle 2019-01-24
From Anna:
For triangle J(1,4) K(-3,-1) L(3,4), find an equation for the median from vertex J
Answered by Penny Nom.
A triangle made from 12 matchsticks 2019-01-20
From Palesa:
How many different types of triangles can be made with a perimeter of 12 matches?
Answered by Penny Nom.
7 spheres on a hexagonal tray 2019-01-14
From herm:
what is the length of each side of a hexagonal tray, with the height of each side 0.75 inch, to hold seven spheres, each with a diameter of 3.00 inches? The spheres are placed such that each side of the hexagon is touched by one sphere at its midpoint (and the seventh sphere is place in the center of the "ring" of the other six spheres.
Answered by Harley Weston.
Finance charge 2019-01-03
From Kenneth:

Hello Math Central:

 

The calculation for determining the finance charge per $100.00 (FC/100) is determined by the following:

Finance Charge/Amount financed * $100.00 = the finance charge per $100.00

Since the denominator (amount financed) is not multiplied by $100.00, how does the result equal the finance charge per $100.00? The numerator (finance charge) is the only amount multiplied by $100.00.
 

Example 1.  3/4 * 6 = 18/4 not 18/24 The 4 is not multiplied by the 6.


Example 2.  $49.00/$200.00 * $100.00 If $49.00 is divided by $200.00 the result is $0.245/$1.00. If $0.245/$1.00 * $100.00 = $24.50/$1.00. This answer is not $24.50/$100.00.
How does the above equal the finance charge per $100.00 when the denominator is not multiplied by $100.00?


If the division is not used to indicate the $0.245 per $1.00, the result becomes $49.00/$200.00 * $100.00 = $4,900.00/$200.00.

 

I thank you for any helpful reply.

 

..........

Kenneth


Answered by Penny Nom.
An octagonal deer blind 2018-12-31
From Alan:
My grandson and I would like to making a 6'x6' octagon floor for a deer blind. He is eight yrs old and the math is outside my wheelhouse. We need to know the length of the boards and angles of the corners please.
Thank you, Sincerely,
Alan

Answered by Penny Nom.
y = f(x) 2018-12-30
From yogita:
What is the actual meaning of "f(x)" ? Please give me full explanation
Answered by Penny Nom.
Solve for x in Y=1.5x + log (1+x) 2018-12-23
From Asadollah:
Please restate below expression in terms of y

Y=1.5x + log (1+x)

Thanks

Answered by Penny Nom.
The surface area of a dome 2018-12-18
From Chithrabhanu:
If a dom dia is 4.552 and the height is 1.21 what is the surface area?
Answered by Penny Nom.
cos(2x) = sin(x) 2018-12-18
From Liz:
Determine all values of x in [0,2pi] that solve the equation Cos(2x)=sinx
Answered by Penny Nom.
The area and weight of sod 2018-12-11
From Susan:
I have equipment that will cut sod 30 inches across and 75 Feet in length? how much will this be in square foot age ? Yards? Approximate weight?
Answered by Penny Nom.
A puzzling equation 2018-12-04
From Manny:
Hello,

Can you explain why this is wrong:
-1 = (-1)^3 = (-1)^{6/2} = ((-1)^6)^{1/2} = (1)^{1/2} = 1 ??

Thank you

Answered by Penny Nom.
Four digit numbers from 0, 1, 2 and 3 2018-11-30
From A parent:
how many four digit numbers can be formed using nos.0,1,2,3 if repetition is allowed?
Answered by Penny Nom.
y = |x| - 1 2018-11-30
From alexis:
what does this mean y=|x|-1
Answered by Penny Nom.
Ordering fractions 2018-11-25
From Mae:
order these from least to greatest:

19/6, 3 5/14, 83/21

Answered by Penny Nom.
2/5 as much time 2018-11-16
From Izzy:
If Patricia spent 3/8 of her time cleaning her room. She spent 2/5 as much time washing the dishes as she did cleaning her room. What fraction of time did Patricia spend Washing dishes? And it wants us to simplify our answer
Answered by Penny Nom.
-4 + 3/2 2018-11-15
From Chloe:
so I have this paper and I'm confused, -4 + 3/2=???? so I originally got -1/2 but I looked at Calculator papa and he says -5/2 and then I looked at google and all I got was -2.5 and i am so confused and lost.
Answered by Penny Nom.
Price before VAT and a discount 2018-11-15
From Carol:
I bought a computer for £330.
The selling price included VAT at 20%.
Then I was allowed a staff discount of £30.
What was it’s original price before the VAT and staff discount?

Answered by Penny Nom.
The price of eggs rises 10 cents per dozen 2018-11-10
From Maddie:
Hi! I'm currently struggling with a word problem that says: "If the price of eggs rises 10 cents per dozen, one will be able to get 2 dozen fewer eggs with $6.00 than was possible at the lower price. What is the lower price? We are working on completing the square and all of the problems I've done so far in this chapter require using that method. If possible and needed, could you explain how to set up the equation? Thank you so much! I appreciate all your help!!
Answered by Penny Nom.
Dividing by a fraction 2018-11-02
From Fadely:
If 8 divided by 4/2 can be expressed as 8 X 2/4, how can 8 X 2/4 be converted back to 8 divided by 4/2?
Answered by Penny Nom.
Why 1/2 in the area of a circular sector? 2018-10-26
From Ariel:
In the equation for area of a sector=1/2r^2theta why is it 1/2? Please explain conceptually not algebraically.
Answered by Penny Nom.
Why 1/3 in the volume of a cone? 2018-10-20
From Keith:
In the formula for the volume of a pyramid, Volume = 1/3 * base * height, where does the 1/3 come from, intuitively?
Answered by Penny Nom.
f(x) and f(-x) 2018-10-19
From Simran:
what is difference between f(x) compared to f(-x) on graph
Answered by Penny Nom.
The area of a trapezoid 2018-09-24
From An other:
AB-272 feet: AC 690 feet; CD-330 Feet; DA-669Feet Calculate the area in feet and Acres.
Answered by Penny Nom.
Two curves 2018-09-23
From Megan:
How do i find the interception points of xy=-2 and y=x+3? Many thanks!
Answered by Penny Nom.
The point-slope equation for a line 2018-09-23
From Sally:
The equation of the line in point-slope form for the line passing through the point ( 4, -6) and parallel to the line y = -6x + 2. Describe the steps.
Answered by Penny Nom.
Predicting the number of fish in a lake 2018-09-16
From April:
A researcher catches 30 fish in a pond, attaches a marker to their fin and releases them. The next day she places a large net in the water and catches 9 fish. 3 have the marker attached to their fin. Can she predict the number of fish in the pond???
Thank you

Answered by Penny Nom.
Exponential form 2018-09-14
From Saquanna:
What is 25 in exponential form
Answered by Penny Nom.
Fixed cost 2018-09-13
From Heather:
On the bus question you answered, what would be the fixed rate? Is that the rate per person or is that also the base rate? ( cost of empty bus)?
Answered by Penny Nom.
Exponential form 2018-09-12
From Grace:
hi, I can't figure out how to do 735 in exponential form. note that im only in 5th grade and its not supposed to be in the scientific notation or whatever. okay, thanks!
Answered by Penny Nom.
A locus of points 2018-09-03
From Ericka:
Find the locus of points three times as far from (0,4) as from (2,0)
Answered by Penny Nom.
Golf schedule for 12 players over 8 days 2018-09-03
From William:
We have a golf trip with 12 golfers

we are playing 8 rounds together......we want to mix the group up at best possible w/o too much overlap.......
thoughts? the math is way over my head...

even if we play the first 6 rounds with different partners, then the last 2 can be repeats ...or whatever the math works out

Answered by Penny Nom.
The sum and difference of two numbers 2018-08-30
From Saranya:
Hi,

The question is to find the numbers from sum and difference.
The Sum = 14 and the difference = 2

I found in this web site some useful answers. but my kid is going second grade only. She don't know how to do a division or multiplication.
So can you please explain on how to find the answer using only the Addition and Subtraction.

Thanks in advance.

Answered by Penny Nom.
A cube in a cylinder 2018-08-13
From Geli:
A cylinder with a radius r and height 2r+4 contains a cube with edge length r√2. What fraction of cylinder volume is taken up by the cube
Answered by Penny Nom.
Form a cone from a circle sector 2018-08-12
From Tinashe:
A 216 sector of a circle of radius 5cm is bent to form a cone. Find the radius of the base of the cone and its vertical angle.
Answered by Penny Nom.
5 mm of water over 1 hectare 2018-08-10
From Joyce:
Please help me with this question.
How many tonnes of water fell on 1 hectare of land during a rainfall of 5mm?

Answered by Penny Nom.
Linear feet and cubic feet 2018-07-16
From Dave:
How do I convert: 1 foot wide x 15 inches high x 19 inches deep

We are trying to figure out how to convert the linear footage into cubic footage the above are the figures represent just 1 linear feet of several thousand

Any thing helps,

Thanks in advance

Dave

Answered by Penny Nom.
Four odd factors 2018-07-04
From Manchali:
Find a number that has 4 factors and all of them are odd numbers.
Answered by Penny Nom.
The surface area of a levee 2018-07-04
From John:
Figuring out the surface area in sq. Ft. Of a levee

Height:10’
Top of levee width: 6’
Width of base (from toe to toe): 40’
Length of levee: 1,250’

Answered by Penny Nom.
The product of two numbers 2018-06-19
From amie:
the product of two numbers is 108.when the larger number is divided by the smaller number the quotient is 3. What are the number.tnx
Answered by Penny Nom.
The domain of f(g(x)) 2018-06-18
From Joshua:
What are the restrictions of the domain of f(g(x))?
Answered by Penny Nom.
The number of terms in an arithmetic sequence 2018-06-15
From Don:
how many terms in arithmetic sequence are there if the first term and the last term are 3&59 respectively in common difference is 4?
Answered by Harley Weston.
The base for a circular pool 2018-06-10
From Adam:
How many sheet of 4ft by 8ft foam would I need to lay under a pool 15dt in diameter
Answered by Penny Nom.
The difference between two numbers is -3 2018-06-05
From Hannah:
write a pair if integers whose difference is -3 ( given that the greater number is 8 )
Answered by Penny Nom.
Pre tax and post tax percentages 2018-05-22
From Tom:
Hi,
I work in a clothes shop which can also do mail-orders. My Head Office has sent a report with which we can check the records we keep in store tally with those at HO.

According to that report we have taken £2578.25 in orders, excluding VAT (20%), which would therefore be £3093.91 including VAT (2578.25+20%=3093.91).

According to my records, we have taken £3179.32 including VAT, which would be £2543.45 excluding VAT (3179.32-20%=2543.45).

Why is it that the difference between the VAT inclusive figures (3179.32-3093.91) is +£85.41, but the difference between the VAT exclusive figures (2543.45-2578.26) is -£34.81?

Now, I can see from the report that 1 order has not been recorded by HO, and I know that our average order value is around £30, so the -£34.81 makes sense. But surely both figures should be a minus, regardless of whether they include VAT?

I'm sure there is an obvious answer, but I just cannot see it! Any help would be gratefully received.
Thanks,
Tom

Answered by Penny Nom.
The area, radius and slant height of a cone 2018-05-18
From joette:
If you have a cone how to find the slant height when given the area and radius?
Answered by Penny Nom.
Write these fractions from greatest to the least 2018-05-18
From angela:
math fraction greater to the least

1\5 ,1\2 or 1\4

Answered by Penny Nom.
Rods,poles and perches 2018-05-13
From roy:
i have some land and its measured in rods poles and perches
how cann i convert to understand it more

Answered by Penny Nom.
Two expressions for the perimeter of a a rectangle 2018-05-13
From Slayde:
A rectangular house has one side 4 metres longer than the other. Create two different expressions for its perimeter. The expressions should be given in factorised and expanded form.
Answered by Penny Nom.
The square root 3 in exponential form 2018-05-09
From Paige:
how do you write the square root 3 in exponential form
Answered by Penny Nom.
Converting decimals to common fractions 2018-05-08
From Ruth:
what does the 0 represent in .018 versus the decimal .180? As in, changing these to fractions? Thank you.
Answered by Penny Nom.
Soccer fans and cricket fans 2018-05-06
From Chibo:
In a survey of 60 sport fans, 45 enjoy soccer, 35 enjoy cricket and 10 do not enjoy either soccer or cricket, but some other sports.What percentage of sports fans enjoys both soccer and cricket?
Answered by Penny Nom.
A fraction 2018-05-02
From Adwin:
The denominator of a rational number is greater than its numerator by 8. If the numerator is increased by 17 the number obtained is 3 by 2. Find the number.
Answered by Penny Nom.
Gauss' Addition of whole numbers. 2018-04-30
From Brad:
I found this on your site. Question: what is the sum of the first 100 whole numbers?? Is there a different formula if the numbers begin at a number other than one? For example What is the series I want to add is goes from 7 - 53?
Answered by Harley Weston.
Spreading soil over 10 acres 2018-04-29
From Joe:
If you have 54000 cubic yards of soil and want to spread it over 10 acres of land, by how many inches would the ground level rise?
Answered by Penny Nom.
Throwing a football 2018-04-26
From Abby:
Daring Danny's threw a football at an angle of 40 degrees to the horizontal. The football hit the ground at 36 feet. Danny is 5 feet 1 inch tall. Find the initial velocity

I have been trying to figure this problem out for forever! Please help! I am really confused because I don't know time either and you are supposed to find that as well as velocity.

Answered by Penny Nom.
A rectangular prism with rounded corners 2018-04-25
From Jackie:
Rectangular prism that is 40 by 20 with rounded corners with a radius of 2.5.

What would the surface area be?

Answered by Penny Nom.
More on the curvature of the Earth 2018-04-23
From will:
the formula for figuring the earth's curve goes against logic, looking at a fixed point and backup 1mi. the point drops 8" then 16" in the next mi. and 32" in the third mi. why shouldn't it be 24" why is the 8" per mi. squared can you tell me in laymens terms why this is it goes against logic it would seem the correct wat would be to add up 8" per mile as you back up from the fixed point 8"- 16"- 24"- 32" not 8/16/32/64"
Answered by Harley Weston.
A volume expression for a pyramid 2018-04-23
From Shaheer:
Do a three sided pyramid and a square pyramid have the same formula if you want to calculate the volume?
Answered by Penny Nom.
The distance between two men 2018-04-21
From Kristin:
If a 5 ft 6 in man gets hit at a downward 45 degree angle by a 5 ft 10 in man, how far apart are the two men? AND, would the shorter man have to be kneeling or standing? Is there a formula and answer for this question?
Answered by Penny Nom.
How to find the area of triangle? 2018-04-19
From g.a:
how to find the area of triangle
Answered by Penny Nom.
What is small four-digit whole number? 2018-04-15
From Rajesh:
What is small four-digit whole number?
Answered by Penny Nom.
The sides of a circle 2018-04-10
From Reid:
Hello,
I recently was wondering about whether or not a circle has an infinite number of sides, and I ended up searching it on your website. I saw that you guys found the question to be too ambiguous or something of that nature, and I thought about your process of reasoning involving vertices and such. I soon realized that I may have come up with the solution to the question, but I want to confirm it with you guys. Allow me to explain:
A circle, unlike any other typical 2 dimensional polygon, can sustain an infinite number of straight lines coming in contact with only one point on it. A square, for example, cannot. A square only has 4 locations that can sustain a such a line, each of those being its corners. The flat edge of a square cannot support a tangential line, because the line would either be crossing the edge of the square or coming into contact with multiple points along its edge.
This concept is consistent in every 2 dimensional polygon: pentagons sustain 5 locations for tangential lines, hexagons 6, nonagons 9, etc. The reason a circle has an infinite number of sides is simply the fact that it must have an infinite number of "corners", assuming it can be defined as a polygon like any of the shapes I described above.
Corners can only exist assuming there is two sides coming into contact with one another. If there is an infinite number of corners in the circle, which is apparent due to the above reasoning, there must be an infinite number of sides coming into contact with each other. Thus, a circle is a polygon that consists of an infinite number of sides coming into contact with each other.
Please review this reasoning and let me know if it is solid.
Thank you!

Answered by Penny Nom.
A side of a square 2018-04-06
From Angie:
I am dyslexic and have problems with spelling and math I'm trying to figure 4.610 Acres Square trying to figure out how many feet or yards of one side if that makes sense to you thank you I'd really appreciate the help
Answered by Penny Nom.
Factoring a quadratic 2018-04-02
From Billy:
I need help factoring the following question: 4x(squared) -10x +16.
Answered by Penny Nom.
The price of a sheet of metal 2018-04-02
From Nydia:
What is the formula to calculate the following:

I nee to find out what the sheet price be. The dimensions are 2000x3000 mm into ft they are 78x1181. The sqft price I have is $22.16. How do I figure out the sheet price from that?

Answered by Penny Nom.
A circle problem 2018-03-30
From ahamed:
the diameters of a circle AB and CD intersect each other at O perpendicularly. Then a chord DF is drawn. AB and DF intersect at E. DE=6cn and EF=2cm.find the area of thw circle
Answered by Penny Nom.
The volume of a tent with a hexagonal base 2018-03-27
From shohel:
A tent has its base in the shape of a regular hexagon whose sides are 10m . If the height of the tent is 12m , then find its volume.
Answered by Penny Nom.
Distance between a chord and its arc on a circle 2018-03-23
From Doug:
Specifically, what is the subject distance for the Earth orbiting for 27 days. Assume the orbit of the Earth to be a circle have a 93 million mile radius. Assume the angle of arc to be (27/365) x 360 degrees. Thank you.
Answered by Penny Nom.
Walking a fraction of a mile 2018-03-20
From Jamie:
Who walked a fraction of a mile that is closer to neither 0 nor 1 ? Explain
Avery 1/6
me.Nunez 5/6
Ms. Chang 1/3
Mr. O’Leary 4/8
Miss Lee 4/6

Answered by penny Nom.
A circle of circumference 4*pi*r 2018-03-19
From Ankith:
If the circumference of a circle is 4*pi*r, what is its area?
i) 4*pi*r^2
ii) 2*pi*r^2
iii)3*pi*r^2
iv) pi*r^2

Answered by Penny Nom.
Which movie lasts longer? 2018-03-14
From YKJAJA:
Which movie lasts longer than the other?
A) “space adventure” lasts for 1 2/3 hour
B) “animal world” lasts for 15/12 hour

Answered by Penny Nom.
A geometric series 2018-03-13
From nathi:
Hi I am really struggling with this question please help !!!!
a pohutukawa tree is 86 centimetres when it is planted. in the first year after it is planted , the tree grows 42 centimetres in height.Each year the tree grows in height by 95% of the growth of the previous year.
assume that the growth in height of the pohutukawa tree can be modelled by a geometric sequence.
A)find the height of the tree 5 years after it is planted and figure out the maximum height the pohutukawa tree is expected to reach in centimetres. The maximum height part is not answered.

Answered by Penny Nom.
The sum and difference of two numbers 2018-03-13
From samima:
Two numbers have a difference of 0.85 and the sum 1.What are the numbers?
Answered by Penny Nom.
The dimensions of a rectangle 2018-03-03
From Mia:
A wire of length 36cm is bent to form a rectangle. Find the dimensions of the rectangle.
Answered by Penny Nom.
Expand (5x-9)(5x+9) 2018-03-01
From adil:
expand the following:
(5x-9)(5x+9)

Answered by Penny Nom.
A four digit pass code 2018-02-26
From Jessica:
I was trying to fond out how many combinations could there be for a four digit pass code to a phone when no numbers are repeated?
Answered by Penny Nom.
The perimeter of a square 2018-02-25
From Alicia:
What's the perimeter of a square with 441cm square
Answered by Penny Nom.
Multiplies and factors 2018-02-24
From Lil:
Is a multiple the same as a factor?
Answered by Penny Nom.
Dividing a region in half 2018-02-24
From mandy:
There is a line through the origin that divides the region bounded by the parabola y=4x−5x2y=4x−5x2 and the x-axis into two regions with equal area. What is the slope of that line?
Answered by Penny Nom.
The slope of a line 2018-02-22
From Steve:
my current slope is 6%. I want to know how many inches higher I will be at 18' Thanks
Answered by Penny Nom.
2.5 gallons of liquid, one half an inch deep 2018-02-21
From Cain:
How many square feet would 2.5 gallons of liquid cover at half an inch deep?
Answered by Penny Nom.
A footballer angle 2018-02-14
From Kim:
Two goal posts are 8m apart. A footballer is 34m from one post and 38m from the other. Within what angle must he kick the ball if he is to score a goal.
Answered by Penny Nom.
Related rates 2018-02-11
From angelo:
hi admin please help me answer this question. thank you! At a certain instant of time, the angle A of a triangle ABC is 60 degrees and increasing at the rate of 5degrees per second, the side AB is 10cm and increasing at the rate of 1cm per second, and side AC is 16cm and decreasing at the rate of 1/2 cm per second. Find the rate of change of side AB?
Answered by Penny Nom.
Water in a cone 2018-02-10
From Shuvo:
The diagram shows a vertical cross-section of a container in the form of an inverted cone of height 60 cm and base radius 20 cm. The circular base is held horizontal and uppermost. Water is pursed into the container at a constant rate of 40 cm3/s.
Show that, when the depth of water in the container is x cm, the volume of the water in the container is (πx^3)/27 cm3.
Find the rate of increase of "x" at the instant when "x" = 2.

Answered by Penny Nom.
3 yards 6 inches times 7 2018-02-08
From Tamara:
3 yards 6 inches × 7
Answered by Penny Nom.
A question about perfect squares 2018-02-04
From Jolyn:
Find the smallest possible value of a whole number m if 648x m is a perfect square
Answered by Penny Nom.
Positive and negative values of a function 2018-01-30
From Grayson:
f(x)=x^6-x^4
Interval: ( negative infinity, negative one )
Test Value: negative two
Function Value f(x): positive forty eight
Interval: ( negative one, zero )
Test Value: negative one
Function Value f(x): zero
Interval: ( zero, positive one )
Test Value: positive one
Function Value f(x): zero
Interval: ( positive one, positive infinity )
Test Value: positive two
Function Value f(x): positive forty eight

What is the sign of f(x) for each Interval?

Answered by Penny Nom.
Simplify 1/2^i 2018-01-28
From Deepak:
How do I simplify this complex equation:

z=1/(2^i)

Answered by Penny Nom.
The sum and difference of two numbers 2018-01-25
From Ali:

Hello,
I was looking at the original question that was posted and answered by
your team:
http://mathcentral.uregina.ca/QQ/database/QQ.09.07/s/donna1.html

I used the 7 and 4 as example and not looking for 39 per original
question:

A=7
B=4

7+ 4 =11 and 7-4 = 3

We end up with 11 +3 = 14

A= 14/2 give use 7

B = how do you solve for B or 4 with out knowing anything about 3 or any other numbers ?

Thank you
Ali


Answered by Penny Nom.
The equation of a line 2018-01-23
From Gloria:
Write the equation of a line in standard form through the point (1, 6) and perpendicular to the line 2x –y = 9.
Answered by Penny Nom.
y as an expression in terms of x 2018-01-20
From Mark:
What is y as an expression in terms of x ?
Answered by pennt.
Profit margin and markup 2018-01-18
From Oyewumi:
Mr A sold a car to Mr B at 20% profit margin and Mr B sold the car to Mr C at 30% markup on cost. If Mr C got the car at $x.
1. How much did Mr A buy the car in terms of x.
2. How much did Mr A buy the car if x is $2800000

Answered by Penny Nom.
The decimal form of a fraction 2018-01-12
From Tom:
Prove that the decimal representation of the quotient of 2 integers must repeat (if it does not terminate).
Answered by Penny Nom.
When are the hands of a clock perpendicular? 2018-01-11
From Nazrul:
How many times in a day are the hands of a clock perpendicular to each other? How can I find the times? Please help me.
Answered by Penny Nom.
A fraction of a mile 2018-01-06
From Sierra:
There are 5280 feet in a mile. what fraction of a mile is represented by 660 feet?
Answered by Penny Nom.
A puzzling sequence 2017-12-19
From Alan:
My son has the following sequence to work out the formula for the nth term (and fing 5th and 6th terms) and I have tried a variety of ways but can't help!

1/2, 5/6, 1, 11/10

I have converted all to 15/30, 25/30, 30/30, 33/30 and so think the next two terms are 37/30 and 45/30 but I have no idea how to prove or if I am right!

Any help appreciated.

Answered by Penny Nom.
Needing the area of an irregular Pentagon 2017-12-06
From Dale:
I'm needing to find the square footage of an irregular pentagon.
I've seen the other questions similar to this one, but without a great deal of difficulty can't get you the angle degrees, or the length between opposing angles.
I'm hoping that a different bit of information can help. So here goes:
The longest side is 67ft long.
One of its legs is 39ft long, and the other is 18ft long.
The leg off of the other end of the 39ft leg is 35ft long.
The leg off of the other end of the 18ft is 49ft long, and meets the other end of the 35ft side.
The information that I DO have, which I'm hoping makes the difference, is that the 39ft and the 18ft sides are parallel.
Also, while the angles at each end of the 67ft side are not right angles, they are fairly close to it, approximately 80 degrees at the 39 ft side corner, and 100 degrees at the 18ft side corner.

Dale.

Answered by Harley Weston.
An arithmetic sequence 2017-11-30
From yo:
x; 2x+1; 11 are three consecutive terms of an arithmetic sequence.calculate x
Answered by Penny Nom.
The derivative of f(x)=2^x/x 2017-11-28
From Chhavi:
f(x)=2^x/x.
Find f'(x).

Answered by Penny Nom.
GCD 2017-11-25
From A student:
Suppose a,b, c are integers such that gcd(a,b)=gcd(a,c)=1. Show that gcd(a,bc)=1.
Answered by Penny Nom.
Abstract algebra 2017-11-25
From A student:
Suppose a,b, c are integers such that gcd(a,b)=gcd(a,c)=1. Show that gcd(a,bc)=1.
Answered by Penny Nom.
Exponential form 2017-11-25
From Sherry:
What is the exponential form of 3sqrt 15?
Answered by Penny Nom.
Heat equation 2017-11-23
From Max:
What does du\dt=a(triangle)^2u mean. Can it be solved for t.
Answered by Penny Nom.
How many miles did he drive in one hour? 2017-11-21
From Ava:
Graham drove 42 1/3 Miles in 1 1/3 hours. How many miles did he drive in one hour?
Answered by Penny Nom.
An expression for the area of a triangle 2017-11-19
From hari:
why is area of a triangle drawn from centre of a circle i.e radii forming the sides is 1/2 *r^2 sin theta where r is radius
Answered by Penny Nom.
Splitting twice 2017-11-11
From Terry:
4 people are splitting 1/3 of something. 1 is to receive twice as much as the other 3. what it the fractional interest for each?
Answered by Penny Nom.
The graph of graph y= -2x-1 2017-11-11
From Natividad:
How do you graph y= -2x-1?
Answered by Penny Nom.
Five coins 2017-11-10
From Allan:
I have five coins..a penny,a nickle,a dime,a quarter and a loonie.....how many different amounts of money could i pay using ny combination of this coins
Answered by Penny Nom.
Mathematical ideas in everyday life 2017-11-08
From Ricita:
Mathematical ideas to solve various problems of our everyday life ,environment related problems.
Answered by Penny Nom.
Scale factor 2017-11-03
From Hannah:
I do not understand it...... What is the scale factor for 9 cenimeters = 1 meter
Answered by Penny Nom.
Surface area of a hemisphere 2017-10-28
From Shaina:
If the total surface area of a hemisphere be 36.22/7 cm then its radius will be 3 cm is it true or false
Answered by Penny Nom.
Adding fractions 2017-10-27
From Sandy:
virinchi distributed 4/7 of cake to his friends 1/3 of it to his family members which part of cake is remaining?
Answered by Penny Nom.
f(x)=-x^2-6x find f(x-2) 2017-10-27
From Kenneth:
f(x)=-x^2-6x find f(x-2)
Answered by Penny Nom.
The distance between the origin and a moving point on a graph 2017-10-16
From Paulina:
Find the rate of change of the distance between the origin and a moving point on the graph of y=x^2 +1 if dx/dt=2 centimeters per second
Answered by Penny Nom.
Five bales 2017-10-08
From John:

two answers were given... I believe both to be incorrect.
five bales weighed two at a time: weightx2 given: 110,112,113,114,115,116,117,118,120, and 121. It stated all combinations ie: ab,ac,ad,ad,bc,bd,be, etc. to arrive at the above 10 combined weights. I come up with the following...

Bale A= 54lbs, Bale B=56, Bale C=58, Bale D=59. and Bale E=62 pounds.

A+B=110, A+C=112, A+D=, B+C=114, B+D=115 ,A+E=116 ,C+D=117, B+E=118 ,C+E=120, and D+E=121


Answered by Penny Nom.
The volume of a cone without calculus 2017-10-02
From Akash:
How to find the volume of a cone without the knowledge of calculus?
Answered by Penny Nom.
The square root of 6 is irrational 2017-10-02
From John:
http://mathcentral.uregina.ca/QQ/database/QQ.09.06/sylvia1.html
In the initial assumption of that proof, root 6 is assumed to be a/b where a and b have no common factors, but why does having a common factor make it irrational?

Answered by Penny Nom.
Four multiples of 10 2017-09-23
From Laudacir:
Four multiples of 10 are added together.the total is a 3 digit number with three consecutive digits. What could the four number be?
Answered by Penny Nom.
Changing a mixed number to a decimal 2017-09-23
From karla:
to change an improper fraction into a decimal 5 3/4 equals
Answered by Penny Nom.
The surface area of a sphere 2017-09-17
From penny:
The surface area of a sphere is 1519.76cm. Taking 3.14 , find the radius of sphere
Answered by Penny Nom.
Average price 2017-09-15
From Annette:
If I sold a widget at $15.50 17.5% of the time and $16.50 88.5% of the time what is the algebraic expression for that. How do I arrive at an average price
Answered by Penny Nom.
Quadrilateral ABCD is inscribed in a circle 2017-09-11
From Joie:
Quadrilateral ABCD is inscribed in a circle such that side DA is the diameter. AB=2m., BC=4m., CD=6m., angle BAD=75.93degrees. Find the area of the quadrilateral.
Answered by Penny Nom.
Simultaneous equations with fractions 2017-09-09
From Farah:
Hi, my name is Farah. I hope you can help me with this question . X/2 + g/5= 3 and 2g - f = 10
Answered by Penny Nom.
The volume of an attic 2017-09-07
From paul:
trying to determine volume of an attic base is rectangular 29 ft by 37 ft peak runs parallel to longest base side and is 7 ft long roof lines run from each end of the peak to the respective corners the peak is 3.5 ft above the base
(so seen from above there are 5 visible lines: the peak and 1 line to each of the 4 corners)

Answered by Harley.
A rope formed in a semicircle 2017-08-29
From Janna:
How do I find the total length of a piece of rope formed in a semicircle that has a diameter of 8 inches?
Answered by Penny Nom.
Factors 2017-08-16
From Gemma:
Why do 315 and 525 have the same number of factors?
Answered by Penny Nom.
A rectangle and a circle 2017-08-12
From Ramzan:
a diagram shows the shape of rectangular framework with length (2x+20)cm and with (y+10)cm . The outline is made of wire . If a circle with diameter 56 cm is to be made from the wire and the area of the rectangle is 420 square centimeter ,find the possible lengths and width of the rectangle
Answered by Penny Nom.
The perimeter of a sector of a circle 2017-08-11
From Alisa:
find the perimeter of the sector with radius 7 cm in which the angle at the centre is 60 degrees leave your answer in terms of pi
Answered by Penny Nom.
(x-18)/7=6 2017-08-08
From David:
(x-18)/7=6
Answered by Penny Nom.
Significant digits 2017-08-06
From Pat:
The directions for a problem are: "Solve the problem, expressing your answer using the correct number of significant figures."

Problem 0.0043 divided by 2.04
Answer before using the correct number of significant figures is 0.002107843

Is the correct number of significant figures the number below?
2107843

Pat

Answered by Penny Nom.
Teams of 4 from 20 people 2017-08-04
From Christopher:
math team has 20 people on the team but only 4 can enter the competition. The team captain has to pick 4 of them but he knows that he cannot put James, Bill or Todd on the team with one another or there will be problems. what would the formula be to find the solution to how many possible team combinations the are
Answered by Penny Nom.
Factorials 2017-07-27
From dinesh:
Is there any shortcut formula for multiply series numbers. like
1x2x3x4x5.......x100 =?

Answered by Penny Nom.
3 consecutive multiples of 11 2017-07-22
From nisha:
using the multiples formula shown at ypur site how can we solve finding 3 consecutive multiples of 11 whose sum is 363
Answered by Penny Nom.
((a-5)/a)/(4/a) 2017-07-18
From Michael:
Find the Quotient:

((a-5)/a)/(4/a)

Answered by Penny Nom.
An arithmetic sequence 2017-07-17
From abbi:
hi there im a student in 11 grade

ive been trying to do this task but i have no idea how to do it the question is
Find the common difference and the n^th term of the arithmetic sequence if the first term is 4 and the twentieth sum of the terms is 1030

Answered by Penny Nom.
Exponential form 2017-07-06
From estela:
what is exponential form of 9×1000
Answered by Penny Nom.
Of cows and men 2017-07-04
From Akash:
In a group of cows and man the number of legs is 32 more than the twice the number of heads.The number of cows is?
Answered by Penny Nom.
The equation of a circle 2017-07-03
From Chen:
Find the center radius and equation of a circle in standard form given the following conditions:

1. Tangent to 3x+2y=0 at the point (0,0) and passing through (1,-1) and (6,0)

Answered by Penny Nom.
y = g(x + 1) 2017-06-27
From Shamsudiin:
HI there i am currently doing a level maths AS

I don't understand how to graph this Question y=g(x+1) i really need help please please respond to this problem

Answered by Penny Nom.
The number of truckloads to fill a hole 2017-06-18
From Jennifer:
How many full size truck bed pickup loads do in need to get to fill a hole that is 18' x 30' that is 4" deep?
Answered by Penny Nom.
The surface area of a hexagonal pyramid 2017-06-16
From Emilie:
Hello, I need help with this question:

A pyramid has a regular hexagonal base with side lengths of 4 and a slant height of 6. Find the total area of the pyramid.

Thank you in advance if you can help me...

Answered by Penny Nom.
The smallest and largest 4 digit integers 2017-06-13
From Avneetpal:
Difference of the smallest and biggest 4-digit number
Answered by Penny Nom.
Solve the equation completely cos 2x = 1 2017-06-08
From Lava:
Solve the equation completely cos 2x = 1
Answered by Penny Nom.
The maximum area of a rectangle with a given perimeter 2017-06-02
From Bob:
How would I go about finding the maximum area of a rectangle given its perimeter (20m, for example)?
Answered by Penny Nom.
4 digit codes with repeating digits 2017-06-01
From Morgan:
what are all possible 4 digit code repeating numbers? cuz i know that you have one website on it but, it doesn't repeat 2 numbers in a code.
Answered by Harley.
3rd Grade math problem 2017-05-30
From Angela:
Following is a problem from my 3rd grader's homework assignment:
Solve. Draw a diagram to represent the situation.
Rachel has 20 one-inch beads. She wants to use all of them to make a square picture frame. What will be the length of each side?
Find Perimeter, Area, and Side Length

Answered by Penny Nom.
Ordering fractions 2017-05-25
From Esmeralda:
How May i order this 30%,3/100,7/20 and 0.33 from least to greatest
Answered by Penny Nom.
Initial velocity 2017-05-23
From Annelle:
a body is thrown vertically downward from a height above the ground. determine the initial velocity and the height of the starting point if after 4 secs it reaches the ground with velocity of 68 m/s.
Answered by Penny Nom.
Two wires between two buildings 2017-05-11
From Bill:
I need to find the height of the intersection point of two building wires< br /> Building A is 860 feet high and building B is 480 feet high. The wires existent from the top of one building to The bottom of the other < br /> The distance between is 32 feet
Answered by Penny Nom.
What is the regular price? 2017-05-11
From Saira:
If the sale price is 55.50$ and the discount is 12% what is the regular price
Answered by Penny Nom.
Canada's population as a percentage 2017-05-11
From bethmarie:
What percentage of the world population (7.5 billion) is Canada's population (36 million). Thank you very much!
Answered by Penny Nom.
Slicing an inverted bowl at various heights 2017-04-30
From Joel:
Find a formula to calculate the circumferences of an inverted bowl at various heights.

E.g. Take an inverted bowl with a diameter of "x" cm and a depth / height of "y" cm.

How can I calculate the circumferences at distances of various heights above the base?

Alternately, what would be the formula to calculate the distance of the line segment resulting from a line which intersects both sides of a parabola in which that line is drawn parallel to the tangent of the vertex of the parabola at any given distance from the vertex?

Answered by Penny Nom.
1 acre as a 2 foot wide strip of land 2017-04-25
From alan:
How can one convert 2 acres (a square - area) of land into a strip of land that is 2 feet wide and (?) feet long?
Answered by Penny Nom.
The length of a train 2017-04-24
From Stefano:
If a freight train is coming towards you and you were traveling at 85 miles an hour and it takes 35 seconds for the train to pass you how long is the train

Figure 50mph for train

Answered by Penny Nom.
Baseball Cards 2017-04-20
From Raquel:
A minor league baseball team gave away baseball cards to each person entering the stadium. One group receives 28 baseball cards. A second group received 68 baseball card. If each person entering the stadium receive the same number of cards, what was the greatest possible number of baseball cards that each person could have received?
Answered by Penny Nom.
A function which is onto but not one-to-one 2017-04-16
From Avinash:
Define a function f(x):N-->N which is onto but not one-one.Where N is set of natural numbers.
Answered by Penny Nom.
Forming a cone from a circle 2017-04-15
From Tasha:
A sector of a circle subtends an angle of 216 degrees at the centre, If this sector is used to form a cone of vertical height ,8cm, calculate the base radius of the cone
Answered by Penny Nom.
The cost of a piece of sheet metal 2017-04-14
From Mohammed:
I bought a 2 mm sheet plate of 1.2m × 2.4m in fijian $95. I used 1.95m × 0.4m from that sheet plate. What will be my total cost for the sheet plate I used. It was a mild steel plate.
Answered by Penny Nom.
Why does 10x10x10 give 1,000.0000000000001? 2017-04-12
From Randolph:
Hi, On your calculator I found that a box 10 by 10 by 10 inches has a volume of 1,000.0000000000001 cubic inches. Can you explain the numeral 1 thirteen places past the decimal? Thanks, Randy
Answered by Penny Nom.
A rectangular box filled with oil 2017-04-11
From morpal:
we want to calculate volume of rectangular box height 1275mm and lengths 2400mm and breadth 1270mm and oil s. gravity is .950 how much kg. of oil i can store
Answered by Penny Nom.
Successive Differences 2017-04-06
From Twaha:
Please find the equation of the sequence 1 2 4 7 11 16 22
Answered by Penny Nom.
The height of a isosceles trapezoid 2017-04-03
From Riham:
Hi
How can I find the height in an isosceles trapezoid if I have the measurements of all of its sides ? Thank u in advance.

Answered by Penny Nom.
13 - 3x = x times the square root of 5 2017-03-29
From Kevin:
A student I am tutoring was sent this question from his teacher.

Solve for x: 13 - 3x = x times the square root of 5

Answered by Penny Nom.
How much money do I make per sq ft? 2017-03-28
From jackie:
If I make $12.00 per hour and I cover 750 sq ft in one hour, how much money do I make per sq ft?
Answered by Penny Nom.
The sum and difference of two numbers 2017-03-20
From Olivia:
The sum of two numbers is 87 and their difference is 29. What are the two numbers
Answered by Penny Nom.
Differentiate y = x^x^x 2017-03-19
From Nafis:
differentiate y = x^x^x
Answered by Penny Nom.
Is infinite a number? 2017-03-18
From Divyansh:
Is infinite a number? If yes why as i think that numbers are used for counting but infinite is undeterminable?
Answered by Penny Nom.
Faster than as a percentage 2017-03-18
From Maximilian:
Hi, i have a strange issue with the phrase "faster than" in percentage, ie. if i have a 10MB/s connection, and somebody has a 7MB/s connection, is it 30% faster or 50% faster.- which ( affirmation, phrase, sentence ) is the right one:

a:) My 10MB/S connection is 50% faster than the 7MB/s ?
or
b:) My 10MB/S connection is 30% faster than the 7MB/s ?

Thanks in Advance, and please if you can teach me why !

Answered by Penny Nom.
2^a, 2^b, 1^1 and 1^0 2017-03-17
From Émile:
If you had 2^a = 4 and 2^b = 4 you could assume that a = b right? Yet we can observe that 1^1 and 1^0 both equal 1, yet 1 = 0 isn't true. So if given a log base 1 number 1 the would the answer be 1 AND 0?
Answered by Penny Nom.
Solve 3x^2-19x-14=0 for x 2017-03-15
From nick:
I'm an 8th grader and am quite confused with this equation: 3x^2-19x-14=0

The answer is:
-2/3 , 7

But I'm not sure how to solve for it.

Answered by Penny Nom.
The ratio of two ages 2017-03-15
From Angel:
The sum of Keith's age and Alan's age is 36. Four years ago, the ratio of Keith's age to Alan's age was 4:3. Find the ratio of their present age
Answered by Penny Nom.
What is the grade of the incline? 2017-03-12
From Jon:
If I am climbing 16 feet in elevation over 80 feet horizontal. What is the grade of the incline
Answered by Penny Nom.
How much does the fish weigh? 2017-03-08
From Jeri:
Fish is caught that weighs 5 pounds plus half its weight. How much does the fish weigh?
Answered by Penny Nom.
More on calculating the area (acreage) of a four-sided lot 2017-03-06
From Kerri:

Can you please show the formula used to calculate areas as provided on this page:

Calculating the area (acreage) of a four-sided lot


Answered by Penny Nom.
Exponential form 2017-03-04
From michael:
I was teaching my student about exponent when we stumbled across this problem.

"write (-125)^-3 in exponent form"

The problem is with the number "-125". I think -125 = (-5)^3. But one of my student thought that it is -125= -5^3.
Both of them is equal to -125 but they totally different in structure. I wonder which one is correct and why it is. I am worried if they get this wrong, it might spell problem in the future when we start substituting the numbers with variable.
Thank you very much for reading my question and answering it.

Answered by Penny Nom.
The inside diameter of a silo 2017-02-28
From JOHN:
I need to find the inside diameter of a cement silo with an outside circumference 118 ft and a wall thickness of 7 inches. thank you John Kerr.
Answered by Penny Nom.
Building a hexagon enclosure 2017-02-26
From Kenneth:
Hello, I have a question for you. I have a special needs daughter and I want to install a ceiling fan into her room. The fan is 42 inches in diameter. I would like to build a wooden hexagon enclosure to protect the fan. I would like the enclosure to be at least 2 inches wider than the fan on each side measuring from flat surface across to flat surface. How do I figure out the length of each panel so that I can miter them and assemble it? Thank you for your time.
Answered by Penny Nom.
Feet per second to miles per hour 2017-01-27
From ron:
if a vehicle travels 50 feet in 2.5 seconds what is the speed in mph. 3600 sec in hour /5280 feet in a mile i get .68181818. how do i figure the 2.5 seconds. all the calculators I've used show 34 mph is that correct or am i forgetting the 2.5 second. please help.
Answered by Penny Nom.
The volume of a slab 2017-01-21
From Russell:
Working with a segment of a circle. The O.D. is 3.440" the length of the chord is 3.130 I need information on the volume of the largest area. I would like to know (in thousands of an inch) how tall would the largest area need to be to equal .1143 cubic inches If you need more info I can send it Thanks in advance
Answered by p.
The average rate of change of cot(t) 2017-01-18
From Brianna:
Hello! It's been a while since I've taken a math course, and I'm stuck on a problem in my calculus course.
The question is this:

Find the average rate of change of the function over the given interval.
h(t)=cot(t) a) [5pi/4, 7pi/4]

Answered by Penny Nom.
Five equations 2017-01-16
From Muhammed:
A x 4 = E
B ÷ 4 = E
C + 4 = E
D - 4 = E
___________
& A + B + C + D = 100
what is the value of E

Answered by Penny Nom.
One millionth of one percent of 90,000,000,000 2017-01-16
From Gene:
What absolute number is one millionth of one percent of 90,000,000,000
Answered by Penny Nom.
8^3/2(2+2) 2017-01-13
From Mary:
8^3/2(2+2)
Answered by Penny Nom.
Water leaking from a trough 2016-12-28
From Kathryn:
A trough is 6 m long, and has uniform cross-section of an equilateral triangle with sides 1 m. Water leaks from the bottom of the trough, at a constant rate of 0.1 m3/min. Find the rate at which the water level is falling when the water is 0.2m deep.
Answered by Penny Nom.
An octagon from a square 2016-12-26
From Chris:
We have a square piece of wood (30 inches x 30 inches) that we need to cut into an octagon. Is there a simple way to figure out where to make the marks/points to cut? I mean like really simple?

Thanks!

Answered by Penny Nom.
20+30*0+1=? 2016-12-26
From Mr:
20+30*0+1=?
Answered by Penny Nom.
Simultaneous equations with fractions 2016-12-21
From Kimi:
I am stuck on this,can someone please help me????

1/2x+1/3y=11
8x+2/5y=102
Can someone please explain how to solve questions like these?? Or else I will never learn...
Thank you

Answered by Penny Nom.
Two concentric circles 2016-12-21
From shrestha:
Two concentric circles have radii of 14 cm and 7 cm respectively. Find the area of space between them.
Answered by Penny Nom.
The inverse of y=x^2+x+c 2016-12-21
From Sam:
What is the inverse of y=x^2+x+c for x?
Answered by Penny Nom.
The inverse of y = x(x - 2) 2016-12-21
From Sasha:
Hello, I recently stumbled upon this question and I haven't been able to figure it out.

Why is the inverse of y=x(x-2) not a function?

Suggest a domain restriction which would ensure that the inverse is a function.

Thank You :)

Answered by Penny Nom.
The area and radius of a circle 2016-12-19
From india:
How do you find the radius of a circle if only the area is given to you?
Answered by Penny Nom.
Joining a fitness center 2016-12-18
From Salem:
A fitness center offers a special price for group-class students. The cost of registration is 30 dollars and then you have to pay 25 dollars per month. What is the rule of the equation in the light of
c=---m+----?

Answered by Penny Nom.
980 cubic yards of dirt 2016-12-17
From Jondi:
I am trying to visualize 980 cubic yards of dirt by using a metaphor.
Would this be the size of a car?
A fire engine?
A semi-trailer?
How many trucks would you need to haul away 980 yards of dirt?

Answered by Penny Nom.
1/1-cosine(2x) - 1/1+ cos(2x) 2016-12-14
From Sean:
1/1-cosine(2x) - 1/1+ cos(2x)
Answered by Penny Nom.
How far apart are the boats? 2016-12-13
From Halley:
Two boats leave port at the same time. Boat A travels east at a speed of 12 km/hr. Boat B travels southwest at a speed of 14 km/hr. After two hours, how far apart are the boats? North is 0 degrees. How do I figure this out. Thanks
Answered by Penny Nom.
Profit and loss on two shirts 2016-12-06
From Mahesh:
A man purchases one shirt and one T-Shirt for Rs. 6000. He sells the shirt at a profit of 20% and T-Shirt at a loss of 10%, as a result he gains 2% on whole transaction. what is the cost price of the T-Shirt ?
Answered by Penny Nom.
Gallons and cubic inches 2016-12-05
From Theresa:
If I did not know the cubic inches in a liquid gallon how would I find it? In other words what is the formula for this?
Answered by Penny Nom.
Covering a floor with plank flooring 2016-12-04
From sandra:
my bathroom is 5'x6'.2"
i want to purchase plank flooring which covers 19.8 sq ft per box.
how many boxes would i need.

Answered by Penny Nom.
2^x= -2x - 11 2016-11-27
From Kathy:
I don't know how to start this problem:
2^x= -2x - 11

Answered by Penny Nom.
The weights of two boxes 2016-11-25
From dearbhla:
One box weighs 3 3/7kg. Another box weighs 1 5/6 as much. How heavy is the second box?
Answered by Penny Nom.
Why are equivalent fractions equal? Part 2. 2016-11-22
From Anandmay:
Hi! I saw one of your answered questions:Why are equivalent fractions equal? The same question I had for why i searched and ended up entering this wonderful site. The answer was so much experimental based.I mean,for example,it was explained how 3/4 was equal to 6/8 by dividing a unit into 4 equal parts and taking four of the parts,and then dividing the same unit into 8 equal parts and then taking 6 of them.The result was that both the taken parts were equal. (That is,3 out of 4 equal parts were equal to 6 of the 8 equal parts.) But,this was very much non-generalized and experimental-based result(proof). And we all know that in mathematics,we confirm some particular thing for all the numbers iff we generalize it.

So,can you explain more clearly 'HOW' the 3 parts of the 4 equal parts and the 6 of the 8 equal parts of the unit equal(that is the same)?I don't want experimental proof.I want clear proof,and thus please give a generalized proof for all fractions and their equivalents.
Thanks!

Answered by Penny Nom.
The amount of material remaining on a roll 2016-11-11
From Yoh:

Question from Yoh:

Hello,
I am trying to find impressions on a roll (either full roll or partial).
Let's say I have the following information.
- Outer Diameter of roll
- Inner Diameter of roll (cardboard core)
- thickness per layer
- Each cut off length (impression length)

Now let's say a roll has a 40in outer diameter, the cardboard core has a 3.75in diameter and a thickness of .002. Each impression has a cut off of 14inches.
With this the roll will have approximately 2,700 impressions. How would I find the remaining impressions if the outer diameter of the roll is only 6.5inches?

Thank you.


Answered by Penny Nom.
An equation with fractions 2016-11-09
From Tina:
How do you solve x+1/3=x+2/5. X+1 divided by 3= x+2 divided by 5. What's x=
Answered by Penny Nom.
A pattern rule for a sequence 2016-10-28
From Grace:
Here's a Grade 7 question, we need to find the pattern rule:

1 - 1
2 - 3
3 - 6
4 - 10
......
It may involve exponents, but we can't seem to figure it out.
thank you.
Grace.

Answered by Penny Nom.
The derivative of the inverse of a function 2016-10-28
From Kate:
Hi, I'm in a College level Calculus course and I can't seem to figure out the answer to this question.
Find the derivative of f^-1(4) if f(3)=4 and f'(3)=1/7

Answered by Penny Nom.
y = mx + c and Ax + By + C = 0 2016-10-18
From Erick:
please explain to me how to convert an equation from the form y=mx+c to Ax+By+C=o form.
Answered by Penny Nom.
The degree of 2x + 7 2016-10-17
From Deejay:
How about when the given has no degree and the other has a constant only, For example 2x + 7, where's the leading term and what is its leading coefficient?
Answered by Penny Nom.
A circle formed from bending a wire 2016-10-16
From Deszaree:
The same piece of a wire is bent to form a circle
calculate:
a, the radius of the circle
b, the area of the circle

Answered by Penny Nom.
A linear relationship 2016-10-16
From Bianca:
x represents the number of hours since 8am. y represents the number of children the school nurse has seen during the school day. The nurse has seen 3 children by 8am and 27 children by noon.
How many children has the nurse seen at 2pm?
If the nurse has seen 18 children, what time is it?

Answered by Penny Nom.
A five digit number 2016-10-12
From Massa:
Explain why any four digit number is less than any five digit number?
Answered by Penny Nom.
Subtraction of two numbers in different bases 2016-10-07
From Wica:
Hello,

So happy I found this site. I am having problems with bases. My question is :
Please perform the subtraction between two number over different bases :
A. (5874)base 12 - (216)base 9
B. (216)base 9 - (5874)base 12

Answered by Penny Nom.
The equation of a line 2016-10-01
From Miriam:
write the equation of the line that passes through (6,7) and is perpendicular to 3x+5y=0
Answered by Penny Nom.
The equation of a circle 2016-09-30
From Trent:
write the equation of circle B with center B(-2,3) that passes through (1,2)
Answered by Penny Nom.
Jenny, Charles and Chase have some butterflies 2016-09-26
From Kyla:
Jenny cares 7 times as many butterflies as Charles. Charles catches 2 times as much as Chase. All together they have 560 butterflies. How many butterflies does each person have? Please help, I am very confused on how to solve the problem.
Thank you!

Answered by Penny Nom.
The length of stock on a roll 2016-09-23
From Ken:
I have a roll that is 13.5 inches wide ( OD) , and is wound on a 3 inch core. the thickness of the material is 3.5 thick.
Do you know of a formula that can calculate the linear feet on that roll??

Answered by Harley Weston.
Standard form 2016-09-22
From anna:
ok. i have b=2/3x+5/7. im supposed to change it to standard form. how do i do that?
Answered by Penny Nom.
Two equations 2016-09-17
From Nilesh:
How to solve (x-y)^1/2 + 3(x-y) = 30 ; xy + 3(x-y) = 11?
Answered by Penny Nom.
The radius of a coffee cup 2016-09-12
From Brett:
What is inside radius, in centimeters of a coffee cup if it holds 350g of coffee when filled to a depth of 9.5 cm? Assume coffee has the same density of water, 1.00g/cm3. A numeric value is expected and not an expression.
Answered by Penny Nom.
Four digit numbers 2016-09-08
From ning:
0000-9999 there is ten thousand 4-digit numbers. How to find the number of ways that 4 digit are the same? How to prove it by using formula?
Answered by Penny Nom.
The equation of a ski slope 2016-08-31
From Hannah:
The information provided is
---A ski slope has a slope of -0.2. You start at an altitude of 10,000ft. Then the question is "Write the equation giving altitude (a) as a function of horizontal distance moved (d).
I know this is probably a very simple question but I didn't take a math class my senior year and now I'm a freshman in college, so some of it is very frustrating to me.
Any help is much appreciated, thank you!

Answered by Penny Nom.
The dimensions of a rectangle 2016-08-31
From Hazym:
How do I find the length(which is 15m longer than the breadth) and the breadth of a rectangle just by its perimeter which is 70 m?
Answered by Penny Nom.
Exponential form 2016-08-31
From Lily:
Hi! I was wondering, I was having a little issue understanding this one problem. It was "write 81 in exponential 2 different ways" and I already got 9(2) but I was wondering what the second one could be! Thanks! -Lily
Answered by Penny Nom.
5/8 of 16 2016-08-14
From Michael:
What is the fraction of 5/8 of 16
Answered by Penny Nom.
Covering an octagonal region with sand 2016-08-13
From Lynn:
How much sand is needed to fill a 20 ft. wide octagon area 6 inches deep?
Answered by Penny Nom.
The area of a circle 2016-08-12
From sandeep:
if circumference of a circle is πd what is the area?
Answered by Penny Nom.
Spraying an acre 2016-08-07
From Sara:
I have a spray rig that is 80' wide. How many feet must I go to have sprayed an acre?
Answered by Penny Nom.
The diameter of a cylindrical tin 2016-08-04
From Scott:
a cylindrical tin is 15 inches high and holds 5 gallons of paint when it is full. Find the diameter of the tin.
(1 gallon = 0.1605 cubic feet)

Answered by Penny Nom.
The dimensions of a rectangle 2016-07-25
From Krunox:
A rectangle is twice as long as it is wide. Its perimeter (the sum of the lengths of its sides) is 60 cm. Find its length and width.
Answered by Penny Nom.
A cubic equation 2016-07-25
From Chinmay:
x³-2x²-7x+5
Answered by Penny Nom.
The number of sides of a polygon 2016-07-23
From Shriya:
All the angles of a polygon are either 155° or 140°.
There are twice as many angles of 155 °or 140°.
Find the number of sides of the polygon

Answered by Penny Nom.
The temperature inside and outside a greenhouse 2016-07-22
From Mehzad:
Every day, the temperature in a greenhouse is at its low temperature, 70 degrees Fahrenheit, at 2 a.m. and at its high temperature, 84 degrees Fahrenheit, at 2pm. Its temperature increases linearly between 2 am and 2pm , and decreases linearly from 2 pm and to 2am. The outside temperature follows the same linear patterns, but has a low temperature of 60 degrees Fahrenheit at 2 am and a high temperature of 78 degrees Fahrenheit at 2 pm. At which of the following times will the temperature inside and outside the greenhouse be the same?

a) 8:00 am
b) 12:00 noon
c) 4:00 pm
d) The two temperatures will never be the same

Answered by Penny Nom.
Two equations with fractions 2016-07-22
From kanesh:
p/2+q/3=3

p/4+2q/3=3

Answered by Penny Nom.
Which term of this sequence has value 8? 2016-07-20
From Lauren:
Hi there

Which term in the sequence -2; -5/3; -4/3 ; -1; ... has a value of 8.

Since term 2 and 3 of this sequence contain fractions which can be converted to recurring decimals. What is the best way to work out the common difference here.

I do however understand that to work out the nth term of an arithmetic series, the following formula Tn = a + (n-1)d. In this series a = -2

Answered by Penny Nom.
Writing numbers in the form a/b 2016-07-17
From Edward:
How to write these numbers in form of a/b: -7, 0.175, -3.25, 37/10, -16, 1/3, 2.04
Answered by Penny Nom.
A congruence theorem for two right angled triangles 2016-07-17
From Sayari:
Hello. Is it possible for two right angled triangles to have the same length of base and height and a different hypotenuse? If not so, then why in the congruence criteria RHS the hypotenuse is given more importance? It can also be like- 'two triangles are congruent if they are right angled and have the same base and height.' Thank You.
Answered by Penny Nom.
The composition of a function with itself 2016-07-17
From Mel:
If f(1 + 3x) = 1 * x, solve f(f(x))
Answered by Penny Nom.
Triakis 2016-06-27
From Gordon:
Some authorities treat "triakis" (and related terms "dyakis", "tetrakis", etc.) as attached prefixes; others treat them as separate adjectives. Thus I see one of the Catalan solids described as both "triakistetrahedron" and "triakis tetrahedron". Which usage is correct?
Answered by Chris Fisher.
Subsets 2016-06-26
From Kats:
How Many sub sets are in set k={6,7,3}
Answered by Penny Nom.
find f+g 2016-06-26
From yzabelle:
find f+g
f(x)=x^2-1
g(x)=square root of x+2

Answered by Penny Nom.
Cubic feet of soil 2016-06-19
From H:
A garden is 8 ft x 7 ft that needs to raised 6 inch's How many cubic feet of soil required ? What am I missing ?

Thanks

H

Answered by Penny Nom.
Profit 2016-06-15
From Toyia:
12.87 profit from 21.99 sale. What is the percentage that was taken off the 21.99
Answered by Penny Nom.
Implicit differentiation 2016-06-06
From Pranay:
Is a circle x^2+y^2=2 a function? If it is not a function, why is it possible to do implicit differentiation on it?
Thanks.

Answered by Penny Nom.
A barn roof 2016-05-29
From Joe:
Is it possible to build a barn roof (irregular pentagon?) with a 12' base and the other 4 sides 4' each? Thanks.
Answered by Penny Nom.
A cylinder in a cube 2016-05-25
From Sonia:
A cylinder has the same height and diameter as the sides of a cube use 3.14~ which has the greater volume
Answered by Penny Nom.
The area of a 4-sided lot 2016-05-25
From prasad:
How to find area of a land whose sides are 41ft,33ft,32.3ft and 33.2 ft. Pl give me the formula and proof.
Answered by Penny Nom.
Two pieces of rope 2016-05-21
From Render:
Fred cuts a 12- inch piece of rope. Then he cuts a second piece of rope that is 3/2 longer than the first piece. Is the second piece shorter or longer than the first piece? Explain.
Answered by Penny Nom.
A trough with a triangular cross section 2016-05-21
From Clarice:
A trough having an equilateral triangle end sections has sides equal to 0.4 m and 7m long.what is the volume of the liquid in the container if the depth of the water is one half the depth of the trough?
Answered by Penny Nom.
Euler's polyhedron formula 2016-05-20
From Antonia:
why does not cylinders follow Euler's polyhedron formula ( V-E+F=2) ?? if a cylinder has 3 faces, 2 edges and 0 vertices it doesn't follow Euler's formula???
Answered by Penny Nom.
Filling a pit with glass pebbles 2016-05-17
From Sam:
I need to know how many pounds of glass pebbles are needed to fill a 24 inch across circular fire pit, if 5 pounds covers 4"H x 4"W x2"D? Thank u for any assistance, Sam
Answered by Penny Nom.
The volume of a truncated rectangular pyramidal pond 2016-05-13
From Paul:
How do you calculate a partially filled truncated rectangular pyramid if you always know the bottom rectangle, the maximum height top rectangle perimeter, but have a varying height. Similar to filling up a pond you know the current height and dimensions at the max rectangle how do you calculate it half full i.e. 10x30 outside perimeter with a 2x8 base and a max height of 6ft how do you calculate it at 3ft without re-measuring the top perimeter. Thanks, Paul
Answered by Harley Weston.
A pair of equations with fractions 2016-05-11
From tiya:
hello, i want to know how to solve this question.

m/6+2n/3=6
-m/10=2n/5=2

Answered by Penny Nom.
A rectangular field 2016-05-07
From Jan:
A rectangular field is 63m long and 21m wide. A fence of 2m in height is needed to fence the field and fencing is also required to divide the field in three square sections. There are three wooden gates measuring at 1.5m to the three sections of the field. What is the total surface area of the fence needed for fencing and diving the field?
Answered by Penny Nom.
A six digit code 2016-05-06
From Ralph:
How many combinations are possible if a "safe" has four buttons and you can use a six digit code? (these are just single press buttons..)For example, this could be one possible combination: Button 4 Button 1 Button 1 Button 3 Button 2 Button 4. Thanks!
Answered by Penny Nom.
Solve 2^2x + 3(2^x) - 4 = 0 2016-04-27
From Lloyd:
Solve the equation 2^2x + 3(2^x) - 4 = 0
Answered by Penny Nom.
sin 2x - sin x = 0 2016-04-24
From lilly:
sin 2x - sin x = 0 0
Answered by Penny Nom.
The inverse function for f(x)=2x^2+5x 2016-04-23
From Ashini:
The inverse function for f(x)=2x^2+5x
Answered by Penny Nom.
A sector of a circle 2016-04-21
From mustafa:
In a sector of a circle, the arc length is equal to half the perimeter of a sector.find the area of a sector in terms of r
Answered by Penny Nom.
15.86 acres 2016-04-21
From Ronald:
If a rectangular piece of land is 1320 ft. on one side, how many feet would be required on the other side to equal 15.86 acres?
Answered by Penny Nom.
A square of side length 4x-3 units 2016-04-20
From Bryan:
Find an expression that represents the area of a square with side length 4x-3
Answered by Penny Nom.
The derivative of x! 2016-04-16
From Sang:
How to find the derivative of x! and integral of x!
Answered by Penny Nom.
Sloping my yard away from my house 2016-04-15
From Harold:
I need to slope a area of my yard away from my house and garage. The length of my house is 80 feet long and i need to slope it 22 feet away. I want to raise the dirt at the foundation by 6 inches and slope it down to nothing at the width of 22 feet. How much soil would i require?
Answered by Penny Nom.
A scale factor 2016-04-13
From tonya:
What is the scale factor for the two figures regular octagon with 15 and 9 written in decimal form
Answered by Penny Nom.
tan15° 2016-04-11
From JOHN:
find the exact value of tan15° in surd form.
Answered by Penny Nom.
The area of a circle and a square 2016-04-10
From Richard:
I cannot understand why if you have a perimeter say of 4 units as the circumference of a circle then you reshape it into a square you lose very approximately some 20% of its volume but the perimeter length stays the same.
Can you please explain this to me as to why the volume decreases.

Answered by Penny Nom.
Filling a hole with 14 tons of rock 2016-04-08
From Barry:
If I had 14 tons of inch and one half sized aggregate rock, how large of a square or rectangular shaped hole would I need to hold that amount?
Answered by Penny Nom.
The last two digits of a phone number 2016-04-03
From Joshua:
I want to find the last two digits of a phone number whose first eight numbers of a ten digit phone numbers are known.
Answered by Penny Nom.
The volume of a cone 2016-03-31
From Odum:
Find the volume of a cone with radius 6.5cm and height 12.6cm
Answered by Penny Nom.
The remainder theorem 2016-03-27
From Pratyasha:
A quadratic polynomial when divide by x+2 leaves a remainder of 1 and when divided by x-1 leaves a remainder of 4. What will be the remainder if it is divided by (x+2)(x-1)?
Answered by Penny Nom.
The volume of a box with a hexagonal base 2016-03-27
From Frank:
how many gallons of water in an 8" hexagon shaped box. 6" sides x 12.5" high ? The walls are 1" thick , beveled @ 30 degrees, so inside dimensions are 6"
Answered by Penny Nom.
Travelling in a spacecraft 2016-03-25
From julie:
if i was travelling in a spacecraft at the speed of light and i turned on my headlight would i see the beam ?
Answered by Robert Dawson.
A rotating schedule for a softball team 2016-03-25
From Caroline:
I couldn't find a similar question, hopefully there isn't one!

I have a softball team of 18 people (7 girls and 11 boys), but am limiting each game to 13 players (6 girls and 7 boys). The season is 10 games long. How do I create a rotating schedule which allows members to play evenly? I tried separating the boys and girls to create rotation for each but I got confused. Some are unable to attend all games but for the simplicity of a rotation schedule can you please help?

Answered by Robert Dawson and Victoria West.
The inverse of a function 2016-03-20
From Billy:
f(x) = Square root(x^2 + 2x)

What is the inverse?

Answered by Penny Nom.
The perimeter of a rectangle 2016-03-17
From Linda:
Noel correctly adds the length of three sides of a rectangle and gets 88 cm her brother Ryan correctly adds the lengths of three sides of the same rectangle and gets 80cm. What is the number of cm in the perimeter of the rectangle
Answered by Penny Nom.
A block wall 2016-03-15
From Robert:
I have a wall 40 ft. long and 4 ft. tall as well as another one that is 30 ft by 3.5 ft. tall that I am trying to build. how many 6 inch by 12 inch landscaping blocks do I need?
Answered by Penny Nom.
The sides of a triangle 2016-03-15
From Hitarth:
Why we cannot construct a triangle ofngiven sides 5cm,5cm and 10cm?
Answered by Penny Nom.
Angles 2016-03-12
From Laurynn:
What are angles in general (please include the 'angle of incline')

Thank you
Laurynn

Answered by Penny Nom.
The integral of a sum 2016-03-10
From Rahul:
How to solve definite integral of a sum. The specific problem is as follows,
Integral of ( 1+ sum of x^k, k=1 to k=n), x=0 to x=b *dx.
The answer is b + sum of b^(k)/k, k=2, to k=n+1. I understand only the integral of first term. But integral of the sum I do not understand at all.

Answered by Penny Nom.
An equation with fractions 2016-03-09
From Ed:
7/10-___ +3/2=6/5
Answered by Penny Nom.
Tiling a floor 2016-03-05
From joanne:
How many floor tiles 20x20 inch do I need for area of 8x 12 ft.?
Answered by Penny Nom.
The area of a rectangle 2016-03-03
From Lucas:
The area of a rectangle is given by A=x2+18x+72
-use factoring to find an expression for the dimensions of the rectangle.
-if the area of the rectangle is 7 square feet, what are the possible values of x?
-what are the dimensions of a rectangle?

Answered by Penny Nom.
Corrosion inhibitor in a pipeline 2016-03-02
From Edu:
I have a 12 in pipeline and 5 miles long. I need to coat it with corrosion inhibitor how much chemical will I need
Answered by Penny Nom.
A rectangular prism 2016-03-02
From Paula:
Trying to help with a math problem. I don't know how to set up the equation for this.

A rectangular prism has a volume of 7.875m3rd and a height of 3.5 m. What is the area of the base of the prism?

Thanks for your help!

Answered by Penny Nom.
An algebra problem with a sphere 2016-02-29
From Sania:
If the number of square centimeters on the surface area of a sphere is equal to the number of cubic centimeters in its volume find its diameter
Answered by Penny Nom.
A deck that is half an ellipse 2016-02-28
From Steve:
On your website, I was reading a question and your response from a girl named Angela in which you provided a formula by which her father, a welder, could figure out points on an arc corresponding to equal 3' intervals on a 30' chord where the vertex was 1' off the chord. Is there an equivalent formula when working with an ellipse? I suspect this change will make the calculations significantly more complex. I am building a deck that is half an oval, and would like to be able to mark out the perimeter by measuring the distance from regular intervals on the primary access to a corresponding point on the perimeter. I will then connect the points on the perimeter and cut a reasonably smooth arc. The length of the primary access will be 22' and width of the deck at the vertex is 9'. I would like to be able to know the distance from the primary axis to a point on the perimeter at equal intervals of 6" along the primary axis. Can you help?
Answered by Penny Nom.
10800 sq. ft equal approx what part of an acre? 2016-02-27
From David:
10800 sq. ft equal approx what part of an acre
Answered by Penny Nom.
The sum of the angles of a triangle 2016-02-24
From Sophia:
Does every triangle add up to 180 degrees? (Such as a unique triangle)
Answered by Penny Nom.
The weight of some dirt 2016-02-24
From Michael:
How many tons of dirt are in a triangle 14M x10M x 8.1M x .8M deep?
Answered by Penny Nom.
Converting a fraction to a percentage 2016-02-22
From Preitty:
How can I turn fractions into percentages when the denominator is not a number that can multiply to 100?
Answered by Penny Nom.
The domain and range of a function 2016-02-19
From Genius:
State the domain and range g(x)=x(x-1)
Answered by Penny Nom.
The interior and external angles of a regular polygon 2016-02-17
From percy:
a regular polygon has n sides .The size of each interior angle is eight times the size of each exterior angle .
1.find the size of each exterior angle
2.calculate the value of n

Answered by Penny Nom.
Backfilling a rectangular hole for a shop foundation 2016-02-08
From Jeremy:
Hello, I'm going to be backfilling a rectangular hole for a shop foundation and need to know how much concrete and gravel I'm going to need to fill it in. The shop foundation is 38 feet wide by 30 feet long and 4 feet deep, there is also a concrete perimeter going inside the foundation that measures 16 inches wide and goes all the way around the inside of the foundation
Answered by Penny Nom.
The equation of a vertical line 2016-02-07
From Kagiso:
The equation for a vertical line that goes through the point (7; -3) is?
Answered by Penny Nom.
An exercise with factorials 2016-02-07
From vidhi:
find the value of n: (1-1/2) (2-2/3) (3-3/4)...(15-15/16) = n!/16
Answered by Penny Nom.
The midpoints of the sides of a quadrilateral 2016-02-05
From Andrea:
The segments, joining, in order the midpoints of consecutive sides of a quadrilateral form a parallelogram.
Answered by Penny Nom.
The height of a triangle 2016-02-03
From Brooklyn:
How do I figure out the height of a triangle when it only tells me the base of he triangle? And vice versa.
Answered by Penny Nom.
Golf for 6 - 2 three balls 2016-02-01
From Andy:
We are shortly having a golf holiday in Phoenix. There are 6 people playing 6 rounds, so we will play in 2 three balls. Is there a combination of pairings that ensures everyone plays with each other the same number of times. Many thanks for your help ! Andy
Answered by Victoria West.
Which term of the series 2+7+12+...is 152? 2016-02-01
From francis:
whice term of the series 2+7+12+...is 152?
Answered by Penny Nom.
Simplify 2016-02-01
From Sabira:
Simplify step by step
5x/8 + 7y/18 = 6

Answered by Penny Nom.
Subtraction in a base larger than ten 2016-01-31
From Jenalee:
Hello,

So happy I found this site. I am having problems with bases. My question is 97B9 - 6A3A base 16 for both.

Please help.

Answered by Penny Nom.
The diagonals of a polygon 2016-01-29
From Sofia:
What are diagonals and if you drew an octagon and drew all the diagonals how many would there be? Please explain because this is pretty confusing! Thank you!
Answered by Harley Weston.
Consecutive angles of a parallelogram 2016-01-28
From Hanna:
The consecutive angles of a parallelogram measures
Answered by Penny Nom.
Exponential form 2016-01-26
From Jufe:
What is the exponential form of ³√11x²
Answered by Penny Nom.
The range of a function with a finite domain 2016-01-25
From Hannah:
Solve y=(1⁄4)x-1 if the domain is (-4,-2,0,2,4). I don't get how to do this. Can anyone help?
Answered by Penny Nom.
A system of linear equations 2016-01-24
From kareem:
my name is kareem
and i am a student i have a math puzzle and i tried to solved it but it always have same mistake
x-y=9
x+z=12
z-n=14
y+n=2

Answered by Penny Nom.
Fractions of two quantities 2016-01-22
From Melody:
Kate ate 1/4 of her orange. Ben ate 2/4 of his banana. Did Kate and Ben eat 3/4 of theit fruit? Explain.
Answered by Penny Nom.
A Max/Min problem with an unknown constant 2016-01-17
From Guido:
Question:

The deflection D of a particular beam of length L is

D = 2x^4 - 5Lx^3 + 3L^2x^2

where x is the distance from one end of the beam. Find the value of x that yields the maximum deflection.

Answered by Penny Nom.
The perimeter of a square 2016-01-16
From dennis:
If I have eleven Square Acres, what would the perimeter dimensions be?
Answered by Penny Nom.
A limit of a rational function 2016-01-16
From selvamani:
F (x) = x^3+3x^2-9x-2 / x^3-x-6 and limit x tends to 2, f (x) exist then limit x tends to 2, f (x) is equal to ? How to answer this problem.
Answered by Penny Nom.
Complex numbers in standard form 2016-01-15
From Michael:
express the following complex numbers in standard form (2+3i)+(5-2i)
Answered by Penny Nom.
Solve 1/3 ( x-5 ) + 4 =1/6 ( 2 - 3 x ) +1 for x 2016-01-15
From edwin:
1/3 ( x-5 ) + 4 =1/6 ( 2 - 3 x ) +1
Answered by Penny Nom.
The range of a function defined piecewise 2016-01-13
From sarah:

f(x)={x-2, x<-2
       {x^2-4, -2_<x_<2
       {x+2, x>2

find the range
Answered by Penny Nom.

Sue's favourite number 2016-01-12
From Maria:
This is my son's homework. He is 7. Can you help?

Sue has a favourite number.
It is an even number.
It has 3 digits.
The digits add up to 15.

Find 10 numbers that could be Sue's number?

Answered by favourite number.
The perimeter of a triangular plot of land 2016-01-11
From Janelle:
I need to fence off 6.5 acres. The plot is triangular shape. How many feet would I be fencing off?
Answered by Penny Nom.
Water flowing through a rectangular drain 2016-01-08
From Tiffany:
the cross section of drain is a rectangle 30 cm wide. If water 3.5 cm deep flows through the drain at a rate of 22 cm/s, how many litres of water will flow through in one minute?
Answered by Penny Nom.
The area of a rectangle 2016-01-08
From ranjan:
if we make a rectangle with a 100cm of wire, then its area will be?
Answered by Penny Nom.
Two transport companies 2016-01-06
From Brynleigh:
The Sugar Sweet Company will choose from two companies to transport its sugar to market. The first company charges $5096 to rent trucks plus an additional fee of $200.75 for each ton of sugar. The second company charges $5500 to rent trucks plus an additional fee of $175.50 for each ton of sugar. For what amount of sugar do the two companies charge the same? What is the cost when the two companies charge the same?
Answered by Penny Nom.
A sequence 2016-01-05
From Mia:
the next three terms in each sequence. 0.4, 0.54, 0.68, 0.82,
Answered by Penny Nom.
A graph of the water level of a river 2016-01-05
From emoni:
Suppose that the water level of a river is 34 feet and that it is receding at a rate of 0.5 foot per day.
Answered by Penny Nom.
Four consecutive integers 2016-01-05
From sherica:
my teacher gave us an assignment and it says here that; THERE ARE FOUR CONSECUTIVE INTEGERS.IF THE SMALLEST ONE IS x ,FIND THE SUM OF THE FOUR INTEGERS. that is the question ...thanks hope that u can answer it
Answered by Penny Nom.
The height of a parabolic arc 2015-12-30
From Tom:
Is there an algebraic means to determine the highest point of a parabolic arc if the base and perimeter are known?
Answered by Penny Nom.
How can he raise his average to 90? 2015-12-29
From Raj:
Harris has an 87 in a class where daily works count 20%, quizzes count 20% and tests count 60%.
If there are only two test scores what must the score on his second test to raise his average to 90?

Answered by Penny Nom.
Strings of characters 2015-12-28
From Ali:
If I have an app and offer a two, three and four digits name and it can be only number, letters or both?
I want to know how many options will I have for each?
For example
the two digits: aa, 11, a1 or 1a
the three digits: 111, a2a, 222
the four digits: 1111, 2ge3 or 1234
So if I used all the letters and all the numbers but nothing starts with a zero How can I calculate them separately? Or what's the result for each?
Thank you
Ali

Answered by Penny Nom.
A diameter of a circle 2015-12-27
From sahil:
Find the equation of diameter of circle x2 +y2-6x +2y-8 which passes through the origin.
Answered by Penny Nom.
Converting a fraction to a percent 2015-12-22
From Sharon:
How do you turn a fraction into a percentage? I'm beyond confused with this. Even though I think of myself as a good math student doing 8th grade math at a young age, this just makes me want to scream! I get the way you turn 1/4 into 25%, but when it comes to something like 7/9 = ? I just can't figure out the answer.
Answered by Penny Nom.
The angles of a triangle 2015-12-17
From Faith:
Does the measure of angle determine the length of its side? For example two angles are congruent then the sides are also congruent because from my understanding the angle determine the shape of triangle.
Answered by Penny Nom.
The perimeter of a 4 acre square 2015-12-16
From samantha:
What is the distance in miles or how many laps to make a mile, if one was walking around 4 acres of land which is like a square?
Answered by Penny Nom.
Proof that an erroneous algebraic statement is false 2015-12-14
From Berteanu:
I need help with this proposition:
"It exists x a real number that for every y real number 5*x-2*y*y=1
This is false.
Let x be from R.
And I need an y real number that 5*x-2*y*y!=1
Please,could you help me?

Answered by Penny Nom.
The line of intersection of two planes 2015-12-09
From Rabz:
Determine the vector equation for the intersection between the two given planes 2x+7y equal to 14,x+z equal to 7
Answered by Penny Nom.
20 golfers playing a 20 week season 2015-12-07
From Jeff:
We have 20 golfers playing a 20 week season. We play in foursomes. How best to set foursomes so everyone can play with every without playing with the same person multiple times?
Answered by Victoria West.
Drawing an arc 2015-12-04
From hassan:
how to draw a curve long 1m with an angle of 22.5?
Answered by Penny Nom.
The height of a truncated cone 2015-12-04
From Jack:
I need to build a truncated cone that has a top of30 inches and a base of 64 inches . The sides need to be at a 64.5 degree angle. This will determine the height. Can this be calculated?
Answered by Penny Nom.
A tangent line to a parabola 2015-12-02
From pei:
Given that the line y=mx-5 is a tangent to the curve y=2x^2+3 find the positive value of M.
Answered by Penny Nom.
The sweep angle of a video camera 2015-11-25
From Franc:
a television camera is 30ft from the sideline of a basketball court 94 ft long. The camera is located 7 ft from the mid court. through what angle must it sweep in order to cover all action on the court?
Answered by Penny Nom.
The range of f(x)=(2x+2)/(x-1) 2015-11-25
From Nazrul:
f : R to R is a function which is defined by f(x)=(2x+2)/(x-1).
Is f(x) an onto function? Please explain.
What is the range of this function?
Thank you very much for your previous help.

Answered by Harley Weston.
How much does the Earth curve over a one foot distance? 2015-11-24
From Sean:
Hi, I am trying to figure out how much the earth curves over a one foot distance. I'd like to be able to draw the exact arc on a piece of paper. I am an artist and am looking to make glass vessels with the exact curvature of the earth. I read on your site that it curves approximately 8 inches per mile. can I just use simple ratios to break it down into inches?? Thank you so much for your help.
Answered by Harley Weston.
The measure of an angle in terms of its complement 2015-11-22
From Pam:
Can you please help me so I can help my daughter the equation is the measure of angle v is 4 time the measure of its complement what is the measure of angle v when the equation is 4x+x=90
Answered by Penny Nom.
Dividing by a fraction 2015-11-21
From Jennifer:

Question from Jennifer, a student:

I want to know how to get the right answer for this math problem, dividing a fraction in to a whole number , example 3/8 using the whole number 6? Thanks


Answered by Penny Nom.
20 million as a percentage of 7.3 billion 2015-11-18
From Nick:
Based on the estimate of the worlds current population (7.3 billion people), I'm trying to work out what percentage of that figure 20 million people would amount to?

(Apologies in advance that I've no idea if the 7.3 billion figure is an American Billion or UK billion)

Thanks for any help.

Nick.

Answered by Penny Nom.
A schedule for 12 high school golf teams 2015-11-15
From Jason:
I have 12 high school golf teams (A - L). There will be 4 games each week for 4 weeks. For each game, 3 teams play at a time (A vs B vs C). I would like it where no team plays the same team twice. Week four is my issue as I seem to always have a team or two playing the same team again. Is it possible to have each team play each other only once?
Answered by Victoria West.
The area of a domed roof 2015-11-13
From Brandon:
I have a tank with a 13' radius that has a domed roof of 3.5' tall How do I figure out the area in SQFT?
Answered by Penny Nom.
Working with fractions 2015-11-12
From muhammad:
There are 1200 pupils in a school, 2/3 of them are girls, 1/4 of the boys are overweight.How many boys are overweight?
Answered by Penny Nom.
A volume in cubic yards 2015-11-12
From James:
The first formula for figuring cubic yards of concrete need is length in feet, times width in feet, times depth in actual inches, divided by 320. Such as, 150x 50 feet x 4inches divided by 320 = cubic yards. How and why does this formula work?
Answered by Penny Nom.
A concrete container 2015-11-12
From Karstin:
The exterior of a concrete container will be 10 feet by 8 feet by 4 feet tall. The walls and the bottom are 6 inches thick. What will it cost to construct it if concrete is $98.95/cubic yard?
Answered by Penny Nom.
Fencing around a rectangular field 2015-11-11
From Darlene:
Question from Darlene, a parent: A farmer has 10,000 meters of fencing to use to create a rectangular field. He plans on using some of the fencing to divide the rectangular field into two plots of land by constructing a fence inside the rectangle that is parallel to one of the sides. Let X be the width of the rectangular field. Write an equation to express the area of the field as a function of X. Find the value of X that maximizes the area of the field.
Answered by Penny Nom.
Roof Square footage 2015-11-11
From Todd:

Question from Todd:

Good Day.
I have to figure out the square footage of a quonset style roof that's not playing by the rules The building dimensions (rectangular) are 63'x135' the height of the roof is 9.25'. It not an entire Quonset, It's that style of roof,(curved). There are concrete block walls 10' up to the metal roof.

Thank you!

Todd


Answered by Harley Weston.
The derivative of x^1/3 2015-11-08
From omar:
hi can help me
am teacher ask me about x^1/3 Derivation definition .

Answered by Penny Nom.
Is 22/7 bigger or smaller than 3.14? 2015-11-06
From Natalie:
Is 22/7 bigger or smaller than 3.14?
Answered by Harley Weston.
A schedule for 8 golfers over 5 days 2015-11-04
From Ged:
Hope you can help with the following to save next years arguments ! Schedule for 8 golfers playing as pairs in 2 X 4 balls on 5 days. No player to pair same player more than once and to have played with each player In the 4 ball. Thanks.
Answered by Victoria West.
The domain of cos2x/(sinx-2) 2015-11-02
From Ameen:
Find the domain of:
f(x) = cos2x/(sinx-2)

Answered by Penny Nom.
Factoring a trinomial 2015-11-02
From Eric:
2v(squared) +11v +5
Answered by Penny Nom.
The sum of two repeating decimals 2015-10-22
From Kaitlin:
Here is the question I am struggling with:

Is the sum of two repeating decimals always repeating decimal? Explain your answer and give an example.

I answered this question thinking that no you cannot but only when adding 0.9 repeating, but my professor said this was incorrect.

Thanks for the help!
-Kaitlin

Answered by Penny Nom.
Simplify -3(x-7) 2015-10-21
From geraldine:
simplify: -3(x-7)
Answered by Penny Nom.
Exponential form 2015-10-17
From Will:
What is exponential form of -8?
Answered by Penny Nom.
4 games in 4 time slots with 8 teams 2015-10-16
From Paul:
I'm trying to set up a game matrix for my kids that will have 4 games in 4 time slots with 8 teams. And every team is playing a different game in a different time slot. I provided the initial matrix and I am trying to ensure that no 2 teams play each other more than once. That's my problem.
       8 pm 9 pm 10 pm 11 pm
Crokinole 1 vs 2
Trivial Pursuit 3 vs 4
Darts 5 vs 6
Pictionary 7 vs 8
Please help, I've been at it for hours.

Answered by Robert Dawson and Victoria West.
11 golfers over five rounds 2015-10-15
From Leo:
11 golfers over five rounds. Will golf as 4-4-3. How to set it up so everyone plays at least once with each player.
Answered by Victoria West.
We are six golfers going away for 3 days of golf 2015-10-13
From Mike:
We are six golfers going away for 3 days golf. We are playing in 2 no threeballs playing concurrently. How can we arrange things so that everyone plays averydody else at least once. Is this possible.?
Answered by Victoria West.
The length of a rectangle 2015-09-26
From Tris:
Hi,
I'm trying to figure out the length of a rectangle but i forgot how. So, the width is 5cm and the perimeter is 34cm.

Answered by Penny Nom.
A 6-person team that has 9 players 2015-09-21
From Sheri:
I am organizing a 6-person team that has 9 players. How do I schedule them fairly over a 9 week league?
Answered by Victoria West.
32 golfers playing 6 rounds 2015-09-19
From scott:
i am looking for a schedule for 32 golfers to play 6 rounds and never play with the same person twice.
I made one for 28 golfers but i am struggling with 32.

Answered by Victoria West.
A word problem 2015-09-17
From Suszanne:
Multiplying a number by x yields the same result as dividing the number by 0.125. What is the value of x?
Answered by Penny Nom.
Order of operations 2015-09-12
From Tanisha:
I would just like to double check if something like 5x squared times 4x cubed equals 20x to the power of 5??
It's just that we were told you can only answer an equation like this if the base is the same...so does that mean the x part or the whole thing like 5x? I'm sorry if that didn't make sense!

Thank you for your help!!

Answered by Penny Nom.
20 golfers for 6 days 2015-09-11
From Dave:
have 20 golfers for 6 days. Each player should pair with 18 golfers used a system on your website which is good for 5 days, but cannot get the 6th day. Thanks for your help Dave
Answered by Victoria West.
18 golfers playing four rounds 2015-09-09
From Paul:
I have 18 golfers playing four rounds. I want two 3 balls and three 4 balls each day. I don't want anyone to play in a 3 ball more than once and I don't want anyone to play with the same person twice. I realise everybody can't play with everybody, I just want the best you can do.
Answered by Victoria West.
HCF and LCM 2015-09-05
From Ally:
the HCF of the two numbers is 3, and the LCM is 15. what could the two numbers be?
Answered by Penny Nom.
A schedule for eight teams and four stations 2015-09-04
From Joel:
We are having a youth activity where we have eight teams and four stations. At each station there will be two teams competing against each other. We need each team to go through all four stations, but without ever competing against the same team, and, of course, without ever doing the same station twice.

If this possible? Thank you!

Answered by Victoria West.
The diameter and circumference of a circle 2015-09-01
From Tracey:
Hello,
I own a custom workroom, I am figuring out fabric quantities or an estimate. When at the job site, I forgot to measure the diameter of the semi circle shape that I have to make a cushion for!!!
If the circumference of the semi circle is 165" what would the diameter be, maybe half this measurement????

I appreciate any help...I basically need the width from left to right to see if I can fit the cushion on one width of fabric that is 52" wide!

Answered by Penny Nom.
Subtraction base 5 2015-08-29
From nakita:
can u explain the quinary subtraction ...
(3000) -(2342) all the numbers are in quinary number system

Answered by Penny Nom.
Factor x^2 + y^2 2015-08-26
From Jerome:
Factor: x^2+y^2
Answered by Penny Nom.
Four equations in 3 unknowns 2015-08-23
From damien:
solve:
S x U - T = 44
T - U x S = 36
U + T - S = 7
U x U / S = 4

Answered by Penny Nom.
Reinforced steel mesh in a circular foundation 2015-08-22
From Padmesh:
in a circular foundation we are laying a reinforced steel mesh . i.e., like chords in both ways. the circle diameter is 2.91m and rods are placed 0.1m spacing. so I want to find the number of rods present in that circle. here by I am attaching an auto cad drawing for your reference
Answered by Penny Nom.
The distance over a Quonset 2015-08-20
From jane:
total base of hemisphere is 30 ft apex height is 20 feet what is total length over dome
Answered by Penny Nom.
20 golfers playing four rounds 2015-08-17
From Paul:
I have a party of 20 golfers playing four rounds in fourballs. I realise that everyone will have 7 players that they do not play with, however I want to ensure that no one plays with the same person twice and that the Captain plays with the 12 players of his choice.
Answered by Victoria West.
Factor 2(x+1)(x-3)^2-3(x+1)^2(x-3) 2015-08-13
From Bella:
The directions are the solve and factor the expression completely: 2(x+1)(x-3)^2-3(x+1)^2(x-3)

I was factoring out an (x+1) and and (x-3) however the answer showing in my book is different and I can't figure out what I'm doing wrong please help

Answered by Penny Nom.
Parallelogram area 2015-08-04
From Nigel:
To find the area of a parallelogram you multiply the base by the height of the parallogram, the height being determined by an imaginary line drawn at right angles to the base. However if I made a parallelogram with joints at each corner and maipulated it to an upright position where the base and sides were at 90 degrees to each other,, I could then simply multiply the base times the height to get the area. Since the sloping side of the parallelogram does not vary in my imaginary jointed model, why can I not simply multiply the base by the sloping side to get the area in the same way that I multiply the base times the height of a square to find its area ? I cannot find the answer to this on the internet although I have searched.
Answered by Chris Fisher.
An oil field barrel 2015-08-02
From Michael:
I have a storage tank that is 12' diameter x 22.5' high
it's capacity is 450 oil field barrels (42 gallon)
Which would hold 19,036 us gallons
I'm trying to find out how many inches it would take to make 1 barrel
I was able to get this far by using your volume calculator, but haven't been able to come up with the rest!
Thank you very much for your time, and I really appreciate this site and the tools you all have made available to everyone for free!
Hope to hear back from you soon and God bless you all!!

Answered by Penny Nom.
Mary, Julie and Peter when on a trip 2015-07-10
From Edward:
How do you solve and set up a problem where: Mary pays one third of the cost of a trip, Julie paid half of what Mary paid, and Peter paid the remaining $75,
Answered by Penny Nom.
Golf for 12 with a proviso 2015-07-07
From Ken:
I hope you can help me. I'm trying to put together a draw for 12 players in 3 groups over 4 days. One proviso, 2 players must play together in each round because they require the use of a golf cart. We are due to play in September this year.
Thanking you in advance for any assistance you can give.

Answered by Victoria West.
Why is the area of square not conserved when it changes to a rhombus? 2015-06-28
From shubham:
Why is the area of square not conserved when it changes to a rhombus, both have equal sides still rhombus have less area than square.??
Answered by Penny Nom.
My final exam score 2015-06-19
From Deirdre:
Hi,
I got 69% on a question worth 20%
68% on a question worth 20%


And the final exam was worth 60%

My final grade overall was 63%

Did I do very bad in the exam????

Answered by Penny Nom.
1 + 1 + 1 + 1 + 1 + 1 + 1 + 1 + 1 + 1 + 1 + 1 x 0 + 1 = ? 2015-06-18
From Sharon:
1 + 1 + 1 + 1 + 1 + 1 + 1 + 1 + 1 + 1 + 1 + 1 x 0 + 1 = ?

I got 1 as my answer despite BODMAS making it 12 because logic tells me I ought to place brackets around the first set of repeated addition. Could you please clarify this for me? Thank you 😊

Answered by Harley Weston.
2^48 - 1 2015-06-13
From Soham:
The number 2^48-1 is divisible by two numbers between 60 and 70. The sum of the two numbers is?
Answered by Penny Nom.
The amount before profit and overhead 2015-06-12
From kelly:
I have an existing amount of $22,750.00. this includes the profit at 10% and overhead at 10%. How do I remove the profit and overhead in order to come up with the original starting figure?
Answered by Penny Nom.
A fantasy football league 2015-06-11
From Jason:
Hello. We have a fantasy football league with 14 teams. The season is 13 weeks long and we would like to figure out a way to have two divisions of seven teams each with each team playing their inter-division opponents twice. Is there a way to set this up? I cannot seem to find a way to do it. Thanks!
Answered by Victoria West.
8 golfers playing 5 round 2015-06-11
From Simon:
We have 8 golfers playing 5 rounds, and want to make the pairings as even as possible (ie all play against everyone else)

2 of the players however must play together each day - due to a buggy being required.

If anyone has a formula to work this out I'd be appreciated

Answered by Victoria West.
The surface area of a round bar 2015-06-06
From chirag:
Please tell me the formula for finding round bar surface in square inches/ mm
Answered by Penny Nom.
The surface area of a triangular prism 2015-06-03
From Alex:
Find the surface area of a triangular prism with Isoceles triangles and dimensions 4 feet by 6 feet
Answered by Penny Nom.
The volume of a pipe 2015-06-02
From gordon:
What is the formula to find out how many gallon of water in a length of pipe tried your answer on 8inch at 50feet did not work out
Answered by Penny Nom.
A pentagon inscribed in a circle 2015-05-30
From Victoria:
find the area of a regular pentagon inscribed in a circle with radius 3 units
Answered by Penny Nom.
The perimeter of a rectangle 2015-05-28
From Imran:
The length of a rectangle is x+3 centimetres.
The width of the rectangle is x-1 centimetres.

Find an expression in terms of x for the perimeter of the rectangle.
The perimeter of a rectangle The perimeter of a rectangle
Give your expressions in it's simplest form.
The perimeter of a rectangle The perimeter of a rectangle
Thank You.

Answered by Penny Nom.
Prove that you cannot factor x squared + 5 2015-05-28
From lily:
the question is: prove that you cannot factor x squared + 5
Answered by Robert Dawson.
y=f(x) and y=-3/2f(x) 2015-05-27
From Kevin:
Could you please show me what change y=f(x) to y=-3/2f(x) has gone through and please graph.

y=f(x) points: (-3,0) ,(0,-2), (2,-2), (3,4)

how does (0,-2) change?

Answered by Penny Nom.
The perimeter of a quarter circle 2015-05-21
From Bethany:
The perimeter of a quarter circle is 3.57. What is the quarter circle's radius?
Answered by Penny Nom.
Golf for 14: 2 fourballs and 2 threeballs 2015-05-20
From martin:
I have to organize 14 players to play four rounds of golf. Each round comprise two fourballs and two three balls .

What's the best combination to ensure that each persons plays with as many of the others as possible without playing with somebody more than twice?

Answered by Victoria West.
Cubic feet and cubic yards 2015-05-17
From Richard:
If I have a bag that covers.8cubic ft how many bags are in 2.65 cubic yards?
Answered by Penny Nom.
An octagonal frame around a pool 2015-05-17
From James:
I have a 20' pool and need to put a frame around it using 2by 4 what r the lengths and angle cuts
Answered by Penny Nom.
The intersection of a plane and a cone 2015-05-16
From Tom:
Is there a way to derive an equation that describes the perimeter of the intersection of a plane and a cone regardless of the angle of the plane to the cone. Assume that the plane does not cut through the base of the cone, the x, y, z location of the vertex is known, the distance from the vertex to the plane through the axis is know., and that the angle of the cone is known.
Answered by Chris Fisher.
A pile of mulch 2015-05-15
From Justin:
I have a pile of mulch that measures 19 feet wide at the bottom and has a height of 6 feet. I need to know how many yards of mulch I have.
Please help and help with the formula for further use. Thank you!

Answered by Penny Nom.
An angle of depression problem 2015-05-14
From haxir:
find the height of the balloon directly above a town if the measure of angle of depression of another town 5km from the first town is 20°!6`
Answered by Penny Nom.
Folding a rectangle to preserve the aspect ratio 2015-05-09
From Anna:
If you fold a rectangular piece of paper in half and the resulting rectangles have the same aspect ratio as the original rectangle, then what is the aspect ratio of the rectangles?
Answered by Penny Nom.
A word problem 2015-05-08
From JAMES:
A businessman bought a number of similar articles for a total of sh. 6000. Three of the articles turned out to be defective with no resale value. He sold the remaining articles at 12.5% each above the cost price making a total profit of sh 480. How many articles had he bought?
Answered by Penny Nom.
The surface area of a hemisphere 2015-05-05
From Mandy:
How can I show that the total surface are of any solid hemisphere is 75% of the area of the full sphere?
Answered by Penny Nom.
The method of elimination 2015-05-01
From oreanna:

Question from oreanna, a student:

How do u solve 2x+9y=3

7x-4y=-25 in elimination


Answered by Penny Nom.
Filling a pool with dirt 2015-05-01
From Mike:
I have a hole which a 24 ft pool in it is 10" deep in the the centre and goes to 1" inch at the edge want to fill it in with dirt how many yards of dirt would I need to fill it in
Answered by Penny Nom.
The volume of a sphere 2015-04-30
From Cassidy:
How do you find the radius of a sphere that has volume 36pI?
Answered by Penny Nom.
A fraction with a negative exponent in the denominator 2015-04-30
From ellen:
7/ab to the -4 power
Answered by Penny Nom.
The volume of a cube 2015-04-30
From megan:
What's the volume of a cube with a side length of 6?
Answered by Penny Nom.
The surface area of a cut pipe 2015-04-27
From ARUN:
dear Sir,

please advise me , how to calculate the surface area of a pipe with diameter of 630 mm and thickness of 67 mm which is cut in a angle of 22.5 degree.

please show me how to calculate the surface area of the pipe which cut in an angle.

Thanking you.

Answered by Harley Weston.
A piecewise function 2015-04-25
From uday:

f(x)={x^2+3x+2 / x+2, x(not equal to)=-1
{4, x=-1
how to find domain and how the graph looks like


Answered by Penny Nom.
Locating the center of a circle that forms an arc 2015-04-23
From Ken:

Find the Cartesian coordinates of the center of an arc with the given location of the beginning and end points and radius length. Not the midpoint of the circumference but the actual point that the arc
is drawn around.

I know their are two answers depending on the direction of the arc. Unless we assume that all arcs are drawn counter clock wise.

Thanks
Ken


Answered by Harley Weston.
A word problem with fractions 2015-04-23
From Cecilia:
if one third of a pole is red, one quarter is white and 15m is black what is the height of the entire pole
Answered by Penny Nom.
Expand and Simplify (3x+5)(5x-9) 2015-04-23
From chloe:
Expand and Simplify (3x+5)(5x-9)
Answered by Penny Nom.
A wireless fence 2015-04-18
From Dave:
I'm buying a wireless fence to keep my pet in my yard. It has a half acre range. In a straight line how far would that be?
Answered by Penny Nom.
Two cars approaching an intersection 2015-04-16
From Engabu:
Car A is traveling west at 50km/h & car B is traveling north at 60km/h. both are headed for the intersection of the two roads. At what rate are the cars approaching each other when car A is 3km & car B is 4km from the intersection?
Answered by Penny Nom.
Two airplanes 2015-04-14
From john:
two planes leave an airport at the same time, one going northwest (N35*W)at 400 mph and the other going east at 332 mph. How far apart are the planes after 4 hours to the nearest mile?
Answered by Penny Nom.
Team selection 2015-04-13
From Peter:
I manage a football team with a list of 28 players.
Only 24 can play each game.
The season is 16 games.
How do I fairly select 24 players each week.

Answered by Robert Dawson.
The area of a circle given the circumference 2015-04-12
From F:
How do you find the area of a circumference of 6?
Thanks.

Answered by Penny Nom.
The perimeter of a quarter circle 2015-04-10
From Riley:
How do you find the perimeter of a quarter circle when the radius is 56?
Answered by Penny Nom.
A word problem with fractions 2015-04-09
From Lorraine:
If the numerator of a certain fraction is doubled and the denominator is increased by 1, the fraction becomes 1/2.

If the numerator of the original faction is squared and the denominator is decreased by 2, the fraction becomes equal to 1.

Let x be the numerator and let y be the denominator of the original fraction. Write down two simultaneous equation in x and y.

Solve these equations to find two possible values for the given fraction.

Answered by Penny Nom.
On what day of the week did 1994 begin? 2015-04-05
From tayyaba:
there were 365 days in the year 1993 the first day of the year was friday. on what day of the week did 1994 begin?
Answered by Penny Nom.
The weight of a steel block 2015-04-04
From Mark:
the weight of a steel cube 37 inches X 28 inches X 7 inches
Answered by Penny Nom.
n^2 is a multiple of 100 2015-03-30
From Rahul:
I have to prove that n^2 is a multiple of 100 is necessary or Sufficient condition (or both) for n being multiple of 10
Answered by Penny Nom.
The game of 24 2015-03-28
From Suzanne:
I love this game! Cannot figure out 4,9,10,16 to equal 24? Help!
Answered by Penny Nom and Claude Tardif.
Ordering fractions 2015-03-27
From latoya:
Put the following amounts in order from greatest to least 60%,2/3,0.599 explain answer
Answered by Penny Nom.
16 golfers for 18 weeks 2015-03-22
From Rose:
We could use a schedule to include the following:
16 golfers
4 foursomes
Each person plays once a week for 18 weeks.
No one plays in same foursome more than 1 or 2 times, or the least amount of time.

Thank you for your kindness.

Answered by Victoria West.
8 golfers playing in 2x2 balls in 5 games 2015-03-18
From Brian:
We are 8 golfers who will be playing in 2x2 balls in 5 games. is there a formula to minimize the chances of playing with a partner or opponent too often please ?
Answered by Victoria West.
The weight of two dogs 2015-03-17
From Renee:
Darlene's dog weighed 5 times as much as Leah's dog. Together, the dogs weighed 84 pounds. How much did each dog weigh? Write an equation and solve
Answered by Penny Nom.
A cone of maximum volume 2015-03-16
From Mary:
I have to use a 8 1/2 inch by 11 inch piece of paper to make a cone that will hold the maximum amount of ice cream possible by only filling it to the top of the cone. I am then supposed to write a function for the volume of my cone and use my graphing calculator to determine the radius and height of the circle. I am so confused, and other than being able to cut the paper into the circle, I do not know where to start. Thank you for your help! -Mary
Answered by Robert Dawson.
A fishfinder 2015-03-13
From Dave:
I have a fishfinder that has a 20 degree cone on bottom of boat going to the bottom of the lake. How do I know the size of base diameter of the cone on the lake bottom depending on depth... such as 10 feet deep, for example?
Answered by Penny Nom.
Olivia and Ray walk to school 2015-03-12
From Rebeccah:
Olivia and Ray walk to school. Olivia walks 1/4 of a mile to school. Her walk is 2/3 of the distance that walks Ray to school. What is the total distance,in miles,that Ray walks to school?
Answered by Penny Nom.
Two sunflowers 2015-03-12
From Maurice:
Sam and sally planted two sunflowers.Each sunflower grows 1cm each day. Sally's sunflower is 4cm high. Sam's is 12cm high. How high will each sunflower be when Sam's sunflower is twice the height of Sally's?
Answered by Penny Nom.
The height of an equilateral triangle 2015-03-12
From anna:
I am anna and i am in 7th grade.
i am trying to find the height of and equilateral triangle, all sides equaling 4 inches

Answered by Penny Nom.
The area of a quadrilateral 2015-03-11
From Joel:
Diagonal ac of quadrilateral ABCD is 60cm and the lengths of perpendiculars to It from the opposite vertices are 4.2cm and 5.8 cm find the area of the quadrilateral ABCD
Answered by Penny Nom.
Angles of elevation and depression 2015-03-08
From Timmy:
Joel is walking down a street and sees a 115 ft tall building in front of him. He stops 190 feet from the base of the building at the tip of the building's shadow. Round answers to three decimal places.

A. If there was a piece of rope from the top of the building to Joel, how long would it be?
B. What is the angle of elevation from Joel to the top of the building?
C. Margaret says that she could find the angle of depression from the top of the building to Joel by subtracting the angle of elevation from 90°. Is she correct? Explain.

Answered by Penny Nom.
A man is standing on top of a building 2015-03-06
From kiki:
a man of height 1.5m standing on top of a building of height 48.5m views another building across the square. he observe that the angle of depression of the bottom of the building is 40 degrees and the angle of depression of the top of the building is 5 degrees. Both buildings stand on the same level
a) calculate the distance of the man from the base of the building across the square measured along the line of sight

Answered by Penny Nom.
Integers in different bases 2015-03-05
From Michael:
Let k be a positive integer so that 28 (subscript)k = 132 (subscript)5
Answered by Penny Nom.
Bricks around a fire pit 2015-03-05
From Jayson:
I have a round fire pit. It measures 25 inches in diameter. I have 12 inch long square bricks to go around it . My question is what degree do I cut the ends of these bricks to make them fit around this circle? The brick dimensions are 12"Lx6"Wx4"D.
Answered by Harley Weston.
Solve x=r(t-sin(t)) for t 2015-02-28
From David:
I don't know how to write the function of t(x) by x=r(t-sin(t)).
Can you teach me that?

Answered by Harley Weston.
The radius of a cylinder 2015-02-26
From Rose:
Hi. I want to know how to find the radius of cylinder.
When I have height (35cm) and area of the curved surface(880sq.cm).
I know the formula is 2πr(h+r). But I can't understand how to find it's radius.
Please help me. I really need your help.

Answered by Penny Nom.
Two equations with fractions 2015-02-26
From Pulane:
Hi math centre I've been trying to solve these equations for days now please help (6/x)-(1/y)=4
(9/x)+1=(-2/y)

Please help me solve them simultaneously
Thank you

Answered by Penny Nom.
f(x)=(x^2-1)/(x-1) 2015-02-21
From Ahmed:
Is f(x)=[(x^2-1)/(x-1) and x=2 at x=1] differentiable at x=1 ? Why ?
Answered by Penny Nom.
A limit 2015-02-19
From genc:
Lim (27x^3-1) / (3x-1)
X-> 1/3

Answered by Harley Weston.
Two equations 2015-02-16
From nigel:
2x+1/2y=1
6x-3/2y=21

Answered by Penny Nom.
A 330 foot long fence 2015-02-15
From Marsha:
I am putting up a fence 330 feet long and want to put the t post every 4 feet how many t post do i need?
Answered by Penny Nom.
A stone is dropped in a lake 2015-02-14
From Wendy:

Hi, I have problems solving this problem. We didn't exactly go over these kind of problems and the book doesn't have an example either. Thank you for your help!

A stone is dropped in a lake, creating a circular ripple that travels outward at a speed of 80 cm/s.
(a) Find a function g that models the radius as a function of time t.

g(t) =

(b) Find a function f that models the area of the circle as a function of the radius r.

f(r) =

(c) Find f compose g.
f compose g =


Answered by Penny Nom.
A word problem with fractions 2015-02-11
From Omokayode:
A vehicle tank was 3/5 full of petrol. When 21 litres of fuel was added it was 5/6 full. How many litres of petrol can the tank hold?
Answered by Penny Nom.
A fence post problem 2015-02-08
From Nicole:
In Susan's rectangular garden plot, she put 12 posts on the long sides and 8 posts on the short sides. How many posts did she use altogether? (The answer is not 40.)
Answered by Penny Nom.
The product of a 2-digit number and a 3-digit number 2015-02-06
From Nathaniel:
The product of a 2-digit number and a 3-digit number is about 50 000 what are the products
Answered by Penny Nom.
2x^2-2x+1=4x+1 2015-02-06
From Caitlyn:
2x^2-2x+1=4x+1
Answered by Penny Nom.
The center and radius of a circle 2015-02-06
From ariana:
I need to find the center and the radius of this circle. I don't know how to put 9/2 than square it into a fraction.

x^2 + 2x + y^2 +9y=5

Answered by Penny Nom.
The volume of a cone 2015-02-05
From Linda:
How do you calculate the volume of a cone that is 25cm high and has an angle of 20 degrees?
Answered by Penny Nom.
Comparing fractions 2015-02-02
From shealla:
Please help me with my assignment please I'm having trouble answering this one..
Write the larger fraction in each pair..
A. 2/3, 4/7 B. 1/3, 2/5 C. 5/8, 3/5

Answered by Penny Nom.
Ratio of gas to oil 2015-02-02
From Chris:
I would like to find the gas oil ratio for a 2 cycle engine. I have the gallons of gas and the ounces of oil used by the engine, but I want to find what the ratio is. For example I used 5.675 gallons of gas and 5 ounces oil what is the ratio? Please explain step by step, and I understand there are 128oz in a gallon.
Answered by Harley Weston.
128/(-16)/(-2) 2015-01-28
From jackie:
128/(-16)/(-2) I was wondering if you can show me how to work this question out
Answered by Harley Weston.
Is a square a trapezoid? 2015-01-26
From Katie:
Can a trapezoid sometimes be a square?
Answered by Penny Nom.
Scheduling meetings with pairs of people 2015-01-22
From Jacey:
I am trying to figure out a formula/system to pair up a list of people so they meet with each other every month, but they rotate who they meet with. Right now I have 13 people and I would like to just type in their names and then have the system put each person with someone else every month and rotate so no one gets the same person twice. Can this be done?
Answered by Victoria West.
The game of 24 2015-01-21
From aaron:
24 game whants me to use 11 14 5 and 9 to make 24 allowed (+ - dividing and multiplying)
Answered by Penny Nom.
The height of a building 2015-01-20
From Emily:
A man 2m high observes the angle of elevation to the top of the building to be 70 degrees. And the angle of depression to the bottom of the building to be 19 degrees. How tall is the building?
Answered by Robert Dawson.
The length of a roll of paper 2015-01-12
From Peggy:
roll of paper 3ft wide and roll equals 500 sq ft how long would the roll be?
Answered by Penny Nom.
A difference quotient 2015-01-12
From Sasha:
Simplify the difference quotient

f(x) − f(a)/ x-a
if x ≠ a.
f(x) = x^3 − 12


Answered by Penny Nom.
The volume of a lake 2015-01-10
From Peter:
If a lake is 8 acres in size and 10 feet deep how many gallons of water does it contain
Answered by Penny Nom.
Selling items at a reduced price 2015-01-09
From Kent:
I have an item I sell for $19.95 and make a profit of $10.97 for each sale. If I reduce the price to $14.97, my profit is $5.99 each sale. Is there a formula to determine how many more I would need to sell at $5.99 per sale to make the same net cash (profit) as selling at $10.97 per sale?
Answered by Penny Nom.
The units digit of a prime 2015-01-08
From Patricia:
Every prime number greater than 10 has a digit in the ones place that is included in which set of numbers below 1 3 7 9 or 1 3 7 or 0 2 4 5 6 8 or 1 3 5 9
Answered by Walter Whiteley.
Scheduling 12 for lunch 2015-01-05
From Sabrina:
I have 12 staff members that need to have lunch together 1 time each week. I am trying to pair them in a way that we each have the same amount of lunches in the same amount of weeks. Please Help! Thanks!
Answered by Victoria West.
The degree of a polynomial 2015-01-05
From Nichole:
How do I determine the degree of polynomials? I've searched this on sites but they are all so confusing! Is there a simple explanation or way to find what the degree is? Some examples are: 6x^4 10x^2yz^5 and 3m^2n^7-10m^8. I also have to say that I am under the impression that this symbol ^ means the number after it is an exponent.
Answered by Robert Dawson and Harley Weston.
Filling a tub 2015-01-02
From Mr:
The cold tap can fill a bath in 5minutes,the hot tap in 15,and the waste pipe empty the full bath in 10 minutes. All three are fully open for 2 minutes,after which the waste-pipe is closed. How much longer will it take to fill the bath?
Answered by Penny Nom.
1.5% of 1 trillion 2014-12-30
From Kgosi:
How much is 1.5% of 1 trillion?
Answered by Penny Nom.
Rates, percentages and units 2014-12-30
From Kenneth:
Hello:

If percentages have no units, why are some percentages called rates, as in interest rate, or perhaps a tax rate of 7% as an example? A rate has units of different quantities.

I thank you for your reply.

Answered by Robert Dawson.
Curvature of the Earth 2014-12-29
From Jimmy:
Both batteries died in my scientific calculator and I have lost my formula for the heigth of the curvature of the earth between two points on the surface. I used degrees and miles. I only had to enter the distance between the two points on the surface and the formula gave me the hieght the earth raised between the two points.
Answered by Robert Dawson.
The perimeter of a quarter circle 2014-12-26
From Maisy:
This is the question that I got for a math worksheet.
The radius of a quarter circle is 5 miles. What is the quarter circle's perimeter?
Use 3.14 for pi.

Answered by Penny Nom.
A trip to Ottawa from Toronto 2014-12-22
From Kristy:
A trip to Ottawa from Toronto will take 4 1/2 hours. Assuming we are 2/3 the way there, how much longer will the trip take?
Answered by Penny Nom.
A line segment of length root 5 2014-12-15
From angela:
On the dot grid below, draw and label a line segment with length square root 5 the dot grid is 8 by 10
Answered by Penny Nom.
Planar curves 2014-12-13
From ann:
what does planar curve mean in your definition of a cone?
Answered by Penny Nom.
A cuboid 2014-12-11
From Nehal:
Six cubes have a surface area of 24 cm2. They are joined together to male a cuboid. What could the surface area of this cuboid be? There are two different answers.
Answered by Penny Nom.
30% profit 2014-12-05
From Lyn:
I would like to know a formula that I would use on a standard calculator to calculate a 30% profit margin on an item. If I purchased an item at $7.70 how would I calculate 30% profit for me, not just 30% mark up?
Answered by Penny Nom.
How do you change a decimal to a ratio? 2014-11-29
From Esmeralda:
How do you change a decimal to a ratio? Express the decimal 0.2 as a ratio.
Answered by Penny Nom.
1999 equations in 1999 unknowns 2014-11-28
From Subrahmanya:
Solve the following system of 1999 equations in 1999 unknowns :

x1+x2+x3=0, x2+x3+x4=0……., x1997+x1998+x1999=0,

x1998+x1999+x1=0, x1999+x1+x2=0.

(in x1,x2,...................,x1999 the numbers 1,2,.............,1999 are subscripts)

Answered by Penny Nom.
What is 20 and 3/5 of 10? 2014-11-26
From Jeannine:
What is 20 and 3/5 of 10? Question is written exactly this way. Is this 20 3/5 times 10 (103/5 times 10), or is it 20 and (3/5 times 10)?
Answered by Penny Nom.
Doubling the dimensions of a cone 2014-11-21
From Hannah:
If the volume of a cone of height 10 cm is 261.8 cm3, show that this volume is increased by a factor of 8 if the dimensions of the cone are doubled.
Answered by Penny Nom.
Factorise 15x + 15y 2014-11-20
From Jordan:
hi i saw a question in class today that i didn't understand and i didn't really understand what my teacher was going on about so how do you factorise 15x + 15y
Answered by Robert Dawson and Penny Nom.
How many square feet in 10 gallons of liquid at 1/4 in thick? 2014-11-14
From Rich:
How many square feet in 10 gallons of liquid at 1/4 in thick
Answered by Penny Nom.
(x-3)^2-(x+3)^2 2014-11-13
From Bernice:
(x-3)^2-(x+3)^2
Answered by Penny Nom.
1/6k=-11 2014-11-13
From Marty:
Using the balance method, what is the answer to 1/6k=-11, trying to find what k equals. Thanks
Answered by Penny Nom.
What is -3 squared? 2014-11-12
From Christine:
On a math test, it said "What is -3 squared?" It did NOT say "What's is (-3)squared"

The teacher's explanation is that if there are no brackets or parenthesis, you ALWAYS square the number first then do the negative, so the answer should be -9, but I can't find anywhere that confirms this.

Help please.

Answered by Robert Dawson.
The area of a circle in terms of the circumference 2014-11-04
From Sarah:
How do you find the area of a circle if you already have the circumference? There was one answer to this question already but it didn't make sense to me, because they rounded up pi which you can't do so the answer is incorrect. thanks
Answered by Penny Nom.
A quadratic function modelling the heightof a flea jump 2014-11-02
From Hannah:
A flea jumps from the ground to a height of 30 cm and travels 15 cm horizontally from where it started. Suppose the origin is located at the point from which the flea jumped. Determine a quadratic function in vertex form to model the height of the flea compared to the horizontal distance travelled
Answered by Penny Nom.
Prime factorization in exponent form 2014-10-31
From Emma:
I need to find out how to make a prime factorization of 120 in exponential form.
Answered by Penny Nom.
Two quadratic polynomials 2014-10-28
From khushboo:
The zeroes of a quadratic polynomial x^2+ax+b are 'c' and 'd' and the zeroes of a quadratic polynomial x^2+cx+d=0 are 'a' and 'b'. find the numerical value of a+b+c+d where a, b, c and d are non zero integers.
Answered by Penny Nom.
A 10-foot piece of PVC pipe 2014-10-21
From David:
A plumber has a 10-foot piece of PVC pipe. How many 9/5-foot pieces can be cut from the 10-foot piece ?
Answered by Penny Nom.
A fact family containing 4, 4 and 16 2014-10-14
From Leslie:
I am helping my son and the question is 4,4,16 in fact family what does that mean
Answered by Penny Nom.
A quarter mile row of trees 2014-10-09
From junna:
a farmer would like to plant trees on one side of his lot which is 1/4 of a mile in a length His friend advised him that if he would plant the trees 5 feet farther apart,he would need 44 fewer trees.If the farmer would take the advice, how many trees would he plant?
Answered by Penny Nom.
Why express fractions in lowest terms? 2014-10-09
From John:
I teach in a Faculty of Education. A colleague in the university asked me about fractions and lowest terms. I am quoting the person below, and would appreciate your insights into the question/thought.

"One I'm struggling with is why (for example) 6/18 is not considered as good an answer to a fraction question as 1/3. The traditional response is that 6/18 is not in lowest terms so the question has not been finished until the fraction is reduced, but what actually makes the lowest terms answer the better one? Is it convention? Is there a way to explain why simplest form answers in fractions are right and and anything else is considered incorrect without alluding to some need for this 'good habit' elsewhere in math or science? Is there a real-life reason?" Thank you for any insights and if you have nothing to offer to this query, that is fine too. I appreciate your consideration. John

Answered by Robert Dawson.
A factoring problem 2014-10-05
From Gina:
3x^2+6x+3
Answered by Penny Nom.
Sieve of Eratosthenes 2014-10-03
From Hope:
using Sieve of Eratosthenes to determine for in instance prime numbers between 1 to 200, to what nth number should i stop?, 4th, which is 11 or what?
Answered by Robert Dawson.
Trig functions and the unit circle 2014-10-02
From Jake:
I was wondering what conclusions can be drawn about the trigonometric functions and how they work about the circle. Can you also please give me an explanation for it? Thank you.
Answered by Penny Nom.
Continuity on a closed interval 2014-09-21
From Pragya:
The trouble I'm having is as follows :
a continuous function is most of the times defined on a closed interval, but how is it possible to define it on a closed interval ,because to be continuous at endpoints of the interval the function's limit must exist at that endpoint,for which it has to be defined in its neighborhood,but we don't know anything about whether the function is always defined in the neighborhood.
Please help...

Answered by Penny Nom.
The perimeter of a parcel of land 2014-09-18
From Shirley:
What is the perimeter of a parcel of land that is 564 acres square
Answered by Penny Nom.
Meetings for 77 students 2014-09-18
From Adam:
We have a new group of students starting, and would like to set up a series of small group meetings so each student meets each other, hopefully without meeting the same people twice.

We have 77 students. In theory I would like to have 11 meeting slots, with 10 groups in each slot, with up to 8 students in each group. Is there a way to do this?

I have wondered if it might be easier to do 9 meeting slots, with 9 groups per slot, with 9 students per group. Thanks!

Answered by Victoria West.
A tangent to a curve passing through a point not on the graph 2014-09-15
From Aquilah:
For the curve y = x2 + 3x, find the equations of all tangent lines for this graph that also go through the point (3, 14).
Answered by Penny Nom.
The diagonals of a cube 2014-09-04
From Rukshanth:
How many diagonals does a cube have?
Answered by Harley Weston.
The sum of two numbers is 52 2014-09-01
From Blake:
The sum of two numbers is 52 and the difference is 10. What are the numbers? i used to be real good at this stuff?
Answered by Penny Nom.
The perimeter of a triangle 2014-08-21
From Phyllis:
What is the perimeter of a five acre triangle with one side being 250 feet and the second side would be at a 90 degree angle.
Thank you.

Answered by Penny Nom.
Filling three holes with stones 2014-08-20
From mark:
how many tonnes of hardcore/crushed stone would it take to fill 1 hole 9ft diameter 5ft deep and 2 holes both 3ft diameter and 5ft deep
Answered by Penny Nom.
Revisiting 8 golfers in two foursomes 2014-08-19
From steve:

I noticed you had an answer on how can you put 8 golfers in even foursomes over 7 games so that everyone plays with everyone equally. However the formula showed the following sequence that was used but did not go past the second day. Can you change the 0s and 1s into the numbers 1 to 8 instead. Here is what you have written:
Day 0 : (0, 0, 0, 0, 1, 1, 1, 1)
Day 1 : (0, 0, 1, 1, 0, 0, 1, 1)
Day 2 : (0, 1, 0, 1, 0, 1, 0, 1)
Day 3 : (0, 1, 1, 0, 1, 0, 0, 1)
Day 4 : (0, 0, 1, 1, 1, 1, 0, 0)
Day 5 : (0, 1, 0, 1, 1, 0, 1, 0)
Day 6 : (0, 1, 1, 0, 0, 1, 1, 0)

Thank you so much for your anticipated help.

Steve


Answered by Harley Weston.
A barge of triangular cross section 2014-08-18
From tushar:
a barge of triangular cross section is 20m long 12 m wide and 6m deep.its floats in SW at a draft if 4m find its displacement
Answered by Penny Nom.
4 rounds of golf as three foursomes 2014-08-18
From Christopher:
For our golf trip, we will be playing 4 rounds of golf as three foursomes. We would like to have each person play with everyone at least once. Is this possible?
Answered by Victoria West.
The equation of a circle 2014-08-14
From jennifer:
hi there My name is Jennifer and residing in Denmark. I am a student and I wrote to you because i am having trouble in finding out the equation for the circle using (x-a)^2 + (y-b)^2 =3D r^2.The diameter of this circle is d=3D 44,514 cm. I have attached a drawing of my problem..thanks
Answered by Penny Nom.
Speed, distance, time 2014-08-13
From Aarti:
a theif running at 8 km/hr is chased by a policeman whose speed is 10km/hr.if the thief is 100 m ahead the policeman then what will b time required for the policeman to catch the thief?
Answered by Tyler Wood.
Ratios ond percents 2014-08-13
From aswathi:
if the ratio of cost price and selling price of an article is be as 10:11,the percentage of profit is
Answered by Tyler Wood.
Golf for 10 2014-08-12
From Fred:
We have 10 golfers playing 4 rounds: one 4-some and two 3-somes each round. Can you suggest the best distribution of players?
Answered by Victoria West.
Golf for 15 2014-08-11
From Brian:
I am trying to arrange 5 rounds of golf for 15 golfers. We will play 5 in a group, 3 groups per round, 5 rounds. I would like everyone to play with everyone else. Secondary objective would be for no one to play with anyone else more than 2 times.

Thank you.

Answered by Victoria West.
The area of a quadrilateral 2014-08-06
From Rahul:
find the area of the quadrilateral whose side measure9cm,40cm,28cm,15cm and in which the angle between the first 2 sides is a right angle
Answered by Penny Nom.
The area of a kite 2014-08-04
From Janis:
How do you calculate the area of a kite with the diagonals 24cm and 48cm?
Answered by Chris Fisher and Penny Nom.
A word problem with fractions 2014-08-01
From nathanielcabalan:
The denominator of a fraction is one more than the numerator. If the numerator is increased by three, the resulting fraction is one more than the original fractional. Find the original fraction.
Answered by Penny Nom.
Expand and simplify [x-3][x+3] 2014-07-16
From symion:
Expand and simplify [x-3][x+3]
Answered by Penny Nom.
16 golfers over 15 weeks 2014-07-11
From Steve:
I need a schedule for 16 golfers over 15 weeks, where each golfer is paired with a different golfer each week and plays 4 matches per week against 4 different pairs (each pair does not have to play each week)
Answered by Penny Nom.
A 14 team golf league 2014-07-11
From Pat:
We have a 14 team golf league that plays a 16 week schedule. Playing once per week, I need a schedule where each team plays every other team at least one time.
Answered by Victoria West.
Fencing around a 6 sided property 2014-07-10
From Annette:
Hi, I've tried every way to figure this out and looked at your other acreage questions and still don't have an answer. It's my Birthday today and it would make me so happy to figure this out. I have this area I want fenced on my property and need to figure out how many acres it is to know if this guy is charging fare price for fencing.
It has 6 sides to it and measured in feet the sizes are: 1400 ft long
807 ft long
1060 ft long
300 ft long
310 ft long
400 ft long

Hope you can help me.
Thanks.
Annette

Answered by Penny Nom.
A 5 sided lot 2014-07-10
From mia:
have the feet numbers of three sides, but the forth side is in a "V" so it has two figures. Question: can i add the two figures of the "v" together to get one figure, so then I would have the figures for all four sides.
Answered by Harley Weston.
4/9 of a set of story books cost Rs 630. 2014-07-09
From YASIN:
If 4/9 of a set of story books cost Rs 630. What will the rest of the set cost?
Answered by Penny Nom.
A ladder against a wall 2014-07-09
From thabo:
A ladder 6.5m long,leans against a wall so that the top of the ladder is 4.8m from the ground.what is the angle of elevation of the ladder to the top of the wall
Answered by Penny Nom.
14 golfers, 7 men and 7 women 2014-07-06
From John:
I have to schedule 14 golfers, 7 men and 7 women, into 4 teams each day, playing for 3 days. So each player plays with as many different players as possible for 3 games.
Answered by Victoria West.
The method of elimination 2014-07-05
From leo:
please explain how can i solve this problem

3x-6y=-38
6x-9y=44

using elimination and simultaneous method thank you :)

Answered by Penny Nom.
The width of a rectangle 2014-07-02
From john:
if the area of a rectangular field is 90 sq. m and its length its is 19 m find its width
Answered by Penny Nom.
Differentiate ln[x(2x-4)^1/2] 2014-06-28
From Igwe:
If y=In[x(2x-4)^1/2],find dy/dx at x=3
Answered by Penny Nom.
A triangle on the surface of the Earth 2014-06-26
From Christine:
A, B and C are three towns, the bearing of B and C from A being 310 degrees and 220 degrees, and their distances from A are 510km and 700km respectively. Find the bearing of B from C to the nearest minute.
Answered by Robert Dawson.
50+9-13 multiplied by 2= 2014-06-25
From Adrian:
50+9-13 multiplied by 2=
Answered by Penny Nom.
12 golfers 2014-06-23
From Derek:
I have a golf group that is set up as follows.12 players broken into 2 teams of 6.We play as a 4 ball ie 2 from each group.I am looking for a formular to have the 2 players from each group not play with each other again & also both of the 2 in each group not play with others in the other group more than once.I don't believe it is possible,so will accept as little duplication as possible-many thanks-Derek
Answered by Victoria West.
An oval pool 2014-06-21
From steve:
I have a 16' x 28' oval pool that is buried 24" deep inground. The dig site is dug 2' wider all the way around the pool. I need to back fill this area with stone. I want to fill this area with 6 to 8" of stone. How many tons of stone will this take?

Thanks you
Steve

Answered by Penny Nom.
The number of standard deviations 2014-06-15
From Dawn:
Suppose that, at a certain college, the average weight of all male students is 160 pounds with a standard deviation of 30 pounds. A certain male student weighs 145 pounds. Determine how many SD's his weight is above or below average.
Answered by Penny Nom.
Profit 2014-06-14
From roudha:
find the profit of one item if it was bought for AED 180 and sold for AED 275. assume that the additional costs are AED 25 .
Answered by Penny Nom.
A model of the sun and the Earth 2014-06-14
From Lynn:
There is a model of the sun that is 2 1/2 inches in diameter. This is the only clue we have! How far away is the earth from this model?
Answered by Penny Nom.
The area of a circle of circumference 32.69 meters 2014-06-11
From coco:
Find the area of a circle with a circumference of 32.69 meters.
Answered by Penny Nom.
Bags of sand 2014-06-06
From Robert:
We have a section that is 20ft long by 14ft wide and 4inches deep, the bags of sand are .5 cubic ft. How many bags of sand do we need to fill this space evenly.
Answered by Penny Nom.
Factorials 2014-06-06
From penny:
What is factorial? For eg. Like 2!, 3! 4! Etc.
Answered by Penny Nom.
10 golfers, playing 7 rounds in 2 x 3balls and 1 x 4ball 2014-06-04
From adrian:
Hi, We are off to Scotland on our yearly golfing trip soon. Question is we have 10 golfers, playing 7 rounds in 2 x 3balls and 1 x 4ball. I would like to ensure that each golfer plays with the other guys an equal amount of times, guess 3ish? and we share the amount of times each player plays in the 4ball. Hope you can help?
Answered by Victoria West.
How do we use logarithms in our daily life 2014-06-01
From ss:
how do we use logarithms in our dailylife
Answered by Penny Nom.
6 inches of dirt in a baseball field 2014-06-01
From Mary:
The baseball field is 20,000 Square feet, 6 inched deep I need to know how many cubic yards of dirt this will generate.
Answered by Penny Nom.
How do I make fractions into decimals? 2014-05-31
From jay:
Hi,
I wanted to know how do I make fractions into decimals example 1 3/8?

Answered by Penny Nom.
A word problem with fractions 2014-05-30
From shadab:
A container of milk is 4/5 full. When 10l milk is poured into it, the container becomes 9/10 full. What is the capacity of the container ?
Answered by Penny Nom.
A word problem with fractions 2014-05-23
From Tanzeela:
There are 1200 pupils in a school. 2/3 of them are girls. 1/4 of the boys are overweight. How many boys are overweight?
Answered by Penny Nom.
The volume of a wedge 2014-05-21
From steve:
I need to figure the volume of a wedge / right triangle, the dimensions of the right triangle is L1(10'-5"), L2 (10'-6"), H (14'-9.5") height of L1 is 0'-0" height at intersect L2/H is 3'-3"
Answered by Penny Nom.
14 golfers for 13 weeks 2014-05-17
From Judi:
I want to schedule each of my 14 golfers to play each of the other girls one time during a 13 week period. We would golf in foursomes , threesomes or twosomes to make this happen. Can you send me a grid that would do this scheduling? I'm at a loss as to how to do it!!
Answered by Victoria West.
The equation of a line 2014-05-16
From Michael:
Find the equation of a line that passes through (2,-1); the sum of the x- and y-intercepts is 2. (There are two answers)
Answered by Penny Nom.
Golf for 14 players over 12 weeks 2014-05-16
From Judi:
How would I develop a playing schedule using foursomes and one twosome for 14 players who will play 12 weeks and each person would play all but one of the others?
Answered by Victoria West.
18 golfers for 5 rounds with a teaching Pro 2014-05-14
From Maree:
We are taking 18 golfers on Tour with a teaching Pro. There are 5 rounds of golf and we want everyone to play with the Pro once, but everyone else to play with as many different people as possible
Answered by Victoria West.
Eleven guys on a fishing trip 2014-05-13
From Mark:
Eleven guys are going on a fishing trip and want to rotate so everyone fishes with the other guys with the fewest number of repeats. They want to fish in groups of two with one fishing alone for three and a half days so will rotate seven times. Is there a combination that works and what is it?
Answered by Victoria West.
How many boards did the carpenter start with? 2014-05-08
From Yanly:
A carpenter has several boards of equal length. he cuts 3/5 of each board. After cutting the boards, the carpenter notices that he has enough pieces left over to make up the same length as 4 of the original boards. How many boards did the carpenter start with?
Answered by Penny Nom.
A schedule for 2 teams of 10 2014-05-07
From John:
How can I set up a schedule for 2 teams of 10 on a team to play each individually against each other over a 10 day span at 10 different course with 10 different start times each day. I was looking to not repeat any competition. As an example, Team A would consist of players 1-10, Team B would consist of players 11-20. I would want to have 10 start times a day for 10 days where a member of team A plays a member of team B each day without repeating any matches and any of the same courses and not repeating any of the same start times. Any help in this would be GREATLY appreciated!!!!
Answered by Victoria West.
A 5-person golf flight 2014-05-05
From Jeremy:
can you have a 5-person golf flight and play only 3 matches so that one person plays one match each?
Answered by Victoria West.
Golf for 8 2014-05-05
From Ivan:
We have an eight member golf team. We field one foursome each week for 16 weeks of league play. How do I roate the eight members so each team member plays with each team member and all play an approximately equal number of times?
Answered by Victoria West.
4 golfers in twosomes 2014-05-04
From Lori:
4 golfers want to play as 2 somes every week, but rotate the 2 players so we all play together With each other evenly thru out the summer
Answered by Victoria West.
Golf :15 players over 4 days 2014-05-02
From michael:
We are playing with 15 players over 4 days. We plan to have 3 people in each group. A total of 5 groups with 3 players each. Can I arrange for each player to play with other players only 1 time?
Answered by Victoria West.
1÷[1-√2(order of surd is 4)] 2014-05-02
From Anoushka:
if t=1÷[1-√2(order of surd is 4)] , then t=?
Answered by Penny Nom.
Surface area 2014-05-02
From vijay:
If a square paper of side 25 cm is rolled to form a cylinder, then its curved surface area is
Answered by Penny Nom.
11 golfers going on a golf trip for 4 rounds 2014-05-01
From David:
I have 11 golfers going on a golf trip for 4 rounds of golf. I want to make sure that everyone gets a chance to play with everyone at least once but not more than twice. Also, as we are n = 11, we will be composed of 3-4-4 everyday. I am trying to ensure that as few as people as possible play in the group of 3 (golf seems to be more enjoyable for whatever reason in foursomes). i have tried 3 differemt times and can seem to figure it out. Can you assist?
Answered by Victoria West.
2 teams of six playing each other 4 times 2014-05-01
From Steve:
I have 2 teams of six (Team A & B)- playing each other 4 times.

Is it possible to have a player from Team A play with a different partner from Team A for each of the 4 rounds, and likewise for Team B. Whilst no player from either team plays an opponent more than twice during the 4 rounds?

Answered by Victoria West.
A cone of vision 2014-04-29
From David:
It is known that a fish in water looking up has a 97 degree "cone" of vision that sees "through" the surface of the water. If a fish lies 4 inches below the surface, the cone forms a window (circle) smaller than if a fish lies 8 inches below the surface. What is the ratio of inches of depth to the radius of the circle on the surface that is its visual window?
Answered by Penny Nom.
Profit and loss 2014-04-29
From Anoushka:
A man sells two tables of the same price. On one he makes a profit of 10 percent and on the other he suffers a loss of 10 percent.Find his loss or gain percent on the whole transaction.
Answered by Robert Dawson.
Fractions and units 2014-04-28
From Aaron:
I can't do this math problem. please help. Find 200m over 1 min times 60 min over 1 hr times 1 km over 1000 m. the choices are a. 300 km/hr b. 12 km/hr c. 300 m/hr and d. 12 m/hr
Answered by Penny Nom.
A triangular chicken pen 2014-04-27
From Cierra:
Margaret has two lengths of fence, 20 meters and 24 meters, for two sides of a triangular chicken pen. The third side will be on the north side of the barn. One fence length makes a 75° angle with the barn. How many different pens can she build if one fence is attached at the corner of the barn? What are all the possible lengths for the barn side of the pen?

Not sure what they are asking here... please show step by step what to do! Thank you so much!

Answered by Penny Nom.
The volume and radius of a sphere 2014-04-27
From grace:
How do you find the radius of a sphere when all you know is the volume?
Answered by Penny Nom.
The derivative of sin(x) 2014-04-26
From Lucky:
f(x)=Sin(x), by first principle its f'(x)...show me how to solve such problem.
Answered by Penny Nom.
A schedule for 9 teams on 3 fields for 18 days 2014-04-25
From Scott:
I need to schedule 9 teams on 3 fields for 18 days with no buys. Two teams play while other practices in field(little guys). Two fields have backstops and other is open field. Would like to have even time on each diamond if possible. Thank you
Answered by Victoria West.
Solve for theta 2014-04-25
From ALASTAIR:
Hi, The question asks Solve for 'theta' cos2theta=sintheta x costheta. Substituting for cos2th either 1-2sin^2th or 2cos^2th does not give an equation in either sine or cosine alone how do I solve this please?
Answered by Penny Nom.
What was the selling price? 2014-04-25
From Con:
Apple built 40 32GB wifi iPads, gave away two in a contest, and sold the rest at two times the manufacturing price. If the company's total profit was $10,782, what is the selling price of one of these iPads?
Answered by Penny Nom.
A quadratic equation given the roots 2014-04-23
From Sarah:
How do you find the quadratic formula when you're given just the roots ... X=-3 X= 1/3 Thanks !
Answered by Robert Dawson.
Two 9 hole golf leagues 2014-04-23
From Sally:
I need help scheduling two 9 hole leagues. Here are the variables:
- Leagues alternate each week starting on the front and back
- League A starts at 4:30 with 7 groups and League B starts at 5:15 with 8 groups (all use carts and must be teed off by 5:30)
- Regular play must continue, customers who have started before league play begins have preference to continue their round whether they are playing 9 or 18 holes.
Thank you!!!

Answered by Victoria West.
Golf for 6 2014-04-21
From barbara:
We are a group of 6 golfers. We play in groups of 3 for 5 days. How can I arrange the groups that everybody plays with everybody else at least once? Thanks.
Answered by Victoria West.
Golf with 3 teams of 20 2014-04-21
From Joshua:
Thank in advance for this great service

Golf - new - 3 teams of 20 - to create 20 unique groups of 3

For example use 3 teams of 3

1a 2a 3a
1b 2b 3b
1c 2c 3c

So groups are
1a 1b 1c
2a 2b 2c
3a 3b 3c

Groups can only have one a, one and one c - and that combo should be unique

Answered by Victoria West.
A 22 team golf league 2014-04-21
From Terry:
We have a team league at our local golf course. We have 22 teams, we would like it so that each team plays the other team only once. I have tried the scenario.
1,21;2,20;3,19;4,18;5,17;6,16;7,15;8,14;9,13;10,11;12,X X+1=1 scenario
2, 1;3,21;4,20,5,19,6,18;7,17;8,16;9,15;10,14;11,13;1,X
using formula from other post in your forums.

I can get all the possible playing arrangements. The big quandary is setting up the schedule so that each week, each team would start on a different hole.

I can get the first 11 weeks scheduled very easily, it is just trying to get the last 11 weeks scheduled?

Any help would be greatly appreciated.

Answered by Victoria West.
Golf League Schedule for 15 weeks and three flights 2014-04-21
From bigdog:
I need help setting up a 12 team (3 flights with 3 teams in each flight) 15 week league play schedule. Each team must play teams in there perspective flight twice during the 15 week season.
Answered by Victoria West.
Simultaneous equations with fractions 2014-04-19
From Maryam:
I looked at your example of simultaneous equations with fractions and applied it to my question from an educate exam papers but I couldn't get it to work. The question is:

x/8 - y = -5/2
3x + y/3 = 13

Answered by Penny Nom.
A frustum of a pyramid with a square base 2014-04-18
From tuba:
a pyramid has a base of 10 m and is 15 m high.what is the volume? if 6m is removed from top what is the volume of the remaining frustum?
Answered by Penny Nom.
Ratios 2014-04-18
From Kenneth:
Should ratios contain only whole numbers and not any fractional numbers?

For example, 8:1, 2:5 have whole numbers. The ratios 2.5:10 and 1:4.5 do not have just whole numbers entirely.

If it is incorrect to express a ratio with fractional numbers, why are complex fractions correct?

I thank you for your reply.

Answered by Harley Weston.
Ratios 2014-04-18
From Kenneth:
Should ratios contain only whole numbers and not any fractional numbers?

For example, 8:1, 2:5 have whole numbers. The ratios 2.5:10 and 1:4.5 do not have just whole numbers entirely.

If it is incorrect to express a ratio with fractional numbers, why are complex fractions correct?

I thank you for your reply.

Answered by Harley Weston.
What is this function? 2014-04-17
From Keta:
What is this function?
-12=-2
-6=-1
6=1
0=0

Answered by Penny Nom.
Barrels of oil 2014-04-16
From daryl:
Looking for an accurate number to use when calculating barrels per inch in a a tank with the following dimensions:
Nominal diameter - 15'-6", nominal height - 16ft, nominal capacity - 500 barrels. These are commonly referred to as 500x16 tanks.
Currently we are using 2.6 barrels per inch. We need 180 barrels for a full load so I'm trying to figure out how many total inches I need to load 180 bbl. This has been a matter of debate for some time. Thanks for your help.

Answered by Harley Weston.
A tangent of the curve (x/a)^n+(y/b)^n =2 2014-04-15
From sudhir:
the equation of tangent of the curve (x/a)^n+(y/b)^n =2. at(a,b) is
Answered by Penny Nom.
The cost before the sales tax 2014-04-13
From Juanda:
Hello,

I know the customer cost with tax and I know the sales tax.
How do I find out the customer cost prior to the added sales tax?

Thank you

Answered by Penny Nom.
Manipulating an equation 2014-04-12
From Paul:
S=4wl+2wh and solve for h
Answered by Penny Nom.
A golf league with 20 players over 17 weeks 2014-04-12
From Chris:
I have a golf league 20 players and 17 weeks. We don't have teams so I just want to have everyone to play with each other the about same number of time and, hopefully, randomly distributed throughout the season. We have had situations in the past where two people play together for three weeks in a row and then not anymore for the rest of the season. From other posts, I see how to schedule the first 5 weeks without repeats but I can’t figure out an algorithm to for the rest of the season. Thanks!
Answered by Victoria West.
Decimals,fractions and percentages 2014-04-11
From Frances:
Ask an expert to find out their real life usage of your topic... that is the question. meaning what is your real life usage of using decimals,fractions and percentages. This is all part of my math assignment to ask an expert real life usage... so please reply back as soon as possible
Answered by Penny Nom.
A golf league consisting of 14 two man teams 2014-04-10
From Kevin:
Hello, I need to make a schedule for a golf league consisting of 14 two man teams playing over the course of 20 weeks. I understand that each team will play at least once and I believe 7 teams will play each other twice over the course of the season. How would I generate this schedule. Thank you, Kevin Perry
Answered by Victoria West.
The surface area of a circular dome 2014-04-10
From Shafiqah:
Is this a dome's surface area formula??
{{2 × π r × h square units}}
Is the surface area of the floor for the dome is calculate too in this formula?

Thanks for answering. =)

Answered by Robert Dawson and Penny Nom.
The surface area of a rectangular prism 2014-04-08
From Manraj:
Find the surface area of a rectangular prism with dimensions 4, 5, and 7 units.
Answered by Penny Nom.
The sides of a triangle 2014-04-06
From Michael:
I am supposed to sove for the length of side "b" of an irregular triangle. I am given the following:
Side a: 65'
Side b: Find this length
Side c: 50'
Angle A: unknown
Angle B: unknown
Angle C: 52 degrees
I am supposed to use the law of cosines to solve for side "b" and my teacher says there is no mistake in the "givens" for the problem. I do not see how this can be done using the law of cosines and i have not figured out how to sove for angle B to use the law of cosines.

Answered by Penny Nom.
The locus of a point 2014-04-04
From srishti:
A point P moves such that the difference between its distance from the origin and from the axis of x is always a constant c . what is the locus of the point?
Answered by Penny Nom.
3/4lbs of nails fills a container 2/3 full 2014-04-04
From Tyler:
If 3/4lbs of nails fills a container 2/3 full, then how many lbs of nails will fill the container?
Answered by Penny Nom.
Golf for 8 playing 7 rounds 2014-04-03
From Evan:
We are going on a golf trip to Scotland and are trying to work out the pairings. There are 8 of us playing 7 rounds. We would like to be partnered up in 2's with everyone once and have evenly split up foursomes. Thanks for the help
Answered by Victoria West.
A schedule for an 8 man golf team playing 4 each week 2014-04-01
From Harold:
We have an 8 man golf team and wish to schedule only 4 to play each week and to have each member play with one of the other 7 at least once. What is the simplest method. Number of weeks could be up to 18 or so.
Answered by Victoria West.
A cubical tank 2014-04-01
From renzo:
A cubical tank contains 2048 cubic ft of water when HALF-FULL. Find the total area of the tank and the length of its diagonal
Answered by Penny Nom.
Reducing the size of a poster by a scale factor of 1/3 2014-04-01
From deeshon:
eric is making a smaller copy of a poster that measures 20.4 cm wide by 34.2 cm long.he reduces the original by a scale factor of 1/3.what are the dimensions of the copy
Answered by Penny Nom.
Golf with 20 players and one compulsory bye 2014-03-31
From Wayne:
I'm trying to run an event with 20 players, I would like if at all possible to limit amount of people playing repetitively together. (Mix it up and play with different people). Here's the conditions. I want groups of 4 over 5 rounds. On each round I want 4 players to drop out and have a bye (to help run the game) but I dont want them helping a group they've played in if I can help it. Is this possible ?
Answered by Victoria West.
The Pythagorean Theorem 2014-03-30
From brenae:
The Pythagorean Theorem, what is it?
Answered by Penny Nom.
A schedule for 6 people 2014-03-29
From John:
How do I set up a schedule where six people are here for ten of twenty days. Arranged in rotating groups of three, so everyone works with everyone else. Everyone works with everyone else at least once and everyone works ten days.
Answered by Victoria West.
Graphing piecewise functions 2014-03-29
From Rayven:
Hi! I'm in eighth grade, taking ninth grade algebra 1. I'm confused as to how to graph piecewise functions. I know that you have already answered a question similar to this (I did my research first) but it didn't completely help me on my homework. I have to graph piecewise functions for the specified domains, and create a table for the absolute values. I know that two bars around a number means absolute value (two bars around -2 makes it +2) , but how do I graph and chart the absolute value for the following:

f(x)= |x+3 | for -5≤x≤3

And then graph and chart: (on a separate graph):

f(x)= {x if x≤0
{x+1 if x <0

thank you!
~Rayven

Answered by Penny Nom.
Curvature of the Earth 2014-03-28
From Max:

Recently I read the answer to a question proposed by someone on this site.

The question : What is the rate of curvature per mile on Earth?
The answer given : Use Pythagoras' Theorem to solve for the answer, given a 1 mile side
and a side as the radius. The hypotenuse minus the radius is your answer of drop/mile or curve/mile.

My conjecture : Why go through all of that work if the distance is one? Something like
{1/diameter} would would fine for such a problem. Seems like a lot of work for no reason.

I understand the practical application of Pythagoras' Theorem in this certain situation, as you would need
to use a^2+b^2=c^2 for any distance greater than one [mile]..
It just seems excessive and unnecessary if you're solving for curve / one mile.


Answered by Robert Dawson.
12 golfers, 6 rounds 2014-03-27
From Bill:
I have a group of 12 golfers playing 6 rounds of golf. I am looking for a pairings schedule that allows each golfer to play at least one round in a foursome with each of the other eleven players.

Can you provide me with a solution?

Answered by Victoria West.
A table of values 2014-03-27
From Marisol:
A table value X 0,1,2,3,4,5,6 and Y 3,7,11,15,19,23,27 . what relationship of X to Y is represent by which equation? A . y= 4x. B y =x+3 . C y =4x+3 or . D y =4x-3
Answered by Penny Nom.
12 golfers, 24 weeks 2014-03-27
From George:
I have 12 golfers and want to set up a season schedule that will avoid duplication of playing partners as much as possible. The saeson is 24 weeks long. Do you have a formula?
Answered by Victoria West.
13 golfers in groupings of 4, 3, 3 and 3 2014-03-26
From Lennart:
How can I organise 13 players playing in groupings of 4,3,3,3 for 3 rounds of golf in a way that all play with all others but a minimum of times with the same players(?)If possible, nobody should play in the 4 ball more than once. I would be grateful for an answer/proposal! Thank you, LR
Answered by Victoria West.
A word problem with fractions 2014-03-26
From lois:
a man has a stack of boards, he cuts 3/5 off each board, after he cuts the boards he finds he has enough pieces left to make 4 boards the length of the original boards. how many boards did he start with.
Answered by Penny Nom.
The faces of a triangle 2014-03-24
From Westun:
Your answer says that a triangle has 0 faces but isn't the part of the shape you see the face!? Because i am having trouble with that so could you help me!
thanks! :)

Answered by Penny Nom.
9 golfers in groups of 3 2014-03-22
From Andrew:
We are a group of 9 golfers.
We play 4 rounds in 3 groups of 3.
What would be the most equitable where everyone plays with another at least 1 time...?. Thank you.

Answered by Victoria West.
Riding on a Ferris wheel 2014-03-21
From Claire:
A Ferris wheel has a radius of 72 feet and its center is 85 feet above the ground. At the top of the Ferris wheel, Sam is in the topmost car. If the Ferris wheel makes two complete revolutions per minute, how high above the ground will Sam be after 10 seconds?
Answered by Penny Nom.
An arithmetic sequence 2014-03-20
From Xabiso:
The 10th term of an arithmetic sequence is 28 and the 7th term is 19. Calculate the common difference and the first term of the sequence.
Answered by Penny Nom.
14 teams divided into two equal divisions 2014-03-18
From C:
We have 14 teams divided into two equal divisions. We want each team to play each team in their division one time and four teams from the other division one time. What is the schedule
Answered by Victoria West.
2 rectangular prisms are similar by a scale of 0.75 2014-03-16
From gina:
2 rectangular prisms are similar by a scale of 0.75. The volume of the smaller prism is 27cm3 (cubed). How many cubic centimeters is the volume of the larger prism?
Answered by Penny Nom.
Two overlapping arcs in a square 2014-03-15
From Jean:
I have a square with side 4 cm. There are two overlapping arcs going from vertex to diagonal vertex. The other two vertices are the center of the arcs, which are shaded. How do I find the area of the shaded arcs? The overlapping arcs when shaded resemble a long thin football
Thank you for your help.

Answered by Penny Nom and Walter Whiteley.
A cable around the Earth 2014-03-13
From fikile:
By how much must an equatorial cable be extended in order that it runs 1meter above the ground?
Answered by Penny Nom.
8 golfers for 7 rounds 2014-03-11
From Brian:
We have 8 guys playing 7 rounds of golf. We want to play with each player as a partner only once, sounds simple but i can not come up with the match ups beyond the first 4 matches. I believe the numbers should work perfectly where each player can play with another as a partner only once each throughout the week. Please help.
Answered by Robert Dawson.
24 golfers playing 3 rounds 2014-03-10
From Jim:
How can I rotate 24 golfers playing 3 rounds of golf so that none of them play with the same person twice.
Answered by Victoria West.
Eleven golfers 2014-03-08
From Leon:
I have a group of 11 golfers wanting to play 10 rounds of golf in grouping of 4,4,3 .What is the best solution so that everyone plays each other as many times as possible
Answered by Victoria West.
The angle of depression 2014-03-08
From Ranger_minor:
A woman of height 1.4 metres standing on the top of a building 34.6 metres high views a tree some distance away. she observes that the angle of depression of the bottom of the tree is 35 degrees and the angle of depression of the top of the tree is 29 degrees.

assume that the building and the tree are on level ground :
1). calculate the distance of the woman from the top of the tree measured along her line of sight.

Answered by Penny Nom.
The equation of a circle 2014-03-07
From Balraj:
I have to draw a circle x^2+y^2 =8x . please tell me how to understand these coordinates ? please elaborate , on how to understand these types of equations.
Answered by Penny Nom.
29% as a fraction 2014-03-07
From kylie:
29% into a fraction or mixed number
Answered by Penny Nom.
Four digit phone numbers 2014-02-25
From Tom:
Hi, I'm an online math teacher, working on lessons for my students.

This question is in their book and I can't do it...! Help!

It has to do with phone numbers, and in this problem, we are only dealing with the last 4 digits of the number.

It asks how many possible numbers we have if at least one digit repeats in the last 4 digits of the number.

Answered by Penny Nom.
The diameter of the base of a cone 2014-02-23
From elwin:
i have a sector of a circle that has 120 degree and 6 cm length. What will be the diameter of the base of the cone.
and what is the diameter of the angle is changed to 180.

Answered by Harley Weston.
A word problem with fractions 2014-02-23
From alex:
In a certain fraction the number by n the denominator is greater than the number by3.If 2 is added to both the numerator and denominator ,the fraction is increased by 6/35..Find the fraction.
Answered by Penny Nom.
x^3+(1/x^3)=18√3 2014-02-23
From Asiv:
if x^3+(1/x^3)=18√3; then what is the value of x?
Answered by Penny Nom.
9 golfers divided into 3 teams 2014-02-22
From Steve:
We have 9 golfers, divided into 3 teams each day. Play is over 6 days. Can you devise a schedule to ensure pairings allow for all golfers to play at least once with all participants? Also, that the pairings have one player from another team.
Answered by Victoria West.
Three investment partners sharing the profit 2014-02-22
From Ayatullah:
Hello;

We are three partners and bought a property at 2050000 and sold at 2300000.
Details of investment of the partners
X= 1500000
Y= 400000
Z = 150000

My questions are
1) How could i calculate percentage of investment of each partner?
2) How could i deduct 30% commission from each partner in profit?
3) How could i distribute the profit amongst the partners?


Thanks in advance

Answered by Penny Nom.
A limit with trig functions 2014-02-22
From pearl:
(what is the value of limit of x as it approaches 0 of sin8x divided by cos6x)
Answered by Penny Nom.
A 7 person golf schedule 2014-02-21
From Claude:
Golf Holiday: 7 guys, play a 4-some and a 3-some each day for 10 days. Want to have the best balance of each player playing with every other player the same number of times and each player playing in an equal number of 4-somes and 3-somes.
Answered by Victoria West.
The growth of a tree 2014-02-16
From Emily:
If a tree is 10 ft tall after 2 years and 25 ft tall after 5 yrs , find the rate of change of growth Of the tree if y is the height in ft and x is the number of years.
Answered by Penny Nom.
What is the exponential form of 1/square root of 6v? 2014-02-16
From Prince:
What is the exponential form of 1/square root of 6v
Answered by Penny Nom.
Simultaneous fractional equations 2014-02-15
From benjamin:
hi math central. benjamin here. during class, i had problem with this topic. normally i wont have problems with math but this topic i just too hard for me. please help i am having exam and test next week on this topic

here is the question:
using substitution method, solve the simultaneous equation.
(x+1)/(y+2)=0.5

(x-2)/(y-1)=1/3

Answered by Penny Nom.
The grade of a road as a percent 2014-02-14
From Emily:
What is the grade of the road shown below expressed as a percent rounded to one decimal place?
Rise is 40feet
Run is 380 feet

Answered by Penny Nom.
Rolls of window film 2014-02-14
From Travis:
This question is probably close to the same question as "roll of paper"

We have Rolls of Window Film that we are trying to figure out an equation for a spreadsheet that we can use to "inventory" our window film.

We use a caliper tool to measure the thickness of the roll in millimeters.

the core thickness = 1.90mm
Full Roll thickness(including core) = 9.08mm to 9.12mm
Film thickness = 0.06

Full Roll of Film is supposed to average 1200" of film

What equation could we use to get the approximate inches left remaining on the roll if we measured the roll including the core with the Caliper tool in Millimeters?

Answered by Harley Weston.
Water in a conical funnel 2014-02-11
From Marcus:
Water is running out of a conical funnel at the rate of 1 inch^3/sec. If the radius of the base of the funnel is 4 in. and the altitude is 8 in., find the rate at which the water level is dropping when it is 2 in. from the top.
Answered by Penny Nom.
8 golfers, 11 rounds 2014-02-09
From Clare:
I am leading a group of 8 golfers who will be playing 11 rounds each at the end of next month and have been tasked with coming up with a schedule that allows, as close as possible, that has everyone playing an equal number of rounds with each other. Do you have any suggestions?
Answered by Penny Nom.
The volume of a cone 2014-02-08
From hibba:
why is the volume of right circular cone divided by 3?
Answered by Penny Nom.
A number with all the digits different 2014-02-07
From Jon:
I'm not a mathematician but just curious. Is there a name for a number of up to ten digits where each digit is different, i.e the equivalent of an isogram in linguistics? My online banking gives me a random 8-digit number each time I log on and it is rare to get one of the type I refer to. There must be calculable odds, but I'm only allowed one question!
Thanks in advance
Jon

Answered by Harley Weston.
Prime factorization 2014-02-06
From Kadeejah:
Write the prime factorization of 37 in exponential form
Answered by Penny Nom.
A triangle has angels in the extended ratio of 2:5:8 2014-02-06
From Rubina:
a triangle has angels in the extended ratio of 2:5:8. find the measure of all three angles?
Answered by Penny Nom.
Order of operations 2014-02-04
From Alex:
Hi there,i got problem with order of operation.Can you please help to find a solution for this?
Thank u beforehand
6-3(7+2)+(2+4)/3-4.

Answered by Penny Nom.
The volume of a frustum 2014-02-02
From mike:
volume of frustum R23", r 18", h 16"
Answered by Penny Nom.
Conics 2014-02-01
From Kassidy:
Hey, I have searched through all the questions about conics and how people use them in the real world, but none of them were very specific on how they are applied and the process, why it's so important etc. I have a project due asking these questions and it's been very difficult finding the right answer, if you could name jobs, how they are use and specifically applied that would be greatly appreciated.
Answered by Penny Nom.
Adding fractions 2014-01-31
From bianco:
2/3+3/4=
9/8-6/8=

Answered by Penny Nom.
An inequality 2014-01-25
From LANELL:
this is a problem to solve: 1/3 + 2/7 >=x/21 -- part of the answer is (-oo) not exactly that similar--it is on a calculator as a symbol- sure you know what it is I am talking about- the x will be a number
Answered by Penny Nom.
A word problem involving fractions 2014-01-23
From Pamela:
John put 1/4 of his flowers in a vase
He gave 1/2 to of the flower to his wife
He gave 1/5 of the flowers to his daughter
He has 8 flowers left
How many flowers did John start with

Answered by Penny Nom.
$9.50 in coins 2014-01-15
From dana:
you have $9.50 in coins. it has twice as many nickels as quarters and three times as many dimes as nickels. how many of each coin do you have?
Answered by Penny Nom.
Five cubes 2014-01-15
From Bob:
Rick has five cubes. When he arranges them from smallest to largest, the difference between the heights of any two neighbouring cubes is 2 cm. The largest cube is as high as a tower built from the two smallest cubes. How high is a tower built from all five cubes?
Answered by Penny Nom.
Fractions and exponents 2014-01-11
From john:
what is the exponent form of 9^4/7^-2 and 6^-3/10^-5
Answered by Penny Nom.
On what day of the week was July 4, 1904? 2014-01-11
From Madi:
Hi guys,
I have a question. July 4, 1903, was a Tuesday. On what day of the week was July 4, 1904? If you could give me an equation that would work for any question like this, that would be GREAT!
Your fellow math scholar (in training),
Madi

Answered by Penny Nom.
4x^3-7x^2-2x 2014-01-10
From Alvi:
4x^3-7x^2-2x
Answered by Penny Nom.
When would one flip the inequality sign? 2014-01-09
From Natasha:
Would one flip the inequality symbol in this equation: (explain why) (-9a)/(-9) > 81
And please explain in what circumstances one would flip the inequality sign
THANKS!

Answered by Penny Nom.
Profit as a percentage 2014-01-04
From Dane:
If i brought a card for £5 and sold it for £10 what is my profit percentage? 50% or 100%
Answered by Penny Nom.
25% profit 2014-01-02
From Finn:
Hello,
The question is all about buy-and-sell business.
Problem:
Pencil - $6 for whole sale price
     $8 if I sell the item How do I get the 25% profit? (you can change the whole sale price and the retail price[if i sell the item])
if I buy the pencil at 24 pieces and sell it at 24 pieces.

Answered by Penny Nom.
A square frame 2014-01-01
From steve:
using 6" wide boards how many inches would each of 4 boards equal in length need to be to produce a square with an inside perimeter of 84 inches on a side? what is the exterior perimeter? There is to be 45 degree cuts at each end of each board. What formula is used to make this calculation?
Answered by Harley Weston.
What does y= f(x) actually mean? 2013-12-31
From John:

I don't understand how to pick coordinates for y=f(x).

I took a look at your answer to a previous question here:

http://mathcentral.uregina.ca/QQ/database/QQ.09.00/monica2.html

What does y= f(x) actually mean?


Answered by Penny Nom.
The slope of the line y=-2x+5 2013-12-29
From victoria:
what is the slope of the line with the equation y=-2x+5
Answered by Penny Nom.
An infinite geometric series 2013-12-24
From Muhammad:
The sum of an infinite geometric series is 15 and the sum of their squares is 45. Find the series
Answered by Penny Nom.
Can 100r^2-81z^2 be factored? 2013-12-08
From Rosa:
Can 100r^2-81z^2 be factored?
Answered by Penny Nom.
Fractions and square roots 2013-12-04
From arionne:
How do you solve a square root with improper fractions like 121 over 49
Answered by Penny Nom.
Problem solving with fractions 2013-12-03
From Jennie:
Paul has 2/3 as many postcards as Shawn. Shawn has 3/5 as many postcards as Tim. If the 3 boys have 280 postcards, how many more postcards does Tim have than Paul?
Answered by Penny Nom.
Exponential form 2013-12-01
From Alina:
My question says to express each number in exponent form in as many different ways as I can; 16, 81 and 64. How do I do this?
Answered by Penny Nom.
Factor 2013-12-01
From Rosa:
How do you factor 9x^2 - 12y^2?
Answered by Penny Nom.
What is the slope of this line? 2013-12-01
From Charlene:
What is the slope of this line?
Answered by Penny Nom.
0.35 as a fraction 2013-11-28
From Laura:
How do you write 0.35 as a fraction or mixed number?
Laura

Answered by Penny Nom.
Coefficients 2013-11-27
From kim:
in the expression below, what number is the coefficient of y? y2+3y+4
Answered by Penny Nom.
Three roommates share the rent 2013-11-23
From Cortlin:
Because of different bedroom sizes, three roommates decide to split the rent this way: Roommate A pays 75% as much as B does, and C pays 40% as much as B does. What fractional part and percent of the rent does each roommate pay?
Answered by Penny Nom.
A schedule for 32 golfers 2013-11-22
From Scott:
I would like a schedule for 32 guys playing in 8 foursomes for 13 week. I would like it so that no one plays in the same foursome more than once or twice. Is this possible to do??? Can you let me know at bogeysm@yahoo.com if you can do it and if you will do it.
Answered by Victoria West.
Adding mixed numbers 2013-11-20
From Kathy:
1 3/4 + 1 2/3= ?

5 1/2 - 2 5/6= ?

Answered by Penny Nom.
Scheduling 10 people for 5 rounds 2013-11-20
From Keith:
Using golf as an example, I would like to schedule 10 people for 5 rounds with 2 groups of 4 and 1 twosome each round. I would like to minimze the number of times any one player plays with another. Can you provide an optimal schedule? Thank you for your help.
Answered by Victoria West.
How many cords of wood is this? 2013-11-17
From melody:
5.5 ft wide and 24ft long and 10ft wide how many cords of wood is this?
Answered by Penny Nom.
Inflection Point 2013-11-15
From Z.:
I was wondering if the function F(x)= x/ln(x) had an inflection point? If yes, why it's not visible on the graph of the function?
Thanks.

Answered by Penny Nom.
A lever 2013-11-09
From Durgesh:
A farmer uses a 2 m long lever to lift large rocks from the ground of his field . He places a fulcrum 20cm from the rock end of the lever and then stands on the other end, levering out the rock. If the farmer weighs 100 kg , what is the heaviest mass of the ro ck that he can lift using this method? (how to solve)
Answered by Penny Nom.
Water flowing out of a tank 2013-11-03
From Carolyn:
The flow of water out of a hole in a tank is known to be proportional to the square root of the height of water above the hole. That is,

dV/dt (proportional to) sq root (h)

The tank has a constant cross-sectional area A, show that the height of water in the tank is given by

h = ((-kt+C)/2)^2

If the tank is 9 metres high, and it takes 5 hours for it to drain from full to half full, how much longer will we have to wait until it is completely empty?

Answered by Penny Nom.
A sloped concrete wall 2013-10-30
From Brad:
We are planning on pouring concrete in a wall form that is sloped. One end is 73 inches tall, and it tapers to 24 inches tall over a distance of 348 inches. It is flat on bottom and sides and is 12 inches thick. Thanks.
Answered by Penny Nom.
Is-5/-11 a positive rational number? 2013-10-30
From tazneem:
Is-5/-11 a positive rational number
Answered by Penny Nom.
The perimeter of a rectangle 2013-10-24
From sana:
if the length is given 70 m and the breadth is given as 40 m find the perimeter of the rectangle
Answered by Penny Nom.
A stem and leaf plot 2013-10-24
From mary:
Hi
I'm trying to answer my child's question on stem and leaf plot. For grade 4, the question is where do you put 57 in this list

6    0
5    4 8 8 9 4 6 4

Answered by Penny Nom.
An equation with fractions 2013-10-22
From Bill:
1/5n= -9
Solve this equation in simple steps!

Answered by Penny Nom.
Factor strings 2013-10-14
From Jane:
What's a factor string? I'm confusey in the head.
Answered by Penny Nom.
A frustum 2013-10-12
From Lily:
A cone of height 6in. and radius of base 4in. has its top cut off by a plane parallel to its base and 4in from it. Find the volume of the remaining frustum.

I have worked out the volume of the entire cone but I don't know how to work out the radius of the top of the frustum.

Thanks

Answered by Penny Nom.
The domain of a derivative 2013-10-10
From Renee:
I am looking to find the domain of a derivative of a radical function, one such as: f(x) = the square root of (8 − x).
I am kind of unclear on how domains work for derivative. I don't understand how you take a function's domain and use that to find the derivative's domain.
Thanks!

Answered by Penny Nom.
Four tangent circles 2013-10-09
From Nilesh:
Four circular cardboard pieces, each of radius 7cm are placed in such a way that each piece touches two other pieces. How to find the area of the space enclosed by the four pieces?
Please let me know.

Answered by Robert Dawson.
An Arithmetic Progression 2013-10-08
From collins:
In an A.P. the difference between the 8th and 4th term is 20 and the 8th is one and half times the 4th term... what is the common difference and the first term
Answered by Penny Nom.
A line through (5, 1) 2013-10-08
From allison:
find an equation in standard form for line passing through point (5,1) and perpendicular to line x=4
Answered by Penny Nom.
A triangle problem 2013-10-02
From raneem:
ABC is a triangle in which : BC=20cm. M(<B) =29 and m(<C)=73 . D is the midpoint of BC Find the length Of AB and AD approximated to 2 decimal places
Answered by Penny Nom.
Problem solving 2013-10-02
From naomi:
My son having trouble a math problem please help with this question a bag of cough drops contains 36 drops. the fraction of each flavor is shown in the table strawberry 5/12 honey 1/12 cherry 1/6 mint1/9 and lemon 1/4 which flavor is the greast in the bag
Answered by Penny Nom.
Two teams of 4 playing 4 rounds of golf 2013-09-29
From Tom:
We would like to play a ryder cup format with two teams of 4 playing 4 rounds of golf. Three of the rounds would be pairs playing each other and one round would be singles. How can we set up the foursomes so we balance the number of times we play with every other golfer.
Answered by Victoria West.
12 golfers for 7 rounds 2013-09-25
From Dennis:
I have 12 golfers playing a round of golf for seven days. How can I set-up teams so that everyone plays with everyone at least once in those seven days. Thanks!
Answered by Victoria West.
6 golfers in threesomes over 7 rounds 2013-09-24
From David:
I have 6 golfers, (2 threesomes) who will be playing 7 rounds of golf. What is the best way to pair the 6 players so they play with as many different players over the 7 rounds?
Answered by Victoria West.
Price, revenue and profit 2013-09-22
From lorraine:
What price maximizes revenue? What price maximizes profit?

The only data I'm given is total output, total revenue, and total cost. I'm not sure how to set up a formula

Answered by Penny Nom.
The number of cords on a pile of wood 2013-09-18
From Gary:
a cord of wood is 4 feet by 4 feet by 8 feet, how much wood is a row that is 16 in by 5 foot by 32 feet?
Answered by Penny Nom.
If the earth was made of a ball of string 2013-09-18
From Tammy:
My 9 year old son asked me a question and I have no idea how to work it out! Please can you help?

If the earth was made of a ball of string and you unravelled the string how long would it be? Assuming the string is 5mm thick.

Answered by Penny Nom.
A garden path 2013-09-06
From mary:
garden path has 8 white slabs, four of the slabs are one third grey, what area of the path is grey
Answered by Penny Nom.
6 golfers in threesomes 2013-09-04
From Gregg:
We have 6 golfers playing in threesomes for three rounds of golf. I would like each golfer to play equally with the other 5. How can this be done.
Answered by Victoria West.
7 golfers, a threesome and a foursome 2013-09-04
From Scott:
I'm leading a golf trip with 7 guys, and we're playing 5 rounds of golf together with a threesome and a foursome at each course..

I want to set up a schedule where each person plays at least two rounds with each of the other 6 guys.

Is that possible? I can't seem to find the right combinations.

Thanks for your help!

Answered by Victoria West.
14 golfers in two person teams 2013-09-04
From Stewart:
How to rotate 14 golfers in two person teams without duplication until all have been paired at least once. Play is once per week.
Answered by Victoria West.
A schedule for 18 golfers 2013-08-29
From Brian:
After searching the data base which helped me solve at least 3 other issues I have one more un answered question. I have 18 golfers split into 6 threesomes for 4 days of golf. What is my best option for a schedule mixing them all up as much as possible over the course of the 4 days
Answered by Victoria West.
More than half of the people said... 2013-08-26
From Lucy:
Hi,
The question is:
Samoa 23%
Brazil 41%
Sri Lanka 52%
Senegal 39%
Armenia 37%
Croatia 20%
Why is the following statement true for the data above: More than half of the people said they were interested in Sri Lanka. The reason I don't understand the problem is that because the percentages add up to be greater than 100%, that means that the 500 people(as stated in the question) who participated in the survey got to pick more than once. Therefore, it is not 52% out of 100% anymore. Thanks for taking the time to answer my question.

Answered by Robert Dawson.
More on the scheduling of 16 golfers in 4 rounds 2013-08-26
From Peter:
I see your formula for 16 players 4 rounds.
Is it possible to get the players into different groups each day.
E.G Player 1 in group 1 the first day,
Group 2 the second day,
Group 3 the third day,
Group 4 the fourth day.
Then so on through the players.

Answered by Victoria West.
Constructing a triangle 2013-08-22
From Nazrul:
The base, the difference of the angles adjoining the base and the sum of the other two sides are given. How can I draw the triangle?
Please help me.

Answered by Chris Fisher.
Golf for 11 2013-08-14
From Don:
2 foursomes and 1 threesome for 6 rounds of golf
Answered by Victoria West and Harley Weston.
Using trig to find the height of a hill 2013-08-14
From Anna:
From the top of a hill, the angles of depression of two successive milestones on a level road, which leads straight away from the hill, are 5degrees and 15degrees respectively. Fine the height of the hill.
Suggestion: BE is drawn perpendicular to AD. Find BE, then BD, finally CD.

Thanks :)

Answered by Penny Nom.
A system of equations 2013-08-13
From Nina:
3x-2y=8
-x+3y=5
What is the solution of the system of equations

Answered by Penny Nom.
An equation in slope-intercept form 2013-08-12
From Brittney:
write an equation in slope-intercept form of the line that is parallel to the grapg of y=-3/4x+6 passes through -8,-5
Answered by Penny Nom.
Differentiate x^x - 2^sinx 2013-08-09
From tarun:
derivative of x^x - 2^sinx
Answered by Penny Nom.
Four equations 2013-08-08
From may:
HI how to solve this 4 equations?
A+C = 0
-4A+B-8C+D=1
3A+16C-8D=-29
-12A+3B+16D=5

Answered by Robert Dawson.
Practical uses of trigonometry 2013-08-06
From tharindu:
use of trigonometry
Answered by Penny Nom.
A golf tournament for 12 people 2013-08-01
From Brad:
I've looked all over the web for a solution to this (including searching the archives here) to no avail.

We're doing a golf tournament with twelve people. Each person has a "handicap" that demonstrates their skill level. We have these numbers for each of the 12 players. We are trying to figure out how to schedule two rounds of golf with the most balance possible.

- We are playing two "real" rounds of golf (18 holes). There are four people per group.
- We want to split each round into two 9 hole games, which gives us a total of 4 games
- Halfway through (after 9 holes), you switch partners within the foursome that you're playing in.
- We want everyone to play with 4 unique different teammates

So say you start the first 18 hole round as a group of Players 1, 2, 3, and 4. Players 1 and 2 are a team, and players 3 and 4 are a team. After 9 holes, Player 1 would either be paired up with Player 3 or 4, and the other two players would pair up as well. Then the next day we will mix up the foursomes and do the same thing. In terms of just pure scheduling this is simple. But I'm trying to figure out how to best balance the handicaps (simple average of each team's two handicaps works here). Assuming I'll need some sort of program or spreadsheet. Any help or even a push in the right direction would be a tremendous help. Thanks in advance to anyone who wants to give this a shot.

Answered by Victoria West.
Golf for 16 2013-07-31
From Graylin:
I am going on a golf trip and we have 16 players and need to know a formula that will allow us to all play together at some point in the 4 rounds we are playing. If not possible please provide best case. Thanks.
Answered by Victoria West.
16 golfers 2013-07-31
From Pat:
16 golfers...3 nine hole rounds a day...for 4 days. what are the best pairings for the fewest repeats of partners?
Answered by Victoria West.
Ann has a bag of marbles. 2013-07-25
From Carmen:
Ann has a bag of marbles. she gave one half to her friend Tina and then one third to her friend Tom. Now left with 15. How many did she had originally?
Answered by Robert Dawson and Penny Nom.
What is the smallest number? (i.e. the closest number to zero) 2013-07-22
From Charlie:
What is the smallest number? (i.e. the closest number to zero)
Answered by Harley Weston.
18/y=30/25 what number does y represent 2013-07-20
From celeste:
18/y=30/25 what number does y represent
Answered by Penny Nom.
10 guys playing 8-9 hole rounds 2013-07-19
From Jason:
I have 10 guys playing 8 9 hole match-play rounds. I need to have everybody play the other 9 men, but need to limit them to only a 9 hole match between them. If we need to go 3-3-4 or 2-4-4 for a better solution, that may be possible.
Answered by Victoria West.
150 litres has been drawn from a tank 2013-07-14
From henry:
When 150 litres has been drawn from a tank, it is 3/8 full, how many litres will the tank hold?
Answered by Penny Nom.
Dirt to fill a pool 2013-07-13
From Neil:
I had a 24 foot diameter pool. The perimeter of it was at ground level. The pool sloped 1 foot deeper to the middle. In other words "a 1 ft. dish. How many cubic yards of dirt do I need to fill this hole?
Answered by Penny Nom.
Scheduling a 12 person fishing 2013-07-09
From Don:
Greetings Math Central!
I searched the index, but could not find a similar quandary.
Here's my predicament:
I am seeking help in scheduling a 12 person fishing outing.
We have four boats.
I would like to schedule three fishermen to a boat.
We fish 2 1/2 days, split into morning and afternoon sessions.
Thursday afternoon, Friday morning, Friday afternoon, Saturday morning, and Saturday afternoon
We switch boats after lunch, with each fisherman moving to a different boat, with different partners.
I would like to have each fisherman fish with as many others in the group as possible, with a minimum of partner duplication.
I would like to have each fisherman fish in all of the four different boats over the course of the trip at least once.

Would you please suggest a boat / fishing partner schedule that fits my parameters as closely as possible.

Thank you!

Don

Answered by Victoria West.
4 couples golfing 2013-07-06
From Brian:
We have 4 couples going on a 4 day golf vacation playing 4 rounds of golf. I have spent hours trying to set up a schedule that allows the 4 spouse to play together, and then each spouse to play with one of the other spouses (men with women) for the] other 3 rounds.
I would like the foursomes to be different as possible. Also, no-one should RIDE in a cart with the same person more than once.
I am not a math guy so I try to do this by working it out on paper, over and over again. It ain't working!!
If you can help, I am very thankful.
Brian

Answered by Victoria West.
The angle of elevation of the sun 2013-07-03
From Maurice:
A vertical pole with a length of 7m cast a shadow with a length of 5m. Calculate the angle of elevation of the sun and include a diagram.
Answered by Penny Nom.
8 golfers playing 8 games 2013-07-03
From Johan:
We have 8 golfers playing 8 games, and we want to schedule them so that they play together equal amount of times, or as close as you can get to that. They will be playing in two four balls each day. Anyone sorted this one out yet, I will really appreciate a solution? Thanks Johan
Answered by Victoria West.
13 golfers playing 5 rounds 2013-07-03
From Alan:
i have 13 golfers playing 5 rounds. want everyone to play together at least once.
Answered by Victoria West.
13 golfers 2013-07-02
From James:
13 golfers playing in 4 groups [4,3,3,3 per group] for four rounds. Can it be scheduled so that no two people play together more than twice?
Answered by Victoria West.
Fantasy football league 2013-06-18
From Chris:
Hello, I am the commissioner of a fantasy football league. In this week, we play all 17 weeks of the season. It's a 12 team league, and we play each team twice. To do that, we have 5 "double header" weeks, which means you play 2 different teams. I am struggling to come up with a schedule to make this work. Does anyone have a solution?
Answered by Victoria West.
{(1+x)^1/3-1/3X(1+x)^-2/3}/(1+x)^2/3 2013-06-17
From STEPHEN:
{(1+x)^1/3-1/3X(1+x)^-2/3}/(1+x)^2/3
Answered by Penny Nom.
a cube-b cube-a+b= ? 2013-06-16
From saryu:
a cube-b cube-a+b= ?
find the answer

Answered by Penny Nom.
16 golfers in 2 groups of 8 2013-06-15
From Derek:
we are 16 golfers.2 groups of 8."A" group & "B" group.We are playing 4 rounds,4 players at a time,2 from each group.We don't want to play with or against the same person twice.IE we want to play with 7 different people each day.Is it possible ?-thanks-Derek
Answered by Victoria West.
4=(1+.08)^x 2013-06-15
From Samantha:
I can't seem to understand how to solve for x when it is in power form, here is the equation:

4=(1+.08)^x

Answered by Penny Nom.
The surface area of a pyramid 2013-06-11
From Sam:
A right rectangular pyramid has a rectangular base measuring 18 cm by 32 cm. The height of the pyramid is 12 cm. What is the surface area?
Answered by Penny Nom.
Two numbers whose difference is 16 2013-06-08
From Johan:
Two numbers whose difference is 16 and whose sum is 120
Answered by Penny Nom.
Four circles 2013-05-29
From varsha:
four circular cardboard pieces each of radius 7cm are placed in such a way that each piece touches two other pieces. find the area enclosed by the four pieces.
Answered by Penny Nom.
21 golfers 2013-05-27
From Linda:
A database search for a question on how to schedule 20 golfers in groups of 4 was very helpful ( Mona's answer referred to by C. Fisher 9-3-2008) I'm very glad I found this sight. Our problem is we will have 21 golfers in the second half of the season. Is it possible to figure out that schedule? I assume we will have to include byes, but I'm not sure how to go about doing that. Thank you.
Answered by Victoria West.
Inverse trig functions 2013-05-19
From ky:
hello, so iv'e been asked to draw a triangle with sides of 3, 4, and 5. And find the measure of all three angle using sin-1, cos-1, tan-1. I got really confuse, I'm taking the SAT pretty soon and it would be great to get this... THANX
Answered by Penny Nom.
The square footage of a 17 inch by 17 inch tile 2013-05-18
From Leroy:
How do I calculate the square footage of a 17inch by 17 inch tile
Answered by Penny Nom.
(4- 4cos^4 x)/(sin^2 x) 2013-05-18
From Agnes:
How I can solve this question :

Simplify (4- 4cos^4 x)/(sin^2 x) and write in terms of sin x

Answered by Penny Nom.
Inverting fractions and percentage 2013-05-18
From Carlos:
If I calculate the percentage change between two fractions, say, 1/100 and 2/100, I get 100% change. Now, if I flip denominator and numerator, 100/1 and 100/2, and repeat the calculation, I get a 50% change. Why are the percentages different?
Thank you very much

Answered by Penny Nom.
The percentage of fat in a container of yogurt 2013-05-17
From Katy:
There is a question that I' having trouble with. It says: "A container of yogurt contains 228 g of water, 54 g of carbohydrates, 12 g of protein and 6 g of fat, what percentage of the yogurt is fat?" How do you convert 6 g into percentage?
Answered by Penny Nom.
Cutting three sections of pipe 2013-05-16
From Michael:
A plumber cuts three sections of pipe from a 12’ length of ABS pipe, the lengths of the sections are 33 3/8”, 56 5/8” and 39 7/8”. What is left over from the full length, if the saw cut is 1/8” wide?
Answered by Penny Nom.
HCF and LCM 2013-05-15
From Kelly:
If HCF and LCM of two numbers is 7 and 20 respectively, then the number/s is/are….
Answered by Penny Nom.
Four digit combinations starting with 7 6 2013-05-15
From Kaye:
How do I find the rest of a four digit combination starting with 7 6?
Answered by Penny Nom.
An equation in two variables 2013-05-14
From Steve:
Verify solutions to an equation in two variables. 4x-2y=8 (3, 2)
Answered by Penny Nom.
Compound interest 2013-05-10
From Kyla:
I am doing correspondents and cannot figure out how to explain the solution to this question, I do not understand how they came to this answer and need it explained step by step so I can complete the following questions in this unit, please help!!! Calculating compound Interest

Johnny invest $800 that pays 8% compounded quarterly for 5 years. How much is the investment worth at the end of the 5th year?

Answered by Penny Nom.
Water use in a rectangular flush tank 2013-05-10
From milo:
A rectangular flush tank 22" by 71/4 contains water to depth of 17" how many gallons of water will be saved if a conservation device reduces the capacity to 3/5 of this amount? And reduced to the nearest tenth
Answered by Penny Nom.
20 golfers over 5 days 2013-05-09
From Russ:
we have 20 golfers over 5 days, with 4 people per foursome which makes 5 groups each day. I would like to each person to play with everyone at least once if possible. Do you have a formula for this?
Answered by Victoria West.
The surface of a solid 2013-05-07
From mustafa:
I am doing a research paper on solid surface applications . Part of the project is to find four at least in our world today and explain what there purpose is. I really need help in this area because I've been searching the internet for where solid surface are used in our world today and I really can't find anything.. Thanking you in advance, mustafa
Answered by Penny Nom.
Profit margin 2013-05-06
From Arron:
I have the retail price of $45.50. This price reflects a gross profit margin of 47%. How do I calculate the original cost of the item?
Answered by Penny Nom.
Points of intersection 2013-05-02
From Bianca:
Find the sets of values of k for which the line y=kx-4 intersects the curve y=x^2-2x at two distinct points.
Answered by Penny Nom.
How do i reverse this formula? 2013-04-20
From MK:
I have 22,000,000 worth of gold and wants to want to resell the gold back with a profit. Each transaction of gold will have a transaction fee of 15%, so if i resell the gold back at 22,000,000, I will only get back 18,700,000 after 15% transaction cut. The current formula is X-15%X = Y where X = 22,000,000 and Y = 18,700,000 Will u be able to come up with a new formula if we don't know the value of X but we know the value of Y
Something like this X-15%X = 22,000,000. How do i reverse this formula to find the value of X
Thanks

Answered by Penny Nom.
8 golfers, 3 rounds 2013-04-15
From justin:
I have 8 golfers going on a trip. We have 3 rounds to play. We would like to play with everyone (with or against.) Is this possible? If so could you send me the formula please and thank you!
Answered by Robert Dawson.
How do i write log_8(P) =7 in exponential form? 2013-04-14
From nancy:
How do i write this in exponential form log8P =7
Answered by Penny Nom.
Bags of pre-mixed concrete 2013-04-12
From Bruce:
Hello there,

I need two cubic yards of concrete for a playgound I am installing for my kids. That much i have figured out. I am stumped over the number of 25kg bags of pre-mixed concrete i will need to accomplish my goal.
Your help would be greatly appreciated.
Thanks
Bruce

Answered by Harley Weston.
Sharing in a car rental 2013-04-09
From Marilyn:
8 persons are planning to share equally the cost of a rental car. If 1 person withdraws from the arrangement and the others share equally the entire cost of the car, then the share of each of the remaining persons is increased by--------------?
Answered by Penny Nom.
10 golfers playing four rounds 2013-04-09
From Terry:
I've searched "golf" and "golf 10" but don't see 10 golfers playing four rounds (each round having one foursome plus two threesomes, and no "flights" as in one earlier answer); each player playing at least once with a maximum number of the other nine over the four rounds?
Answered by Victoria West.
The equation of a line 2013-04-04
From Miranda:
My coordinates provided are (6, 6) and (-8, 9) and i need to put this into ax=bx=c format. I remember the equation to find the slope of y2-y1/x2-x1 to get the slope. However when i put that back into the y=mx+b format it still leaves me with a lot of variables. Please help :)
Answered by Penny Nom.
A gravel pile in the shape of a triangular pyramid 2013-04-04
From Casey:
Hello
Right now I am stuck and I feel embarrassed because I feel like the answer is so easy I should know it.

I am working on a project and need to find a volume of gravel it will take to occupy this triangular prism like area. I am not sure what formulas I should use whether it be that for the volume of a pyramid or something more complex? Basically it forms a right triangle at one side then from there all points slope to one singular point about 10412mm away.
I am attaching a picture drawn up in paint with the actual dimensions to clear up any confusion.

Thank you for any help. Casey

Answered by Penny Nom.
Question 2013-04-04
From Casey:
Hello
Right now I am stuck and I feel embarrassed because I feel like the answer is so easy I should know it.

I am working on a project and need to find a volume of gravel it will take to occupy this triangular prism like area. I am not sure what formulas I should use whether it be that for the volume of a pyramid or something more complex? Basically it forms a right triangle at one side then from there all points slope to one singular point about 10412mm away.
I am attaching a picture drawn up in paint with the actual dimensions to clear up any confusion.

Thank you for any help. Casey

Answered by Penny Nom.
16 golfers over 15 days 2013-04-02
From Mark:
16 golfers, each plays the other 15 golfers once each over 15 days. played in four foursomes each day. What is schedule that allows each golfer to play with other golfers an equal number of times?
Answered by Victoria West.
We can't write sinx and cosx as a finite polynomial. 2013-03-31
From rimoshika:
prove that we can't write sinx and cosx as a finite polynomial.
Answered by Walter Whiteley.
Simultaneous equations with fractions 2013-03-31
From Terence:
5/x-6/y=1 17/x+30/y=16 I been spending whole day to solve this question. Would be very grateful if you can help I try The denominator value is a equations term which make is simultaneous equations so hard.
Answered by Penny Nom.
A 4 digit code 2013-03-26
From chris:
I saw a previous question about a 4 digit code type thing and I was wondering if you could email me that same type of thing. I changed the pin number on my phone's lock screen and I tried hard to remember it, then I had to go help my mom with something and I automatically forgot it and there is no way for me to get back on my phone. I need all of the possible 4 digit number combinations without the numbers 0, 1, or 2. It cannot be repeating, ascending, or descending.
I know you may not want to do all of them, but I really need it.

Answered by Robert Dawson.
A markup followed by a discount 2013-03-23
From edward:
A shopkeeper marks his goods to gain 35%. He allows 10% discount for cash payment. What is his profit percent when he sells the goods for cash?
Answered by Penny Nom.
Cumulative Frequency Math Question 2013-03-20
From Primadonna:

Hi,

Please help me solve this question.

Thank you so much.

The cumulative frequency table below shows the length of time that 30 students spent text messaging on a weekend.

Minutes Used Cumulative Frequency
31-40 2
31-50 5
31-60 10
31-70 19
31-80 30

Which 10-minute interval contains the first quartile?

(1) 31–40

(2) 41–50

(3) 51–60

(4) 61–70


Answered by Penny Nom.
Five players in a 4-player partnership game 2013-03-19
From Dee:
I have 5 players who are playing in a 4-player partnership game & want to rotate the 5th person in in such a way that each person is each other person's partner for a game. How can such a rotation be done?
Answered by Penny Nom.
Tiling a floor 2013-03-18
From whitley:
Question from Whitley, a student:

How many square feet of tile do you need to cover the floor of a room that is 20ft, 25ft, 15ft, 20ft, 5ft, and 5ft

I made a replica of the floor

Answered by Penny Nom.
An algebraic exercise 2013-03-18
From autumn:
given: f(x)=x/(x^2+2) determine: [f(x-h)-f(x)]/h
Answered by Penny Nom.
Row echelon form 2013-03-15
From Shawn:
Please help me put matrix in row echelon form:

1 9 8 0
5 8 1 35
1 -4 -1 17

Answered by Penny Nom.
Percent profit: State exam question 2013-03-12
From tony:
which is the correct way to figure 30% profit?
1)
1000 product
500 labor
50 permit
1550 total cost
1550x1.3=2015

2)
1000 product
500 labor
50 permit
1550 total cost
1-.3=.7
1550/.7=2214

Answered by Harley Weston.
state exam question 2013-03-12
From tony:
which is the correct way to figure 30% profit?
1)
1000 product
500 labor
50 permit
1550 total cost
1550x1.3=2015

2)
1000 product
500 labor
50 permit
1550 total cost
1-.3=.7
1550/.7=2214

Answered by Harley Weston.
Fencing a parcel of land 2013-03-11
From Koleen:
How much fencing do I need for an 56,850 parcel of land for a 15 stable horse barn
Answered by Penny Nom.
f(x) + f ''''(x)=0 2013-03-05
From Andreea:
Hei. I don’t speak lot of english but here is my question,hope u understand: f(x) + f ````(x)=0. so, my question. what is f(x), where f ````(x) is f(x) derivative by four time ? i tried to find the answer and i knew f(x) is something like that f(x)=e^x*sinx but miss something.
Answered by Brennan Yaremko.
Four digit "combinations" using 0124 2013-02-27
From Ayana:
What are all the possible four digit combinations using 0124. Please list them for me. Thank you.
Answered by Penny Nom.
A messy arithmetic expression 2013-02-24
From niiaryee:
(((((5^2*4^3)1/5+9^3/3)+(25+(4*5)/15)^2-10)/10/2)3)))))
Answered by Penny Nom.
The tangent to a circle at a point on the circle 2013-02-22
From Andrew:
What is the equation of the line tangent to the circle with equation x^2+y^2=25 at the point (-4,3)
Answered by Penny Nom.
Fractions over fractions 2013-02-22
From Kenssa:
Question from Kenssa, a parent:

Simplify: 3-(7/x)-(6/x^2) / (3/x^2)+(5/x)-2

Answered by Harley Weston.
Your profit should be 20% of your cost 2013-02-21
From anis:
if the customer says your profit should be 20% of your cost..

1) my selling value should be cost +20% of cost = SP

or

2)cost / 0.80 = SP which is correct...if the second answer is right how should i justify my customer

Answered by Penny Nom.
Related rates 2013-02-17
From Ishaak:
A hemispherical bowl is filled with water at a uniform rate. When the height of water is h cm the volume is π(rh^2-1/3 h^3 )cm^3, where r s the radius. Find the rate at which the water level is rising when it is half way to the top, given that r = 6 and the bowl fills in 1 minute.
Answered by Penny Nom.
The continuity of f(x,y)=ln(x^2+y^2) 2013-02-17
From anu:
the question says we have to find the points in the plane where the function is continuous: f(x,y)=ln(x^2+y^2) . here we aren't given a particular point (x,y) where we have to check a function's continuity. what is to be done if we have to check continuity over the whole domain of the function? please help .
Answered by Harley Weston.
The equation of a circle 2013-02-11
From mhd:
Complete the equation of the circle centered at(0,4) with radius 3
Answered by Penny Nom.
4*4+4*4+4-4*4=??? 2013-02-06
From Jacky:
4*4+4*4+4-4*4=???
Answered by Penny Nom.
What is the domain of f(x)=sin(ln(x))/ln(x)? 2013-02-06
From Behrooz:
Hi, the following problem may be interesting: What is the domain of f(x)=sin(ln(x))/ln(x)? Be careful, domain is not obvious. Best regards Behrooz
Answered by Penny Nom.
Exponential form 2013-02-05
From Sandra:
What is 37,008 in exponential form? Help me please!!!
Answered by Penny Nom.
We have 10 golfers and four rounds of golf in two days 2013-02-05
From Steve:
We are having a golf tournament. We have 10 golfers and four rounds of golf in two days. We are going to break the group of 10 into two flights. Flight A and Flight B. We would like all members in Flight A to play one nine hole match against each other. All players in Flight B to play one nine hole match against each other. Could we play in groups of 4,3 and 3? What is the best way to make the pairings work?
Answered by Robert Dawson.
7x-1/4-1/3[2x -1-x/2]=19/3 2013-02-03
From M:
7x-1/4-1/3[2x -1-x/2]=19/3
Answered by Penny Nom.
The fourth side of an irregular polygon 2013-02-01
From Emran:
I have a irregular polygon. I know 3 of the 4 sides, and 2 of the angles. A-B is 285, B-C is 149, and C-D is 310. Angle B is 135 degrees. and Angle C is 45 degrees. Is there a formula to solve for the final side? Thanks.
Answered by Penny Nom.
The multiplication table for the different bases 2013-02-01
From sylvia:
I am having a difficult time trying to figure out how to fill in the multiplication table for the different bases. i don't know how to get the numbers.
Answered by Penny Nom.
We have 2 teams of 6 couples each that play 6 games 2013-01-25
From Denise:
We put together a game night where we have 2 teams of 6 couples each that play 6 games. We haven't been able to figure out an arrangement that allows each couple to play each game with a different couple from the opposite team (i.e. Team A couples play every game with a different couple from Team B). Is this possible? It works with 2 teams of 5 couples each.
Answered by Chris Fisher.
Factor x^6 - 64 2013-01-25
From Taylor:
Hi, I am having trouble factoring x^6-64. Could you help me?
Answered by Penny Nom.
Golf for 17 2013-01-23
From Richard:
I have looked through the examples on your site but can't find one that suits. I am fiddling with one to see if I can get a result; but not confident. I have 17 players playing 6 rounds in 3 groups of 4 and 1 group of 5. Two players will not be included in any 5 ball group. Can you help me please.
Answered by Victoria West.
The zeros of a fourth degree polynomial 2013-01-23
From Dakota:
My problem has multiple steps and I have done everything but the last one. 8The original problem is f(x)=x^4-5x^3+7x^2+3x-10 and I have to find the zeros of the equation. I used synthetic substitution like my teacher taught us to get the equation of x^3-3x^2+x+5=0 but now I don't know how to get the zeros of that equation, or solve it.
Answered by Penny Nom.
Golf for 18 for 18 weeks 2013-01-22
From Al:
We have 18 two man golf teams and we play once a week for 18 weeks. We need a formula to schedule these teams so that they play each other one time only.
Answered by Victoria West.
Plotting y=f(x)+2 2013-01-21
From Gavin:
how would you plot y=f(x)+2
Answered by Penny Nom.
Maximize profit 2013-01-19
From Chris:
A firm has the following total revenue and total cost function.
TR=100x-2x^2
TC=1/3x^3-5x^2+30x
Where x=output
Find the output level to minimize profit and the level of profit achieved at this output.

Answered by Penny Nom.
The weight of a pipe full of water 2013-01-16
From joe:
how do you figure out the weight of a pipe full of water
Answered by Harley Weston.
Percent as a fractional relationship 2013-01-14
From Kenneth:
What does the following indicate?

"A percent is another way of expressing the ratio or fractional relationship of two numbers."

I know what a ratio is, but the term "fractional relationship" confuses me.

I thank you for your reply.

Answered by Penny Nom.
A shed roof 2013-01-12
From christine:
A roof on a shed is 7.3 ft wide has an incline of 20 degrees what is the height?
Answered by Penny Nom.
A trig identity 2013-01-04
From Tehmas:
Prove sinC+sinD=2sin(C+D/2)cos(C-D/2)
Answered by Harley Weston.
The quadratic formula 2013-01-03
From itsel:
Find the discriminant ans use it to determine the use the quadratic formula to solve the equasion -2x^2+3x+2=0
Answered by Penny Nom.
A 4 digit phone lock using the digits 4, 2, 3 and 0 2012-12-24
From Ragu:
I would like to know formula for searching a combination of 4 digit. eg.

I knew my Phone Lock combination is my car plate number 4230. But this time, i used another way around instead of 4230.

But I am sure, there is only this 4 digits only. 4 & 2 & 3 & 0

Answered by Penny Nom.
2 1/4 x 1/8 x 1 3/4 x 12 4/9 x 3 2012-12-21
From Tony:
Ok my question

2 1/4 x 1/8 x 1 3/4 x 12 4/9 x 3

I just can't figure out the question ?

Answered by Penny Nom.
3x6)+(12divided by2)-8= ? 2012-12-17
From christina:
hi my question that's been bothering me is what's
(3x6)+(12divided by2)-8= ?????

Answered by Penny Nom.
An area bounded by lines 2012-12-16
From sidra:
find area bounded by functions:
y=x
y=2x
and y=5-x

Answered by Penny Nom.
If n is odd, then n^2 - 3 is even 2012-12-11
From Tracy:
Prove the statement:

For all integers n, if n is odd, then n2 - 3 is even.

Answered by Penny Nom.
The angles of elevation and depression 2012-12-03
From Chelsey:
a person on a balcony of one building looks towards a second building. if the angle of elevation to the top of the second building is 25 degrees, the angle of depression to the bottom of the second building is 17 degrees, and the balcony of the first building is 22 feet above the ground, what is the height of the second building?
Answered by Penny Nom.
Sinx=logx+x^2 2012-11-28
From yasmin:
Sinx=logx+x^2
Answered by Harley Weston.
Two quadratic equations 2012-11-22
From fahmie:
graph and solve the intersection of the following equations:
x^2+y^2=4
x^2+2y=4

Answered by Penny Nom.
Golf for 2 groups of 4 for six rounds 2012-11-21
From Andy:
You have very kindly responded to a couple of questions I asked previously and I have tried to find the answer to my current query on the various links etc. My question: There are 8 of us going on a Golf Tour and we will be playing 2 groups of 4 for six rounds of golf. Is there a suitable schedule that works to ensure everyone plays with each other the same number of times (or as close as possible). Many thanks again for your help.
Answered by Victoria West.
Difference of squares 2012-11-19
From Qelibar:
Please factorise x^2y^2 - 4
Answered by Penny Nom.
A function problem 2012-11-18
From nahla:
f: IN --> IN
n --> f(n)
for every n that belongs the IN : fof(x) = 4n - 3
and for every n that belongs to IN f(2^n) = 2^(n+1) -1

Calculate f(993)

Answered by Penny Nom.
The square of any odd number, decreased by 1, is divisible by 8 2012-11-16
From bailey:
Prove that the square of any odd number, decreased by 1, is divisible by 8
Answered by Penny Nom.
Factor (6a^2-5a+1)(8a^2-6a+1)(12a^2-7a+1) 2012-11-16
From bailey:
Factorise f(a)=(6a^2-5a+1)(8a^2-6a+1)(12a^2-7a+1) thus find [f(a)]^1/2
Answered by Penny Nom.
How much should I price the earrings? 2012-11-13
From Sharon:
I want to sell some earrings and make a $12.00 profit on them I have to pay a 35% commission on them. How much should I price the earrings?
Answered by Penny Nom.
Practical uses of trigonometry 2012-11-11
From Michael:
Where can I find books or information on real life function of sine and cosine?
Answered by Penny Nom.
The derivative of y = sin (30º + x) 2012-11-07
From Saskia:
derivative of y = sin (30º + x)
Answered by Harley Weston.
Joe and Chris each bought a six pack of cola 2012-11-01
From Loulou:
Joe and Chris each bought a six pack of cola. Joe gave 2/3 of his away and Chris gave half as many as Joe. how many more colas did Chris have than Joe
Answered by Penny Nom.
4+4x5+4= ? 2012-10-31
From Bob:
4+4x5+4= ?
Answered by Harley Weston.
Frosting a three layer cake 2012-10-30
From johnathan:
Each layer of a 3 layer cake is a cylinder with height 7.5 cm. The bottom layer has diamegter 25 cm. The middle layer has dimegter 22.5 cm. The top layer has diameter 20 cm. The surface of the cke is frosted. What area of the cake is frosted. I get 1695.56 but the answer in the book is 2081.3
Answered by Penny Nom.
Introductory algebra 2012-10-30
From kevon:
if x = 7 is used in the expression 2x + 5 what is the output
Answered by Penny Nom.
An implicit differentiation problem 2012-10-26
From Katie:
find y' of x^2y-2y^3=3x+2y
Answered by Harley Weston.
Some 4 digit numbers 2012-10-25
From samira:
How many 4-digit numbers are there in which the first and last digits are the same?
Answered by Penny Nom.
How fast is the distance between the aircraft and the car increasing? 2012-10-24
From Steven:
At a certain instant an aircraft flying due east at 240 miles per hour passes directly over a car traveling due southeast at 60 miles per hour on a straight, level road. If the aircraft is flying at an altitude of .5mile, how fast is the distance between the aircraft and the car increasing 36 seconds after the aircraft passes directly over the car?
Answered by Penny Nom.
A label to cover a plastic cup 2012-10-23
From Kevin:
I'm trying to make a label to cover the entire outer area or a plastic cup. I know there must be a way to figure out the dimensions needed, but I can't seem to figure it out. The circumference of the bottom of the cup is 21.4cm and the circumference at the top of the cup is 29.8cm. The cup is 14.5cm tall. What should the height of the arc from the plane connecting the two ends of the 21.4cm arc. I attached a diagram where x is the value I'm looking for. I'm guessing there is some simple relationship between the length of a line and the arc needed to turn that line into a perfect circle, but I don't know what it is. Can you figure this out and share it with me? Thanks.

-Kevin

Answered by Penny Nom.
Differentiation rules 2012-10-23
From Morgan:
Use the derivative rules to differentiate each of the following:
1. f(x)=1/x-1
2. f(x)= sqrt(x)

Answered by Penny Nom.
Converting a fraction to a decimal 2012-10-20
From A Teacher:
My students want an easy way to convert a fraction to a decimal, can I get one from you?
Answered by Penny Nom.
A function with 3 asymptotes 2012-10-19
From rimoshika:
find a function that have 3 Asymptotic : y=x/y=sinx/x=-1
Answered by Penny Nom.
Whites and Sulphurs butterflies 2012-10-19
From Arlene:
The Whites and Sulphurs are one major family of butterflies. There are about 720 varieties. Eleven times more Whites and Sulphurs live outside North America, mostly tropical Asia and Africa, than live inside North America. How many of this family of butterflies live in North America? How many live in the rest of the world?
Answered by Penny Nom.
Composition of functions and one to one 2012-10-17
From Ariana:
If f o g are one to one function,does it follow that g is one to one? Give reasons for your answers
Answered by Penny Nom.
The degree measure of the central arc of a circle 2012-10-17
From Crystal:
On a circle with radius of 12 cm is an arc of length 20 cm. What is the degree measure of the central angle used to make this arc?
Answered by Penny Nom.
3 acres 2012-10-16
From Katy:
If 3 acres of land were a complete square, how many feet would be on each side?
Answered by Penny Nom.
A word problem involving a fraction 2012-10-12
From Derrick:
If the numerator and denominator of a fraction are both decreased by 1 the fraction becomes 2/3. If the numerator and denominator are both increased by 1 the fraction will be 3/4. Find the original fraction. How to do?
Answered by Penny Nom.
A four digit number 2012-10-09
From cyndi:
find this 4 digit number: the tens digit is 1/3 of the hundreds digit; the 2 digit number formed by the tens and ones digits i the largest prime number smaller than 40; the sum of all 4 digits is the same as the number of face on an icosahedron ? what is the number?
Answered by Penny Nom.
Profit 2012-10-05
From kathie:
find the profit income is $500 expenses $120
Answered by Penny Nom.
Painting 400 drums 2012-10-05
From Jordan:
This cylinder is given:

Cylinder with a height of 84 cm and a diameter of 54 cm.

Question: The exterior of the drum (cylinder), except the base, is to be painted blue. calculate how many litres of paint will be needed to paint 400 drums if one litre of paint covers 8.8meters Squared. Answer to nearest tenth of a litre.

Answered by Harley Weston.
\mbox 2012-10-05
From Jordan:
This cylinder is given:

Cylinder with a height of 84 cm and a diameter of 54 cm.

Question: The exterior of the drum (cylinder), except the base, is to be painted blue. calculate how many litres of paint will be needed to paint 400 drums if one litre of paint covers 8.8meters Squared. Answer to nearest tenth of a litre.

Answered by Harley Weston.
Four tangent circles 2012-10-04
From renu:
inside of a circle K of radius length measure R,three circular discs A,Band C each of radius r are placed so that each touches the other two and K . express R in terms of r. in the space between K, A and B , another circular disc D is placed which just touches K, A and B. if the radius is s, show that (6+root3)s=(2+root3)r
Answered by Penny Nom.
Solve 8t-r=12t for t 2012-09-29
From Monejah:
I need help solving 8t-r=12t for t
Answered by Penny Nom.
Profit and discount 2012-09-24
From ally:
A wholesaler purchased a wheat for $150 per tonne.He sold it at $170 per tonne.Calculate the percentage profit to (1 decimal place)if the wholesaler offered a 12% discount on the selling price.What could be the new selling price?
Answered by Penny Nom.
Adding two fractions 2012-09-19
From jeanette:

Question from jeanette, a parent:

can you help me figure this according to BEDMAS I am lost
steps please:
\[\frac{12.75^2}{50} + \frac{13.49^2}{60}\]
I hope this is not too confusing for you it is to me...


Answered by Harley Weston.
What percent of her monthly take home pay are her monthly expenses? 2012-09-19
From tina:
a girls monthly take home pay is 1932.00. her monthly expenses total 1910.00. What percent of her monthly take home pay are her monthly expenses?
Answered by Penny Nom.
Functions 2012-09-18
From nayeem:
I tried with many functions but I am not getting the exact values please help me
A give an example of a function whose domain equals the set of real numbers and whose range equals the set the set {-1,0,1}?
B Give an example of a function whose domain equals (0,1)and whose range equals [0,1] C.Give n example of a function whose is the set of positive integers and whose range is the set of positive even integers D. Give an example of a function whose domain is the set of positive even integers and whose range is the set of positive odd integers E give an example of function whose domain is the set of integers and whose range is the set of positive integers. F. Give an example of function whose domain is the set of positive integers and whose range is the set of integers.
please show me the work
Please give me the trick of finding such functions

Answered by Robert Dawson and Harley Weston.
Functions 2012-09-18
From nayeem:
I tried with many functions but I am not getting the exact values please help me A give an example of a function whose domain equals the set of real numbers and whose range equals the set the set {-1,0,1}?
B Give an example of a function whose domain equals (0,1)and whose range equals [0,1] C.Give n example of a function whose is the set of positive integers and whose range is the set of positive even integers D. Give an example of a function whose domain is the set of positive even integers and whose range is the set of positive odd integers E give an example of function whose domain is the set of integers and whose range is the set of positive integers. F. Give an example of function whose domain is the set of positive integers and whose range is the set of integers.
please show me the work
Please give me the trick of finding such functions

Answered by Robert Dawson and Harley Weston.
Equivalent fractions 2012-09-13
From lily:
if i had 1/3 and i had to find the problem an they gave me 20 what will be the denominator
Answered by Penny Nom.
Factoring 2012-09-10
From saravanan:
Dear Sir ,
I felt difficulty solving the following problem . So please help me .

a+b+c=7
ab+bc+cd= 20
what is a^2+b^2+c^2 ?

Answered by Lorraine Dame and Robert Dawson.
Golf for 12 2012-09-10
From John:
To have 12 players in groups of 4 playing in 4 games with different people each day
Answered by Victoria West.
Adding more than 2 fractions 2012-09-10
From Anonymous:
How do you add or subtract more than 2 integer fractions?

OK, here is an example.

-5/6 + 7/4 + 13/8

Please tell me the steps.

Thanks- grade 9 student

Answered by Robert Dawson and Penny Nom.
A profit of 35% 2012-09-07
From Jack:
If a product cost me $10.00 and I want a profit of 35% what is my sell price? How do I figure the sale price to make a 35% profit on the sale?
Answered by Penny Nom.
The volume of fluid in a tank 2012-09-06
From Dave:
I have vertical round flat bottom tanks of various dimensions. It is easy to find total volume but I would like to know how to figure out the coversion factor to go from how many centimeters of liquid is in the tank to the number of total cubic meters of volume. So if my float gauge on the tank says there is 50 cm of fluid in the tank, how many cubic meter of total volume is there?
Answered by Harley Weston.
(2y+1)/3=(1y-1)/2 2012-09-06
From jenna:
(2y+1)/3=(1y-1)/2
Answered by Penny Nom.
Row echelon form 2012-09-05
From Jennifer:
Hello, my name is Jennifer. I am a 12th grader in Pre-Calculus and I was wondering if you may help me with Row Echelon Form.

x + 2y - 3z = -5
-2x - 4y - 6z = 10
3x + 7y - 2z = -13

Answered by Penny Nom.
Exponential form 2012-09-04
From angie:
How do i write 9^-5 in exponential form?
Answered by Penny Nom.
Exponntial form 2012-08-30
From Robin:
My math teacher has asked the question:

"Write mmmxxjmyy2 in exponential form"

How would you solve this problem?

Answered by Penny Nom.
Golf for 20, two teams of 10 2012-08-30
From Mark:
Hi I have to teams of 10 people playing Ryder cup format(against each other) Can I have all 20 people playing with someone different over 3rounds of golf while remaining on there respective team I.e. players 1 to 10 must team up with a player of that group against a pair for players 11 to 20
I hope you can solve this for me
Thanks a bunch Mark

Answered by Victoria West.
12 golfers, 4 rounds 2012-08-29
From Paul:
We have 12 golfers who will play four rounds of golf - how can we organize the foursomes so that each player has the most exposure to the other 11 players?
Answered by Victoria West.
Ordering fractions 2012-08-22
From Imani:
I am a 6 th grader and on my homework it asks to order the fractions from least to greatest 11/12, 9/10, 10,/11, 15/16. Please help with solving this question.
Answered by Penny Nom.
The difference of the numbers on two dice 2012-08-19
From dilys:
Two fair dice are thrown. Find the probability that the difference of the two numbers is divisible by 4?
Answered by Chris Fisher and Lorraine Dame.
The volume of a sphere 2012-08-18
From Rohit:
why isn't volume of a sphere = Area of a semicircle x the circumference. i.e. if we revolve a semicircle around its axis we get a sphere
Answered by Penny Nom.
Golf for 16 in 7 rounds 2012-08-16
From Elizabeth:
Hello, I am trying to sort out golfing for 16 guys for 7 rounds. However the twist is that the teams are broken down into 8 old boys vs. 8 young boys. Ideally, each of the players would only play each other once within their own team and then twice within all of the matches. Is that possible? Thanks for your help, my brain is hurting!
Answered by Victoria West.
The exponential function form f(x)=a^x 2012-08-13
From Lucy:
Hi,
Why does the "a" value in the exponential function form f(x)=a^x have to be negative?

Answered by Penny Nom.
The range of h(x) = 1/x 2012-08-11
From Lucy:
What is the range of the function: h(x)=1/x (and can you please explain why)?
Answered by Penny Nom.
A function from {a,b} to {p,q} 2012-08-10
From Lucy:
How can a function relate each element of a set with exactly one element of possibly the same set?
Answered by Penny Nom.
A 5 pointed star in a circle 2012-08-05
From wiljohn:
find the area of a 5 pointed star in the circle with a radius of 12.517
Answered by Chris Fisher.
5 + 5 + 5 - 5 + 5 + 5 - 5 + 5 x 0 = 2012-07-29
From Tom:
5 + 5 + 5 - 5 + 5 + 5 - 5 + 5 x 0 =
Answered by Harley Weston.
John's electronic store 2012-07-25
From Jora:
Electronic Store John opened an electronic store in December. During his first month, He sold 10 LCD TVs and 20 Plasma TVs. His income during that month was $12800. In January, he sold 25 LCD TVs and 40 Plasma TVs for an income of $27600. In February he projects to sell 30 LCD TVs and 50 Plasma TVs. If John expenses are $28900 in February, how much money will he have after paying the expenses?
Answered by Penny Nom.
0.687 acres in square feet. 2012-07-18
From Roy:
The land parcel I am looking at says Acreage .687 how do I determine square feet?
Answered by Robert Dawson.
2 threesomes for golf over 4 days 2012-07-16
From Jim:
How to arrange for 2 threesomes for golf over 4 days equitably.
Answered by Victoria West.
A volume of revolution 2012-07-15
From Tewodros:
Let f(x) = e^x and g(x) = x^1/2 both be defined on [0,1]. Consider the region bounded by f(x), g(x), x = 0, x = 1. Rotate this region about the y-axis and determine the volume using the shell method.
Answered by Harley Weston.
The angular elevation of the sun 2012-07-14
From VINEET:
WHAT IS ANGULAR ELEVATION OF THE SUM
Answered by Penny Nom.
Two equations involving fractions 2012-06-12
From Fatima:
Hi ,teacher gave two question to my daughter as follows
Solve 2/x+3=(1/xx-9)-(1/x-3)and
Solve (4/x-2)-(x/x+2)=16/xx-4


Please help me
Thanks & regards fatima

Answered by Penny Nom.
A cylinder is to be filled with peas. 2012-06-12
From Silje:
Hi! How can I solve the following question without the use of a calculator?

"A cylinder is to be filled with peas. It is done like this: At 12:00 o'clock you put 1 pea in, at 12:01 you put 2 peas in, at 12:02 you put 4 peas in, at 12:03 you put 8 peas in, and so on. This continues until 14:00 o'clock (two hours later), when the last peas are put in and the cylinder is full. At what time is the cylinder half full?"

Answered by Robert Dawson.
Expanded exponential form? 2012-06-12
From Ramon:
express the numbers 990 614 000 005 in expanded exponential form.
Answered by Harley Weston.
9-3(2+6)/6-2*5 2012-06-08
From Sammi:
Hi there,

I am doing a practice test for my admittance into a college accounting program and I am really confused by this equation.

The answer I got was way off what the test answer sheet says it should be.

The question is

9-3(2+6)/6-2*5

If you could explain how the answer beccomes 35 that would be greatly apprciated!!

Thank you!

Answered by Robert Dawson.
The surface area of a dome 2012-06-06
From sravya:
how to calculate surface area of a dome if its base radius and height are known and neither it is hemisphere nor radius of sphere is known??
Answered by Walter Whiteley.
A reel of cotton thread 2012-06-05
From Grace:
91 meters of cotton goes round the cotton reel. About how many times does the cotton go round the reel?
Give your answer to the nearest ten.
(The Diameter of the cylinder is 3cm)

Answered by Penny Nom.
29 golfers 2012-06-04
From robert:
i have 29 golfers and have to make 8 teams for a golf scramble there will be 5 teams of 4 and 3 teams of 3 what is the fairest way to make up the teams?
Answered by Victoria West.
16 golfers, 4 rounds 2012-06-04
From Gerry:
16 golfers, 4 rounds - best pairings so that everyone plays with each other at least once.
Answered by Harley Weston.
A cyclist a jogger and a fly 2012-06-04
From Emd:
A cyclist & a jogger are 20 miles apart. Cyclist goes 20 mph & jogger 7mph towards each other. A fly starts on nose of cyclist & flies at 20 mph from cyclist to jogger & back & forth until they meet. How far has the fly travelled?
Answered by Penny Nom.
A parabola 2012-06-04
From Madeline:
In a parabola, I need to know what "a" b and c determine. I think that a determines the width of the parabola, but I am not exactly sure what b, and c do.
Answered by Robert Dawson.
Golf 16 2012-05-29
From marknp:
16 players only want tp play with one player each one time
Answered by Victoria West.
Multiples 2012-05-28
From Kenneth:
If I understand correctly , a multiple is a product of two numbers. For example some of the multiples of 6 are 6, 12, 18, 24, 30, etc. I just multiplied 6 by 1, 2, 3, 4, 5, etc.

Are the multiples of a fraction, for example, 2/3, determined in the same way? Are they 2/3, 4/3, 6/3, 8/3, 10/3, etc., or are they instead, 2/3, 4/6, 6/9, 8/12, 10/15, etc.?

Or do fractions have no multiples?

Answered by Penny Nom.
16 golfers 2012-05-17
From Nohemi:
Can 16 golfers each play in foursomes for 6 days? I found the answer for 5 days, but is it possible to do it for 6.
Answered by Robert Dawson.
Sharing the profit 2012-05-14
From Mafiza:
A and B started a business by investing $6000 and $8000 respectively. At the end of the year, a profit of $2100 is made. How much amount will A get as his share of profit?
Answered by Penny Nom.
Solve for x 2012-05-12
From Desiree:
3x -5^2=8
Answered by Penny Nom.
Golf for 6 2012-05-12
From Steve:
We have six golfers playing in two threesomes for six rounds of golf. Can you help solve this so that everyone plays with everyone else the same amount of times or close to it? Thanks!
Answered by Victoria West.
A 25% sulfuric acid solution 2012-05-03
From Scott:
In order to make a 25% solution with 96% Sulfuric Acid, how much of the Sulfuric do I add to DI water to make up 2000ml??
Answered by Penny Nom.
Golf: 8 players over 4 days in 4-balls 2012-05-03
From JimB:
I cannot seem to get this to work. Previous answers cover 2-balls, but we are 4-balls.

What we really want is for each player to play with each other, at least once and no more than twice.
Ideally, each would also play equally in the first and the second 4-ball of the day.

Answered by Victoria West.
Factoring 2012-04-26
From Amanda:
y=x^2-9x
Answered by Penny Nom.
A sloped yard 2012-04-24
From William:
Trying to calculate fill dirt for leveling a sloped yard to place a slab. Slab is 30'x40', yard slopes from corner #1 south 40' to Corner #4 drop is 11". From corner #1 east 30' to corner #2, drop is 11 1/4". From corner #1 southeast at a 45 degrees, 50' to corner #3, drop is 19 3/4". Corners numbered in a clockwise direction.
Answered by Penny Nom.
2x^2+5x+2 2012-04-23
From eddie:
2x^2+5x+2
Answered by Penny Nom.
The sarsen circle 2012-04-21
From Firdous:
I found the circumference but after what to do I am not getting the question is.The diameter of the sarsen circle is 33 meters,since there were originally 30 sarsen stones located on the circumference, how far apart wold the centres of the stones have been?Round to the nearest tenth of a meter.
Answered by Penny Nom.
The area of a quadrilateral 2012-04-21
From Rajat:
calculate the area ABCD in which AB is 48'1'',BC is 98'4'',CD is 61'4'',DA is 102'10'',AC is 110'3'',BD is 116'9''.
Answered by Penny Nom.
Golf for 12 2012-04-20
From glenn:
need to schedule 12 players for a 20 week season
Answered by Victoria West.
Golf for 12 2012-04-20
From Bram:
We are heading on a golf trip with 12 guys, playing 6 rounds. Can you please provide the formula whcih which have the least duplication.
Thanks in advance,
Bram

Answered by Victoria West.
16 golfers 2012-04-20
From steve:
i have 16 golfers, 4 in "A" flight , 4 in "B" flight , 4 in "C" and 4 in "D"

We are playing 4 rounds (4 teams of 4) and every group must have a player from the A B C an D flight, and in no round may 2 people have already played together!

Answered by Victoria West.
The inverse of y = x^2 - 2x 2012-04-20
From Shona:
How do I Find the inverse of the function y= x^2 - 2x ?
Answered by Penny Nom.
Golf for 12 2012-04-18
From Brent:
I have a golf group that is set up as follows: 12 players, broken into 2 teams of 6. I am looking for a formula to have one player from team A play against one player for team B each round, not repeat the match, and ride with as little duplication as possible. I know that it is not statistically possible with these numbers and will have at least one round that has duplication.
Answered by Robert Dawson.
Golf for 10 2012-04-18
From Bob:
Question from Bob:

Hi,
I am trying to set up pairings for 4 rounds of golf with only 10 golfers (2 foursomes and 1 twosome) and trying to minimize duplications. Any formulas or help?

Answered by Victoria West and Harley Weston.
An equation of the form y=mx 2012-04-17
From Samiya:
what would be an equation of the form y=mx with the points (-2, 6) and (1, -3) for the line?
Answered by Penny Nom.
What is the prime factor form of 7/5? 2012-04-17
From David:
What is the prime factor form of 7/5
Answered by Robert Dawson.
Profit as a percent 2012-04-14
From Anson:
The cost price of a CD is $50 and its marked price is $105. If the selling price is $88, find the profit percent.
Answered by Penny Nom.
A 14 man golf league 2012-04-12
From Paul:
14 man golf league. Need to have alternating 2 man teams for 18 weeks. Need to rotate foursomes so no one is stuck in twosome more than once. Can you help?
Answered by Robert Dawson and Victoria West.
A golf league with 2 man teams 2012-04-11
From Cosmo:
wanting to set up a golf league with 2 man teams. There are 14 A players and 14 B players.We want each A player to team with each B player once and each A player to play against each other A player once.Is this possible
Answered by Victoria West.
Golf for 28 2012-04-10
From EARL:
HI, I HAVE 28 GOLFERS AND I NEED A SCHEDULE FOR 5 DIFFERENT DAYS THERE WILL BE 7 GROUPS OF 4 PLAYERS EACH DAY. ALL PLAYERS WOULD LIKE TO PLAY WITH ONE ANOTHER ONCE AND NOT PLAY WITH EACH MORE THAN ONCE IF POSSIBLE OR LIMIT THE TIMES A GOLFER PLAYS WITH ANOTHER TO A MINIMUM IF POSSIBLE. (PLEASE SEND AN EXAMPLE OF THE SCHEDULING OF THE GROUPS FOR THE TEE OFF ORDER)
THANKS FOR ANY SOLUTIONS YOU CAN PROVIDE.
EARL

Answered by Robert Dawson.
The spread of a rumor 2012-04-09
From Roohi:
The function f(t) = a/(1+3e^(-bt)) has also been used to model the spread of a rumor. Suppose that a= 70 and b=3 0.2. Compute f(2), the percentage of the population that has heard the rumor after 2 hours. Compute f'(2) and describe what it represents. Compute lim t approaches infinity and describe what it represents.
Answered by Penny Nom.
A number puzzle 2012-04-07
From Kenya:
my number is less than 25
my number of tiles will make only one rectangle
my number is odd
my number is a factor of 36

Answered by Penny Nom.
Water is flowing from tank A to tank B 2012-04-06
From Noel:
At First, Tank A was completely filled with water. The tap from Tank A was then turned on and water flowed out at a constant rate. The water which flowed out was collected into Tank B. At the end of 6 minutes, Tank A was 2/3 full. After a further 8 minutes, Tank A had 2.4 liters of water left while Tank B was completely filled with water. Find the capacity of Tank B.
Answered by Penny Nom.
A 12 foot long board 2012-04-05
From Lesley:
If I have a board 12 feet long and am building shelves 2 1/4 feet in length, how many inches are left?
Answered by Jaymi Peterson and Penny Nom.
The surface area of the tetrahedron 2012-04-04
From youssef:
how to find the total surface area of the tetrahedron ?
Answered by Penny Nom.
1+2+4+8....= -1 2012-04-02
From Andy:
In this minutephysics video, it's claimed that 1+2+4+8....= -1 Is this true, and if so, how?
< href="http://www.youtube.com/watch?v=kIq5CZlg8Rg">http://www.youtube.com/watch?v=kIq5CZlg8Rg

Answered by Robert Dawson.
A lot with a slope 2012-03-30
From Carlos:
I have a lot with a slope. On the right I need to bring it up 2', From that point to the left is 112' which at this point I need to bring it up 5' for it to be level . The distance front to back is 80'. What is the cubic yards of this area that I need to fill in.
Answered by Harley Weston.
Golf for 17 2012-03-29
From Richard:
Hi Guys,
I have a scheduling problem which I don't think you have covered before.
Apologies if you have!
I have 17 golfers due to play 4 rounds of golf.
Each round will consist of 3 threeballs and 2 fourballs (ie. 17 golfers in 5 groups)!
Is it possible to come up with a schedule where each golfer plays
with different partners in each round?

Answered by Robert Dawson.
The period T of a pendulum 2012-03-27
From Ashley:
The period T of a pendulum is given in terms of its length, l, by T=2pi sqrt(l/g) where g is the acceleration due to gravity(a constant)
a. find dT/dl
b. what is the sign of dT/dl
c. what does the sign of dT/dl tell you about the period of the pendulums?

Answered by Penny Nom.
A uniform probability density function 2012-03-26
From Noeline:
The label on a bottle of liquid detergent shows contents to be 12 ounces per bottle. The production operation fills the bottle uniformly according to the following probability density function:
f(x) = 8 for 11.975 ≤ x ≤12.10
and
f(x) = 0 elsewhere

a. What is the probability that a bottle will be filled with 12.02 or more ounces?
b. What is the probability that a bottle will be filled between 12 and 12.05 ounces?
c. Quality control accepts production that is within .002 ounces of number of ounces shown on the container label. What is the probability that a bottle of this liquid detergent will fail to meet the quality control standard?

Answered by Penny Nom.
The equation of a line 2012-03-26
From Danielle:
Hello, could you help me figure out what the equation of the line is in the form of Ax+By=C with the points (-2,0) and (0,3)? Thank You!
Answered by Penny Nom.
The derivative of 2sin cubed x - 3 sin x 2012-03-25
From holly:
suppose f(x) = 2sin cubed x - 3 sin x

show that f 1(x) = -3 cos x cos 2x

Answered by Harley Weston.
A cubic inequality 2012-03-25
From Victoria:
The flight path of a bumblebee above the ground can be modelled by the function f(x)= 2x^3- 17x^2+ 11x + 130. Where x is the time in seconds and f(x) represents the height in inches above the ground. The entrance to the bee's hive is located 100 inches above the ground. Determine when the bumblebee's height is greater than 100 inches. The bee's height was monitored from 0 to 25 seconds inclusive. (over the domain 0
Answered by Penny Nom.
A vertical line passing through (10,4) 2012-03-16
From Isabelle:
Hi! My question is this:

Write the equation of each line described:
A vertical line passing through (10,4)

I would really appreciate your help!
Thanks,
Isabelle

Answered by Penny Nom.
An increase/decrease in profit 2012-03-13
From Thanikasalam:
It is understood that increase/decrease in profit for current year compared to the previous year is done by the formula (x-y)/y, with x=profit on current year and y=profit on previous year. How do we derive this formula?

If we work on this formula of calculating increase/decrease in profit, we get x/y-1

* = (x-y)/y
= x/y - y/y
= x/y - 1

How do you justify the 1 in the equation above. How to derive the formula and how do i prove it?

Answered by Penny Nom.
Factoring a quadratic 2012-03-13
From Makell:
X^2 + 32x + 220 = 0
Answered by Penny Nom.
A clock takes 3 seconds to strick the hour of 3 o'clock 2012-03-09
From saptarshi:
[ A clock takes 3 seconds to strick the hour of 3 o'clock. How many seconds will it take to strick 9 o'clock?]
Answered by Robert Dawson.
Four digit combinations from 1,2, ..., 9 2012-02-26
From Errol:
show me a list with all four digit combination numbers from 0 -9
Answered by Penny Nom.
Integral 1/(25-x^2)^3/2 2012-02-22
From John:
Integral 1/(25-x^2)^3/2
Answered by Harley Weston.
Dt[sin t tan (t^2+1)] 2012-02-21
From Ayu:
Ayu
Dt[sin t tan (t^2+1)]
derivatives

Answered by Harley Weston.
Fractions and proportions 2012-02-16
From Kenneth:
Hello:

I read the following in an old textbook:

Display the numbers 27, 18, 26, and 39 so as to form a proportion.

Which set of equal fractions is correct?

18/27 = 26/39

18/26 = 27/39

I thank you for your reply.

Answered by Penny Nom.
36 golfers 2012-02-15
From Steve:
We will have 36 golfers playing 4 rounds of golf and would like to have different foursomes each round. In other words, no player will play with another more than once.
Answered by Harley Weston.
A cylindrical container 2012-02-14
From KRIS:
YOU ARE PaCKAGING 4 EQUAL SIZED PAINT CANS WITH A RADIUS 8CM AND A HEIGHT OF 20CM, IN A CIRCULAR CONTAINER.What is the surface area of the packing container?
Answered by Penny Nom.
A storage box with a slanted roof 2012-02-13
From Sophia:
Hi!
I have another problem.

The diagram shows a side view of a box which is used to store small logs of wood for burning in a fire place. The slopping lid has an overhand of 15cm.
a) Calculate the total length of the slopping lid to the nearest cm.
b) When the lid is open above ground will the end of the lid be?

P.S. Please see attached.

Answered by Penny Nom.
Simplify the expression 2012-02-12
From Christy:
Hi guys, ok so I'm having problems with solving this equation since it's been so long I've done precal.
I have to find critical points so I have to find the derivative first. This is what I've done so far s(t)= (t-6)^4 * (t+1)^2
t'= 4(t-6)^3*(1)*(t+1)^2+ 2(t+1)(1)*(t-6)^4
so this is where I got stuck, which is the algebra part. How can I simplify this to get the answer to figure out the critical points.

Answered by Penny Nom.
Wallpapering a room 2012-02-08
From jimmy:
Ms. Frank is going to wallpaper a living room with dimensions 24 feet long, 18 feet wide, and 8 feet high. What surface area does Ms. Frank plan to wallpaper?
Answered by Penny Nom.
The curvature of the earth 2012-02-08
From sean:
Question from sean, a student:

Two people 1.8 metres tall walk directly away from each other until they can no longer see each other (due to the curvature of the earth, which has a radius of about 6378 km).
A) Find a function relating the height of two identical objects with the distance between them using the scenario above as an example.
B) Sketch this function (you may use Graphmatica if you wish). Over what domain and range does the function exist?
C) Describe this relation in practical terms.

Answered by Harley Weston.
Expressing 0.5% 2012-02-06
From Kenneth:
Hello:

It is correct to express 0.5% as 5 tenths of one percent.

Is it also correct to express 0.5% as 5 tenths of one-hundred percentage points or 5 tenths of one percentage point?

I think it would be 5 tenths of one percentage point because 5 tenths of one-hundred percentage points would be 50 percentage points, but I'm not sure.

I thank you for your reply.

Answered by Penny Nom.
Notation for the second derivative 2012-02-06
From Shafira:
In all math textbooks, it is written that d/dx ( d/dx) (y)= d2y/dx2. Why do they write it as d2y/dx2, not as d2y/d2x2?
Answered by Robert Dawson.
A quartic polynomial that is a perfect square 2012-02-05
From archit:
If P(x)=x^4+ax^3+bx^2-8x+1 is a perfect square then (a+b)=?
Answered by Penny Nom.
Two sectors of a circle 2012-02-03
From Wayne:
I am not even sure what to ask. I have to two lengths of feet and an angle of degrees, and I have to find the area?
**Problem attached

Thanks,
Wayne

Answered by Penny Nom.
Tons of fill dirt 2012-01-27
From Brian:
about how many tons of fill dirt would I need to level a area 30 foot by 30 foot and 6 foot deep
Answered by Penny Nom.
1 + 3 + 3^2 ...+3^(n-1) = 3^n - 1/2 2012-01-27
From Vicki:
I am trying to find out how to do show how this proof was worked.
Here is the end result 1 + 3 + 3^2 ...+3^(n-1) = 3^n - 1/2

This equation was used to find the number of white triangles in the Sierpinski Triangle

Answered by Walter Whiteley.
A fountain 2012-01-24
From kris:
A fountain has a radius of 14 meters to its outer edge. Their is an inner ring in the center of the fountain, where a statue of Sir Isaac Newton stands, that does not contain water. The inner ring has a diameter that is 6m less than the diameter of the outer ring of the fountain. What is the circumference of the inner ring? What is the area that is covered by water in the fountain?
Answered by Penny Nom.
The difference in latitudes of two cities 2012-01-18
From Renee:
Assuming that the earth is a sphere of radius 6378 kilometers, what is the difference in latitudes of two cities, one of which is 600 kilometers due north of the other?
Answered by Robert Dawson.
Tangent of theta 2012-01-17
From stahl:
explain what the 'tangent of theta' means. Draw and label a diagram to help with your explanation.
Answered by Harley Weston.
Exponential form 2012-01-15
From Helen:
How do you write (1/x) in exponential form?
Answered by Penny Nom.
The derivative of x^-(1/2) 2012-01-14
From Eric:
I have an problem figuring out the derivative of the negative square root of x i.e. x^-(1/2) using the first principle.
Could someone please show me?
Thanks in advance!

Answered by Harley Weston.
Four apples and two oranges cost Rs. 30... 2012-01-13
From nasr:
Four apples and two oranges cost Rs. 30, and one apple and 3 oranges costs Rs.15.How much does each apple and each oranges cost?
Answered by Harley Weston.
A volume of revolution 2012-01-11
From john:
find volume of solid generated by revolving the region in the first quadrant bounded by the curve y squared=x cubed, the line x=4 and the x-axis about the line y=8. The answer in the back of the book is 704 pi divided by5
Answered by Penny Nom.
One acre of dirt 6 inches deep 2012-01-11
From Cameron:
Ok what im wanting to know is how many pounds of dirt are in 80 acres of land that is 6 inches deep? My friend and I are stumped on this question
Answered by Penny Nom.
A picture in a frame 2012-01-10
From nnadozie:
a picture, which measures 46cm by 38cm, is surrounded by a frame, which is 2cm wide, find the area of the frame.
Answered by Penny Nom.
An equation with fractions 2012-01-06
From Balkees:
3/4(4-8x)=2x-2/3(6-12x)
Answered by Penny Nom.
Two shapes 2012-01-06
From muhammad:
Question from muhammad, a student:

a boy has two pieces of wire each 99cm long .he bends them into the shapes shown below.calculate the radius of each figure,giving each answer to two decimal places.figures are one is semicircle and other on is half of the semicircle.

Answered by Harley Weston.
Adding fractions 2012-01-05
From laurie:
if shaded parts of a circle are 4/7 and the other is 1/3 what fraction is left
Answered by Penny Nom.
Sum and difference of squares 2011-12-31
From Anne:
Se x e y são números reais distintos, então:
a) (x^2 + y^2) / (x - y) = x + y
b) (x^2 + y^2) / (x - y) = x - y
c) (x^2 - y^2) / (x - y) = x + y
d) (x^2 - y^2) / (x - y) = x - y
e) Nenhuma das alternativas anteriores é verdadeira.

Answered by Harley Weston.
The graph of a quadratic function 2011-12-27
From Thomas:
sketch a quadratic function with zeros at -3 and 1
Answered by Penny Nom.
The volume of a frustum of a cone 2011-12-24
From CV:
If I know the height, taper, and volume of a frustum cone, what are its Radii?

Calculating frustum cone volume is straight forward.
v=Pi/3*h*(R*R+r*r+R*r) where;
v=volume;
h=frustum height;
R=major radius;
r=minor radius;
Pi=3.1415926;
t=taper, ('slant angle' where t=0 is a disk)

Here is the tricky part.
Knowing 'h', 't', & 'v';
Calculate one or both of 'R' & 'r'.

Specifically, what is the formula for 'R=' or 'r='?

Answered by Penny Nom.
Three sides of a triangle 2011-12-24
From saba:
the three sides of a triangular lot have lengths 10,11and 13cm,respectively. find the measure of its largest angle and the area of the lot?
Answered by Penny Nom.
The volume of a cylinder 2011-12-22
From luke:
what is the volume of a cylinder that is 30 mm in diameter and 70 cm high?

can you please write the equation out in full for me to help understand it better. thanks

Answered by Penny Nom.
The number of digits in a number base m 2011-12-22
From Jash:
Assume there is a number system of base m.

The one property of this system is: If 2 numbers written in this system, which have 'a' and 'b' as the number of digits are multiplied, then the product of the 2 numbers will have a number of digits which is a function f(a,b).

In other words, as long as the number of digits of the 2 numbers are constant, the number of digits of their product is a constant.

Find m.

Answered by Robert Dawson.
Water is flowing into a cup 2011-12-19
From Tim:
A cup has a radius of 2" at the bottom and 6" on the top. It is 10" high. 4 Minutes ago, water started pouring at 10 cubic " per minute. How fast was the water level rising 4 minutes ago? How fast is the water level rising now? What will the rate be when the glass is full?
Answered by Penny Nom.
The circumference and area of a circle 2011-12-13
From Mable:
A circle that going 70mi across using 22/7 I need the area,radius, and the circumference and how to set up the steps can you help?
Answered by Penny Nom.
i+i^2+i^3+i^4...+i^102 2011-12-13
From megan:
how do you solve i+i^2+i^3+i^4...+i^102
Answered by Robert Dawson.
Extended real numbers 2011-12-12
From Justin:
Hi there, I was wondering does +infinity=+infinity in the extended real number system? Basically, I was wondering does +infinity=+infinity since -infinity and any extended real number (except +infinity) are less than +infinity?

Sincerely,

Justin

Answered by Robert Dawson.
Exponential form 2011-12-12
From Zoey:
What is the exponential form of 1/8??
Answered by Penny Nom.
factor 6x2-19x+3 2011-12-10
From pedro:
factor 6x2-19x+3
Answered by Penny Nom.
Jennifer wants to fence in her backyard 2011-12-09
From Val:
Jennifer wants to fence in her backyard. A yard is rectangular and is 15 feet by 12 feet. One 12-foot side is against the house so it does not need fencing. How many feet of fencing does Jennifer need?
Answered by Penny Nom.
The height of a flagpole 2011-12-07
From Grail:
at a certain point the angle of elevation of the top of a flagpole which stands on a level ground is 35 degree. 75 ft. nearer the pole, the angle of elevation is 50 degree. How high is the pole
Answered by Penny Nom.
Uniform acceleration 2011-12-06
From Android:
At a certain instant, two cars A and B are 2000 ft apart. At this instant, car A is traveling at 15 miles/hr. and accelerating at 3 ft/s^2 while B is moving towards A at 30 miles/hr. and an acceleration of 2 ft/s^2.

Find the ff:
a.) The possible time of their collusion in minutes.
b.) The distance in ft. each has traveled before collusion.

Answered by Penny Nom.
A cube of ice is melting 2011-12-05
From Emily:
a cube of ice (i.e.) each side is of the same length) is melting at a rate such that the length of each side is decreasing at a rate of 5cm per hour. how fast is the volume of the cube decreasing (in cubic cm per hour) at the instant the length of each side is 25cm?
Answered by Penny Nom.
Solve for theta if 8cos^2 theta-3=1 2011-12-02
From Katherine:
Hi, I have just learned to solve trigonometric problems for theta and have one specific question in order to find the solutions to my homework. I will use one example for this question. If I have 8cos^2 theta-3=1 I first divide by 8 and get cos^2theta=3/8 then I have cos theta= plus or minus the square root of 3/8 Then I assume that I plug in inverse cos (the square root of 3/8) to my calculator. How do I find the four solutions (we are typically supposed to find four, I believe?) Can you help me with finding the solution to this problem? Thank you!
Answered by Penny Nom.
The height of a flag pole 2011-11-21
From Micah:
A flagpole stands vertically at the edge of a roof of a building 200 ft high. the angle of elevation of the pole from a point 100 ft from the bottom of the building is 67 degrees. Find the length of the pole in meters.
Answered by Penny Nom.
Water pouring into a conical tank 2011-11-21
From Patience:
Hi my name is patience and I'm having a problem with this question.
Water pours into a conical tank of semi vertical angle 30 degrees at the rate of 4 cm^3/s, where h is the depth of the water at time t. At what rate is the water rising in the tank when h = 10 cm?
Thank you

Answered by Penny Nom.
Reducing by a scale factor of 1/3 2011-11-21
From Sandra:
A rectangle with the dimensions of 6.3 cm and 5.8 cm is reduced by a scale factor of 1/3. What are the dimensions of the new image?
Answered by Penny Nom.
2 + f(x) and f(x) + 2 2011-11-21
From Beth:
Do you do y=2+f(x) the same way as if the 2 came after the equation such as y=f(x) +2 when graphing?
Answered by Penny Nom.
Ordering Fractions 2011-11-16
From Chloe:
hey okay im having trouble with a math question. how do you put these fractions in number order
8/14 11/28 3/7

Answered by Jaymi Schmidt.
Ordering fractions 2011-11-16
From Laya:
Order from least to greatest
2/3, 1/2, 5/12

Answered by Jaymi Schmidt.
A log equation in exponential form 2011-11-11
From Sarah:
Express the equation in Exponential form

log2 to the 16th power=4

Answered by Penny Nom.
The slope of a line 2011-11-10
From Sara:
Line k lies in the xy-plane.
The x-intercept of line k is -4.
And line k passes through the midpoint of the line segment whose endpoints are (2,9) and (2,0).
What is the slope of line k?
Give the answer as a fraction.

Answered by Penny Nom.
1 third plus 5 sixths 2011-11-10
From Matt:
1 third plus 5 sixths =
Answered by Penny Nom.
What was the cost price of the article? 2011-11-10
From tashana:
By selling an article for $320,the profit was 20% of the cost price.what was the cost price of the article.
Answered by Penny Nom.
0.5 acres of land 2011-11-09
From Kim:
I have .5 acres of land. I want to divide it into 20x20 squares. How many squares will I have?
Answered by Penny Nom.
The sum of a series 2011-11-07
From Rattanjeet:
Find the sum of 1(1/2) + 2(1/4) + 3(1/6) + 4(1/6)(3/4) + 5(1/6)(3/4)2 + 6(1/6)(3/4)3+ ... where 1/6 + (1/6)(3/4) + (1/6)(3/4)2 + ... constitutes a geometric series.
Answered by Penny Nom.
A scale drawing 2011-10-30
From aretha:
i have a scale drawing of a house floor plan, don't understand how to find the answer. the drawing is 1in: 3ft. need to find the length / width/and the actual length/width of the living room,kitchen, 3bedrooms deck,sitting room, entry, and closet
Answered by Penny Nom.
Factor 2011-10-30
From Dawn:
x^2 -6x+9-4y
Answered by Penny Nom.
A mythical soccer ball 2011-10-27
From Joel:
We've been working on this problem diligently and can't seem to come up with the answer book's answer. We think it may be wrong, yet want to check it with an expert. Here goes. The school's new soccer balls are covered with 64 regular hexagonal panels. Each hexagon measures 2 inches between opposite corners and 1.5 inches between opposite sides. What is the total surface area of the soccer ball?
Answered by Robert Dawson and Lorraine Dame.
3 fish 2011-10-27
From Ann:
You have 3 fish weighing a total of 21 lbs. The largest fish weighs twice as much as the second fish and the third fish weighs half the amount of the second fish. So how much does the third fish weigh?
Answered by Penny Nom.
1 divided by 0 and infinity 2011-10-24
From ritika:
we say that one divided by zero gives us infinity, then why zero multiplied by infinity does not gives us one?????????????
Answered by Robert Dawson.
Implicit differentiation 2011-10-20
From Monica:
Find dy/dx in terms of x and y, if sin(xy)=(x^2)-y.
Answered by Penny Nom.
The area of a circle 2011-10-17
From Winney:
If the diameter of a semicircle is 3 feet what is the area.
Answered by Penny Nom.
One central circle and three tangent circles 2011-10-16
From Margaret:
You have one central circle and three or more circles tangent to the outside of the circle of varying radii. You know the x,y coordinates of the centers of the other circles. If you now remove that central circle (and pretend you never knew where it was), can you calculate its center in x,y coordinates?
Answered by Chris Fisher.
Expanding the size of a table 2011-10-16
From Ericka:
You are working as a carpenter in an industrial Shop. A customer came to you and inquired about the size of the table which would be fitted in her room. She informed you that she had already a 1.5 x 1m table in her room but she wanted to maximize the space by adding the same amount to is length and width. She is planning to occupy a 3 square meter place on her room for her to work comfortably. She is requesting you a written recommendation before she asked to make a table. What amount should be added to both sides to maximize a 3 square meter area?
Answered by Penny Nom.
Expanded form 2011-10-12
From Chris:
Write this number in expanded form
4 326 180 501

Thanks!!!

Answered by Penny Nom.
Six golphers for 27 weeks 2011-10-04
From ken:
I have 6 golfers alternating playing in a group of 4 each week for 27 weeks. How can I create a schedule so that each golfer both plays equally and plays against the other golfers an equal amount of time
Answered by Victoria West.
A reservoir has the shape of an inverted cone 2011-10-03
From Roger:
a reservoir has the shape of an inverted cone whose cross section is an equilateral triangle. if water is being pumped out of the reservoir at a rate of 2m^3/sec, at what rate is the depth of the water changing when the depth is 40 meters?
Answered by Penny Nom.
Three men sharing a task 2011-09-27
From Olaniyan:
Three men do a piece of work, the first does 7/15, the second does 5/6 of the remainder while the third does the rest.if the rest done by the third person equals 24 units, find the whole piece of work?
Answered by Penny Nom.
A hemispherical bowl with a lead ball inside 2011-09-27
From Jean:
"(a) Water is being poured into a hemispherical bowl of radius 3 inch at the rate of 1 inch^3/s. How fast is the water level rising when the water is 1 inch deep ?

(b) In (a), suppose that the bowl contains a lead ball 2 inch in diameter, and find how fast the water level is rising when the ball is half submerged."

Answered by Penny Nom.
A tank is 3/8 full 2011-09-24
From Olaniyan:
When 150 liters has been drawn from a tank, it is 3/8 full; how many liters will the tank hold?
Answered by Penny Nom.
One factor array 2011-09-16
From Hana:
hi my brother a 5th grade have a question. this is what it say "i have only one factor array, and i am between two numbers 12 and 16. what number am i?"
Answered by Penny Nom.
A four digit number 2011-09-16
From gina:
i am having a hard time figuring this out.. i believe the answer is 3105 but the 0 is throwing me off... i am a 4 digit number between 3100 and 3200, none of my digits are even, the sum of my digits is 9, the largest digit is in the ones place, what number am i ??
Answered by Penny Nom.
- 16 x 6 / -2 2011-09-09
From Nadiyah:
i dont understand how to answer this question ;
- 16 x 6 / -2
i still dont understand it with bedmas
/ = divison
please help!

Answered by Penny Nom.
Composition of functions 2011-09-05
From Jenna:
Let f(x)=x^2-1 and g(x)=1-2x. Find the indicated values.
1. f(g(1)) and g(f(1))

Thanks,
Jenna

Answered by Penny Nom.
The derivative of f(x) = (x+1)^1/2 2011-09-05
From Carla:
Find the derivative using the limit process of
f(x) = (x+1)^1/2

Answered by Harley Weston.
Exponential form? 2011-08-30
From Emily:
I have a 6th grader that needs to find out what is the exponential form of 461,885,811
Answered by Penny Nom.
A border around a flower garden 2011-08-28
From jessica:
A Landscaper, who just completed a rectangular flower garden measuring 6 feet by 10 feet, orders 1 cubic yard of premixed cement, all of which is to be used to create a border of uniform width around the garden. if the border is to have a depth of 3 inches, how wide will the border be? ( 1 cubic yard=27 cubic feet)
Answered by Penny Nom.
Exponential form 2011-08-23
From kiara:
The African bush elephant is the largest land animal and weighs about 8 tons . Write the amount in exponential form .
Answered by Robert Dawson.
A step function 2011-08-22
From Giovanna:
An average school bus holds 45 people. Show a graph between the relationship of students who need bus transportation and number of buses required?
Answered by Robert Dawson and Penny Nom.
An octagonal fire pit 2011-08-19
From terry:
i have a 18 ft diameter gravel circle laid down for a fire pit. i want to lay the paver stones out in a octagon pattern. How long would the sides of my octagon be to fit center on the 18 ft diameter of gravel base?
Answered by Penny Nom.
Ordering fractions 2011-08-17
From Myon:
My problem is putting fractions from least to greatest and they are 1/4 2/5 1/3 4/6 1/2 8/9 3/4 1/1 2/6 1/10 thanks
Answered by Penny Nom.
Two line segments in the plane 2011-08-15
From Tim:
For this problem I actually have tried to visualise the image in my head many times. This question makes my head spin.

Four points lie in a plane. They are partitioned into two pairs so that the sum of the lengths of the segments joining the points of each pair has the minimal possible value. Prove that these segments have no common points.

Answered by Chris Fisher.
A ratio as a fraction in lowest terms 2011-08-12
From jonecia:
write this ratio as a fraction in lowest terms 707days of 112days?
Answered by Melanie Tyrer.
Infinite Logarithmic Series 2011-08-08
From Sourik:
Dear Expert,

In my Amithabha Mitra and Shambhunath Ganguly's "A Text Book of Mathematics" I found the formula of log (1+x) where the base is e and x lies in between -1 and +1.As I want to learn Mathematics,I am not satisfied with the mere statement of the formula.Please help giving me the full proof.
Thanking you,
Sourik

Answered by Robert Dawson.
A cone with an oval as a base 2011-08-03
From Emily:
Hi, I was wondering how to calculate the surface area of a cone with an oval as a base (which I think is referred to as an elliptic cone or something like that). I have the both the maximum and minimum radius as well as the height, but I don't have a slant height and I'm not sure how to calculate it and then calculate surface area. I would really appreciate some help. Thanks!
Answered by Robert Dawson.
The height of a fluid in a horizontal tank 2011-07-24
From jason:
Same set up as many others, cylindrical tank on its side, but I am interested in defining the change in volume and/or fluid level as a function of time at a constant volumetric outflow. I plan on hooking a pump to the tank so "gpms' will be constant. I have a couple different sized tanks and pumps so I want a general equation. Thanks for your help.
Answered by Harley Weston.
A linear equation with fractions 2011-07-13
From Simon:
I would like to ask if you would be able to explain : linear equation: 2 1/2 (X-1) - x+3 /3 = 4 , the first step shows multiplying by 2 then multiplying by 3. etc. What I don't understand is where the 2 in multiplying by 2 comes from? ( the three is pretty obvious being x+3 /3 (to get rid of the divided by) ) .
Thanks for your help.

Answered by Penny Nom.
Fencing around a pool 2011-07-12
From Anita:
The measurement of a swimming pool are 32 feet by 18 feet. How much fencing is needed if the fence is to be built 7 feet from each side of the pool?
Answered by Penny Nom.
A group of 40 golfers 2011-07-11
From Doug:
We are a group of 40 golfers who play in 10 x foursomes playing 3 rounds. Can we all play with different players each round yet maintain the foursomes format.
Answered by Victoria West.
Golf for 14 2011-07-11
From Doug:
We have 14 players playing over 4 days. can we play with different players every day and keep the 3 x foursomes and 1 x pair format
Answered by Victoria West.
Golf for 12 2011-07-07
From Jim:
I have a golf trip scheduled with a total of 12 guys. We are playing three rounds of golf. For round 1 Players A B C D must play together. Is there a formula that would allow me to set up the other 8 foursomes with no repeats or minimal repeats? Thanks.
Answered by Victoria West.
17/20 of way around around an octagon 2011-06-05
From Priscilla:
What side of an octagon is a snail on if it is 17/20 way around? I think it is 6.8? Does that mean it is on side 6 or 7?
Answered by Penny Nom.
A reel of paper 2011-06-03
From keith:
if a full reel of paper has 7750 metres on it with a diameter of 1240mm and a core diameter of 100mm how can work out the thickness of each strip of paper therefore allowing me to work out the metreage of the roll when its half in diameter at say 620mm
Answered by Penny Nom.
The surface area of a hemisphere 2011-05-29
From Brenda:
What is the surface area of a hemisphere with a radius of 2 inches?
Answered by Penny Nom.
16 golfers and 8 rounds 2011-05-26
From Gerry:
16 golfers and 8 rounds. To have each player play with as many others as possible.
Answered by Victoria West.
pi/2 + 12/13 2011-05-24
From Jen:
If you have (pi/2) and you want to add it to (12/13), how would you go about it?
Answered by Chris Fisher.
U(n+1) = 2Un + 1 2011-05-22
From Cillian:
In a certain sequence, to get from one term to the other you multiply by 2 and add 1, i.e. This is a difference equation of form: U(n+1) = 2Un + 1. prove that there is a maximum of 2 perfect squares in this sequence
Answered by Claude Tardif.
Problem solving with fractions 2011-05-21
From emilyd:
I had a packet of sweets. I put 1/5 of the sweets into Container A, 1/3 of them in Container G, 10 sweets in container C and the rest in container D. If the number of sweets in container D is equal to 75% of the total number of sweets in container A and B, how many sweets are there altogether?
Answered by Penny Nom.
Draping a room with fabric 2011-05-19
From Maria:
I need to drape an entire room with fabric. The width of the fabric is 120 inches. How many yards of fabric do I need if the room is 4000 sq ft. (50 x 80 ft) and the ceiling is 12 ft. high.
Answered by Penny Nom.
Exponential form 2011-05-18
From Kara:
How do I write this in exponential form?

log 1/8 = -3

Answered by Penny Nom.
A rational number 2011-05-17
From owais:
according to definition of irrational number "a number which cannot be expressed in p/q form is called irrational number" square root of 2 is a irrational number but if i round off the digit correct to two decimal places so it becomes 1.41 and we can easily convert into p/q form so it is a rational number ???
Answered by Penny Nom.
Find two numbers whose difference is 153. 2011-05-10
From Scott:
Find two numbers whose difference is 153.
Answered by Penny Nom.
Golf with 36 players 2011-05-09
From Terry:
What is the calculation to have 36 players play in a different foursomes each week. I would guess this would take 12 times but do not know how to figure this out
Answered by Victoria West.
The domain of a function 2011-05-08
From Minks:
(x^2 + 1 / x^2 +2x - 3 ) X ( x^2 -9 / x^2 - 2x - 3)
Answered by Penny Nom.
The game of 24 2011-05-06
From shaylin:
how do you use 3,9,7,9 once in any operation equal to 24?
Answered by Penny Nom.
A scale of 1:160 2011-05-02
From Dave:
a 4 foot by 8 foot sheet of plywood is how many acres in a scale (1:160)
Answered by Penny Nom.
The radius of a cylinder 2011-04-27
From Jazmin:
Hi, I don't understand how to find the radius in a cylinder with only the surface area (143.7) and the height (0.8)? I know that the formula is 2pir2+2pirh, but I don't see how to isolate the r? I appreciate your help.
Answered by Penny Nom.
(27^r^-1)^-2/3 2011-04-24
From Yung-Ju:
(27^r^-1)^-2/3
Answered by Penny Nom.
Dividing the profits 2011-04-19
From Tracy:
I need to know how to come up with the answers to the following problems. I need to be able to show my work. So can you help me on how to come up with the answers to questions like these. Thanks.

Three individuals form a partnership and agree to divide the profits equally. X invests $9,000, Y invests $7,000, Z invests $4,000. If the profits are $4,800, how much less does x receive compared to having the profits divided in proportion to the amounts invested by X, Y, and Z?

In printing an article of 48,000 words, a printer decides to use two sizes of type. Using the larger type, a printed page contains 1,800 words. Using a smaller type, a page contains 2,400 words. The article is allotted 21 full pages in a magazine. How many pages must be in smaller type? ___________

thank you

Answered by Penny Nom.
Solve x(x-3) = 5 (x+4) 2011-04-19
From Ashley:
I have to solve using the quadratic formula. And my answer should be in simplest exact form. And I can't approximate.
I really need help to prepare for a test ....
x(x-3) = 5 (x+4)

Answered by Penny Nom.
The equation of a sphere 2011-04-16
From kanika:
equation of sphere and how to know its center
Answered by Penny Nom.
A golf league of 60 golfers 2011-04-14
From Keith:
I have a golf league of 60 golfers and I want to have each golfer play in a different group of 4 each week for 24 weeks. How can I do this
Answered by Victoria West.
Coefficient of variation 2011-04-14
From Braden:
When determining coefficient of variation (CV) or %CV is it possible to calculate %CV for two variables? For instance can %CV be used to determine the precision of 5 data points on a graph using the X and Y coordinates? or does %CV need to be determined for each variable separately?
Answered by Robert Dawson.
2 cubic foot bags of mulch 2011-04-11
From Ron:
if I buy 2 cubic foot bags of mulch, how bags would I need to equal 1 yard? (13 bags?)
Answered by Penny Nom.
PEDMAS 2011-04-10
From Ross:
48 ÷ 2 (9+3)

Is the answer 2 or 288?

Answered by Harley Weston.
x/6 + 4/5 = x/9 2011-04-10
From Khadija:
x/6 + 4/5 = x/9
Answered by Penny Nom.
Real-life applications of trigonometry 2011-04-10
From Angela:
I am a teacher and I desire to show the students the real-life application of trigonometry. Of course, one application is to use a clinometer and find the heights of various things. However, I am trying to provide a real-life scenario which also answers the question "why" the height of the object needs to be found. Not being an engineer, I do not know the specifics examples, but I want my information to be accurate and my example to to be as real-life as possible. I mean, I can say that someone wants to know the height of a flagpole; however, I also want to answer the question "why" they want to know this. I would like to give an actual real-life scenario. Do you know of some? Thanks!
Answered by Penny Nom.
8 golfers playing 2 groups of 4 each day for 5 days 2011-04-08
From Chris:
Hi,
8 golfers playing 2 groups of 4 each day for 5 days. Trying to work out how they can organise playing at least once, and no more than twice, with everyone over the 5 days. We always come up with someone playing 3 times with another in one of the 4's!
Hope you can help!
Regards.

Answered by Victoria West.
The volume of a flower pot 2011-04-07
From kp:
I have 2 flower pots with the following dimensions that I am trying to calculate the volume of Soil I will need to fill them. pot #1 29"tall, top of pot 31.5 inches across( radius of 15.5") the bottom of the pot is 21"across (radius 10.5) Pot #2 29"tall, top of pot 26 across (radius 13") the bottom of the pot 17'"a cross (radius 8.5")
thank you
KP

Answered by Penny Nom.
Eliminate y 2011-04-07
From Lynn:
2x + y = 8
y + 3z =5
z + 2w =1
5w + 3x = 9

Form three equations with y eliminated

Answered by Penny Nom.
A conical container and a spherical balloon 2011-04-06
From Steven:
Water is running out of a conical container 12 feet in diameter and 8 feet deep (vertex down) and filling a spherical balloon. At the instant the depth of the water in the cone is 4 feet, the radius of the sphere is approximately 4 feet.

The rate of change of the depth of the water in the cone at the instant is approximately ______________ times the rate of change of the radius of the balloon.

Answered by Penny Nom.
Administration costs and profit 2011-04-05
From brian:
Hi , If I have direct job costs of $100. and my administration is 20 % and I want to make a 15 % profit , how would I calculate the administration and profit and what would be the total of each be and also the final total?

Thanks,
Brian

Answered by Penny Nom.
Two ships 2011-04-05
From Gevork:
Ship A is sailing due south at 16 mph. At the same time, a second ship B, 32 miles south of A, is sailing due east at 12 mph.
(a) at what rate are they approaching or separating at the end of one hour?
(b) At what rate are they approaching or separating at the end of two hours?
(c) When do they cease to approach each other and how far apart are they at that instant.

Answered by Penny Nom.
The rate of change of the area of a parallelogram 2011-04-05
From Gevork:
Let a parallelogram have sides of 8 and 12 and let vertex angle A be decreasing at a rate of 2degrees per minute. Find the rate of change of the area of the parallelogram when angle A equals 30 degrees.
Answered by Penny Nom.
[(90+36-4) ÷ 2] x 15 = 2011-03-30
From ken:
[(90+36-4) ÷ 2] x 15 =
Answered by Penny Nom.
I need to make $1000 profit 2011-03-28
From Kat:
I need to make $1000 profit selling plants for $3 each. My direct cost is $2 per plant and my overhead is $5oo. How many pots (plants = 1 per pot) do I need to make $1000?
Answered by Penny Nom.
Write 3 1/5 in decimal form 2011-03-24
From Stephenie:
converting fractions????

3 1/5 into decimal???

Answered by Penny Nom.
The number of points on a line is equivalent to that of a surface 2011-03-24
From Gary:
I I was reading about how the number of points on a line is equivalent to that of a surface. This was done by taking any point on a line then taking alternating digits and making them as points on an x and y axis therefore points on a surface.The problem is as i see it if you just take a line then hold it over a surface you have just put the points on the line in a one to one correspondence with the points directly under it on the surface.Now you have all the rest of the surface which cannot be mapped onto the line since it is already used up.What am i missing?
Answered by Penny Nom.
A true or false trig question 2011-03-24
From Abeth:
True or False: Since cot (theta) = cos (theta)/sin (theta), if cot (theta) = 1/2, then cos (theta) =1 and sin (theta)=2. My answer before was true, but not my answer is false. Can you give me a solution on this matter. thanks.
Answered by Penny Nom.
If 25 items are evenly spread over 7 days 2011-03-23
From michael:
What is the formula to calculate:
If 25 items are evenly spread over 7 days, but one day has 3 items less than the other days, what is the number if items for each of the remaining 6 days?

Answered by Penny Nom.
What is x to the power of 0? 2011-03-23
From Jason:
What is x to the power of 0?
Answered by Robert Dawson.
Ordering fractions 2011-03-20
From Yasmine:
what's the answer, greatest to least

3/8 1/4 2/3

Answered by Penny Nom.
(3x+4y)^2 - (2x-y)^2 2011-03-16
From Taiwo:
pls could some one help me with this question? thanks as lot

factorize:
(3x+4y)^2 - (2x-y)^2

Answered by Penny Nom.
6(M-1/9)=55/12 2011-03-15
From Colleen:
6(M-1/9)=55/12
Answered by Penny Nom.
Fencing a park 2011-03-07
From taniel:
Find the number of feet of fence needed to fence a park that is 1 3/8 mi long and 5/8 wide.
Answered by Penny Nom.
6 faces and 8 corners 2011-03-04
From noemi:
jeff made a figure with 6 faces the figure has 8 corners all of the faces are the same size and shape what figure did jeff make
Answered by Penny Nom.
Vertices and sides of a polygon 2011-03-04
From RINA:
My questions is : would a vertices and sides of a polygon be the same number.  For example, a octagon it has 8 points and 8 sides. Am i correct?
Answered by Robert Dawson.
Factor 10x^2+17x-6 2011-03-03
From Jeff:
Factor 10x^2+17x-6 with steps and explanation.
Answered by Penny Nom.
A family of circles 2011-03-01
From steffi:
Find the equation of the family of the circle passing through the the point of intersection of x^2+ y^2 -4x-28=0 and x^2 +y^2 -4x-20+52=0; the member tangent to x=7.
Answered by Penny Nom.
Ordering fractions 2011-03-01
From amanda:
I am utterly confused my son needs to order 3 different fractions from least to greatest such as 3/6, 5/6 & 4/6 I read your explanation on how to do it but I was still confused is there a more simpler method?
Answered by Robert Dawson.
A camera's line of sight 2011-02-26
From MJ:
A rocket that is rising vertically is being tracked by a ground level camera located 3 mi from the point of blast off when the rocket is 2 mi high its speed is 400mph At what rate is the (acute) angle between the horizontal and the camera's line of sight changing
Answered by Penny Nom.
Cutting the top off a conical tent 2011-02-22
From tom:
how far from the top must you cut a conical tent in order to cut the cloth in half...
Answered by Penny Nom.
How much gravel do I need to make it level? 2011-02-22
From Robert:
I have an area that is 55 feet long x 75 feet wide and goes from 0 to 13 inches deep. How much gravel do I need to make it level?
Answered by Penny Nom.
Prove sin x = sin (pi - x) 2011-02-15
From Janet:
Prove sin x = sin (pi - x)
Answered by Penny Nom.
(1-1/2)(2-2/3)(3-3/4)...(15-15/16)=n!/16 2011-02-15
From Fiona:
Could you help me find the value of n: (1-1/2)(2-2/3)(3-3/4)...(15-15/16)=n!/16
Answered by Penny Nom.
Factoring with fractions 2011-02-15
From Megan:
Hi there, I'm working on factoring polynomials but this question has me quite puzzled.. im a college student in my first year.

(x^2-a^2/xy)(xy/x+a)

Answered by Penny Nom.
The equation of a circle 2011-02-14
From Cristela:
find the equation and all the information in General Form and Standard Form of the Circle that will passed trough the point (2,3) (6,1) (4,-3)
Answered by Stephen La Rocque.
1/0 and 2/0 2011-02-11
From Dixit:
How are the infinite number obtained by dividing 1 / 0 and 2 / 0 are different?
Answered by Penny Nom.
The equation of a sphere 2011-02-09
From kelly:
what is the equation of a sphere if one of it's diameters has a giving end point of (4,4,4), (8,8,8)?
Answered by Penny Nom.
Throwing a football 2011-02-08
From Janet:
a football is thrown into the air from 2 meters high. After 1 second it is 6.9 meters high. After another second it is 2 meters high. How high is it after 1.02 seconds and .05 seconds? when will it reach 5.136 meters? I am not sure how to extract the numbers I need and what equations to use them in?
Answered by Stephen La Rocque.
Rearrange for x 2011-02-06
From sue:
(x+7)/3 = (3xy+2x)/4 rearrange for x
Answered by Penny Nom.
I have nine players, three rounds of golf 2011-02-05
From Pat:
My problem is I have nine players three rounds of golf, please could you give me the best formula so that no two players play twice together
Answered by Victoria West.
1 and four eights inches 2011-02-01
From Miranda:
WRITING TO EXPLAIN:If a line is measured as 1 and four eighth inches long,explain how you could simplify the measurement?
Answered by Penny Nom.
A fence around a water tank 2011-02-01
From Heath:
I am building a fence around a water tank. the fence is to be in the shape of a normal octagon. The tank has a circumference of 57 ' 6''. I would like the fence to be 3 ft from the tank at the skinny point . How would I calculate(for the simple guy) where to set each of my 4x4 posts at the 8 corners. Any help would be greatly appreciated.
Answered by Harley Weston.
22 people playing 6 rounds of golf 2011-01-31
From angi:
I'm having a problem scheduling matches for a golf vacation. We have 22 people playing 6 rounds of golf. We know that we will need to have two 3 man pairings each day. Is there a way to make sure that is not a great deal of duplication. I did see how Penny answered a question in 2005 with the circles. Is there a way mathematically to do this or through excel?
Answered by Victoria West.
A mixed number 2011-01-30
From Joyce:
express the fraction as a mixed or whole number 94/5
Answered by Penny Nom.
(7cubed * 8(exponent6))exponent 6 2011-01-29
From Shalaine:
What is the exponential form of (7cubed•8(exponent6))exponent 6?
Answered by Penny Nom.
Scale factor 2011-01-25
From Mackenzie:
hi my problem is:

a man that is 6 feet tall in real life and only .5 inches in a = photograph. what is the scale factor and scale of the photograph?

another:

a girl in the same photograph is 4.8 feet tall. How tall is she in the = picture?=

Answered by Chris Fisher.
16 golfers 2011-01-22
From Jack:
I am responsible for setting up groups for 16 golfers for 7 days, is there a formula to calculate playing partners so that everyone gets to play with each person?
Answered by Victoria West.
1/a^2 + 1/b^2 2011-01-19
From robert:
If (a + b)^2 = 81 and ab = 18, find the value of 1/a^2 + 1/b^2 ?
Answered by Penny Nom.
Flipping a cone over 2011-01-17
From Fionna:
The height of your cone is "x". Holding the cone so that the vertex is at the bottom, fill it half full with water. The depth of the water is "1/2x". Put a lid over the top, and flip it over. What is the new depth of the water, now that the cone is flipped?
Answered by Penny Nom.
Profit as a percentage 2011-01-17
From Connie:
I am trying to determine the profit margin on selected items. ex: A bottle of juice costs me $.70, I sell it at $3.75. How can I determine what my profit percentage is?
Answered by Penny Nom.
16 golfers 2011-01-16
From Kenneth:
Our group will play 12 rounds. We are 16 golfers.
How can I schedule 12 days of differing foursomes limiting repeats in the foursomes

Answered by Victoria West.
tanθ=1.192 2011-01-15
From Adori:
Use a calculator to approximate two values of the θ (0 ≤θ≤2π) that satisfy the equation. a) tanθ=1.192

I do not understand how to find the second value of θ.

Answered by Harley Weston.
Identify each polynomial by its degree and number of terms 2011-01-10
From betty:
Write in standard form and identify each polynomial by its degree and number of terms. How do you do this?
Answered by Penny Nom.
Tiling a swimming pool 2011-01-09
From rustom:
(a) Find the volume of water in swimming pool with vertical ends and sides . The length measured at the water line is 50 ft. and the breadth is 20 ft. The bottom of the swimming pool is a plane sloping gradually downward so that the depth of the water at one end is 4 ft. and 8ft. at the other end. (b) If the sides, ends, and bottom of the swimming pool are constructed of tile blocks whose glaze surface dimensions are 3in by 6in. , and if the ends and sides of the pool extend 2ft.above the water level, find the number of blocks used if 1/20 of the surface area is covered by sealing material.

I got the (a) question but I don't know the (b) question which have the answer of 16,136 blocks. I hope I can get the procedure for this, THANK YOU!

Answered by Penny Nom.
10 golfer playing 5 rounds 2011-01-09
From Jeff:
need pairing for 10 golfer playing 5 rounds of golf.
Answered by Victoria West.
A golf holiday 2011-01-04
From Andy:
You kindly responded to a question I previously sent with regard to a Golf Holiday and I wonder if I might ask another question please. There are now 7 players playing 5 rounds of golf and I wonder if it is possible to devise a schedule where everyone plays with each other a minimum of two times i.e. they will play twice with some and three times with others. As always, many thanks for your kind assistance.
Answered by Victoria West.
If ac = bc ... 2011-01-04
From jamielle:
if ac=bc, then a is not equal to b, c is not equal to zero
Answered by Penny Nom.
The intercept form 2011-01-04
From sussie:
How do you find the intercept form of y=X^2-2x-3?
Answered by Penny Nom.
A tangent line 2011-01-03
From Amanda:

Question from Amanda, a student:

an equation of the line tangent to y=x^3+3x^2+2 at its point of inflection is
(A) y=-6x-6
(B) y= -3x+1
(C) y= 2x+10
(D) y=3x-1
(E) y=4x+1


Answered by Penny Nom.
Exponential form 2011-01-02
From jumana:
find any five examples of exponential form from real life situation
Answered by Penny Nom.
Profit as a percent 2010-12-19
From gaurav:
In the following, which calculation is right to calculate the profit percent.
if profit is 20%- 100*1.2 = 120
if profit is 20%- 100/0.8 = 125?

Answered by Penny Nom.
Golf pairings 2010-12-17
From AL:
I have a golf tourney where we have fourteen players and are playing five times/rounds of golf. I am trying to pair up two threesomes and two foursomes so we don't play with the same person twice or the least amount of times.
Answered by Victoria West.
The perimeter of a quarter circle 2010-12-15
From kim:
how do you figure the perimeter of a quarter circle that has a radius of 7 inches? I would like to have the explanation of how you figure it out. Thank you
Answered by Penny Nom.
Ordering fractions 2010-12-14
From serita:
1/2,2/4,4/8 how do you do least to greatest
Answered by Penny Nom.
Angle of elevation 2010-12-10
From PANKAJ:
angle of elevation of the sun perpendicular72 and base 88 find angle
Answered by Penny Nom.
The length is twice its breadth 2010-12-09
From jenu:
The length of the rectangular field is twice its breadth. A man jogged around it 5 times and covered a distance of 3 km. what is the length of the field?
Answered by Penny Nom.
Simultaneous equations 2010-12-05
From ryan:

Question from ryan, a student:

3         4
--   -    --      =     1      (1)
x         y

7       2             11
--   -  --        =   --       (2)
x       y              12


Answered by Chris Fisher and Stephen La Rocque.
Simplify the logarithmic expression 2010-12-02
From Adori:
Use the properties of logarithms to simplify the logarithmic expression. Log(1/250) base 5
Answered by Penny Nom.
Can determine if it is scalene, isosceles, or equilateral 2010-12-01
From Jessie:
find the measures of the sides of triangle KPL and classify each triangle by its sides. my first problem would be K(-3,2) P(2,1) L(-2,-3) ...The three points they give you are the vertices of the triangle and you need to match them up. Draw the triangle and write in the vertices and the related point with the vertex. You will then do the distance formula three times to find the distance of all three sides. Once you have the three sides you can determine if it is scalene, isosceles, or equilateral...using the distance formula how do i solve this?
Answered by Penny Nom.
6 rounds of golf 2010-11-29
From Andy:
I wonder if you are able to assist me with scheduling pairings for a forthcoming Golf Tournament ?

There are 7 players and we will be playing 6 rounds - each round will be a 3 ball and a 4 ball. I am trying to work out a schedule that means everyone plays with each other roughly the same number of times. Also, we are looking to ensure everyone plays in roughly the same number of 3 and 4 balls. Any suggestions would be much appreciated - thank you.

Answered by Chris Fisher.
Ordering mixed numbers 2010-11-23
From kate:
what are the mixed numbers 5 7/9, 5 1/2, 5 11/18. put least to greatest?
Answered by Penny Nom.
Two girls agree to mow a lawn 2010-11-23
From Milorad:
Two girls agree to mow a lawn 60m by 80m so that the first girl cuts one half by cutting a uniform strip around the lawn. How wide a strip should the first girl cut?

I've been struggling with the question above, mainly with the equation and I have set it up as 2400=(60-2x)(80-2x), if this is incorrect I would like to know where I've gone wrong and also would like a full answer to see it properly done. I need an algebraic answer and not one with a diagram.

Answered by Penny Nom.
The rate of change of (8e^3x)+(27 e^-3x) 2010-11-23
From Aleo:
I am unable to solve this problem: Find the rate of change of (8e^3x)+(27 e^-3x), with respect to x when x= 0.5
Answered by Penny Nom.
i^i 2010-11-21
From trale:
Can we use e^ix=cosx+isinx for finding i^i like that: x= pi/2 => e^(ipi/2)=0+i then [e^(ipi/2)]^i=i^i.then we find i^i= 0,207879576.... is it true? can we give value for x for free?thank you.
Answered by Harley Weston.
A 35% profit 2010-11-19
From Ana:
You have a stamp collection and make a 35% profit by selling it on E-Bay for $2700. Find the original cost of the collection by using the formula:New value × 100 / 100 + percent change = Original value
Answered by Stephen La Rocque.
A scale drawing 2010-11-17
From chandler:
A parking lot 200 meters wide; 1 centimeter:25 meters, find the length on the scale drawing and the scale factor
Answered by Penny Nom.
Ordering fractions 2010-11-14
From Alexia:
I am having real trouble with how to order 18/11 , 2/5 ,6/15 from least to greatest can you give me a step by step real easy way to do it?
Answered by Penny Nom.
Will the ball clear the tree? 2010-11-14
From MK:
Sam hits a golf ball with a five-iron a distance of 120m horizontally. A tree 45m high and 35m in front of Sam is directly in the path of the ball. Will the ball clear the tree if the ball makes a parabolic curve and has a maximum height or 80m?
Answered by Brennan Yaremko.
What numbers can I be? 2010-11-13
From john:
my thousands digit is 2 times my ones digit.my hundreds digit is my ones digit plus 2. my tens digit is the difference between my thousands and hundreds digit what numbers can i be?
Answered by Penny Nom.
Four-digit numbers using 0,3,4,5,6,and 7 2010-11-12
From Katy:
Using only the digits 0,3,4,5,6,and 7, how many distinct four-digit numbers exist that are greater than 4002 and less than 6732?
Answered by Stephen La Rocque.
z^5 - 3z^4 + 2z^3 + z^2 - 3z + 2 2010-11-06
From Kumar:
would you please solve this problem, related to complex numbers.

Find all the roots of :

z^5 - 3z^4 + 2z^3 + z^2 - 3z + 2

Answered by Robert Dawson and Penny Nom.
Terminal zeros 2010-11-04
From morgan:
if I have to multiply 1*2*3*4*5*6*7*8*9*10*11*12*13*14*15*16*17*18*19*20 how many terminal zeros do i get?
Answered by Penny Nom.
How far must the pitcher travel to get to the ball? 2010-11-04
From ken:
A baseball player bunts a ball down the first base line. It rolls 35ft at an angle of 26 degrees with the first base path. The pitchers mound is 60.5 ft from the plate. How far must he travel to get to the ball.
Answered by Penny Nom.
What is the speed of each car? 2010-11-04
From Jennifer:
two cars are 420 miles apart and traveling towards each other along the same road. They meet in 3.5 hours. One car is traveling 15mph slower than the other. What is the speed of each car? This must be solved using a system of equation, I have no idea?
Answered by Stephen La Rocque.
A fourth degree polynomial function with real coefficients 2010-10-30
From Ryan:
Question from Ryan, a student: What is the fourth degree polynomial function with real coefficients that has -1,1 and I as zeros and such that f(x)=160
Answered by Harley Weston.
I want to make a 50% profit 2010-10-29
From Debbie:
If my cost on an item is $32.00 and I want to make a 50% profit- What would the retail be?
Answered by Penny Nom.
Two algebraic expressions 2010-10-29
From chaptay:
what is the difference between x(n^2) and x(n)^2
Answered by Penny Nom.
The game of 24 2010-10-26
From gabrielle:
What is the answer using the numbers 0 0 0 0 (trig and calc are allowed, all math is allowed) in the game 24?
Answered by Claude Tardif.
A polynomial with integer coefficients 2010-10-23
From Adori:
Question from Adori, a student: Find a polynomial with integer coefficients that has the given zeros: 2/3, 4, root of 3i, root of -3i.
Answered by Harley Weston.
Powers 2010-10-20
From dylan:
how do you write 20736 in exponential form .same for 1728 and 50625.

is there a formula to figure out how to express large know numbers in exponential form.

Answered by Penny Nom.
The equation of a circle 2010-10-20
From Silvan:
Hi, I just want to find the x,y values for the circumference of a circle...

Lets take a clock having its centre at (0,0) in a graph. I just want to know how to find the (x,y) co-ordinates for the curved path or the surface of the circle.. Is there any formula to directly align the curved path or the circumference of the circle in a graph for a known radius of a circle..

I feel it will be useful for me to draw a clock in a graph... :-)

Answered by Penny Nom.
A scale factor 2010-10-17
From david:
We have to rectangles with measurements and we need to find the scale factor. The rectangles are W=6 L=4 for first and second is W=4 and L=3. thanks for your help
Answered by Penny Nom.
Simplifying fractions 2010-10-13
From Alice:
14times (9divied3) + 12
______________________
18 divied (26-24)

don't get this.......

Answered by Penny Nom.
Can a function be both even and one to one? 2010-10-09
From Tracy:
Can a function be both even and one to one?
Answered by Harley Weston.
Exponential form 2010-10-06
From celina:
what is the exponential form of 4x*4x
Answered by Penny Nom.
Rectangular arrays 2010-09-29
From Kim:
My 4th grader has started working on rectangular array that shows 2 factors of a number.

The problem reads on centimeter grid paper draw as many arrays as you can for each of the #'s 2,3,4, 5,6,7,,11,12,15 and 16. That would be great except I have no idea what they are talking about and I cannot offer any help to my daughter.

Answered by Penny Nom.
Factoring cubics and quartics 2010-09-29
From Tori:
How do you factor these types of questions:

* x^3 - 4x^2 - x + 4
*2x^3 - 3x^2 - 4x - 6
*x^4 - 15x^2 - 16

Answered by Robert Dawson.
4√3 + 2√1/3 2010-09-25
From Michelle:
How do you solve 4√3 + 2√1/3? Please show step by step.
Answered by Penny Nom.
The distance between the origin and a moving point 2010-09-24
From Norma:
I am having problems with this question find the rate of change of the distance between the origin and a moving point on the graph of the function below if dx/dt=5 cm/sec y=x^2+2
Answered by Penny Nom.
20% profit on the selling price 2010-09-24
From ata:
My cost is 100 and I want to make 20% profit on my selling price (not 20% of the cost). So, what should be calculation formula? Let's say my cost is 493.71 and I want to make 53.28% profit on the selling price; so how to determine the selling price?
Answered by Penny Nom.
Limits 2010-09-20
From rodrigo:
what's the limit of x+1/x-1, when x tends to 1?
Answered by Janice Cotcher.
order fractions from least to greatest 2010-09-20
From Kameron:
How do I work this out to find the answer from least to greatest ? 48/8, 12/16, 3/5
Answered by Melanie Tryer.
Elimination and substitution 2010-09-18
From Lauren:
Solve one using the method of substitution and the other with the method of elimination.

v a. y=5x+4
x=2y+1

b. 4x+3y=7
6x-3y=13

Answered by Penny Nom.
A strip of land 2010-09-15
From Walter:
How many linear feet of strip of land 15 feet wide does it take to make 1 acre?
Answered by Penny Nom.
La racine carrée et l'exposant une demie 2010-09-14
From Alain:
Bonjour. Je cherche une explication sur l'équivalence entre les exposants fractionnaires et les racines nième. Par exemple, comment prouve-t'on que la racine carrée correspond à  l'exposant une demie? merci
Answered by Pierre-Louis Gagnon et Claude Tardif.
An algebraic equation with fractions 2010-09-10
From leah:
If 6/x - (x-1)/2 = 4 then the LCD is 2x, right? so you get 12/2x - (x2-x)/2x = 8x/2x, right? then what do you do with the 2x when you turn it into a quadratic equation?
Answered by Penny Nom.
Mrs. Johns graded 121 math tests. 2010-09-10
From Tums:
Mrs. Johns graded 121 math tests. It took her 3 hours to grade all of the tests. Each test took the same amount of time to grade. About how many test did Mrs. Johns grade in 2 hours?
Answered by Penny Nom.
A building and a flag pole 2010-09-09
From paul:
A flag pole and a building stand on the same horizontal level. From the point p at the bottom of the building,the angle of elevation of the top t of the flag pole is 65 degrees. From the top q of the building the angle of elevation of the point t is 25 degrees.If the building is20 meters high. Calculate the distance pt
Answered by Penny Nom.
Decimal form 2010-09-08
From Tracy:
What is the decimal form of 0.14 x 10 the the first power?
Answered by Penny Nom.
A pattern 2010-09-01
From Hien:
How to find the pattern in this function?
x=1, y=o
x=2, y=1
x=3, y=1
x=4, y=2
x=5, y=2
x=6, y=3
x=7, y=3 and so on...

Answered by Tyler Wood.
Percentage improvement 2010-08-26
From shahrukh:
Hi...

We had a web page that was taking 12 secs to load. Now it loads in 3 secs. What is the percentage of improvement or how much faster is it working now. TO me it looks like it is working 3 times faster or there is a 300% improvement in the loading time. However my client says its ((12-3)/12*)100 = 75% improved. Which seems illogical on the face of it.

Thanks for your help
Shahrukh

Answered by Robert Dawson and Chris Fisher.
What is -2^2? 2010-08-25
From alex:
I know that (-2)^2 is 4 but what is -2^2?
Answered by Robert Dawson and Penny Nom.
Can a fraction be an even or odd number? 2010-08-25
From Kenneth:
Can a fraction be an even or odd number? If not , why?
Answered by Stephen La Rocque and Harley Weston.
Ordering fractions 2010-08-24
From COCO:
Hello. I am having lots of trouble with these types of problems. Here is one of them:1/2 1/18 3/6. And I need to order them from greatest to least. Please also explain how to do it.
Answered by Robert Dawson.
Exponential form 2010-08-24
From Courtney:
What the Exponential form? For Ex. X2*X5*X4??
Answered by Penny Nom.
A circular 20 acre lake 2010-08-13
From Ken:
I jog around a circular 20 acre lake. Please help me learn the distance of the perimeter of the lake so I will know my jogging distance. Thanks
Answered by Stephen La Rocque and Tyler Wood.
How many labels are left on a roll? 2010-08-13
From Melissa:
Is there a simple way to calculate how many labels are left on a roll? I thought there was a way to look at the individual label length, the label thickness and the core diameter and then measure the overall diameter to calculate how many pieces were left, but I'm struggling. I've seen a tool before- I thought we called it a circle calculator, but those parameters were all that I needed to input to spit out the correct count. I never saw what was behind the scenes for this calculation. Ie factoring in revolutions or the number of pieces on each layer, etc. Please let me know if you have any suggestions. Any assistance will be greatly appreciated!!! Thanks!
Answered by Penny Nom and Tyler Wood.
Differentiable on an interval 2010-08-12
From Dave:
Hi I was wondering if a function can be differentiable at its endpoint. For example if I have Y = X^2 and it is bounded on closed interval [1,4], then is the derivative of the function differentiable on the closed interval [1,4] or open interval (1,4). They always say in many theorems that function is continuous on closed interval [a,b] and differentiable on open interval (a,b) and an example of this is Rolle's theorem. Thank you for your help.
Answered by Robert Dawson.
A square field has an area of 289 meter squared 2010-08-02
From daniel:
a square field has an area of 289 meter squared. How do you calculate the dimensions
Answered by Stephen La Rocque and Penny Nom.
A fruit seller 2010-07-31
From Nazrul:
5% of total number of fruits of a fruit seller has been rotten and also 5% of them has been damaged during transportation. What is the percentage profit by which he sells the rest so that he profit 20% as a whole?
Answered by Penny Nom.
Difference quotient 2010-07-26
From Brandon:
i'm trying to figure out how to use the difference quotient with x^3, i have tried the following...(x+h)^2*(x+h)-(x+h)/h and i have only gotten as far as...(x^2+2xh+h^2)-(x+h)/h....i cannot see where i am goin astray, i know i am overlooking something simple, what am i overlooking?
Answered by Penny Nom.
Reflection in the line y = x 2010-07-21
From tousif:
Find the 2 x 2 matrix which represents a reflection in the line y = x? Please help....
Answered by Penny Nom.
Simplify (2x)3 2010-07-16
From emily:
how to Simplify (2x)3
Answered by Melanie Tyrer and Penny Nom.
The area of a 4 sided region 2010-07-14
From Cliff:
If I wanted to know the square footage of an area that has four different sides and only had the length of one side and the angles at the ends of that length; could the other lengths be found if I wanted to have a total of 130,680 square feet?
Answered by Robert Dawson and Tyler Wood.
Fibonacci and induction 2010-07-12
From James:
I'm trying to prove by induction that F(n) <= 2^(n-1) where f(1)=f(2)=1 and f(k)=f(k-1)+f(k-2) for k >=3 is the Fibonacci sequence
Answered by Stephen La Rocque and Tyler Wood.
Thickness of sand 2010-07-07
From RHONDA:
In an 18 x 33 ft area how much is 4 yards of sand in inches to cover this area.
Answered by Penny Nom.
An algebraic fraction 2010-06-28
From luis:
3x-2//4-3x=12
Answered by Stephen La Rocque.
Volume of a styrofoam cup 2010-06-23
From Stacy:
how do you find the volume of a truncated cup with height of 3, top diameter of 2.5, and a bottom diameter of 2
Answered by Penny Nom.
Selling price, cost and profit 2010-06-22
From Alex:
The profit on a certain television set is 16.5% of the cost price. If the profit is $330, Find:
a) the cost price
b)the selling price of the television

Answered by Penny Nom.
Exponential form 2010-06-16
From charlotte:
what is the exponential form of 6x6x6x5x5x5x5=
Answered by Robert Dawson.
Expand and simplify 2010-06-15
From Alaa:
Expand and simplify
(x-9)(x+2)

Answered by Penny Nom.
Distribution with Fractional Terms 2010-06-11
From Vincent:
Compounded semiannually. P dollars is invested at annual interest rate r for 1 year. If the interest iscompounded semiannually, then the polynomial P(1 + r/2)^2 represents the value of the interest after 1 year. Rewrite this expression without parentheses. Evaluate the polynomial if P=$200 and r = 10%.
Answered by Janice Cotcher.
Roosters, hens and chicks 2010-06-10
From Kamarah:
If a rooster is worth 5 coins and a hen is worth 3 coins and chicks 3 together are worth 1 coin, how many roosters, hen and chicks totaling 100 can be bought with 100 coins?

notes:
you must buy at least one of each type of fowl
no fractional fowl
chicks are purchased in multiple of 3
find 3 solutions

Answered by Penny Nom.
An input-output table 2010-06-07
From Brian:
Make an Input -output table to represent the function. Use 0,1,2,3 as the Domain. f(x)= 3+7x
Answered by Harley Weston.
Using the limit definition 2010-06-06
From Meagan:
Using the limit definition find the derivative of 3/(2x^2)
Answered by Harley Weston.
The outside surface area of a U shaped tube 2010-06-03
From Adolfo:
What is the formula for calculating the outside surface area of a U shaped tube?
Answered by Robert Dawson.
Compound interest 2010-06-01
From Kenneth:
Hello: I have a question regarding the following compound interest and future value calculation.

Year 1 P + rP equals balance after the first year.
Year 2 (P + rP) + r(P + rP) equals balance after the second year.
Year 3 ? equals balance after the third year.

This question is in two parts.

1. What would follow for year three?

2. I know that a pattern will develop. What will it be so that I can determine the extended pattern for following years ?

Answered by Penny Nom.
A cylindrical vessel with belled ends 2010-06-01
From George:
surface area of a vessel with a cylindrical length of 60 feet, a diameter of 10 feet, and with belled ends of 5-foot radius
Answered by Penny Nom.
Who is more extravagant? 2010-06-01
From likhitha:
kunal and ramit have Rs.40 and Rs.50 respectively. Kunal spends Rs.18 and ramit spends Rs.21.Who is more extravagant ?
Answered by Penny Nom.
graph y=(2x^2-3x)e^ax 2010-05-31
From James:
graph y=(2x^2-3x)e^ax
Answered by Robert Dawson.
A fuel tank with a broken gage 2010-05-28
From Kinsley:
Hello, I have a fuel tank with a broken gage. It is a 2000 gallon tank. The dimensions are 64" dia. x 146" long. The fuel gage doesn't work, so the only measurement I can do is put a stick in it to see what is left. Currently it is at 19". PLEASE NOTE: the tank is laying on its side, meaning the long side is on the ground. So, how do I determine how much fuel is in the tank by the inches remaining? Thank you very much.
Answered by Tyler Wood.
Weighing a 1000 lb cylinder on a 500 lb scale 2010-05-28
From Jerry:
I am trying to weigh a large possibly 1000 lb cylinder on a 500 lb scale, is this possible? If so what is the actual formula to figure out the correct weight? I am told it is possible with the fulcrum method but have not been able to find a formula. Thanks Jerry
Answered by Robert Dawson.
More on a truncated cone 2010-05-28
From Mike:

Question from Mike, a parent:

I was reviewing this question and answer:
http://mathcentral.uregina.ca/QQ/database/QQ.02.06/phil1.html

But I have trouble with this part:
Now if we express the radius of the inside circle as r and the outside circle's radius is R, then this means r/R is 911/1728. But earlier we said that the outside radius R is simply w more than the inside radius r, so R = r + 282. That means that r/R = r/(r + 282). Now we can simply solve the equation for r:
r/(r+282) = 911/1728
This means r = 314 mm (with rounding).

Can I get more detail on the method to solve for r?

Thank you,
Mike


Answered by Penny Nom.
The number of gallons of water in a basement 2010-05-23
From George:
A basement with dimensions 68 ft long by 42 ft wide by 10 ft deep is flooded to the ceiling calculate the number of gallons of water in the basement. 70
Answered by Penny Nom.
Four digit numbers 2010-05-22
From tyeisha:
Whats all the four digit numbers you can come up with using numbers 0-9
Answered by Tyler Wood.
How many bags will it take to fill this? 2010-05-22
From David:
A garden lot..360 square feet and 6 inches deep..you want to fill this with mulch and use 2 cubic. foot bags...how many bags will it take to fill this? Thank you...Dave
Answered by Tyler Wood.
Difference of cubes 2010-05-22
From Anad:
how can we prove a^3 - b^3 is equal to (a-b)(a^2+ab+b^2)?
Answered by Penny Nom.
(x+1)(x+2)(x+3)/(x+1)(x+2) 2010-05-19
From Nazrul:
Simplify : (x+1)(x+2)(x+3)/(x+1)(x+2)
Which answer is correct:
(i) x+3
(ii) (x+2)^2(x+3)
Please help me.

Answered by Harley Weston.
A fact family 2010-05-19
From PRISCILLA:
ELLIE HAS 12 BOOKS. SHE GIVES AWAY 3 BOOKS. ELLIE HAS _LEFT. _ - _ = _ _- _ = _ _+ = _ _ + _ = _
Answered by Robert Dawson.
A scale model of a city 2010-05-18
From Briana:
Well I have to create a three dimensional city for a geometry project and it requires us to use precise scal factor and well we want to build a medium sized city but our teacher said that If we make or buildings to high our houses would look like super small compared to the tall building were trying to figure out if we were to build a medium sized city what scale factor should we use to make the houses big enough to see and design?
Answered by Walter Whiteley and Tyler Wood.
A quartic equation 2010-05-18
From Austin:
Z^4-10z^2=-9
Answered by Penny Nom.
How many sq. ft in 2 1/2 ft x 2 1/2 ft? 2010-05-16
From jim:
how many sq. ft in 2 1/2 ft x 2 1/2 ft
Answered by Penny Nom.
The circumference of a circle 2010-05-12
From Morgan:
find the circumference of a circle with radius 8. im having trouble i just don't understand. thanks for your help i really do appricate it.
Answered by Penny Nom.
Painting a hemispheric dome 2010-05-09
From Jone:
If a hemispheric dome has a height of 20m and it take 5 gallons of paint to paint the floor, how many gallons of pain are needed to paint the inside of the dome?
Answered by Penny Nom.
Line of sight 2010-05-06
From David:
I live in St. Joseph, Michigan and there is an ongoing argument regarding line-of-sight over the horizon.

Standing on a 200 foot high bluff here, people swear they can see the top of the Willis (nee Sears) Tower in Chicago, which is about 1653 feet high.

It is my contention that this is actually a "refracted reflection" and not direct-line-of-sight.

So, to settle the argument, I'd sure like some simple explanation for this, even if-and I hope not-I am incorrect.

David

Answered by Harley Weston.
f(x) = x + 3 2010-05-02
From becca:
Find the function value F(1)if f(x)=x+3
Answered by Penny Nom.
How much concrete do I order? 2010-05-01
From Carol:
We are having a block foundation installed under our house that measures 80ftX28ft and I need to know how to figure out how much concrete to order. The footer will be 12 inches wide and 4 inches deep all the way around the house. How do I figure the square footage of my house? I know area = LxW but do I double that since there 4 sides to my house?
Answered by Penny Nom.
The rate of change of y with respect to x 2010-04-29
From Tom:
I just had a quick calc question about wording that wasn't ever addressed in class. When the book says "the rate of change of y with respect to x", should it be considered how fast y is changing in comparison to x?

I ask because the textbook says that "y is changing 3 times faster than x, so the rate of change of y with respect to x is 3." I'm use to rate being like velocity, as in units of distance per units of time. All we're told in class is that it's the slope of the tangent line, I was hoping you could clarify for me what exactly is meant by the wording of a "rate of change of something with respect to something else". More specifically, what "rate" and "with respect to" mean within this context?

Thanks for your time

Answered by Harley Weston.
A trig equation 2010-04-28
From Steve:
2cos^2(X)-2sin^2(x)+1=0
Answered by Harley Weston.
The width of a picture frame 2010-04-28
From Daniel:
The frame of a picture is 28 cm by 32 cm outside and is of uniform width. What is the width of the frame if 200 cm ^2 shows?
Answered by Penny Nom.
cos(x) = sin(x - 1) 2010-04-28
From alex:
In the equation cos x = sin x-1 for -pi/2 A: solve for x graphically
B: solve algebraically and prove the solution is correct.

Alex

Answered by Penny Nom.
y = - log(x) 2010-04-28
From Alex:
y= - log(x), where y = 4.3
solve for x.

Answered by Harley Weston.
Adding five positive integers that have a product of 48 2010-04-27
From Lorraine:
What is the smallest possible sum that can be obtained by adding five positive integers that have a product of 48.
Answered by Chris Fisher.
Tiles 2010-04-25
From Pat:
how many 6 inch tiles will I need to purchase for an area that is currently covered with 351 4 inch by 4 inch tiles. Each box of 6 inch tiles states it contains 16 pieces and covers 4 square feet.
Answered by Penny Nom.
The height of a flag shaft 2010-04-25
From Sarah:
A man standing 20metres away from a tower observes the angles of elevation to the top and bottom of a flag shaft standing on the tower as 62degrees and 60degrees respectively. Calculate the height of the flag shaft.'
Answered by Penny Nom.
Algebraic fractions 2010-04-22
From rory:
3x/(x²-64)+4/(x²-6x-16)=
Answered by Robert Dawson and Harley Weston.
Profit 2010-04-21
From joyann:
a Raleigh road runner 16" unisex bike which cost $892 was sold for $1052.56 calculate
A. the profit made on the bike
B. the percentage profit

Answered by Penny Nom.
If (x, 4) is equidistant from (5, -2) and (3, 4), find x. 2010-04-21
From abeth:
If (x, 4) is equidistant from (5, -2) and (3, 4), find x.
Find the point on the y - axis that is equidistant from (-4, -2) and (3, 1).

Answered by Penny Nom.
Two problems 2010-04-13
From Dorothy:
1. Explain why the number 123, 456, 789, 101, 112 cannot be a perfect square. (Hint: What is the units digit?)

2. A substance doubles in volume every minute. At 9:00A.M., a small amount is placed in a container. At 10:00A.M., the container is just full. At what time was the container one-eighth full?

Answered by Robert Dawson.
Exactly two lines of symmetry 2010-04-11
From debbie:
i am looking for a quadrilateral with exactly two lines of symmetry. please help! thank you.
Answered by Tyler Wood.
The derivative of y=x^x 2010-04-09
From David:
So, its David, and I was wondering about the derivative of y=x^x. I have often seen it be shown as x^x(ln(x)+1), but when I did it through limits it turned out differently. Here's what I did:
It is commonly know that df(x)/dx of a function is also the limit as h->0 of f(x+h)-f(x)/h. To do this for x^x you have to start with lim h->0 ((x+h)^(x+h)-x^x)/h. The binomial theorem then shows us that this is equal to lim h->0 (x^(x+h)+(x+h)x^(x+h-1)h+...-x^x)/h
This is also equal to lim a->0 lim h->0 (x^(x+a)+(x+h)x^(x+h-1)h...-x^x)/h.
Evaluating for a=0 you get lim h->0 (x^x+(x+h)x^(x+h-1)h...x^x)/h
Seeing as the last 2 terms on the numerator cancel out you can simplify to a numerator with h's is each of the terms, which you can then divide by h to get:
lim h->0 (x+h)x^(x+h-1)... which when evaluated for h=0 gives us: x(x^(x-1)). This statement is also equal to x^x.
This contradicts the definition of the derivative of x^x that is commonly shown. So, my question is: can you find any flaws in the logic of that procedure? I do not want to be shown how to differentiate x^x implicitly because I already know how to do that.

Answered by Robert Dawson.
If a third is a fourth of a number, what is the number? 2010-04-08
From larry:
If a third is a fourth of a number, what is the number?
Answered by Penny Nom.
Backfilling the area around the outside of a pool 2010-04-07
From Craig:
I need to get dirt to fill an area 25ft x 25ft (think of a giant sandbox) HOWEVER, there is a pool 18 ft round in the middle of it. We need to backfill the area around the outside of the pool 6" deep. How much dirt will I need?
Answered by Penny Nom.
The area of a field of view 2010-04-07
From Frank:
I know we have a field of view of 90 degrees and the distance we can cover is 6 miles. What is the area we can cover/view?
A formula would be nice to have since the diameter or distance of the wedge area can change and the degree of the field of view can also change.
Thank you and I look forward to hearing from you.
Frank

Answered by Penny Nom.
A model for the population of a Canadian city 2010-04-07
From jess:
Let t^2 represent t-squared. The population of a Canadian City is modelled by: f(t) = 12t^2 + 800t + 40,000, where t is the time in years. When t= 0, the year is 2007.
a) According to the model, what will the population be in 2010?
B) In what year is the population predicted to be 300,000?

Answered by Penny Nom.
The derivative of cos^3x 2010-04-06
From Erson:
Find y' of the given function: y = cos^3x.
Answered by Harley Weston.
Body measurements 2010-04-06
From Amirul:
Recently I'm proposing my research question to my teacher for my extended essay. I'm an IB student. My research question is regarding the estimation of human in buying trousers through reference of neck. What does the relation between the diameter of the neck and the diameter of the waist? I want to see how far does the estimation theory is true for different type of people with different BMI(body mass index)..

But teacher said that it is golden ratio...so nothing interesting... =( really??? But i search on net.... state that my idea seems do not have any relation with the golden ratio so far..... i just want ask you... am I able to perform in my extended essay if i continue with this research question??

Answered by Robert Dawson.
Fitting a sofa through a doorway 2010-04-06
From Yvonne:
Hi, I would be most grateful if you could help me, because I have no idea how to work this out! I am moving from a house to a flat, and the doors in the flat are 70 cm wide. I have a sofa 88 x 88 x 215. Will it go through the door if we tip it sideways at an angle?

Thanks, Yvonne Beck

Answered by Robert Dawson.
Pyramids and parallelograms 2010-04-01
From Vivi:
Hi, my name is Vivi, and math teacher is giving a geometry test. I tend to forget how to find the surface area and volume of triangular pyramids and parallelograms. Do you have a way of remembering how to find the surface area?
Answered by Harley Weston.
Tiling a floor 2010-03-31
From shane:
a floor in a house is 12'6" in width and 10'4" in length. Tiling the floor with each tile 5" on each side. First express the square footage into an improper fraction. Second express the area of each tile in square feet. Third how many tiles needed to tile the floor. Fourth explain how answers relate to real world
Answered by Harley Weston.
A system of equations 2010-03-30
From samantha:
the equations of two lines are 6x-y=4 and y=4x +2.what is the value of x in the solution for this system of equations?
Answered by Penny Nom.
A 25% discount 2010-03-29
From Susan:
If something costs 1.00/per lb. and that price includes a 25% discount. How do you calculate what the price was before the discount?
Answered by Penny Nom.
Sample size 2010-03-29
From Rae:
What sample size was needed to obtain an error range of 2% if the following statement was made? "75% of the workers support the proposed benefit package. These results are considered accurate to within + or - 2%, 18 out of 20 times. This seems like a straight forward question but I'm getting it wrong. Could you please help me out even just the set up would be appreciated so I can see if that's where I'm going wrong. Thanks
Answered by Harley Weston.
The sides of a 30-60-90 triangle 2010-03-29
From maris:
Find the sides of a 30-60-90 triangle whose perimeter is 6. The solution must use systems of equations using 3 variables.
Answered by Penny Nom.
The area of a polygon 2010-03-28
From John:
Question from John, a student:

How do you find the area of a polygon with any given amount of sides?

Answered by Harley Weston.
The height of a hill 2010-03-26
From Amber:
A surveying team determines the height of a hill by placing a 12-foot pole at the top of the hill and measuring the angles of elevation to the bottom and to the top of the pole. They find the angels of elevation. Describe how to find the height of the hill.
Answered by Penny Nom.
A radio tower 2010-03-26
From Alex:
The height of a radio tower is 450 feet, and the ground on one side of the tower slopes upward at an angle of 10 degrees. How long should a guy wire be if it is to connect to the top of the tower and be secured at a point on the sloped side 110 feet from the base of the tower?
Answered by Harley Weston.
A roll of paper 2010-03-25
From keith:
how many yds of paper for a roll with a width of 6.875 and a core of 3 in and a diameter of 40in?
Answered by Robert Dawson.
The angle of intersection between two lines 2010-03-25
From Madhumitha:
How to find the angle of intersection between a line x=249 and another line which is of the form y=m*x+c; Line1 has a slope of infinity so what does the angle of intersection turn out to be? Or how can i find it?
Answered by Chris Fisher.
Modelling the roof of a house 2010-03-23
From Sandi:
This question has been haunting my dreams at night. It states the cross section of the roof of a house is modelled by the function y= -5\12|x-12|+5, where y>0 or y. Now I'm suppose to graph the function. b) Find the slope, height, length of sides, and base of the isosceles triangle. c)Explain what transformation must be applied to the graph of y=|x| to obtain the graph of y= -5\12|x-12|+5, where y>0 or y=0. 1st I put the equation into my calculator and graphed it. I got 10 for the height. We have had 2 sub teachers this past week. The first one told me the height should be 12 the other one told me 5. So I'm really confused. If I use my 10 units as height and the Pythagorean Theorem I get my sides to be 26 units the base 48 and my slope I calc to be 5/12. Both teachers are pretty sure I'm wrong and they are right but we all have different answers. Please help put this problem to rest so I can rest soundly at night. Thank you for your time.
Answered by Penny Nom.
Factorials 2010-03-23
From Leah:
When should you use factorials?
Answered by Robert Dawson.
What is 8.6597 divided by two thirds? 2010-03-22
From patsy:
what is 8.6597 divided by two thirds
Answered by Penny Nom.
A quadrilateral with 4 known sides and 1 known angle 2010-03-19
From samuel:
Name: Samuel
Status: Student

I have a quadrilateral with 4 known sides and 1 known angle, and I'm trying to evaluate the other angles of my quadrilateral.

By the law of cosines, I can easily find my opposite angle (using the diagonal as a basis for the equation).

However, to find the two remaining angles, I have found no other way so far than to use the other diagonal, which can be found with the equation attached (from geometry atlas).

Is there any simpler way?

Answered by Robert Dawson and Harley Weston.
How many zeros? 2010-03-18
From jeff:
1 light yr is 6,500 billion miles, the galaxy "Andromeda" is 2,500,000 million light yrs, how many zeros are in this equation ?
Answered by Robert Dawson.
Factors and zeros of a polynoial 2010-03-17
From Susan:
find a polynomial function of degree three with -3 as a zero of multiplicity 2 and 4 as a zero of multiplicity 1.
Answered by Tyler Wood.
Metric prefixes 2010-03-16
From ali:
How do you convert zectometres to yoctometres
Answered by Robert Dawson.
Decimals and fractions 2010-03-16
From fay:
why do we use decimals in place of fractions in everyday life?
Answered by Robert Dawson.
Constants and coefficients 2010-03-12
From lourdes:
identify the coefficients, constant term(s), and like terms of the expression.
8x + 9 - 3x
17 - 2a + 5a - 1
7m - 7 + 6m - 6
-10 -15r -22r + 8

Answered by Penny Nom.
The nth derivative of x^(n-1) log x 2010-03-10
From shambodeb:
This is a successive differentiation problem by Leibnitz theorem

If y = xn-1 log x ; Proof nth derivative y(n) = (n-1)!/x

Answered by Harley Weston.
Cooling 2010-03-07
From Lori:
If a house is always at 20 degrees celsius. Water (tea) boils at 100 degrees celsius. 5 minutes later the tea was 70 degrees celsius. Use an equation to predict the temperature after 20 minutes.
Answered by Tyler Wood.
Prove A intersect B =X iff A = X and B = X 2010-03-06
From Gloria:
how would you prove A intersect B =X iff A = X and B = X
Answered by Tyler Wood.
Standard form 2010-03-05
From Laura:
How do I change 3y=4x+1 into standard Ax+By=C standard form. This is my son's schoolwork. I homeschool him. I got the answer to the problem as 3y=4x+1, but it needs to be written in standard form and I know it needs to read 4x-3y=-1, but how do I get that answer into standard form. I need to know how to explain it to him. Can you show me? Thank you so much.
Answered by Penny Nom.
The volume of a frusta of a hexagonal based pyramid 2010-03-04
From sarah:
Volume of a frusta of a hexagonal based pyramid
Answered by Penny Nom.
How many CDs and videos did the store sell? 2010-03-01
From dawn:
A used book store started selling CDs and videos. In the first week,the store sold 40 used CDs and videos,at 4.00 per CD and 6.00 per video.The sales for both CDs and videos totaled 180.00 she wrote a system of equations to represent the situation.Then she graph the system of equations

thanks- Dawn

Answered by Tyler Wood.
Ordering fractions 2010-02-26
From jessie:
I'm having alot of trouble with orderig fractions from least to greatest! I'm in 5th grade....and my problem is, 5/8,1/2, and 3/4
I need to know HOW to order them.........can anyone give me a simple explanation?

Answered by Penny Nom.
Ordering fractions 2010-02-26
From Brianna:
I'm a seventh grader and we're learning about fractions. How would you list the numbers in order from least to greatest?
The options:
A. 9/14 , 17/28 , 11/21
B. 17/28 , 9/14 , 11/21
C. 9/14 , 11/21 , 17/28
D. 11/21 , 17/28 , 9/14

Answered by Penny Nom.
The test for some disease is 99% accurate 2010-02-24
From baaba:
Assume that the test for some disease is 99% accurate. If somebody tests positive for that disease, is there a 99% chance that they have the disease?
Answered by Chris Fisher.
Is 0.1234567891011.... periodic? 2010-02-23
From Adriana:
0.1234567891011.......Is this fraction periodic ?
Answered by Penny Nom.
15% profit 2010-02-21
From daviann:
calculate the cost price,given
selling price=$304
percentage profit=15%
what is the answer?

Answered by Penny Nom.
A perfect square 2010-02-20
From george:
can this number 400....006 be a perfect square . how to solve problems like this.
Answered by Chris Fisher and Tyler Wood.
The game of 24 2010-02-20
From haley:
hello.... we have these very hard make 24 stumpers. i am stuck on one.... how can you make 24 using the numbers 7-7-13-18?? thanks!
Answered by Claude Tardif.
15 men can do a piece of work in 7 days 2010-02-20
From Kenneth:
If 63 books cost $126, what will 125 books cost?

If 15 men can do a piece of work in 7 days, in how many days can 21 men do the same work?

Answered by Penny Nom.
An algebraic equation with fractions 2010-02-19
From Ingrid:
What is the solution set of the equation x over x plus 4 = 1 over x plus 3 = 28 over x to the 2nd power - x- 12?
Answered by Penny Nom.
1/6 x 5 x 12 = what 2010-02-19
From dana:
1/6 x 5 x 12 = what
Answered by Lorraine Dame.
Square poles 2010-02-16
From Steve:
If 1 pole=16.5', and you have an area that is 146 sq. poles, how many sqaure feet would you have? I calculate that to be 146(16.5)X146(16.5)=5,803,281 sq. feet Is that correct? I am told I need to divide by 146, which makes the answer 39,748. I do not understand why I need to divide by 146....
Answered by Penny Nom.
The dimensions of a flag 2010-02-15
From dawn:
a state flag was twice as long as it was wide and had an area of 288 sq inches, what were its, dimensions
Thanks

Answered by Penny Nom.
The price before tax 2010-02-15
From dawn:
the sale tax on a cape was 6%,and total price with tax was $92.75,what was the price of the cape?
Answered by Penny Nom.
Birds and cats 2010-02-13
From Sophia:
While visiting a pet store, you notice that there are only birds and cats in the cages. You can't help but wonder how many of each animal there is in the yard. But when you ask the store manager how many of each animal he has, he refuses to give a direct answer. He says there are 16 animal heads and 42 animal feet. How many birds and cats are there in the pet store? It said to show all your work. Explain in words how you found your answer. Tell why you took the steps you did to solve the problem.
Answered by Penny Nom.
Least common denominator 2010-02-13
From Priscila:
3/8 + 4/5 + 7/3 + 9/10 = ?

Thank you for your assistance.

Priscila

Answered by Penny Nom.
Combining fractions 2010-02-10
From Nick:
Combine the fractions

2m/t + 5/mt

Answered by Penny Nom.
The angle at which the road is inclined 2010-02-10
From abeth:
a mountain road drops 5 m for every 22 m of road. Calculate the angle at which the road is inclined to the horizontal to the nearest degree.
Answered by Penny Nom.
Mixed numbers and multiplication 2010-02-08
From Nick:
Can you explain how this works, I have been going at it for hours and have a test friday and just cant get my head around the question
3 1\4 * 1/7 * 8

I have to express in lowest terms

Answered by Penny Nom.
Order of operations 2010-02-07
From addie:
(3+10) x 10 - 8 x 4 =
Answered by Penny Nom.
Fencing a garden 2010-02-04
From kris:
How many meters of fencing do i need to enclose a garden if the yard is 26 meters long and half as wide?
Answered by Penny Nom.
Bending a wire to form shapes 2010-02-04
From Geraldine:
a wire bent into the shape of a square encloses an area of 25cm squared. then the same wire is cut and bent into two identical circles. what is the radius of one of the circles round to the nearest hundred
Answered by Penny Nom.
Scale factor 2010-02-02
From Darlene:
A towers actual height is 300m. An architect makes a scale drawing using a scale factor of 0.0001

A) how tall is the tower in the drawing?

B) if the drawing was 4cm high, what would the scale factor be?

Answered by Penny Nom.
Everything in a cube 2 miles long? 2010-01-31
From Naresh:
in a book, i got to read this :

Is it possible to pack the entire population of earth and everything that was created by humankind in a cube whose edges are 2 miles long ?

Is it possible? Thanks.

Answered by Stephen La Rocque.
The height of a roof 2010-01-31
From carl:
Width of my roof I am building is 5M at baseline, and the pitch is 40%. What will the height be, and how can I work this out in the future.
Answered by Penny Nom.
1^3 + 2^3 + 3^3 +4^3 ... n^3 = ? 2010-01-29
From ireimaima:
Hi..
Can u please help me with this question.. I find that when i test eg: n=2 for n (n+1) /4, it seems that it does not giving me the right answer of 1^3 + 2^3 = 9 but 3/2... i'm confuse..can u please help me..thanks so much

Prove that: 1^3 + 2^3 + 3^3 +4^3………………………………..n^3 = n (n+1) /4

Answered by Penny Nom.
Solving a triangle 2010-01-25
From Paige:
how do i solve a triangle with one angle of 73 degrees, one angle of 32 degrees, and one side of 23cm?
Answered by Harley Weston.
An altitude of the triangle intersect a side of the triangle 2010-01-23
From Sarah:
In what type of triangle does an altitude of the triangle intersect a side of the triangle at its midpoint??
-Sarah

Answered by Walter Whiteley.
Angle of incline 2010-01-20
From Alan:
how do I fnd the angle of an incline with a measurement of 0.042 with an adjacent of 1.2mtrs?. Thank You
Answered by Penny Nom.
Expressing the natural numbers in terms of 4 fours. 2010-01-19
From Steve:
When I was studying mathematics, there were several of us that tried to express the natural numbers in terms of 4 fours. Allowing the use of any known math operation, for example n to the zeroth power is 1.Is there more info on this exercise. This was done mentally, nothing written down.
Answered by Robert Dawson.
Cubic feet 2010-01-16
From deb:
1 room 49x24 1 room 18x18 need 6 inch deep can u convert to cubic feet for me thank you
Answered by Penny Nom.
A circular photo in an octagonal frame 2010-01-13
From Mike:
I have a circle photo 18 and one quarter inches round. I want to cut frame for it in a octagon shape. What would the angle and the length of cut be?
Mike

Answered by Robert Dawson.
Trigonometry and picture hanging 2010-01-13
From george:
The top of a picture 1m high 0.8m from the ceiling. At a point on the ceiling directly in front of the picture, we wish to install a light so that the angle subtended by the picture equals to the angle of depression of the top of the picture. How far out from the wall should the light be installed?
Answered by Penny Nom.
Mixed numbers 2010-01-13
From Linh:
Perform the indicated operations:
6 1/4 + (- 2/3) x 1 1/2

Answered by Penny Nom.
A proof by induction 2010-01-12
From Bhavya:
Prove by induction that if Xi >= 0 for all i, then

(Summation Xi from 1 to n)^2 >= Summation Xi^2 from 1 to n

Answered by Penny Nom.
Improper fractions 2010-01-12
From Blanca:
Between which two consecutive natural numbers does the improper fraction 57/56 lie?
Answered by Robert Dawson.
A proof involving real numbers 2010-01-11
From Amper:
Let a,b is an element of real numbers, and suppose that for every x>0 we have a is lesser than or equal to b+x.
(a) Show that a is lesser than or equal to b.
(b) Show that it does not follow that a is lesser than b.
i'm feeling bad of having no idea with this, hope i you can help me. GRACIAS!!

Answered by Penny Nom.
The intervals where the function is positive and negative 2010-01-10
From Ron:
Hello

I'm trying to find out the intervals where the function is positive and negative. It's for a polynomial function y= -(x+2)^2 (x-2) and y= (x+1)(x+4)(x-3)

I have tried the right and left side of each x-intercepts, but I still don't understand the results

thank you for your help

Answered by Penny Nom.
(x+3) is a factor of 2x^3-3x^2+px+30 2010-01-09
From Ruby:
(x+3) is a factor of 2x^3-3x^2+px+30.
Determine the value of p

Answered by Penny Nom.
Factoring 2010-01-08
From jim:
i am having trouble with grade 12 factoring in general, i just do not understand it, please help! here is and example of a question:
12ac+28bc-3ad-7bd and it gets tougher from there like 7a(squared)-14ab+7b(squared) any help is greatly appreciated

Answered by Stephen La Rocque.
A triangular pyramid 2010-01-05
From ally:
how many vertices, edges, faces in a triangular pyramid?
Answered by Robert Dawson.
Holiday for 7 golfers 2010-01-05
From Andy:
I am having difficulty scheduling pairings for a forthcoming Golf holiday. There are 7 players and we will be playing 7 rounds - each round will be a 3 ball and a 4 ball. I am trying to work out a schedule that means everyone plays with each other roughly the same number of times (with a minimum of, say, 3 times if possible). Also, we are looking to ensure everyone plays in roughly the same number of 3 and 4 balls. Any suggestions would be much appreciated (we leave in 10 days time). Thank you.
Answered by Victoria West.
How many 4x4 tiles would I need for 10 sq ft? 2010-01-04
From Chris:
how many 4x4 tiles would I need for 10 sq ft
Answered by Robert Dawson.
Profit increase from 10% to 15% 2010-01-03
From taha:
A shopkeeper sold a pen for $13.20 to a make a profit 10 percent.In order to earn a profit of 15 percent,he should have sold it for?
Answered by Penny Nom.
Four digit numbers with no repeats 2010-01-02
From Daniel:
I want to know how many different numbers there would be between the numbers 0000-9999 that do not have any numbers that duplicate or repeat in the number? For example the first number would have to be 0123 and the last would be 9876 (I think) Thanks!
Answered by Penny Nom.
Write a function for the area in terms of x 2010-01-01
From Almarie:
Lauren has 40 feet of metal fencing material to fence three sides of a rectangular garden. A tall wooden fence serves as her fourth side. Write a function for the area of the garden A in terms of x, the width in feet.
Answered by Penny Nom.
A linear equation with fractions 2009-12-31
From Michelle:
How would I solve: (-2)(-1/2x + 5) = (-2)(8)?
Answered by Penny Nom.
The volume of a frustum 2009-12-29
From dave:
I have a frustum top 1.7r bottom .55r and 2.14 h I have to calculate cement in a silo every week. I am not very good at maths but i have been adding the top and bottom to get an average so as to turn it into a cylinder and i come up with 8.5m3 I know that the correct volume is 9.24 m3. Can you tell me why getting an average width on the top and bottom of a frustum doesn't work.
Thank you.

Answered by Chris Fisher.
Finding Specific Heat of a Substance 2009-12-26
From erin:
Hi- 2 kg metal requires 1.00 X 10^4 J of heat to raise its temperature from 20 degrees C to 40 degrees C. What is the specific heat capacity of the metal?
Answered by Janice Cotcher.
15 golfers 3 rounds fivesomes 2009-12-23
From Dewey:
I have 15 golfers playing 3 rounds of golf. We will be playing in fivesomes. Any formula that has everyone playing with a different person each round?

Thank you. Dewey

Answered by Robert Dawson.
Perimeter 2009-12-23
From Jalyn:
I'm doing this math project comparing measurements and I want to know the perimeter of the Upper Peninsula of Michigan. Please Help me!

-Jalyn

Answered by Robert Dawson and Walter Whiteley.
The tens digit of 0! + 1! + 2! + 3! + ... + 2000! 2009-12-21
From Alicia:
What is the tens digit of 0! + 1! + 2! + 3! + ... + 2000!?
Answered by Robert Dawson and Penny Nom.
Maximizing the area of a rectangle 2009-12-17
From rachel:
A rectangular field is to be enclosed by 400m of fence. What dimensions will give a maximum area?
Answered by Penny Nom.
A trig question 2009-12-15
From A trig question:
Hey, my name is Candle
I'm in academic math10 and am stuck on my trig... one question I thought I had right because i used the cosine law I got wrong and can't figure out why... here's a copy of the question. (i guessed it was D... but my teacher said it's B)
Thanks
Candle

Answered by Robert Dawson.
A 4 digit number 2009-12-15
From Lisa:
Billy Club was assigned the task of putting numbers on all the playground balls used during daily recess. Billy will number the balls using the following rules:

1. It will be a 4-digit number.

2. The digit in the thousands place will be a 1 or a 2.

3. The digit in the hundreds place will be a 2, 4, or 6.

4. The digit in the tens place will be an odd number.

5. The digit in the ones place will be greater than six.

How many balls can Billy number if he follows these rules?

Answered by Robert Dawson.
A telephone pole on a slope 2009-12-14
From Marissa:
A 10 meter telephone pole casts a 17 meter shadow directly down a slope when the angle of elevation of the sun is 42 degrees. Find the angle of elevation of the ground. Its a law of sines problem.
Answered by Penny Nom.
Decimal equivalent 2009-12-13
From Asia:
In a forest,it rained 36 out of 90 days. What is the decimal equivalent of 30 out of 90 days
Answered by Penny Nom.
4x +3/4 = 1/4 2009-12-12
From Lynda:
4x +3/4 = 1/4

I know the answer is -1/8 but I don't know how to show the process of getting the answer

Answered by Penny Nom.
A number base that's not a positive integer 2009-12-11
From Nick:
Is it possible for a number to have a base that's not a positive integer? Base 1 gives a result that's meaningless but possible. I have no concept of a base 0, a negative base, or a fractional base, never mind any other base. I think there isn't any base that's not a positive integer, but, knowing that math keeps jumping ahead and sometimes has inventions before anyone knows how to exploit them, I think I'd better ask.

Thank you.

Nick

Answered by Robert Dawson and Claude Tardif.
A concrete slab 2009-12-10
From Mark:
there are 30 cubic yards of concrete, how many feet will it cover with a 4" slab?
Answered by Penny Nom.
e = m c^2 2009-12-09
From Jim:
Please explain the following formula: E=MC2 (squared)
Answered by Robert Dawson.
Game of 24 2009-12-09
From Tony:
Checked out all of the previous inquiries into the crazed game, and found none with these particular numbers: 1, 4, 6, 7. I've tried every method I can think of, please Help! Tks.
Answered by Robert Dawson, Penny Nom and Claude tardif.
How fast is the distance between the two cars decreasing? 2009-12-08
From Jenny:
Two cares are on a collision course toward point P. The paths of the two cars make a 30 degree angle with each other. The first car is 40 km from P, and traveling toward P at 16 km/hour. The second car is 50 km from P, traveling at 20 km/hour. How fast is the (straight line) distance between the two cars decreasing. (Hint: Law of Cosines)
Answered by Harley Weston.
Simplifying a quadratic expression 2009-12-07
From Sabbie:
Hi, I need help solving this equation for my physics homework,so I can solve the quadratic equations for x and y. I've derived the following equations from the information provided in the question.

2a+3b = 4 -- 1
2a^2 + 3b^2 = 62 -- 2

a=2-(3/2)b is substituted into equation 2.

so, i get
2 (2-(3/2)b)^2 + 3b^2 = 62

this is where the problem begins. i cannot work through this to get to the correct quadratic equation

5b^2-8b-36=0

I can't seem to be able to figure out where I keep going wrong. I would really appreciate it if you could give me a step-by-step breakdown of the workout.

thanks!

Answered by Penny Nom.
(9 - x^2)/(x - 3) 2009-12-04
From Sandy:
9-x^2/x-3
I need to know how to solve this.
Thanks

Answered by Penny Nom.
(24x3)+(7x8)-(20/5)+(4x7) 2009-12-01
From Marilyn:
(24x3)+(7x8)-(20/5)+(4x7)
Thought I had the answer but the form says no. Could you solve it for me please.

Answered by Penny Nom.
Four fours 2009-11-29
From linda:
Four Fours order of operations challenge i need problems using 4's only that have an answer of 13, 18, and 19 than you
Answered by Penny Nom.
The graph of f(x)=1/x -2 2009-11-29
From Kapilan:
Please answer the following question:
Sketch the graph of f(x)=1/x -2

Answered by Harley Weston.
A simplification 2009-11-21
From Lou:
In the following problem, [(7)/(x+2)]-[(x+8)/(4-x^2)]+[(3x-2)/4-4x+x^2)] what steps do you use to convert the middle portion to become +[(x+8)/(-4+x^2)]? The answer to the problem is supposed to be (11x^2-18x+8)/(x+2)(x-2)^2
Answered by Penny Nom.
The graph of f(x-2) 2009-11-19
From Kapilan:
Hi please answer the following question:
Given the function f(x)=x^2 sketch the graph of f(x-2)

Answered by Robert Dawson.
Factoring 2009-11-19
From danyelle:
81x^2+36x+4

factoring i dont understand it at all

Answered by Robert Dawson.
Ascribing a value to 1/infinity 2009-11-19
From Jack:
Hello, and, in advance, thanks for answering.

I came across the problem of ascribing a value to 1/∞ (one divided by infinity) recently, I heard many things: that it is infinitesimally small (i.e. .0000000000...1 the most intuitive), that it is 0 (the most ludicrous of them all in my mind), and that it is not definable (which makes the most sense, although is a bit of a let down).

I know that lim (x->∞) 1/x = 0 and this is often used as an argument for all three possibilities. So what's the ruling on this? And, I know this question has already been answered, but for a little modification; is there any way to prove the answer that seems to be the most prevalently used (not definable as ∞ is a concept) with mathematical logic? Or is it just because of the definition of ∞?

Answered by Robert Dawson.
Coefficients 2009-11-17
From Da'mon:
The Coefficients of the expression 8 - 5x - 4 + 3x are 5 and 3 Whether the statement is true or false.
Answered by Penny Nom.
Three angles and one side of a triangle 2009-11-16
From Esther:
How do i find the sides of an acute triangle if i know the angels are 60,45,75 and i only know one side which is 10? Thanks!
Answered by Penny Nom.
Maximize profit 2009-11-14
From Willie:
Profit is the difference between Total Revenue and Total Cost. Therefore, to MAXIMIZE PROFIT you must maximize Total Revenue. True or False? Explain answer.
Answered by Penny Nom.
A rate of 67 per 1000 2009-11-10
From ANTHNONY:
if the rate is 67 per 1000 the fraction is 67/1000 how do I present the expanded form
Answered by Penny Nom.
A minute hand 2009-11-05
From Pardha:
A minute hand of table clock is 3cms long. How far its tip move in 20 minutes
Answered by Penny Nom.
At what rate are the people moving apart? 2009-11-01
From saira:
A man starts walking north at 4 ft/s from a point P. 5 minutes later a woman starts walking south at 5 ft/s from a point 500 ft due east of P. At what rate are the people moving apart 15 minute after the woman starts walking ?
Answered by Harley Weston.
Where is the Father? 2009-10-30
From Jeffrey:
A mother is 21 years older than her kid and within 6 years the child will be 5 times younger than the mother. Where is the Father? Jeffrey
Answered by Robert Dawson and Claude Tardif.
Factors 2009-10-30
From brenden:
The numbers 3,5,7 are factors of n. Find four other factors of n besides 1
Answered by Penny Nom.
Painting a dome 2009-10-30
From Jessica:
A hemispherical dome with a radius of 50 ft will be given a coat of paint .01 inch thick. The Contractor for the job wants to estimate the number of gallons of paint needed. Use a differential to obtain an estimate (231 cubic inches/gallon) HINT: Approximate the change in volume of hemisphere corresponding to increase of .01 inch in the radius.
Answered by Robert Dawson.
An array 2009-10-28
From felicia:
what factors are shown in the array below?
**
**
**
**
**
**

Answered by Penny Nom.
Sarim has $1 in coins. 2009-10-27
From Elizabeth:
Here is the question asked: Sarim has $1 in coins. One-fifth of the coins are dimes, two-fifteenths are nickels, and two-thirds are pennies. Tell how many of each coin he has. I am not sure how to start this problem.
Answered by Penny Nom.
Four consecutive odd integers 2009-10-24
From Tonya:
the sum of four consecutive odd intergers 2008 what are the intergers
Answered by Penny Nom.
A linear equation 2009-10-22
From Ryan:
How do you convert the equation 2x-6y=7? This is on my homework and I am really stuck!!!
Answered by Penny Nom.
The inverse of f(x)=-1/x^2 2009-10-22
From Jasmine:
how to solve f(x)=-1/x^2 --> find f-1[x], than state if its a function
Answered by Harley Weston.
A linear system 2009-10-20
From marissa:
Solve this linear system
2x-y=5
3x+y=-9

Answered by Penny Nom.
How much fabric will I need? 2009-10-20
From Coach Jones:
I have a new ideal and need to know how much fabric will I need to order.

My design is 16'6" long, 10' wide and 24" tale. How much fabric will I need for the

Top, Bottom and the Walls?

Thanks

Coach Jones

Answered by Penny Nom.
Proof that the root of 27 is irrational 2009-10-18
From Scarlet:
How do you prove that the square root of 27 is irrational?
Answered by Victoria West.
What fraction of my goal has been raised? 2009-10-13
From sally:
If my goal is to raisel $300 and I have only raised $240 what fraction of my goal has been raised
Answered by Penny Nom.
How many gallons does it contain when it is 1/6 full? 2009-10-13
From Bren-Nesha:
A tank contains 200 gallons when it is 4/5 full. How many gallons does it contain when it is 1/6 full? I already know the answer but i would like to know how to arrive at the answer.
Answered by Penny Nom.
Integration of cos (x^2) dx 2009-10-12
From Venkat:
How do you evaluate the integration of cos (x^2) dx ?
Answered by Harley Weston.
A right triangle 2009-10-12
From Josseph:
A right triangle has an area of 84ft sq. and a hypotenuse 25ft long. What are the lengths of the other two sides?
Answered by Penny Nom.
Golf for 8 2009-10-10
From Chris:
I have a group of 8 golfers who will be playing three rounds, What is the best formula so we can all try to play with different people during the 3 rounds .
Answered by Victoria West.
24 feet of pipe in 3 pieces 2009-10-09
From Sam:
I had a carpenter being a smarty pants with me. Question: I have a 20ft piece of pipe. I need three equal lengths cut. What is the measurement of each cut? He told me the answer right away because he knows the measuring tape like the back of his hand. The answer is 6'8". Is there an equation I can use to get the answer quicker than him looking at his tape measure?
Answered by Penny Nom.
24 golfers, 6 days 2009-10-03
From patrick:
i need a pairing schedule for 24 golfers for six days playing in foursomes. Please
Answered by Victoria West.
Fill required for a driveway 2009-10-03
From Rose:
We are going to be pouring a driveway and it is needing a lot of fill. I can't seem to figure this out. Measurements are 30 ft x 30 ft and it goes from 2' deep to 4" (which is what I need overall as it will be 4" of concrete for the driveway. Please HELP Rose in GA.
Answered by Penny Nom.
Game of 24 2009-10-02
From trami:
game of 24 by using -4 -7 -7 1?
Answered by Claude Tardif.
Prove by induction 2009-10-02
From Anonymous:
How can you prove the following by induction:

Any fraction (A / B), where 0 < (A / B) < 1, can be expressed as a finite sum
(1 / c(1)) + (1 / c(2)) + (1 / c(3)) + ... + (1 / c(k)),
where c(1), c(2), ..., c(k) are natural numbers greater than 0.

[ex. (20 / 99) = (1 / 9) + (1 / 11)]

Answered by Claude Tardif.
24 golfers 2009-10-01
From Peter:
I am working on a 19 week golf schedule with 24 golfers, and I would like to know how to make up six foursomes each week without having a player paired with the same golfers.
Answered by Victoria West.
-4 squared and (-4) squared 2009-09-29
From Andrea:
Is -4 squared the same as (-4) squared? I am thinking the first is -16 and the second is +16. I am trying to clarify for my students.
Answered by Penny Nom.
Differentiating y= square root(x-1) 2009-09-29
From edith:
describe the x-values at which f is differentiable.
y= square root(x-1)

Answered by Penny Nom.
Sawdust 2009-09-29
From joel:
What is the density of saw dust
Answered by Harley Weston.
Comparing fractions 2009-09-28
From TeeTee:
I am in 6th grade. I have to order these fractions from least to greatest: 3/4, 2/5, 5/8, 1/2.....How do I order them if I dont know where to start, and how would I find these answers...Please Help! :)
Answered by Penny Nom.
Evaluating -x^2 + (yz - 3)^2 2009-09-28
From Kathy:
My daughter and I are having trouble solving this equation:

-x²+ (yz – 3)²

if x=-4,y=2 and z=0 We are having difficulty with the -x² part of the equation. Thanks

Answered by Penny Nom.
How many terms are there in this sequence? 2009-09-28
From tabby:
How many terms are there in this sequence?
5,1,-3,...,-111

Answered by Penny Nom.
Constructing a scale model 2009-09-27
From Colleen:
What is the formula i need to use to find out how to state a reasonable scale in the question below; Suppose you were constructing a model that must sit on a piece of cardboard that is 22cm by 28cm. STATE THE REASONABLE SCALE that you can use to build a model to represent the following items. Determine the dimensions of the model.

a) a rectangular prism with a length of 5m and a width of 2m?

Answered by Penny Nom.
Probability 2009-09-27
From Ed:
My mother died 3 years to the day after her daughter died. what are the odds of that happening by chance? thanks
Answered by Chris Fisher.
Game of 24 2009-09-24
From vanessa:
Using the following numbers 13,6,6, and 12 to equal 24. Using 5th grade math
Answered by Penny Nom and Claude Tardif.
Order of operations 2009-09-24
From aman:
my question is regarding the fact i have a formula which looks like this :

(ratio 1 + ratio 2 divided by 2)

so my question is do i add first then divide by 2 or do it all together

Answered by Penny Nom.
5 x 8 + 6 divided 6 - 12 x 2 2009-09-24
From Susan:
5 x 8 + 6 divided 6 - 12 x 2. I am not sure of the rules of operation for this type of question
Answered by Penny Nom.
24 golfers: 4 days 2009-09-21
From Steven:
We have 24 golfers (6 A's, 6 B's, 6 C's & 6 D's) playing 4 days. Each day a foursome consist of an A, a B, a C & a D player. Can you give me a pairing list so that no two golfers will play on the same foursome for the 4 days?
Answered by Chris Fisher and Victoria West.
What is my number? 2009-09-18
From Hanna:
What is my number?
My number is a perfect square.
The only number in its prime factorization is 2.
My number is a factor of 32.
The sum of its digits is odd.

Answered by Penny Nom.
Repeating decimal to fraction 2009-09-18
From joan:
how can i answer it easily to convert terminating decimal to fraction? example of this is that, convert ...143143143... to fractional form
Answered by Robert Dawson.
Exponential form 2009-09-18
From karen:
Q: Write the following in exponent from: a * a * a * a * * * a, where there are n -1 a's

note: *asterisk means times.

Answered by Penny Nom.
Two equations in two unknowns 2009-09-18
From Citizen:
x+-3y=7
-x+4y=7

Answered by Penny Nom.
The method of substitution 2009-09-17
From laura:
ok! i really need help with this question plz help!!

x= - 4y +5
x+2y = 7

Answered by Penny Nom.
Surface area of a dome 2009-09-16
From Charlie:
Why is the calculation for the surface area of a domed tank less than the surface area of a flat top tank. Using domed tank formula 2*pi*r(r-(r-dome ht) works out to be less surface area than a flat top tank area formula pi*r x r, Shouldn't the dome have more surface area than the flat surface ?
Answered by Harley Weston.
at+b=ar-c, solve for a 2009-09-16
From kimberly:
at+b=ar-c, solve for a
Answered by Penny Nom.
10x-8 /3 =6x+24 /5 2009-09-16
From Heidi:
10x-8 /3 =6x+24 /5.....I know the answer is 7/2 but how do I get there?
Answered by Penny Nom.
Ordering fractions 2009-09-15
From stacy:
I need help with listing fractions from the least to greatest. last problem of math is 5/6,7/9,23/27,13/18?
Answered by Penny Nom.
A trillion grains of rice 2009-09-14
From akaila:
can 1trillion rice grains fit in a classroom with the area of 144.4m3
Answered by Penny Nom.
Friends in a class 2009-09-14
From pran:
it is known that among any group of three students in a class two of them are friends. the total number of students is 25. prove that there is a student who has at least 12 friends
Answered by Claude Tardif.
The slope-intercept form 2009-09-11
From Shelby:
Write an equation in slope-intercept form of the line that is parallel to the graph of 3y-4x=1 and passes through (0,6)
Answered by Penny Nom.
Exponential form 2009-09-08
From hanna:
hi, well i am really bad in math and i have a math paper and i was just wondering what 3x3x3x3x3 was in exponential form and what is exponential in a way a person like me would understand?
Answered by Penny Nom.
f(x + h)-f(x) 2009-09-07
From Marie:
For each function defined as follows, find a.) f (x+h) b. f(x+h)-f(x) c. [f(x+h)- f(x)]/h.

Problem: f(x)= -1/ x^2.

Answered by Harley Weston.
10 golfers 2009-09-06
From Bill:
We have 10 golfers scheduled to play over five days on vacation in November. Each day we will play in three groups of three and one foursome. Is there an optimum solution that would assure that every golfer plays with every other golfer at least once while no golfer plays an inordinate number of times with any other golfer or too often in a foursome?
Answered by Victoria West.
Divide and reduce to lowest terms. 2009-09-06
From Karen:
Divide and reduce to lowest terms. Use the cancellation technique as needed 4 2/3 divided by 12 14/3 divided by 12/1 not sure wheer to go from here with the cancellation technique
Answered by Penny Nom.
A four-digit positive integer 2009-09-04
From TRACEY:
I AM A FOUR-DIGIT POSITIVE INTEGER SUCH THAT THE SUM OF MY DIGITS IS 18 AND MY DIGITS REVERSED ARE EXACTLY FOUR TIMES GREATER THAN MYSELF. WHAT NUMBER AM I?
Answered by Robert Dawson.
Simplify 2009-09-04
From Sarah:
How do you simplify: (x-4)^3?
Answered by Robert Dawson.
What is 8 tons in exponential form? 2009-09-01
From Tyler:
what is 8 tons in exponential form
Answered by Penny Nom.
Cardinality of infinite sets 2009-09-01
From Brian:
I was reading an answer to a question on your site regarding infinite sets (http://mathcentral.uregina.ca/QQ/database/QQ.09.01/carlos1.html), and I think they may have got the answer wrong.

I his example, he claims that the set of real numbers BETWEEN 0 AND 1 is larger than the set of positive integers.

Please correct me if I am wrong, but I believe those two sets are -- pardon the expression -- equally infinite. For any integer, there is a corresponding real number between 0 and 1, and vice versa.

For instance, using the decimal as a "mirror", you can create a mirror image of any real number between 0 and 1 as an integer (i.e. 0.1234 gets mirrored as the integer 4321 -- I could write it out algebraically, if you want, but you get my point)

Am I wrong?

Thanks, Brian

Answered by Victoria West.
I have 9 players 4 rounds of 3 2009-08-31
From Larry:
I have 9 players 4 rounds of 3. I need each player to play one timw with everyone in the group
Answered by Victoria West.
Exponential form 2009-08-31
From cecil:
what is the exponent form 564000?
Answered by Stephen La Rocque and Harley Weston.
Simultaneous equations 2009-08-28
From onias:
solve 3/a - 2/b = 1/2 , 5/a + 3/b = 29/12
Answered by Robert Dawson.
y=2x+1 and y=2x-1 2009-08-28
From MARICELA:
Need help of how to work this problems

y=2x+1 and y=2x-1

First of all what is the difference?

Second I think is easy just to work with the X y with the line in the middle

but how do you get the numbers for each side.

Answered by Penny Nom.
Chickens and pigs 2009-08-28
From David:
A farmer has a number of chickens and pigs. There are 24 heads and 80 ft. How many pigs and how many chickens are there?
Answered by Robert Dawson.
The game of 24 2009-08-27
From ANITA:
Help I'm suppose to be a math expert, yet I can not yield 24 from the numbers 8,4,9,9, using each only once. HELP
Answered by Penny Nom.
The game of 24 2009-08-26
From melvi:
how can you get the numbner 24 but only useing the numbers 7,5,4,5
Answered by Penny Nom.
Fact families 2009-08-24
From Camasha:
I am trying to help my daughter with fact families. I understand the premise from your previous explanation on the site . Generally when applying addition and subtraction their are two add and two subtraction utilizing all three numbers. However what do you do when you have a series like like 9, 9, 18
Answered by Penny Nom.
Factoring 2009-08-23
From Bj:
Solve for x: 5x^2 - 35x = 0
Answered by Stephen La Rocque.
Graph 2x-3y+15=0 2009-08-20
From Diem:
Graph 2x-3y+15=0
Answered by Leeanne Boehm.
Comparing Fractions 2009-08-20
From smilegrlal:
please order these number in least to greatest 7/8, 5/9, 2/3
Answered by Leeanne Boehm.
A 5 foot strip around a building 2009-08-20
From Robert:
I am having a problem at work that I am hoping someone can help me figure out. I have a pest control company and we are pretreating the soil for a new hospital being built. The company that we are working has asked to also treat a 5 foot strip around the entire building and for us to get paid we need to figure out what the square footage of this 5 foot strip around the building that we are treating. The structure being built is a hospital and as you can imagine the building has all kinds of nooks and offsets and even some of the wall are cruved more like a circle. I already know the square footage of the building itself because they gave us this information. The square footage of the building is 120,000 SqFt. I would greatly appreciate if someone can help me figure out the square footage of a 5 foot strip around a 120,000 SqFt building who's walls have a lot of insets and odd shapes. I don't if someone can figure out the square footage of the 5 foot strip with the information I gave. I would greatly appreciate any help that you can give me. I need this information for work because we charge for pretreatment of soil by the square footage of the area to be treated and I need this information so that I don't over charge the company we are working for or so I don't cheat myself either. Any and all help we be greatly appreciated because I don't know who else that can help me.
Answered by Victoria West.
Fourth and fifth degree polynomials 2009-08-20
From Evin:
hello.i am a student . ax^4+bx^3+cx^2+dx+e=0 x=? i want to learn the solution or formula of equations of the fourth and fifth degree...PLEASE
Answered by Robert Dawson.
Golf for 15 2009-08-18
From Judy:
We have a group of 15 golfers that play a scramble with groups of 3. The problem is we play for 14 weeks. We want every player to play at least once with everyone and no more that 3 times. I am working on this and have just about accomplished the schedule, but with a few playing more than 3 times. I'm sure there must be some mathematical calculation to do this but I have yet to figure it out.
Answered by Victoria West.
Sequence problem 2009-08-18
From James:
I have the equation: tn = (n)(n+1)/2 Which gives the sequence: 1, 3, 6, 10, 15 etc. However, I'd like to be able to solve for n. So that a t of 6 would give me the answer of 3, and t of 10 would give me the answer 4 etc.
Answered by Stephen La Rocque and Victoria West.
How do i convert 6 feet into yards? 2009-08-14
From taylor:
how do i convert 6 feet into yards?????????????????
Answered by Robert Dawson.
A football schedule 2009-08-14
From Bill:
I am having trouble with a football schedule that has two divisions of five teams each, 1-5 and 6-10. 1-5 will play against each other in the first 4 weeks and also the the last 4 weeks with teams 6-10 completing the middle of the schedule. The schedule is 13 weeks. Thank you for your time
Answered by Chris Fisher and Victoria West.
An antiderivative problem 2009-08-13
From Indrajit:
∫4e^x + 6e^-x/(9e^x + 4e^-x)dx = Ax + Bloge(9e2x - 4) + C

then A=?......B=?.....C=?

plz solve it...."^" stands for "to the power of"....

Answered by Harley Weston.
Fitting the curve y=a*exp(b*x)+c 2009-08-12
From aika:
Could one show me the complete algorithm and formula for finding the coefficients (a,b, and c) in exponential regression model y=a*exp(b*x)+c
Answered by Robert Dawson.
The leaning tower of Pisa 2009-08-09
From MF:
Would you have any idea how the 'latitude of 44 degrees N" has anything to do with this question and how I would apply it?

The leaning tower of Pisa leans toward the south at an angle of 5.5 degrees. One day near noon its shadow was measured to be 84.02 m long and the angle of elevation from the tip of the shadow to the top of the tower was measured as 32.0 degrees. To answer the question, assume that the tower is like a pole stuck in the ground, it has negligible width. Also, it is important to know that Pisa Italy is at a latitude of approx 44 degrees North because this affects the direction of the shadow.)

Answered by Stephen La Rocque.
Eye Height Level Found from Visual Angle 2009-08-07
From Jolie:
Hello, I am trying to figure out what height from the ground a person's eyes are if they have a 12 degree visual angle to a screen that is 58cm away from their eyes? Thank you.
Answered by Janice Cotcher.
An infinite set 2009-08-07
From Islam:
How can I prove that the set of all odd natural numbers is an infinite set. Thank you.
Answered by Robert Dawson.
Exponential form 2009-08-05
From motaz:
Let x = log_2 1/8 Write the exponential form of the equation and solve the equation for x
Answered by Stephen La Rocque.
Susan's RRSP 2009-08-05
From Polly:
Susan contributed $500 every 6 months for 14 years into a RRSP earning interest @ 7.5% compounded semi-annually. Seven years after the last contribution Susan converted the RRSP into a RRIF which is to pay her equal 1/4 payments for 16 years. If the first payment is due 3 months after the conversion into the RRIF and the interest on the RRIF is 9% compounded 1/4, how much will Susan receive every 3 months?
Answered by Stephen La Rocque.
A spherical dome 2009-08-03
From Eric:
I have recently been asked to resurface a dome sculpture for my local council but i'm having problems working out the area. Here are the dimensions. The height of the dome is 3m from the ground to the top of the arc. The arc itself from the ground rising up to the 3m point and back down, is 10m. The dome is 7m wide from one side to the other through the centre, at ground level.
I hope there is enough detail here. It's been a long time since i was in a maths class. Thank you, hope to here from you soon!

Answered by Stephen La Rocque.
HCF 2009-08-03
From Nazrul:
If (x+ a) be the H,C.F. of x^2+px+q and x^2+mx+n, prove that (p-m)a=q-n
Answered by Stephen La Rocque.
Math in everyday life 2009-08-03
From Naveen:
Dear sir, We are advised to do a project on "Mathematical modeling to solve various problems of our everyday life/environmental related problems...... Can u plz help us by mailng some ideas, suggestion,reference to make my project successful.... Thanking you...... Waiting for your favourable reply......
Answered by Penny Nom.
A walkway around a flower garden 2009-08-01
From Barbara:
A rectangular flower bed measuring =C2=A0 10 feet by 15 feet =C2=A0 has a surrounding walkway on all sides that measure 186 square feet. How wide is the walkway?
Answered by Harley Weston.
Fencing five acres 2009-07-31
From Aaron:
Ok So I need to order fencing for my property I have 5 acres and would like to fence in the perimeter, I cant for the life of me figure out how many feet of fencing to order for help please im an idiot.
Answered by Robert Dawson.
Dimensions 2009-07-29
From Deborah:
The blueprint for landscaping a yard has a scale of 1/2" to 1 foot. If the blueprint is a rectangle 18 inches by 22 inches, what are the dimensions of the yard?
Answered by Robert Dawson.
Torricelli's trumpet 2009-07-29
From Gary:
I was reading about torricelli's trumpet which is described by the equation1/x which is then rotated around the x axis which results in a figure which looks like a trumpet. Now in order to find the volume the integral 1/x^2 dx is used which diverges when integrated so the volume is finite.However if you integrate 1/x dx which is the formula on the plane the answer diverges. Now if you took an infinite area then rotated it around the x axis shouldn't you get an infinite volume? Notice the area I am talking about is under the line 1/x not the surface area of the trumpet which is what the painters paradox is about What am I missing? Thanks
Answered by Robert Dawson.
Highest Common Factor of Two Polynomials 2009-07-28
From Nazrul:
If x+a be the h.c.f. of x^2+px+q and x^2+mx+n, how can I prove that (p-m)a=q-n.
Answered by Robert J. Dawson & Janice Cotcher.
Inequalities Proof 2009-07-24
From ABOU:
good morning.......a b c are real positive no zero......proof that sq root(2a/(a+b))+sq root(2b/(b+c))+sq root(2c/(c+a))inferior or equal 3 thank you
Answered by Janice Cotcher.
Properties of Natural Numbers 2009-07-24
From nazrul:
If m,n,k are natural number how can I prove that (m+n)k=mk+nk. In the proof the properties of natural number should be used.
Answered by Janice Cotcher.
Proof of a Unique Solution 2009-07-24
From muele:
Find matrix A such that A is not invertible, and b such that Ax=b has a unique solution
Answered by Robert J. Dawson.
A degreaser for a fish tank 2009-07-23
From charlie:
I have a tank that is 10 feet in diameter that I want to put floating degreaser in.The rate for the degreaser is 1/2 gallon per square foot of surface water.How do I figure out how many gallons I need?
Answered by Penny Nom.
A father and son problem 2009-07-23
From yomi:
five years ago, a father was 3 times as old as is son, now their combines age's amount 110 years. what is the present age of the father?
Answered by Penny Nom.
Golfing with an unpopular golfer 2009-07-23
From Ian:
"We are a group of 8 golfers, one of whom is unpopular. How can we construct a schedule, of two foursomes, so that each person is scheduled to play with him the same number of times. What does the week by week schedule look like?"
Answered by Robert Dawson and Victoria West.
Selling fish 2009-07-22
From Hemantee:
A fishmonger bought 150kg of fish. he sold 70%of it at the price of Rs90 per kg and the rest at Rs 75 per kg. he made a profit of 42.5 per cent. how much did he pay for the fish.
Answered by Penny Nom.
Area under curve 2009-07-21
From Morag:
What is the area bounded by the curve y=X^2(2-X) and the x axis between x=0 and x=2?
Answered by Janice Cotcher.
A golf league with 12 players 2009-07-17
From Jane:
My weekly golf league has 12 players in 4 threesomes. How many weeks would it take to play everyone once and not have too many duplications? What would be the schedule each week? Thanks
Answered by Victoria West.
Portion of Work 2009-07-17
From Ellen:
A girl can shampoo the dog, clean his ears, and clip his nails in 45 minutes. Her sister can do it in 60 minutes. How long will the job take it they work together?
Answered by Stephen La Rocque.
Profit as a percentage 2009-07-15
From Jay:
If I buy an item for $3.00 and sell it for $6.00. What is my percentage of profit?
Answered by Penny Nom.
Filling a hole 2009-07-14
From BARRY:
I HAVE AN AREA THAT i NEED TO FILL WITH DIRT AND AM TRYING TO FIGURE OUT THE CU. YDS NEEDED. tHEY CIRCUMFRANCE OF THE HOLE IS 28' ROUND AND SLOPES FROM 2.5 FEET AT ONE END TO 0 ' AT THE OTHER. OVERALL THE HOLE IS 30' LONG BY 30' WIDE . THE SLOPE RANGES FROM 2.5 ' TO APPROX 4"
Answered by Harley Weston.
The surface area of a rectangular pyramid 2009-07-13
From Shivani:
What is the surface area of a rectangular pyramid with a length of 6 inches, a width of 7 inches and a SLANT height of 4 inches.
Answered by Penny Nom.
Fractional Exponents 2009-07-07
From bob:
2^5/2 - 2^3/2
Answered by Janice Cotcher.
Using the Remainder Theorem to Find an Unknown Coefficient. 2009-07-06
From Mukulu:
What is the value of a if 2x2-x-6 ,3x2-8x+4 and ax3-10x-4 have a common factor.
Answered by Janice Cotcher.
Vector spaces 2009-07-05
From Nazrul:
How can I prove that m(a+b)=ma+mb where a and b are two vectors. Thank you.
Answered by Harley Weston.
A roll of paper 2009-07-05
From mark:
is there a simple way of finding out how much is left on a roll of paper. i have read similiar questions and answers on here but all seem very complicated and not being very good at maths does not help me much. some are in inches and all have different figures to mine so if i give my figures hopefully i wil understand it better. the radius of the cardboard core is 52.25mm, the radius of the paper at 2000 linear meters is 158.625mm. the thickness is 0.17mm. when nearing the end of the paper how do i work out how much is left in a simple way. thank you in advance for any help you can give me
Answered by Harley Weston.
The extended real number system 2009-06-30
From Justin:
Hi again, thanks a lot for answering my previous question! I was also just wondering again if the extended real number system has a potential or actual infinity because it includes positive infinity as a point that exists at the end of it?

All the Best,

Justin

Answered by Robert Dawson.
The surface area of a fire pit 2009-06-30
From eric:
whatw is the square footage of the inside wall of a fire pit 2 feet deep and five feet in diameter? I need to estimate both the floor and wall square footage to order bricks.
Answered by Robert Dawson and Harley Weston.
Potential infinity and actual infinity 2009-06-29
From Justin:
Hi there, I was just wondering what is the difference between the potential infinity and actual infinity in math? Thanks a lot for your help with this question!

All the Best,

Justin

Answered by Robert Dawson.
Integers and decimals in day to day life 2009-06-29
From mitichie:
can you tell use of integers and decimals in our day to day life.
Answered by Penny Nom.
Coffee scoops per cup 2009-06-29
From charity:
if my coffee machine takes 10 scoops for every six cups, how many scoops do i need for 4 cups. each scoops equals one tbls(.25 oz)
Answered by Penny Nom.
Designing a shelf 2009-06-28
From Bjorn:
I'm putting up a shelf and thought I'd be nice and cut the sharp and pointy corners off. I want to the cut to be at a 45 degree angle, but I also want the exposed edge to be the same length as from the wall to the beginning of the cut -- so the end of shelf will look like the first two sides of an octagon. I've managed to construct the solution, but I haven't been able to calculate it...
Please help!

Answered by Stephen La Rocque.
Eight golfers 2009-06-26
From E:
Hello,
We have a golf tournament. 8 players, 3 days, 2 fourballs per day. How can each player play with each other no more than twice, and at least once? Would like a speedy answer as we're going tomorrow!
Thank you

Answered by Laura Morrison.
Height of a Flag Pole 2009-06-26
From Jake:
Can anyone answer this I am very stumped. The flagpole on the roof of a Chocolate Factory is 13 cm in diameter and a perfect cylinder. If one end of a 2300-cm-long string is attached to the top of the flagpole, and wraps around the flagpole exactly 16 times before ending at the bottom of the flagpole, how tall is the flagpole? Need answer by Wednesday
Answered by Penny Nom.
Twelve golfers 2009-06-26
From Fred:
Hello- There are twelve golfers in 3 foursomes for three days. Is there a combination of four man foursomes that allow for no duplicates? In other words, is there a formula to insure the greatest amount of variety in the foursomes. Looking at it math like you have 1,2,3,4,5,6,7,8,9,10,11,12 grouped into four numbers each for three days. So if your have the first day 1,2,3,4 in one group, 5,6,7,8 in the second and 9,10,11,12 in the third what can you do for the next two days to have the fewest duplicates or have the greatest variety in the foursomes? Thanks
Answered by Karen Meagher and Victoria West.
Infinite-Dimensional Spaces 2009-06-26
From Justin:
Hello again, I was also just wondering (in Hilbert Space and Function Space) are there infinite-dimensional spaces larger than each other in terms of cardinality? Thanks a lot for your help again! All the Best, Justin
Answered by Victoria West.
Finding Force When Pressure and Area Are Known 2009-06-25
From chris:
i need to calculate the force for a tube. given the follwing data. pressure is 900 PSI Tube is 6 m long by 1.5m Outside Diameter
Answered by Janice Cotcher.
The surface area of a tank 2009-06-25
From Charles:
I need to calculate the external wall and top sqft surface area of a tank 54ft od x 15ft high with a dome top 54ft od x 8ft high.
Answered by Penny Nom.
Dividing infinity by infinity 2009-06-24
From Justin:
Hello again, I just had one other question nagging question about infinity. I read this article on "Types of Infinity" on Paul Hawkins calculus website and he stated that one infinity cannot be divided by another or that the answer is inderterminate because fundamentally infinity comes in different sizes with respect to infinite sets and that this applies also to calculus. And so I was wondering (if this is true) is this why when you divide infinity by infinity (in the extended real number system) the answer is indeterminate since fundamentally one inifnity is larger than another like in infinite sets or is there another reason? Thanks sooo much for answering my question again! I greatly appreciate it!

All the Best, Justin

Answered by Robert Dawson.
The Pythagorean theorem 2009-06-24
From supreet:
What are some real-world applications of the Pythagorean theorem?
and
How are the Pythagorean theorem and the distance formula related?

Answered by Harley Weston.
Is one Infinity larger than another in the extended real number system? 2009-06-24
From Justin:
Hello there, I was wondering if one infinity is larger than another in the extended real number system (just like in the transfinite ordinals and cardinals with respect to infinite sets) or are all infinities the same size in the extended real number system? Thanks sooo much for answering my question! I greatly appreciate it!

All the Best,

Justin

Answered by Robert Dawson.
Prove that the set of all positive odd integers is an infinite set 2009-06-20
From Nazrul:
How can I prove that the set of all positive odd integers is an infinite set.
Thank you in advance.

Answered by Victoria West.
Maximum Volume of a Cylinder Inscribed in a Sphere 2009-06-18
From Jim:
Hello I have a hard time finishing this question: A right circular cylinder has to be designed to sit inside a sphere of radius 6 meters so that each top and bottom of the cylinder touches the sphere along its complete circular edge. What are the dimensions of the cylinder of max volume and what is the volume?
Answered by Janice Cotcher.
Successive differences 2009-06-18
From Jonathan:
I'm trying to find the next number sequence for this equation: 1 11 35 79 149 251, my problem is that I worked it out and ended up with a single number 17. What am I doing wrong. Thank you for any help.
Answered by Robert Dawson and Penny Nom.
A difference quotient 2009-06-17
From Sue:
When s(x)=x^3+x, compute and simplify the difference quotient s(x+h)-s(x)/h.
Answered by Harley Weston.
Factoring a quadratic 2009-06-16
From Rebecca:
solve the quadratic equation by factoring.
a) x(squared)+7x+10=0

Answered by Stephen La Rocque.
11 golfers playing 4 rounds 2009-06-14
From Brian:
I have 11 golfers playing 4 rounds of golf. It would be great if we could play at least once with everybody. I realize we will have 2 foursomes and 1 threesome each round...can you help?
Answered by Victoria West.
The last non cero digit of a factorial 2009-06-12
From Wilson:
How can I find the last non cero digit from a factorial calculation of a big number. For example 10! = 3628800, the last non cero digit is 8. What is the last non cero digit of 10! ??
Answered by Robert Dawson.
The product of gradients between 2 perpendiculars lines 2009-06-11
From Alister:
how do i prove that the product of gradients between 2 perpendiculars lines equal to -1....
Answered by Penny Nom.
Vectors and the Law of Cosine 2009-06-08
From lauren:
once force of 20 pounds and one force of 15 pounds act on a body at the same point so that the resultant force is 19 pounds. Find, to the nearest degree, the angle between the two original forces
Answered by Janice Cotcher.
Apparent Weight in salt water 2009-06-06
From Rob:
If an object weighs 100kg in air how much will it weigh when suspended in salt water? Is there an easily used conversion calculation?
Answered by Janice Cotcher.
Solving an Algebraeic Equation with Fractions 2009-06-04
From olivia:
solve for x (3x-1)/4 + (x+3)/6 = 3
Answered by Janice Cotcher.
How long are the rafters? 2009-06-03
From Tina:
An architect designs a house that is 12 m wide. The rafters holding up the roof are equal in length and meet at an angle of 70 degrees. The rafters extend 0.3 m beyond the supporting wall. How long are the rafters?
Answered by Penny Nom.
Omega 1 2009-06-03
From Justin:
Hello there, I was just wondering if the infinity in the extended real number system is the same as w1 (or Omega 1, the order structure of the real numbers) in the transfinite ordinals? Thanks so much for your help with this question, I really appreciate it!

Sincerely,

Justin

Answered by Robert Dawson and Harley Weston.
how do I factor 45 - 5x^2 2009-05-31
From tracey:
how do I factor 45 - 5x^2
Answered by Penny Nom.
Profit margin 2009-05-26
From andreya:
what is the profit margin of a business that spends $200 in order to make $1000
Answered by Penny Nom.
Factor 6x^2+19x+3 2009-05-26
From michele:
how do you factor this trinomial. 6x^2+19x+3
Answered by Penny Nom.
Find out the length of a cup when its volume is halved 2009-05-25
From Thomas:
I'm having trouble with a question. What kind of formula would i use to find out the length of a cup when its volume is halved?
Answered by Stephen La Rocque.
The quotient of 5/8 divided by7/4 2009-05-25
From tammy:
find the quotient of 5/8 divided by7/4
Answered by Stephen La Rocque.
The position of the fulcrum 2009-05-23
From jim:
I think I need a formula. I need to know how far an object will be lifted. A beam is 246 inches long on one side of the fulcrum, and 41 inches on the other side, if I push down 36 inches on the long side of the beam, how much will the short side move up?
Answered by Stephen La Rocque.
The dimensions of a larger cup 2009-05-23
From Elizabeth:
Hey If i have a cup that holds a volume of 477mL and the bottom radius is 2.8cm and the top radius is 4.9cm and the height is 10cm. If i increase the volume by one and a half times what is the new measurements if the cup is directly proportional to the first one. Thank you
Answered by Stephen La Rocque and Penny Nom.
Game of 24 2009-05-22
From Sam:
Using the numbers 7, 3, 2, and 2, how do you get the number 24?
Answered by Claude Tardif and Harley Weston.
Equivalent fractions 2009-05-22
From Jocelyn:
give two other ratios that are equal to each

example: 6/9

Answered by Penny Nom.
(x + 1) / 3 + (x + 2) / 7 = 2 2009-05-22
From MzDonna:
Question from MzDonna, a parent:

We can't seem to get an answer for this problem:
X + 1      X + 2
_____ + _____   = 2

   3              7

The book gives the answer as 10 ; But, we struck. Please help us.
                                               ___
                                                21       

Thanks in advance for your assistance.
Answered by Stephen La Rocque.

Fraction Word Problem 2009-05-13
From Sonya:
Debbie wants to eat 1/6 of her 12 pieces of candy. How many pieces did she eat ?
Answered by Janice Cotcher.
differentiate y sin[x^2]=x sin[y^2] 2009-05-11
From mamiriri:
derivate y sin[x^2]=x sin[y^2]
Answered by Harley Weston.
Maximum profit 2009-05-11
From Sally:
a manufacturer of dresses charges $90 per dress up to 100 units and the average production cost is $60 per dress. to encourage larger orders the company will drop the price per dress by .10 for orders in excess of 100. I need to find the largest order the company should allow with the special discount to realize maximum profit.
Answered by Harley Weston.
Unit Conversion for Force and Mass 2009-05-08
From Ray:
Good Day! I was confused about lbm and lbf and slug. I knew that lbm is a unit for mass and lbf is a unit for weight. my problem is some books follow this relationship where lbf=(lbm times the gravitational pull) which is 32.2 ft/(s^2) while other books treat lbm=lbf. other books also treat lbm=(lbf times 32.2)... which is correct? Is 1kg=2.205 lbm or 1kg=2.205lbf? Is 1BTU=778.16 lbf-ft or 1BTU=778.16 lbm-ft? what is the relationship of slug on these two, lbm and lbf?
Answered by Janice Cotcher.
Cubic feet and cubic yards 2009-05-08
From TJ:
21 bags at 2 cubic feet per bag. How many yards would that be?
Answered by Penny Nom.
Area and perimeter 2009-05-07
From Kelly:
Why do objects w/ the same exterior linear feet have different interior sq ft? I.e. a 25x25 room has 625 sqft while a 20x30 room has only 600 sqft? I can visualize that by cutting and moving parts of the rectangle, you loose a 5x5 section, but I just can't get my head around why.
Answered by Robert Dawson and Stephen La Rocque.
How many eighths are there in 1.25? 2009-05-06
From Tamara:
How many eighths are there in 1.25?
Answered by Penny Nom.
3h+10 divided by 2 +9=20 2009-05-05
From CJ:
what is the solution for the equation 3h+10 divided by 2 +9=20
Answered by Penny Nom.
A 6 team social softball league 2009-05-02
From Don:
Hi We have a 6 team social softball league with only two fields. Each team plays two games each Saturday. We have three time slots 10:00, 12:00 and 2:00 We play for 11 Saturdays and than have a 2 weekend playoff. All teams prefer double headers vs the split at 10:00and 2:00. How do we schedule as balanced a schedule as possible and minmize the splits ? Thanks Don
Answered by Laura Morrison and Victoria West.
Game of 24 2009-05-02
From Barbara:
using the numbers 1,3 7 and 9 only once how do you get the answer of 24
Answered by Claude Tardif.
Fractions of fractions 2009-05-02
From Sonya:
9/10 of the students came to school today and 8/9 of the students brought their lunches how many students brought their lunches?
Answered by Stephen La Rocque.
The volume of 3-D figures 2009-04-29
From Serena:
What is the formula for finding the volume of 3-D figures
Answered by Robert Dawson.
Eight golfers, four days 2009-04-28
From Billy:
A golf Pairing question:

8 Golfers are Playing golf - one round per day
Four days of golf
(four twosomes) are paired each day to play
***No one can play with the same partner more than once
***Everyone must play with every other golfer in his foursome at least once

Is this possible?

Answered by Robert Dawson.
The integral of a to power x squared 2009-04-28
From JIM:
WHEN I ATTENDED U.OF T. (TORONTO ) MANY YEARS AGO WE WERE TOLD THE FOLLOWING INTEGRAL COULD NOT BE SOLVED : a to power x squared . is this still true ?

CURIOUS , JIM

Answered by Robert Dawson.
Completing the square 2009-04-27
From Daniel:
I tried following an example of yours but I still couldn't figure it out. Here is my question,

2x^2 - 6x + 1 = 0.

Any help would be greatly appreciated.

Answered by Stephen La Rocque.
The callaway system 2009-04-26
From Earl:
Do you have a formula to enter players scores using the callaway system that calculate each player score after their score is entered?
Answered by Victoria West.
A word problem involving y = mx + b 2009-04-24
From Devon:
I need to put the following question in to y=mx+b form

I rent a gym for $150.00 for 30 students. Another time I rent the gym for @270.00 for 70 students. I need to also find a fixed rate.

Answered by Stephen La Rocque.
Negative nine minus 7x = negative 5 2009-04-23
From gigi:
what is negative nine minus 7x = negative 5
Answered by Robert Dawson.
8 golfers in 3 rounds 2009-04-22
From patrick:
We have eight people, playing three rounds of golf. Would like to know the optimal pairings so that the most people can play with everyone. Thanks.
Answered by Robert Dawson.
Miles per hour and feet per second 2009-04-22
From mary:
A car traveling at 60 miles per hour, how many feet per second has it traveled?
Answered by Robert Dawson.
A golf schedule for 18 teams 2009-04-21
From laura:
Could you help me form a golf schedule for 18 teams, playing a total of 8 weeks. Rotating the schedule each week so every team plays with a different team each week? I am using a shotgun format with 9 holes every week. Thank you for your input.
Answered by Victoria West.
The center of an ellipse 2009-04-21
From Nae:
what is the ellipse center of 5x^2+3y^2=15
Answered by Stephen La Rocque.
A perpendicular line in standard form 2009-04-21
From Kristy:
Can you help me with this equation? Find the equation, in standard form of the line perpendicular to 2x-3y=-5 and passing through (3,-2) With the equation in standard form with all integer coefficient.
Answered by Stephen La Rocque.
The sum of the roots of a quartic 2009-04-21
From dave:
This is a algebra problem that i am confused about: The sum of the roots of x^4-x^3+5x^2+4=0 is: i tried graphing it, but it shows that there are no roots, but the answer is 1. are they wrong?
Answered by Penny Nom.
The derivative using limits 2009-04-21
From Kirstin:
I am trying to take the limit of f(x) = [f(x+h)-f(x)] / h If you try taking the limit by substituting the limiting value h=0, you get 0/0, which of course is not the right answer. You rewrite f(x+h)-f(x) so it has a factor of h in it, which you cancel with the h in the denominator before you substitute h=0. But I am not sure how to do this. Thanks.
Answered by Robert Dawson.
12 oz. cup 2009-04-19
From Tom:
I am a ceramic teacher and wanted my students to make a 12 oz. cup, what formula should we use?
Answered by Chris Fisher.
Factor x squared(x-3)+(x-3) 2009-04-19
From Victor:
How do I factor x squared(x-3)+(x-3) ?
Answered by Harley Weston.
28 golfers 2009-04-18
From DON:
HI, I HAVE 28 GOLFERS AND I NEED A SCHEDULE FOR 24 DIFFERENT DAYS THERE WILL BE 7 GROUPS OF 4 PLAYERS EACH DAY. ALL PLAYERS WOULD LIKE TO PLAY WITH ONE ANOTHER AS EVEN AS POSSIBLE (SAY 2 TO 4 TIMES EACH OVER THE 24 DAYS)...THANKS DON
Answered by Victoria West.
Four-digit combinations 2009-04-18
From Carri:
How many 4-digit combinations can be made out of digits 1, 2, 3, 4, & 5 using each number once? What are the combinations?
Answered by Penny Nom.
Make x the subject in this equation 2009-04-16
From victoria:
hi,
i need help trying to make x the subject in this equation:
7y + 2x = 15
i'm really stuck so any help will be good,

thanks

Answered by Penny Nom.
Exponential form 2009-04-16
From Pete:
Hi, How do you express ³√h^-4 in exponential form. I am having a lot of trouble with this one.
thanks
Pete

Answered by Stephen La Rocque.
Change 5/6 into a percent 2009-04-15
From Shelly:
My question is when it says change 5/6 into a percent
Answered by Penny Nom.
Infinity and Aleph-Null 2009-04-14
From Justin:
Yes, I am reading the Paul Halmos book on Set theory, thanks for telling me how to get it! I was just wondering from your last answer though if the positive real infinity of calculus then corresponds to Aleph-null? I am sorry if this is a similar question to the one I asked before but I was just wondering about this!

All the Best,

Justin

Answered by Robert Dawson.
A four-sided lot 2009-04-12
From Robert:
back of property is 137' across,with an angle of 122deg.on the left side and an angle of 140 deg. on the right side,the left side is 123' and the right side is 93' long,the left front is 130 deg. and the right front side is 150 deg. and the front of the property measures 78'
Answered by Harley Weston.
Four-digit numbers 2009-04-11
From jp:
How many four-digit numbers are there?
Answered by Penny Nom.
2sinB=3tanA 2009-04-10
From Xanathax:
ABC is a right-angled triangle. 2sinB=3tanA. Calculate the measure of angle A.
Answered by Penny Nom.
Winding paper after a break 2009-04-10
From Olen:
Question from Olen:

I work in a paper mill and have been handed the task to search for a formula to determine how much paper needs to be added to a parent roll to make up the difference at the winder. (Ex. The spool diameter at the reel is 18.25" we measure roughly 33.5" to make two 58" rolls in the winder. If the is a paper break and the roll diameter in the winder is 30" how much do I add to a single parent roll (22" roughly) to make one 58 " and the 28" needed at the winder. I would appreciate any help to complete this task. I would like to be able to build a chart that operators can refer to based on what is needed. Thank you.

Answered by Harley Weston.
The axiom of choice and constructibe sets 2009-04-10
From sydney:
The axiom of choice asserts the existence of certain sets, but does not construct the set. What does "construct" mean here? For example, does it require showing the existence and uniqueness of some function yielding the set? In general, what does it mean to require the existence of a mathematical object be tied to a construction of it?
Answered by Claude Tardif.
Positive real infinity and Aleph-null 2009-04-09
From Justin:
Hello, I was just wondering why does the positive real infinity correspond to Aleph-null? Thanks a lot for answering my question!

Justin

Answered by Ami and Robert Dawson.
Game of 24 2009-04-08
From Joe:
How do you get 24 using 6,9,10,10 once. Its called the 24 game.
Joe

Answered by Harley Weston.
Constructing a cube 2009-04-08
From Jill:
you have a sheet of cardboard that measures 9ft x 6ft. To make the entire sheet into a closed box tht is a perfect cube, what would be the surface area of the box?
Answered by Robert Dawson.
The integral of the square root of the sine function 2009-04-07
From Indrajit:
how to integrate this derivative???

∫√sinx

Answered by Harley Weston.
Actual Food Cost is 3% or more above Projected Food Cost 2009-04-07
From Kam:
Your store guidelines states that when Actual Food Cost is 3% or more above Projected Food Cost, immediate corrective action be taken by the Unit Manager. In January, your Actual Food Cost was $11,000, and your Projected Food Cost was $10,850. To find out what percentage the difference is do I divide the difference ($150) by Actual Food Cost ($11,000) or Projected Food Cost ($10,8500)? What would be the answer? Thanks
Answered by Harley Weston.
Multiplication of polynomials 2009-04-07
From Carla:
I am struggling to understand Multiplication of Polynomials. No matter how hard I try to understand Multiplication of Polynomials, I just can't get it!

The problem that I am trying to solve is this :

-3x^3y(-y + 2 -x^2 + x)

Answered by Robert Dawson.
A golf trip for eight 2009-04-05
From Stuart:
8 of us are off on a golf trip later this month. We are playing 4 rounds in fourball format. Can we put together groups of four so that we play with each other at least twice? I have spent ages on this but can't get it to fit! Your help would be appreciated
Answered by Chris Fisher.
Game of 24 2009-04-05
From Ramona:
math 24 using 16, 9, 20, 7
Answered by Penny Nom.
Pounds per square foot 2009-04-04
From kevin:
Ineed to know the pds per sq ft of a machine that weighs 1,550 pds and measures 78.5 inches long x 32 inches wide.
Answered by Harley Weston.
A fraction in its simplest form 2009-04-02
From Michael:
I'm in 4th grade and need to express decimals as a fraction in its simplest form. Is there a step by step method to figure out?

ex 0.64 = 64/100 = ?

Answered by Robert Dawson.
Fractional part 2009-04-01
From Galina:
What fractional part of eight is eight hundredths
Answered by Stephen La Rocque.
A five game average of 148 2009-04-01
From Shelly:
(128 + 145 + 139 + 157 + x) divided by 5 = 148 matches the situation stated below:

A person bowled four games and scored 128, 145, 139, and 157. Find x, the score the person would need to have in the fifth game to average 148 for all five games.

How do I solve this and how am I supposed to add x like it says in the equation?

Answered by Penny Nom.
Why learn math? 2009-04-01
From Uno:
I got myself in a lot of trouble today. I got into an argument in school with my math teacher because while learning geometry I said that this was useless.
I don't understand why I need to learn algebra, geometry & trigonometry. I don't see how we use this in real life and it is almost like my teachers don't know either. They say I have to learn it because... I don't think that is a good enough answer. The only way I don't get suspended is if I can come up with real world applications of why we learn math.

I need help... I am already in deep trouble with my parents. Any resources on how learning a proof is used in real life?

Answered by Claude Tardif and Harley Weston.
Leading terms and leading coefficients 2009-03-31
From Susan:
Write the polynomial P(x)=x^3-3x^4+17x+11-4/3x^2 in descending order. Identify the leading term and the leading coefficient.
Answered by Penny Nom.
Square feet and acres 2009-03-31
From ROZELL:
how much land is 660'x400' also how many feet in one hundred acres
Answered by Harley Weston.
More on the square root of 0.75 2009-03-30
From Blaine:
I read your response to How is the square root of 3/4 is greater than 3/4?

What I'm hoping for is a way for my students to use their own experience and number intuition to be able to make sense of the issue. As soon as my kids see "if y is this and x is this then..." their little eyes glaze over. Unfortunately, I can't come up with a way myself. Thank you for your help.
Answered by Penny Nom and stephen La Rocque.

A rolling wheel 2009-03-29
From Jules:
How far does a wheel of radius 2 feet roll along level ground in making 300 revolutions?
Answered by Penny Nom.
Profit as a percent 2009-03-26
From Danny:
If I have something that cost me .38 and I want to sell it for $1.00 what is my profit % on this item
Answered by Robert Dawson.
The rate of change of the volume of a sphere 2009-03-25
From Kaylin:
why the rate of change of volume of a sphere is not constant even though dr/dt is constant?
Answered by Walter Whiteley.
x^2/5 - 4/5 = -3/5x 2009-03-25
From Lorrie:
x^2/5 - 4/5 = -3/5x
Answered by Penny Nom.
The optimal retail price for a cake 2009-03-25
From Shawn:
Your neighbours operate a successful bake shop. One of their specialties is a cream covered cake. They buy them from a supplier for $6 a cake. Their store sells 200 a week for $10 each. They can raise the price, but for every 50cent increase, 7 less cakes are sold. The supplier is unhappy with the sales, so if less than 165 cakes are sold, the cost of the cakes increases to $7.50. What is the optimal retail price per cake, and what is the bakeshop's total weekly profit?
Answered by Robert Dawson.
Dividing by fractions 2009-03-25
From Mitch:
-2 divided by -2/3 = ??
Answered by Penny Nom.
Comparing positive and negative fractions 2009-03-24
From Christyrose:
Hello. I am a sixth grade math student and i need help on how to compare negative and positive fractions.

Here is is:
3/4 -2/3 7/10 -5/6

I'm not sure how to do the negative parts because i was absent for a couple days. Thank you!

Christyrose

Answered by Penny Nom.
An infinite number of solutions 2009-03-24
From Sean:
this is a linear equations problem;

first:
3535.5 + Fbd (.866) + Fbc (.5) - Fab (.5) = 0
and
-3535.5 - Fab (.866) - Fbc (.5) - Fbd (.5) = 0

Answered by Harley Weston.
Conversion factor 2009-03-23
From kab:
Would like to photocopy a Landscape Design. What is the Conversion Ratio or magnification required to convert 1" = 10 feet to 1/4" = 1 foot?
Answered by Harley Weston.
Find a polynomial function with the indicated zeros and satisfying the given conditions 2009-03-23
From Kristen:
Find a polynomial function with the indicated zeros and satisfying the given conditions. Simplofy your answer (no imaginary numbers or parentheses in the answer) Zeros: 1+2i, 1-2i, 5 ; f(-2)=1
Answered by Harley Weston.
The radius of a circle 2009-03-22
From Justin:
Find the radius of a circle with a circumference of 9.43cm.
Answered by Penny Nom.
An equation for a line with a changing slope 2009-03-21
From Ben:
Is there anyway to write an equation for a line with a changing slope. e.g. A cell [hone company has a initial fee of 50$ paying for the first 100 minutes, the rate then increases to 10 cents per minute, then at 500 minutes the rate is 5 cents per minute.

If so please tell how.
Thanks

Answered by Penny Nom.
A dead fly is stuck to a belt that passes over two pulleys 2009-03-21
From Jules:
A dead fly is stuck to a belt that passes over two pulleys 6 inches and 8 inches in radius. Assuming no slippage,how fast is the fly moving when the large pulley (8 inches) makes 21 revolutions per second ? How many revolutions per second does the small pulley make ? How long will it take the dead fly to travel 1 mile ?
Answered by Penny Nom.
0.0007 as a percent 2009-03-18
From Shelly:
I need to know how to divide 0007 by 100 and soon! and I have no idea how I can get 100 to go into a number like that! here's the question this problem originated from:

change the decimal into a percent:
0.0007 _________

How do I change THAT number into a percent?

How do I change 12.455 and 92.348 and 29.005 into percents as well? I know how to change numbers like 0.6 and 0.70 into percents but not the numbers I told you I'm having problems with. Please someone explain this to me and also please focus mainly on the first number I asked about but don't forget the others please. Please reply ASAP I need to know this in about a week or less!

Answered by Penny Nom.
The sides of a parallelogram 2009-03-17
From Sami:
If ABCD is a parallelogram, prove that line AB is congruent to line CD. Clearly state your reasons and conjectures.
Answered by Penny Nom.
The midpoints of two sides of a triangle 2009-03-17
From Manis:
Prove that the line joining the midpoint of two sides of a triangle is parallel to the third and half of it.
Answered by Robert Dawson.
How many bags do I need? 2009-03-16
From Dawn:
I need to purchase an item that is .4 cubic feet in size and the amount I need is 5.625 cubic feet. How many bags do I need?
Answered by Penny Nom.
Related rates 2009-03-14
From Jeevitha:
The side of an equilateral triangle decreases at the rate of 2 cm/s. At what rate is the area decreasing when the area is 100cm^2?
Answered by Stephen La Rocque.
A linear system 2009-03-13
From Rasanga:
Use the row echelon form of the augmented matrix to solve the following linear system.
X1+2X2+X4=6
X3+6X4=7
X5=1

Answered by Harley Weston.
Four digit numbers 2009-03-10
From Theresa:
I need to know the number of 4 digit numbers that can be formed using 1-2-3-4 without repeating numbers
Answered by Penny Nom.
66 2/3% 2009-03-10
From Traci:
Please convert 66 2/3% into a decimal and a fraction
Answered by Penny Nom.
A polyhedron 2009-03-10
From Mollie:
Hi, my name is Mollie and I'm in 5th grade. Here's the math problem I have for homework tonight:

"Larissa made a model of a polyhedron using 8 pieces of clay for the vertices and 18 straws for the edges. What type of polyhedron did Larissa make?" Thanks.

Answered by Penny Nom.
Related rates 2009-03-09
From Megan:
A plane flying with a constant speed of 330 km/h passes over a ground radar station at an altitude of 3 km and climbs at an angle of 30°. At what rate is the distance from the plane to the radar station increasing a minute later?
Answered by Harley Weston.
Trifecta 2009-03-09
From Annie:
I want to know how many combinations are there for the first 3 horses in a race of 20 horses. For example, the horses can come in 1, 5, 7 or 7, 5, 1 which are totally different combos. I've only found formulas where the 1, 2, 3 and 3, 2, 1 are counted as the same combo. In horse racing every combo is different. Thank you.
Answered by Penny Nom.
Cubic feet 2009-03-07
From A student:
What is cubic feet and how do I use it in this question? (picture below)
Answered by Stephen La Rocque.
A 14 team league that is divided into 2 divisions 2009-03-07
From Ronnie:
I have a 14 team league that is divided into 2 divisions. We have a total of 18 weeks to play. I would like for each team to play the team in their division twice. The remaining 6 weeks would be played with the other division. Any help would be appreciated.
Answered by Chris Fisher and Victoria West.
Infinite sets and infinite limits 2009-03-06
From Justin:
Hello, I know I have asked a similar question before but I was just wondering if set theory applies to the lim x->0, y=1/x=infinity and if so, what type of infinity would it be? Thanks a lot for your help with this question!

Regards,

Justin

Answered by Robert Dawson and Harley Weston.
A taxi charges .20 every 1/9th of a mile 2009-03-03
From ronald:
i need to know if a taxi charges .20 every 1/9th of a mile and i have to go 6.55miles how much would that be?
Answered by Robert Dawson.
Symmetries of a polyhedron 2009-03-02
From Vincent:
I want to ask is there a formula to find out the number of axes of rotation & plane of reflection of a 3D figure, like pentagon, pyramid?
Answered by Chris Fisher.
Implicit differentiation 2009-03-01
From Emily:
determine the derivative y' at the point (1,0)
y= ln(x^2+y^2)

y'(1)= ??

Answered by Stephen La Rocque.
The game of 24 2009-03-01
From Jeff:
My son in the 5th grade was given a problem for extra credit and could use parents help. He was given the numbers 21, 13, 9, 6 and asked to use each number only once by +, -, /, x the numbers in any order to obtain an answer of 24.
Answered by Harley Weston.
An annuity 2009-02-27
From Jules:
R 20 000 is deposited in the bank,at the end of each year R 5 000 is withdrawn from it, the interest is 13 percent compounded monthly. Calculate how many years that person will be able to use his/her money ?
Answered by Stephen La Rocque.
Simplify the square root of 125 x^2 2009-02-27
From Ashley:
√125x²

How do you simplify this equation assuming that variables can represent any real number?

Answered by Robert Dawson.
A triangular pyramid 2009-02-26
From Craig:
Can you help with this question:
Write the names of the faces and the number of each kind of face on a triangular pyramid?

Answered by Penny Nom.
Multiplying in different bases 2009-02-25
From Susan:
11 base 2 X 22 base 3 + 33 base 4 = _________ base 5
Answered by Robert Dawson.
Collecting an army 2009-02-25
From bevaz:
Question from bevaz, a student: A ruler orders his chamberlain to collect an army from 30 houses. The servant goes to the first house alone and collects one man. At each house after that he takes the same number of men as he has already collected, so at the second house he goes with one other and so on. How many men did he collect in all?
Answered by Penny Nom.
Trig functions without geometric data 2009-02-24
From bob:
I do not understand how it is possible to find the sine, cosine, or tangent of an angle if there is no hypotenuse, opposite or adjacent side?!
Answered by Robert Dawson.
Amount of Water 2009-02-23
From Jeanette:
I received a water bill for 660,000 gal. of water. I don't think it is possible to pass this much water through a 1 1/2 inch pipe in 30 days. Is it possible to send that much water? I am assuming that there is a water pressure of 50 gpm, but I am not sure of that. It is a rural water system.
Answered by Janice Cotcher & Robert Dawson.
Pressure at Given Depth 2009-02-22
From Tamara:
A sea is 3km deep. The average density of sea water there is 1020 kg/m^3. What is the water pressure at the bottom?
Answered by Janice Cotcher.
Time in a Swing 2009-02-22
From Barb:
I looked at the questions concerning a pendulum as I know I need to use this formula but I am stiil not able to figure this problem out. Can you help? If a child is on a swing with a 10 foot chain, then how long des it take to complete one cycle of the swing? I know I am suppose to use the formula 8T^2 = pi^2 L but I do not understand how to do this. Thanks
Answered by Janice Cotcher.
Intercepts in a Quadratic Function 2009-02-22
From Barb:
I have a question I need help with please. What are the number of x and y intercepts that quadratic function can have and why? Thanks.
Answered by Janice Cotcher.
Surface area of a cube 2009-02-22
From JOHNNIER:
determine the surface area of a cube with volume 125 mm3, thank u very much
Answered by Penny Nom.
Percentage difference in pipe volumes 2009-02-22
From mike:
what is the % difference between a 3 inch diameter pipe compared to a 4 inch diameter pipe? and how do i find the answer?
Answered by Penny Nom.
16 golfers playing one round of golf on each of four days 2009-02-22
From Jim:
I have seen solutions for other combinations of setting up a golf trip but I haven't seen one for my group. We have 16 golfers playing one round of golf on each of four days. Is there a way for all of them to play in foursomes so that everyone gets to play with everyone else? Thanks for your consideration.
Answered by Chris Fisher.
Express the HCF of 1232 and 573 as 1232x + 573y = 1 2009-02-22
From Anonymous:
Express the HCF of 1232and 573 as 1232x + 573y = 1.
Answered by Victoria West.
Find the resultant of this displacement pair 2009-02-22
From katydidit:
Find the resultant of this displacement pair:
500 miles at 75 degrees east of north and 1500 miles at 20 degrees west of south.
How do I graph this and how do I solve this problem?

Answered by Penny Nom.
Surface area and scale factors 2009-02-21
From tina:
the surface area of a sphere is about 86 ft (2) find the surface area of a larger sphere that has a scale factor of 4
Answered by Penny Nom.
6 golfers playing 8 rounds 2009-02-21
From evan:
I have a group of 6 golfers playing 8 rounds. we would like to rotate the 3 somes so each person golfs with different people as many times as possible
Answered by Chris Fisher.
The focal point of a parabolic surface 2009-02-20
From kishore:
how to find out the focal point of a parabolic surfaces
Answered by Chris Langdon.
Vertices, edges and faces 2009-02-19
From Deb:
I have been reading the responses you have to whether or not a cone and cylinder have edges, and whether a cone has a vertex. I have also found differing answers in different books and websites. Your idea that it depends why we want to call it a vertex or edge is interesting but does not help the students next year when they have a different teacher who tells them that whatever decision I make is wrong! It also doesn't help them when they do the provincial testing. Does anyone know if EQAO considers them vertices or edges? I want to know because textbooks and provincial tests often ask the students to count the number of faces, edges, and vertices or to describe a solid by using these, or to identify a solid that they have described. We need to have consistency in what we are teaching!
Answered by Chris Fisher.
Implicit differentiation 2009-02-18
From Sunny:
Find slope of the tangent line to the curve 2(x^2+y^2)2=25(x^2–y^2) at (3,-1)
Answered by Robert Dawson and Harley Weston.
More on cardinal numbers 2009-02-18
From Justin:
Hello again, I was just wondering that since the rules of Cantor's cardinal numbers in set theory do not apply to the infinity obtained by limits in calculus (ex. x->0, y=1/x=infinity), does that mean that this infinity is the largest quantity in both calculus and mathematics?

Justin

Answered by Robert Dawson.
Angle of depression 2009-02-18
From Meeka:
An aircraft flying at an altitude of 2000m is approaching an airport. If the angle of depression of the airport is 5 degrees, what is the distance from the plane to the airport measured along the ground? Round your answer to the nearest tenth of a kilometer.
Answered by Robert Dawson.
How can other infinites can be larger than each other? 2009-02-17
From Justin:
Hello again, I was just wondering even in the context of set theory, how can other infinites can be larger than each other, I thought infinity itself is the largest possible quantity?

Justin

Answered by Victoria West and Robert Dawson.
More on infinity and Set Theory 2009-02-17
From Justin:
I greatly appreciate your help I was just wondering from your previous answer, why doesn't Cantor's cardinal numbers in set theory apply to the limit x->0, y=infinity?

Justin

Answered by Robert Dawson.
Infinity and Set Theory 2009-02-17
From Justin:
I was just wondering is the limit x->0, y=1/x=infinity, the biggest uncountable infinity according to Cantor's cardinal numbers in set theory?

Justin

Answered by Robert Dawson.
The surface area of a cube and rectangular prism 2009-02-16
From Mhiko:
how to derived the total surface area of a cube and rectangular prism using geoboard??
Answered by Robert Dawson.
The product of two integers their LCM and their HCF 2009-02-15
From Anonymous:
Two numbers have LCM = 60. If their product is 180, what is their HCF?
Answered by Harley Weston.
0/0 2009-02-15
From Justin:
Hello, I was just wondering, what is the difference between 0/0 being represented as nullity or as an indeterminate form?

Justin

Answered by Harley Weston.
In what base is 3x3= 10? 2009-02-14
From David:
In what base is 3x3= 10, 3x3=11, 3x3 = 12? is there a fast way to see this or do I have to create multiplication tables until I find the right one?
Answered by Penny Nom.
The height of a triangle 2009-02-14
From GARIMA:
IF THE SIDES OF TRIANGLE ARE GIVEN . HOW WE WILL CALCULATE THE HEIGHT OF TRIANGLE.
Answered by Penny Nom.
A fraction with negative exponents 2009-02-14
From Gabriel:
What does the line seperating the top from the bottom mean? what does it tell you to do and what is the answer to the question?

Question 3

Simplify. Write the answer using only positive exponents. Assume all variables represent nonzero numbers.

(x^-5)^-4(x^-1y)^3
________________
(xy^4)^3


Answered by Penny Nom.
Tearing down a wooden privacy fence 2009-02-14
From Dezra:
we are tearing down a wooden privacy fence, it is 180' in length and the boards are 4"w x 6' H=A0x 1" thick i have a truck that holds 15 cubic yards. I would like to=A0know how many loads it's going to take to haul it off.

thank you for your help
dezra

Answered by Harley Weston.
8 golfers, 5 rounds 2009-02-12
From John:
8 of us are going on a golf trip next month. We are golfing 5 rounds. Is there any way that everyone can play with each other at least twice? I have been trying to figure it out but it always seems that someone plays with another golfer only once.
Answered by Chris Fisher.
9 golfers 2009-02-12
From Beth:
I am organizing a golf trip for 9 women. We are playing in threesomes and are playing four days. I would love to mix it up so that everyone gets to play with each other at least once. Any mathematical suggestions as to how I can make this happen??
Answered by Robert Dawson.
A four digit number 2009-02-12
From alex:
the number of tens plus the number of hundreds equals the number of thousands. the number of ones is half the number of tens and one more than the number of hundreds the hundreds digit is 2. what is this four digit number
Answered by Penny Nom.
Cubic feet gas to grams of gas 2009-02-11
From tim:
I was trying to convert cubic feet gas to grams of gas. Is there a formula to complete this task?
Answered by Robert Dawson.
What percent is 314.70 of 693.20? 2009-02-11
From steph:
what percent is 314.70 of 693.20
Answered by Penny Nom.
Solid figures 2009-02-10
From Rebecca:
I need to know the names of solid figures. I have several pictures that I need to answer what type they are. Thanks, Rebecca 3rd grade
Answered by Penny Nom.
A definite integral 2009-02-09
From Mathata:
Evaluate: integral from 0 to 1, x^2 e^x^3dx
Answered by Harley Weston.
Fencing 5 acres 2009-02-06
From bob:
How many feet of fencing is required to fence in 5 acres
Answered by Harley Weston.
A rectangular park 2009-02-06
From Debbie:
A park named Writer's Rectangle opened in town. When asked about the dimensions of the rectangle, the city planner, responded with these clues: ---The diagonals of the rectangular park plus its longer sides together measure seven times one of the shorter sides. ---The length of one diagonal is 250 m longer than one of the shorter sides. Use this information to find the area of the park.
Answered by Stephen La Rocque.
How much did the statue originally cost him? 2009-02-06
From Debra:
Jim is able to sell a hand-carved statue for $670 which was a 35% profit over his cost. How much ded he statue originally cost him?
Answered by Penny Nom.
The substitution method 2009-02-06
From amber:
Pleasse help me with this problem
x+y=-2
y-2x=1

Answered by Penny Nom.
Fill behind a retaining wall 2009-02-05
From Fausto:
i'm trying to figure out how many cubic yards i will need. I built a retaining wall that it's 5 feet high, running 90 feet long. There is a slope running to the wall of 24 degrees. The distance from the top of the slope to the wall is 12 feet. I don't know how to calculate how many cubic yards of fill dirt i need in there. Please help. Thanks!
Answered by Harley Weston.
Faces, edges and vertices 2009-02-04
From sheila:
what is the relationship between the number of faces and the number of edges of a triangular pyramid ?
Answered by Penny Nom.
Transformations 2009-02-04
From Vickie:
Hello,

My daughter had a question on her Week By Week about transformations and I can't remember what transformations will result in proportional figures. If a transformation looks like it is smaller what is that called? Also if the figure looks bigger what is that called?

Vickie

Answered by Robert Dawson.
A company fish fry 2009-02-04
From shaelisa:
At a company fish fry, ½ in attendance are employees. Employees’ spouses are 1/3 of the attendance. What is the percentage of the people in attendance who are not employees or employee spouses?
Answered by Robert Dawson.
12 golfers 2009-02-04
From david:
12 golfers playing in 3 four balls over five rounds. can everyone play with everyone else at least once.
Answered by Victoria West.
Fertilizer in a bin 2009-02-03
From Todd:
Hello I am looking for a formula to figure out the fertilizer volume in a hopper bottom bin not only when it is full but part full as well. When you are filling it is heaped up in the middle to make a cone and when you are emptying the bin the cone is inverted so it would be nice to be able to quickly figure out the tonnes partly filled and when full.
Lets say the bin is 32 feet high from top of bin where you fill to the bottom where the product goes out and it is 16 feet in diameter. I know how to calculate the cylinder it is the cones on the top and bottom of the bin I have the main question on.

Answered by Harley Weston.
A piecewise function for monthly salary 2009-02-03
From Josh:
A sales rep for a drug company is paid on the following scale. His base salary is $1600 per month. He is also paid a commission of 2% of sales over $20,000. When his gross sales reach $30,000 he is then paid 4% on the amount above that until his sales reach $50,000 after which point his commission becomes 6%. His commission reaches 8% when his sales total goes over $100,000 for the month. Set up a piecewise function for the information above.
Answered by Harley Weston.
Calculating markup 2009-02-03
From Tonya:
I'm trying to figure out the formula for the following, I have a product that I want to receive a certain profit, but with this product I also have to pay a percentage of commission to someone. I need to know how to calculate my new selling price taking into account the markup percentage, but I don't want my cut to be lowered. I have calculated the markup percentage to get my new selling price with the commission, but it lowers my cut once I pay the commission and I don't want it to do that. Thanks
Answered by Penny Nom.
What's the derivative of y = e^2x-e^-2x / e^2x+e^-2x? 2009-02-02
From Heather:
What's the derivative of y = e^2x-e^-2x / e^2x+e^-2x ?
Answered by Harley Weston.
4 times as many or 4 times more? 2009-02-02
From Jackie:
Given : Here are 3 squares and 4 sets of 3 circles.

I wonder it is right to write in the below manner to represent the following Conclusion that
can be made from the above given information:

  1. There are 4 times as many circles as there are squares,

  2. There are 4 times fewer square than circles;

  3. There are 4 times more circles than squares.

  4. ...
Jackie

Answered by Robert Dawson.
The sequence 2n! - 1 2009-02-01
From Penny:
I am trying to help my son with this problem. Find the first five terms of the sequence that can be written from the formula A= 2n !-1.
Answered by Penny Nom.
7 golfers playing 4 days of golf 2009-01-30
From Trish:
I have 7 golfers and we are playing 4 days of golf. I would like to do pairings so that everyone gets to play with each other at least one time. Can you help with this? Not a math whiz here and looking for help. Thank you.
Answered by Victoria West.
The amount of concrete in a football stand 2009-01-30
From jacques:
A football stand 150 yd long has 20 tiers of seats; each tier has a rise of 2ft and tread of 3ft.Its construction of reinforced concrete with a cross section as shown. Find the amount of the material used in its construction.
Answered by Penny Nom.
What is 301 million in word form? 2009-01-30
From chyamber:
What is 301 million in word form?
Answered by Penny Nom.
Four digit numbers using 1, 0, 2 and 8 2009-01-27
From Mike:
How many combinations of this four digit number 1028 can this produce only using those numbers. Example would be 0000, 0001,0002,0008,0010,0011,0012,0018,etc...
Answered by Penny Nom.
The vertical line test 2009-01-26
From bob:
what is a Vertical line test
Answered by Stephen La Rocque.
Three consecutive natural numbers 2009-01-26
From abby:
1.what is the smallest four-digit number that is the sum of the three consecutive natural numbers? what are the three consecutive natural numbers that give this sum?

2.what is the largest three digit number whose square root is a prime?

Answered by Penny Nom.
The difference quotient (f(x+h)-f(x))/h 2009-01-26
From helen:
Find the difference quotient of f, f(x+h)-f(x)/h, h does not equal 0, for this function. Be sure to simplify. that is, find f(x)=x^2-x+4
Answered by Robert Dawson.
How many students were there 2009-01-25
From sooos:
After a math lecture in university, the students go to the cafeteria. EACH ONE of them buys a cheese sandwich and a cup of tea. They ALL pay $18.49. How many students were they?
Answered by Chris Fisher.
How fast is the visible surface of the earth decreasing? 2009-01-24
From Ray:
A dive bomber loss altitude at a rate of 400 mph. How fast is the visible surface of the earth decreasing when the bomber is one mile high?
Answered by Harley Weston.
Perimeters and scale factors 2009-01-24
From dennis:
I have this work that says how are the value of the ratios for the perimeters related to the scale factor? PLEASE HELP ME OUT
Answered by Harley Weston.
An integral from 1 to infinity 2009-01-24
From Ray:
Determine the area bounded by the x-axis and the curve y=1/(x^2) from x=1 to x=infinity.
A. 1.00
B. infinity
C. indeterminate
D. 2.00

Answered by Harley Weston.
20 of us golf together in groups of 4 2009-01-24
From D.:
Every Sunday, 20 of us golf together in groups of 4. I am looking for a way that each of us play with 3 other people each week and ultimately get to play in groups that are unique. For instance if week 1, I play with 2, 3, 4 and then the next week I play with 5, 6, 7, and the 3rd week I play with 8, 9, 10 and so forth until I have played with everyone. Everbody else should be doing the same thing. Can you give me a schedule for this and how many weeks would it take for all of us to accomplish this where we all play with different combinations of people. (We should not play with the same person very often or even the same pairs of people but everyone should play with everybody else) I hope this makes sense........whew and thanks!
Answered by Victoria West.
The tens digit of 0! + 1! + 2! + 3! + 4! .............+9999! + 10000! 2009-01-23
From Kaylyn:
determine the tens digit for the expression:
0! + 1! + 2! + 3! + 4! .............+9999! + 10000!

Answered by Robert Dawson and Penny Nom.
Archimedes' formula for parabolic arches 2009-01-23
From La:
Use calculus to verify Archimedes' formula for y=9-x^2. Prove Archimedes' formula for a general parabolic arch.
Answered by Harley Weston.
3/4 0f what=75 2009-01-22
From tyree:
3/4 0f what=75
Answered by Harley Weston.
In the shadow of a flagpole 2009-01-22
From La:
How fast is the length of the shadow of an 18 foot flagpole growing when the angle of elevation of the sun is 45 degrees and is decreasing at a rate of 10 degrees per hour?
Answered by Harley Weston.
The parabola with vertex (7,-2) and directrix y = -3 2009-01-21
From Deann:
Find an equation of the parabola with vetrex (7,-2) and directrix y =(-3)
Answered by Penny Nom.
Factor x^2 - y^2 2009-01-20
From Shell:
complete Factor: x^2-y^2
Answered by Penny Nom.
PST and GST 2009-01-20
From janet:
Hello I need to figure what one would pay pst 8% gst 5% on a total of $1700.00 I have a small business some pay with cash I need to calculate the taxes in reverse Can you help with formula that the government might like Thank you Janet
Answered by Harley Weston.
Square footage 2009-01-20
From david:
15 3 5/8 x 18 7"
Answered by Penny Nom.
A 28 week golf schedule 2009-01-17
From Don:
You have a 12 man golf schedule playing 8 weeks. Can you go further and make it a 28 week schedule. 3 foursomes. any help would be greatly appreciated.
Answered by Victoria West.
Fuel in a tank 2009-01-16
From Steve:
I am not a math expert and want help seeing how much fuel is in a tank at one given time the tank that i have is 27 inches round. It is 69 inches long. laying length ways. it holds 140 gallons of fuel at fill up. I do know that it would hold less at the bottom of the tank an more in the center. Is there anybody that could just give me the figures by the inch without a formula. thanks for your time
Answered by Stephen La Rocque.
Problem solving with fractions 2009-01-16
From Linda:
Sam and Mary each owned one-half stock in a printing company. Sam sold 2/5 of his stock to Mary. What fractional part of the printing business does Mary now own?
Answered by Stephen La Rocque.
A golf trip with 12 golfers 2009-01-16
From Brad:
I am organizing a golf trip with 12 golfers. We will play 5 rounds (3 foursomes each round) total and play two 9 holes matches per round (18holes). So two matches are created within each foursome. What are the possible combinations of foursomes so that everyone plays each other at least once with the least number of repeat matches? For example: Here are the foursomes for round 1:(1,2,3,4) (5,6,7,8) (9,10,11,12) match 1 round 1: 1vs2, 3vs4, 5vs6, 7vs8, 9vs10,11vs12 we can not split foursomes up for the second match (9 hole matches): So match 2 round 1: 1vs3, 2vs4,5vs7,6vs8,9vs11,10vs12
Answered by Victoria West.
Segments of a sphere 2009-01-16
From Herman:
How do I solve for a segment of a sphere so that the orange peel or pie shaped section is converted to a flat surface with dimensions. I form large diameter domes, elliptical and sphere heads on a press. I enter the diameter say 30 feet with two segments above and below the equator and the total number of segment around the circumference at say 18 so the widest part of the pie shaped section will fit my press. How do I take the upper or lower course above or below the equator and figure the height of the orange peel shape, the chord length at top and bottom, and solve for the right angle at 2 degree increments down the arc length of the pieces so I can layout the flat plates prior to pressing. Thanks for your help.
Answered by Robert Dawson.
Saving for his daughter's education 2009-01-15
From George:
To save for his daughter’s university education at the end of each month Mr McLean sets aside £50 in a savings scheme paying 10% p.a. compounded monthly. He begins saving when his daughter is 3 years old. How much will he have saved by the time his daughter is 18?
Answered by Harley Weston.
Does probability apply here? 2009-01-15
From Paul:
Suppose I have a dice in my hand that I am about to roll. The probability that I roll a six is, all things being equal, 1/6. I accept that.

Suppose now the I roll the dice and immediately cup my hand over the result. What is the probability that I have rolled a six? People seem to want to say it is still 1/6. But it can't be can it!? It is surely either 1 or 0, depending on whether I have in fact rolled a six?

Answered by Robert Dawson.
Ordering fractions 2009-01-14
From Shea:
I am a special education teacher, who is trying to to show my students how to order fractions. I found your response to this questions while looking for information. I need a visual to explain to my students and myself on how and why you multiply the fractions together.
Answered by Harley Weston.
The circumference of a table 2009-01-14
From Maggie:
My dinning room table is 48 inches. I need to know what its Circumference is? I do not know algebra so a formula will not help me.
Answered by Penny Nom.
A rectangular prism 2009-01-13
From nika:
How many bases does a rectangular prism have? What are they?
Answered by Harley Weston.
Negative rate of change 2009-01-12
From hemanshu:
when i have to find rate of change of decrease in any value my ans comes in negative why??????????
Answered by Penny Nom.
A four digit passcode 2009-01-12
From john:
i have a machine at home that has a four digit passcode and the numbers range from 1 to 4 how can i get all the ways those numbers can be used
Answered by Harley Weston.
16 golfers 2009-01-11
From Bill:
I have read many of your answers to similar questions but still need your assistance. We have 16 golfers that will be playing in 4 groups of 4 each day on an upcoming golf vacation. Can you suggest the best way to scheudle the individuals assigned to each daily foursome so that we have the most diversity in foursomes throughout the vacation. I am trying to have evryone play with as many members of the group over the week with the least amount of repetition. If you would please provide it for both 6 and 7 days, since we are not sure yet how many days we will be playing.
16 golfers-playing in groups of 4 each day-6 days
16 golfers-playing in groups of 4 each day-7 days

Answered by Victoria West.
The slope intercept form 2009-01-11
From Norma:
write equation in slope intercept form and then find the slope and y-intercpet of the corresponding line.

please show step by step

1). 2x-3y-9=0 and 2). 3x-4y+8=0

Answered by Penny Nom.
A T-shirt fundraiser 2009-01-11
From Andre:
Lou's class is selling T-shirts for a fundraiser. The supplier charges 750$ for the initial design and the set-up plus 5$ for each imprinted shirt. The students sell the shirts for 15$ each.
a)How many T-shirts do the students need to sell to break even?
b)How much profit will the students make if they sell 150 shirts?

Answered by Harley Weston.
Constants, and coeficiants 2009-01-10
From Kelsey:
I am having trouble trying to remember what like terms, constants, and coeficiants are. My math teacher hasent been explaining it very well and i was wondering how to remember what they are, and what they are? I dont understand the concepts.
Answered by Stephen La Rocque.
18 golfers and 5 rounds 2009-01-10
From steve:
We have 18 golfers 5 rounds . Would like 3 foursomes and 2threesomes each day Different groups each day.
Answered by Victoria West.
The table of values for y=3x-10 2009-01-09
From Jade:
How can I know the value of x for y=3x-10 in this table of values pls. include the deatails

if x=-2 y=-16
if x=3 y=-1
but how if y=2 what is x

Answered by Penny Nom.
The top half of a cone vs. the bottom half of the same cone 2009-01-08
From Tammy:
What is the ratio of the volume of the top half of a cone vs. the bottom half of the same cone? Is it seven times larger, or four times larger? and is the ratio consistent regardless of the size of the cone opening?
Answered by Penny Nom.
Grade 4 Foundation Skills 2009-01-07
From Susan:
Hi, I'm taking over tutoring my 15 yr old niece, who has dyslexia. She has been being tutored by someone else,but my sister had problems with her. My niece needs help in math and english and spelling. Is there a test that I can do with her to see where she stands at this point? I do believe the math is at a 3rd, or 4th grade level. I'm not sure about the other two. Do you have any suggestions? All help is appreciated.
Answered by Victoria West.
30% profit 2009-01-06
From bob:
this is a basic question that I just can't get thru my head.

I know all my cost for a product, if I want to sell at a given profit can I divide by a given number, example for 30 % divide by .7 or do I multiply? Is there a simple answer for this? Sure would make my work a lot easier.

Thank you for your time

Puzzled in Ohio

Answered by Penny Nom.
The factors of an integer 2009-01-06
From Madison:
what are the factors of 878
Answered by Robert Dawson.
Determine y'' by implicitly differentiating twice 2009-01-04
From Walter:
Given x^3 - 3xy + y^3 = 1 , determine y'' by implicitly differentiating twice. I cannot solve this. Would you be kind enough to perform the mathematics and show the steps involved in obtaining the solution?
Answered by Harley Weston.
The sum of the roots of a quartic 2009-01-03
From peter:
How do you find the roots of an equation without graphing? like, i have a problem that says what is the sum of the roots of x^4-x^3+5x^2+4=0.
Answered by Harley Weston.
What is f^-1(3) when f(x)=2x-1? 2009-01-03
From Peter:
how do you do functions like f^-1(3) when f(x)=2x-1?
Answered by Penny Nom.
System of equations - comparison method 2009-01-02
From nick:
Hi, I am doing a project on system of equations by using comparison method, but for some reason or another, i can't seem to find any information on the comparison method at all. So, please would you help me by giving me a website that you know of that has some information on my method. And, if you know the answer, can you please tell me why or when do we use system of equations by using comparison method? Thank You
Answered by Robert Dawson.
The integral of f(x)dx and the integral of f(x-4)dx 2009-01-02
From Katie:
Calculus: If the definite integral from -2 to 6 of f(x)dx=10 and the definite integral from 2 to 6 of f(x)dx=3, then the definite integral from 2 to 6 of f(x-4)dx= ?

I don't understand how to solve definite integrals when the function has something more than just x inside the parenthesis such as f(4-x).

Answered by Robert Dawson.
The square root of 164 2008-12-31
From Melissa:
How do you simplify the square root of 164?
Answered by Stephen La Rocque.
What is i^i? 2008-12-27
From randomness:
i have learnt that 'i' is square root of -1. What is then i^i ? It baffles my maths teacher...
Answered by Robert Dawson and Penny Nom.
The area of a triangle 2008-12-23
From ashiya:
Hi,

I have attached figure of the triangle, Can you please help to find the area of Triangle ABC.

Thanks.

Ashiya

Answered by Stephen La Rocque.
24 golfers playing golf for 9 rounds 2008-12-23
From Duane:
Got 24 golfers playing golf for 9 rounds. Any formula where everyone can play with everyone else at least once. We are playing 4somes only. Thanks, Duane
Answered by Victoria West.
A spill of oil on a flat surface 2008-12-19
From Phillip:
Hello, my question is: If I spilled 25 gallons of diesel fuel on a flat surface how much area would the spill cover? How would I mathematically figure it out? Thanks!
Answered by Robert Dawson and Harley Weston.
Coefficient of variation 2008-12-17
From JR:
I have read your reponses regarding the coeffcient of variation (CV) and find them very useful. I still have a question about interpreting the CV. Let's that the CV of sample #1 is 3% and that of sample #2 is 12%. Can I report that Sample #2 is 4 times more variable than sample #1? Thanks in advance!
Answered by Robert Dawson.
The angle between two lines 2008-12-17
From abhi:
how to calculate the angle between two lines, given the length of the lines.. angle should vary from 0 - 360 in the counterclockwise direction
Answered by Robert Dawson and Harley Weston.
Order from least to greatest 2008-12-16
From staci:
order from least to greatest on a number line,
7/10 3/5 5/10

Answered by Robert Dawson.
Surface area of an irregular shape 2008-12-15
From Patrick:
An irregular shaped object (lets say a gold nugget, not smooth with pockets) can have its volume determined by comparing its mass in water.

Is there any method or means or anything that could be used to determine the surface area of this shape? Whether that be theoretical mathematical formula to using a special infrared technique,etc...

The problem I foresee is that the component parts cannot be divided into smaller geometric shapes. I would propose an answer although I don't know if it is a good one: A liquid material that dries super-thin, but has a very specific and easily determined volume/mass is coated over the object. Measure the mass difference between the beginning sample of fluid and the mass after the object has been coated. Then determine the surface area of the same mass of fluid in a geometric shape. Is this feasible?

Answered by Robert Dawson.
Do all square numbers have an odd number or factors? 2008-12-11
From Amy:
Do all square numbers have an odd number or factors?
Answered by Victoria West.
A line in slope intercept form 2008-12-08
From laurie:
the question: Write and equation of the line that passes through the point and has the give slope. Then rewrite the equation in slope intercept form. (2, 5 ) m= 3
Answered by Penny Nom.
The cost of a foundation 2008-12-08
From carl:
Hi, I need to work out the cost of a foundation.

The foundation is 1m deep x 500mm wide and is 6.3 meters long.

The cost per cubic meter for concrete is 1000 rand escuvation of earth will be 100 rand per cubic meter and back filling wil also be 100 rand per cubic meter.

What will the cost be

Answered by Harley Weston.
Golf - 24 guys 4-somes 6 rounds 2008-12-07
From Duane:
Thank you. What about 24 guys 4-somes 6 rounds everyone playing with each other at least once.

Thanks again.
Duane

Answered by Victoria West.
Gravity's Effect at a Distance 2008-12-06
From ROHAN:
AT WHAT HEIGHT FROM THE SURFACE OF EARTH,THE WEIGHT OF THE BODY BECOMES HALF? (RADIUS OF THE EARTH = 6400)
Answered by Stephen La Rocque.
Determining the Unknown Numbers 2008-12-06
From jacob:
the sum of two numbers is 45 and their difference is 15. what are the two numbers?
Answered by Janice Cotcher.
Combinations of Golf Groups 2008-12-05
From Duane:
We have a golf outing coming up with 25 golfers playing 5 rounds in 5-somes. Is there a formula to figure out where everyone can play with different people each round? Thanks, Duane
Answered by Victoria West.
6 golfers play threeball 2008-12-04
From Ian:
I have a group of 6 golfers wanting to play 3 rounds as 2x threeballs, but with different players each day. Is this possible? Can you provide some threeball combinations for this please?
Answered by Victoria West.
Decimals and fractions used in our daily lives 2008-12-04
From josh:
i have a projecct due friday and i need to know, how are decimals and fractions used in our daily lives. i am having trouble coming up with ideas. i need seven more that dont involve money or recipes. please help me.
Answered by Harley Weston.
Some number theoretic speculations 2008-12-04
From Andrew:
Another way of looking at the 'alternating parity polynomial', again based on Fermat's Little Theorem, is to substitute (a - b) for x in x^(p-1) - 1 as this is always divisible by any prime p. So, if one removes the "- 1", there is always a remainder of (1/p)! (I took up your challenge!)
.
.
.

Andrew

Answered by Chris Fisher and Victoria West.
How many four-legged animals are in the field? 2008-12-02
From evelyn:
Some chickens, cows and sheep are in the field. Their number of legs is 12 more than twice their number of heads. How many four-legged animals are in the field?
Answered by Harley Weston.
Division in different bases 2008-12-02
From MICHELLE:
DIVIDE 538 BY 14 IN BASE 2, 3, 4 & 5
Answered by Penny.
How high is the flagpole? 2008-12-02
From michael:
100 m from the base of a flagpole the angle of elevation of the top of the flagpole is 7degree 16'12".how high is the flagpole
Answered by Harley Weston.
Topsoil on a soccer field 2008-12-02
From rita:
a park foreman decides that a soccer field needs 4 inches of topsoil. soccer field is 100 feet by 250 feet. 1 truckoad is 5 cubic yards. how many truckloads do we need to order. i keep confusing myself with formulas. please explain the steps to me thank you
Answered by Penny.
Fact families with 4 facts 2008-12-01
From Shazia:
Question from Shazia, a parent:

When does a fact family not have 4 related facts? Give an example.

Answered by Harley Weston.
What is so important about quadratics? 2008-11-29
From zoe:
what is so important about quadratics?
Answered by Harley Weston.
Related rates 2008-11-26
From Lyudmyla:
How fast is the volume of a cone increasing when the radius of its base is 2 cm and growing at a rate of 0.4 cm/s, and its height is 5 cm and growing at a rate of 0.1 cm/s?
Answered by Harley Weston.
A periodic function 2008-11-26
From Ali:
A periodic function f(x) has a period of 9, if(2)=-3 and f(5)=13, determine the value of f(11)?

tks /rgrds

Ali

Answered by Penny Nom.
Fractions of fractions 2008-11-26
From DIANNE:
Question from DIANNE, a student:

5(16-9) divided by 7x2+14 top part of equation
_____________________________________
12-3x2 bottom part
Answered by Penny Nom.

Six golfers 2008-11-25
From Joe:
I have a group of 6 golfers wanting to play 3 or 4 rounds but with different players each day if possible. We know we cannot make it happen without some playing with the same person. Can you provide some pairings for this?
Answered by Victoria West.
A cereal box that has a volume of 12000cm^3 2008-11-24
From William:
Hi, I have this math question that says construct a ceral box that has a volume of 12000cm3 and the surface area of the box has to be between 3200cm2 and 400cm2, I found some dimensions that are 20 by 20 by 30 and it works, but when I attempt to make the box, or make a net I would need this hugh piece of paper/cardboard, which I don't have, is there an easier way to find different dimenshions so I dont need a hugh piece of paper to create a net easily? Because i've tried trial and error but it has taken to long and I cant find anything,else, thats resonable.
Answered by Harley Weston.
The game of 24 2008-11-24
From Davey:
this has us all stumped. 5 ,5 ,7 ,and9 = 24. Oh may you assist in ending our brain pain
Answered by Penny Nom.
Cubic yards of fill dirt 2008-11-22
From Donnie:
I need to know the formula to figure cubic yds of fill dirt to fill an area from 4ft. deep to 18 inches deep this lot is on a slope and I am wanting to level 176 ft long by 4ft. deep the sides is 115 ft. wide the upper side is 18 inches deep.
Answered by Harley Weston.
Profit as a percentage 2008-11-21
From Nomar:
Cost Price is $ 556,109.64 and Selling price is $ 1,020,936.00, What is the Profit Percentage?
Answered by Harley Weston.
Factor 9x^2 + 6x + 4 2008-11-21
From Jonah:
how can i solve this by factoring: 9x^2 + 6x + 4
Answered by Harley Weston.
The path of a small sailboat 2008-11-19
From jane:
a sailor in a small sailboat encounters shifting winds. she sails 2.00 km East then 3.40 km North East, then an additional distance in an unknown direction. Her final position is 6.68 km directly east of the starting point. find the magnitude & direction of the third leg of the voyage.
Answered by Harley Weston.
Factoring 2008-11-19
From Neji:
How do you factor (y-z) (y+z) (y^4+y^2z^2+z^4) and get (y+z)(y^2-yz+z^2) (y-z) (y^2+yz+z^2) as the answer?
Answered by Harley Weston.
An algebraic exercise with fractions and radicals 2008-11-18
From Rubén:
Question from Rubén, a teacher:

I have the ecuation:

U = [yb / (ab+a^2)]^1/2 + [ya / (ab+b^2)]^1/2

I know for sure this reduces to

U = [y (1/a + 1/b)]^1/2

but I cannot find a way to get into that result!

Thanks

Answered by Rubén Osuna.
How do I compare fractions and decimals? 2008-11-18
From Alexis:
How do I compare fractions and decimals ?
Answered by Penny Nom.
How do you solve " (x-4)^3/2 = -6 "? 2008-11-17
From Jim:
How do you solve " (x-4)^3/2 = -6 " ?
Answered by Penny Nom.
How far are the boats apart? 2008-11-14
From dom:
Two boats leave port at the same time. They leave at 150 degree angle. One boat travels at 10mph and the other at 20mph. After two hours how far are the boats apart?
Answered by Penny Nom.
A cross-sectional area 2008-11-13
From David:
I am doing a science project examining the tensile strength of seaweed fronds. I need to calculate the cross-sectional area from the major and minor diameters of the elliptical central axis of the seaweed frond. What formula would I use to get this. The strength of the segments is expressed in terms of cross-sectional area of the frond at the test site.
Answered by Penny Nom.
What is -5x+3y=24? 2008-11-13
From Robert:
What is -5x+3y=24???????
Answered by Stephen La Rocque.
The angles and sides of a triangle 2008-11-13
From JAMIE:
a triangle with a side(b)37m an angle(C)70degrees and (a)79m find values of angles A and B and length of side c
Answered by Stephen La Rocque.
A barrel on its side 2008-11-13
From Dave:
Question from Dave:

How many gallons are left in a 36x60 in. barrel (laying on its side) and has 16 in. of gasoline left. I have attached a diagram.

Answered by Harley Weston.
Real and imaginary zeros 2008-11-12
From David:
Find all the real and imaginary zeros for each polynomial. Factor each polynomial. Leave factors with imaginary zeros in quadratic form.

h(x)= x^5 +2x^4 - 10x^3 -20x^2 +9x + 18

Answered by Harley Weston.
A conical funnel 2008-11-12
From Rachael:
Hello, I am a 10th grader in AP Calc, and can not figure out this question: Water is running out of a conical funnel at the rate of 1 inch^3/sec. If the radius of the base of the funnel is 4 in. and the altitude is 8 in., find the rate at which the water level is dropping when it is 2 in. from the top.
Answered by Harley Weston.
How fast is the distance between the airplanes decreasing? 2008-11-10
From Crystal:
At a certain instant, airplane A is flying a level course at 500 mph. At the same time, airplane B is straight above airplane A and flying at the rate of 700 mph. On a course that intercepts A's course at a point C that is 4 miles from B and 2 miles from A. At the instant in question, how fast is the distance between the airplanes decreasing?
Answered by Harley Weston.
A flagpole and a yardstick 2008-11-07
From Wanda:
One boy holds a yardstick vertically at a point 40 feet from the base of the flagpole. The other boy backs away from the pole to a point where he sights the top of the pole over the top of the yardstick. If his position is 1 ft 9 in from the yardstick and his eye level is 2 ft above the ground, find the height of the flagpole.
Answered by Penny Nom.
A repeating decimal 2008-11-07
From mike:
what is 0.0028282828 recurring as a fraction?
Answered by Penny Nom.
Algebraic equations with fractions 2008-11-07
From John:
Solve each equation. Check each solution.
1/8+5x/x+2=5/2
10/2y+8 - 7y+8/y(squared)-16 = -8/2y-8

Answered by Harley Weston.
The height of an equilateral triangle 2008-11-06
From touqeer:
My question is that how can we find the height of an equilateral triangle without using pythagoras theorem?
Answered by Penny Nom.
Forming 10 from 4 fours 2008-11-06
From Corderro:
I need a answer for 10 using only the number 4
Answered by Penny Nom.
The cost of a steel plate 2008-11-05
From Tammie:
What is the formula to calculate the following:

I have a plate of steel that is 1/4" x 48" x 192" at a price of $15.23 per square foot What is the formula to calculate the cost if I only use 1/4" x 47-1/4" x 68-1/8" of that steel plate?

Answered by Penny Nom.
Separating variables 2008-11-04
From Terry:
by separating variables solve the initial value problem

(x+1)y' + y = 0 y(0) = 1

Answered by Harley Weston.
Job applicants 2008-11-03
From dave:
Three-fourths of the applicants for a position had previous experience. If 144 people applied, how many had previous experience? How many did not have previous experience.
Answered by Penny Nom.
The prime factorization of one billion 2008-11-02
From Alta:
The prime factorization of 1000 is 2 cubed times 5 cubed. How do you write the prime factorization of one billion using exponents?
Answered by Penny Nom.
A circle tangent to a line and with its centre on another line 2008-11-01
From liza:
Find the equation of the circle of radius squareroot 26 tangent to the line 5x+y=13 and having its center on the line 3x+y+7=0.
Answered by Chris Fisher.
How far does the ball travel? 2008-11-01
From Betty:
This is a question that is perplexing me. I tried to solve it with the Pythagorean Theorem but have not been able to get the right answer. A ball attached to the moving end of the 5-meter arm of a pendulum. The pendulum swings through a 90 degree arc once. Approximately how far, in meters, does the ball travel?
Answered by Victoria West.
A homework problem 2008-10-28
From Shawn:
I'm checking my son's homework and we disagree on the solution. The problem reads: 135mn^4 (n to the 4th power) over 50n^2 (n squared) I think the answer is: 27mn^2 (squared) over 10 Can you help?
Answered by Penny Nom.
2x^3+x^2-2x-1=0 2008-10-26
From bobby:
2x^3+x^2-2x-1=0
Answered by Penny Nom.
Order the #s from least to greatest 2008-10-24
From morgan:
order the #s from least to greatest

18 7/12, 18 2/3, 183/4, 18 1/3

Please help
Morgan

Answered by Stephen La Rocque.
Put in lowest terms 6a^2c/8ab 2008-10-24
From hana:
put in lowest terms 6a^2c/8ab
Answered by Stephen La Rocque.
Antiderivative of 1/(x(1 - x)) 2008-10-22
From Matt:
derivative of dx/(x(1-x))

From what I've seen I should break apart the equation as such derivative of dx/x - dx/(1-x) and then get the 2 corresponding log functions.

If that is correct why does this factoring work, if that is incorrect what is the proper way to find the derivative.

Answered by Harley Weston.
Factor completely 2008-10-22
From Monica:
I have to factor the following two items completely, but I am lost. I've tried factoring by substitution and grouping but they don't seem to work:

1. 5(w-3z) -10(w-3z)=

2. ab-21+a3-b7=

Answered by Penny Nom.
The chairs on a ski lift 2008-10-21
From Travis:
If you are sitting on chair 33 of a ski lift, which is a loop. Chairs are numbered consecutively, starting with 1. Chair 97 passes you halfway up the ski slope. How many chairs are on the lift?
Answered by Penny Nom.
Exponential form 2008-10-21
From layla:
how we wite 256 in exponential form
Answered by Harley Weston.
Scaling a logo 2008-10-20
From Carl:
Hello! I hope that you can help me out here. I've been trying to figure this out for awhile now and can't come up with the answer logically. I am a graphic designer by trade and need to figure out how large my client's logo should be. The logo will need to be sized proportionally with the surface area of the page. The logo will always be sized at 1.2938(w) x .75(h) on a letter size page (8.5 x 11).
My question is...
When you increase the size of the page (not always proportionally sized - dimensionally) how do figure out the NEW size of the logo. I could have a page 20" x 40" or 30' x 4'... or ANY size really. I believe I have the new surface area of the logo but can't find the new dimensions.

Answered by Harley Weston.
Melting ice on a hemisphere 2008-10-20
From heather:
The top of a silo is the shape of a hemishere of diameter 20 ft. if it is coated uniformly with a layer of ice, and if the thickness is decreasing at a rate of 1/4 in/hr, how fast is the volume of ice changing when the ice is 2 inches thick?
Answered by Penny Nom.
I have 6 vertices and 10 edges 2008-10-20
From Jacquelin:
I have 6 vertices and 10 edges. One of my faces is a regular polygon.
Answered by Chris Fisher.
How much does Luc pay for his tools?? 2008-10-20
From imran:
Luc makes $1 profit on each tool he sells. Marc makes a $2 profit while paying $5 less for his tools than Luc. Marc thus makes $6 more profit for every $100 invested. How much does Luc pay for his tools??
Answered by Penny Nom.
The area of a square 2008-10-20
From Danny:
Is it possible to use a circumference measurement to find the area of a square when all 4 corners have a 1.75" radius?
Answered by Harley Weston.
Prime factorization 2008-10-19
From nick:
while im doing prime factorization for one number and it cant be divided 2,3 or five so what next?
Answered by Penny Nom.
A football field covered in garbage 2008-10-17
From AMie:
a small city produces 500,000 cubic ft of garbage per week. If all of this garbage were stacked neatly (in a nice vertical pile) on a 100-yard by 60-yard football field, how high would the pile be (in feet)?
Answered by Penny Nom.
An arithmetic series 2008-10-17
From Laura:
In an arithmetic series 5+9+13+...+tn has a sum of 945. How many terms does the series have? What formula do I use?
Answered by Penny Nom.
A roll of film 2008-10-16
From John:
I need to know how to calculate the build up on a roll of film is calculated. Example: I start with a 6" diameter core, and I start winding .005" thick film on the core, so I am adding a total of .010" to the diameter each wrap. If I continue to do this for a total length of film of 3000 feet, what will the roll diameter be? So what I need is the formula to perform this type of calculation. Can you help me? I want to be able to plug the formula in a spread sheet and to be able to input a core diameter, a film thickness and a total length and get a roll diameter.
Answered by Penny Nom.
Related rates 2008-10-16
From Gisela:
As sand leaks out of a hole in a container, it forms a conical pile whose altitude is always the same as its radius. If the height of the pile is increasing at a rate of 6 in/min, find the rate at which the sand is leaking out when the altitude is 10in.
Answered by Penny Nom.
The rate of change of the volume of a cone 2008-10-15
From Barbara:
Suppose that both the radius r and height h of a circular cone change at a rate of 2 cm/s. How fast is the volume of the cone increasing when r = 10 and h = 20?
Answered by Harley Weston.
Concavity and the second derivative 2008-10-15
From Christina:
I'm having trouble solving for a second derivative for the following graphing question.

f(x) = (X^2+2x+4)/2x

using the quotient rule, I found:
f'(x) = (x^2-4)/(2x^2)

however, using the quotient rule again I can't seem to solve it (concavity):
f'''(x)=[(2x)(2x^2)-(x^2-4)(4x)]/[(2x^2)^2]
f''(x)=[(4x^3-(4x^3 -16x)]/4x^4
f''(x)=16x/4x^4
f''(x)=4/x^3

and making the equation equal to zero result in 0=4 which doesn't seem to make sense...

Answered by Penny Nom.
The average rate of change of gasoline used 2008-10-06
From JHulie:
What is the average rate of change of gasoline used, measured in miles per gallons if you travel 212 miles, then you fill your gas tank up again and it takes 10.8 gallons. If you designate your change in distance as 212 miles and your change in gallons as 10.8?
Answered by Penny Nom.
equivalent fractions 2008-10-06
From marcella:
a fraction equivalent to 16/20
Answered by Harley Weston.
Algebraic fractions 2008-10-06
From Kayla:
(x^2-9)/x times (x^3-4x)/(x^2+5x+6)
Answered by Harley Weston.
A different approach to a word problem 2008-10-03
From Kenneth:
Sarah's age is 2/3 of Mary's age and 3/4 of Ruth's age. The sum of their ages is 46 years. How old is each?
Answered by Penny Nom.
The unusable space in a container 2008-10-02
From Billy:
an object is 42 inches long, 29 inches wide, and 69 inches high. The usable capacity of this container is 36.4 cubic feet. How much space in percentage is left over from the container?
Answered by Penny Nom.
Conversion to cubic feet 2008-10-01
From Dorothy:
What is the cubic feet of an item that is 7"x16 7/8"x19"
Answered by Harley Weston.
Cost before and after the GST 2008-10-01
From bruce:
Hi, i seem confused on this topic:

If i need to add say 10% GST to a cost then i would times it by 1.1.
If i need to reduce the cost by 10% then i need to divide it by 1.1. that is all well and good.


What if i need to add 12%, 18% or 20% what is the formula then?

Answered by Penny Nom.
The vertex of a parabola 2008-09-30
From Anne:
How do you find the vertex of the function f(x)=-x squared+7x-6
Answered by Penny Nom.
Lines that pass through a point 2008-09-30
From Abigail:
what is the name for a point that a group of lines pass through that contains the letters c e n p i l?
Answered by Penny Nom.
z(z+1)-x(x+1) / z-x 2008-09-30
From sylvia:
z(z+1)-x(x+1) / z-x

HOW DO I SIMPLIFY THIS

Answered by Penny Nom.
a(a+1) - b(b+1) 2008-09-30
From Shaun:
I need to factor (a-b) out of the following: a(a+1) - b(b+1). I know it is simple but I cannot remember how.
Answered by Penny Nom.
Half life 2008-09-29
From aisyah:
if dA/dt = kA and A(0) = A as the model for the decay of a radioactive substance, how can show that in general, the half life T of substance is T = -(In 2)/k
Answered by Harley Weston.
How tall is the wall? 2008-09-29
From ash:
you and bob are separated by a tall wall you stand 10 feet further from the wall than bob your angle of elevation is 37 degrees and his 44 degrees how tall is the wall?
Answered by Penny Nom.
Is it possible to a function to be both even and odd ? 2008-09-28
From Mohammed:
Is it possible to a function to be both ( Even and Odd )
Answered by Harley Weston.
3,6,10,15,and 21 2008-09-28
From jarred:
i am currently stumped on a math project that requires me to find out the recursive formula for a sequence of numbers. the numbers in the sequence are 3,6,10,15,and 21. Thank you for your time.
Answered by Walter Whiteley.
Simplify 6(a+2b)+8a-16b 2008-09-27
From madison:
6(a+2b)+8a-16b
Answered by Penny Nom.
The standard form Ax+By=C 2008-09-26
From Susan:
I am having difficulty changing the point-slope form of the equation of the line y-y1=m(x-x1) to the standard form Ax+By=C. You must express A,B,C in terms of the constants m,x1,y1. I have been able to insert numbers and switch easily from the general to the point slope equation, but I am stumped on how to convert it this way.
Answered by Penny Nom.
Exponential form 2008-09-25
From Pat:
The African bush elephant is the largest land animal and weighs about 8 tons. Write this amount in exponential form.
Answered by Penny Nom.
A father and daughter problem 2008-09-19
From Lamarr:
A father is four times as old as his daughter is now. In 20 years he will be only twice as old as his daughter. How old are the father and daughter now?
Answered by Penny Nom.
Simplify [[1/(x+3)]-1/3]/x 2008-09-19
From diana:
How does [[1/(x+3)]-1/3]/x simplify
Answered by Harley Weston.
Oils and fats 2008-09-18
From Barry:
I'm doing a nutrition assignment and I'm adding up my Oils and Fats category. I have 10ml of butter to add to 31g of fat so I'm trying to figure out what to do or I should say how to figure out the answer. Barry
Answered by Penny Nom.
Simplifying equations 2008-09-18
From diana:
I do not know how to simplify equations. How does 5x^2-7X+2 become (x-1) (5x-2)?
Answered by Penny Nom.
Number of factors 2008-09-18
From Austin:
I am stuck on finding what type of number has exactly two factors and what type of number has an odd number of factors? I need some examples.
Answered by Penny Nom.
Adding fractions 2008-09-17
From amanpreet:
please can you help me to simplify:

2(c-3) / 7 + 3(4c-2)/2

Answered by Penny Nom.
Art and Integers 2008-09-17
From pamela:
how do artists use integers?
Answered by Janice Cotcher.
The volume of a box 2008-09-16
From Andre:
volume of the box cubic centimeters

____ X _____ X ______ X =_______

The length is 8.5 Width is 3.4

Answered by Penny Nom.
Exponential form 2008-09-16
From shawn:
Write the product in exponential form: 5.5.5.2
Answered by Penny Nom.
What is the derivative of (2^sinx)/(logbase4(2x+1))? 2008-09-16
From Jesse:
What is the derivative of (2^sinx)/(logbase4(2x+1))
Answered by Harley Weston.
Limit as it Approaches a Constant 2008-09-16
From Brooks:
what is the limit as x approaches 10 for f(x) = sqrt(x-1) = 3 pleeeeeaase help me out here
Answered by Janice Cotcher.
The nth term of a sequence 2008-09-13
From lavett:
what is the Nth term in the sequence when the sequence is 2,4,8,16... and the term numbers are 1,2,3,4...
Answered by Stephen La Rocque.
Putting fractions in order 2008-09-11
From kaby:
whats the answer for this? put it from least to greatest: 1/4, 1/9, 1/8
Answered by Penny Nom.
A line through two points 2008-09-10
From Conor:
Find the equation of the straight line which passes through the points (-2,14) and (8,-1)
Answered by Penny Nom.
Angle of elevation 2008-09-09
From kristy:
A man on the tenth floor of a building shouts down to a person on the street. If the angle of elevation from the street to the man in the building is 35° and the man in the building is 40 feet up, about how far away from the building is the person on the street?
Answered by Penny Nom.
(5x - 4) / 6 = -9 2008-09-09
From Conor:
Solve to find x:

(5x - 4) / 6 = -9

Answered by Penny Nom.
Repeating fractions 2008-09-09
From Juli:
My teacher recently put my math class to the test... We were deiscussing repeating fractions and she asked us to find out what the bar over a repeating decimal is called. I found out it was called the vinculum. But she also said to find out what the number under the vinculum was called. I can't seem to find it anywhere.
Answered by Penny Nom.
The height of a tree 2008-09-09
From danice:
At a certain time of day, the angle of elevation of the sun is 30°. A tree has a shadow that is 25 feet long. Find the height of the tree to the nearest foot.
Answered by Penny Nom.
How would put .12 with 2 repeating into a fraction? 2008-09-09
From Savanna:
How would put .12 with 2 repeating in fraction?

Savanna!

Answered by Penny Nom.
0.151515...=15/99 2008-09-08
From Emma:
This week, my Algebra teacher told us about the pattern between infinitely repeating decimals and their corresponding fractions. (ex. .2222222...= 2/9, .151515...=15/99, 456456456...=456/999, etc.) I was just wondering the reason why this pattern occurs. Is there a certain element that causes this pattern to occur?
Thanks
-Emma

Answered by Penny Nom.
1/3(2x-1)=1 2008-09-07
From Jen:
1/3(2x-1)=1
Answered by Penny Nom.
A security camera 2008-09-07
From Rita:
A security camera in a neighborhhod carnival is mounted on a wall 9 feet above the floor inside a video gallery. What angle of depression should be used if the camera is to be directed to a spot 6 feet above the floor and 12 feet from the wall?
Answered by Stephen La Rocque and Harley Weston.
Simplify ((5/7)/(5/14)) + (3/4) 2008-09-06
From Emal:
How do you simplify completely: ((5/7)/(5/14)) + (3/4)?
Answered by Penny Nom.
Mathematical induction 2008-09-05
From James:
I need to prove a problem by induction regarding the Triangle Inequality. The problem is

abs(a1 + a2 +...+an) <= abs(a1) + abs(a2) +...+ abs(an).

Answered by Victoria West.
A mixed number 2008-09-04
From irena:
Need help writing this fraction as whole or mixed number 124/6
Answered by Penny Nom.
Half a circle 2008-09-04
From brandon:
what are the area and perimeter and area of half a circle??
Answered by Penny Nom.
A line parallel to y= 1/2x + 5 2008-09-03
From Michelle:
Find the equation of a line parallel to y= 1/2x + 5 if the required line passes through ( 3,4)
Answered by Penny Nom.
I need an equation that best fits these numbers. 2008-09-03
From Vallatini,:
Attached, please find a plotted curve (pdf file). I have pulled values from this curve (see below). I need an equation that best fits these numbers. Can you help?
Answered by Harley Weston.
20 golfers playing 5 rounds in 4-somes 2008-09-03
From joe:
20 golfers playing 5 rounds in 4-somes. 5 players in group A play with all 15 golfers in groups B,C,D but not with any in their own group (A). 5 players in group B play with all 15 players in groups A,C,D but not with any in their own group (B) and so on. So each 4-some has an A,B,C,&D player each of the 5 rounds.
Answered by Chris Fisher.
1,679,616 as a power of 6 2008-09-03
From Denise:
express 1,679,616 in exponential form as a power of 6.
Answered by Penny Nom.
Cows, chickens, heads and feet 2008-09-01
From marina:
Iam helping my 6th grader son solve this problem. I found similar problem in the Q&A.I understood the simple answer without using an equation. I want to solve this using the equation.This is the question about chicken and cows. There are cows and chickens in the farm. There are 65 heads and 226 legs. How many chicken are there?
Answered by Harley Weston.
How many litres of rain is this? 2008-09-01
From Todd:
If i have 10mls of rain over one square metre how many litres would I and how do I do the maths for this?
Answered by Penny Nom.
A roof angle 2008-08-30
From carla:
Use vectors to find the angle in the attached diagram
Answered by Chris Fisher and Stephen La Rocque.
A fraction word problem 2008-08-29
From Sylvia:
The denominator of a fraction is 1 less than 4 times the numerator. If the numerator is doubled and the denominator is increased by 6, the value of the resulting fraction is 2/5. Find the original fraction.
Answered by Penny Nom.
A force vector 2008-08-28
From moumita:
|F|=20N...this force is acting along a vector A=4j+3k.what is F?
Answered by Penny Nom.
How many miles of track? 2008-08-27
From Stephanie:
I have a 50 ac tract of Railroad I need to know approx how many miles of tr= ack would be included in 50 ac
Answered by Penny Nom.
An equivalence relation 2008-08-27
From Francesca:
In this problem we are told to find out if the given set is an equivalence relation or not. {(x,y) such that 4 divides x-y}. What I am confused about is whether or not for instance if I picked (1,5) as a part of the set whether I could put in (5,1) as well or whether that would be wrong. Thanks!
Answered by Victoria West.
Ordering fractions 2008-08-25
From Ghita:
Order these distances from least to greatest
1/2 , 5/8 , 3/4
What is a quotient?

Ghita

Answered by Penny Nom.
Convert Repeating Decimal into a Fraction 2008-08-25
From Craig:
Can you help me to convert: 0.444444 (repeating decimal - base 10) to base 9?
Answered by Janice Cotcher.
Hidden Faces of a Row of Cubes 2008-08-25
From Mhiko:
how many hidden faces of a cubes are there in a 5 cubes in a row? how many hidden faces of a cubes are there in a 10 cubes in a row? how many hidden faces of a cubes are there in a 15 cubes in a row? how many hidden faces of a cubes are there in a 20 cubes in a row? please show me the solution or formula....
Answered by Janice Cotcher.
A cubic equation 2008-08-25
From RAM:
The following Cubic Eqn should have three roots - what are they?

x^3-27=0

Answered by Penny Nom.
The square root of a fraction 2008-08-24
From Lauren:
How do you solve square roots of fractions? Does the format change if it is either a proper or an improper fraction?

Ex. the square root of 1/4 or the square roots of 80/25

Answered by Penny Nom.
A linear system 2008-08-23
From dipesh:
consider the system of equations
x-6y+2z=5
2x-9y-z=14
4x+12y-3z=19
by reducing the augm matrix to row echelon form, find the solution to the equation.

leaving the first two equations the third is changed to -3x+6y+14z=-31 show the equations have infinitely many solutions and give a general formula for them

Answered by Penny Nom.
Radii and Chords Create a Non-Right Triangle 2008-08-22
From Beary:
AOC is a diameter of circle O. Line AB is 12, and lines OA and OC (the radii) are 10. Find the length of line BO and chord BC.
Answered by Janice Cotcher.
Simplifying Algebraic Expressions 2008-08-22
From Jacky:
x^2-y^2+4x+4y
Answered by Penny Nom.
Function or Not? 2008-08-22
From Katie:
Determine whether y is a function of x: [(x^2)y]-(x^2)+(4y)=0 ....thx
Answered by Janice Cotcher.
Trough Filling with Water 2008-08-21
From lanny:
a triangular trough is 10 feet long, 6 feet across the top, and 3 feet deep. if water flows at the rate of 12 cubic inches per minute, find how fast the surface is rising when the water is 6 inches deep.
Answered by Janice Cotcher.
The game of 24 2008-08-20
From Tasha:
I have four numbers a 6, a 6 a 5 and a 3, i can add subtract divide or multiply these numbers in anyway but i have to use all of them and call only use them once i have to come up with a total of 24
Answered by Penny Nom.
Exterior square footage of a dome 2008-08-19
From Richard:
I have to gold leaf a dome with a circumference of 29'. What would the exterior square footage be?
Answered by Penny Nom.
Factoring x^2 + 729 2008-08-19
From peter:
hello I,am having trouble factorising a polynomial into polynomial factors with real coefficients please can you help the polynomial is x^2+729
Answered by Harley Weston.
A string around the world 2008-08-18
From Terry:
I heard this on T.V. and was wondering if it was true. Place a string around the world, you would need 25,000 miles of string. If you raised the string off of the ground by 1 foot all the way around. How much more string would you need?
Answered by Penny Nom.
Solving for Shared Height of Two Right Triangles 2008-08-17
From Heidi:
find the height of a triangle, which can be split into two right triangles, but the base (50m) is not split equally in half. one end of the base is 40 degrees, while the other is 30 degrees.
Answered by Janice Cotcher.
A triangular field 2008-08-15
From joneka:
Barb walked five times aroun a triangle field all together she walked 435 feet the first side of the triangle fiel is 23 feet the length of the other two sides are of the same measure. What are the dimensions of the triangle field?
Answered by Penny Nom.
22 cubic feet 2008-08-14
From julio:
how to measure to get 22 cubic feet, do i need the
Height
Length
width

Answered by Penny Nom.
Is zero a perfect square? 2008-08-14
From peter:
is zero a perfect square?
Answered by Penny Nom.
Coefficients, Like Terms, and Constants 2008-08-14
From Maddie:
How Would I Answer This Problem?

  Coefficients Like Terms Constants
3x+7      
4m+(-3n)+n      
6kp+9k+kp-14      
-8y+6ab+7-3ba      
c+2c+c-5c+1      


I Would Really Appreciate It If You Could Help Me
Sincerely,
Maddie

Answered by Penny Nom.
[f(x)-f(1)]/(x-1) 2008-08-14
From katie:
Evaluate (if possible) the function of the given value of the independent variable:

f(x)=(x^3)-x:

[f(x)-f(1)]/(x-1)

Answered by Penny Nom.
Percent difference 2008-08-12
From mike:
What is the percent difference between two homes if one is worth $525,000 and the other is $425,000. (525,000 - 425,000 = 100,000)
Answered by Penny Nom.
Graphing a piecewise function 2008-08-12
From Shanti:
how do you graph a step function or piecewise function such as
u^2 if u is greater than or equal to -2 and less than 1
2- u if u is greater than or equal to 1 and less than 4

Answered by Penny Nom.
BEDMAS 2008-08-12
From Rebecca:
I have 3 questions. 1) I don't really understand BEDMAS.Im going in to the 6th grade and im kind of nervis about it.? 2)When you're doing BEDMAS what does the small 3 or 2 above the other numerals mean? 3)How would you answer this:5+2 x 9 - 9 x 12= ??
Answered by Janice Cotcher.
How many decimeters are in a meter? 2008-08-11
From Julia:
How many decimeters are in a meter?
Answered by Penny Nom.
Solve for y in terms of x 2008-08-10
From Rosie:
Solve equation for y

-4x=12+3y

Answered by Penny Nom.
A linear function 2008-08-07
From Warren:
What are the conditions of the numerical coefficients of the standard form Ax+By=C in Linear Function? How will you transform y=square root of 3 x + 1 in standard form Ax+By=C?
Answered by Penny Nom.
1/X-2 + 3/X+3 = 4/X^2+X-6 2008-08-06
From Molly:
Hi, I'm beginning pre-calc this semester, and for practice over the summer, the teacher has given us a review packet with algebra 2 stuff. Well, I've forgotten most of the fraction stuff on it. Would you please help? Here's the question:

1/X-2 + 3/X+3 = 4/X^2+X-6

Answered by Penny Nom.
Arc-length and sector-angle 2008-08-06
From Benson:
If chord length, radius are given, How to find the sector angle and arc-length
Answered by Janice Cotcher.
The angle between two faces of a pyramid 2008-08-06
From Carla:
A pyramid has its vertex directly above the centre of its square base. The edges of the base are each 6cm, and the vertical height is 8cm. Find the angle between 2 adjacent slant faces.
Answered by Penny Nom.
24 in by 14 in by 14 in in cubic feet 2008-08-05
From Tom:
I have a box that has internal dimensions of: 24in. H x 14in. W x 14in. D. How many cubic feet is this internal space?
Answered by Penny Nom and Victoria West.
Find the product of 2^35 and 5^38 in sci. notation. 2008-08-03
From Peter:
I am preparing for a competition and a lot of the non-calculator problems are like find the product of 2^35 and 5^38 in sci. notation. How would you do that?
Answered by Penny Nom.
The volume of a fuel tank 2008-08-01
From G.:
I have a fuel tank that is 70 inches by 48 inches and 24 inches deep, how many imperial gallons does it hold?
Answered by Victoria West.
Product of Two-Digit Numbers 2008-07-31
From pete:
The number 4641 can be expressed as the product of two two-digit whole numbers. What is the sum of these two numbers?
Answered by Janice Cotcher.
algebra 2008-07-31
From Eric:
Would you please solve & explain this equation to me: x^2+2x=x(x+2)? Thank you
Answered by Penny Nom & Stephen La Rocque.
Limit as it Approaches Infinity 2008-07-29
From mary:
i was trying to find the limit of this problem the limit as x approches infinity of x minus cosx divided by x lim x-cosx/x x->oo
Answered by Harley Weston.
Problems understanding math 2008-07-27
From Cindy:
why do so many people have problems understanding math. is it because at some point along the way they didn't fully understand a concept and they then lose confidence. Learning math is cummulative so what you don't learn at one point just gets more confusing as you move along. how do we correct this? how can i help students in junior and senior high who do poorly in math.
Answered by Stephen La Rocque and Victoria West.
The product of the digits of a four digit number 2008-07-26
From Pete:
I am a student preparing for a competition and this was one of the prep problems: The product of the digits of a four digit number is 6x5x4x3x2x1. how many such numbers are there with this property?
Answered by Stephen La Rocque.
Proofs 2008-07-26
From Taylor:
when doing a proof, how do i figure out the steps in which i find the statements? i find the reasons pretty easily but i do not understand how to get the proving part. that would be great if you can help me! Thanks
Answered by Victoria West.
Braking Distance Relationship 2008-07-23
From Mr.Adjetey:
you are travelling at 30km/h.According to the rule of thumb,the braking distance is 9 metres when braking normally.How long is the braking distance under the same conditions when travelling at 60km/h?
Answered by Janice Cotcher.
Alpha level 2008-07-23
From anonymous:
What is the advantage of using an alpha level of .01 versus a level of .05, What is the disadvantage of using a smaller alpha level?
Answered by Janice Cotcher.
How many gallons of fuel still in the barrel? 2008-07-22
From Charles:
I have barrel 6 feet long and 3 feet diameter that is laying on it's side with 5 inches of fuel, how many gallons of fuel still in barrel
Answered by Penny Nom.
Time with Non-Uniform Motion 2008-07-21
From jenina:
An unmarked police car, travelling a constant 95km/h, is passed by a speeder travelling 145km/h. precisely 1.00s after the speed passes, the policeman steps on the accelerator. If the police car’s acceleration is 2.00m/s², how much time elapses after the police car is passed until it overtakes the speeder (assumed moving at constant speed)?
Answered by Janice Cotcher.
The number of digits in a really big exponent 2008-07-21
From Pete:
how would you find the number of digits in a really big exponent without a calculator?
Answered by Penny Nom.
Factoring a quadratic trinomial 2008-07-21
From Jeff:
c^2+4c+4

x^2-7x-18

Answered by Penny Nom.
Four Positive Integers 2008-07-20
From william:
let a, b, c and n be positive integers. If a+b+c=(19)(97) and a+n=b-n=c/n, compute the value of a.
Answered by Janice Cotcher.
Number of factors 2008-07-20
From pete:
how do you find the number of factors in a really big number without using a factoring program or listing the out?
Answered by Penny Nom.
solving four simultaneous equations 2008-07-18
From Muhammad:
-2B-2C+4E=1 A+B+C+D=0 -2B-2C-2D+E=0 B+C+4D-2E=0
Answered by Janice Cotcher.
Does the sequence 1 2 4 8 16 32 etc have a name? 2008-07-17
From Richard:
Just an idle thought really. Does the simple sequence 1 2 4 8 16 32 etc have a name?
Answered by Victoria West.
n choose r equals n-1 choose r plus n-1 choose r - 1 2008-07-14
From fae:
Prove that

( n ) = ( n – 1) + ( n - 1 )
( r )         ( r )        (r-1)

NOTE: the ( ) should be one for n taken r and so on. but there is no one big ( ) that will cater two lines

Answered by Janice Cotcher.
A reflected line segment 2008-07-14
From Peter:
Segment AB has endpoints A(-1,2) and (3,1). Segment AB is reflected over the y-axis such that A becomes A' and B becomes B'. What is the positive difference between the lengths of segment AA' and segment BB'?
Answered by Janice Cotcher.
A silo with a flat side 2008-07-14
From Amy:
Without using the Pythagorean Theorem, determine the capacity of a silo in cubic feet of grain if: the cylinder-shaped silo has one flat, rectangular face that rests against the side of the barn; the height of the silo is 30 feet and the face resting against the barn is 10 feet wide; the barn is approximately 5 feet from the center of the silo.
Answered by Harley Weston.
Express as a function of x 2008-07-14
From Rita:
Express the area A of an isosceles right triangle as a function of x of one of the two equal sides.

NOTE: I always see the phrase "Express as a function of x" in math textbooks.
What exactly does that phrase mean?
I also have seen it written as "Express in terms of x."

Answered by Penny Nom.
A Falling Rock 2008-07-11
From Rita:
This question has 3 parts. If a rock falls from a height of 20 meters on the planet Jupiter, its height H (in meters) after x seconds is about H(x) = 20 - 13x^2 (a) What is the height of the rock when x = 1.2 seconds? (b) When is the height of the rock 15 meters? (c) When does the rock strike the ground? Thank you
Answered by Janice Cotcher.
A difference quotient 2008-07-10
From Rita:
Find the difference quotient of f, that is, find [f (x + h) - f (x)]/h, where h does not = 0 for the given function. Be sure to simplify.

f(x) = 1/(x + 3)

Answered by Janice Cotcher.
Egyptian fractions 2008-07-09
From vivek:
An Egyptian fraction has numerator equal to 1, and its denominator is a positive integer. What is the maximum number of different egyptian fractions such that their sum is equal to 1, and their denominator are equal to 10 or less?
Answered by Janice Cotcher.
A four sided lot 2008-07-09
From michelle:
the front is 65.29
left side 162.26
the back 155.00
right side 101.01

right angle=50.00
left angle= 153.30

Answered by Harley Weston.
Profit as a percentage 2008-07-08
From ANNORIS:
Question from annoris, a student:

How to calculate percent of profits using step by step division formula.
I know how to get the profit.
Selling cost was $15.00
Cost amount was $10.00
Total profits its $ 5.00.
But i do not know how to get the percent because i dont know how to divide to well . Can you helppppppp me ?

Answered by Penny Nom.
Pounds per cubic foot 2008-07-08
From Peter:
A column has a diameter of 42 feet, a height of 40 feet, and weighs 15 tons. What is the mean number of pounds per cubic foot?
Answered by Penny Nom.
A net profit of 4% on a sales volume of $100 million 2008-07-07
From Muriel:
If a firm has a net profit of 4% on a sales volume of $100 million and the company has instituted logistics programs that result in cost savings of $1 million, what equivalent sales increase would be required to equal such a cost savings?

What I do know is that the net profit % has increased from 4% to 5% and the profit has gone from $4,000,000 to $5,000,000 but I have no idea what the formula is to figure out how much sales would have to increase to result in that same profit.

Answered by Janice Cotcher.
Composition of two functions 2008-07-04
From Kristian:
f(x)= the square of x add to 1 and g(x)=1/x

find: (f o g)x

Answered by Penny Nom.
Sum and product of the roots of a quadratic 2008-07-03
From Gautam:
If a and b are the roots of x^2+px+1=0 and c and d are the roots of x^2+qx+1=0
prove that
(a-c)(b-c)(a+d)(b+d)=q^2-p^2
Regards
Gautam

Answered by Harley Weston.
Exponential form 2008-07-03
From Kent:
How to write 8 365 005 106 in expanded and exponential form? May I request you also to explane how does it solve.
Answered by Stephen La Rocque.
A series solution of y' = xy 2008-07-03
From sasha:
I've to find the power series solution of the differential equation: y' = xy.
I don't know how to find the recursive equation. Can you please help me. Thanks

Answered by Harley Weston.
What is the largest perfect square factor of 1512? 2008-07-02
From peter:
What is the largest perfect square factor of 1512?
Answered by Penny Nom.
The sum of the digits of two-digit number is 9 2008-07-01
From Audrey:
the sum of the digits of two-digit number is 9. if the digits are reversed, the new number if 63 greater than the original number. find the number.
Answered by Penny Nom.
The square root of (18*n*34) 2008-07-01
From Peter:
What is the least possible positive integer-value of n such that square root(18*n*34) is an integer?
Answered by Penny Nom.
Fish in a pond 2008-06-28
From Peter:
At the beginning of a study 24 fish are caught, tagged, and released back into a large pond containing fish. A few days later, 19 fish are caught from the pond and three of them have tags. Using this ratio, how many total fish would you expect to be living in the pond? The answer is 152. Why is that?
Answered by Leeanne Boehm.
Differentiate y= (x^x^x)^x 2008-06-27
From emril:
Differentiate y= (x^x^x)^x
Answered by Harley Weston.
How many positive integer factors of 2^2*3^2*5 are multiples of 12? 2008-06-27
From Peter:
How many positive integer factors of 2^2*3^2*5 are multiples of 12
Answered by Janice Cotcher.
Write log_2 (1/8) = -3 in exponential form 2008-06-27
From Jaime:
how do you write log2 (1/8) = -3 in exponential form?

the 1/8 is a fraction and the 2 is a base of 2

Answered by Penny Nom.
Four fours 2008-06-25
From jen:
Use four 4's and some of the symbols: +,-,X, / and (), to give expressions for the whole numbers from 0 through9. Example : (4X4+4)/4=5
Answered by Janice Cotcher.
A golf tournament 2008-06-24
From Roland:
We have a golf tournament, 12 players, three rounds. We want to pair so that we have pairings with as few duplications as possible.
Answered by Janice Cotcher.
Slope and rate of change 2008-06-23
From Lee:
What is the difference between a slope and a rate of change?
Answered by Stephen La Rocque.
111...111 * 2003 2008-06-23
From Peggy:
Number a = 111...111 formed by 2003 1's. Find the sum of the digits of the number 2003 * a.
Answered by Penny Nom.
The sum of the digits of a number 2008-06-23
From Ben:
Question: Using mathematical induction, prove that if the sum of the digits of a number is divisible by three, then the number itself is also divisible by 3.
Answered by Penny Nom.
How many face does a cone have? 2008-06-21
From Vanessa:
How many face does a cone have?
Answered by Harley Weston.
Two forces acting on a body 2008-06-21
From Rita:
Before answering the question below, what exactly is, in basic words, resultant force? QUESTION: Two forces of 50 and 68 pounds act on a body to produce a resultant force of 70 pounds. Find, to the nearest 10 minutes or nearest tenth of a degree, the angle formed between the resultant force and the smaller force.
Answered by Harley Weston.
30% profit 2008-06-19
From Julie:
how do i calculate the price per person that will return a 30% profit on the sales revenue generated?
Answered by Penny Nom.
Cows and chickens 2008-06-16
From damebochie:
I got this problem and want to write a system of equation for it:
Farmer Joe has cows and chickens on his farm. One day he count 76 legs and 24 heads. How many cows and how many chickens are on the farm? write a system of equations and solve.
I don't have any problem to solve it, but I just can;t see how I am going to write this system of equation. Thank you so much for helping?

Answered by Harley Weston.
Percent difference? 2008-06-14
From Cathy:
I am trying to verify a "percent of difference" as stated on a tax form. I cannot seem to come up with the equation to do so.
Here are a few examples on the form:
Last year $62.91 this year $66.05 tax difference $3.14 percent difference -5.64%
Last year $ 2014.97 this year $1982.93 tax difference $-32.04 percent difference 57.56%
Last year $97.66 this year $79.58 tax difference $-18.08 percent difference 32.48%
I have tried several different ways and per some of Q&A's already on your site I still have a problem and would appreciate some assistance.
Thanks-Cathy

Answered by Penny Nom.
A right prism with a square base 2008-06-13
From Malik:
Question from Malik: A company wants to alter the dimension of container. The new container is in the shape of right prism equivalent to the current one. The current container has height of 12cm and square base. The new container has also square base, but in the dimension base is increased by 3cm and height is decreased by 5cm. if the cardboard is used to manufacture the new container, what area of cardboard is used to make new container?
Answered by Penny Nom.
-2(x - 1)^2 + 8 2008-06-12
From kasondra:
y=-2(x-1)2[thats a square] +8 that need to be put in standard form
the answer is y= -2x2 -4x + 6
i got y= -2x2 -4x -4
i just don't get how my teacher got that answer

Answered by Penny Nom.
The rate of change in the depth of the water 2008-06-12
From Liz:
A rectangular pool 50ft long and 30ft. wide has a depth of 8 ft. for the first 20 ft. for its length and a depth of 3 ft. on the last 20ft. of its length and tapers linearly for the 10 ft in the middle of its length. the pool is being filled with water at the rate of 3ftcubed/ min at what rate is the depth of the water in the pool increasing after 15 hours?
Answered by Harley Weston.
Scientific notation 2008-06-11
From ASTRID:
how can you solve the following problem...
the mean distance between the sun & plut0 is about 3,700,000,000 miles. write this distance in scientific notation?

Answered by Penny Nom.
x=(-14x+16)/x-8 2008-06-11
From Kory:
Solve the equation.
x=(-14x+16)/x-8

Answered by Penny Nom.
Which fractions should go next? 2008-06-11
From Sharon:
Please help me with these fractions
1/4, 2/3, ___, 5/6
which fractions should go next
A.1/2
B.1/3
C.3/4
D.11/12

Answered by Janice Cotcher.
The surface area of a lemon 2008-06-11
From Susan:
How would I find the surface area of a lemon
Answered by Harley Weston.
What is 6 over 8 as a percentage? 2008-06-11
From stephen:
what is 6 over 8 as a percentage?
Answered by Penny Nom.
What is the GST component? 2008-06-11
From Jilly:
I need to find a quick and easy formula for finding the gst (10%) of the total amount in $ terms, eg: the total is $500 what is the gst component?
Answered by Penny Nom.
Packing a truck 2008-06-10
From Ann:
I am renting a 24ft box truck and have been told it is 1401 cubic feet. I want to know if I have 23 display cases to move and each is 6ft X 2ft X 3ft will the all fit? Or do I need additional info in order to figure this out?
Answered by Penny Nom.
How many different ways can cards fall when shuffling? 2008-06-09
From Twyla:
How many different ways can cards fall when shuffling? In other words, if you have 52 numbers, each one unique, how many different ways can they be arranged, using each number only once?
Answered by Penny Nom.
All wiffs are miffs and all miffs are kiffs. 2008-06-08
From Allison:
All wiffs are miffs. all miffs are kiffs. There are 25 wiffs. There are 76 kiffs. 33 kiffs are not miffs. How many miffs are not wiffs?
Answered by Penny Nom.
g(m-1,2n)+n 2008-06-07
From Florence:
What does g(m-1,2n)+n for m>0, n>=0 mean.
Answered by Penny Nom.
90% of gross leaseable footage 2008-06-07
From Rosalind:
90% of gross leaseable footage in the shopping center, being 394,577, excluding that square footage leased by major anchor tenants, being 284,747. Would you deduct the 284,747 from the 394,577, then multiply by the 90%. Or would you multiply the 394,577 by the 90%, then deduct the 284,747?
Answered by Penny Nom.
The mean and variance 2008-06-05
From Donny:
An investment will be worth $1,000, $2,000, or $5,000 at the end of the year. The probabilities of these values are .25, .60, and .15, respectively. Determine the mean and variance of the worth of the investment.
Answered by Harley Weston.
A recursive formula for 9, -18, 36, -27, ... 2008-06-05
From Rita:
I am having trouble understanding how to write formulas (I should say create formulas) from a given sequence. It does not matter what sort of sequence it is. The confusion remains. I have not found a textbook or easy math book that explains this process for the average Joe to grasp. Here are the two questions:

(1) Write a recursive formula for the sequence
9, -18, 36, -72, ...

(2) Write a recursive formula for the sequence
3, 3(sqrt{3}), 9, 9(sqrt{3})

Answered by Penny Nom.
The number of hours of daylight 2008-06-03
From Marilyn:
Hi, could you please help me with this question? In a city (in the Northern Hemisphere) the minimum number of hours of daylight is 9.6 and the maximum number is 14.4. If the 80th day of the year (March 21) has 12 hours of daylight, determine a sine function which gives the number of hours of daylight for any given day of the year. (Jan 1 = 1, Jan 2 = 2, etc). Thank you!
Answered by Harley Weston.
4x^3 - 6x + 2 = 0 2008-05-28
From paul:
4x^3 - 6x + 2 = 0
Answered by Janice Cotcher.
The surface area of an "egg shaped" object 2008-05-28
From Tom:
How do you calculated the surface area of an "egg shaped" object (ovoid) ?
Answered by Harley Weston.
Scale factor 2008-05-28
From Riley:
I have a math problem, and a big test coming up on Friday or Thursday (my teacher hasn't decided yet.) I need help understanding how I would do this problem. Here it is: they wanted me to find the perimeter and area of a rectangle when they only give me the scale factor. I've delt with scale factors before, but this is quite confusing. I have done the scale factor way by having a rectangle that maybe has a width of 10 and a height of 20, and I needed to find what the scale factor would be for a rectangle with the width of 5 and a height of 10. I know that they scale factor would be 2, right? But this just gives me the scale factor, and I have to FIND the perimeter and area. Hopefully you can get back to me tonight.
Answered by Harley Weston.
The area of a rectangle 2008-05-27
From Becky:
the area of a rectangular lot is represented by 6t^2+13t-5. What are the length and the width of the rectangular lot?
Answered by Harley Weston.
.5 cu ft of sand to lbs. 2008-05-25
From rogersimmons:
needtocovert .5 cu ft. of sand into lbs.
Answered by Penny Nom.
I want to fill my backyard slope with fill dirt. 2008-05-25
From Piero:
I want to fill my backyard slope with fill dirt. The slope is 50 feet wide and the distance from the top of the top of the slope to the bottom is about 15 feet deep and the slope angle is at a 45. I want to know how many yards of dirt do I need to fill a space that is 20 feet out, 15 feet deep at a 45 degree angle.
Answered by Stephen La Rocque.
Applications of trigonometry 2008-05-24
From Mohita:
I have got a project in the school and i am not getting anything about the topic. The topic is that we need to find the application of trigonometry on any one of the real life situations using 3-dimensional figures. I mean how can trigonometry can be used in real life situations like navigation, architecture, survey, astronomy etc.
Answered by Penny Nom.
3 radical 48 +11 radical 75 2008-05-24
From Maria:
Add 3 radical 48 +11 radical 75. Write answer in simpliest form.
Answered by Penny Nom.
The surface area of a cube 2008-05-24
From cailtlin:
hi.. if the surface area of a cube is 384 cm^2, how could i find the length of one edge? thank you
Answered by Penny Nom.
Prime factors 2008-05-21
From john:
the first 5 prime numbers are 2,3,5,7,11. which is the smallest number that has these prime factors.
Answered by Stephen La Rocque.
1+1/1+1/x 2008-05-21
From nadean:
1+1/1+1/x
Answered by Penny Nom.
Percentage difference 2008-05-20
From Cavell:
Hi, the projected cost was 71,700. The actual cost was 56,876. What is the percent difference?
Answered by Penny Nom.
The cube of -2y in exponential form 2008-05-19
From Xiu:
I dont get this question.. Write "the cube of -2y" in exponential form. I know the answer is -(2y)^2 but i really don't understand why. Can you explain how to do this whole problem for me?
Answered by Leeanne Boehm.
GCF 2008-05-19
From Brenda:
what is GCF????? =c
Answered by Leeanne Boehm.
Converting a decimal to a fraction 2008-05-19
From judy:
please explain how to get from the decimal number .3437 to the fraction 11/32
Answered by Leeanne Boehm.
Guy wires for a tower 2008-05-19
From larissa:
a radio tower 500 feet high is located on the side of a hill ( the hill has an inclination to the horizontal of 5 degrees.) How long should two guy wires be if they are connected to the top of the tower and are secured at two points 100 feet directly above ( up the hill ) and directly below the base of the tower?
Answered by Penny Nom.
1/2 of infinity 2008-05-16
From Pamela:
Hello. My question was posed to me by my husband, who says he knows the answer and that there is an answer....I have tried to research it myself but to no avail. Here is the question, as he posed it to me. ?? = 1/2 of infinity.
Answered by Stephen La Rocque, Penny Nom and Claude Tardif.
Fencing around a playground 2008-05-15
From Vanessa:
If the playground is a rectangle with it's width 10ft shorter than it's length, and it can be enclosed by exactly 140ft of fence, what is the length and width of the playground?
Answered by Penny Nom.
Selling price 2008-05-10
From Roxane:
I'm trying to figure out the formula for the following, I have a product that I want to receive a certain profit margin, but with this product I also have to pay a percentage of commission to someone. I need to know how to calculate my new selling price taking into account the markup percentage, but I don't want my gross margin to be lowered. I have calculated the markup percentage to get my new selling price with the commission, but it lowers my profit margin once I pay the commission and I don't want it to do that.
Answered by Penny Nom.
The surface area of a tank 2008-05-09
From jack:
i'm looking for the square footage of a tank that is 10'diameter x 23'high,floor,ceiling and walls
Answered by Penny Nom.
The factor theorem 2008-05-06
From Nomaan:
factorise
x³-2x²-x+2
using factor theorem

Answered by Penny Nom.
Factor ac^2 - a^3 - a^2b + ab^2 + b^3 - bc^2 2008-05-05
From erson:
factorize:
ac^2 - a^3 - a^2b + ab^2 + b^3 - bc^2

Answered by Harley Weston.
Identify the graph 2008-05-05
From A parent:
Refer to the graph given below and identify the graph that represents the corresponding function. Justify your answer.
y = 2x
y = log2x
I can figure out what line on the graph is which equation but unsure what is meant by justify your answer.

Answered by Penny Nom.
How many presses should be used? 2008-05-04
From Sarah:
Hi! I am in Calculus and this problem is on my study guide and i just cant figure it out!? A printing company had eight presses, each of which can print 300 copies per hour. It costs $5.00 to set up each press for a run and 12.5+6n dollars to run n presses for an hour. How many presses should be used to print 6000 copies most profitably? Let h equal the number of hours used to print the 6000 copies.
Answered by Harley Weston.
Zero divided by infinity 2008-05-03
From ANNE:
There was a question about what do you get when you divide Zero by infinity. There was an example using Potatoes. Could someone please explain a little bit more in detail, so that I can help my son who has Schizophrenia understand. He is big into Mathematics and is consumed by this question. Thankyou so much, Anne
Answered by Penny Nom.
The inverse of a function 2008-05-03
From keith:
please help me find the inverse of this function:

h(t) = 2 + 4 ln(1-5t)

Answered by Stephen La Rocque.
Surface area 2008-05-01
From John:
I am working with concrete and need some assistance please. I am trying to figure out what the surface space is for a one cubic foot that is four inches thick?
Answered by Penny Nom.
The standard form of an ellipse 2008-04-30
From Rebecca:
I have to write the following equation into standard form of an ellipse:

9(X-1)^2 + (Y+1)^2 = 1

Answered by Stephen La Rocque and Harley Weston.
(9x^2-4)/(3x^2-5x+2) * (9x^4 - 6x^3 +4x^2)/(27x^4+8x) 2008-04-29
From Jonathon:
I don't remember how to solve an equation in the form of the following:

(9x^2-4)/(3x^2-5x+2) * (9x^4 - 6x^3 +4x^2)/(27x^4+8x) =

Answered by Stephen La Rocque.
The ratio of their perimeters to two regular hexagons 2008-04-29
From td:
Two regular hexagons have areas of 54 square root of 3 and 150 square root of 3, what is the ratio of their perimeters?
Answered by Stephen La Rocque.
Factor and solve s1^2 - 10s1 - 200 = 0 2008-04-29
From KIM:
please factor and solve s1^2 - 10s1 - 200 = 0
Answered by Stephen La Rocque and Penny Nom.
Adding rational functions 2008-04-29
From Jonathon:
1/(x+3) + 1/(x^2+5x+6) =
Answered by Penny Nom.
A bowl is the shape of a hemisphere 2008-04-28
From josh:
a bowl is the shape of a hemisphere with diameter 30 cm and water is poured into the bowl to a height h cm. how do i find the volume of the water in the bowl
Answered by Harley Weston.
An octagonal prism 2008-04-27
From Melanie:
My son is identifying geometric shapes in the real world? We are stuck on octagonal prism, rectangular prism and square prism. Can you help me out with some examples. Thanks
Answered by Penny Nom.
The sum of the squares of the fibonacci numbers 2008-04-27
From Thomas:
Hey I have a question for a research topic that our teacher set us, It is regarding the sum of the squares of the fibonacci numbers.

The question says describe the pattern that exists and write a general formula that describes the relationship illustrated above. I can see the pattern that is occurring but i cannot put this into a general formula. Any help would be appreciated. Thanks Tom

Answered by Victoria West.
A circle inscribed in a triangle 2008-04-26
From Amar:
I have been given a circle inscribed in a triangle and have been told to prove that the ratio of the perimeter of the triangle to the circumference of the circle is the same as the ratio of the area of the triangle to the area of the circle. How would this be done?
Answered by Stephen La Rocque and Walter Whiteley.
3-3+3-3+3.........up to infinite terms = ? 2008-04-25
From Jatin:
3-3+3-3+3.........up to infinite terms = ?
Answered by Stephen La Rocque.
Naming a figure 2008-04-24
From P.:
What is the name of a figure that has 2 sides that measure 25 feet each and 2 sides that measure 10 feet each?
Answered by Penny Nom.
A volume of revolution 2008-04-24
From Sabahat:
Hi, i have a region enclosed by both axes, the line x=2 and the curve y=1/8 x2 + 2 is rotated about the y-axis to form a solid . How can i find the volume of this solid?. (Please note that y equation is read as y =1 over 8 times x square plus 2.) I will be really grateful if you answer this question. :)
Answered by Harley Weston.
Why does -2^2 = -4 when -2 * -2 = 4? 2008-04-22
From blaine:
Why does -2^2 = -4 when -2 * -2 = 4?
Answered by Penny Nom.
f(x)=sin^3(3x^2) find f ' (x) 2008-04-21
From Michael:
f(x)=sin^3(3x^2) find f ' (x)
Answered by Harley Weston.
The perimeter of a half acre square 2008-04-20
From s:
Measured in feet. What would be the perimeter of a square area of land measuring 1/2 acre?
Answered by Penny Nom.
Ratios and fractions 2008-04-20
From Alissa:
What is the formula for writing the ration of two numbers as a fraction?
Answered by Leeanne Boehm.
WHY 144 where does this figure come from 2008-04-19
From Sean:
Why do you use the number 144 to divide

EG. length of timber 2 inches x 8 inches x 14 foot = 1.5 cubic feet

Calculation is 2x8x14 =224 divide by 144 = 1.5 WHY 144 where does this figure come from

Answered by Leeanne Boehm.
The three sides of a triangle 2008-04-16
From Bridgett:
The sum of the lengths of any 2 sides of a triangle must be greater than the third side. If the triangle has one side that is 11 CM, and the second side of the triangle is 4 cm less than twice the third side, what lengths do the 2nd and 3rd side have to be?

between 0000-00-00 and 9999-99-99


Answered by Stephen La Rocque.
A quadratic equation with odd coefficients 2008-04-15
From Manashi:
If p,q,r be any odd integers, then prove that the roots of the quadratic equation px^2+qx+r=0 can't be rational .
Answered by Stephen La Rocque.
What is 300% as a fraction? 2008-04-14
From Carli:
what is 300% as a fraction
Answered by Stephen La Rocque.
A 13 cubic foot cylinder filled with sand 2008-04-12
From Thomas:
I have a 100 pound bag of sand. I need to know how much sand I will need to fill a 13 cubic foot cylinder
Answered by Harley Weston.
The slope intercept form 2008-04-11
From Dave:
Hi, why is there a y-intercept in the slope intercept form?What does the y-intecept have to do with the equation?
Answered by Penny Nom.
Two function problems 2008-04-10
From keith:
find the domain: F(x)= 1/ {ln (x+5) - ln (7-x)}

find the inverse: g(t)= sqrt {2-3 ln (1-t)}

Answered by Penny Nom.
The chain rule 2008-04-10
From joey:
pls help me to Differentiate
y=(3x^2-4x)^8

Answered by Harley Weston.
The game of 24 2008-04-10
From sherwin:
ok is like this you must get 24 using 1,5,5,5 but you can only use +, - , / and *
Answered by Claude Tardif.
The zeros of a cubic 2008-04-09
From Ryan:
what are all the zeros of f(x)= x^3-1/2x^2+1/3x-1/6
Answered by Penny Nom.
cosec [arccos (1/y)] 2008-04-09
From Tom:
I am having trouble with this question. For y > 1, the value of cosec [arccos (1/y)] is?
Answered by Penny Nom.
How many bags of mulch? 2008-04-07
From MJ:
I have an area that is 35' X 50' and I want to cover it with 2" of mulch.

Do I take length X Width X depth? Bags of mulch are sold in 2 or 3 cubic feet per bag.....I just want to know how to do the math. can you please send me the equation?

Answered by Penny Nom.
What fraction of 45 is 10? 2008-04-06
From Scott:
45 x fraction? = 10
Answered by Penny Nom.
A volume of revolution 2008-04-04
From ted:
Consider the region bounded by y=x^2 + 1, y=5-3x and y=5. Sketch and shade the given region; then set up but dont evaluate teh integrals to find the following:

a) The volume of the solid generated by rotating the region about the line y=5

b) the volume of the solid generated by rotating the region about the y-axis

Answered by Penny Nom.
A shape 2008-04-03
From Joshua:
what shape has 4 faces, 4 corners and 9 edges.
Answered by Penny Nom.
Factor 2x^3 + 3x^2 - 32x + 15 2008-04-02
From sarah:
I have a course review I'm working on and I'm not sure how I'm supposed to factor the following 2x^3 + 3x^2 - 32x + 15.
Answered by Penny Nom.
The number of positive integer factors 2008-04-02
From lisa:
If n is a positive integer such that 5n has 36 positive factors, and 3n has 40 positive factors: How many positive factors can 15n have? How many values are possible for n?
Answered by Penny Nom.
The number of faces and vertices in a pyramid 2008-04-02
From Laurie:
Can you find a connection between the number of faces and vertices in a pyramid and the number of sides of it's base???
Answered by Penny Nom.
Common fraction to decimal fraction 2008-04-01
From chadwick:
how do I turn 3/32 into a decimal
Answered by Penny Nom.
-2 ^ 2 + (6 - 9) / (-3) + 4 * (-2) 2008-04-01
From Jeth:
How do I simplify: -2 ^ 2 + (6 - 9) / (-3) + 4 * (-2)
Answered by Penny Nom.
Square feet and square yards 2008-04-01
From Victor:
6200 sq. ft.into yards
Answered by Penny Nom.
The game of 24 2008-04-01
From Frank:
Hi, my name is Frank. I have been really stumped on this 24-card. It has the 4 # of 21,3,5,10. If you have any suggestions, I would be grateful!
Answered by Claude Tardif.
A rectangular prism 2008-03-31
From Chanlie:
i need need to find a rectangular prism with a total surface area of 210 units.
Answered by Penny Nom.
How many bags of cement do I need 2008-03-30
From Joe:
How many bags of cement do I need if I want to extend my carport 6 inches all around? The carport is 15ft. x 15ft. and the existing slab is 4 inches thick.
Answered by Harley Weston.
The exponential form of x radical y to the third power 2008-03-30
From Jennifer:
what is the exponential for of x radical y to the third power
Answered by Penny Nom.
A fish tank in the shape of an irregular pentagon 2008-03-29
From richie:
i am building a fish tank. it is going to be an irregular pentagon. the sides are going to be
24"
24"
8"
8"
32"(approximately)

there will 3 right angles A, B, E

my question is how to figure out the degree of the angles that are not right angles (C,D)?

Answered by Chris Fisher.
The average rate of change 2008-03-29
From Tom:
For the function x/3x-1 find the average rate of change between the interval x=1 and x=5?
Answered by Harley Weston.
A car tire full of concrete 2008-03-27
From robert:
I want to build a volleyball net support. I am using a car tire 24"odx16"id filled with concrete. how much will this weigh? thanks
Answered by Penny Nom.
Operation on the mixed numbers 2008-03-26
From bradley:
1) Perform the indicated operation on the mixed numbers below; write answer in simplest form:
8 1/3 – 2 1/4

2) Perform the indicated operations on the mixed numbers below; write answer in simplest form; note;: "•" denotes multiplication:
2 1/2 • 3 2/3 • 5 3

Answered by Stephen La Rocque.
A function satisfying f(x) + 2f(1/x) = x 2008-03-25
From Joan:
Let f be a function satisfying f(x) + 2f(1/x) =x for all real numbers (x does not equal 0)
a) Find f(1) justify your answer

b) Find f(2) justify your answer

Answered by Penny Nom.
The length of the ramp on a motorcycle trailer 2008-03-25
From Joshua:
I am currently building a motorcycle trailer. I am trying to figure out the length of the ramp so that the bikes don't scrap the trailer/ramp as they are loaded. This is the info I have: the motorcycle is 6" off the ground in the center, the point where the tires touch the ground are 80" apart, the trailer deck is 20" high. How do I figure the length of the ramp? Please show equation so I have for future reference with different measurements.
Answered by Stephen La Rocque and Harley Weston.
The decimal equivalent of a fraction 2008-03-24
From Mario:
What is the decimal equivalent of the fractions 5/8?
Answered by Penny Nom.
(x-3)/(x-cube+6x-square-25x-150) 2008-03-24
From rafia:
(x-3)/(x-cube+6x-square-25x-150)
Answered by Penny Nom.
A hydraulic cylinder 2008-03-24
From james:
I am trying to adjust the placement of a hydraulic cylinder that raises a dump bed up from the frame of a truck. How long would the cylinder (height of a triangle) have to be to raise the bed to a 70 degree angle? The base from pivot to cylinder is 132.5 inches.
Answered by Stephen La Rocque.
The volume of a footing 2008-03-23
From Brett:
You need to pour an 8"x16" footing that is 100ft. long. At a cost of $65 per cubic yard, how much will this cost?

(A) $205 or less
(B) more than $205, but less than $220
(C) $220 to $235
(D) more than $235

Answered by Penny Nom.
How would I factor 2x^3 + 11x^2 - 7x - 6? 2008-03-22
From Sean:
How would I factor this polynomial? 2x^3 + 11x^2 - 7x - 6
Answered by Stephen La Rocque.
The radius of a circle 2008-03-22
From danny:
waht is the radius of a circle, if the circumference is 800?
Answered by Penny Nom.
How would I graph: x - 2 divided by x^2 - 4? 2008-03-19
From Sean:
How would I graph: x - 2 divided by x^2 - 4 using a method in extreme values of functions or completing the square

x - 2/ x^2-4

Answered by Stephen La Rocque.
The product of the roots 2008-03-19
From Rebecca:
Determine the product of the roots of the equation (x-1)(x-2) + (x-2)(x-5)=0
Answered by Penny Nom.
A pen with 40 post on each side 2008-03-18
From Manpreet:
how many post do you need to make a pen with 40 post on each side
Answered by Penny Nom.
Graph the equation P(x)= x^3 + x^2 -12x 2008-03-18
From Sean:
I want to graph the equation : P(x)= x^3 + x^2 -12x I was thinking that I would combine the x-values, but then I could not use the complete the square technique. Would I have to just solve the second half of the equation and then graph x^3 seperately
Answered by Penny Nom.
Twenty golfers 2008-03-16
From john:
I have a group of 20 golfers. We plan on playing 6 rounds of golf in 4 days. I would like to have the most possible combinations of players without each player playing with anyone if possible, in the 4 days of golf
Answered by Victoria West.
The sum and product of the roots of a quadratic 2008-03-16
From Katelyn:
Write the quadratic equation....

Sum of the roots = 16
Product of the roots = -80

Answered by Penny Nom.
The surface area of a cube and a cylinder 2008-03-16
From sunday:
I'm doing a unit assessment, and there is something I don't understand. Now in my textbook, it says that surface area for a cube is different from a cylinder. Surface area for a cylinder has two numbers, but surface area for a cube has 3 or more numbers. And plus, I can't figure out the volume for a cube because it only shows volume for a cylinder. So how do I get this right?
Answered by Harley Weston.
Mathematics in cooking 2008-03-15
From Emily:
What forms of Mathematics are used in Cooking?
Answered by Harley Weston.
1, 1, 2, 3, 5, 8, 13, ___ ___ ___ 2008-03-13
From Karen:
What is the rule for the following pattern, and what would come next?

1, 1, 2, 3, 5, 8, 13, ___ ___ ___

Answered by Penny Nom.
Inverses of functions 2008-03-13
From jess:
if g(x)=2x+3 find g^-1(g(4)) how do I sovle I cant's figure out what to do with the 4
Answered by Stephen La Rocque.
A dilation project 2008-03-12
From john:
I am working on a dilation project. How do I dilate an irregular octagon on a scale of 3.5
Answered by Stephen La Rocque.
Factor x^6-5x^3+25 2008-03-11
From jacob:
how do you factor equations for algebra 2
x^6-5x^3+25

Answered by Harley Weston.
A width-to-length ratio of 2 : 3 2008-03-11
From Stephanie:
The national flag of France has a width-to-length ratio of 2 : 3. What is the width of an 18-inch long French flag?
Answered by Penny Nom.
A linear system 2008-03-10
From sandhiya:
one group of people purchased 10 hotdogs and 5 soft drinks at a cost of $12.50. a second group bought 7 hotdogs and 4 soft drinks a ta cost of $9.00 find the cost of a single hot and a single soft drink.(using the row echelon form to solve)
Answered by Penny Nom.
Repeating decimals 2008-03-10
From Blaine:
Is it possible to put a repeating decimal number into a calculator to solve a problem?

EX: Write 39.3939... as a fraction.

Answered by Penny Nom.
Saving to buy a coat 2008-03-09
From alayna:
Jackie is saving her money to buy a coat that costs $121. If she already has $61 and saves $15 each week, in how many weeks can she buy the coat ? Make a function table to answer the question.
Answered by Penny Nom.
The volume of a countertop 2008-03-08
From Matthew:
I need to calculate volume down to cubic feet. I pour concrete countertops and cant figure out the formula to convert inches and feet. If I have a 2.5 inch by 2 foot by 16 foot concrete countertop how do I convert it to cubic feet and hence know how many bags of concrete at .6 cubic feet per bag I will need. I am trying to input the formula into a spreadsheet. thank you so much Matt
Answered by Penny Nom.
Composition of functions 2008-03-07
From sharon:
Find FoG(x)
F(x)= the square root of x-9
G(x)= x^2
and also find GoF and their domains

Answered by Penny Nom.
Cubic feet to liters 2008-03-06
From chase:
if 1cu.ft=6.229 imp. gal and1 imp.gal=4.546 liters how many liters does of 128cuft. hold
Answered by Penny Nom.
The dimensions of a rectangle 2008-03-06
From Julie:
Find the dimensions of a rectangle with an area of 400. Width x ft. Length (2x-7)ft.
Answered by Stephen La Rocque and Penny Nom.
Parametric equations 2008-03-05
From kumi:
I have made a picture of a football pass 45 degrees with 30yds and height of 7ft and 4ft. But I am just stuck there. I have stared at the worksheet for soo long. I have no idea how to solve this, and I don't even know where this is going (I don't understand how parametric equations work). Can you help me with this problem?

1.The quarterback of a football team releases a pass at a height, h, of 7ft above the playing field, and the football is caught by a reciever at a height of 4ft, 30yds directly downfield. The pass is released at an angle of 45 degrees with the horizontal with an initial velocity of vo. The parametric equations for the position of the football at time t are given, in general, by x(t)=(vo cosθ)t and y=h+(vo sinθ)t-16t^2.

a. find the initial velocity of the football when it is released
b. write the specific set of parametric equations for the path of the football
c.use a graphing calculator to graph the path of the football and approximate its maximum height
d. find the time the reciever has to position himself after the quarterback releases the football

Answered by Harley Weston.
How large is a container that will hold 3.5 cu meters? 2008-03-05
From Robert:
If a company allows you to ship personal belongings while on an over seas assignment and the figure is 3.5 cubic meters. Just how much is the total area in square feet?? 10X10X10 or??? I just wanted to understand the size of a container that would hold 3.5 cubic meters!!
Answered by Harley Weston.
A roll of paper 2008-03-05
From Sheik:
How I Convert My Roll Weight Into Meters?
Roll Width : 241mm
Thickness : 56gsm
Weight :81 Kg

Answered by Harley Weston.
Adding feet and inches 2008-03-04
From mike:
3.25'+3.25"
Answered by Penny Nom.
What was the ostrich's speed in miles per hour? 2008-03-03
From jacob:
If it took an ostrich 2.5 seconds to travel 110 feet and the average speed is 44 feet/ seconds, what was the ostrich's speed in miles per hour?
Answered by Penny Nom.
The diameter of a circle 2008-03-03
From Kathie:
I know the length of the curved portion of a semi-circle is 200 ft. but I need to find the diameter to get a total perimeter and area. How do I find the diameter to get the perimeter?
Answered by Penny Nom.
Time and a half 2008-03-01
From Travis:
Bill receives time and a half for working overtime. If Bill is paid $55 for each hour of overtime, how much is his regular hourly wage?
Answered by Stephen La Rocque.
The profit was what percent of the cost? 2008-02-29
From sierra:
The profit was what percent of the cost?
Here is some more of that whole problem.
Selling Price-$26.60
Discount-$6.65
Sale Price after the Discount-$19.95
The Profit-$5.95

Answered by Penny Nom.
A new carpet 2008-02-28
From Rhiannon:
The Gatlins are buying a new carpet for their house. They need about 1,175 square feet of carpet. The carpet they buy is sold by the square yard. Estimate the number of square yards of carpet they need. Do I use the formula to find the volume or are?
Answered by Penny Nom.
Some solid figures 2008-02-28
From tiffany:
What solid figure could have? It has 4 more edges than vertices,
Answered by Penny Nom.
The radius of a circle 2008-02-28
From SteVonee:
Estimate the radius of a circle with the given circumference that is 192ft
Answered by Penny Nom.
A proof in geometry 2008-02-27
From Kimberly:
I'm trying to write a proof for the following: If all altitudes are equal in an equilateral triangle then all sides are equal.
Answered by Stephen La Rocque and Penny Nom.
Two regular pentagons 2008-02-27
From Shirley:
Regular pentagon QRSTU has a side length of 12 centimeters and an area of 248 square centimeters. Regular pentagon VWXYZ has a perimeter of 140 centimeters.

I need to find its area.

Answered by Stephen La Rocque.
What number has exactly 51 divisors? 2008-02-26
From Lisa:
What number has a Tau which equals 51? In other words, what number has exactly 51 divisors? It must be a square of some kind!
Answered by Penny Nom, Victoria West and Harley Weston.
The surface area of an ellipse 2008-02-25
From paritosh:
how is the surface area of a elliptical dome be calculated when the two diameters are 55 metres and 35 metres and the height of the structure is 13.4 mts?
Answered by Harley Weston.
Comparing fractions 2008-02-21
From Melissa:
Hello i am a sixth grader, On my homework sheet there is the question : 5/45 _ 9/81 But it says to Compare the fractions. Use <,>, or = and i find it difficult to answer, please help me find a simple way to answer questions like this.
Answered by Stephen La Rocque and Penny Nom.
Fact family 2008-02-21
From Anand:
Can you think of a fact family that has only two facts? Write the fact family
Answered by Penny Nom.
Algebraic fractions 2008-02-21
From sergio:
how to simplify
(x^2 +5x+6)/(x^2 - 4) x (x^2 -5x+6)/(x^2-9)

Answered by Stephen La Rocque.
10 - 3 + 2 2008-02-21
From Amrit:
what 10-3+2

is it 5 or 9

Answered by Penny Nom.
Row echelon form 2008-02-21
From cOCO:
USE ELEMENTARY ROW OPERATIONS TO WRITE THE MATRIX BELOW IN ROW ECHELON FORM.
1 1 8 - 2
2 -1 1 11
2 1 14 -11

Answered by Penny Nom.
Factoring a trinomial 2008-02-20
From Adrienne:
a field an rea of (x^2+x-6) square metres. State expressions for its length and width.
Answered by Penny Nom.
Subtracting two algebraic fractions 2008-02-17
From Willie:
Subtract and express the answer in simplest form. 5x-3/6-x+3/6
Answered by Penny Nom.
The equation of a line 2008-02-17
From Jennifer:
Given the point A(3,2)B(5,8) find the equation of the line AB ( in standard form )
Answered by Penny Nom.
The slope intercept form of a line 2008-02-17
From chris:
find the slope and y intercept of the line (2x+4)-3y=11(x-1) is this equation in slope point form.
Answered by Penny Nom.
Exponential form 2008-02-16
From annna:
i already got the answer for what is the prime factorization of 648 which is 2x2x2x3x3x3x3 but how do i write that answer in exponential form?
Answered by Penny Nom.
The length of the third side of a triangle 2008-02-16
From mary:
I have an angle of 72 degrees and each of the sides are 5' long. What is the distance from each of the ends of the 5 feet to form a triangle.
Answered by Stephen La Rocque.
Five digit combinations 2008-02-15
From Debbie:
I would like to get a list of all the possible 5 digit combinations of the numbers 12345.
Answered by Penny Nom.
Growth factor and growth rate 2008-02-13
From william:
what is the difference between growth factor and growth rate?
Answered by Stephen La Rocque and Harley Weston.
Scale factors 2008-02-13
From Dayne:
My teacher doesn't now how to teach us scale factors and i have a problem to make it easier to understand. The problem is: A map of Levi's property is being made with a scale of 2 cm : 3 meters. What is the scale factor?
Answered by Stephen La Rocque.
I am a 4-digit number 2008-02-12
From Nickie:
I am a 4-digit number with no repeating digits. I am divisible by 5, my first two digits (left to right) make a number divisible by 3, and my first three digits make a number divisible by 4. Also, my digits have a sum of 19 and I have the digit 7 in the thousands place. Who am I?
Answered by Penny Nom.
A real life example of a decagon 2008-02-12
From Htet:
I have a math dictionary to complete by February 13, 2008, Wednesday, and I need to know what a real life example of a decagon can be. I need help on this!
Answered by Penny Nom.
5x squared - 15x - 200 2008-02-11
From Catherine:
I am having a problem solving this question:

5x squared - 15x - 200

Answered by Penny Nom.
5x^2 - 45 2008-02-11
From Tiana:
factor: 5x^2 - 45
Answered by Stephen La Rocque.
The circumference and radius of a circle 2008-02-10
From Ray:
How do you find the circumference or radius of an area presuming it is a circle. Or in other words how do you find the c or r given only the area is 50 sq metres
Answered by Penny Nom.
A flagpole and a statue 2008-02-09
From Krista:
A flagpole casts a ten meter (m) shadow at the same time as a six metre (m) statute beside it casts a two metre shadow. What is the height of the flagpole??
Answered by Steve La Rocque and Penny Nom.
What percent profit is this? 2008-02-07
From Vanessa:
Mrs Henderson buys a chair for $215.00 and later sell the chair for $336.00 what percent profit is this?
Answered by Penny Nom.
What is the outside footage of a 50 acre parcel square? 2008-02-07
From dan:
what is the outside footage of a 50 acre parcel square
Answered by Penny Nom.
Find two numbers with HCF of 3 and LCM of 180 2008-02-07
From matthew:
Hi, please help me with this, Find two numbers with HCF of 3 and LCM of 180
Answered by Stephen La Rocque and Penny Nom.
Four digit numbers containing 85 2008-02-06
From Vonda:
How many four digit numbers contain the digit pattern 85 at least once and only once?
Answered by Stephen La Rocque.
The game of 24 2008-02-06
From Jameeka:
What is a solution for 10,11,9 and 2 (Using fractions) in the game 24 square.
Answered by Penny Nom.
Total profit 2008-02-03
From Rhonda:
I help to manage a concession stand. We sell a variety of items. Some have a profit of 60%, some 80%, etc. We have done nothing scientific to choose our prices. We just try to stay in the ballpark of schools around us. Our administrator wants to pay those that run the concession stand a percent of what is in the cash box after start-up money is taken out. We have some gathered some data (not much) on how many of each item we sell and can certainly calculate percent profit. Is the profit for the entire concession stand a weighed average? I'm thinking (% profit on candy)(number candy sold) + (percent profit nachos)(number nachos sold) + (percent profit pop)(number pop sold), etc. Will this work? Does the AMOUNT of profit for these different items need to be figured-in? Hmmm. . .
Answered by Victoria West and Harley Weston.
A inear system 2008-02-03
From Kelly:
is (-1,5) a solution of each system? y=-x+4 y=-1/5x
Answered by Penny Nom.
Tiles on a basement floor 2008-01-31
From ORIETTA:
I have a basement 1000 sq ft I want to purchase tile that are 18x18inches and they are charging 4.15 sq ft how much would it cost since I will need less tiles to cover the area since it's not 12X12
Answered by Penny Nom.
Fibonacci numbers 2008-01-30
From Beth:
You've answered a question for a Linda Bach on Oct. 25, 1999 that has to do with a pet rabbit and stairs. (Working Backwards) We were wandering if there is a formula we can use to plug the numbers in? My 7th grader got that exact same problem. I've spent several hrs trying to come up with a formula. I've tried the combinations formula, the Pascal's triangle and the Permutations formula. It seems like it is something real close to those, but can't quite get anything to work. What kind of formula would work for finding the outcome of this problem? Thanks so much for your help. Beth
Answered by Penny Nom.
The standard form of a line 2008-01-30
From Thomas:
I'm attempting to get this in standard form

slope=3;(1,5)

Answered by Penny Nom.
Sum and difference of cubes 2008-01-30
From Amanda:
It has been a really long time since I was in Algebra and I can't remember how to factor cubes such as x^3 +81 or subtracting/adding fractions with variables such as [1/(x+h)+2]-[1/x+2]. Please help!!!
Answered by Penny Nom.
The acre measurement of a property 2008-01-30
From Kim:
I would like to know the acre measurement of a property that is 673.68 feet by 820.76 feet.
Answered by Penny Nom.
What solid figure has 1 flat surface and 0 vertices? 2008-01-30
From Cheronda:
What solid figure has 1 flat surface and 0 vertices?
Answered by Stephen La Rocque and Harley Weston.
Converting decimal fractions into common fractions 2008-01-29
From Donna:
Question from DONNA:

I need to change 6.9375 by 10.0625 into inches.

Answered by Harley Weston.
Belled-out pier 2008-01-28
From Gina:
I need to know how to find the total yards needed to fill a concrete pier that is 54"/ 108" and 26' deep. That is...54" @ the top of the pier belled to 108" @ the bottom...26' deep.
Answered by Stephen La Rocque.
18/95 x t =3 3/5 2008-01-28
From emily:
18/95 x t =3 3/5
Answered by Penny Nom.
A fraction comprised as 2 numbers having the same units 2008-01-27
From nancy:
term=a fraction comprised as 2 numbers having the same units
Answered by Stephen La Rocque and Harley Weston.
The volume of a tank 2008-01-27
From Fred:
A cubic tank holds 1,000 kilograms of water what are the dimensions of the tank in meters?
Answered by Harley Weston.
Sum of factorials 2008-01-26
From Emily:
What is the tens digit in the sum 7! + 8! + 9! ... + 2006!
Answered by Stephen La Rocque.
Inflection points 2008-01-25
From Armando:
Hi, Im trying to write a program that takes an equation ( f(x) = 0 ) and returns a list of the inflexion points in a given interval. there must be (I think) a mathematical method or algorithm to do this, probably involving the (second) derivate of the function. However I have not found such a method yet. Any help on this will be much appreciated.
Answered by Stephen La Rocque and Harley Weston.
Linear feet to metres 2008-01-25
From Jillian:
i have a measurement of something in linear feet. i dont know what a linear foot is so i want to convert it to meters to have an idea. how do i convert.
Answered by Penny Nom.
A developer subdivides a plot 2008-01-25
From Katherine:
A developer buys a plot of land that is 48,000 sq.feet. She sets aside 1/2 of it to bulid a house for herself, then divides the remaining half into 3 equal plots. Find the area of each lot.
Answered by Penny Nom.
The standard form of a line 2008-01-24
From James:
I need to put it in Standard from ax+by=c the problem is (-4,0); parallel to y=-2+1 how can I put this in Standard from
Answered by Stephen La Rocque.
Find the slope of the line with equation 7-3y=2x 2008-01-23
From nell:
Find the slope of the line with equation 7-3y=2x
Answered by Stephen La Rocque.
Percentage of what? 2008-01-23
From Jenny:
What is the difference in percentage between 31990 and 29990?
Answered by Stephen La Rocque.
Change a mixed fraction into a decimal 2008-01-23
From Qiana:
I need to know how to change a mixed fraction into a decimal. the mixed fraction is 9 1/2
Answered by Penny Nom.
Out of school applications of Pythagoras Theorem 2008-01-23
From Laura:
Hi, I am currently working on a math summative in which I have to choose a real life subject and relate it back to the material in my grade 12 math class. I find the history and discovery behind the Pythagorean Theorem and Identity very interesting, but I have yet to find a real-life application of the equations. Yes, I know they are used for finding distances, heights etc., but realistically, how many people actually use it in those situations? Very few. I was hoping for a new application. Is the pythagorean theorem (sin^2x + cos^2x = 1) even applicable? Thank you, Laura
Answered by Harley Weston.
A parallelogram and a rhombus 2008-01-22
From miguel:
i have a problem proving a parallelogram a rhombus.. if a diagonal of a parallelogram bisects an angle of the parallelogram , then its a rhombus prove
Answered by Stephen La Rocque and Walter Whiteley.
A toad climbs the stairs 2008-01-22
From Kate:
The question is: A small toad is sitting at the bottom of the stairs. He wants to get to the 10th step, but is having some difficulty. He jumps up 2 steps and then back 1 each time. How many jumps will he take to reach the 10th step?
Answered by Stephen La Rocque and Penny Nom.
1/2, 1/2, 3/8, 1/4, 5/32, 3/32, 7/128 2008-01-22
From Neil:
Find the next two terms in the following number sequence

1/2, 1/2, 3/8, 1/4, 5/32, 3/32, 7/128

Find a general rule for the nth term of the sequence

Answered by Penny Nom.
The equation of a line 2008-01-20
From Sara:
The question is:

Write the equation of a line that is parallel to y=3x-2 through the point (-5,9). I think the correct equation for this is y=3x+24. But I have to write it in standard form, and I can't seem to figure out how to do that. Please answer soon! Thanks!

Answered by Stephen La Rocque and Penny Nom.
The line through (-22 ;-1) and (-23, -2) 2008-01-20
From Zachery:
i have a slope that equals 1/1 and my coordinates are (-22;-1) and (-23;-2) and i am trying to change it into the equation y=mx+b and the farthest that i've gotten is y=1x_ (i can not find out how to find b.) So i was wondering if you could help me find out what b is because i have got no clue!!!!!
Answered by Penny Nom.
A word problem 2008-01-18
From Marouf:
How can you make a word problem for 6 2/3 and solve the word problem? I'm having trouble. Please help
Answered by Stephen La Rocque.
The game of 24 2008-01-17
From Raina:
I have four numbers 3 3 9 5 that need to equal 24 using addition subtraction multipliction and division and I'm so stuck.
Answered by Chris Langdon, Stephen La Rocque and Claude Tardif.
The angles of a triangle given the three sides 2008-01-17
From Lucy:
Is there a way to find the angles of a triangle just by knowing the lengths of it's sides? It seems like the would be a relationship between the two, but I'm not sure.
Answered by Stephen La Rocque and Harley Weston.
GCF and LCD 2008-01-17
From Peter:
I need to know the differences between GCF and LCD.
Answered by Penny Nom.
A club made a quilt 2008-01-17
From joanna:
math question is club made a quilt 16 1/2 square feet 2/3 of the quilt was red how many square feet were red ?
Answered by Penny Nom.
Fact families with multiplication and division 2008-01-16
From Donna:
I am trying to help my granddaughter with some math homework. I looked at what you have on fact families. Am I correct to assume that the same process applies to multiplication and division as well? Her question asks for fact families for 2, 8, & 16.
Answered by Penny Nom.
Explaining the factoring for the difference of cubes 2008-01-16
From Bill:
A student asked me where did the "difference of cubes" and "sum of cubes" come from. I did not have an answer for her. She is very bright and understands how they work but wanted to know where they derived from. Any help you can offer would be great. Thanks
Answered by Stephen La Rocque.
How many rolls of stock do I need to buy? 2008-01-16
From Dan:
How many rolls of stock do I need to buy?
Given:
The rolls are 40" wide, 20" roll o.d., 4" core o.d., gauge is 0.008" I need to cut these master rolls in 6" strips (4" of waste per master roll) and I need 1,270,500 linear feet of this strip.

Answered by Stephen La Rocque.
Balancing on a fulcrum (net torque and equilbrium) 2008-01-15
From Eric:
If I have a 4" bar that has no weight value with a 275lb. weight on one side and a 125lb. weight on the other side what would be the folcrum point? Please keep the answer very simple. Also is there a fixed equation to figure folcrum points? Is there an equation for if the 4" bar had a weight value?
Answered by Stephen La Rocque.
Two solutions using the law of sines 2008-01-14
From Kate:
I am working on the Law of Sines and I have a problem that says: Find a value for b so that the triangle has 2 solutions.

I am given that A = 36 degrees and a = 5. Now, I learned that for a triangle to have 2 solutions, h < a < b. BUT...my answer key says the answer is: 5 < b < 5/sin 36. I can't figure out how to make this fit with h < a < b.

Answered by Harley Weston.
The surface area of a hemisphere 2008-01-14
From Nicholas:
the surface area of a hemisphere is 1062 cm2. What is the radius?
Answered by Penny Nom.
(303,000,000,000,303x3,300,000,033)/1,000,100,010,001 2008-01-13
From Adrian:
Give the result of the following computation as an integer.

(303,000,000,000,303x3,300,000,033)/1,000,100,010,001

N.B. No calculators allowed

Answered by Penny Nom.
1.2 as a fraction 2008-01-12
From Confused:
i have confused my brain and its driving me crazy i am trying to write 1.2 as a fraction but each time i do it i come out with a different number please help
Answered by Penny Nom.
Hoisting an anchor 2008-01-10
From ron:
four men on a boat are using a capstan for hoisting an anchor weighing 2000lbs. The diameter of the barrel of the capstan at the section in use is 8in., and the pull on the chain carrying the anchor and passing around the capstan barrel is assumed to act at a distance of 1in. from the surface of the barrel. If the distance from the axis of the barrel to each point at which effort is applied to a capstan bar is 40in., what force must be applied by each man hoisting the anchor?
Answered by Stephen La Rocque.
Forces on an inclined plane 2008-01-10
From Ron:
A body that weighs 540lbs is caused to slide up an inclined plane with a uniform velocity by a force that acts parallel to the plane. For each foot of horizontal distance, there is a vertical rise of 2in. If the coefficient of sliding friction is 0.16, what force is required to move the body?
Answered by Stephen La Rocque.
Sqrt(x^2 + y^2) 2008-01-10
From Darcie:
Can you simplify this problem?: square root of (x squared plus y squared). Note: x squared plus y squared is under the radical.
Answered by Penny Nom.
How do I write the following in exponential form? 2008-01-09
From Mandy:
How do I write the following in exponential form?

( square root of a^-2 b)^5

Answered by Penny Nom.
0.18 of an acre 2008-01-09
From susan:
what is the square footage of .18 of an acre of land?
Answered by Penny Nom.
Exponential form 2008-01-08
From liam:
i was wondering if anyone could help me with this question and tell me how and what the did; exponetial form of 4 326 180 501
Answered by Stephen La Rocque.
How far is the jet from the lighthouse? 2008-01-07
From Natalie:
Question: A ship spots a lighthouse that is 53m high, at an angle of elevation of 7 degrees that is directly north of the ship. The same ship spots a jet travelling N62E at an altitude of 1500m with an angle of elevation of 15 degrees. How far is the jet from the lighthouse?

Natalie

Answered by Harley Weston.
Partial fractions 2008-01-06
From taiwo:
i copied the question below from john bird's higher engineering mathematics fourth edition (pg 19) after solving all the partial fraction questions from the book except this:

x^3 + 4^2 + 20x - 7 over (x-1)^2 (x^+8). i will appreciate it if you help me out so i can know whether their answer is correct. thanks alot

Answered by Harley Weston.
The area of a piece of property 2008-01-05
From Greg:
I need to know the area of a six sided piece of property. I sent a map by regular mail.
Answered by Harley Weston.
Factoring x^2 + 25 2008-01-04
From Geri:
what does x squared plus twenty five factor out to?
Answered by Penny Nom.
Multiplying exponentials 2008-01-02
From jessie:
Can you please explain how to put x^3 y^2 times x^1 y^3 in simplest exponential form?
Answered by Penny Nom.
Radicals in exponential form 2008-01-02
From Andre:
The title of the section in my textbook is to write each of the following radicals in exponential form.

My question is how do u write the squareroot of 10 in exponential form?

Answered by Penny Nom.
Practical applications of sequences 2008-01-01
From carl:
can you give me examples of different kinds of practical applications of sequences?
Answered by Penny Nom.
Ordering fractions 2007-12-31
From MARLYN:
Which list shows number in order least to greatest?

A. 07;2/3. 1/5;0.6

B 1/5;06; 07;2/3

C 1/5; 06; 2/3;0.7

D 2/5;0.6; 1/5;0.7

Answered by Penny Nom.
Adding fractions 2007-12-31
From Lisa:
As an educator in Adult Education, I am preparing a young man for a US Government test. The equation I have trouble with is explaining the following equation. It asks: If it takes A 3 days to dig a certain ditch, whereas b can dig it in 6 days, and C in 12, how long would it take all three to do the job? I know the equation is 1/3 + 1/6 + 1/12 and it equals = 7/12 but I don't understand how it equals 7/12. This simple math eludes me. So the question is not the final answer of 1 and 5/7 of days but where one comes up with the 7/12?
Answered by Penny Nom and Claude Tardif.
The surface area of the rectangular solid 2007-12-30
From Sara:
For my math homework i found the volume of a rectangular solid whose deminsions were in feet and the numbers were 10 by 20 by 30
10 = hight
20 = short side base
30 = long side base
and now i have to find the surface area of the rectangular solid....
i know how to do this but i can't get the right answer it should be 2,200 ft squared but i keep getting 13,00 feet squared

Answered by Penny Nom.
The standard normal distribution 2007-12-29
From GEORGE:
Statistics texts state that in a normal distribution, 1 standard deviation covers 68%, 2SD 95% 3SD 99.7%. However, on looking at the tables for normal curve areas, the percentage for a z value of 1 is 84.13. 68 % of values are covered at a z value of 0.47. The discrepancies exist even for other values but are smaller. So why is the rule of thumb so different from the Table for Normal Curve areas?
Answered by Harley Weston.
Differentiate 2007-12-28
From taiwo:
i am finding it difficult to use first principle to differentiate this question: y=xcos2x. can u help me.
Answered by Penny Nom.
An irregular octagon 2007-12-23
From Sheldon:
I am attempting to construct an irregular octagon picture frame out of bamboo. The bamboo is 1" in diameter and the opening should be 20" H X 16"W.
What measurements should be used?

Answered by Penny Nom.
Volume in cubic feet 2007-12-20
From kim:
if i have the inches and need to convert them to cubic feet, how do i do that? i thought it was height X width X length.
Answered by Penny Nom.
F( x - 2) = (x + 3)/ (x - 4) 2007-12-20
From Sean:
If F( x - 2) = (x + 3)/ (x - 4) , then F(5) = 10/3

How do you solve this problem and what section of calculus can I learn the technique to solve this type of problem?

Answered by Stephen La Rocque.
Simplifying square roots 2007-12-19
From Ciara:
How would you calculate 2 to the square root of 8 plus 4 to the square root of 2 minus 5 to the square root of 2?
Answered by Stephen La Rocque.
Partitioning a clock face 2007-12-19
From Kim:
Using a clock with a regular circular faced dial, draw two straight lines such that the sum of the numbers in each of the three areas is equal.
Answered by The team at Math Central.
Profit percentage (markup and margin) 2007-12-18
From Laura:
If my husband says he sold $11,000 worth of tools and made $1400 on the deal, what is the percent of profit?
Answered by Stephen La Rocque.
For how many positive integers n is n^2 + 96 is a perfect square? 2007-12-18
From Rajesh:
For how many positive integers n is n^2 + 96 is a perfect square ?
Answered by Penny Nom.
What size room would I need for 220 cubic feet? 2007-12-18
From Andy:
What size room would I need for 220 cubic feet? What is the calculation I would use to find the answer to this problem?
Answered by Penny Nom.
A bronze statue is made in two sizes. 2007-12-17
From Sara:
Hello! A bronze statue is made in two sizes. The taller statue is 15 cm high and the shorter one is 9 cm high. The taller statue weighs 3.75 kg. What is the weight of the shorter statue?
Answered by Penny Nom.
What length scale factor should he use? 2007-12-16
From Lisa:
Hi! Joe wants to enlarge a picture so that its area is doubled. What length scale factor should he use?
Answered by Penny Nom.
The perimeter of circle 2007-12-15
From shaquilla:
what is the perimeter of a circle.
Answered by Penny Nom.
Negative numbers 2007-12-14
From mannal:
what negative integers do we use in the real world? like -30 temperature?
Answered by Penny Nom.
Inflation rate 2007-12-14
From cassie:
In 1972, Bob could fill his car with gasoline at a cost of $3.00. In 2007 it cost him $45.00 to fill his car. From 1972 to 2007, what is the annual inflation rate for gasoline? How does this inflation rate change when you change the cost to $0.25 per gallon in 1972 to $3.20 per gallon in 2007?
Answered by Stephen La Rocque.
Improper fractions and mixed numbers 2007-12-12
From Selah:
How do I change an improper fraction to a mixed number?
Answered by Penny Nom.
How many houses must they paint? 2007-12-12
From Natalie:
The question is: Four boys work together,painting houses for the summer.For each house they paint they get 256 pounds. If they work for four months of the summer and their expenses are 152 pounds per a month, how many houses must they paint for each of them to have one thousand pounds at the end of the summer?
Answered by Penny Nom.
Shooting an arrow over a wall 2007-12-11
From Amy:
The path of a large arrow fired from a non torsion catapult can be modeled by y=-0.0044x^2 + 1.68x, where x is the distance the arrow travelled (in yards) and y is the height of the arrow (in yards). Given the height of a castle wall, find the safest distance from the wall to launch an arrow over a 120 yard-high wall.
Answered by Victoria West and Stephen La Rocque.
The tangent to a curve 2007-12-10
From Christy:
I know this question is simple but I can't figure out what I'm doing wrong.
Find the equation of the tangent line to the curve 2x^2 - y^4= 1 at the point (1,1).

Answered by Penny Nom.
System of equations 2007-12-06
From Jenn:
change the equation,x-y=4 to form y=mx+b the solution to the system of equations y=2x and y=-x+3 is
Answered by Stephen La Rocque.
Chicken and goat feet 2007-12-05
From Kim:
Old McDonald raises goats and chickens. The animals have a total of 100 heads adn 360 feet. How many goats and how many chickens does Mr. McDonald have?
Answered by Stephen La Rocque and Penny Nom.
How many gallons are in the tank? 2007-12-03
From bobby:
my tank holds 740 gallons and the dimensions are 8 ft tall and 4 ft diameter the wine is 3 ft 7.5 inches from the top can you figure out how many gallons are in the tank ?
Answered by Penny Nom.
Sharing a cake 2007-12-03
From Nikheel:
Amar baked a cake. john ate 1/6 of the cake. susan ate 1/5 of what was left. chan ate 1/4 of what was left. cindy ate 1/3 of what was left. luigi ate 1/2 of what was left. how much of the original cake was left.
Answered by Stephen La Rocque.
Square yards 2007-11-30
From Chad:
I need to convert 4,599 Cubic Yards to Square yards. The thickness is 10 inches. Is there a formula for this conversion. Thanks,
Answered by Penny Nom.
The square footage of a property 2007-11-30
From Warren:
The west property line is 240' long with a 90-degree angle to the south property line.
The south property line is 493' long with a 90-degree angle to the east property line.
The east boundary is 165' long.
The length of the north property line is not known but it is a straight line from the north end of the east property line to the north end of the west property line.

Could you please tell me the total square feet and how many acres?
And how you calculated the answer

Answered by Penny Nom.
Exponential form 2007-11-30
From Sasha:
How do I write the number 127398 in exponential for?
Answered by Penny Nom.
Transformations and compositions 2007-11-29
From mary:
Is there any possible relationship between composite functions and the concept of function transformations?
Answered by Harley Weston.
(32-2 x 5) divided by 2 + 8 2007-11-28
From Kim:
Solve

(32-2 x 5) divided by 2 + 8

Answered by Leeanne Boehm.
How many cubic feet are in this box? 2007-11-26
From Julio:
My son has a box that measures 27X18X19 inches. We want to know how many cubic feet are in this box.
Answered by Penny Nom.
Straight lines 2007-11-26
From Divyansh:
hello i am in eleventh class and am, preparing a project on straight lines i cant really find uses of straight lines and its equations in daily life i am also thankful to you in advance and am waiting for your answer eagerly because i need to submit my project only this week thanking you
Answered by Penny Nom.
A percent as a fraction 2007-11-24
From Abby:
write 43.75% as a fraction or mixed number in lowest terms.
Answered by Penny Nom.
Ordering fractions 2007-11-23
From Joy:
what are the orders of these fractions?
2/3, 1/2, 1/4, 2/10, 3/4, 1/3, 9/10, 2/2.

Answered by Penny Nom.
A curve sketch 2007-11-22
From Ahson:
Find critical points, determine the monotonicity and concavity and sketch a graph of f(x) with any local maximum, local minimum and inflection points labeled:

1. f(x) = x^4 - x^3 - 3x^2 + 1

Answered by Harley Weston.
The definition of the sine function 2007-11-22
From Indrajit:
I need a explanation in this theory.......if sinθ = p/h...then
sin 90 deg. = p/h
or 1 = p/h
or p=h .....how can a perpendicular be equal to a hypotenuse.???

Answered by Harley Weston.
Area of a 17-sided lot 2007-11-21
From Lynda:
My uncle is wanting to buy this piece of land [a 17-sided polygon] but we are questioning the acerage total. the measurements are [on the attached diagram].
Answered by Stephen La Rocque.
Bundles of wood 2007-11-19
From Darcy:
When you purchase a "bundle" of wood at the store, it is marked .75 cubic feet. What is the formula for figuring out that measurement for cubic feet and what fraction of a true cord of wood (128 cubic feet) is it. I would appreciate the formula so I can build a spreadsheet to help my secretaries and myself be in compliance to the law when selling firewood. Thanks.
Answered by Harley Weston.
The derivative 2007-11-19
From ralf:
Find the derivative of the function
1-. y=1+2x8
2-. y=(1+2x )8

Answered by Harley Weston.
Confidence level 2007-11-19
From Fara:
It is common for public opinion polls to have a " confidence level" of 95%, meaning that there is a 0.95 probability that the poll results are accurate within the claimed margins of error. If six different organizations conduct independent polls, what is the probability that all six of them are accurate within the claimed margins of error? Does the results suggest that with a confidence level of 95%, we can expect that almost all polls will be within the claimed margin of error?
Answered by Harley Weston.
Cubic feet and gallons 2007-11-19
From Leonard:
Every conversion I find to convert a cubic foot to a gallon is in liquid. Question, how would 1 cubic foot convert to gallons in dry measurement? Example say you have 1 Gallon of packing peanuts how many cubic feet would that be? I can do this and have done it for the heck of it, I took a box 12x12x12 inches, which is a cubic foot, filled it with packing peanuts the tried to fill a 1 Gallon container, that amount almost fills a 1 gallon bucket twice. I even ask a math teacher and was told that 1 cubic foot equaled 7.481 gallons, then I said that is liquid, teacher ansered yes, then i ask what about dry and got a response of I do not know.
Answered by Harley Weston.
The game of 24 2007-11-19
From M:
Please help...how would you compute 2,2, 6, and 9 to equal 24?
Answered by Penny Nom.
Red balls and blue balls 2007-11-18
From Md:
In a group of 15, 7 have Red Balls, 8 have Blue Balls and 3 have neither. What fraction of the group has both Red Balls and Blue Balls?
Answered by Stephen La Rocque.
Find the radius of a circle given the center and a point on the circle 2007-11-18
From Raymund:
Find the radius if the center is at (0, -5) and one point on the circle is (2,3)
Answered by Stephen La Rocque.
Expand (a^4 - b^4) 2007-11-17
From Saif:
how would you expand (a^4 - b^4) ???
Answered by Stephen La Rocque and Victoria West.
The game of 24 2007-11-17
From Todd:
Hello. My son is very good at the Math 24 game and when he is stumped, I am usually able to figure it out. However, we are both stumped with this one. The numbers are 6, 6, 9 and 5. You can use addition, subtraction, division and multiplication. Please help us equal 24 with those 4 numbers.
Answered by Claude Tardif.
Exponential form 2007-11-16
From ashley:
write a sentence for exponential form.
Answered by Penny Nom.
How many yards of fencing do I need? 2007-11-16
From Terrie:
I have to order fencing for a 25-acre, rectangular field.One side of the field is 1/4 mile. How many yards of fencing do I need to enclose the field?
Answered by Stephen La Rocque.
A geometric proof 2007-11-16
From Julie:
Prove that tangents to a circle at the endpoints of a diameter are parallel. State what is given, what is to be proved, and your plan of proof. Then write a two-column proof.
Answered by Walter Whiteley.
A variable over a number 2007-11-15
From Silver:
3/8x - y/2= 1

I do not understand what you do with the variable over a number.

Answered by Penny Nom.
Why do they use the letter b in y=mx+b? 2007-11-15
From virginia:
why do they use the letter b in y=mx+b? im asking this because my math teacher said he will give extra points to anyone who finds this out first and i was hoping you guys would be the ones who could help.
Answered by Penny Nom.
Ordering fractions 2007-11-15
From DEL:
Hi. I feel really stupid ! I'm a mature student and i have completely forgotten How to find out the order of fractions from largest to smallest. I Have been put this poser; 7/4...1/6....7/2 Can you please tell me what is the largest and lowest? Will be very grateful....thank you
Answered by Gabe Potter.
How do you convert eleven fifteenths into a decimal? 2007-11-14
From Lauren:
How do you convert eleven fifteenths into a decimal?
Answered by Penny Nom and Stephen La Rocque.
Local maxima, minima and inflection points 2007-11-13
From Russell:
let f(x) = x^3 - 3a^2^ x +2a^4 with a parameter a > 1.

Find the coordinates of local minimum and local maximum

Find the coordinates of the inflection points

Answered by Harley Weston.
Whole numbers and improper fractions 2007-11-13
From Jennifer:
I would like to know how you convert whole numbers into improper fractions
Answered by Melanie Tyrer.
Maximize his profit 2007-11-12
From apoorva:
During the summer months Terry makes and sells necklaces on the beach. Last summer he sold the necklaces for $10 each and his sales averaged 20 per day. When he increased the price by $1, he found that he lost two sales per day.

a. Find the demand function, assuming it is linear.
b. If the material for each necklace costs Terry $6, what should the selling price be to maximize his profit?

Answered by Penny Nom.
Family of functions 2007-11-12
From Russell:
Consider the family of functions
f(t)= Asin3t + Acos3t +Bsin8t + Bcos8t

find exact values of parameters A and B so that f(0) = 2 and f ' (0) = 1

Answered by Stephen La Rocque.
What number best completes the series? 2007-11-12
From Grace:
What number best completes the series?

2 3 7 13 27 ____

Answered by Stephen La Rocque, Penny Nom and Harley Weston.
8^2+64^2 2007-11-12
From Daniel:
I am having trouble factoring equations like 8^2+64^2, I just don't know the correct steps to get the answer.
Answered by Penny Nom.
6 consecutive multiples of 6 2007-11-11
From jeff:
find 6 consecutive multiples of 6 whose sum is the least common multiple of 13 and 18
Answered by Penny Nom.
Prime factorization 2007-11-11
From jeff:
find the prime factorization and use exponential notation for 432
Answered by Penny Nom.
Changing units and sizes of things 2007-11-08
From Carol:
volume of cube = 216 cubic inches how do I calculate cubic feet and calculate its surface area in square feet. a rectangular box has dimensions of 3 x 6 x 15 how do I calculate surface area in square yards and calculate volume in cubic inches. if I double the dimensions how does it affect the volume
Answered by Stephen La Rocque.
Related Rates (a water trough) 2007-11-07
From Christina:
A rectangular trough is 3ft long , 2ft across the top and 4 ft deep. If water flows in at the rate of 2ft^3/min, how fast is the surface rising when the water is 1 ft deep ?
Answered by Stephen La Rocque.
Equivalent Fractions 2007-11-07
From Marlene:
find equivalent fractions with a denominator of 8 1/2
Answered by Stephen La Rocque.
Ordering fractions 2007-11-06
From Mene:
this is the weight of three packages 4/5 kilograms 7/10 kilograms 1/2 kilograms order these weights from least to greatest
Answered by Penny Nom.
The height of a triangle 2007-11-03
From William:
I need to find the height of a triangle if the base is 15, one side is 14 and the other side is 13.
Answered by Stephen La Rocque and Penny Nom.
Stem & Leaf Plots 2007-11-02
From Jen:
Trying to help my Gr.4 son with his math and i am a little lost here are the stem and leaf plots..
Stem Leaves
4          02
3          1226699
2          011255789
1          33357899

Stem Leaves
2          00011111223334455
1          range566788899

Question - How are the ranges for the stem & leaf plots different ?

Answered by Penny Nom.
Questions of the Month 2007-11-01
From Judith:
Where would I find comprehensive questions, that cover the majority of skills in a unit such as number sense, to use as "Questions of the Month"?
Answered by Victoria West.
f(x+y) = f(x) + f(y) + 2xy 2007-11-01
From Marcia:
For all real numbers x and y, let f be a function such that f(x+y) = f(x) + f(y) + 2xy and such that the limit as h -> 0 of f(h) / h = 7, find: f(0), use the definition of the derivative to find f'(x), and find f(x).
Answered by Penny Nom.
A decimal to a fraction 2007-11-01
From Lisa:
i am trying to turn a decimal into a fraction. my example is to turn 10.5 into a fraction at its lowest form
Answered by Penny Nom.
A wiffleball field 2007-10-31
From Svitlana:
The Adam's family has set up a wiffleball field in their backyard. The bases are arranged like a typical baseball diamond, where the distance between consecutive bases is the same. First base is opposite of third base, and second base is opposite of home plate. The distance between consecutive bases is 50 feet. Now, the pitcher stands 25 feet from home plate and lies on the line between home plate and second base. How far is the pitcher from first base? Round your answer down to the nearest inch.
Answered by Penny Nom.
Write each frequency as a decimal 2007-10-31
From Jazzy:
Twins are born once in every 89 births. Identical twins are born 4 times in every 1000. Triples are born once in every 6900 births. write esch frequency as a decimal and order the decimals from least to greatest frequency.
Answered by Penny Nom.
Measurements on a test tube 2007-10-30
From Bruce:
I figure that the surface area (SA) of a round bottom test tube can be determined by the equation SA = 2pi*h*r where r is the radius of the hemispheric bottom & column part, and h is the height (h) at any point upwards from the bottommost portion in the center of the tube.

I haven't been able to come up with an equation yet for the circumference of such a test tube at any point upwards along the tube based on the height in its center or on the distance along the outside of the tube.

Answered by Harley Weston.
The rate of change of the concentration of a solution 2007-10-30
From Nicholas:
A barrel initially has two kg of salt dissolved in twenty liters of water. If water flows in the rate of 0.4 liters per minute and the well-mixed salt water solution flow out at the same rate, how much salt is present after 8 minutes? I tried working backwards given the answer but I can't seen to get their answer of ~1.7kg. Any help would be great! Thanks
Answered by Harley Weston.
Board feet 2007-10-28
From Carolyn:
Lumber is measured iin board feet, where a board foot in volume of a piece of lumber one ft sq and one inch thick. How many board feet are in a two by four inch that is 10 feet long?
Answered by Stephen La Rocque.
How to solve related rates problems 2007-10-27
From David:
Can you plz explain how and where you come up with an equation to solve this?
Find the rate of change of the distance between the origin and a moving point on the graph of y = sin x if dx/dt = 2 centimeters per second.

Answered by Stephen La Rocque.
Is there a practical use for radian measure? 2007-10-26
From Paula:
Is there a practical use for radian measure in any profession? Which professions might us radian as opposed to degree measure?
Answered by Harley Weston.
Area of a triangle formed by three points on a graph 2007-10-26
From Betty:
My question is 'find the are of a triangle whose vertices have coordinates (3,5),(6,-5), and (-4,10)
Answered by Stephen La Rocque.
Two mirrors 2007-10-24
From Peter:
The reflecting surfaces of two intersecting flat mirrors are at an angle θ (0° < θ < 90°). For a light ray that strikes the horizontal mirror, show that the emerging ray will intersect the incident ray at an angle β = 180° – 2θ.
Answered by Stephen La Rocque.
A sixfold increase 2007-10-24
From Fred:
If I have $500 and it grew to $3,000, what is the correct description of the increase? Is it a sixfold increase (sextupled), or a fivefold increase (quintupled)? When I divide $3,000 by $500, the result is six, therefore, a sixfold increase?
Answered by Stephen La Rocque.
Turn 81 into the power of three 2007-10-22
From Victoria:
I do not understand how to turn 81 into the power of three.
Answered by Penny Nom.
Maximize profit 2007-10-22
From Dina:
A meat market purchases steak from a local meat packinghouse. The meat is purchased on Monday at a price of $2 per pound, and the meat market sells the steak for $3 per pound. Any steak left over at the end of the week is sold to a local Zoo for $0.50 per pound. The demand for steak and the probabilities of occurrence are as follows: Demand Probability
      20          10%
      21          10%
      22          15%
      23          20%
      24          20%
      25          15%
      26          10%
Determine the amount of stock to maximize the profit. Draw the graph and explain.

Answered by Penny Nom.
The rate of change of the area of a triangle 2007-10-22
From Ahlee:
So my question is: The included angle of the two sides of a constant equal length s of an isosceles triangle is ϑ.
(a) Show that the area of the triangle is given by A=1/2s^2 sinϑ
(b) If ϑ is increasing at the rate of 1/2 radian per minute, find the rate of change of the area when ϑ=pi/6 and ϑ=pi/3.
(c) Explain why the rate of change of the area of a triangle is not constant even though dϑ/dt is constant

Answered by Penny Nom.
A Farmer's pasture 2007-10-19
From Jennifer:
A Farmer's pasture measures 100 feet by 108 feet. Each cow needs 4 square yards of pasture to have enough grass.
How many Cows should the farmer put in this pasture?
If he builds a fence around the pasture 5 feet inside the edge, how much fencing would he need?
Does this change the number of cows that can be kept in the pasture? If so, why ?

Answered by Penny Nom.
I need to order 3/11, 1/8, 2/9 from least to greatest 2007-10-19
From Andrew:
I need to order 3/11, 1/8, 2/9 in least to greatest.
Answered by Penny Nom.
The nth term 2007-10-18
From shannon:
Ok , what i am having problems with is the nth term. I get how the numbers come together, but i am having trouble with finding the nth term.
Answered by Penny Nom.
How far away is the town? 2007-10-17
From georgia:
You are driving to town 15 miles away. If you have already driven 3 1/4 miles, how far away is the town.
Answered by Penny Nom.
A triangular lot 2007-10-17
From Brian:
Seeking the square footage of a triangular property lot, dimensions are: 620 X 620X 720.
Answered by Penny Nom and Melanie Tyrer.
Order of operations 2007-10-17
From Devon:
What function precedes the other? ie; 18 - 4 x2 =
Answered by Penny Nom.
Surface area of an open-ended cone 2007-10-16
From Lorne:
What is the surface area of an open ended cone? Measured at 10' high, 16' diameter on the bottom and 2' diameter at the top.
Answered by Stephen La Rocque.
0,1,2,3,6,11,20,37,68... 2007-10-16
From Pat:
Describe the rule for this pattern: 0,1,2,3,6,11,20,37,68... Thanks
Answered by Penny Nom.
Four triangles in a square 2007-10-15
From Kristina:
A square with side lengths of 6 cm is divided into 3 right triangles and a larger isosceles triangle. If the three right triangles have equal area, find the exact area of the isosceles triangle.
Answered by Stephen La Rocque.
15 divided by 5a 2007-10-12
From Susan:
Is "5a" considered an expression to be solved first, or do you simply go in PEMDAS order with the 15 divided by 5 in the following problem: Evaluate when a = 2 15 divided by 5a
Answered by Penny Nom.
How much water is in a hose? 2007-10-12
From Bryan:
I need to know how much water (gal) is in a hose thats 100' long by 5" in dia. Thanks
Answered by Stephen La Rocque.
Domain and range 2007-10-12
From Dawn:
Hi, I've been out of school for 8 years and recently picked up a math correspondence course. I'm having trouble trying to figure out the range and domain of a linear function. I've read everything I can find in my text, lessons and on your site and I still can't figure out what I'm suppose to be doing. The function is y=2x+1. Please help.
Answered by Penny Nom.
13 year and 17 year locusts 2007-10-12
From stefan:
how many years pass between the years when both 13 year and 17 year locusts are out at the same time?
Answered by Penny Nom.
A stem leaf and plot with decimals 2007-10-12
From Brandi:
My child came home with a worksheet regarding stem and leaf plot. I figured out how to work it with whole number but not with decimals. Can you tell me how to do this with stem and leaf graph? They are even asking for a conclusion on the information. This makes no sense to me. Please explain and send an example problem also. Thank you!
Answered by Harley Weston.
What is a friendly Factor? 2007-10-12
From Christine:
Hi mi niece and I are trying to figure out what friendly factors are. An example of one of the questions: Use friendly factors to multiply the following. 8x25=. Can you please help? thank you
Answered by Penny Nom.
The percentage difference between 1/20 to 1/15 2007-10-11
From jake:
what is the percentage difference between 1/20 to 1/15....
Answered by Stephen La Rocque.
Inverses of Quadratic Functions 2007-10-11
From Elliot:
I'm a special education tchr trying to demonstrate how to find the inverse (using algebra) of quadratic functions. Any help would be greatly appreciated.
Answered by Steve La Rocque and Harley Weston.
The average rate of change of a function 2007-10-11
From vern:
Find the average rate of change of the function over the given interval. Compare this average rate of change with the instantaneous rates of change at the endpoints of the interval. f(X)=sinX for the inverval [0,pi/6]?
Answered by Harley Weston.
Substitution method 2007-10-11
From Kevin:
3xx+2y=-36-y=11
Answered by Stephen La Rocque.
Area of a quadrilateral 2007-10-10
From Courtney:
how would i find the area of a quadrilateral..
the sides are a (/) is 6cm, b (—) is 9 cm, and c (\) is 7 cm..
the angle between a and b is 140 degrees and b and c is 115 degrees..

Answered by Stephen La Rocque.
Simplifying algebraic expressions 2007-10-09
From Sakeena:
(2^2*3)^x+1/2^2x*3x
Answered by Stephen La Rocque.
Simplifying rational expressions 2007-10-09
From Bama:
x(x-2)+1 divided by x^2-3x+2
Answered by Stephen La Rocque.
Factoring quadratics 2007-10-09
From Abbie:
I have two problems same topic. I belive they cannot be factored but i want to make sure.
x2+6x-40
x2-20x+99

Answered by Penny Nom.
Solving four simultaneous equations (system of four linear equations) 2007-10-07
From Johan:
I need some help in solving this question
x + 2y - 3z + 4w = 12
2x + 2y - 2z + 3w = 10
0 + y + z + 0 = -1
x - y + z - 2w = -4

Answered by Stephen La Rocque.
Triangle perimeter and area 2007-10-05
From divakar:
The sides of the triangle are integers. The perimeter is 8. What is the area?
Answered by Stephen La Rocque.
Solving arithmetic problems in the right order (BEDMAS) 2007-10-05
From Kim:
3+4x2-(10 divided by 5)= what?
Answered by Penny Nom.
Find the equation of a line given two points 2007-10-05
From Tiffany:
The equation for the line through the points (11,2) and (18,12) can be written in the form Ax+By=C. Find A and B.
Answered by Victoria West.
Maximizing profits II 2007-10-05
From a student:
Suppose there are three firms with the same demand function. The function is Q=1000-40P. Each firm also a a cost function.
Firm 1: 4000+5Q, Firm 2: 3000+5Q, Firm 3: 3000+7Q.
What price should each firm charge if it wants to maximize profits.

Answered by Harley Weston.
Maximizing profit 2007-10-05
From a student:
Use the following equation to demonstrate how a firm that produces at MR=MC can also maximize its total profit. The equations to use are P=170-5Q TC=40+50Q+5Q^2
Answered by Harley Weston.
Show that there is no five-digit number which 2007-10-04
From Greg:
show that there is no five-digit number which uses each of the digits 1,2,3,4,5 such that the numbers formed by the first digit is divisible by 1, by the first two digits is divisible by 2 by the first three digits is divisible by 3, by the first four digits is divisible by 4, by the first five digits is divisible by 5
Answered by Stephen La Rocque, Claude Tradif and Victoria West.
Sum and difference of two mystery numbers 2007-10-04
From Donna:
The sum of two numbers is 39. The difference is 11. What are the numbers?
Answered by Stephen La Rocque.
Yards to feet 2007-10-04
From Celina:
A football field is 100 yards how many ft is it?
Answered by Stephen La Rocque.
Finding the last non-zero digits of large factorials 2007-10-04
From Mukesh:
i have to find last five non zero digits of integer which can be very large ( upto 10^12) . i can find last non zero digit of of any factorial. Now my problem is that i have to find last five non zero digit of factorial and also i want to general method for last K non zero digits of factorial n. For example 10!=3628800 so last non zero digit is 8 ,last two non zero digit is 88 .....and last five non zero digit is 36288.
Answered by Victoria West.
What is the total surface area of a cylinder? 2007-10-03
From MATT:
what is the total surface area of a cylinder with outside diameter 40 cm. thickness 10cm. and length 2m. Work in cm^2
Answered by Stephen La Rocque.
A sheet of corrugated iron 2007-10-03
From Ashutosh:
A sheet of corrugated iron has corrugations based on the arc of a circle of radius of 3 cm and a central angle of 180 degrees. How many corrugations are formed when a flat sheet of steel 1 metre wide is bent to form a sheet of corrugated iron?
Answered by Stephen La Rocque.
Equivalent fractions 2007-10-03
From kiki:
I want to know how to this equivalent decimal for 6.0250.
Answered by Stephen la Rocque and Penny Nom.
8x^3 + 4x -3 2007-10-02
From Indrajit:
8x^3 + 4x -3
Answered by Stephen La Rocque.
Golf pairings 2007-10-02
From Mike:
Regarding arranging golf players so no person plays with anyone more than once. You have given examples for 16 and 24 players. If it can be done, i need a solution for 20 players, 4 players per team one round per day for 5 days
Answered by Victoria West.
Equivalent mixed numbers 2007-10-02
From negra:
an average-sized person can burn about 6 1/2 calories a minute while ridinng a bike. Which of the following is equivalent to that amount?
a) 1 2/2

b) 5 6/2

c) 6 2/4

d)6 2/6

Answered by Stephen La Rocque and Penny Nom.
How do you calculate work when force is negative? 2007-10-02
From Snigdha:
how do you calculate work when force is negative?
Answered by Stephen La Rocque.
Irrational functions 2007-10-01
From alicia:
i have a question about irrationals functions.
i have been using them quite some time now, but i wonder where they can be found in daily life?
i hope you can help me,

Answered by Harley Weston.
3 times x to the power -2 2007-10-01
From Jennifer:
The problem is 3 times x to the power -2.

I was just supposed to simplify, and I have no idea how.

Answered by Penny Nom.
y squared over 3 times 8 over y 2007-09-30
From John:
y squared over 3 times 8 over y
Answered by Stephen La Rocque.
Dividing the surface of a sphere 2007-09-30
From Jaroslaw:
Is there a formula or method to divide the surface area of a sphere into equal parts. For example; 36 equal parts, 60 equal parts or 92 equal parts and so forth.
Answered by Chris Fisher and Stephen La Rocque.
A 6 digit safe combination 2007-09-29
From rich:
hi, i have a safe with a 6 digit combination. and i know the only numbers used are the numbers 2 and 5. how many combinations are there? and can you tell me all of them?
Answered by Penny Nom.
The area of a roof 2007-09-27
From RANDY:
I have an area of roof. It's in a sort of triangle. it's 25' across the top 30' at each edge 86' across the bottom
Answered by Stephen La Rocque.
The game of 24 2007-09-27
From Amanda:
I'm really stumped on a 24 math game question from math class. The numbers are 7,11,17,and 20. Help??
Answered by Claude Tardif.
Four digit odd numbers 2007-09-26
From sonia:
how many 4 digit odd numbers can be made using the digits 2, 5, 7, and 8
Answered by Penny Nom.
Exponents 2007-09-26
From Calvin:
Exponential form 5 3exponent x 5 4exponent divide 5 2exponent
Answered by Stephen La Rocque.
Cubic meters, square feet and office space 2007-09-25
From Kathryn:
Can someone please help and let me know how do you convert 11 cubic meters into square footage?
Answered by Steve La Rocque and Harley Weston.
A piece of wire is bent in the form of a circle 2007-09-24
From Renece:
a piece of wire is bent in the form of a circle and it encloses an area of 154cm a) calculate the radius of the circle b) the circumference of the circle use 22/7 The same piece of wire is then bent into a square d) calculate the area enclosed by the square.
Answered by Penny Nom.
Two equations in two unknowns 2007-09-22
From Mary:
Having problems doing this problem, looking for a solution with the work. I would like to see how you got your answer, to see what I was doing wrong.

solve using the substitution method, is there "no solution" or "infinitely many solutions"

4x+y=4
2x+8y=0

Answered by Stephen La Rocque and Leeanne Boehm.
How much water to fill my bedroom? 2007-09-22
From hector:
how can i find out how much water i can fill my bedroom with when it measures 12 ft.x12 ft.x10 ft.
Answered by Steve La Rocque and Penny Nom.
The derivative of f(x)=1/(x-1) 2007-09-21
From Michelle:
im having trouble finding the derivative of f(x)=1/(x-1) using the f(x+h)-f(x)/h method.
Answered by Stephen La Rocque.
Algebraic fractions 2007-09-21
From Yvonne:
Would you please help me on the following 2 questions?

1. Given that 3x-5y / 2x-y = 2/3, find the value of 2x/5y.

2. If 3x-5y / 7x-4y = 3/4, find the value of x/y.

Answered by Penny Nom.
Solving an equation with fractions 2007-09-20
From Len:
I am having a brain cramp or maybe I just forgot some basic math, but I am having trouble solving for "r" in your truncated cone example where r/(r+w)=r/R or r/(r+282)=911/1728. Could you refresh my memory by showing the steps to solve for "r"?
Answered by Harley Weston.
Prove that any two consecutive integers are relativley prime. 2007-09-18
From Michael:
Im not very good at proofs and I was wandering if you would be able to help me with the following question: Prove that any two consecutive integers is relativley prime. Thanks a million.
Answered by Penny Nom.
A fraction that cannot be simplified 2007-09-17
From Kevin:
Make a fraction that cannot be simplified and has 24 as its denominator?
Answered by Stephen La Rocque and Chris Langdon.
A 5 by 5 checkerboard 2007-09-17
From Darren:
Hi, I'm Darren and i have some questions to ask you about this problem: In a 5 by 5 checkerboard : how many 2 by 2 squares are there, what other sizes of squares do you need to count and how many of of each size of squares can you find; how many squares did you find in all
Answered by Victoria West.
An equation with fractions 2007-09-16
From Stuart:
4/5+3x/8=x/4-1/10
Answered by Victoria West.
how do you simplify 3a + 13b = 37 2007-09-16
From brandon:
how do you simplify 3a + 13b = 37
Answered by Penny Nom.
Differentiate x^(1/3) using first principles 2007-09-14
From Sheila:
our teacher gave us this question as a challenge and even he couldnt figure it out: Differentiate x^(1/3) [aka the cube root of x] using first principles. i know the answer is 1/(3.x^2/3), but how is it possible using first principles?
Answered by Harley Weston.
Two equations in two unknowns 2007-09-14
From CAMEILA:
NEED T0 SOLVE THIS EQUATION

Y = -2X + 6
Y + 6 = 2x

Answered by Stephen La Rocque.
The area of half a circle 2007-09-14
From Heather:
We need the formula for area of half circle please explain where each number comes from.
Answered by Penny Nom.
What are the factors of 111? 2007-09-13
From Miska:
What are the factors of 111?
Answered by Penny Nom.
1-4/square root 3 2007-09-12
From Prudence:
How to solve this question?
1 - 4/√3

Answered by Penny Nom.
How many gallons per minute? 2007-09-12
From Diane:
HI I have a natural water spring and I am trying to determine how many gallons per minute will flow in an 8" pipe? I know one gallon is 231 cubic inches and V=nr2h - so if i had one foot of 8" pipe it would hold 2.6 gallons but I'm looking for the flow rate of how many gallons per minute? Thanks for your help.
Answered by Stephen la Rocque.
Exponential form 2007-09-12
From Ericha:
How do I express 1,679,616 in exponential form as a power of 6 using a calculator?
Answered by Stepehe La Rocque and Penny Nom.
The circumference of a circle 2007-09-11
From annette:
circumference of a 3.75 ft circle I have not done this in years and forgot how.
Answered by Stephen la Rocque and Victoria West.
How much space we are talking about when they say 500 cf? 2007-09-11
From Rhonda:
I was informed while planning a move that the moving company will allow 500 cubic feet of moving space. Anything over that is $5.99 per cf. How do I find out exactly how much space we are talking about when they say 500 cf ?? Thank-you in advance for the advice.
Answered by Stephen la Rocque.
The factors of 35 2007-09-11
From johh:
wats the factors for 35
Answered by Penny Nom.
A sequence of fractions 2007-09-10
From mitch:
find an expression for T(n), the nth term of the sequence
1/5, 3/8, 5/13, 7/20, 9/29

Answered by Stephen La Rocque.
24 * 1/6 2007-09-09
From Laurie:
24 * 1/6

I don't no where the 24 goes and if the number must be the same to devide into the numerator and the denominator . Thankyou for your time.

Answered by Penny Nom.
Find All 24 Factors Of 360 2007-09-08
From Jennifer:
Find All 24 Factors Of 360
Answered by Penny Nom.
The slope of a line 2007-09-06
From Danielle:
Hello. I need some help with this: i am supposed to find the equation of a line with a point (2/3,5) and (-5/6,-4) the answer in the book says y=6x+1. My question is how did they get for the slope?
Answered by Penny Nom.
2x+1+x-4+4x+1 2007-09-06
From laurie:
2x+1+x-4+4x+1
Answered by Penny Nom.
0.24 1/2 2007-09-06
From Kenneth:
The fraction, 1/2, in 0.24 1/2 occupies the 1/100 place along with the 4.
If the fraction, 1/2, is changed to a decimal, as in 0.245, the last 5 in 0.245 occupies the 1/1000 place.
Why doesn't the 1/2 in the decimal 0.24 1/2 occupy the 1/1000 place (0.001) instead of the 1/100 place?

Answered by Stephen La Rocque.
Is x/y + y/x = 2 a function? 2007-09-06
From Farzan:
Is x/y + y/x = 2 a function or not.
Answered by Penny Nom and Stephen La Rocque.
How many two digit numbers contain at least one 7? 2007-09-06
From Janet:
How many two digit numbers contain at least one number seven?
Answered by Penny Nom.
Quotient of proper fractions 2007-09-06
From Tim:
The question asks me to find a counter example for: the quotient of 2 proper fractions is a proper fraction
Answered by Stephen La Rocque.
Prime factors in exponential form 2007-09-06
From Michelle:
how would you write this: factor 48 as a product of primes written in exponential form
Answered by Stephen La Rocque.
Liquid in a pipe 2007-09-05
From christian:
how many cubic feet are in a pipe that is 3" (inside diameter) and 459 feet long?
Answered by Stephen la Rocque.
A sequence of fractions 2007-09-05
From Arjun:
1/2 3/5 5/8 7/11- need to find the nth term.

I did search the data base & found one for fractions but what I want to know is when calculating nth term for the denominator in the example give in your database how do we get (n-1)? When we deduct the actual term with the one that is in the table give in your example it is more that one. Could you please explain solving the above example?

Answered by Penny Nom.
Exponential form 2007-09-05
From jeanette:
the African bush elephant is the largest animal and weighs 8 tons. write this amount in exponential form.
Answered by Stephen la Rocque and Penny Nom.
Factoring a quadratic 2007-09-04
From Arlene:
Question from arlene, a student: x^+2x-24=0
Answered by Stephen la Rocque.
Speed of sunrise 2007-09-04
From Robert:
Facing west in a car, you are watching the sun rise (through the rear view mirror). It peaks the horizon, you take off driving. How fast would you have to drive, to see it rise again?
Answered by Stephen La Rocque.
Four digit combinations 2007-09-04
From Natalie:
Using The Numbers 7 4 5

How many different 4-digit combinations are possible, you can use any of the numbers twice.

Answered by Penny Nom.
A rectangular sheet of paper is folded along the diagonal 2007-09-01
From Amit:
A rectangular sheet of a paper with dimensions a and b is folded along its diagonal. What is the area of the overlapped region?
Answered by Harley Weston.
Is there a way I can calculate correct square footage for odd shapes? 2007-08-31
From Tom:
I manage a golf course, and we are about to over seed the course for the upcoming winter months. We will be over seeding all greens, fairways, and tee boxes. Noone of the area that we will be placing seed on is a regular shape. The closest shapes they would be is a circle, rectangle or square, but most of them are all odd CURVED shapes. I know what the equation is for Square ft., and I know how many Square ft. are in an acre, but I need to be precise with the amount of seed I order. Is there a way I can calculate correct square footage for odd shapes.
Answered by Harley Weston.
Equivalent fractions 2007-08-31
From Darlene:
What is the equivalent decimal for 0.3 and 6.53
Answered by Penny Nom.
Exponential form 2007-08-31
From Victoria:
The cost of each space shuttle is about $10,000,000. Write this amount in exponential form.
Answered by Stephen La Rocque.
Exponential form 2007-08-29
From Emily:
Write the Exponential Form of 125 x 25 and 81 x 27.
Answered by Stephen La Rocque.
The digital lock for a cell phone 2007-08-29
From Hollie:
i have a four digit lock code on my cell phone and cannot remember what it is. i know that it does not contain a 2 or a 4 and none of the numbers are repeated. the numbers range from 0 to nine and the only way to figure out what the code is, is to try randomly. I was wondering if you could give me a list of possible combinations using the information i gave you to make it easier to crack the code.
Answered by Penny Nom.
Rotational energy needs 2007-08-29
From will:
how much energy in KW do i need to start and subsequently rotate a 4 ton fly wheel of 1 metre radius at 12 rpm?
Answered by Stephen La Rocque.
Percentage differences 2007-08-29
From Camilla:
If a store sells a certain syle of skirt, an in the first week they sell 13, in the second week 20, in the third week 17 and in the fourth week 23, calculate the percentage difference between the highest and the lowest weekly sale.
Answered by Stephen La Rocque and Harley Weston.
Fractions, ratios and percentages all mixed together 2007-08-29
From Charon:
Example:
x : 1/4% :: 9 3/5 : 1/200

Answered by Stephen La Rocque.
Counting stitches and changing units 2007-08-28
From Lim:
Jack is making stuffed toys. Each toy needs 360 stitches. Ten stitches take 15 seconds.One toy needs 57cm of cotton for stitches. How many centimetres of cotton does Jack use in an hour?
Answered by Stephen La Rocque.
Factor completely 2x^2-3x-5 2007-08-28
From melissa:
how do you factor this completely:

2x^2-3x-5

Answered by Stephen La Rocque and Penny Nom.
Applications of sequences and series 2007-08-27
From Trish:
I'm a grade 12 learner working on a math project based on sequences and series. I'd like to know the different types of sequences and series such as fibonacci, fourier, farey, etc.

I've already used the Fibonacci Sequence and Harmonic Series and need two more. The simpler the sequence or series type the better.

I'd also like to know in which non-mathematical areas use sequences and series and how. Areas such as engineering or science.

Answered by Penny Nom.
Filling an old swimming pool 2007-08-27
From Russ:
I would like to know how much fill material I would need to fill an old swimming pool. The pool is 18' wide x 36' long and is 4' to 10' deep.
Answered by Penny Nom.
Reference angles 2007-08-25
From Jenny:
find the reference angles for the angles given below, find the quadrants in which the angles lie
1. 0=6n/7
2. 0=3.3

Answered by Stephen La Rocque.
How many cubic feet in a cubic yard? 2007-08-25
From George:
How many cubic feet in a cubic yard?
Answered by Stephen La Rocque.
z+81 = 9z-7 2007-08-24
From Erin:
Question from erin, a student:

z+81=9z-7

Answered by Stephen la Rocque.
Two-column proof for a circle geometry problem 2007-08-24
From Kendra:
i have to prove that tangents to a circle at the endpoints of a diatmeter are parallel by stating whats given, whats to prove and a plane, then write a two column proof i dont understand this
Answered by Stephen La Rocque.
A frustum of a right pyramid 2007-08-24
From Andrew:
Find the volume of a frustum of a right pyramid whose lower base is a square with a side 5 in., whose upper base is a square with a side 3in., and whose altitude is 12 in. Round your answer to the nearest whole number.
A. 47cu in.     C. 226 cu in.
B. 196 cu in.     D. 1036 cu in.

Answered by Stephen la Rocque.
More on a percentage question 2007-08-23
From Dee:
In a previous question, you answered:

"What percentage of $34.51 is $10.71?

I translate this English statement into a mathematical statement (equation) using the following

"what" is the unknown, call it x
"percent" is per-one-hundred, that is over 100
"of" is multiplication
"is" is the verb, which in an equation is the equal sign
Thus the English statement above becomes


x/100 $34.51 = $10.71

Solving for x yields

x = 31.03

Hence the gross profit is 31% of your selling price."

How did you solve for 'x'

Answered by Penny Nom.
Four fours 2007-08-23
From Belinda:
My son came home with a math question that I can figure out. He needs to use only the number 4 and only four 4's, no less no more. The problems need the answers 11,13,14,and 18. He can use any mathematical sign except square root. He's been able to get all but these so far and I've worked them, but can't come up with the problems. Can anyone help?
Answered by Stephen la Rocque.
32,648 square footage = how much land? 2007-08-22
From Elizabeth:
1. What amount of Land = an acre?
2. 32,648 square footage = how much land?

Answered by Penny Nom.
Converting a repeating decimal to a common fraction 2007-08-22
From isabelle:
how do you turn 6.333... into a fraction in simplest form?
Answered by Stephen La Rocque and Penny Nom.
Where do you use trigonometry? 2007-08-21
From jenny:
where do you use trigonometry besides architecture and engineering?
Answered by Stephen La Rocque.
Percentage difference between a microcentury and 50 minutes 2007-08-21
From Wonder:
A lecture period (50 min) is close to 1 microcentury. Find the percentage difference from the approximation?
Answered by Stephen La Rocque.
The surface area of a prism 2007-08-21
From Iliyas:
State the area of prism with logical proof?
Answered by Penny Nom.
A geometry problem 2007-08-20
From samhita:
ABC is a triangle. Let D be a point on side BC produced beyond B such that BD=BA. Let M be the mid-point of AC. The bisector of angle ABC meets DM at P. Prove that angle BAP=angle ACB.
Answered by Chris Fisher.
The area of a five sided lot 2007-08-19
From Karyn:
I have been looking at your site, but I am still confused as to how to figure out the exact square footage of my irregular lot (since it is a gradual fan lot from the front, and triangles out in the back).
Answered by Harley Weston.
Identifying a conic from its equation 2007-08-19
From Robin:
Hi, Do you have any tips how to identify a conic from its equation?
Answered by Leeanne Boehm and Steve La Rocque.
Common factors 2007-08-19
From John:
I have problem factorising 2p^3 - 34p. Please advise me.
Answered by Leeanne Boehm, Steve La Rocque and Penny Nom.
Relations and functions 2007-08-16
From virginia:
determine whether each relation is a function provide reasons for identifying relation
(3, 4) (5, 9) (9, 9) (2, 3)
(0, 0) (0, 1) (1, 4) (2, 4)
(2, 1) (4, 5) (8, 4) (1, 0)
(8, 3) (8, 0) (7, 7) (4, 7)

Answered by Steve La Rocque.
Point-slope form 2007-08-16
From jenny:
write an equation in the point-slope form and the intercept form slope=4,passing through (1,3) slope=8,passing through (4,-1) slope= -5,passing through (-4,-2) slope= -2,passing through(0,-3)
Answered by Leeanne Boehm.
Angle of depression to a football 2007-08-15
From Tosin:
The angle of depression to a football from the top of building 10m high is 17 degrees. Find the distance of the ball from the foot of the building.
Answered by Stephen La Rocque.
How many square feet are in 1.24 acres? 2007-08-15
From Karen:
how many square feet are in 1.24 acres
Answered by Penny Nom.
The length of the third side of a triangle 2007-08-15
From Brooklyn:
What is the equation to find the length of the third side of a triangle if you have the length of A, B, and the angles(s)?
Answered by Stephen La Rocque.
A round reservoir 2007-08-14
From lee:
if you have a 46 acre round reservoir, how far around in feet would that be ??
Answered by Penny Nom.
Adding algebraic fractions 2007-08-14
From John:
Ive completely forgot anything to do with the subject mentioned, so my question is straight to the point..

I need to know how to do the following problem (Preferably do not give me an answer though) (k/3k-8) - (4/k+2)

Answered by Penny Nom.
Explain how you can shift a parabola up two units 2007-08-14
From Marsia:
Explain how you can shift a parabola up two units (y axis).
Answered by Stephen La Rocque.
The tension in a single strand of wire 2007-08-13
From Gene:
Seeking an equation that calculates the tension in a single strand of wire or rope. Conditions: the wire is fixed at both ends, A known weight is suspended from the center of the span. The weight will dipslace the strand downward some distance Y. Let the distance between the fixed ends be X= 500mm. Weight W=0.5kg. Original tension in the strand is T= 10 newtons.Your help sincerely appreciated.
Answered by Gabriel Potter and Stephen La Rocque.
Percentage change 2007-08-13
From Karen:
I need a formula for calculating percentages in Excel. For example, we = sold 4 units of an item one month, and 105 units the next. I need a = formula to calculate the percentage increase in sales units. Also, if = 5251 were sold one month and 651 the next, what is the formula for = calculating the percentage difference? Please help, thank you.
Answered by Penny Nom.
10r^2 - 35r = 0 2007-08-13
From Aranxa:
how do u solve this equation by factoring: 10r^2 - 35r = 0
Answered by Penny Nom.
Twenty dining tables 2007-08-13
From priya:
Utkarsh bought 20 dining tables for Rs 120000 and sold these at a profit equal to the S.P of 4 dining tables.Find the S.P of one dining table.(give the answer in statements)
Answered by Stephen La Rocque.
Can you solve this by factoring? 10r^2 - 35r = 10 2007-08-12
From Aranxa:
how do u solve this equation by factoring:
10r^2 - 35r = 10

Answered by Stephen La Rocque.
Fencing cost 2007-08-11
From Ethel:
If ann wants to put up a fence for her dog around her 75ft x 55ft retangular yard, the hardware store has fencing materials for 7.50 per yard. How much would it cost her to purchase fencing material?
Answered by Stephen La Rocque.
Induction - divisibility 2007-08-04
From Jerry:
How would you prove that for any positive integer n, the value of the expression 3^(2n+2) - 8n -9 is divisible by 64.
Answered by Chris Fisher and Penny Nom.
Factorial fraction 2007-08-03
From Sekhoane:
Expand completely: (N-2)!/N!(9N-1)!
Answered by Stephen La Rocque.
What fraction of the letters of the alphabet is each word? 2007-08-02
From Clayton:
How do I figure this out, the math question has the answers but I have no idea how to get it. my Mom can't figure it out either.

What fraction of the letters of the alphabet are each word
Man = 3/26
Glasses = 7/26
Integrity = 1/4
computer = 2/9

Answered by Paul Betts and Harley Weston.
A statistics example 2007-08-02
From Claudia:
A particular employee arrives to work some time between 8:00 am - 8:30 am. Based on past experience the Company has determined that the employee is equally likely to arrive at any time between 8:00 am - 8:30 am.

On average, what time does the employee arrive?

What is the standard deviation of the time at which the employee arrives?

Find the probability that the employee arrives exactly at 8:12 am?

Find the probability that the employee arrives between 8:20 am - 8:25 am?

Answered by Har.
The height of a pole 2007-08-02
From lalaine:
Hi, this is my problem.. From a point 50.2 m to the pole, a student measured the angle of elevation to the top of the pole to be 32°. Find the height of the pole if the student's height from his feet to his eyes is about 4 ft.
Answered by Penny Nom.
An irrigation ditch 2007-07-31
From Kevin:
I would like to know how many cubic yards of concrete I would need to fill a section of irrigation ditch that measures 12 inches on the bottom, 40 inches on top, 16 inches high and 20 feet long with a 18 inch diameter, 20 foot long culvert sitting in the ditch (to remain open). I am trying to build a roadway across the irrigation ditch. Thanks very much.
Answered by Penny Nom.
Four digit numbers 2007-07-30
From Spence:
My son has a problem that he has been unable to figure out. I was asked to try, but I am confused on how to get the last two parts. Any help would be great.
Thanks
Spence

How many four-digit numbers can be formed under each condition?

A) The leading digit cannot be zero: 9(10)(10)(10)=9000

B) The leading digit cannot be zero and no repetition of digits allowed: 9(9)(8)(7)=4536

C) The leading difit cannot be zero and the number must be less than 5000: ??

D) The leading digit cannot be zero and the number must be even: ??

Answered by Penny Nom.
Building a garage 2007-07-29
From charles:
I want to build a garage that is 24 feet 4 inches wide by 50 feet long. can you please tell me what the length of one corner is to the other?
Answered by Penny Nom.
A 20,000 gallon fuel bag 2007-07-28
From Kevin:
I have a 20,000 gallon fuel bag Length 32'x Width 29'x Height 4.2'. I need to build a containment around it. If the bag was to rupture the containment would have to hold the 20,000 gallons of fuel plus ten percent. What size would the containment have to be in order to hold the fuel.
Answered by Stephen La Rocque.
Solve y'' + y = 0 2007-07-28
From Shih-ya:
How do you solve y’’ + y = 0
Answered by Stephen La Rocque and Harley Weston.
More on the cardinality of sets 2007-07-27
From Mac:
Can you please help me to find and verify whether the following are finite, countably infinite and uncountable ?
Answered by Harley Weston.
Multiplcation of two negative numbers 2007-07-26
From Brett:
Someone asked a question about multiplication and division of two negative numbers yielding a positive result here: http://mathcentral.uregina.ca/qq/database/QQ.09.99/butler1.html I was not fully happy with the explanation b/c I want to give me daughter a real-world example and I can't seem to find one.

The following illustrates why multiplying negative numbers has become difficult to explain:

2 X 2 = 4

----(-4)---(-2)---0---2---4
In this example we start with 2 and then want 2 more of them. When we move across the number line from 2 to our answer, which is four, we have moved only 2 units to the right.

-2 X -2 = 4

----(-4)---(-2)---0---2---4
In this example we start with -2 and then want -2 more of them. When we move across the number line from -2 to our answer, which is four, we have moved 6 units to the right.

How can the phenomenon of multiplying two negative numbers being more powerful than multiplying two positive numbers be explained? -Brett

Answered by Stephen La Rocque and Harley Weston.
Simplifying an algebraic fraction expression 2007-07-25
From Jessica:
How do I simplify b/(b2-25) + 5/(b+5) - 6/b?
Answered by Stephen La Rocque.
Countable and uncountable sets 2007-07-24
From Mac:
Hi, i tried to read few webpages related to the countably infinite and uncountable sets. Even i read few questions from this forum.

But i am not convinced with this explanation. If you have any good book that explains this in layman term, please redirect me to that.
1) Can you please explain what is the difference between these too ?
2) How could you say set of Natural number and set of even numbers are countably infinite ?
N={1,2,3,...} and Even= {2,4,6,...}
When an element in the even set is some 2n, we will map it to 'n'.So now we have a bigger number(2n) right ?
Sorry, i didn't understand that.
...

Can you please help me out to understand that ?

Answered by Harley Weston.
A complex number in polar form 2007-07-23
From roland:
write the given complex number z in polar form lzl(p+qi) where lp + qil=1 for 3 - 4i.
Answered by Harley Weston.
Angle of depression 2007-07-23
From joyce:
hello, here is my problem......
As you stand on a bridge w/c is 100 ft. above the water you are looking @ an approaching barge. If the A of top of the front of the bridge is 29.04 degrees and the angle of depression of the rear is 17.36 degrees . Find the length of the barge?

Answered by Harley Weston.
A triangular pyramid 2007-07-22
From shani:
my homework question is confusing (yr 7 beginner) "How many vertices does a triangular pyramid have." (Mum thinks all triangles are triangular& vertices are sides) I want to be sure.
Answered by Penny Nom.
A football stand 2007-07-19
From jean:
a football stand 150 yd. long has 20 tiers of seats; each has a rise of 2 ft. and tread of 3 ft. it constructed of reinforced concrete with a cross section of 36*54. find the amount of material used in its construction.
Answered by Harley Weston.
What fraction of the company does Mary own? 2007-07-18
From Bridget:
Sam and Mary each owned one-half stock in a printing company. Sam sold 2/3 of his stock to Mary. What fractional part of the printing business does Mary now own?
Answered by Stephen La Rocque.
Fractions of fractions 2007-07-18
From Bridget:
Sam and Mary each owned one-half stock in a printing company. Sam sold 2/3 of his stock to Mary. What fractional part of the printing business does Mary now own?
Answered by Penny Nom.
Angle of depression (declination) between sailboat observations 2007-07-18
From Joyce:
From a cliff 150 ft above a lake, we see a boat sailing directly towards us. The angle of depression of the boat is seen to be 5 degrees and 7 inches and 11 degrees and 18 inches. Find the distance sailed between observations.
Answered by Stephen La Rocque.
Angle of inclination from the horizontal 2007-07-18
From Joyce:
In flying upward for 1260 yards along a straight inclined path airplane rises 156 yards. Find the climbing angle ( the angle of inclination from the horizontal) Thank you in advance
Answered by Stephen La Rocque.
Calculating the area (acreage) of a four sided lot 2007-07-18
From A property owner:
I have a real estate property and the lot size is something I need to find out. I know the lengths of the four sides, but it isn't a rectangle, it is an odd shape. How do I find the acreage?
Answered by Stephen La Rocque.
Pythagoras theorem in daily life 2007-07-17
From sana:
i would like to what are the 5 practical uses of the Pythagoras theorem in daily life??? its for a math project thanx a lot sana
Answered by Penny Nom.
The area of a property 2007-07-17
From Mary:
I currently have an opportunity to sell my property to a business and they would like me to send a proposal to them with details. I need to know the square footage of my property and have tried using your formula from similar questions. The dimensions of my property are as follows
front line facing street: 19.908 m
right side : 37.338 m
back line : 29.779 m
left side : 43.983m
The front line and the right side are at right angles to each other. Can you formulate and reply? Thank you Mary

Answered by Harley Weston.
The domain and range of a quadratic function 2007-07-17
From Linda:
I have been trying to solve this problem but I'm unable to figure it out. How do I find the domain and range of y=-(x+1)^2-3? Please explain...thanks!
Answered by Stephen La Rocque and Penny Nom.
A model for a building 2007-07-17
From Carl:
A model for a building is 50 centimeters long, 38 centimeters wide, and 225 centimeters tall on model 1 centimeter represents 2.5 meters. I need to find the volume of the actual building in cubic meters,and the surface area of the building including ceilings and floors. round to the nearest tenth of a square meter.
Answered by Harley Weston.
Radical 96 plus radical 27 divide by radical 3 2007-07-17
From linda:
radical 96 plus radical 27 divide by radical 3
Answered by Penny Nom.
Trig functions for angles not between 0 and 90 degrees 2007-07-16
From Tim:
My question: Why is the value of a trigonometric function, the same, for an angle over 90 degrees and its reference angle? How are the angle and its reference related? Do they both form a triangle that has equal sides?
Answered by Penny Nom.
Height of a tower from two observations 2007-07-16
From joyce:
An observer wishes to determine the height of a tower. He takes sight @ the top of the tower from A & B w/c are 5oft. apart @ the elevation on a direct line w/ the tower. The vertical angle @ point A is 30 degrees & the point B is 40 degrees. What is the height of the tower? Find the value of x in angle tangent 40 degrees and 30 degrees? Show the solution of the value of x?
Answered by Stephen La Rocque.
Height of an antenna (angle of elevation) 2007-07-16
From Fhay:
An antenna stands on the edge of the top of a 52 story building from a point 320 ft. from the base of the building, the angle of elevation to the top of the antenna is 64 degrees in each story is 12 ft. high. Find the height of the antenna
Answered by Stephen La Rocque.
Finding the area of a circle from its circumference 2007-07-16
From Amanda:
what is the area of a circle with a circumference of 3000 metres?
Answered by Stephen La Rocque.
The two towers (angles of elevation trigonometry) 2007-07-14
From joyce:
The angle of elevation of tower B from the top of tower A is 28 degrees and the angle of elevation of the top of tower A from the base is 46 degrees Find the height of tower A if tower B is 120 m high?
Answered by Stephen La Rocque.
Proving a quadrilateral is a rectangle 2007-07-14
From Sonja:
I was having this discussion with another teacher and we need a third opinion. When you are trying to prove a quadrilateral is a rectangle which method should you use:
  1. Prove the shape is a parallelogram by doing slope 4 times by stating that parallel lines have equal slopes. Then proving a right angle by stating that perpendicular lines have negative reciprocal slopes.
  2. Doing the slope 4 times and stating that the shape is a rectangle because opposite sides are parallel because of equal slopes and it contains a right angle because of negative reciprocal slopes.
I guess the real question is do you have to first state that the shape is a parallelogram?

Answered by Stephen La Rocque.
Comparing flow rates of two pipes 2007-07-14
From Kenneth:
If two water pipes are 3 feet long, but one of them has a 1 foot diameter and the second has a 1 1/2 foot diameter, what simple mathematical method can be used to determine how much faster one pipe can drain water than the other pipe?
Answered by Stephen La Rocque.
Implicit Derivatives 2007-07-13
From Charles:
I need help computing y' by implicit differentiation the question is: y^2 + x/y + 4x^2 - 3
Answered by Stephen La Rocque.
Any regular polygon inscribed in a circle 2007-07-12
From DJ:
Circle with r=12" is inscribed in a regular octagon. What is the length of each octagon segment? Note: Our answer works for any regular polygon inscribed in any circle.
Answered by Stephen La Rocque.
Length of a circle 2007-07-11
From Debra:
i have a radius of 73 inches, i need to know the length of the circle please.
Answered by Stephen La Rocque.
Grams of vodka 2007-07-10
From Andrew:
milliliters to grams..vodka 80 proof?
Answered by Stephen La Rocque.
Derivative of a Function 2007-07-09
From Bob:
What is the derivative of the function a sub n = [n/(n+1)]^n ?
Answered by Stephen La Rocque.
Proof that any side of a triangle is less than half the perimeter. 2007-07-07
From Omkar:
Any side of a triangle is smaller than half of its perimeter, prove this in short ?
Answered by Stephen La Rocque.
Can't find circumference of an ellipse 2007-07-06
From Michele:
I need to figure the circumference of an oval and I know the height and width.
Answered by Penny Nom.
Surface area of a coin 2007-07-05
From Anne:
Assuming both sides of a coin are identical, and a line tangent to the inner circle is 23 mm from edge to edge of the coin, and excluding the edge of the coin in the calculation, what is the surface area of the coin, in square millimetres?
Answered by Stephen La Rocque.
A statistical difference 2007-07-03
From a student:
I am writing a paper for a grad class in which I am comparing achievement levels of boys and girls. I have data that shows the average marks for girls and boys in each grade 12 subject for the entire province. My prof want me to show there is a statistical difference. What do I have to do to show this? I don't have access to the raw data, so don't know the SD. Is there anything I can do mathematically with the data that I have? (I can determine n for each set of data, it ranges between 2400 and about 13000 for each subject area.)
Answered by Chris Fisher and Harley Weston.
Algebraic EquationsAx+By=C 2007-07-02
From Juan:
The general "Standard" form of an equation is Ax+By=C. Re-Write this equation in slope-intercept (y=mx+b) form.
Answered by Harley Weston.
A sand base for a pool 2007-07-01
From Clark:
I am adding sand for a base for a pool. It is 20' round and I would like the sand to be 2" deep. Can you tell me how many cubic feet that would be? Thanks!
Answered by Harley Weston.
How do I convert milliliters(powder form) to grams? 2007-06-28
From ed:
How do I convert milliliters(powder form) to grams
Answered by Harley Weston.
Splitting the profits unequally 2007-06-27
From Maci:
We recently hosted a fundraiser. Not everyone will receive a whole share of the profits because they did not participate the entire time. Several will receive only 1/2 or 1/3 share. How do I divide the profits?
Answered by Stephen La Rocque.
Using Heron's Formula to help maximize the area of a triangle 2007-06-27
From Claire:
Given one side of a triangle is 4 cm and the ratio 1:3 for the other 2 sides. What is the largest area of the triangle?
Answered by Stephen La Rocque and Harley Weston.
An odd number with 12 factors 2007-06-27
From Wilson:
Do you know of any odd number that has 12 factors? Reply back today. Because I have hand in a project tomorrow.
Answered by Stephen La Rocque and Harley Weston.
Edging around a pool 2007-06-26
From Celeste:
I have a 27ft. above ground pool. I want to put edging around it but do not know how many feet of edging I need. Can You help? The edging will be approximately 2 feet away from the pool.
Answered by Penny Nom.
sin|x| and cos|x| 2007-06-25
From Mac:
Can anyone tell me whether sin|x| and cos|x| is differentiable at x=0 ? As far as i know, cos(x) and sin(x) is differentiable at all x.
Answered by Penny Nom and Stephen La Rocque.
Limits as x approaches a constant 2007-06-25
From Mac:
can you please tell me what is the reason they say "denominator is a negative quantity" in the solution 11 and "denominator is a positive quantity" solution 10 ??
If i guess correctly, for solution 10, its because of x^2 in the denominator.

Answered by Penny Nom.
Selling price and revenue 2007-06-24
From nick:
hello, this is my first time asking for help from this website in which by the way, i think is great for everyone. My question is as follows... the relationship between the selling price of a sleeping bag, s dollars, and the revenue at that selling price, r(s) dollars is represented by the function: r(s)= -10s^2+1500s
evaluate, interpret, and compare: a) r(29.95)

Answered by Stephen La Rocque and Penny Nom.
Exponential form: x^y 2007-06-22
From Kishor:
whats the easy way to calculate X raised to Y where y is much greater than x.
Answered by Stephen La Rocque.
Simplify 2007-06-21
From A student:
what is the answer of 3y+5/2(4y+1)+5y/2
Answered by Penny Nom.
Rewrite 2x+4y+2=3y+5 in general form 2007-06-21
From dawn:
how do you rewrite 2x+4y+2=3y+5 in general form?
Answered by Stephen La Rocque.
Simplifying complex denominators 2007-06-21
From Krys:
How do I simplify completely? ((4+i ) / (3+i )) - ((2-i ) / (5-i ))
Answered by Stephen La Rocque.
Factor X^6 - 64 2007-06-20
From Krystal:
How do I Factor X^6 - 64 into linear factors?
Answered by Penny Nom.
Making a box with a particular volume 2007-06-20
From Kimberly:
How do you convert 487 cf into box?
Answered by Stephen La Rocque.
Period of a sum of trig functions 2007-06-17
From Aakash:
the period of the function f(x)=cos3x+sin4x+tan4x
Answered by Stephen La Rocque.
What is the 100th term? 2007-06-15
From George:
I have the sequence 50,120,235,406,644

the 1st difference is 70,115,171,238
the 2nd difference is 45,56,67
the 3rd difference is 11,11


i used the a*n^3+b*n^2+c*n+d rule but the a turns out to be a decimal because 11/6 is 1.833333333 and so i cant find the formula for the 100th term term.

Answered by Penny Nom.
Simplifying a quartic rational expression using long division 2007-06-14
From Megan:
x+2/12x^4+17x^3+0x^2+8x-40=
Answered by Stephen La Rocque and Penny Nom.
Circumscribing a golden cuboid with a sphere: surface areas 2007-06-14
From Ainslie:
A golden cuboid is defined as a rectangular prism whose length, width and height are in the ratio of phi : 1 : 1/phi.
Prove that the ratio of the Surface Areas of the golden cuboid to that of the sphere that circumscribes it is Phi : Pi.

Answered by Stephen La Rocque.
Angles of depression 2007-06-13
From Phonda:
The pilot of a small private plane can look forward and see the control tower for a small airstrip. Beyond that is a large factory that is 3 milies from the airstrip. The angles of depression are 12.5 degrees and 4.8 degrees respectively. Find the airplane's altitude, to the nearest ten feet.
Answered by Stephen La Rocque.
The size of a lot 2007-06-13
From Larry:
This is driving me crazy, I know i learned this in college but that was so many years ago haha. I am trying to determine how many acres my property is. Unfortunately everywhere i look is like another language to me when seeing these formulas. My property is square shaped so starting from one side and just working around the dimensions are: 114 feet x 95.88 feet x 100 feet x 91.91 feet.
Answered by Penny Nom.
What is the size of the circle it would make? 2007-06-13
From ed:
i have a piece of sheet metal 56' 6" long what is the size of the circle it would make
Answered by Penny Nom.
Percentage increase and decrease 2007-06-13
From Anna:
I just want to if the following formula is used for the computation of a PERCENTAGE DECREASE:

Old Value - New Value/ Old Value x 100%

My immediate superior is under the impression that the said formula works ONLY for the computation of a percentage INCREASE. He believes that the formula for a percentage DECREASE:

Old Value - New Value/ NEW Value x 100%

On this score, I want to know the correct formula for the computation of percentage decrease. Thank you so much.


Answered by Penny Nom.
Express a certain variable as a function of another variable 2007-06-12
From Gilligan:
The volume V of a right circular cone is V = (1/3)pi(r^2)(h). If the height is TWICE the radius, express the volume V as a function of r.
I see the phrase "express a certain variable as a function of another variable" a lot in math books. What exactly does that mean? Is there another way to say the same thing?

Answered by Stephen La Rocque.
Composition functions 2007-06-12
From Gilligan:
Find functions f and g so that f(g(x)) = H.
(1) H(x) = (1 + x^2)^(3/2)
(2) H(x) = int(x^2 + 1)
I don't know where to start.

Answered by Stephen La Rocque.
Converting dollars to gallons 2007-06-12
From Cynthia:
How do I solve this problem? If a family uses .20 per 100 gallon of water. How many gallons would $10.00 cost, divided amoung 3 people? Thanks, Cynthia
Answered by Stephen La Rocque.
Comparing and ordering fractions 2007-06-12
From virgil:
Order the fractions
2/5, 2/3, 2/7

Answered by Penny Nom.
The area of a quadrilateral 2007-06-10
From Lucy:
Calculate the area of the quadrilateral ABCD. AB= 4.1cm, BC = 7.6cm, AD= 5.4 cm, CD= ? Angle ABC = 117, Angle ADC = 62. Give your answer correct to 3 significant figures.
Answered by Stephen La Rocque and Penny Nom.
The law of sines 2007-06-09
From Felicia:
A parallelogram has one side that is 12.0 cm and one angle that is 65°. The shorter diagonal is 25.0 cm. To the nearest tenth of a centimetre, how long is the other side of the parallelogram? Use the sine law.
Answered by Penny Nom.
Percentage below the monthly quota 2007-06-07
From Greg:
Sales quantities for the month were 12,834 with a monthly quota of 14,796. What is the percentage below the monthly quota?
Answered by Penny Nom.
Rate of change of distance between the clock hands 2007-06-05
From Jonathan:
A certain Clock has a minute hand with a length of 4 inches long and an hour hand with a length of 3 inches long. How fast is the distance between tips of these hands changing at 9:00?
Answered by Stephen La Rocque.
Solving a quadratic equation using the Quadratic Formula 2007-06-05
From Stuart:
Solve the following quadratic equation: 3xsquared-5x-4=0
Answered by Stephen La Rocque.
Probability tree - two switches failing 2007-06-05
From Maura:
draw a probability to show the outcomes of two new switches (a) of both switches being faulty (b)both switches are not faulty (c) that one switch is faulty. The failure rate is 1/10.
Answered by Stephen La Rocque.
What happens when you have zero's on both sides? 2007-06-05
From Lily:
On the substitution method what happens when you have zero's on both sides of the equation? Is that considered no solution or infinitely many?
Answered by Stephen La Rocque and Penny Nom.
Finding the area of a lot without knowing the angles 2007-06-05
From Cristin:
I have a land-locked piece of land. I need to know the square footage in order to determine the acreage. My deed only gives the lengths of its five sides.
Answered by Stephen La Rocque.
A flagpole and a telescope 2007-06-04
From Fabiola:
A telescope is mounted on a tripod 5 ft above the ground and 20 ft from a flagpole. The telescope must be rotated 48° from horizontal to see the top of the flagpole. How tall is the flagpole?
Answered by Stephen La Rocque.
How many faces does a triangle have? 2007-06-04
From Daniela:
how many faces does a triangle have?
Answered by Stephen La Rocque.
A species of beetle has a 3yr life span 2007-06-03
From Gilligan:
A species of beetle has a 3yr life span. Half the females in their first year will survive into the second year the rate from second to 3rd year is 1/3. Each female breeds in the 3rd year of life, producing 6 females and 1 male. No females survive beyond the 3rd year.

Let xsub1,xsub2,xsub3 denote the number of females in the 1st, 2nd and 3rd year.

(a) What will be the number of females in the 1st, 2nd and 3rd years of life next year?

(b)Show that the numbers in each group will return to the same values every 3 years?

(c) Given a total of 3600 beetles, over the 3 age groups, calculate the number in each group?

Answered by Penny Nom.
The difference between two dice 2007-06-02
From smiley:
Two standards dice are rolled. Determine the probability that the difference between the two numbers on the dice is 2
Answered by Penny Nom.
Converting units 2007-06-01
From SAMANTHA:
My hot tub is 61 square feet, how many gallons would it be?
Answered by Stephen La Rocque.
John ate 1/4 of a cake 2007-06-01
From Jemina:
John ate 1/4 of a cake, Maria ate 1/4 of the remaining cake. What was the unit or whole in John's case? In Maria's case? How much of a full pan was there in each case?
Answered by Penny Nom.
Area of an isosceles triangle 2007-06-01
From Josh:
In a previous question answered by Sue regarding the area of a regular polygon you gave a formula for the area of an isosceles. My question is how did you get this formula? Can you please explain to mean the process that you used to get that formula? Thanks
Answered by Stephen La Rocque.
A 5 digit key 2007-05-31
From Michael:
I have a 5 digit key pad lock on my truck. I want to print out all of the combinations to try. pad one=0 pad two =1 pad three=2 pad four=3 pad five =4. can you help me find and print all combinations?
Answered by Harley Weston.
Cubic inches and square feet 2007-05-31
From keith:
is it possible to convert cubic inches to square feet.thanx
Answered by Stephen La Rocque.
The number of grams of sulfuric acid 2007-05-31
From cookie:
Hi,
Can you please let me know if I've done this correctly?

Calculate the number of grams of sulfuric acid in 0.500L of battery acid if the solution has a density of 1.28 g/mL and is 38.1% sulfuric acid by mass.

Answered by Stephen La Rocque.
The intersection of two planes 2007-05-31
From Sarim:
How to find a intersection of two planes?
Answered by Penny Nom.
A bag of concrete 2007-05-29
From rick:
I have a bag of concrete that will cover .525 cubic feet of space. How many square feet will the same bag cover?
Answered by Penny Nom.
Water tank volume 2007-05-29
From Cecelia:
I HAVE A SEMI CIRCULAR TANK AGAINST A WALL MEASURING 6'10" ACROSS THE WIDEST PART AND 3' FROM THE WALL TO THE FRONT OF THE TANK. THE DEPTH IS 14". PLEASE HELP ME TO CALCULATE THE VOLUME OF WATER IN THE TANK.
Answered by Stephen La Rocque.
Factor Completely 2007-05-29
From Ema:
Question 1:
Factor completely. 3x(x – 4) + 5(x – 4)
Question 2:
Factor completely. x^2 + 2x + 3

Answered by Penny Nom.
Fractions, decimals and profit 2007-05-29
From fatima:
how can i change a fraction into a decimal and how do i change a decimal into a fraction................
anther question please what does making a profit have to do with percentage change!!!!!!

Answered by Penny Nom.
A scale factor 2007-05-28
From Chandler:
I am rather confused about finding the area of a triangle they give me the area of the larger triangle it is 25 units but I can't figure out how to find the area of the smaller triangle the scale is 3:5 please help me ASAP. Thanks so much I really appreciate it oh the triangles are similar
Answered by Stephen La Rocque and Penny Nom.
The limit of a rational function 2007-05-28
From Imad:
     3 _______ 3 _______
lim \/ 1 + x -    \/ 1 - x
x->0 ---- ----------------------------
              x

Answered by Penny Nom.
The surface area of a fire hose 2007-05-26
From Vanessa:
Why do we need to find the surface area of a fire hose (cylinder).
Answered by Stephen La Rocque.
More on quadrilateral shape names 2007-05-26
From Don:
If North Americans call a quadrilateral with no parallel sides a trapezium, is a kite merely a special type of trapezium? Can a rhombus be a kite?
Answered by Walter Whiteley and Penny Nom.
The circumference of an ellipse 2007-05-25
From Graham:
I have an ellipse that I need to find the circumference of. It is 40ft at its longest point and 25ft at its shortest
Answered by Penny Nom.
Simplifying a square root 2007-05-24
From Leigh:
I am trying to help my son. Simplify sq root 81 x to the 2nd power y to the 12th power.
Answered by Stephen La Rocque.
System of equations 2007-05-24
From Chris:
Find all real solutions (x,y,z,w) of the system of equations:
2y= x + x/17, 2z= y + y/17, 2w = z + z/17, 2x= w + w/17

Answered by Penny Nom.
Mulch price comparison (price per cubic yard versus price per cubic foot) 2007-05-23
From Dianna:
This is a 2-fold question. I am getting some mulch that is measured in 1 cubic yard for $25.00. Walmart has a 2 cubic bag for $3.00. I need to cover a 376 sq ft area. I need to know which is the best deal and how much I need to buy to cover the 376 sq ft.
Answered by Stephen La Rocque.
Factoring a trinomial in two variables 2007-05-23
From Ema:
How do you factor this completely. x^5y^3-3x^4y^2-28x^3y
Answered by Stephen La Rocque.
Using a stem and leaf plot 2007-05-22
From Maegan:
The students in Mr.Smiths PE class collected data on the number of sit ups the students were able to do in three minutes. Use the data to create a stem and leaf plot. What is the median for the class data? range? mode?
Answered by Penny Nom.
Parabolas in the real world 2007-05-18
From Katherine:
Hi, my name is Katherine, and my mean old math teacher just assigned us a test in which we have to write two examples of how parabolas are used in the real world, each one page single spaced, size 12!! I know you have already answered some questions like this, but I still don't understand the whole baseball thing, and any other way parabolas are used. And how I can write a whole page on it. But that's my problem, not yours, I just need help with a little explanation on how parabolas are actually used today. I know this might be kind of confusing for you, but imagine how it is for me!!
Answered by Stephen La Rocque.
Using FOIL for (2x - 6)^2 2007-05-18
From TONI:
I am having a problem finding examples of how to solve (2x-6)^2.
Answered by Stephen La Rocque.
Lateral area of a right cone 2007-05-17
From Crystal:
In my homework the question says the lateral area of a right cone is 226.08 cm cubed. the slant hieght is 12 cm. Find the total surface area. How do I do that?
Answered by Stephen La Rocque.
Hexagonal pyramids 2007-05-17
From Tracey:
How many faces, edges, and vertices does a hexagonal pyramid have?
Answered by Stephen La Rocque.
Percent difference 2007-05-17
From Carolyn:
What is the % difference between these two numbers 211373 & 185420
Answered by Gabriel Potter, Penny Nom and Steve La Rocque.
4 digit lock 2007-05-16
From Kathy:
Hi, My name is Kathy and I'm a REALTOR, I have 2 different lockboxes that I cannot remember the combinations for, so what I need is all the different combinations for 4 digits that cannot be the same, i.e. 0123. Can you help me out? I would hate to throw them away as they cost about $30 each. Thank you, Kathy
Answered by Stephen La Rocque.
What's a vertex? What's an edge? 2007-05-16
From Sophie:
I don't understand what is a vertex and what is an edge. Please put a picture up and explain. I am a fifth grader.
Answered by Brennan Yaremko and Steve La Rocque.
Surface area of a cylinder 2007-05-13
From Nina:
if a cylinder is 23 ft high with a 12 ft radius, how do I find the surface area?
Answered by Penny Nom.
Find the sample size needed 2007-05-13
From Mini:
Find the sample size needed to be 98% confident thata marketing survey on the proportion of shoppers who use the internet for holiday shopping is accurate within a margin of error of 0.02. Assume that the conditions for a binomial distribution are met, and that a current estimate for a sample proportion does not exist.
Answered by Penny Nom.
Find the equation of a line passing through two given points 2007-05-12
From Kenzie:
I am having problems solving an equation of the line passing through (-2,4) and (6,0)
Answered by Penny Nom and Stephen La Rocque.
Angle of Elevation 2007-05-10
From Micky:
Two Buildings are on opposite sides of a street 40 feet wide. The taller of the two buildings is 580 feet tall. The angle of depression from the top of the tallest building to the shorter building across the street is 57 degrees. Find the height of the shorter building.
Answered by Stephen La Rocque.
How many faces on a prism? 2007-05-10
From Samad:
How many faces does a rectangular prism have?
Answered by Penny Nom.
Area of a pentagonal lot 2007-05-09
From Mimi:
I would like to know the square footage of a pentagon with the following measurements: Property is .43 of an Acre Front: 57.11' NW: 105.78' S: 120' SE: 145' E: 102.86' I don't want you to try to show me how to work thru the problem since I will never comprehend it. For some reason, I am unable to perform these kinds of equations. I will greatly appreciate your help.
Answered by Penny Nom.
Combinations 2007-05-09
From Michael:
Show that 5 X C(n,5) = n X C(n-1,4)
Answered by Stephen La Rocque.
Probability of getting an A 2007-05-09
From Christine:
In a class of 15 people, exactly 3 got an A. If 2 people are randomly chosen from this class, what is the probability that at least one of these 2 got an A?
Answered by Paul Betts.
Ordering fractions 2007-05-08
From Mary:
Can you place these in order? 1/8, 15/64, 15/16, 7/32
Answered by Stephen La Rocque.
Area and a square-based pyramid 2007-05-08
From Nick:
For a regular square pyramid with base perimeter 40 and slant height 50, what is the lateral and surface areas?
Answered by Stephen La Rocque.
Fractions and mixed numbers 2007-05-08
From Amanda:
I am having truble with this problem: WRITE THE FRACTION 8/3 AS A MIXED NUMBER
Answered by Stephen La Rocque.
Converting from mm to cm 2007-05-08
From Amanda:
How do you convert mm into centimeters?
Answered by Gabriel Potter and Steve La Rocque.
Uses of pi 2007-05-08
From Kari:
How is pi used in real life?
Answered by Penny Nom.
Are proofs important in geometry? 2007-05-07
From BJ:
Are proofs very important to know how to do?
My daughter has been in Geometry & the teacher skipped proofs.

Answered by Penny Nom.
Probability of H.I.V. tests 2007-05-07
From Danielle:
A medical test detects H.I.V. Among those who have H.I.V., the test will detect the disease with probability 0.95; among those who do NOT have H.I.V., the test will falsely claim that H.I.V. is present with probability 0.0125. Among those who take this test, 4% have H.I.V. The test is given to Lucille, and indicates that she has H.I.V. What is the probability that Lucille actually has H.I.V.?
Answered by Stephen La Rocque.
A problem involving squarefree integers 2007-05-07
From Andrew:
I was told that if x > y (integers); then x would never exactly = divide y^n (n integer > 1) if (x,y) =3D 1 ; or if x is "square = free". Is the latter true and why?
Answered by Stephen La Rocque and Penny Nom.
The circumference of part of a circle 2007-05-06
From Wallis:
I have more then a quater of a circle, but less than half. What is the circumference if the two side are 7cm and the angle between them is 110 degrees?
Answered by Penny Nom.
Simplify -36x squared over 18x 2007-05-01
From Emily:
Simplify -36x squared over 18x
Answered by Penny Nom.
A couch sliding off a truck 2007-04-30
From William:
A couch with a mass of 1 X 10^2kg is placed on an adjustable ramp connected to a truck. As one end of the ramp is raised, the couch begins to move downward. If the couch slides down the ramp with an acceleration of .70 meters per second when the ramp angle is 25 degrees, what is the coefficient of kinetic friction between the ramp and couch? I drew a force diagram and if I did it correctly I identified the forces involved as "mg" (mass x gravity), "Fn" (normal force) and the "Ff" (frictional force). I know that we have the couch sliding down the ramp a .70 m/s but I don't think this a force and I'm not sure how this info fits into the problem. I know that the formula for calculating the coefficient of friction is Ff/Fn. Based upon the force diagram I drew and calculated Fn to be 1082N. I can't seem to get past this point. How do I determine what the frictional force is?
Answered by Stephen La Rocque.
LCD 2007-04-29
From Jackson:
I am having trouble getting the LCD for the following question. 1 3/8 x 8/9=
Answered by Penny Nom and Melanie Tyrer.
Simplifying a rational expression 2007-04-29
From Tamika:
X^2 + 10X + 24
______________
X + 6
SIMPLIFY EACH RATIONAL EXPRESSION (IF YOU CAN'T SIMPLIFY IT, WRITE ALREADY SIMPLIFIED)

Answered by Stephen La Rocque.
The area of a pyramid 2007-04-28
From Alexander:
Total area of the plate required to fabricate a vessel(pyramid) the base is 0.6mx0.6m and height of 1.0m.
Answered by Stephen La Rocque.
The grade of a shoulder 2007-04-28
From Robert:
I am building a road with a 1.2 meter shoulder. The plan calls fo a 6% shoulder grade. When I use my metric calculator I come up with a different answer when I multiply 3/4 of an inch by 1.2 meters and when I multiply 1.2 meters by .06%?
Answered by Penny Nom.
What is the square root of 729 to the third power? 2007-04-25
From Tori:
What is the square root of 729 to the third power?
Answered by Penny Nom.
The size of a freezer 2007-04-23
From Jacqui:
How many can packages can fit? I have a freezer that is 7cf. But it contains a little tray component. The freezer measurements without the tray is(bear with me, I'm not good with LxWxD) 23 across x 16 W x 25 deep (I'm assuming depth) and the tray is 8 across x 16 w x 15 D. We are trying to see how many 9 L x 6 W x 3.5 D packages can fit in the freezer. Please help me. I'm getting app. 60. I need 84 to be able to fit.
Answered by Penny Nom.
24 four-digit numbers 2007-04-21
From Megon:
Of the 24 four-digit numbers formed from the digits 1,2,3,&4 is there an easier way to find their sum other than writing them all out and totaling them by hand?
Answered by Stephen La Rocque.
A square contains five circles with the same radius. 2007-04-21
From Jamie:
A square has a side length on 1 m. The square contains five circles with the same radius. The centre of one circle is at the centre of the square and it touches the other four circles. Each of the other four circles touches two sides of the square and the center circle. Find the radius.
Answered by Penny Nom.
Vertices, edges and faces 2007-04-19
From Kimberly:
I am trying to help my son find the vertices, edge, and face of a 16 sided figure. I am not sure of the process to use to find these, could you please tell me how to do this?
Answered by Penny Nom.
Fahrenheit and centigrade 2007-04-18
From kyrie:
help ? temperature formula: f and c represent degree's of temperature.
exact formula:
f = 9c / 5 + 32
approx formula:
f = 2c + 30

at what temp in c do these formulae give an equal value for f ?

Answered by Stephen La Rocque.
Ax + By = C 2007-04-18
From Diana:
find an equation in general form Ax + By = C with the following properties passing through points (a,b) and (2a, 2b)
Answered by Penny Nom.
Radical form 2007-04-18
From Cassia:
Hi Um, I had a question for algebra, I'm just curious of how to put x^3/8y^1/4 in radical form. Can you help me?
Answered by Stephen La Rocque.
Scientific notation 2007-04-17
From Cynthia:
Hi I was wondering if their was a shorter way to figure out this problem... The answer has to be in scientific notation.

A machine in a soft drink bottling factory caps 3 bottles per second.
How many bottles can it cap in 15 hours?
3,600 (forgot how I got that) multiplied by 15= 54,000
54,000 multiplied by 3 = 16,2000
answer 1.6times 10^5

Answered by Stephen La Rocque.
Exponential form 2007-04-16
From Cassia:
I was just wondering, how do you write the squared root of 7x(to the 5th) in exponential form? If you could help explain that I'd be grateful. Thanks
Answered by Brennan Yaremko.
Fractions! Fractions! Fractions! 2007-04-16
From Maria:
Why are fractions important for:
1) The study of Mathematics
2) Real life

Answered by Haley Ess.
A gardener has 140 ft of fencing to fence 2007-04-16
From Johann:
A gardener has 140 ft of fencing to fence in a rectangular shaped vegetable garden. To plant all his vegetables, the gardener needs more than 825 sq ft of space. What are the possibles values for the length of the rectangle that would meet the space requirements? You may assume that the variable x represents the length of the rectangle.
Answered by Stephen La Rocque.
Factor by grouping 2007-04-16
From Vincent:
How do you factor

the ( ) are exponents

4b4b(3)+ a(2)b - 4a - ab(4)

I still don't understand if there is no a in 4b4b

Answered by Penny Nom.
A rotated rectangle 2007-04-16
From Graham:
There is a circle inside which from the centre to the top is a rectangle. Size is unimportant. If I the rectangle is at 0 degrees and I know the co-ordinates of the 4 corners I can do a check to see whether a given point x,y is inside the rectangle or not.

Question : If the rectangle is rotated by 50 degrees how do I check then as the lines of the rectangle are not perfectly straight with regards to x and y ie at 0 degrees x changes but y is constant and vice versa. At 50 degrees x and y change together.

Answered by Penny Nom.
The second derivative 2007-04-14
From Gerry:
In mathematical context,what do you understand by the term "Second Derivative"
Answered by Penny Nom.
The area of a heptagon 2007-04-14
From Chantel:
Would you be able to tell me the formula to work out the area of a heptagon? thank you.
Answered by Stephen La Rocque.
The angle of depression 2007-04-14
From Mary-Beth:
Two towers are 30 m apart. From the 15th floor, 40 m up, find the angle of depression to the base of the taller tower? I think the answer is 53, but the answer book says 37.
Answered by Penny Nom.
Factoring by grouping 2007-04-13
From vincent:
How would you factor the number in ( ) are exponents 2b(3)+3a(3)+3ab(2)+2a(2)b = and 4b4b(3)+a(2)b - 4a - ab(4) =
Answered by Haley Ess.
Intersection of a line and a circle 2007-04-12
From gaby:
The sum of two numbers is 9. The sum of the squares of the two numbers is 41. Find the numbers.
Answered by Steve La Rocque and Melanie Tyrer.
A wooden hexagonal pyramid 2007-04-12
From David:
Im trying to form a hexagonal pyramid out of plywood and need to know the height to the center point of the pyramid. I want to use standard 4x8 sheets of plywood and split them diagonally. Then with the resulting triangles combine them to form one triangle. Then i want to accumulate 6 of these triangle total and put together to form a hexagonal pyramid.
Answered by Stephen La Rocque.
An arc shaped groove into a peice of metal 2007-04-12
From daniel:
hello i work at an engineering workshop the other night i was asked to machine an arc shaped groove into a piece of metal the cord length was 6 mm and the height from the middle of the cord to the arc was 1mm i was hoping to find the diameter of the cutter needed to do the job and also the formula to work out how to find the diameter. i believe it is 10mm dia thankyou for your time and knowledge
Answered by Stephen La Rocque.
Swimming pool water 2007-04-12
From tina:
How much water does a 24' round, 4' deep swimming pool hold?
Answered by Haley Ess.
Reducing the over all size of a irregular pentagon 2007-04-11
From Gilbert:
I'm having trouble figuring out how to reduce the over all size of a irregular pentagon. The pentagon I'm trying to reduce has 2 right angles adjacent to each other. Basically the shape of a house. I have width, length of the legs and the height of the peak. But cannot figure out how to get the size of a reduced version that keeps the same proportion and angles. Thanks Gilbert
Answered by Penny Nom.
Who has the most pecans? 2007-04-10
From Oreste:
Julian gather 3/5 of a bag of pecans Nikki filled 5/7 and Brian 5/9 who filled the most pecans.
Answered by Stephen La Rocque.
Yards of sand in a ton 2007-04-09
From Teresa:
how many yards of sand are in a ton and how many yards of limestone does it take to make a ton
Answered by Penny Nom.
A cabinet with an arched front 2007-04-09
From Joe:
I am building an arched front cabinet that is 71 inches wide, 12 inches deep at both ends and 16 inches deep at the center. To accurately build this cabinet I need to known the radius of the circle that would form that arch. Thanks, Joe
Answered by Penny Nom.
Surface area of a sphere 2007-04-08
From maria:
I've read your solution to the problem of how to derive the formula of the surface area of the sphere. But i've got a question that initially you demenstrate the area is apprximately equal to that of the lateral area of the cylinder. Afterwards you simply say that the two areas are exactly equal. I'm very confused, how you connect your two statements together. Can you explain to me how they are exactly equal? Your quick reply would be appreciated. Thanks for your help!
Answered by Stephen La Rocque.
A volume of revolution 2007-04-08
From christina:
find the volume of the solid formed when region bounded by y=x/3, y=2 and the y-axis. it is revolved about the x-axis.

the assignment was to use both the washer method and the shell method but when i solved for the volume, i got different answers. i think my shell method is wrong because i know i'm having difficulties with using "dy" instead of "dx" here's my work so far:

Answered by Penny Nom.
An arched opening for a large doorway 2007-04-08
From Richard:
I am trying to build an arched opening for a large doorway...I know the vertical sides of the opening to be 8'-9" from the floor to the lowest point of the arch on each side...I know it is 15 1/2" from the center horizontal point to the top of the arc...I know the vertical sides are 11'-11" apart...what I need to know is the radius to create the proper arc. Can you help?
Answered by Stephen La Rocque.
3 divided by 3 to it's fifth root 2007-04-06
From Annie:
How do I transform the equation 3 divided by 3 to it's fifth root to simple radical form (getting the radical out of the denominator)?
Answered by Penny Nom.
How much did it originally cost? 2007-04-04
From Megan:
My question is simple:

"-Jim is able to sell a hand-carved statue for $650 which was a 35% profit over his cost. How much did ie originally cost?"

I don't even know where to start could you help?

Answered by Penny Nom.
Game of 24 2007-04-04
From teri:
How do you find the answer of 24 by using 12,24,7 and 2 only once.
Answered by Stephen La Rocque.
Fuzzy sets 2007-04-04
From Vasile:
What is the theory fuzzy and what example and/or applications can give me? Thanks
Answered by Penny Nom.
Simplify (1/3 + 1/3x) / (1/x + x/3x) 2007-04-04
From Farina:
how do i simplify (1/3 + 1/3x) / (1/x + x/3x)
Answered by Penny Nom.
The area of a parcel of land 2007-04-03
From Ricky:
I need the attached land parcel (#0436) converted to acreage. This would be of great help to me. Thanks, Ricky
Answered by Penny Nom.
Understanding fractions and their equivalents 2007-04-02
From Gail:
How can I get fourth grade students to understand fractions and their equivalents?
Answered by Diane Hanson.
Using the "difference of squares" formula how do I compute 27 * 33? 2007-04-02
From Sarah:
Using the "difference of squares" formula how do I compute 27 * 33?
Answered by Penny Nom.
What is the intensity 5m below the surface? 2007-03-31
From david:
I have this question which I am supposed to set it up and solve as a differential equation. I know how to solve the diffrential equation but I am having hard time understanding this question. Here is the question: The intensity of light in the ocean decreases the deeper you dive. In fact, the rate at which the intensity decreases is proportional to the current intensity. Setup the corresponding differential equation and solve for I(Y), the intensity I as a function of current intensity Y. If the light intensity 2m below the surface is 25% of the intensity at the surface, what is the intensity 5m below the surface. Can you please explain to me what does it mean by current intensity and how do I set this equation up. Thanks for the help.
Answered by Penny Nom.
How much gravel do I need 2007-03-31
From linda:
how much gravel do I need for 254 square feet to be 4 inches deep?
Answered by Penny Nom.
A "claw setting" for a gemstone 2007-03-30
From Stephanie:
I'm trying to make a cone out of a flat sheet of metal for a "claw setting" for a gemstone. The cone must be 8mm wide at the top and 11mm long tapering to a point. But because the prongs must be cut out of the top the cone should not start to taper for a length of 3mm from that top 8mm. The 3mm prong is then bent over the 8mm stone. That probably doesn't make enough sense. But I don't know how to explain it. If it helps a claw setting is the very common prong setting for engagement rings or earrings. Please help as soon as possible as this is a commissioned piece for someone and I'm running out of time. I don't remember any math really from high school so please make the instructions really easy to follow. Thank You!!
Answered by Penny Nom.
Golf schedule 2007-03-30
From Jim:
I am organising a golf break for 12 golfers playing in 3 fourballs over 5 days. I would like to maximise the number of different golfers each player can play with. Any help would br greatly appreciated.
Answered by Penny Nom.
What does inverse graph represent? 2007-03-30
From san:
what does inverse graph represent?
Answered by Penny Nom.
Is the inverse of a function always a function? 2007-03-29
From San:
Is the inverse of a function always a function? Please justify. Thank You!
Answered by Penny Nom.
A four digit number 2007-03-28
From Paulo:
Hi , if i have to discover a keypad number of four digit , Which the closest combination of getting right?? i have this tips ---> i know the second and thirth number is odd . dont have the number 7 in any digit , Doesnt begin with 8 or 4 ... and dont have the 2 as a last number . Thanks !!
Answered by Stephen La Rocque.
Names of solid figures 2007-03-28
From Debbie:
What is the name of the solid figure: 1) base with 5 equal sides and faces that are 5 triangles 2) all 6 faces are squares and 3) 2 congruent pentagons for bases and 5 rectangular faces. Please help! Thanks
Answered by Penny Nom.
The foci of an ellipse 2007-03-27
From Brad:
I am trying to figure out how to find the foci of an ellipse x^2/7 + y^2/16 = 1. Since 16 is the largest denominator I know the major axis is going to be the y axis. Do I now take 7-c^2=16. c^2=16-7, c^2=9, c=3. So is my foci (0,+-3).
Answered by Penny Nom.
Factoring a trinomial 2007-03-27
From Kim:
Hi, could you please help me solve this.

3x(squared) +20x - 7

Answered by Leeanne Boehm.
The area of an irregular pentagon 2007-03-26
From grace:
hi I am doing graphics and need to find the area of the section given to us (it is just drawn). the sides measure 15m, 35.4m, 5m, 17.8m, 35.7m. the angle between the 15m side and the 35.4m side is 17degrees, the angle between the 35.4m side and the 5m side is 73degrees. the angle between the 5m side and the 17.8m side is 151degrees, the angle between the 17.8m side and the 35.7m side is 90degrees and the angle between the 35.7m side and the 15m side is 95degrees. if you could tell me the area i would be extremely grateful. thanks very much.
Answered by Penny Nom.
Painting a pool 2007-03-25
From vincent:
Determine the total cost to put 2 coats of paint on the interior pool and amount of water needed to fill pool

facts:
a gallon of paint will cover 50 square feet and cost $18.00 the pool will be filled when water is leveled 6" from the top of pool there are approximately 7.48 gallons of water per cubic foot Rectangle shape pool 36.0 long and 20.0 wide from top

Answered by Stephen La Rocque.
Friction of a skier 2007-03-24
From William:
An olympic skier moving at 20.0 m/s down a 30 degree slope encounters a region of wet snow and slides 145m before coming to a halt. What is the coefficient of friction between the skis and the snow?
Answered by Stephen La Rocque.
The distance between two fire towers 2007-03-23
From tony:
Two fire towers are 30km apart, tower A is due west of tower B. A fire is spotted from the towers, and the bearing from A and B are N76degreesE and N56degreesW, respectively. Find the distance from the fire to the straight line connecting tower A to tower B.
Answered by Stephen La Rocque.
Graphing a line 2007-03-23
From mitchell:
Graph each equation. y = 2x + 4
Answered by Jaymi Peterson and Haley Ess.
The volume of a hopper 2007-03-23
From amitesh:
Let me know how to calculate the tonnage of hopper / bins / chutes of different sizes and dimensions.
Answered by Penny Nom.
The law of cosines 2007-03-23
From chetna:
Q 1) In triangle LMN, l=7, m=5 , n=4. find ANGLE N. After applying the rule and substituting values i'm getting Cos n= 58/40. Is there something wrong. The answer at the back of the book is 34 degrees.
Answered by Penny Nom.
The velocity of an aircraft 2007-03-22
From William:
Hello, I don't understand how to work this problem. Thanks in advance for your help!

A jet moving at 500.0 km/h due east moves into a region where the wind is blowing at 120.0 km/h in a direction 30.0 degrees north of east. What is the new velocity and direction of the aircraft relative to the ground?

Answered by Penny Nom.
Which solid figure has 4 more edges than vertices? 2007-03-22
From dawn:
which solid figure has 4 more edges than vertices?
Answered by Penny Nom.
Angles of depression 2007-03-21
From romaine:
a woman of height 1.4m standing on top of a building of height 34.6m veiws a tree some distance away. she observes that the angle of depression of the bottom of the tree is 35 degrees, and the angle of depression of the top of the tree is 29 degrees. assume that the building and the tree is on level ground.
(a) calculate the distance of the woman from the top of the tree measured along her line of sight.
(b) determine the height of the tree.

Answered by Stephen La Rocque.
What makes a relation a function? 2007-03-21
From san:
What makes a relation a function?
Answered by Haley Ess.
Examples of algebra used in life 2007-03-21
From Arianna:
i need to find about 5 different examples of algebra used in life, and im not sure where to start. does anyone have any suggestions as to topics that i should choose for my project? thank you so much for all of your input. Arianna Leigh
Answered by Penny Nom.
28 golfers playing golf over 7 days 2007-03-20
From ian:
I have 28 golfers playing golf over 7 days I would like everybody to play with each other at least once Is there a schedule? Thank you Ian
Answered by Penny Nom.
n-1/(n+1)! + n+1/n! 2007-03-18
From Cody:
How do you go about simplifying something like this; n-1/(n+1)! + n+1/n!?
Answered by Steve La Rocque and Claude Tardif.
How may litres are in my fish pond 2007-03-18
From beryl:
I need to work how may litres in my fish pond. I have measures it in two sections because of it shape. One section is 1600w, 1100L, 540D. the other section is 1060W, 1800L, 380D. Thank you beryl
Answered by Penny Nom.
What will our square footage size be after appropriation? 2007-03-17
From Melissa:
I have a rectangular lot of 110.0 feet by 396.0 feet. When we sell the District will take 2.38 meters off the front for appropriation. I believe we currently have a 1 acre lot? What will our sqaure footage size be after appropriation? Thanks, Melissa
Answered by Penny Nom.
The Geosphere at the the Epcot Center park 2007-03-16
From stephanie:
The epcot center has a diameter of 265 ft the park needs to paint the park one can will cover 400 sq ft how many cans do they need ?
Answered by Stephen La Rocque.
y-intercept form 2007-03-16
From Ken:
Simple way on how to convert to & from y-intercept form & Std. form. Eg: Convert into y-intercept form: 3x+4y+7=0, Eg.: Convert into Std. Form: y=-1/2x +3, y=50x +300. I would appreciate your help in this matter, as I find it hard to understand. Kind Regards: Ken
Answered by Penny Nom.
Triple angle tangent formula 2007-03-15
From sam:
Hi I am trying to derive a triple angle formulae for tan. I know i need to use compound and double angle formulae but am finding it difficult to "clean" up my fraction to get the triple angle formulae can you show me a worked derivation?! thanks
Answered by Penny Nom.
The area of a property 2007-03-14
From Greg:
Hi my name is Greg. I have a piece of property I am trying to buy, but need to find out the square feet of it first. The four sides are unequal lengths and none at a right angle or parallel. Here are the measurement in feet. The base of the square is 29.12 feet in width.
The right side of the square is 44.33 feet high.
The top of the square is 28.80 feet in width.
The left side of the square is 46.20 feet high.
I have not done this math in ages and really need your help.

Answered by Harley Weston.
A stem and leaf plot 2007-03-14
From Hazel:
Hi, I wanna ask about my son's homework re: reading stem and leaf plots. It's really hard for me to understand about this..please show me how. Ex. Number of dogs competing in dog shows

Stem Leaf
1 62024
2 5295
3 166
4 05

1. What was the least number of dogs at a dog show?
2.What was the greatest number of dogs at a dog show?
3. How many dogs shows are reported in the stem and leaf plot? thanks...Hazel

Answered by Penny Nom.
Angle of elevation 2007-03-13
From Joslyn:
A ship at sea sights a 12m high lighthouse on a cliff which is 80m above sea level. If the angle of elevation to the top of the lighthouse is 27 degrees, calculate the distance from the ship to the shore.
Answered by Haley Ess.
Kicking a field goal 2007-03-13
From san:
A football player attempts a field goal by kicking the football. The ball follows the path modelled by the equation h= -4.9^2+10t+3, where h is the height of the ball above the ground in metres, and t is the time since the ball was kicked in seconds. *the ball must clear the uprights for the field goal to count. The uprights are approximately 5 m high. How long does the ball stay above 5 m in height?
Answered by Penny Nom.
(x+2) is a factor of x^3+bx^2-14x+24. Find the value of b? 2007-03-11
From victoria:
hi my names victoria and i have been stuck on this question for ages can you please help me! (x+2) is a factor of x^3+bx^2-14x+24. Find the value of b?
Answered by Penny Nom.
2x/3+x-5/4=1/6 2007-03-10
From Nomvuyo:
2x/3+x-5/4=1/6
Answered by Penny Nom.
The period of a simple pendulum 2007-03-10
From Melissa:
The period of a simple pendulum of length L feet is given by: T=2pi(sqrt(L/g))seconds. It is assumed that g, the acceleration due to gravity on the surface of the earth, is 32 feet per second per second. If the pendulum is a clock that keeps good time when L=4 feet, how much time will the clock gain in 24 hours if the length of the pendulum is decreased to 3.97 feet? (Use differentials and evaluate the necessary derivative at L=4 feet.) Answer is in seconds. Melissa
Answered by Penny Nom.
If the product of A,B,C is 15834, what is A+B+C? 2007-03-10
From rachel:
if the product of A,B,C is 15834, what is A+B+C?
Answered by Penny Nom.
A truncated cone 2007-03-10
From Russell:
Hello, I have attempted to use two of your answers already given and had no real success. This young lady is making a cat food dispenser using a truncated cone. The top of the cone as a diameter of 5 inches with a height of 6 1/2 inches and diameter of 3 inches for the bottom.

Could you please map out a solution for the both of us to understand? Thank you so much for your time and for this wonderful service.

Russell

Answered by Penny Nom.
What is pi? 2007-03-09
From Billy:
What is pi?
Answered by Jaymi Peterson and Sara Ulmer.
The game of 24 2007-03-09
From colleen:
We are playing the 24 math game where four numbers need to combine using any sign (+, - X, \) to equal 24. The four numbers are 22, 21, 6, 20.
Answered by Penny Nom.
A fountain as a parabola 2007-03-08
From Emily:
I have to do a math project proving that something in real life is a parabola. I really need some help here because i don't know where to start. I want to do it on a fountain and prove it's a parabola but how do i do that? I would really appreciate it if you could help Emily
Answered by Stephen La Rocque.
Chords and arc lengths 2007-03-08
From Angela:
my dad, who is a welder, asked me a question pertaining to chords and points on an arc to which I cannot for the life of me find an answer or an equation. if you could help, it would be much appreciated. I am sending an attachment of the problem.
Answered by Stephen La Rocque.
Rotor-ride 2007-03-07
From Jessie:
In a "Rotor-ride" at a carnival, people are rotated in a cylindrically walled "room". The room radius is 4.6m and the rotation frequency is 0.550 revolutions per second when the floor drops out. What is the minimum coefficient of static friction so that people will not slip down? People on this ride say they were "pressed against the wall". Is there really an outward force pressing them against the wall? If so, what is its source? If not, what is the proper description of their situation (besides scary)?
Answered by Stephen La Rocque.
An even function 2007-03-07
From Dana:
Find a function t(x) such that t(x)/x+3 = y is an even function
Answered by Haley Ess.
4th grade equivalent fractions 2007-03-07
From Raymond:
5/8=?/16 ?=?
Answered by Melanie Tyrer and Sara Ulmer.
An odd function 2007-03-06
From Dana:
If g(x) has a domain of all real numbers, name one point on a graph and explain why it must be there. g(x) is an odd function.
Answered by Penny Nom.
How much fill I would need 2007-03-06
From Tracey:
Oh am I confused! I need to know how much fill I would need to cover an area that is 28 feet in diameter by 8 inches deep. Thank you ...
Answered by Penny Nom.
A metal shroud for a outdoor fireplace 2007-03-06
From Arnold:
I am making a metal shroud for a outdoor fireplace, it is basically a lampshade type pattern,like the bottom of a cone.The top has to be 6 inches to fit the 6 inch stovepipe,and the bottom will be a 24 inch circle. the sides will be 18 inches in length.With the cost of the sheet metal,I can only afford to cut this out once,can you help me with the pattern ?
Answered by Penny Nom.
One acre of water, one foot deep 2007-03-05
From Jeff:
If you have one square acre of ground flooded with one foot of water and a pump that pumps 1000 gallons a minute how long would it take to drain the one acre of ground?
Answered by Penny Nom.
A counterfeit coin 2007-03-05
From Delian:
Hi there, I have a counterfeit penny problem - however this one is slightly more challenging. I have twelve pennies, and one of them is counterfeit. The only difference between the counterfeit coin and the real ones is that the counterfeit is EITHER heavier or lighter than the others (We don't know which). I need to be able to solve this in only three weightings, and I REALLY NEED HELP!!!
Answered by Penny Nom.
Area of a five-pointed star 2007-03-05
From Ashley:
What is the area of a five pointed star inscribed in a circle with a radius of 10cm?
Answered by Stephen La Rocque.
Scientific notation 2007-03-05
From mackenzie:
How would you answer this question? Write 0.000661 in scientific notation?
Answered by Sara Ulmer.
What exactly is a factor of a number? 2007-03-04
From ASHLEY:
What exactly is a factor of a number?
Answered by Penny Nom.
The factor theorem 2007-03-04
From Sophia:
hi this is sophia, 10th grade from the philippines. im really having a hard time in factor theorem im just asking if you could help me solve these problems because i really dont have an idea on how to do solve these.. please

find the values of A and B so that (x+3) (x-2) are both factors of x^3+Ax^2+Bx-12

and

find the values of A, B, and C so that (x-3) (x+1)and (x-2) factors of P(x)=x^4+Ax^3+Bx^2+Cx+12

please help me.. i really do not understand these problems

Answered by Penny Nom.
A long narrow cone 2007-03-04
From Amy:
I am an art student making a piece of jewelry out of a flat metal sheet. I'm trying to make a long narrow cone that's roughly 3 1/4" long and just slightly under 1" wide at the base. Since I'm no math wiz, I'm having a really hard time. Please help.
Answered by Penny Nom.
The percentage difference between 64.24% and 74.24% 2007-03-03
From M:
What is the percentage difference between 64.24% and 74.24%?
Answered by Stephen La Rocque and Penny Nom.
Slope 2007-03-02
From Lacey:
My algebra 2 class is researching graphing and slope and we would like to know how we use graphing and slope in everyday life. So if you could get back to me with some real world examples i would really appreciate it. Thanks for your help!
Answered by Steve La Rocque, Penny Nom and Sara Ulmer.
Finding the x-intercept 2007-03-02
From Ash:
Y = -1/100X^2 =27/10X +0 = 0 THE Y INTERCEPT IS 50 HOW DO I FIND THE X INTERCEPT
Answered by Gabriel Potter.
y = f(x) and y = 2f(x-6) 2007-03-02
From carl:
if P(4,-5) is a point on the graph of the function y=f(x), find the corresponding point on the graph of y=2f(x-6).
Answered by Penny Nom.
How many yards of concrete? 2007-02-28
From jeremy:
I need to figure how many yards of concrete to use. I work for an asphalt company, and we are expanding into concrete. Currently we measure everything in sq. ft. So we came up with 6,960 sq ft of concrete to remove and replace. Currently there is 6" of concrete, and 2" of bedding material. We would like to replace only 4.5" of concrete, the rest in bedding. Here is how I figured, am I correct. 6,960 Sq ft (to simplify I made the Sq ft 350 X 20 ) 350 ft X 20 ft X .33 (1/3 ft or 4") = 2,310 cubic feet 2,310 cb ft / 27 = 85.55 cubic yards Is this correct?
Answered by Penny Nom.
sin(3a), cos(3a) and tan(3a) 2007-02-28
From mailene:
hi, i...indeed,to..need..your..help
how..cn..i..prove..this,formula???
sin3a=3sina-4sin^3a
cos3a=4cos^3-3cosa
tan3a=3tan-tan^3a /1-3tan^2a

the..symbol...^is..the..expOnent

Answered by Haley Ess and Penny Nom.
Adding fractions 2007-02-27
From real:
A person spends 3/7 of her monthly earnings on rent and 1/7 on food. How Much does she spend on food and rent? how much does she have left? How do u work out these questions
Answered by Pam Fowler.
Four digit numbers 2007-02-26
From Anton:
Hello! I would like to ask you how many combinations are possible using 4 digit number. # # # # (1...9).There is no zero,numbers don't repeat,second and forth are odd.Thanks
Answered by Penny Nom.
At what rate is the area of the triangle changing? 2007-02-24
From mac:
two sticks 3.5 feet long are hinged together and are stood up to form an isosceles triangle with the floor. The sticks slide apart, and at the moment when the triangle is equilateral, the angle is increasing at the rate of 1/3 radian/sec. At what rate is the area of the triangle increasing or decreasing at that moment?
Mac

Answered by Penny Nom.
Circles 2007-02-22
From Erika:
I have a research paper due on real life uses of conic sections I've looked through all your conic topics and uses of them, but and i cant seem to find real life uses for circles. What are real life uses of circles?
Answered by Penny Nom.
Surface area of a triangular prism 2007-02-21
From Katrina:
What is the formula to finding the surface area of a triangular prism?
Answered by Gabriel Potter.
Find the area of the triangle 2007-02-20
From Christina:
Graph the function f(X)= x+1/x-1 and graph the tangent line to the function at the points A:(2,3) and B:(-1,0). The point of intersection of the two tangent lines is C. Find the area of the triangle ABC.
Answered by Stephen La Rocque.
The surface area of a hexagonal prism 2007-02-19
From Cari:
How do you find the surface area of a hexagonal prism? Thank you.
Answered by Penny Nom.
Surface Area of a cone 2007-02-19
From Cari:
I am doing a math project. I am very confused on how to find the surface area of a cone. I have looked at other equtions but i still don't understand. How do you find the surface area of a cone that has a 15cm length and a 3.5cm radius?
Answered by Penny Nom.
Weightng coins 2007-02-18
From Gary:
You are considering buying 27 silver coins that look alike, but you have been told that one of the coins is a lightweight counterfeit. Find the least number of weighings on a balance scale that you can use to be certain you have found the counterfeit coin.
Answered by Penny Nom.
Factoring polynomials 2007-02-14
From Joe:
I am in the eighth grade, and we are learning the equivalent of Algebra 2. I have no ides how to factor (x-2)(x^2-1)-6x-6 You help is most aprreciated. Thank you! Joe
Answered by Stephen La Rocque.
Things that are shaped like a parabola 2007-02-14
From carra:
i can't find other examples of things that are shaped like a parabola except for bridges.............. Pls. help it is due tomorrow. thank you very much:)
Answered by Steve La Rocque, Penny Nom and Walter Whiteley.
What constitutes a face for a geometric solid? 2007-02-14
From Erica:
What constitutes a face for a geometric solid? We have been getting conflicting answers from different textbooks. Some teachers are saying that cylinders and cones do not have faces. I'm thinking that cylinders have 2 and cones have 1. Please enlighten me. Thanks!
Answered by Walter Whiteley.
Cost per square foot 2007-02-14
From llinden:
how do you figure out the cost of square footage? the amount per sq ft is $28 Thank you
Answered by Gabriel Potter.
Trig - Ferris wheel 2007-02-13
From Anthony:
A ferris wheel is 250 feet in diameter and revolves every 40 seconds when in motion. Your step up to seat on the wheel at the bottom 2 feet above the ground so you are sitting 4 feet above the ground to start. Derive the formula for the height of your seat at time (t). If I go three times around, how long is the ride in ditance traveled?
Answered by Stephen La Rocque and Penny Nom.
A person jumps from the roof of a house 2007-02-13
From Jessie:
A person jumps from the roof of a house 3.9m high. When he strikes the ground below, he bends his knees so that the torso decelerates over an approximate distance of 0.70m. If the mass of his torso (excluding legs) is 42kg, find a) his velocity just before his feet strike the ground , and b) the average force exerted on his torso by his legs during deceleration.

Please provide calculations and answers for comparison.

Answered by Stephen La Rocque.
Algebra 2007-02-13
From B.Wilson:
The teacher has asked to solve the following, but I have never seen this before: 6x/2 = 4x/5
Answered by Haley Ess.
Countable and uncountable sets 2007-02-13
From piyush:
we se that union of countably infinite no of sets having countably infinite number of elements is a countable set we can express p(n) (i.e power set of natural number) as a union of countable infinite number of sets i.e p(n)=s1Us2Us3..... where s1=null s2={1,2,3,4,5..........} s3={{1,1},{1,2},{1,3},..............{2,1},{2,2}........} using the same statement can we prove that power set of natural number is a infinit countable set
Answered by Penny Nom and Claude Tardif.
Exponential form of complex numbers 2007-02-12
From Austin:
When dealing with imaginary numbers in engineering, I am having trouble getting things into the exponential form. The equation is -1+i now I do know that re^(theta)i = r*cos(theta) + r*i*sin(theta). Just not quite understanding the order of operations. Thanks
Answered by Penny Nom.
The surface area of a dome 2007-02-11
From chris:
I need to figure the surface area of a dome which is 96" across and 48" tall. Could you help please? Thanks!
Answered by Penny Nom and Melanie Tyrer.
Large exponents 2007-02-09
From Nick:
I am trying to figure out an extremely large number. It relates to the estimated number of bacterial divisions in 12775 generations of bacteria.
The problem I need to solve is:
2^12775 or 2 to the power of 12,775.

Answered by Stephen La Rocque and Penny Nom.
The area for a pentagon 2007-02-09
From nicole:
how do you find the area for a pentagon
Answered by Stephen La Rocque.
The capacity of a tank 2007-02-08
From lorn:
what is the capacity of a tank height is 110 feet diameter is 24 feet
Answered by Brennan Yaremko.
Chickens and cows 2007-02-08
From Norman:
My grandson is testing me on a question he recently solved.

The question is "A farmer has cows and chickens - there are 252 heads and 81 legs - how many of each animal does he have?"

I immediately thought of the quadratic equations I learned 50 years ago at school, but cannot remember how to do it. It is further complicated by the fact that 1 animal must have 3 legs as the total number of legs is 81 (or perhaps he his trying to catch me out) Any helpful advice would be welcome.

Many thanks
Norman

Answered by Stephen La Rocque and Penny Nom.
An even positive integer cubed minus four times the number 2007-02-07
From Rachael:
I can't figure out the proof or the method to get the proof for this question: any even positive integer cubed minus four times the number is divisible by 48
Answered by Haley Ess and Penny Nom.
Find the next 3 terms of the sequence 2007-02-07
From rose:
find the next 3 terms of the sequence 2,3,9,23,48,87
Answered by Penny Nom.
Equations with fractions 2007-02-06
From Juan and Derek:
3/4-x=1/2
3/4+1/5x+-1/3=4/5x

Answered by Penny Nom.
how many hours are in a fortnight? 2007-02-04
From Rosegretta:
how many hours are in a fortnight
Answered by Stephen La Rocque and Penny Nom.
How to calculate margin of profit???? 2007-02-03
From A student:
If cost price is Rs. 100/- and our selling price is 139/-. I want to know the gross profit & how much margin in this????
Answered by Penny Nom.
14 golfers--6 days 2007-02-02
From tom:
14 of us are going golfing-- over the course of 6 days is there a formula that would allow us to have the chance to play with as many different partners as possible--2 foursome, and two threesomes
Answered by Chris Fisher.
Price of topsoil 2007-02-02
From Tyler:
I need to convert cubic yards to square feet. I have topsoil that is $34.30 a cubic yard and I need to know what the price is in square feet. The depth is 18".
Answered by Penny Nom.
Derivatives 2007-02-01
From Jacob:
Find the derivative of: y= pi^2+x^2+3xy+sin(y^2)
Answered by Penny Nom.
The volume of a cone 2007-01-31
From ajay:
WHY VOLUME OF CONE IS ONE THIRD OF THE VOLUME OF CYLINDER?
Answered by Penny Nom.
Simplify 18e^2fg/32eg^2 2007-01-30
From Morgan:
how do you simplify 18e^2fg/32eg^2
Answered by Penny Nom.
The height of an isosceles triangle 2007-01-27
From Brendan:
I need to find the height of an isosceles triangle whose angles are 52, 52 and 76 degrees. The base is 100, and the two equal sides are unknown. How would I go about this?
Answered by Stephen La Rocque.
Greatest common factor 2007-01-26
From Alexis:
What is the greatest common factor of 52,and 100?
Answered by Penny Nom.
The area of a ellipse 2007-01-25
From Ranjit:
I have a task in which i have to find the area of a ellipse. i find this difficult because i have only been provided with the perimeter, which is 1000m.
Answered by Chris Fisher.
An algebra exercise 2007-01-24
From Clara:
Given x + 2y = -2 and x - 2y = 18, find:
a) x^2 - 4y^2
b) x^2 + 4y^2

Answered by Stephen La Rocque and Penny Nom.
What is the point of intersection of two sides of a polygon? 2007-01-23
From Tyler:
What is the point of intersection of two sides of a polygon?
Answered by Stephen La Rocque.
A lumber yard receives 24 studs in a bundle 2007-01-23
From Roxanne:
A lumber yard receives 24 studs in a bundle that contains 25 row (or layers) of studs with 20 studs in each row. A 2X4 stus ia actually 1 1/2'' X 3 1/2'' X 92 5/8 Find the cross sectional area of a bindle in square inches Find the volume of the bundle in cubic feet
Answered by Stephen La Rocque.
Is -2^2 = -4 or 4? 2007-01-23
From Joan:
Is -2 squared, when written without parentheses around the -2, -4 or could this correctly be solved by squaring -2 (-2 x -2) for an answer of 4? Or, to correctly get an answer of 4, would the problem have to read (-2) squared?
Answered by Stephen La Rocque.
Maximizing profit 2007-01-23
From Denise:
Total Profit= Total Revenue-Total Cost P(x)=R(x)-C(x) Where x is the number of units sold. Find the maximum profit and the number of units that must be sold in order to get that profit. R(x)=5x C(x)=.001x^2+1.2x+60
Answered by Stephen La Rocque.
The volume of a pool 2007-01-23
From Steve:
how do we measure the cubic feet of a pool. The pool is 50 feet long by 40 feet wide by 4 feet at the shallow end and 9 feet at the deep end.
Answered by Penny Nom.
The square footage of a lot 2007-01-22
From A parent:
I HAVE A LOT THAT IS 50FT. BY 95.5 FEET HOW MANY SQ.FEET.
Answered by Penny Nom.
Simplify (2x)^4 (1/2x^3)^2 2007-01-21
From Sarah:
Simplify (2x)^4 (1/2x^3)^2
Answered by Penny Nom.
Calculating a person's volume 2007-01-20
From James:
Hi i am currently trying to calculate my volume. I have tried to do this using a tank of water and measuring the difference. i worked out that my volume is 112 litres. my question is that i want to make a box that would hold my volume and so i need to convert it into centimeters. i know that there are 100 cm^2 in a litre. if i divide this by 4 i would be 91x91x91ft. is this simply to do with density. if so how could i make this more realistic?
Answered by Stephen La Rocque.
A formula for figuring a tanks volume at various levels 2007-01-20
From Dave:
I know there has to be a general formula to figure out a tanks volume where as you know the total volume but you need to know what it will hold at various levels measured in 1/8th inches in depth and that you may be able to plug this formula into a program like say excel and thus create a chart unique to that tank. But for the lack of me I can not figure it out.
Answered by Stephen La Rocque.
Irregular multi sided polygon 2007-01-20
From Graham:
I have an irregular polygon. I know the length of all the sides and know approximately all the internal angles. Is there a formula or table that can calculate the area for me?
Answered by Steve La Rocque, Chris Fisher and Penny Nom.
Fractional interest in property (fractions of fractions) 2007-01-19
From Threasa:
Please explain how to solve this problem. Someone inherits 1/2 of 1/3 of 1/4 of 1/8 of 8/8 interest in 100 acres of land would equal what. Please show show in detail how to fiqure this answer. I will be working with a lot of fractional interest.
Answered by Stephen La Rocque.
The area of an irregular pentagon 2007-01-18
From Ranjit:
I would like to find the area of an irregular pentagon where the total perimeter is 1000m. Is there a Heron's formula for this?
Answered by Stephen La Rocque.
Answer to skill testing question 2007-01-18
From Mike:
This question was on a ballot. Is the right answer 36 or 1? 5+2-1X6=
Answered by Stephen La Rocque.
Comparing two fractions 2007-01-18
From Kayla:
Why does eight over twelve compared to one half work when you use cross multiplication.
Answered by Penny Nom.
Points of intersection 2007-01-18
From Mark:
Find the points of intersection of the given pair of curves. 2x - 3y = -8 and 3x - 5y = -13
Answered by Steve La Rocque and Haley Ess.
How do you calculate cubic feet from inches? 2007-01-17
From Sue:
How do you calculate cubic feet from inches:
8 1/2 X 11 sheet of paper that is 3/4 inches thick?

Answered by Steve La Rocque and Haley Ess.
The height of a triangle 2007-01-17
From Sharon:
I am needing to determine the height of a triangle with a 50 ft base and one side is 40 ft the other side being 30 ft.
Answered by Penny Nom.
The volume of a frustum of a pyramid 2007-01-17
From Sam:
Find the volume of a frustum of a pyramid with square base of side b, square top of side a, and height h.
Answered by Penny Nom.
The volume of a sphere 2007-01-16
From Anshul:
How to prove or find the volume of sphere?
Answered by Harley Weston.
How much lemon oil do you need to make 10.15L of polish? 2007-01-16
From Sajad:
to make an environmentally safe furniture polish, you can combine lemon oil and mineral oil in the ratio 3:200, by volume. How much lemon oil do you need to make 10.15L of polish?
Answered by Penny Nom.
An octagonal bird house 2007-01-13
From Soren:
I'm in the process of building a birdhouse that is an octagon (based on previous questions, looks like that's a familiar tune). The essential elements are known, but I get stuck when trying to determine the angle for the cuts that would be made to the thickness of the wood so that they all fit together when assembled. Each octagonal section is 7 inches in width and the peak of the roof will be 2 inches higher than the sides. My sense is that the angle cuts that need to be made to the 'height' of each piece of wood. By height I mean the thinnest part of the wood that is neither the length nor the width to use colloquial terms. While it's clear that a slight angle is needed, it would seem that the angle would necessarily change as the distance from the top of any one side to the peak changes. Please advise if more clarification is needed. The 2 inches is random and can be changed if more convenient. Whew!
Answered by Harley Weston.
Input-output table 2007-01-10
From Richard:
Question is make an input - output table for the function using the domain 0,1,2,3,4, Then state the range of the function.

y =x + 8 ?

y =0.4x +3 ?

Answered by Penny Nom.
Fact families with fractions 2007-01-10
From Joe:
I am trying to help my son with a math problem dealing with fact families and fractions. The question only gives 2 numbers 1/16 + 1/12 and 5/4 - 4/5. Can you help me understand how fact families work with fractions?
Answered by Penny Nom.
y = 5 x - 3 2007-01-10
From Richard:
Question is the function y = 5 x - 3 for the given value of x 7 , 1 or 4
Answered by Stephen La Rocque and Penny Nom.
A cylinder has how many flat surfaces, vertices, and edges? 2007-01-09
From Stacy:
A cylinder has how many flat surfaces, vertices, and edges?
Answered by Diane Hanson and Penny Nom.
How much money is that? 2007-01-05
From Stephanie:
The question is asking fill in the whole box. It is set of 15 nickels. It says circle 2/5 of the nickels? The next question ask How much money is that?
Answered by Penny Nom.
Ordering farctions 2007-01-05
From Samantha:
I am having trouble ordering these fractions: 2/15, 1/5, 3/5

And also would like to know what is $6.67 divided by 8 rounded to the hundredths place

Answered by Penny Nom.
Order the fractions from least to greatest 2007-01-04
From Justin:
I am a sixth grader, and I am having trouble with the last question in my homework assisgnment. 1/6, 2/5, 3/7, 3/5!
Answered by Stephen La Rocque and Penny Nom.
The slope-intercept form 2007-01-04
From Earnestine:
Given a line containing the points (1,4), (2,7), and (3,10) determine the slope-intercept form of the equation, provide one additional point on this line, and graph the function.
Answered by Penny Nom.
Annualized 2007-01-03
From Dan:
What would my rate of compound interest be per month? Invested $80,000 and in 150 days have increased that amount by $14,300.
Answered by Stephen La Rocque.
Where do the medians of a triangle meet? 2007-01-02
From Brittany:
Where do the medians of a triangle meet?
1. Inside the triangle
v 2. On the traingle
3. Outside the traingle

Answered by Stephen La Rocque.
6/x-2y - 15/x+y = 0.5 2006-12-21
From Emeka:
6/x-2y - 15/x+y = 0.5
Answered by Stephen La Rocque.
Prove that square root of 3 is irrational. 2006-12-21
From Humera:
Prove that square root of 3 is irrational.
Answered by Stephen La Rocque.
Miles, square feet and acres 2006-12-18
From Emily:
How do you convert miles to square feet? I have to convert 2.86 miles to square feet and then convert 72.5 acres to square feet and add them together. I looked on your website and what does the word hence mean?
Answered by Penny Nom.
The slope-intercept form 2006-12-17
From Anthony:
I'm really confused with this, tried to figure it out on my own but i can't seem to grasp it here's the problem:
Two points are given
a. Write an equation in slope-intercept form for the line containing the two points.
b. Graph the line from Part a and check that both points are on the line.

Answered by Penny Nom.
The product of of two rotations 2006-12-17
From Katie:
Is the product of of two rotations over a different center point always a translation?
Answered by Walter Whiteley.
Four 9s 2006-12-15
From A student:
Can you make 67 with four 9's without using the ceilling method?
Answered by Stephen La Rocque.
cos(n)pi = (-1)^n 2006-12-14
From Idrees:
How can I prove the following: cos(n)pi = (-1)^n
Answered by Steve La Rocque.
The absolute value of imaginary and complex numbers 2006-12-11
From Keith:
i don't get how to find the absolute value of imaginary and complex numbers here is an examples from the text book the answers are given but they don't show the work so i can follow along just show me the work please and explain how it is done

problem
3+4i

Answered by Stephen La Rocque and Penny Nom.
Multiplying fractions 2006-12-09
From Nicole:
When you are Multiplying Fractions why is the answer always smaller?
Answered by Stephen La Rocque.
Fill in the signs 2006-12-07
From Ujwala:
4$4$4$4=20 Fill in the signs in the place of dollor symbol.
Answered by Stephen La Rocque.
Factoring m^4-9^n 2006-12-07
From Josh:
I can not figure out how to completely factor m^4-9^n.
Answered by Penny Nom.
The square footage of an irregular lot 2006-12-07
From Lori:
If I have a rectangular irregular sized lot with the following:

A - 110.53'
B - 159.87'
C - 127.28'
D - 165.86'


How do I determine the square footage?

Answered by Penny Nom.
The volume of an irregular tetrahedron 2006-12-06
From Hai Van:
Could you please show me the way to calculate the volume of an irregular tetrahedron
Answered by Penny Nom.
Star of Bethlehem 2006-12-05
From Stephen:
I need to construct a star for a Christmas play being done at our church. I want it to be a four-pointed star. The top and side points should be 12 inches long and the bottom point 24 inches. It will be flat on the back but raised on the front with lights inside for illumination. Each point will be made of two triangles with a ridge in the center. I would like the width of the rays at the center (widest point or center of star) to be about five inches and the center ridge at this same point to be raised about two inches from the flat back. I need to know the dimensions and angles of each of the 8 triangles that make up the star so that all of them meet properly at the center of the star so as to have the correct height from the flat back.
Answered by Penny Nom.
The height of a triangle 2006-12-01
From Jeni:
I need to know how to figure out the height of a triangle when all I know are the sides and base. The sides are 5 and the base is 8 What do I do?
Answered by Penny Nom.
The game of 24 2006-11-28
From John:
Math 24 with 9, 8, 4, 2?
Answered by Stephen La Rocque and Penny Nom.
What proportion of cups of coffee have at least one defect 2006-11-28
From Ayman:
coffee machine may be defective since it dispenses the wrong amount of coffee (C) & or the wrong amount of suger ( S ) the probability of these defects are P(C) = 0.05, P(S) = 0.04 & P( C&S ) = 0.01 find
1- what proportion of cups of coffee have at least one defect
2- what proportion of cups of coffee have no defect

Answered by Penny Nom.
(x - 5)/3 - 3/2 = (4x + 7)/6 2006-11-23
From Terri:
I have never had algebra before and I am finding it difficult.Can you please help me work this problem out?

(x - 5)/3 - 3/2 = (4x + 7)/6

Answered by Penny Nom.
Percentages 2006-11-23
From Hyaqcinth:
I just do not understand percentages...if the cost of a product is $1.20, how can I make a profit of 55% ? how do I calculate this problem?
Answered by Stephen La Rocque.
Profit margin 2006-11-22
From Kris:
I just started a new sales position and need to calculate profit margins based on my cost of a given product. For example I have a piece of equipment that costs me $102,077.73. I need to figure a profit margin of 18% from cost to arrive at my sale price. What is the best way to do this?
Answered by Penny Nom.
Factors 2006-11-22
From Jacqueline:
Why do the numbers 16,36,48,60,64,90 and 100 have an odd number of factors?
Answered by Stephen La Rocque and Penny Nom.
How do I factor 10x^2 + 17x + 6? 2006-11-21
From Sheldon:
How do I factor 10x^2 + 17x + 6?
Answered by Penny Nom.
The Fibonacci sequence 2006-11-21
From Ross:
Let f0 = 0; f1 = 1,... be the Fibonacci sequence where for all n greater than or equal to 2 fn = fn-1 + fn-2. Let Q = (1+square root of 5)/2. Show that for all positive n greater than or equal to 0, fn less than or equal to Q^(n-1).
Answered by Penny Nom.
The square footage of 1.29 acres 2006-11-20
From Steve:
What is the approx. square footage of 1.29 acres?
Answered by Penny Nom.
Composition of functions 2006-11-19
From RJ:
Let f0(x) = 2/2-x and fn+1 = f0 o fn for n greater than or equal to 0. Find a formula for fn and prove it by mathematical induction. Recall that o represents function composition. i.e., (f o g)(x) = f(g(x)).
Answered by Stephen La Rocque.
Conic sections 2006-11-19
From Joyce:
My son has a project on conic sections. I need the following information on Parabola, Circle, ellipse,and hyperbola. He can't find the following information for each conic section: equations with explanations, four uses for each shape and Shape explanation.
Answered by Penny Nom.
Scale factors and ratios 2006-11-19
From Maggie:
I have a BIG math test coming up, and it's about scale factors and ratios, but i don't understand them at all. How can you tell the difference between scale factors and ratios? and How are they used in the everyday world?
Answered by Penny Nom.
A particle moving along a curve 2006-11-18
From Rachel:
a particle is moving along the curve whose equation is: (xy^3)/(1+y^2)=8/5 assume the x-coordinate is increasing at the rate of 6 units per second when the particle is at the point (1,2).
a. at what rate is the y-coordinate of the point changing at that instant?
b. is the particle rising or falling at that instant?

Answered by Penny Nom.
Composition of functions 2006-11-18
From Oryan:
Given f(x)=-2x^3 and g(x)-4x-5, find g(f(-1))
Answered by Stephen La Rocque.
An aircraft and a missile 2006-11-18
From Sarah:
an aircraft is flying at a constant altitude with a constant speed of 600mph. an antiaircraft missile is fired on a straight line perpendicular to the flight path of the aircraft so that it will hit the aircraft at a point P. at that instant the aircraft is 2 miles from the impact point P the missile is 4 miles from P and flying at 1200 mph. at that instant, how rapidly is the distance between missile and aircraft decreasing?
Answered by Stephen La Rocque.
Wheat is poured on a conical pile 2006-11-17
From Rachel:
wheat is poured through a chute at the rate of 10 cubic feet per minute and falls in a conical pile whose bottom radius is always half the altitude. how fast will the circumference of the base be increasing when the pile is 8 feet high?
Answered by Penny Nom.
The volume of a box 2006-11-16
From Sam:
I have this box to mail and i would like to find out the cubic foot of the box.The W-52 -D-13- H- 37 inch cubic
Answered by Penny Nom.
Sigma from 0 to infinity of (n^3 / 3^n) 2006-11-15
From Cedric:
I'm wondering how you would find if this series converges or diverges?

Sigma from 0 to infinity of (n^3 / 3^n)

Does the n^3 dominate, or does the 3^n dominate? What about higher powers like n^10 / 10 ^ n ? Which one would dominate then?

Answered by Penny Nom.
The fraction 2/3 is found between which pair of fractions on the number line? 2006-11-14
From Anita:
How can i find the answer? What is the formula? The fraction 2/3 is found between which pair of fractions on the number line? A.3/7 and 4/8 B. 1/2 and 3/7 C. 6/7 and 9/8 D. 3/5 and7/9
Answered by Steve La Rocque and Diane Hanson.
How to show my work when I simplify this 2 - 2[4 - 9] ÷ (-5)? 2006-11-14
From Remada:
How to show my work when I simplify this 2 - 2[4 - 9] ÷ (-5)
Answered by Penny Nom.
A system of equations 2006-11-13
From Katrina:
solve the system of equations using either the substitution or the elimination method. 2x-3y= -12 and -4x+6y=15
Answered by Penny Nom.
Some applications of conic sections 2006-11-13
From Burt:
how are circles, ellipses, and hyperbolas used in everyday life
Answered by Penny Nom.
15g/50g^3 2006-11-12
From Nicole:
How do u simplify the variable expression? 15g/50g^3
Answered by Penny Nom.
When kx^2+3x+1 divided by (x-2) the remainder is 3 2006-11-09
From Brianna:
For the dividend, find the value of k if the remainder is 3. (kx^2+3x+1) divided by (x-2)
Answered by Penny Nom.
Piecewise functions 2006-11-08
From Kait:
We discussed how to graph piecewise functions today and I'm very lost!! I'm sitting here staring at this problem that says:
f(x){2x+1, if x <1
f(x){-x+4, if x is greater than or equal to 1.

Answered by Penny Nom.
The slope-intercept form 2006-11-08
From Kilihea:
I need to graph this equation but first need to turn it into a slope-intercept form. 6x - 3y = -3 I would really appreciate it if you could help me turn it into a slope-intercept form.
Answered by Penny Nom.
The rate of change of the perimeter of a square 2006-11-07
From Karli:
Find the rate of change of the perimeter of a square with respect to its area.
Answered by Stephen La Rocque.
A container to hold 600 gallons of water 2006-11-07
From Cissy:
What size rectangular tank would I need to hold about 500-600 gallons of water.
Answered by Penny Nom.
Simplifying exprressions 2006-11-06
From Jessica:
-21(a+2b)+14a-9b
Answered by Stephen La Rocque.
A boat is being pulled towards a dock. 2006-11-06
From Cassie:
A boat is being pulled towards a dock. If the rope is being pulled in at 3 feet per second, how fast is the distance between the dock and the boat decreasing when it is 30 feet from the dock?
Answered by Penny Nom.
Profit margin 2006-11-04
From Beverly:
Our costs to make a product is $73.00. We want to sell this product and make a 30 percent profit margin. What is the selling price? What is the simple formula to follow? Is there a chart we can go to that shows us these percentage of profit breakdowns
Answered by Penny Nom.
Newton meters and ft-lbs 2006-11-04
From Michael:
What is the correlation between Newton meters and ft-lbs of torque? If an engine has 100ft-lbs of torque,what would that equate to in Newton meters?
Answered by Stephen La Rocque.
The sum of two numbers is 30 2006-11-03
From GG:
The sum of two numbers is 30 and the product of the same two numbers is 221. Find the two numbers.
Answered by Stephen La Rocque.
3x+4y+7x+9+2 2006-11-03
From Chavon:
3x+4y+7x+9+2 I don't get it please help
Answered by Stephen La Rocque.
Coefficients 2006-11-02
From Kilihea:
How do you factor the following expression so that the coefficient is 1? 4x+8 and -3x-27
Answered by Haley Ess.
Selling Price using Profit Margin 2006-10-29
From Chris:
What is the selling price of my inventory, find using the profit margin? Inventory A costs $2,300. The profit margin is 55% Inventory B cost is $4,500. The profit margin is 210%.
Answered by Penny Nom.
The surface area of a triangular prism? 2006-10-29
From Amanda:
What is the formula for the surface area of a triangular prism?
Answered by Haley Ess.
An odd number of factors 2006-10-28
From Anthony:
What is the common name used for numbers that have an odd number of factors? This answer should be a two word answer only.
Answered by Stephen La Rocque.
How much labor should the firm employ? 2006-10-28
From Christy:
A dressmaking firm has a production function of Q=L-L(squared)/800. Q is the number of dresses per week and L is the number of labor hours per week. Additional cost of hiring an extra hour of labor is $20. The fixed selling price is P=$40. How much labor should the firm employ? What is the resulting output and profit? I am having a difficult time with this, HELP!
Answered by Stephen La Rocque.
LCM and HCF 2006-10-28
From Henry:
Is there a unique solution to the question: If the LCM and HCF of two numbers are 180 and 15 respectively, what are the two numbers? I got 45 and 60. I got a feeling there are others.
Answered by Stephen La Rocque.
Math and using fingers 2006-10-26
From Portia:
Should students be allowed to use their fingers to learn math and why
Answered by Walter Whiteley.
Four digit combinations 2006-10-26
From Sasha:
CAN YOU LIST ALL OF THE POSSIBLE 4 DIGIT COMBINATION'S, THAT DO NOT START WITH 0?
Answered by Stephen La Rocque.
Ratios, proportions and medicine 2006-10-25
From Steven:
I don't understand why we need to learn about ratios and proportions because i want to become a doctor and i don't know if i'll ever use this type of math.
Answered by Penny Nom, Claude Tardif and Walter Whiteley.
3-6+4X3= 2006-10-25
From Tom:
3-6+4X3=
Answered by Penny Nom.
(2/3)^2/(4/2)/1/3 2006-10-25
From Jena:
how do i simplify this problem? (2/3)^2/(4/2)/1/3
Answered by Stephen La Rocque.
(a^(1/3) – b^(1/3)) ( a^(2/3) + a^(1/3)b^(1/3) + b^(2/3)) 2006-10-24
From Julie:
(a1/3 – b1/3) ( a2/3 + a1/3b1/3 + b2/3)
Answered by Haley Ess.
Water is being pumped into the pool 2006-10-24
From Jon:
A swimming pool is 12 meters long, 6 meters wide, 1 meter deep at the shallow end, and 3 meters deeps at the deep end. Water is being pumped into the pool at 1/4 cubic meters per minute, an there is 1 meter of water at the deep end.

a) what percent of the pool is filled?

b) at what rate is the water level rising?

Answered by Stephen La Rocque.
Simplifying radicals 2006-10-23
From Christin:
i would like for you to solve this problem?? 6 square root 8
Answered by Stephen La Rocque.
Octagonal Stop Sign 2006-10-23
From Phil:
My colleagues and I are in a discussion about how many true "sides" an octagonal stop sign has. I say that it has 10 true sides because of the front back sides of a stop sign (3D octagon). Is this correct terminology or should the front and back "sides" be referred to as faces -- equaling 2?
Answered by Paul Betts, Steve La Rocque and Harley Weston.
Percentage differences 2006-10-23
From Michael:
how do I calculate the percentage difference of a certain box size. for example. How much percent smaller is this box: 8.4mm x 6.3mm to this box: 9.35mm x 7mm
Answered by Stephen La Rocque.
The area of a regular octagon 2006-10-22
From Farhana:
I have a regular octagon with sides of 0.8m and I need to find the area,
Answered by Karen McIver and Penny Nom.
The height of a triangle as a function 2006-10-19
From Ryan:
Let 2s denote the length of the side of an equilateral triangle. Express the height of the triangle as a function of s
Answered by Penny Nom.
How do we use fractions in every day life? 2006-10-19
From Tori:
I am doing fractions in class and we need to write a paper about them. So my question is:"how do we use fractions in every day life?"
Answered by Claude Tardif.
How fast is the water level rising when the water is 1 meter deep? 2006-10-19
From Don:
The cross section of a 5-meter trough is an isosceles trapezoid with a 2-meter lower base, a 3-meter upper base and an altitude of 2 meters. Water is running into the trough at a rate of 1 cubic meter per minute. How fast is the water level rising when the water is 1 meter deep?
Answered by Stephen La Rocque.
How many acres in the following land description? 2006-10-18
From Tammy:
How many acres in the following land description? south 1/2 of the northwest 1/4 of the southwest 1/4 of a section
Answered by Stephen La Rocque.
An octagonal room 2006-10-18
From Rick:
I am a cabinetmaker, and I have a client who has an octagonal room which he would like to be used as a walk-in closet. Each side of the octagon is 60" in length. Each wall section is to have a 24" deep cabinet installed on it. I am trying to figure out the width of each cabinet allowing for a 3" space between cabinets at the front corner.
Answered by Stephen La Rocque.
Putting things in exponent form 2006-10-18
From Sheila:
Write in exponential form 2 sq. root a with an exponent 5.
I do not understand on how to do it.

Answered by Stephen La Rocque.
A dome with a square base 2006-10-17
From Danielle:
I need to know how to calculate the area of a dome that has a square base. In order to insulate the room I need to find the area of it but I am not sure how to do so because there is no formula for domes with square bases. Basically, the floor has 4 corners but the ceiling is dome shaped starting directly from the floor.
Answered by Stephen La Rocque and Penny Nom.
The greatest common factor 2006-10-17
From Myrna:
What is the greatest common factor of 49,164 and 80,860
Answered by Stephen La Rocque.
(2324/4)-525+214 2006-10-15
From Sherin:
(2324/4)-525+214
Answered by Penny Nom.
How many 1 square 3/8 thick pieces of steel can you get from one cubic foot of steel? 2006-10-12
From Jana:
How many 1 square 3/8 thick pieces of steel can you get from one cubic foot of steel?
Answered by Stephen La Rocque.
Plotting percentage change 2006-10-11
From Beckie:
I have worked out a set of percentage differences which are all percentage decrease. Should i express them as a negative percentage as -20%? I also need to plot them on a graph and am not sure whether to just use the percentage difference as it is, in which case i get a positive gradient, or include a negative axis and plot them in this way, in which case i would get a negative gradient. I think the slope of the curve will be the same either way so it might not matter!
Answered by Penny.
I have a 3000 sq ft house ... 2006-10-10
From Ann:
I have a 3000 sq ft house and need to use flea foggers. Each fogger treats 6000 cubic feet. How many do I need?
Answered by Stephen La Rocque.
Four fours 2006-10-08
From Prabh:
Find out 10 BEDMAS problems using order of operation with only four 4's in the problem. The solution must be the digits 1-10. Example 44/44=1
Answered by Stephen La Rocque.
A proof by induction 2006-10-02
From Zamira:
i'm studying induction but i don't get how to proof that 1+2+2^2+2^3+...+2^(n-1) = (2^n) - 1.
Answered by Penny Nom.
What is 1.7142857143 as a fraction? 2006-10-01
From Lizzy:
What is 1.7142857143 as a fraction?
Answered by Stephen La Rocque and Penny Nom.
The focus of a parabola 2006-10-01
From Lily:
I have a mathematical assignment which includes applications of parabolas, hyperbolas and ellipses in the real world. I have been searching the internet and now I am ware that most of the applications of parabolas have a connection with what people call "the focus". However, I do not think I clearly understand what "the focus" of a parabola is. Would you please explain it to me?
Answered by Penny Nom.
Prove that 2nCn is less than 4n, for all positive integers n? 2006-10-01
From Anna:
How can I prove that 2nCn is less than 4n, for all positive integers n?
Answered by Penny Nom.
Family of lines 2006-09-29
From Sasha:
could a "Family of Lines" have both parallel and intersecting equations that aren't related directly but otherwise related by each other to create a curve
Answered by Chris Fisher.
X^1/3-X^1/9=60 2006-09-26
From Harini:
I need the answer for

The value of X in X^1/3-X^1/9=60.
Answered by Penny Nom.

Equivalent fractions 2006-09-25
From Orlanda:
How do I write two equivalent decimals for 5.8?
Answered by Penny.
BEDMAS 2006-09-24
From Partick:
if you have a question like this
-(4)-(-4)+(-4)+4
how do you solve it step by step

Answered by Penny Nom.
How many items must the company produce to begin to make a profit? 2006-09-24
From Devon:
The profit a manufacturing company makes can be found using the formula P=120n - n^2 - 2200 How many items must the company produce to begin to make a profit?
Answered by Stephen La Rocque.
Piecewise functions 2006-09-24
From Claudia:
hi! i was just looking at a question by someone else about piecewise functions, but i still don't get it.
my problem is
g(x){x+2 if x <-2}
g(x){2x - 1 if x> or = -2}

Answered by Penny Nom.
I need to cut an octagon 2006-09-23
From Freddie:
I have a 48 inch square piece of wood that I need to cut into an octagon, help. What's an easy way to just measure and cut it.
Answered by Penny Nom.
Constructing a cone 2006-09-20
From Suresh:
i want to know the required size of plate for cone rolling,sizes are 2950mm is bottom dia,894 is top dia and 600 is height.I have already read u r answers but i little bit confused ,harely and sue have given useful answers but when i worked both method the required plate size is different. so i like to know which method is easy and correct.and also i like know whether it can be rolled without segment my rolling machine width is 1500.
Answered by Penny Nom.
Equivalent fractions 2006-09-20
From Sarah:
I am the aunt of a 5th grader and we were doing math homework and the question was What is the equivalent decimal for .05 and 2.875? I am college education and was completely stumped. Please Help. Thought the question should have been what is the equivalent fraction.
Answered by Penny Nom.
The square footage of my property 2006-09-17
From Jack:
I have been trying to calculate the square footage of my property, without success. The measurements are below. Is there a simple formula that can be used based upon the outside parameters? Or, is there a simple way to calculate the square footage?
Answered by Stephen La Rocque and Penny Nom.
Parameters 2006-09-15
From Chase:
What is the meaning of the word "parameters" when used in reference to Algebra.
Answered by Penny Nom.
Convert a standard form to a slope intercept form 2006-09-14
From Maiko:
I need to know step by step procedures to convert a standard form to a slope intercept form?
Answered by Penny Nom.
Four 4's 2006-09-10
From Ying:
How can I use 4 four times, use any mathematical signs, such as +, -, times, division, ( ), !, square root to get answer 33, 41, or 45?
Answered by Stephen La Rocque.
How do i factor x^3 - 6x^2 + 8x 2006-09-10
From Lauren:

how do i factor

x3 - 6x2 + 8x


Answered by Stephen La Rocque.
what is 8+(-2)(-5)/4-(-1)-4(-2+3) 2006-09-08
From Jordan:
what is 8+(-2)(-5)/4-(-1)-4(-2+3)
Answered by Paul Betts.
The velocity of a pendulum, part II 2006-09-07
From Erin:
We saw the question in your database about the velocity of a pendulum swinging.....It is the same exact question....but there is another question......it says....

"estimate the instantaneous rate of change of d with respect to t when t = 1.5. At this time, is the pendulum moving toward or away from the wall? Explain."

Answered by Harley Weston.
Cubic feet 2006-09-06
From Tom:
How do I calculate cubic feet ? What is the formula?
Answered by Penny Nom.
Converting a decimal into a fraction 2006-09-06
From Andres:
How do i convert a decimal into a fraction?
Answered by Penny Nom.
Metric Conversions 2006-09-02
From Patricia:
I have a new difficulty, metric conversion when it involves the prefixes. Some problems I need help with are:
Answered by Stephen La Rocque.
Rays 2006-08-31
From Natasha and daughter:
My daughter, Natasha, is in 4th grade. She had the following homework question about rays that confused us: Name as many rays as you can in the figure below.
Answered by Penny Nom.
The lower half of a sphere 2006-08-30
From Yvette:
I am interested in finding out how to represent the lower half of a sphere in the form of z=f(x,y) with r=2 and a centre point (3,1).
Answered by Stephen La Rocque and Penny Nom.
How many total worker are in the factory? 2006-08-26
From Sherine:
One quarter of the workers at the factory are clerical, one fifth are technical, half are are administrative,and the remaining 25 are managerial. How many total worker are in the factory?
Answered by Penny Nom.
The area of an irregular octagon 2006-08-23
From Billy:
How do you figure the area of an irregular octagon? Measurements are 4 equal sides of =BD", and 4 equal sides of =BE". It measures 1 1/2" = from flat to flat.
Answered by Penny Nom.
Differentiate Y= sin3x + cos7x 2006-08-22
From james:
Differentiate the function of x using the basic rules.

Y= sin3x + cos7x

Answered by Stephen La Rocque.
Find the divisors 2006-08-21
From Minh:
Using combination or permutation or both, find the
a. Divisors
b.even divisors
of 2160

Answered by Stephen La Rocque and Penny Nom.
16X27+32÷4-26 2006-08-20
From Amanda:
The question was 16X27+32÷4-26.
Answered by Stephen La Rocque.
Points of intersection 2006-08-20
From Gianella:

Find the points of intersection by solving this problem analytically.

y= x cubed
y= x


Answered by Stephen La Rocque.
Find the points of intersection 2006-08-18
From Ingrid:

The question says find the points of intersection of the graphs of the equation, and check your results analytically.

x squared + y squared=25
2x+ y=10


Answered by Stephen La Rocque.
A golf schedule for three days 2006-08-18
From Jon:

I was wondering if I could trouble you to assist me with the following: I am working on a playing schedule for 28 golfers. 7 groups of 4. I have 3 golf days to schedule.

Ideally, I would like to schedule all 28 golfers in 7 different groups of 4 on each day. Here is the catch.....no golfer in any group can be grouped togther more than once. Every group of 4 each day will have 4 new golfers that have never played together before. Is this possible?


Answered by Penny Nom.
Mini Golf 2006-08-17
From Sarah:
I am a sixth grade teacher in Minnesota. I want to have my students explore mini golf and calculate the reflections and angles so that they can figure out how to hit a hole in one. I know that my daughter had various problems like this in eighth grade geometry, but I can't seem to find any internet activities of the appropriate level.
Answered by Stephen La Rocque.
The development of trigonometry 2006-08-15
From Eugene:
Can you please give the exact time line of trigonometry.
Answered by Penny Nom.
1/x + 1/y 2006-08-11
From Sonya:
what is 1/x+1/y = ? is it equal to 1/x+y or what?
Answered by Penny Nom.
A problem with exponents 2006-08-09
From A student:
(8a to the negative 2 b cube c to the negative 4/4a squared b to the negative 3 c squared) to the negative 2
Answered by Stephen La Rocque.
Find x 2006-08-08
From Lucas:

is it possible to circle x when a question says find x? is that the answer or is that just being a smart ass?

i was wondering if that is the real answer and could I get full marks for it in a test?


Answered by Claude Tardif.
HCF and LCM 2006-08-05
From Bharath:
The HCF and LCM of polynomials p(x) and q(x) are h and l respectively and p(x) + q(x) = h + l, show that
[p(x)]2 + [q(x)]2 = h2 + l2

Answered by Stephen La Rocque.
Converting sq ft to sq meters 2006-08-03
From JB:
I would like to convert 2700 square feet into square meters.
Answered by Penny Nom.
x+ 6 / 3 = x+ 6 / x 2006-08-01
From Lynne:
x+ 6 / 3 = x+ 6 / x
How do I solve this equation?

Answered by Penny Nom.
When 391758 and 394915 are divided by a certain three digit number, ... 2006-07-26
From Nick:
When 391758 and 394915 are divided by a certain three digit number, the three digit reminder is the same in each case. Find the divisor.
Answered by Chris Fisher and Steve La Rocque.
The capacity of our hexagon fish tank 2006-07-26
From Doug and Andrea:
My name is Andrea and my son, Doug, and I were wondering what the capacity of our hexagon fish tank was but it’s been so long since I’ve been to school that I’ve forgotten the formula to do so. Would you please help us? The dimensions are: 27 ¾ W x 24L x 29H or one side (glass panel) is 14W x 29H.
Answered by Penny Nom.
Converting fractions to decimals 2006-07-25
From Diane:
HOW DO I CONVERT A FRACTION TO A DECIMAL? (E.G,. 8/14 to ? )

And HOW DO I DIVIDE A FRACTION (e.g., 1/14 divided by 3) and then convert that answer to a decimal?

Answered by Penny Nom.
Percentage Difference 2006-07-25
From Sandra:

I have two numbers (1530 and 1315). How do I calculate the percentage difference.

For example: I have one loader that lifts 1530 lbs. and another loader that lifts 1315 lbs. I know the difference is 215 lbs. How much greater lift capacity does the loader that lifts 1530 lbs. have than the loader that lifts 1315 lbs.?

The formula that I used was:

1530 - 1315 = 215
215/1315 = .163498 or 16.3%

Is this correct?


Answered by Stephen La Rocque.
54x14x8 at $28.00 per cubic foot 2006-07-24
From Toi:
How much would it cost if I had an item that was 54x14x8 and it was $28.00 per cubic foot?
Answered by Stephen La Rocque.
How many miles are there in 30 furlongs? 2006-07-22
From Jennifer:
If there are 8 furlongs in a mile, how many miles are there in 30 furlongs?
Answered by Steve La Rocque and Paul Betts.
The lot is a very odd shape 2006-07-20
From Liz:
I have a square footage question. I'm looking at buying a piece of land and would like to know how to calculate the square footage of it. The lot is a very odd shape and I have attached a drawing of the lot:
Answered by Stephen La Rocque.
Mini Golf geometry 2006-07-18
From Sarah:
I want to have my students explore mini golf and calculate the reflections and angles so that they can figure out how to hit a hole in one. I know that my daughter had various problems like this in eighth grade geometry, but I can't seem to find any internet activities of the appropriate level. If you can steer me towards any resources, I'd be most grateful.
Answered by Natasha Glydon.
Volume of rainfall 2006-07-18
From Randy:
Calculate the amount (in gallons)of rainfall .98 inches will put over an area of 3.76 acres.
Answered by Stephen La Rocque.
The greatest common factor of two numbers 2006-07-16
From Fadwa:
What is the greatest common factor(GCF) of the following algebraic expressions? 1680 and 6048
Answered by Stephen La Rocque.
Finding square footage from lengths in inches 2006-07-14
From Ann:
How do I find out the square footage of a window 45" by 22" and 43" by 35.5"?
Answered by Stephen La Rocque.
Concrete for a chicken coop floor 2006-07-13
From Chelsea:
How much concrete for a chicken coop floor? The dimentions are 3 slabs, 30 feet long and 4 inches thick, 6 feet 6 inches wide. One slab is 41 feet long, 6 feet 6 inches wide, and 4 inches thick.
Answered by Stephen La Rocque.
How old is y = mx + b? 2006-07-05
From Beckie:
I am taking intermediate algebra this summer (Math 95), and have a question for you. My instructor will award extra credit points to any of us who find out how old the formula y = mx+b is. Would you happen to know the answer?
Answered by Penny Nom.
Two algebra questions 2006-07-02
From Majeedah:

I'm upgrading thru correspondence and haven't been in school for a long while, so I have no class or teacher to explain the basics to me.

I have a couple of questions:

The problem is this:

  1. f(x) = x2 - 2, Find the expression.

    Q f(-x)
    A x2 - 2

    Why is the answer not -x2 - 2?

  2. A relation f, is given by f(x) = x - 2/x. Find the expression.

    Q 2/f(3)
    A 6

    How do you get the answer.


Answered by Stephen La Rocque.
Order of operations 2006-06-29
From Jaden:

(1) A triangle has vertices at (-3,-3) (7,-3) and (-2,5). What is the area of the triangle?

Is there a way of doing this so that you dont have to graph it? If the x axis is horizontal can i say it is the base and the y-axis is horizontal meaning the height. Would I be able to do this...-3 to 7-->9 spaces, -3 to 5-->7 spaces-----9(7)63/2=31.5

(2) A cube has a surface area of 54 square centimeters. What is the volume of the cube in cubic centimeters?


Answered by Stephen La Rocque.
Order of Operations 2006-06-29
From Marcy:
I have been having problems with this skill testing question. I know the rules of operation but I don't understand why they would put it in brackets. Can you help me. I have two answers but I would like to know which one is right. Im having a disagreement with a friend over this matter. Thank you so much

[ (20 + 82 - 6) ÷ 16 ] x 14

Answered by Steve La Rocque and Paul Betts.
Marking out a circle 2006-06-28
From Peter:
given a straight line. how do i work out the off sets ( at right angles) at several intermediate points. to set out a 5.0m arc that has a 18.0m radius.
Answered by Stephen La Rocque and Penny Nom.
An octagon shape flower bed 2006-06-24
From Brandy:
hello my name is brandy my husband and I would like to build an octagon shape flower bed to put around the tree in the front yard we would hope to have the whole shape about 4 ft-5 ft around the tree. what would be the way to find out how to cut each side to that they fit together equally
Answered by Penny Nom.
A trig problem 2006-06-24
From Greg:
A and B are two towers, B being 4 km due east of A. The true bearings of a flagpole, C, from A and B are α east of north and α west of north respectively. The true bearings of a second flagpole, D, from A and B are (α + β) east of north and (α - β) west of north respectively. Assuming A, B, C, and D are on level ground, and that α = 25, β = 10, find the distance between C and D.
Answered by Penny Nom.
simplify 7x + 4y + 2x - 6y 2006-06-21
From Alan:
simplify 7x + 4y + 2x - 6y
Answered by Paul Betts.
Factor 81x^4 - 72x^2 + 16 2006-06-19
From Jesse:

how would i factor this?

81x4 - 72x2 + 16


Answered by Penny.
Convert 13ft by 10ft room area into square feet 2006-06-18
From Marlene:
want to convert 13ft by 10ft room area into square feet
Answered by Penny Nom.
What are the 3rd and 4th terms of (2x-y)^7? 2006-06-18
From April:
What are the 3rd and 4th terms of this sequence: (2x-y)7?
I'm having an issue with this...is there any easier way to get it without completely factoring the whoooole thing out?

Answered by Penny Nom.
How many bags will it take to cover the area? 2006-06-14
From Ken:
I have area of 160 sq ft. I want to put mulch in the area 2 inches thick. The bag of mulch comes with 2 cu ft of mulch per bag. How many bags will it take to cover the area?
Answered by Penny Nom.
The chord length of a polygon 2006-06-14
From Krishna:
I have to find out the chord length of a polygon - Tetradecagon ! The Radius of the Circle is 11.5 Cms. The Circle is intersepted by 14 arcs. Then how to find out the chord length?
Answered by Stephen La Rocque.
Fahrenheit and Celsius 2006-06-12
From Doris:
I just know how to do problems with fahrenheit or celcius. Could you show me step by step how to do these? Then I can do it if you can show me each step of the way.
Answered by Stephen La Rocque and Penny Nom.
The zeros of a function 2006-06-12
From Sky:
find all the real zeros of the function:
f(x)=2x3 + 4x2 - 2x - 4

Answered by Stephen La Rocque.
Square feet and linear feet 2006-06-10
From Richard:
Need to know how many linear ft are in 2800 sq.ft
Answered by Penny Nom.
Flipping a coin 2006-06-08
From John:
If a coin is flipped n times, where H is the number of heads after n flips, and T the number of tails, then will the quantity (H-T) change signs infinitely often as n goes to infinity?
Answered by Chris Fisher.
Designing a garage 2006-06-08
From A builder:
I'm currently designing a garage and came upon this interesting math problem. I've tried using various methods to solve it but have so far been unsuccessful. I've included a picture as its far easier to show you my question than explain it verbally. I realize it could be done by trial and error but i'm looking for a real solution.
Answered by Stephen La Rocque and Penny Nom.
A volume of revolution 2006-06-07
From Colleen:
Find the exact volume in cubic units generated by rotating a region, R, around the
y-axis, given that R is the region bounded by the curve y = x3 and the lines x = 1 and
y = 8.

Answered by Penny Nom.
Buying some carpet 2006-06-05
From Russ:
My room is 24 ft. long and 17 ft. wide. I need to know how many square feet are in 1 yd.²
Answered by Penny Nom.
The area of part of a circle 2006-05-29
From Larry:
need to find area of a circle between a given line (cord) to the circumference of the circle (see attachment). I often review blue prints of homes and many times have to know the area the home.
Answered by Stephen La Rocque and Penny Nom.
what is the cubic feet of 18... 2006-05-27
From A student:
what is the cubic feet of 18"x18"x24"?
Answered by Penny Nom.
differentiate the volume of a cylinder with V respect to h 2006-05-24
From A student:
differentiate the volume of a cylinder with V respect to h
Answered by Stephen La Rocque.
The area of a triangle 2006-05-21
From A student:
how do you find the area of a triangle?
Answered by Penny Nom.
The coefficient of variation 2006-05-20
From Glenn:
What is the correct formula for coefficient of variation for a binomial distribution?
Answered by Penny Nom.
Square feet and acres 2006-05-19
From Lisa:
1.12 acre= ???? sq ft
1.043 acre= ???? sq ft

I work in the loan department and I do appraisal reviews but I use square feet to determine my values.

Answered by Penny Nom.
A pole 22 5/6 feet long is broken in two 2006-05-19
From Kenneth:
A pole 22 5/6 feet long is broken in two. One piece is 2 3/4 feet longer than the other. What is the length of each piece?
Answered by Penny Nom.
The length of the diameter 2006-05-18
From Steven:
A circle has a circumference of 312in. Find the length of its diameter to the nearest integer?
Answered by Penny Nom.
What is the radius of the sphere? 2006-05-17
From Jimmy:
The surface area and volume of a sphere are both equal to the product of 4-digit whole numbers and pi. What is the radius of the sphere?
Answered by Stephen La Rocque.
What are the dimensions of the field? 2006-05-14
From Angie:
The length of fence required to enclose a rectangular field is 3000 metres. What are the dimensions of the field if it is known that the difference between the length and width is 50 metres?
Answered by Penny Nom.
The volume of water in a sphere shaped water tower 2006-05-14
From Dan:
I work in the water industry, providing automatic controls for water systems. I was wondering if there was a formula for calculating the volume of water in a sphere shaped water tower? With the use of a pressure gauge we know the elevation of zero water {the bottom of sphere) and the overflow elev. (near the top of sphere) I would like to calculate how many gallons are at any elevation in between.
Answered by Stephen La Rocque.
I want to calculate 0.00353 to the power 1.3 2006-05-14
From Richard:
I want to calculate 0.00353 to the power 1.3 How do I do it?
Answered by Stephen La Rocque.
A person is given 3 true or false questions 2006-05-11
From Larry:
A person is given 3 true or false questions. They have no idea of the answers. What is the probability of the person getting them all correct? Could you show me a tree for this.
Answered by Penny Nom.
What is 2/3 divided by 5? 2006-05-11
From KH:
What is 2/3 divided by 5?
Answered by Penny Nom.
An aircraft going from city A to city B 2006-05-10
From Preston:
An aircraft going from city A to city B on a bearing of S69 degrees East is traveling at a speed of 430 mph. The wind is blowing out of the north to south at a speed of 25 mph. Find the ground speed and the plane's true bearing.
Answered by Stephen La Rocque.
10^100 in binary form 2006-05-10
From Headmaster:
How many digits has the number 10100 if we write it in binary form?
Answered by Chris Fisher, Paul Betts and Steve La Rocque.
Small pipes and large pipes 2006-05-09
From geece:
A large fresh water reservoir has two types of drainage system. Small pipes and large pipes. 6 large pipes, on their own, can drain the reservoir in 12 hours. 3 large pipes and 9 small pipes, at the same time, can drain the reservoir in 8 hours. How long will 5 small pipes, on their own, take to drain the reservoir?
Answered by Penny Nom.
Twenty golfers 2006-05-08
From Mona:
We have twenty golfers and four rounds of golf. We would like to figure out how to make foursomes so that each golfer plays with as many different golfers as possible.
Answered by Penny Nom.
A hallway in an old house 2006-05-06
From Sabrina:
I am trying to determine how many square feet is in a room that is not square or rectangular. It has basically 5 walls of different lengths, it is acutely considered a hallway in an old house, how can I figure out the footage, the Length times Width requires to much wasted materials
Answered by Penny Nom.
More on the game of 24 2006-05-05
From Kathy's daughter:
My daughter's 5th grade class plays the 24 math game. She came home the other day with a real stumper! The numbers on the card are 5,5,2,and 6. The rules say you must use every number,you may only use a number once,you can only add, subtract, multiply and/or divide and the answer is 24. Can you help!!!! No one where I work has been able to figure this one.
Answered by Paul Betts and Penny Nom.
A fountain of water jets forms parabolic arches 2006-05-03
From Jennifer:
Let's say in you have a fountain and the water jets form parabolic arches. The center of the fountain, being the origin of the coordinate system, it is elevated 5 feet off the ground, . The equation formed the water arch is y= -x2+4x, what is the radius of the basin needed to catch the water at ground level?
Answered by Stephen La Rocque.
Simplifying with negative exponents 2006-05-02
From Kristine:

I am having a hard time understanding factoring and multiplying polynomials. I have a couple of problems I just can't get, can you please help me?

Simplify, do not use negatives in the answer:
(5a-1b-7)(-2a4b2)

and:
(5x2y-7z)(-4xy-3z-4)


Answered by Stephen La Rocque.
A word problem with fractions 2006-05-02
From Stuart:
I have 54 1/4 yards of material. i need to cut as many pieces as possible out of it that are 3 1/12 long. how many pieces will you get and how much is left over?
Answered by Stephen La Rocque.
Rate of ladder falling 2006-04-30
From Harsh:
A ladder 4 m long rests against a vertical wall. If the bottom of the ladder slides away from the wall at a speed of 30 cm/s, how quickly is the top of the ladder sliding down the wall when the bottom of the ladder is 2 m from the wall?
Answered by Stephen La Rocque.
How much does 1300cc of liquid weigh? 2006-04-28
From Debra:
How much does 1300cc of liquid weigh, and how is the formula equated?
Answered by Stephen La Rocque.
Ladder and wall 2006-04-27
From Lori:
A 12-foot ladder is leaning across a fence and is touching a higher wall located 3 feet behind the fence. The ladder makes an angle of 60 degrees with the ground. Find the distance from the base of the ladder to the bottom of the fence.
Answered by Stephen La Rocque.
If the perimeter of a square is 1 foot, what is its area? 2006-04-27
From Larry:
If the perimeter of a square is 1 foot, its area is how many square inches?
Answered by Stephen La Rocque.
Pythagorus and cone dimensions 2006-04-26
From Glynnis:
How do you find the measure of a side that is not the hypotenuse using the Pythagorean Theorem? Also, how do you figure the surface area and volume of a cone when the radius is 5 yards and the height is 8 yards?
Answered by Stephen La Rocque.
Marine shipping - fuel consumption 2006-04-26
From Sandy:
When a ship is 800 nautical miles from port its speed is reduced by 20%, thereby reducing the daily fuel consumption by 42 tonnes and arriving in port with 50 tonnes onboard. If the fuel consumption per hour is given by the expression (0.136 + 0.001V^3) where V is speed in knots; Estimate a) reduced consumption per day b) fuel onboard when speed reduced c) normal consumption for 800 nautical mile voyage d) increase in steaming time
Answered by Stephen La Rocque.
How many gallons in a cubic foot? 2006-04-24
From Jana:
How many gallons in a cubic foot?
Answered by Stephen La Rocque.
Proof by induction 2006-04-24
From Meshaal:
Find an expression for: 1-3+5 - 7 + 9 - 11 + ... + (-1)^(n-1) * (2n-1) and prove that it is correct.
Answered by Stephen La Rocque.
Geometry proof 2006-04-23
From Jade:
From a point P outside a circle with centre O, tangents are drawn to meet the circle at A and B. a) Prove that PO is the right bisector of the chord AB. b) Prove that
Answered by Stephen La Rocque.
The perimeter of a regular octagon 2006-04-20
From Martin:
I would like to make an octagon out of 2x4 lumber. I know that the lumber needs to be cut at 67.5 degree angles, but how do I determine the length of each piece if I want to make, say, a 2.5 ft diameter octagon?
Answered by Stephen La Rocque.
What is the surface area of a cone with a hemisphere on top? 2006-04-20
From Jordan:
How do you find the surface area of a cone with a hemisphere on top? The only information I have is the radius of the hemisphere, which is 4, and the total height, which is 7.
Answered by Stephen La Rocque.
Geometric sequence and basic functions (graphs) 2006-04-20
From Marlene:
Which of the basic functions is related to the geometric sequence: Linear, Quadratic, Rational, or Exponential? Can you give me an example of how it would be used in normal life?
Answered by Stephen La Rocque.
What is the original cost? 2006-04-19
From Charlie:
Selling a piece of furniture for $670 and the profit is 35%. What is the original cost?
Answered by Stephen La Rocque.
Proving a summation formula by induction 2006-04-19
From Sharon:
Prove by induction that the sum of all values 2^i from i=1 to n equals 2^(n+1) - 2 for n > 1.
Answered by Stephen La Rocque.
How many numbers are relatively prime with 250? 2006-04-19
From David:
How many positive integers less than or equal to 250 are relatively prime with 250?
Answered by Stephen La Rocque.
How do you round off 89,100? 2006-04-19
From Jeannie:
My home work says to round each number to the place of the underlined digit. It has 89,100. The underlined number is 9. What do I do?
Answered by Stephen La Rocque.
What is the cost price? 2006-04-14
From Purushotham:
If the selling price is 120 and the profit is 20%, what is the cost price?
Answered by Stephen La Rocque.
How to find the odds for a lottery jackpot 2006-04-12
From Harvey:
Is it best to use factorials to calculate the odds of winning a lottery, such as the MegaMillions that is popular in the US, or is there a better way?
Answered by Stephen La Rocque.
Finding the inverse of a function 2006-04-12
From Sam:
If g(x) = 2x+3/5 then how do I find the inverse?
Answered by Walter Whiteley.
A proof by induction 2006-04-09
From Sharon:
prove by induction: For every n>1, show that
2 + 7 + 12 + ...+ (5n-3) = n(5n-1)/2

Answered by Penny Nom.
Given three angles and a side 2006-04-09
From Jon:
How do you figure out the length of all sides of a scalene triangle if given the measure of all angles, and one side?
Answered by Stephen La Rocque.
An octagon shaped soffit 2006-04-08
From Ken:
I am trying to make an octagon shaped soffit in my ceiling and need help figuring how to divide the circle on the ceiling into an octagon- The circle is 7 foot in diameter.
Answered by Penny Nom.
Interest computed on a 360 per year basis 2006-04-06
From Sonya:
Okay, I am reading this promissory note that says interest is computed on 365/360 basis by applying the ratio of annual interest rate (24%) over a year of 360 days. I sure do not remember anything like this in school, and am stumped.
Answered by Stephen La Rocque.
The average of monthly averages 2006-04-06
From Ray:
I have the averages of some data in percentages. the data is rather large, how can i find the average of these averages without going back to the original data? Is there a Factor or Constant that i could multiply or devide or add and get the average of these averages without resorting to the raw data.
Answered by Steve La Rocque and Harley Weston.
x^2+2x-3 is a factor f(x)=x^4+2x^3-7x^2+ax+b 2006-04-06
From Maha:
Given that x2+2x-3 is a factor f(x)=x4+2x3-7x2+ax+b, find a and b and hence factor f(x) completely.
Answered by Penny Nom.
An exponential function 2006-04-03
From Meadow:
The graph of an exponential function passes through the points (0,1) and (4,10) find the constant of growth or decay. State the function.
Answered by Stephen La Rocque.
Find the point of inflexion for the curve y = e^x/(x^2-1) 2006-03-31
From Sam:
Hi, i am trying to find the point of inflexion for the curve y = ex/(x2-1) and i got a really complex expression for y". I can't seem to solve x4-4x3+4x2+4x+3=0 so does that mean there is no point of inflexion?
Answered by Penny Nom.
A fence around a pen 2006-03-30
From Daryl:
I hope you can help me out with the attached problem, It has been driving me crazy.
Answered by Stephen La Rocque and Penny Nom.
Superimposing images 2006-03-30
From Laurie:
I am a parent and trying to find real world applications for the math strand that includes superimposing one item on another to determine identical. The Alberta strand is match size and shape of figures by superimposing one on top of the other. I am trying to find a way to put this into context for everyday life. Any ideas?
Answered by Penny Nom.
Adding with mixed fractions 2006-03-30
From Becca:

can anyone help with how we change to get the same denominator with adding whole number.

example:8+7 3/8

6 1/2 + 2 5/6


Answered by Stephen La Rocque.
cos(3X) 2006-03-29
From Joshua:
I'm having trouble proving that cos(3X)=cos3X -(cosX)(sin2 X)
Answered by Penny Nom.
A table of values 2006-03-28
From Cindy:
Create a table of values for each of the following functions: y=5x, y=52, and y=5x. I dont know to do this.
Answered by Stephen La Rocque.
Adding fractions 2006-03-26
From Barbara:
I know that to subtract 1/4 from 2/3 I must find a common denom. Now the 2/3 becomes 8/12.....i understand the 12, but where does the 8 come from?
Answered by Penny Nom.
The area of a block of land 2006-03-26
From Ronald:

I have a building block of land with four unequal sides and only one right angle. I want to know the total area (in metres) and how the calculations were carried out.

The four sides are: Rear of property: 9.14 metres
left side: 36.9 metres
Right side: 32.61 Metres
front to street: 27.43 Metres

The front to street and right side constitute a right angle. but there are no others.


Answered by Penny Nom.
A scale factor 2006-03-25
From Jackie:
I need a formula or step by step help to show my child how to find missing dimensions for example: The scale factor for a model is 10 inches = ________ feet
Model 39.7 in
Actual 11.9 ft

Answered by Penny.
Acres 2006-03-25
From Judy:
How do you turn land measurements into acres
Such as piece of property is 45 x 167

Answered by Penny Nom.
The cartesian product of a countably infinite collection of countably infinite sets 2006-03-25
From Geetha:
Is the cartesian product of a countably infinite collection of countably infinite sets countable infinite?
Answered by Penny Nom.
How much fabric do I need? 2006-03-25
From Michelle:

I AM DECORATING A STAGE FOR A BOARD OF DIRECTORS MEETING AND I NEED TO BUY FABRIC AND I DON’T WANT TO WASTE $$ BY BUYING TOO MUCH FABRIC OR NOT ENOUGH FABRIC.

THE AREA THAT I’M NEEDING MATERIAL FOR IS, 12ft HIGH BY 34ft WIDE.

THE FABRIC COMES IN ROLLS OF 60” (inches) x 100 yards.

SO HOW MUCH FABRIC SHOULD I NEED TO FOR THE AREA I’M WANTING TO COVER?


Answered by Stephen La Rocque and Penny Nom.
Comparing Fractions 2006-03-24
From J.:
how can you compare fractions with like numerators and UNlike denominators, like 2/3 and 2/5, without renaming or using fraction strips?
Answered by Stephen La Rocque and Penny Nom.
given that p is a prime and p|a^n, prove that p^n|a^n 2006-03-24
From Janna:
given that p is a prime and p|an, prove that pn|an
Answered by Stephen La Rocque.
x+6/7 = 3/5 2006-03-22
From Cavell:
i couldn't find in the book on how to solve x+6/7 = 3/5 i know its simple but i don't remember how and the book never explains step by set on how to solve problems and i don't have a teacher to ask because i do work through mail so i cant ask questions vary easily
Answered by Stephen La Rocque.
Piecewise functions 2006-03-21
From Kris:
First Problem:
Southeast Electric charges .09 cents per kilowatt-hour for the first 200 kWh.The company charges .11 cents per kilowatt-hour for all electrical usage in excess of 200 kWh. How many kilowatt-hours were used if a monthly electric bill was $57.06? The answer I came up with is 360, is that right? and also how do I set it up in an equation form?

Second Problem:
A construction worker earned $17 per hour for the first 40 hr of work and $25.50 per hour for work in excess of 40 hr. One week she earned $896.75. How much overtime did she work? I came up with 8.5 hrs over-time worked. Again I don't know how to set up the equation to come up with the answer.

I need some pointers on how to figure out story problems! If you have any suggestions that would help me out I would be very grateful.

Answered by Penny Nom.
How far from Earth is Neptune? 2006-03-21
From Scott:
A laser beam travelling at the speed of light bounces off the planet Neptune and returns to Earth in 29 960.72 s. The speed of light is 299 792.5 km/s. How far from Earth is Neptune?
Answered by Penny.
BEDMAS 2006-03-21
From Andie:
It's been a long time since I've used bedmas. I'm still confused as to what the answer to this question would be.
30 - (2 x 9) + (15/3)=

Answered by Penny Nom.
Three towns are located at the vertices of an equilateral triangle 2006-03-20
From A student:
three towns are located at the vertices of an equilateral triangle. The towns are 8, 5, and 3 miles, respectively, from a store. How far apart are the towns?
Answered by Chris Fisher.
Linear feet on a paper roll 2006-03-20
From Vishal:
I WOULD WISH TO KNOW OF A STANDARD FORMULA BY WHICH I CAN CALCULATE THE LINEAR FEET OF A PAPER ROLL AS PER THE FOLLOWING DETAILS:-

THICKNESS: 150 GSM (GRAMS PER SQUARE METER)
REEL WIDTH: 85 CM
REEL DIAMETER: 140 CM
REEL INNER CORE: 10 CM
REEL WEIGHT: 946 KG's


Answered by Stephen La Rocque.
GCD 2006-03-20
From Jana:
How can i prove: if gcd(a,b)=1 then gcd(a+b,ab)=1
Answered by Penny Nom.
Quarts, liters cubic yards and soil 2006-03-19
From Kathy:
I am trying to find out how many quarts of dirt there are in a yard of dirt. The local flower store sells bags of dirt that are 20 quarts or 22 liters for $1.25 and another store sells a yard of dirt for $25.00. Which is the better deal?
Answered by Penny Nom.
The day of the week 2006-03-19
From Neil:

A friend of mine has an excellent memory for dates and events that occurred on that date, but his party piece is the ability to tell you the day of the week that fell of any given date. This talent is all the more extraordinary because the answer comes back in less than 2 seconds (often under 1 second). By his own admission he is no mathematician.

Now his memory and knowledge are without question, but I challenged him that because of its diversity this party piece could not be based upon memory, but on mathematics. I believe that with the correct mathematical approach and the use of a common algorithm, anyone with a basic mathematical mind can do this in their head.


Answered by Chris Fisher.
Gross profit as a percentage 2006-03-18
From Mark:
How do I calculate the gross profit by percentage on and a product I purchased for $23.80 marked up 45% sold for $34.51 = profit of $10.71 GP by percentage????????
Answered by Penny Nom.
The square footage of an area in my backyard 2006-03-17
From Kim:
I need to find out how to calculate the square footage of an area in my backyard that is in the shape of a "slice of pie". There are two sides that are straight lines that come together at the top to form a point, and then at the bottom is a curved line that joins the two other lines together. I need to figure out how to calculate the square footage that is inside the area.
Answered by Penny Nom.
Building a flower bed 2006-03-17
From Bobby:
I am building a flower bed 60 ft long by 8 ft wide by 3 ft deep. How much dirt will it take to fill it with top soil/dirt.
Answered by Penny Nom.
9, 8, 7 and 2 in the game of 24 2006-03-17
From Alana:
I just began student teaching in a 5th grade math class and found that the students play 24 in class, a game that I used to play in elementary school. The students choose a card which shows 4 numbers and then have to use any combination of addition, subtraction, multiplication, and division to reach 24. There was one card that we found that seems to have all of us stumped. The four numbers are 9, 8, 7 and 2. Can you figure out how to make 24 with those? Thanks so much for your help!
Answered by Claude Tardif.
A proof by contraposition 2006-03-16
From Eban:

1)by mathematical induction prove that 12 + 32 + 52 + ...... + (2k-1)2 = (1/3)k(2k-1)(2k+1) for all positive integers k.

2)show that the contrapositive of the following statement is true. if 1 + M7 is even, then M is odd.


Answered by Stephen La Rocque.
Adding a cellar 2006-03-14
From Spencer:

You want to add a cellar to your store. The construction company told you it would cost $5.90 per cubic yard to dig the cellar plus $20.00 per square ft. to finish it.

How much will it cost to add a cellar 36 feet long, 14 feet wide, and 8 feet deep?


Answered by Stephen La Rocque.
f(x) is a polynomial of degree 3. 2006-03-13
From Meadow:
f(x) is a polynomial of degree 3. It leaves a remainder of 10 and 4 when divided by x + 1 and x - 2 respectively. Given also that f(1) = f(-2) = 0, find the remainder when f(x) is divided by x - 3.
Answered by Penny Nom.
Cubic feet to cubic yards 2006-03-12
From Sandy:
We need to convert feet to cubic yards. The concrete is 45ft by 10ft by 4ft. How many cubic yards is this?
Answered by Penny Nom.
The volume of a freezer 2006-03-11
From Marsha:
I have a stand up freezer that measures 2ft deep, 23 inches wide, and 44.5 inches in height. I multiplied them (2x23x44.5) and came up with 2047. I guess my question is, when you look at an ad for this item, they always say 17, 14, or 19 cubic feet. How do they (and I) come up with a number like that?
Answered by Penny Nom.
Profit 2006-03-11
From Wendy:
If I have a product I have bought for 33.58 and I mark it up by 30% sell at 43.65 how do I work out the gross profit.
Answered by Penny Nom.
Factor 2006-03-08
From Brad:
Factor:
x3 + 64m3 and 125p3 - q6

Answered by Penny Nom.
Factors of a cubic 2006-03-08
From Skye:
Show that (x+2) is a factor of f(x) where:

f(x) = px3 + (4p+2)x2 + (5p+4)x + 2p

Find the range of values for which the equation f(x) = 0 has 3 distinct real roots

Answered by Penny Nom.
A hat contains between 10 and 25 marbles 2006-03-06
From Kerry:
A hat contains between 10 and 25 marbles. Some marbles are green, and the rest are yellow. Without looking you are to reach into the hat and pull out a marble. The probability of pulling out a green marble is 2/9. How many marbles are in the hat and explain?
Answered by Stephen La Rocque and Penny Nom.
1/infinity and 1/0 2006-03-04
From Evan:
I was thinking the other day when i was in math class that when you divide 1 by say n you'll get 1/n. As the value of n increases the smaller the number you get. So if you divide 1/infinity would that equal zero? And if that is true then would 1/0=infinity be true also?
Answered by Penny Nom.
The quadratic formula 2006-03-03
From Miriam:
Please show work and indicate "answer", using the quadratic formula -

(x+2) (2x+3) = 6

Answered by penny Nom.
The meaning of numbers 2006-03-03
From John:
I'm having a philosophical debate on the meaning of numbers, equations, or the world of math in general. Would it be possible if you could help me by giving me a resource that talks about it or if you have your own opinion I would be most grateful.
Answered by Harley weston.
8/11 of 900 2006-03-02
From Cavell:
what the number is 8/11 of 900?
Answered by Penny Nom.
6y^10 + 19y^5 + 14 2006-03-02
From Bahija:
Factor 6y10 + 19y5 + 14
Answered by Penny Nom.
The volume & surface area of a rectangular pyramid. 2006-02-27
From Cheryl:
My daughter is working on a math project & we are having a hard time finding the volume & surface area of a rectangular pyramid. Can you help us with this?
Answered by Penny Nom.
Profit 2006-02-25
From Vernessa:
ABC company wants to sell enough products to earn a profit of $40,000, if unit sales are $10 and variable costs $8 , fixed cost $80,000 how many units must be sold to earn a profit of $40,000.
Answered by Penny Nom.
The area of a flag 2006-02-24
From Kim:
I have a flag that's very similar to the canadian flag. Two smaller red areas and i larger white area. If I am given the length is 3 and the height is 1.4 how do I determine what the area of the two red areas is?

Here's the answer but I can't figure out why they are multiplying 1.4 x 1.4?

(3x1.4-1.4x1.4)divided by 2= 2.24 divided by 2 = 1.12

Answered by Penny Nom.
Traveling from Asheville to Indianapolis 2006-02-21
From Grace:
Carol was traveling from Asheville, North Carolina, to Indianapolis, Indiana, by bus. At the halfway point of her trip, in terms of distance, she fell asleep. When she awoke, her distance to Indianapolis was half the distance that she had traveled while she slept. For what fraction of her trip did Carol sleep?
Answered by Penny Nom and Stephen La Rocque.
Stem-and-leaf plots, box-and-whisker plots and medians 2006-02-21
From A student:
explain which plot would be easiest to use to find the median....box-and-whiskers plot or stem-and-leaf plot
Answered by Penny Nom.
Find the xy equation of the curve on which Brian is located. 2006-02-20
From Skye:
Andrew, located at (0, -2200) fired a rifle. The sound echoed off a cliff at (0,2200) to Brian, located at point (x,y). Brian heard the echo 6 seconds after he heard the original shot. Find the xy equation of the curve on which Brian is located. Assume the distances are in feet and that sound travels at 1100 feet/second. (hint: find the equation of the hyperbola)
Answered by Stephen La Rocque.
A man sells a refrigerator for $171 2006-02-20
From Skye:
A man sells a refrigerator for $171, gaining on the sale as many percent (based on the cost) as the refrigerator cost, C, in dollars. Find C.
Answered by Stephen La Rocque.
Translate and magnify a triangle 2006-02-19
From A student:
Triangle CAT has vertices located at C(-2,5), A(-5,1), T(1,1)

Translate triangle CAT 4 units right and magnify the triangle by 3. List the new coordinates and explain the process of computing the new coordinates.

Graph triangle CAT and the new triangle form part A on the same coordinate plane....

Will the area of the new triangle be 3 times as large as the original?? Explain why or why not

Answered by Penny Nom.
Whose photo is he looking at? 2006-02-19
From Adeyeri:
Looking at a picture a man says, that the father of the person (in photo) is the son of my father. whose photo is he looking at?
Answered by Penny Nom.
Potting soil 2006-02-17
From Cheryl:
I need to buy some potting soil for a raised planting bed that is 6'x24'x1'. How do you figure how many 2 cu. ft. bags of potting soil that I will need?
Answered by Stephen La Rocque.
The height of a cylinder 2006-02-15
From Autumn:
the surface area of a cylinder with circumference 24pi cm. is 300pi cm2 . what is the height???
Answered by Penny Nom.
How many flat surfaces does a cylinder have? 2006-02-14
From Kimia:
How many flat surfaces does a cylinder have?
Answered by Stephen La Rocque.
Victor's profit 2006-02-13
From Millie:
victor is trying to make money buying and selling cars. he saved up and buys a car for $2500 and then sells it for $4327.he then buys another car using 2/3 of his profit and sells it for twice what he paid for it. how much money did victor make during these transactions?
Answered by Steve La Rocque.
Golfing with 8 or 12 2006-02-13
From Carolyn:
I have a couple of questions that my dad asked me and I do not think it can be done in the manner he wants it to come out to.

He is going on a golf trip in June. There are going to be either 8 or 12 players playing 4 rounds of golf.

The questions that he says will not work and I do not know how to figure it out either is

4 Rounds of golf is for sure. 18 hole Rounds each

4 Players to a group

Either 8 or 12 players can every player play with every player in a group at least once during those 4 rounds.

If this can be done can you give me the combinations for both 8 and 12 , so I can help out my dad. Along with how you you figure this out.

Answered by Penny Nom.
A concrete slab 2006-02-13
From Cheryl:
A concrete area that will be used for picnic benches will measure 20 ft on each side. The concrete slab will be 8 inches deep. How many cubic feet of concrete will be needed to construct the patio?
Answered by Penny Nom.
the size of a lot is 300 sqm, what is the square feet conversion? 2006-02-11
From JC:
the size of a lot is 300 sqm, what is the square feet conversion?
Answered by Penny Nom.
X^2 + 4X + 6 is a factor of X^4 + rX^2 + s 2006-02-11
From Katie:
If X2 + 4X + 6 is a factor of X4 + rX2 + s, then find the value of r+s?
Answered by Steve La Rocque and Penny Nom.
Is there a solid shape with 5 faces, 8 edges, and 4 vertices? 2006-02-11
From Annie:
Is there a solid shape with 5 faces, 8 edges, and 4 vertices?
Answered by Penny Nom.
Proof by induction 2006-02-10
From Victoria:

how do i prove by induction on n that
n
Σ 1/i(i+1) = n/(n+1)
i=1

for all positive integers n


Answered by Penny Nom.
Equal fractions 2006-02-10
From Dylan:
write three equal ratios

3/4 _________________________


Answered by Penny Nom.
sinh(i/2) 2006-02-09
From Louis:
How can you set up an equation to find sinh(i/2)
Answered by Penny Nom.
Simplify the variable expression: 3g^2h over 12gh 2006-02-09
From Andrew:
okay the question says simplify the variable expression: 3g2h over 12gh
how would u do this?

Answered by Penny Nom.
How many 16 foot 2 by 4's are there in a 1000 board feet of lumber. 2006-02-08
From Fred:
how many 16 foot 2 by 4's are there in a 1000 board feet of lumber.
Answered by Penny Nom.
preuve par neuf 2006-02-08
From Thonet:

Je suis enseignante en primaire en Belgique et j'aii actuellement une stagiaire étudiante qui va devoir apprendre à mes élèves comment fonctionne la preuve par 9. Elle sait expliciter concrètement celle-ci et a trouvé l'explication du pourquoi de sa fonction mais nous ne savons pas l'origine de cette application mathématique...

Qui l'a inventée ? Pourquoi ? ...
Pourriez-vous nous éclairer ?


Answered by Claude Tardif.
How many divisors does the number 138600 have? 2006-02-08
From Joe:
How many divisors does the number 138600 have?
Answered by Steve La Rocque and Penny Nom.
Highest common factor 2006-02-06
From Sajid:

1.A rectangular field has dimensions 156 m and 84 m. It is to be fenced with wire netting supported by stakes, equally spaced, a stake being placed at each corner of the field. What is the greatest possible distance between any two stakes, given that it should be more than 2 m and less than 8 m?

2. What is the length of the side of the largest square tile which can be used to cover a floor 525 cm long by 275 cm wide without leaving a gap?

3. Find the greatest number which will divide 140 and 249 to leave remainders 8 and 18 respectively?

4. Find the greatest number which will divide 304 and 298 so as to leave remainder 4 in each case.


Answered by Penny Nom.
Prove that p^n >= (p!)/(p-n)! 2006-02-02
From Rhydian:

PROVE:

pn >= (p!)/(p-n)!


Answered by Penny Nom.
More on the game of 24 2006-02-02
From Cathy:
10,16,9,4 this numbers have to equal up to 24. we can't figure it out.
Answered by Claude Tardif.
BEDMAS 2006-01-31
From Janielle:
What Do All the Letters In BEDMAS Stand for?
Answered by Penny.
How do you find the angles in a triangle? 2006-01-27
From Keith:
How do you find the angles in a triangle if you know the lengths of the sides?
Answered by Chris Fisher and Penny Nom.
Four people are in a race 2006-01-26
From Tammy:
If 4 people are in a race, how many different placements, i.e., 1st, 2nd, 3rd, 4th, can there be and what is the equation?
Answered by Penny Nom.
Our client wants to make 25% from their sale... 2006-01-24
From Noelle:
I need to figure out how to figure pricing for our clients. If our client wants to make 25% from their sale and they purchase it from me for $35.95, how do I figure what their cost is?
Answered by Penny Nom.
Finding the nth term in fractions 2006-01-23
From Zarina:
Each term in this sequence is made by increasing the numerator by 1 and the denominator by 3. here are the first five terms:

1/4, 2/7, 3/10, 4/13, 5/16, ...

(a) write an expression for the nth term:

Answered by Penny Nom.
The angles in a hexagon 2006-01-22
From Linda:
My problem is in relation to wood and making a six sided object from it. On my saw, there is a place to set the angle to which you wish to cut. I cannot for the life of me, figure this out. I am starting with a piece of plywood (1/4" x 6" x 18") and need to know what the angle degree would be to make each of the sides match perfectly to form a hexagon. Trial and error just is not working. Can someone help me?
Answered by Penny Nom.
A confidence interval 2006-01-21
From Jonathan:

I am attempting to calculate how my confidence interval will widen at the 95% confidence level if my response universe increases from 100 to 150 or to 200.

There is a universe of 54,000. I take a 5% sample for a test universe of 2,700

If my "yes" universe is 100, at the 95% confidence level, what is my +/- range? (i.e +/- 3? +/-5?)

Historically, 6.6% of the 2,700 you say "yes". I am trying to determine how the confidence interval would change if the number of "yes" responders increased to 150 or to 200.


Answered by Penny Nom.
1/3 of 1/4 2006-01-21
From Nicole:
WHAT FRACTION IS EQUAL TO 1/3 OF 1/4
Answered by Penny Nom.
The circumference of a larger circle is twice the circumference of a smaller circle. 2006-01-20
From Amanda:
The circumference of a larger circle is twice the circumference of a smaller circle. What is the ratio of the radius of the smaller circle to the diameter of the larger circle?
Answered by Penny Nom.
One boundary of a pond is parabolic in shape. 2006-01-20
From Glenn:
One boundary of a pond is parabolic in shape. The boundary passes through the points A(-20,45), B(40,40) and E(30,35). The equation of the parabola is of the form y=ax2+bx+c. Find the equation of the parabola and the coordinates of the vertex of the parabola. Any assistance you could provide would be greatly appreciated.
Answered by Penny Nom.
An irregular octagon 2006-01-20
From Robert:
I am building a poker table which is in the shape of an irregular octagon. I know the table measures 72 inches long and 48 inches wide with two parallel straight sides of equal length and six smaller sides of equal length ( three at each end of the table), what I don't know are the lengths of the any of the sides.
Answered by Harley Weston.
5x^2 - 27x - 18 2006-01-13
From Katy:
How would you factor 5x2 - 27x - 18?
Answered by Penny Nom.
Convert to decimal form 2006-01-12
From A student:
I would like to know the full answer on how to turn the fraction 3 3/8 into a decimal please.
Answered by Penny Nom.
The volume of a piece of furnature 2006-01-10
From Kimberly:
What is the cubic feet for a piece of furniture that is 38" wide X 45" tall X 18" deep
Answered by Penny Nom.
How do I calculate feet and inches to sq ft? 2006-01-09
From Rebecca:
How do I calculate feet and inches to sq ft?
Answered by Penny Nom.
y - y1 = m(x - x1) 2006-01-08
From Greg:
How do you change 2x + 3y = 7 into y - y1 = m(x-x1)?
Answered by Penny Nom.
GCF, LCM, primes and the ladder method 2006-01-07
From Linda:
How would I teach both finding the GCF and LCM with prime factors...I recall the ladder method vaguely.???
Answered by Penny Nom.
Differentiation, powers and logs 2006-01-06
From Claudia:

Question: how do I find the derivative of

x* ln(x+(e^2))^2

x^lnx

x^(e^(-x^2))


Answered by Penny Nom.
The area of an octagon 2006-01-03
From Nikki:
I want to figure out the square footage of an octagon. i have 8 panels that are 24" wide. Its for my dogs and i wanna know how much room they'll have.
Answered by Penny Nom.
3x^4 - 81 2006-01-02
From Julio:
How can I factor the following?:

3x4 - 81

Answered by Penny Nom.
A probability question which resulted from a game of Yahtzee 2005-12-31
From Robert:
Could someone please assist me with this probability question which resulted from a game of Yahtzee we were playing in Melbourne, Australia on our holidays.

The object on this turn was to throw a “large straight” which is 5 numbers in sequence from 5 dice numbered 1 – 6. A player initially throws all 5 dice and then selects those dice they want to throw again for a further two more times. In this instance the player on their first throw, threw a 1,2,3,4 and 6.

Question:- What is the respective probabilities of gaining a straight if they were to –

a) put back say the 6 and try and throw a 5 on the two further throws or…..
b) put back the 1 and 6 and try and throw a 1 and 5 or 5 and 6 on the two further throws bearing in mind that if one of the numbers was a 5 on the second throw they could hold that number and try for a 1 or 6 on the third throw.

I would be most appreciative if someone could assist in showing me how to calculate the probabilities particularly in the second instance (b).

Answered by Penny Nom.
Twelve golf balls 2005-12-28
From Todd:
You have 12 golf balls and 11 of them are the exact same weight, but one of them is either a little heavier or a little lighter. You only have three attempts to weigh the balls. How can you determine which ball weighs more/less than the rest?
Answered by Penny Nom.
A goat is tied to the corner of a 50 ft square outbuilding 2005-12-24
From Danielle:
Topic: A goat is tied to the corner of a 50 ft square outbuilding with a 40 ft. rope.
a) What is the measure of the partial circumference created as the goat walks at the full length of the rope?

b) Since the goat is trimming the grass from part of the outbuilding, how much of the perimeter of the outbuilding will the building owner have to trim?

Answered by Penny Nom.
Why do we bother learning pre-algebra and algebra 1? 2005-12-23
From Priya:
My students always ask "Why do we bother learning pre-algebra and algebra 1?" and I haven't found an answer to satisfy them yet. Can you help me? My students are from grade 9 to 11. I have tried giving them real life examples in each topic but it just feels like they are not satisfied!!!
Answered by Penny Nom.
Cubic yards of fill 2005-12-23
From Mike:
And I have a question in regards to the amount of fill needed to bring a vacant lot up to "grade level". The lot is 112' x 87' and I need to bring in fill to elevate the finished level up 4'.
Answered by Penny Nom.
Percentages in our daily life 2005-12-23
From Naina:
what are the uses of percentages in our daily life
Answered by Penny Nom.
Two dogs and a flea 2005-12-23
From Michelle:
Two dogs, each traveling at 10 ft/sec, run toward each other from 500 feet apart. As they run, a flea flies from the nose of one dog to the nose of the other at 25 ft/sec. The flea flies between the dogs in this manner until it is crush when the dogs collide. How far did the flea fly?
Answered by Penny Nom.
Simultaneous Equations 2005-12-21
From Matt:
I have these two equations,
336 = 60a + 10b
and
432 = 84a + 6b
Am I right in saying both a and b are 4.8?

Answered by Penny Nom.
The game of 24 with 7, 5, 5 and 4 2005-12-18
From Nicole:
i was playing the 24 game the other day. you get 4 digits and you have to turn them into 24. You can use addition, subtraction, division, and multiplication. You have to use all the numbers but you can only use each number once. an example is 2 4 4 1 the answer is (2x4)(4-1)=24. I got stuck on one. The numbers were 7 5 5 4 .
Answered by Penny Nom.
Folding a sheet of paper 2005-12-15
From Victoria:
The current problem is to take a normal 8 1/2 x 11 sheet of paper, take a corner and fold it to meet the opposite corner, and (without actually measuring) produce a formula to describe the result fold/crease.
Answered by Penny Nom.
Solving with fractions 2005-12-14
From A student:

Solve:
1. 3/10k - 1 = 1/2
2. 7 3/4 = 3/8h + 6
3. About how heavy should an object be before you start to measure the object in tons instead of pounds? Explain


Answered by Penny Nom.
5(x-3)/4 =x+1 2005-12-14
From Jennifer:

I have two problems i am stumped on I don't know how to help my daughter.

5(x-3)/4 =x+1
3(x+12)/5 =x +2


Answered by Penny Nom.
Inclusive definitions 2005-12-14
From Layla:

recently the solvable quandary of 5+5+5=550 came up (the question says that you have to put 1 straight line somewhere in the equation to make it true with out turning the "=" into a "not=" sign).

So two answers were put forward:
545+5=550 (the use of a line converting a + into a 4)
AND
5+5+5(less than or equal to)550

There is currently an argument about the second solution. The disagreement is about whether this sign can be used. One person is arguing that the "less than or equal to" sign defines that the number on the left is in the range 550 and below. The other is saying that since the number (which is clearly defined with no variables) can never equal 550, then the "less than or equal to" sign cannot be used in this case.

Which one is the correct definition?


Answered by Walter Whiteley.
LCM 2005-12-12
From Alex:
what is the LCM of 210 and 54 and the LCM of 42 and 126
Answered by Penny Nom.
2/n +4 = 10- 4/3n 2005-12-11
From Jennifer:

2/n +4 = 10- 4/3n

5(x-3/4) = x+1


Answered by Penny Nom.
The stair problem 2005-12-06
From Arnold:

My daughter had me help her with some of her college math problems that require finding the pattern. The problem was the stair problem where you can climb either 1 step or 2 steps at a time. How many combinations are there to get to the 10th step. I found the data set that solves the answer to the question, but is there an equation that expresses the answer in terms of n?

1 2 3 4 5 6 7 8 9 10 stair number
1 2 3 5 8 13 21 34 55 89 number of possible combinations


Answered by Harley Weston.
A sheet metal cone 2005-12-05
From Laura:
I am an art student and in the process of making a cone out of sheet metal. I am unable to work out the template I need to produce my final cone. The dimensions I have are that the final cone will be 58mm high and will have a diameter of 102mm.
Answered by Penny Nom.
How do you simplify a fraction if one of the numbers is negative? 2005-12-05
From Stephanie:

Question: How do you simplify a fraction if one of the numbers is negative?

Ex. -32/40


Answered by Penny Nom.
How is trigonometry applied to everyday life? 2005-12-03
From Yadira:
My question is how is trigonometry applied to everyday lives and functions. Ex: Builders use it but how and what are some examples of the trig-functions or formulas that they use?
Answered by Harley Weston.
Area of a lot 2005-12-03
From Ben:

How do you calculate the area of the following Lot?

I figured the following angles from the deed, which read:

N 86 degrees, 45 minutes E, for 322 feet.

S 10 degrees, 30 minutes W for 113 feet.

N 84 degrees, 30 minutes W for 368 feet.

N 50 degrees, 42 minutes E for 76 feet.

N 40 degrees, 40 minutes E for 15 feet.

There is a discrepancy between two surveyors and I'd like to figure out how to calculate the Area of such a shape.


Answered by Harley Weston.
11 /12 + b = 5/6 2005-12-02
From Jennifer:
11 /12 + b = 5/6
Answered by Penny Nom.
Common fractions to decimals 2005-11-25
From A student:
how do u turn 8 over23 into a decimal and how do u turn 8 over 29 into a decimal
Answered by Penny Nom.
The sum of the angels in a triangle 2005-11-25
From Rachel:
how do you prove, without knowing any of the measurements or degrees, that the three angles of a triangle equal 180? what are the steps for proving that?
Answered by Penny Nom.
Adding improper fractions 2005-11-25
From Paula:
I would like a simple step by step explanation on how to add improper fractions.
Answered by Penny Nom.
BEDMAS 2005-11-24
From Judy:
My name is Judy and I am a grade 6 teacher.
We have just started our lessons on the order of operations and my students have asked me why we have BEDMAS and what is the logic to it.

Is there a reason that that we do math in this particular order?

Who invented this rule or how was it decided on and when?

Answered by Walter Whiteley.
Numbers with exactly 5 divisors 2005-11-23
From Mary Lou:
Show the 10 numbers with exactly 5 divisors.
Answered by Penny Nom.
Gallons and cubic feet 2005-11-19
From Just interested:
I have found that 1 cubic foot will hold 7.48 gallons of gasoline, but I would like to know how to come by this answer.
Answered by Penny Nom.
I = PRT 2005-11-16
From Ryan:
Use the formula to find the value of the variable that is not given:
I=PRT;I=$2880, R=0.08, P=$12,000

Answered by Penny Nom.
What shape do I say a football is? 2005-11-14
From Peter:
I am a 43 year old man. I am no longer in school, I have a bachelors degree. I like to tell stories about sports in ways to make people think about the amount of time they devote to the sports. For example, while attending a church in Chicago on the north side, I would pass by Wrigley field. I began to call it the church of the flying orb or sphere. Now if I want to tell a similar story about football (not soccer), what shape do I say a football is?
Answered by Chris Fisher.
An isosceles triangle 2005-11-14
From Chris:
PX and QY are attitudes of acute triangle PQR, and Z is the midpoint of PQ. Can you write a proof that triangle XYZ is isosceles?
Answered by Chri Fisher.
Factoring quartics 2005-11-13
From Kyle:
How do I factor y4 + y2 +1?? I think the answer is (y2 + y + 1)(y2 - y + 1), but I'm not sure how to get that...
Answered by Chris Fisher.
fog 2005-11-12
From Janice:
I am having problem with the (fog) function

(fog) (x). Given f(x)= 2x2 ; g(x)= 3-4x

Answered by Penny Nom.
Cubic feet and cubic yards 2005-11-12
From Mike:
I need to know how to calculate square feet into yards of material. Example: 1320 sq. ft. surface area by 1 in. depth. What does this equal in cubic feet and yards. Thanks for any help you can give me.
Answered by Penny Nom.
Notation for the second derivative 2005-11-08
From Mussawar:
my question is d/dx( dy/dx) = d2y/dx2. why it is not equal to d2y/d2x.
Answered by Penny Nom.
7000 sq feet = ? acres ??? 2005-11-05
From Victor:
7000 sq feet = ? acres ???
Answered by Penny Nom.
The sides of an octagon 2005-11-02
From Royce:
I understand there is a simple calculation to determine the sides of an octagon when you know the distance across the parallel flats. something like .447 . can you help?
Answered by Penny Nom.
Factoring 2005-10-30
From A student:
Factor 18x3 y3 + 12xy2
Answered by Penny Nom.
A sum of 75 and a difference of 13 2005-10-30
From Vicki:
Find two numbers that have a difference of 13 AND A SUM OF 75? We need to learn about the formula or procedure.
Answered by Penny Nom.
Velocity and acceleration 2005-10-27
From Candace:
When taking the integral of the position function, you get the velocity function, and the same for velocity to acceleration. So when you do each of these, you get a function. But when you integrate on a graph, you get an area under a curve. The area is un units squared- where do the units go when you make it an equation? How can a function be an area?
Answered by Harley Weston.
A slump cone 2005-10-27
From Wendy:
we are trying to make a slump cone (used to measure the slump in concrete). It has dimensions of 8" diameter on the bottom, 4" diameter hole on the top and a height of 12". Please help, it is getting frustrating.
Answered by Penny Nom.
How many cubic yards of dirt will be needed? 2005-10-27
From Mike:
We are filling in a hole left by the removal of an aboveground pool that was dug into the ground. The diameter is 25' across and the depth is 5'. I wondered how many cubic yards of dirt will be needed to fill this hole.
Answered by Penny Nom.
Cubic feet and imperial gallons 2005-10-27
From Dave:
What is the volume in cubic feet which is occupied by 1080 Canadian gallons of water?
Answered by Harley Weston.
Improper fractions 2005-10-21
From Joe:
Is a whole number an improper fraction. For example, is 4 an improper fraction? Is 4 an improper fraction? Is 4 the simplest form for 20/5? Are 4 and 20/5 equivalent fractions?
Answered by Penny Nom.
How many cubic feet are in 8 yards? 2005-10-20
From Neisha:
I am trying to figure out how many cubic feet are in 8 yards. I am trying to pour a concrete slab. The width is 24 ft, lenth is 30 ft, and it is 4 inches thick
Answered by Penny Nom.
Painter's caps 2005-10-19
From Duk:
The equation C=10 +2n represents the cost in dollars, C, for n painter's caps advertising the walkathon. Which pair of values could represent a number of caps and the cost for that number of caps, (n, C)?

(0, 10) (7, 24) (15, 30)

Answered by Penny Nom.
Coefficient of variation 2005-10-19
From Jan:
I am currently teaching the coefficient of variation and am wondering if there are some guidelines as to the interpretation of this statistic. I understand that it measures the variation in a variable relative to the mean - but what is the cut off for "too much" variation expressed in this way???
Answered by Andrei Volodin and Penny Nom.
Can we take the derivative of independent variable 2005-10-18
From Mussawar:
why we take derivative of dependent variable with respect to independent variable .can we take the derivative of independent with respect to dependent.if not why.
Answered by Walter Whiteley.
Divisibility by each of the first ten counting numbers 2005-10-17
From Simon:
determine smallest positive integer that is divisible by each of the first ten counting numbers
Answered by Penny Nom.
How many gallons will it hold? 2005-10-16
From Scott:
I am building a pond and need to convert the cubic dimensions into gallons in order to know what size fountain pump to purchase.

My pond is 1 foot deep x eight feet long x three feet wide.

How many gallons will it hold?

Answered by Penny Nom.
The coefficient of thermal expansion for steel 2005-10-14
From Jim:

Is the following statement true?

The coefficient of thermal expansion for steel is 0.00000645in/in/deg. Doesn't sound like much but when you run out the numbers it comes to .405504 ft/mile/deg. Still doesn't sound like much, only about 5". Then multiply by 40 degrees and you get a piece of rail that has grown by 16.22 feet in that one mile. It's not at all unusual for the rail temp to go from say, 40 deg to 80 deg on a spring or fall day. Remember that on a sunny day, the rail temp can be significantly higher than the air temp as well."

I ran the math and came up with an answer closer to 16 inches, instead of 16 feet. Which is closer to being correct?


Answered by Penny Nom.
The bathtub curve 2005-10-13
From David:

My father asked me to submit a question about the so-called 'bathtub
curve'. If you cut a bathtub in half lengthwise down it's middle, the
edge of the tub would describe the 'bathtub curve' which can be used
to demonstrate typical failure rates of products. This curve is
characterised by high initial (infant mortality) failure rates at
it's beginning, which drop quickly to a very low level. Failures then
increase gradually to the "end of life" stage where the failure rate
takes off dramatically again.

If anyone in the math department knows about the so-called 'bathtub
curve' my father would really appreciate the equation.


Answered by Chris Fisher and Edward Doolittle.
The game of 24 2005-10-10
From Kim and Chris:
My 4th grade son came home with a math game called 24 game. In it each card has 4 numbers on it. Each number can only be used once, in any order, using multiplication, subtraction, division, or addition. We are stuck on one and wondered if you could help. The four numbers are: 9, 9, 9, 9. And they must equal 24. Any clues?
Answered by Claude Tardif.
The remainder theorem 2005-10-07
From Ron:
If f(x) is divided by (x-3), the remainder is 5, and if it is divided by (x+1), the remainder is -3. Find the remainder when f(x)is divided by (x-3)(x+1).
Answered by Chris Fisher.
An odd number of factors 2005-10-06
From Ramneek:
What is the common name used for numbers that have an odd number of factors?
Answered by Claude Tardif.
Coefficients, constants and like terms 2005-10-05
From Elizabeth:
In the equation -8y+6ab+7-3ab what are the coefficients; the like terms and constants?
Answered by Penny.
Prove that a rhombus' diagonals are perpendicular 2005-10-02
From Tania:
How do you prove that a rhombus' diagonals are perpendicular using the 2 column proof method?
Answered by Walter Whiteley.
2x/x^2-9 - 6/x^2+x-12 2005-09-27
From Emily Ann:
2x/x2-9 - 6/x2+x-12
Answered by Penny Nom.
6x+7=8x-13 2005-09-27
From A student:
show me how to do 6x+7=8x-13
Answered by Penny Nom.
14p - 8 = 22 + 20p 2005-09-27
From Tiffany:
14p - 8 = 22 + 20p
Answered by Penny Nom.
A field with the largest possible area 2005-09-25
From Louise:
A FARMER HAS FENCING OF 1000M AND WANTS A FIELD WITH THE BIGGEST POSSIBLE AREA HOW DO I GO ABOUT DOING THIS
Answered by Penny Nom.
U'(X) - U(X) = 0; U(0) = 2 2005-09-23
From David:
Out of interest could you please answer the following questions?

U'(X) - U(X) = 0; U(0) = 2

and

U''(X) - U'(X) = 0; U'(0) = U(0) = 2


Answered by Harley Weston.
Isn't 1/3 written as a decimal .33? 2005-09-21
From Lisa:
Isn't 1/3 written as a decimal .33? My son informs me I'm wrong, but doesn't know the answer
Answered by Chris Fisher and Penny Nom.
How do you solve 3x^2 + x - 2 = 0 2005-09-15
From Lizzy:
How do you solve 3x2 + x - 2 = 0
Answered by Penny Nom.
A piecewise function 2005-09-15
From Duncan:
A child is assigned to your care and she has a headache. The parent has authorized the administration of Children's Tylenol. Read a box of Children's Tylenol (or any other similar drug) and find the directions for administration. Note the child's weight and the corresponding dosage. (Be sure the drug you choose has a minimum of 4 weight intervals.)
Answered by Penny Nom.
A rational arithmetic expression 2005-09-14
From Vijay:
How to simplify following Grade 9 math problem:

(4/-9) X (-21/-32) X (-3/14)

Answered by Penny Nom.
How do you differentiate y=(x)^(x^x)? 2005-09-14
From Calebius:
How do you differentiate y=(x)(xx)?
Answered by Penny Nom.
Equivalent fractions 2005-09-13
From Cindy:

I am a parent and aunt of a 5th grader.

My nephew has asked me what the equivalent decimal is of 0.4. Does he change it to a fraction or just add a zero either before or after?


Answered by Chris Fisher and Harley Weston.
A quadratic 2005-09-06
From Ginger:
I took a pre alg test today. There was a question on it that I am not sure and is bothering me. Can you tell me the answer?

x square + 13 + 14

Factor this

Answered by Penny Nom.
A system of equations 2005-09-02
From Gina:
How do you interpret the solution of a system of equations by the corresponding graph? Demonstrate your answer by the use of an example.
Answered by Penny Nom.
BEDMAS 2005-09-01
From A student:
I am a student and am wondering about the answer to this question.

56/2(31)

is the answer 7?


Answered by Harley Weston.
A typical farm 2005-09-01
From Pagedi:
A typical farm is about 700 acres. How many square miles is this? Please show me the work so, that I will know I to perform this problem myself.
Answered by Penny Nom.
12-42, 8-26, 10-34, 9-30, 16-58 2005-08-31
From Elizabeth:
here is a math question my niece gave me, grade 7

12-42, 8-26, 10-34, 9-30, 16-58

first day of school and this is what they give her no explanation

Answered by Harley Weston.
The area of a lot 2005-08-29
From Richard:
My wife and I are interested in buying property in Idaho but the owner can't give us a square footage of the lot. The dimensions are as follows:

121.0 on the left side
157.0 on the right side
135.0 on the bottom
162.0 on the top

The bottom right corner of the lot is a true right angle, the rest are not.

Answered by Penny Nom.
The length of a lot 2005-08-28
From Billy:
If I have 3 acres and the front is 300 ft across how many feet would I go down the sides to equal 3 acres?
Answered by Penny Nom.
y = log(x) + x. Solve for x. 2005-08-26
From Alain:

I have the following equation:

y = ln(x) + x

How do I solve for x?


Answered by Penny Nom.
Labour efficiency 2005-08-23
From Rob:

The problem, on the surface, seems very simple and yet has created some controversy among a group of accountants. The problem itself has to do with labour efficiency rates and only involves two variables; standard working hours, and actual working hours. The difficulty lies in deriving an efficiency % from these two numbers.

Standard working hours or the targeted number of labour hours required to produce one widget, which I will represent as "s". Actual working hour or the actual number of labour hours require to produce one widget, which I will represent as "a". Labour efficiency I will represent with "E". The prevailing calculation with which I have a problem with is this:

s/a=E or if s=3000, and a=4000 then 3000/4000=75%

What bothers me about the calculation is that the standard hours get represented as a percentage of the actual hours and in my opinion changes the focus of the calculation from standard or target, where it should be, to the actual hours. I cannot define why, but this just seems inherently wrong to me.
The calculation that I use:

(1+((s-a)/s))=E or if s=3000, and a=4000 then (1+((3000-4000)/3000))=66.67%

My calculation is like a %change from standard calculation. However, there is something that also concerns me about my calculation.

If you substitute 100 for a and 50 for s, then you come to a quandary, because if you plug those numbers into the second equation the result is of course zero % efficient which doesn't sit right with me either. If you plug them into the first calculation you get 50% efficiency which doesn't really seem to work either, because you require 100% more hours to do the same work in this case. ???

Is the first calculation correct? Am I missing something altogether? Are both calculations off base?


Answered by Harley Weston.
The range of a function 2005-08-21
From Kelsey:
What is the range of the function f(x) = 4x - 6 when the domain is {-3,-1,1,3,5}?
Answered by Penny Nom.
3A/4 +1=(2A-1)/3 2005-08-21
From Rachael:

I'm doing a practice test question and the answer I keep turning out is different from the answer on the answer sheet. I think I might be forgetting a step. The problem is:

3A/4 +1=(2A-1)/3

The answer is -16. If anyone could explain the steps to me to getting the answer I would be grateful. Whenever I do it I get something completely different.


Answered by Penny Nom.
Which number is greater? 2005-08-18
From Dante:
Which number is greater: 888....88 X 333...33 or 444.......44 x 666........67 (each of the numbers has 1989 digits)?
Answered by Claude Tardif.
My fantasy football league 2005-08-16
From Jim:
I have to make a schedule for my fantasy football league. There are 10 teams in the league and we play a 13 week season. The challenge is we want to play each team only once during the first 10 weeks. This way no team will play another team twice without having played each team at least once. Is this possible?
Answered by Penny Nom.
Stock in a printing company 2005-08-12
From Tracy:
Sam and Mary each owned one-half stock in a printing company. Sam sold 2/3 of his stock to Mary. What fractional part of the printing business does Mary now own?
Answered by Penny Nom.
Framing an arched wall 2005-08-12
From Mike:
I'm framing a building wall with a curved (arcing) top section. The radius of the section is 74'6" with a height above finish floor of 16'0". The horizontal run of the arced section is 23' 1 1/2" with a low height above finish floor of 12'4". If I start with a 16' stud at the high end how long are the subsequent studs if they are on 16" centers? Short of laying this out on a tennis court how can I work out the lengths of the studs?
Answered by Penny Nom.
Proof by induction? 2005-08-10
From Peter:

I am a lecturer and am having a problem with the following Proof by
Induction.

If

(N x N x N x N) + (4 x N x N x N) + (3 x N x N) + (N) = -4000

Prove that N is even!


Answered by Chris Fisher and Penny Nom.
Cubic feet 2005-08-10
From Selene:
This is my math question I need to convert these inches to cubic feet 34"W x 34"H x 34"L
Answered by Penny Nom.
Constructing a fence 2005-08-09
From Andres:
I was constructing my fence and was having some problems trying to do a perfect (or as close as possible) arch. If i know a section of a fence is for example 85 inches wide and I want a 4 inch rise from the top rail how do i figure out the radius?
Answered by Penny Nom.
Converting rational numbers 2005-08-07
From Joe:

I am helping my son with his converting rational
numbers in the form of A/B where A and B are integers and
B not equal to zero

I think I am doing this right but I am not sure so
below are the problems and our answers are beside
them, please let me know if these answerers are correct


Answered by Penny Nom.
A golf outing for 16 golfers 2005-08-05
From Bob:
I have a golf outing fast approaching and I’m having a problem matching the golfers so that everyone plays a round of golf with every other golfer in the group.

My problem is this: I have 16 golfers playing 5 rounds of golf. I would like a different foursome for every round of golf, that is each round (5 total) an individual is playing with 3 other players he hasn’t played with as of yet. Can you crunch the numbers and get back to me?

The way I see it, we have 20 combinations (5 rounds x 4 golfers) with numbers 1 thru 16.

Answered by Chris Fisher.
A perfect square that is equal to 1 acre 2005-07-31
From Shannon:
If you have a perfect square that is equal to 1 acre, what is the length of the sides and how do you do it?
Answered by Penny Nom.
Cubic feet in a mobile home 2005-07-22
From Jeanne:
my mobile home is 24 x 56. I'm trying to find out how many cubic feet this is. I'm going to set of some bug bombs and it says 1 can will cover 4000 cubic feet. I don't know how to figure my cubic feet. the ceilings are 8 ft high everywhere but the living and dining room. They are vaulted and approximately 10 feet at the peak. I just need an estimate.
Answered by Penny Nom.
Solve for x 2005-07-20
From Ed:
(x-1)/3 - 1 = (x + 2)/2
Answered by Penny.
How many tiles do I need? 2005-07-20
From Jeannette:
I am putting up backsplash tile in my kitchen and I need to know how many tiles I would need to use for about 45 square feet using 4x4 tiles.
Answered by Penny Nom.
The area of a lot 2005-07-17
From John:
A plot of land has the following dimensions: 391 ft. x 757 ft. x 208 ft. x 788 ft. Q: How many acres is this?
Answered by Harley Weston.
Profit of more than 100% of cost 2005-06-22
From sam:
I know I can work out what percentage profit I am making by : profit/cost x 100, and I can work out up to 99% profit (the way I currently calculate it is profit x 1.99) But how do I work out any amount over 100%, say for instance I have something that costs me $0.52 that I wish to sell for 250% profit?
Answered by Penny Nom.
An algebraic remainder 2005-06-22
From Raymond:
Given that 4x3 - kx is divisible by 2x+1, find the remainder when the expression is divided by 4 - x2.
Answered by Penny Nom.
20% profit 2005-06-12
From Shawn:
If I know that I will be paid, say, $150.00, for labor, and I want to make 20% profit on what I pay for the labor, what is the calculation and factoring process?

I see it like this

(x*.2)+x = 150

but I don't know how to factor it out from there.

Answered by Penny Nom.
The sum of the digits of 2^100 2005-06-11
From Richard:
The sum of the digits was calculated for the number 2100, then the sum of the digits was calculated for the resulting number and so on, until a single digit is left.
Answered by Penny Nom.
The size of a lot 2005-06-09
From Bob:
Is it possible to determine the square footage and acreage of a residential lot that's straight on 3 sides, and triangular on the 4th side?

As shown in the crude sketch below, the two legs of the triangle at the back end of the lot are 61' and 62'.

Answered by Penny.
Digging a hole for a pool 2005-06-08
From Debra:
I need to have dirt hauled away from my built in pool and the haulers are asking me how many yards of dirt there is. How would I figure that out? It is a 24 ft round pool and 3 inches of soil depth was dug out.
Answered by Penny.
An octagonal deck 2005-06-07
From Scott:
I want to build a octagonal deck. The wood I am using are cut in 8 foot lenghts. What I want to know is if the sides of the octagonal are 8 feet, what is the diameter. Also what are the angles of each side?
Answered by Penny Nom.
Square feet and cubic feet 2005-05-30
From Kim:
Is there a relationship between square feet and cubic feet?
Answered by Penny Nom.
The volume of a hopper 2005-05-28
From Brian:
I would like to know the volume of this rectangular hopper. can you help
Answered by Penny Nom.
Logarithmic differentiation 2005-05-23
From Richard:
I need to convince myself that I understand the process of differentiating y=xx.
The specific question is that if I have to take the logarithm of both sides of the equation how can differentiate the following?
y= {(x+2)(x+2)}/{(x+1)(x+1)} - {(x+1)(x+1)}/(xx),
I have an idea that the differential of this fairly complex function is itself ... am I right or wrong.

Answered by Penny Nom.
Cubic feet of concrete 2005-05-23
From Robert:

REQUIRED CUBIC FEET OF CONCRETE TO POUR AN EXTENDED PAD

8 FEET X 8 FEET X 6 INCHES DEEP


Answered by Penny Nom.
A triangular lot 2005-05-21
From Linda:
I am looking to purchase a piece of property, and I need some help trying to figure out the square footage of the land. I know that normally if I multiply the length and width I could get my answer. However, this piece of land is an odd shape and has 3 measurements. Can you please help me.
The measurements for this piece of land is:

97.8'
201.2'
139.7'

Answered by Penny Nom.
Factoring 2005-05-20
From Abraham:
Can you please explain to me how to factor completely the following problem:
(sqrt(x+1))-((x2+2x+1)( -7/4)).

Answered by Penny Nom.
The circumference of an oval pool 2005-05-18
From John:
I have an oval pool of which I am trying to find the circumference. it is 38 feet long and 19 feet wide
Answered by Penny Nom.
Jobs relating to the topics of ratios and proportions 2005-05-17
From Alexa:
What are some jobs relating to the topics of ratios and proportions other than architects and map makers?
Answered by Walter Whiteley.
How many cubic feet in a yard? 2005-05-13
From Carol:
How do you measure a cubic foot? How many cubic feet in a yard?
Answered by Penny Nom.
Surface areas 2005-05-11
From Jessica:
How can I demonstrate to my high school students the reason for the formulas for the surface area of a prism, right cylinder, and regular pyramid, and right cone?
Answered by Penny Nom.
sin x + sin 2x + sin 3x + sin 4x = 0 2005-05-10
From Elia:
I tried many times, but can't get to solve the following question:
sin x + sin 2x + sin 3x + sin 4x = 0

Answered by Chris Fisher.
Some triangle problems 2005-05-06
From A student:
1.use the heron's formula to find area of triangle ABC to nerest tenth

A.a=21 b=28 c=11

B.a=23 b=28 c=12

2.Find the nearest tenth the altitude of the longest side if a=3 b=3 c=5

3.the length of each side of a rhombus is 10 cm the length of one diagonal is 12 cm find the area to the nearest square centermeter

Answered by Penny Nom.
y = x+2 and y = 2x - 1 2005-05-03
From Kimberley:
Question: y=x+2 y=2x-1
Answered by Penny Nom.
16x^2+20x-12x-15 2005-05-03
From Tanya:
how they got (4x+5)(4x-5) from
16x2+20x-12x-15=4x(4x+5)-3(4x+5)=(4x+5)(4x-5)

Answered by Penny Nom.
Volume and surface area of a prism 2005-04-29
From Melissa:
Can you please tell me if the volume and surface area of a prism can ever be the same? Could you please explain why or why not.
Answered by Walter Whiteley.
40% profit 2005-04-19
From Carol:
Could you please tell me the formula to calculate for example a 25% profit on cost. I.e a sandwich cost .75p for the cost of ingredients, I wish to make a 40% profit on that cost so what do I multiply and divide it by to arrive at the selling price in order to achieve a 40% profit?
Answered by Penny Nom.
A flaw in a problem 2005-04-15
From Bryce:

Question:

(x2-x2) = (x2-x2)
x(x-x) = (x+x)(x-x) [divide both sides by (x-x)]
x = x + x
x = 2x [divide both sides by x]
2 = x/x = 1

Where is the flaw in this problem?


Answered by Paul Betts.
Difference in latitude 2005-04-05
From Gretel:
Assuming the the earth is a sphere of radius 6378 kilometers, what is the difference in latitude of two cities, one of which is 400 kilometers due north of the other? 500 kilometers?
Answered by Penny Nom.
Rational functions 2005-04-05
From Nicole:
My name is Nicole and I am a teacher at Weyburn Comprehensive School. I am currently teaching both Math B30 and Calculus 30 at the school and I have a question about rational functions. I know that if a rational function (by definition) has common factors in the numerator and the denominator then it is not a rational function (math b30) however in calculus this common factor creates a hole in our graph. Can you explain to me why a common factor or constant does not give us a rational function?
Answered by Penny Nom and Leeanne Boehm.
Bayes Theorem 2005-04-03
From wei:

In the January 11,1988,issue of the Oil&Gas Journal, R.A.Baker describes how the Bayesian approach can be used to revise probabilities that a prospect field will produce oil. In one case he describes, geological assessment indicates a 25% chance the field will produce oil. Further,there is an 80% chance that a particular well will strike oil given that oil is present on the prospect field.

  1. Suppose that one well is drilled on the field and it comes up dry. What is the probability the prospect field will produce oil?
  2. If two wells come up dry, what is the probability the field will produce oil?
  3. The oil company would like to keep looking as long as the chances of finding oil are greater than 1%. How many dry wells must be drilled before the field will be abandoned
  4. If the first well produces oil,what is the chance the field will produce oil?

Answered by Andrei Volodin and Penny Nom.
A wishing well 2005-03-28
From Don:

I am building a wishing-well out of pieces of 2-by-4. I have included a picture of a miniature version of what I want. There are to be ten 2-by-4 pieces around the well and I want the circle around the outside of the structure to have a diameter of approximately 3 feet. How long to I cut the 2-by-4's to build the wishing-well.

Thanks,
Don


Answered by Harley Weston.
A torus and a sphere 2005-03-27
From Tony:
Is it possible to shrink a torus into a sphere?
Answered by Andrei Volodin and Penny Nom.
The answer to a subtraction problem 2005-03-22
From Libby:
What is the name of a number that is the answer to a subtraction problem?
Answered by Diane Hanson and Claude Tardif.
Dimensions of a roof 2005-03-18
From A roofer?:
A right triangle (roof of a house) has a base of 7 feet and a 22 degree angle. What is the height of the roof and what is the hypothenus of the triangle.
Answered by Penny Nom.
The volume of a stock tank 2005-03-13
From Cord:
I am a farmer and I have recently installed a new stock tank which is 1650" in circumference and 10' deep. How many gallons of water will it hold?
Answered by Penny Nom.
21yd 2ft divided by 5=what? 2005-03-10
From A student:
1gallon 3qt divided by 6= what?
21yd 2ft divided by 5=what?

Answered by Penny Nom.
Perfecting an ideal gambling system 2005-03-06
From Gaz:
I am a screenwriter, currently in the fortunate position of having the development of a Screenplay funded by the South Australian Film Corporation. The (anti)hero of this screenplay is a statistician whose life is falling apart around him, thanks in part to his obsession with perfecting an "ideal" gambling system.
Answered by Andrei Volodin.
The area of a quadrilateral 2005-02-27
From Jonathan:
I want to know how to find the area of a quadrilateral.
Answered by Penny Nom.
A countably infinite collection of countably infinite sets 2005-02-26
From Feroz:
Suppose a set can be divided into a countably infinite number of countably infinite sets.Then can the original set be considered as a countably infinite set?
Answered by Penny Nom.
A 3-digit product 2005-02-25
From A student:
I have an odd 3- digit product. My factors are a 1 digit number and either a 2 or 3 digit number. My factors are odd, but neither is 1. Find my factors and my product.
Answered by Penny Nom.
Trading fish for bread 2005-02-25
From Mike:
Byron lives where people trade goods they produce for other things they need. He has some fish and wants to trade them for bananas. He finds the following:
5 fish = 2 loaves of bread
6 oranges = 2 melons
1 loaf of bread = 1 banana and 3 oranges
4 loaves of bread = 14 oranges
How many bananas can Byron get with 5 fish?

Answered by Penny Nom.
Cubic feet and feet 2005-02-25
From Nancy:
How do you convert cubic feet into feet?

The problem is to convert 90 cubic feet into feet.

Answered by Penny Nom.
Factoring a cubic 2005-02-20
From Abraham:
How would you factor completely,
x3-x2+5x-5?

Answered by Claude Tardif.
Understanding fractions 2005-02-11
From C.J.:
I have some questions about the lesson, "Understanding Fractions" by Diane Hanson, Regina Catholic Schools. How successful was the lesson? Are there any changes you would recommend?
Answered by Diane Hanson.
Differentiating F(x,y) = 0 2005-01-23
From Jacob:
In calculus, we often mention to the students that if F(x,y) = 0, then we can differentiate both sides and still get an equality. The problem is that we can't perform the same operation on F(x) = 0, say x = 0, otherwise 1 = 0, which is absurd. What is the reason?
Answered by Walyer Whileley and Harley Weston.
The diameter of a 800 km circle 2005-01-22
From Mechelle:
What is the diameter of a 800 km circle?
Answered by Penny Nom.
A 6 sided (hexagonal) pyramid 2005-01-22
From Steve:
im trying to make a 6 sided (hexagon) pyramid, from 6 triangles of 12mm plywood, i know all the angles to cut apart from the one one to join all 6 triangles together. Rough measurements are outer edge (A) of each triangle is 13cm's, length of other 2 sides (B&C) of triangle outside to center is 14cm's with a height of the whole thing together about 6cm's.
Answered by Chris Fisher and Harley Weston.
Finding the missing side of a triangle 2005-01-20
From Jason:
I found a geometry problem that reads as follows:In triangle ABC,
Answered by Penny Nom.
The angle between two forces 2005-01-19
From Abraham:
One force of 20 pounds and one force of 15 pounds act on a body at the same point so that the resultant force is 19 pounds. Find, to the nearest degree, the angle between the two original forces.
Answered by Penny Nom.
Numbers that John likes 2005-01-16
From Garrett:
John likes 400 but not 300; he likes 100 but not 99; he likes 3600 but not 3700. Which does he like?

900
1000
1100
1200

Answered by Penny Nom.
The game of 24 2005-01-10
From Manny:
HI - my daughter and I are being driven crazy by the following "24" question 7,7,3,3. She's good at the game (well, better than I am) and her teacher gave her this one. Any insight would be appreciated!
Answered by Claude Tardif.
Profit 2005-01-10
From Abraham:
The profit a coat manufacturer makes each day is modeled by the equation P(x)=-x2+120x-2000, where P is the profit and x is the price for each coat sold.For what values of x does the company make a profit?
I don 't understand this problem(how to do it) and hope you can help me.

Answered by Penny Nom.
ln(hx+n) 2005-01-07
From Anibal:
if I have the eq. y=hx+n and I want to know ln(y), how do I process ln(hx+n)?
Answered by Penny Nom.
Quadratics 2005-01-05
From Usman:
Hi, in my Grade 11 Functions math class we have been assigned the task of finding jobs and careers related to quadratics, I have done many searches but have been unsuccessful, then I saw your website and e-mailed. I also have to use an example of a math problem that the job uses, then solve it, this will all compile on bristol board for a presentation. I would greatly appreciate it if you could send me some links and references of sources that refer to this subject.
Answered by Harley Weston.
An isosceles triangle 2005-01-03
From Abraham:
The question is,"Triangle ABC is not isosceles.Prove that if altitude BD were drawn, it would not bisect AC."My question is If an altitude is drawn wouldn\'t that mean automatically its isosceles because, In a triangle the sides opposite congruent angles(in this case the right angles)are congruent? What am I thinking wrong?
Answered by Harley Weston.
A sequence of transformations 2005-01-03
From Abraham:
Please go to this web site and look at number 31.

http://www.nysedregents.org/testing/mathre/matharch/mathbtestau02.pdf

That is the New York state education web site. Can you tell me if my answer would be marked correct? My answer was Transformation of 4,-3 (T 4,-3) followed by a reflection in the x-axis.If I graphed the question and wrote that, would I get full credit

Answered by Harley Weston.
An elliptical table 2005-01-03
From Roger:
Want to make an elliptical table, say the long (major) axis is 4 feet, and the short (minor) axis is 3 feet. I can construct this figure, but I'm trying to figure out what the exact dimension of a rectangle within this ellipse will be if I make the table a drop leaf type where the drop dimensions are equal for each end of both the long and short axes. Intuitively, it looks like there is one and only one solution.
Answered by Penny Nom.
The area of a triangle 2004-12-30
From Perry:
What is the area of a triangle with dimensions 3"X5"X7"? Could you provide the formula?
Answered by Penny Nom.
2^9833 days from today 2004-12-26
From Pauline:
Today is Monday, 1 October 2001. What day of the week will be 29833 days from today?
Answered by Penny Nom.
Half of five 2004-12-26
From Pauline:
Why is four half of Five? Hint: It's the middle half?
Answered by Penny Nom.
252 x ? is a cube 2004-12-22
From Andrea:
What is the smallest positive interger by which 252 can be multipled so the result is a perfect cubed?
Answered by Penny Nom.
Divisibility by 15 2004-12-19
From Lisa:
My son was asked to find divisiblity rules for 15. We have been unable to find the answer. Does it exist?
Answered by Leeanne Boehm and Denis Hanson.
A two equation fact family 2004-12-17
From Tony and Hailey:
My Daughter Hailey had this question as part of her 4th grade math homework. Name 2 fact families that have only two equations. For the life of me I can't figure it out. Any ideas?
Answered by Penny Nom.
0.999..., asymptotes and infinity 2004-12-17
From Mike:
My Name is Mike and I teach high school. I had a student ask me to explain why .9 repeating is equal to 1. Then he asked me about an asymptote, or why a parabola or any other curve for that matter can continually approach a value (like 1) and yet never attain a value of 1. He is thinking that these two should represent the same concept and yet they contradict each other. Do you have a solid explanation for him? Of by the way he is a 7th grader. Great little thinker!!!!!
Answered by Claude Tardif and Harley Weston.
ln(x)/x 2004-12-11
From Tina:
What is the derivative of (ln x)/x? The double derivative?
Answered by Penny Nom.
A geometric proof 2004-12-11
From Hanna:
Given: ABCD is a quadrilateral;
Prove: ABCD is a parallelogram

Answered by Penny Nom.
"a" cubed minus "b" cubed 2004-12-02
From Denise:
"a"cubed minus "b"cubed equal (a-b) times
("a"squared plus "ab" plus "b"squared)?

I know this is a formula, but why is it true?

Answered by Penny.
Select a card from the deck. 2004-12-02
From Heidi:
Select a card from the deck. What is the probability that this card will be red? Show the number of expected outcomes versus the number of total possible outcomes. What type of event does this represent?
Answered by Penny.
The substitution method 2004-11-28
From A student:
I Was working and i ran into this problem can you help me solve it using the substitution method?

-3x-2y=-10
x+5y=-27

Answered by Penny Nom.
The tangent line at an inflection point 2004-11-28
From Louise:
the equation of the tangent line to the curve y = x3 - 6x2 at its point of inflection is...
Answered by Penny Nom.
Computing confidence intervals 2004-11-26
From Christie:
I was given a question with N=100, sample proportion is 0.1- compute the 95% confidence interval for P? I have tried this several ways but do not know how to do without means, standard deviations, standard error of the mean? I asked my teacher and she said I have all the info I need. Can you help????
Answered by Penny Nom.
Is a square a rectangle? 2004-11-21
From Carol:
I am a teacher. In an FCAT sixth grade review test, there was a question to the students to draw a square and then they referred to it as a rectangle.

What is the definition that makes a rectangle a square that can be taught to the students without confusing them.

Answered by Walter Whiteley.
Proof by induction 2004-11-20
From Vic:
Problem: Find the first 4 terms and the nth term of the infinite sequence defined recursively as follows:

a(1) = 3 and a(k+1) = 2a(k) for k -> 1.

Note: Quantities in brackets are subscripts
-> means 'equal to or greater than'.

Using the recursive formula, the first 4 terms are; a(1) = 3, a(2) = 6, a(3) = 12, a(4) = 24

The nth term a(n) = 2n-1 x 3 (equation 1)

Equation 1 must be proven using mathematical induction. This is where I am having a problem.

Answered by Penny Nom.
Solving quadratics 2004-11-20
From Aaron:
Can you solve the following equation (quote the value of x,y and t to 3 sig figs)

((i have changed figures so that i can work through my own example))

x2 + x - 6 = 0

5y2 +19y = 3 10cos2 t + 3cos t = 4

Answered by Penny Nom.
Construction of a cone 2004-11-20
From John:
I am a builder working on a project where I need to make a cone. It's a right circular cone with 15" base radius and slant angle of 30 degrees. I want to cut it out of flat sheet metal then bring the edges together to form the cone. Is this enough information?
Answered by Penny Nom.
1/4 tsp salt = ??? grams 2004-11-16
From Julie:
if i have 1/4 teaspoon of salt how many grams does that equal
Answered by Penny Nom.
A fixed point problem 2004-11-14
From Bob:
I am trying to solve an existence of fixed point problem. I need to show that a function f (on reals) with f'(x)=>2 has a fixed point.
Answered by Claude Tardif.
An ODE 2004-11-10
From David:
I have a question that i really cant do, it is as follows:

The ODE dy/dx + 0.5y = 0.5e^(-1.5x) ; y(5) = 2

Solve the ODE subject to the given condition using exact methods and evaluate the solution y for x = 5 x=5.2 x=5.4 x=5.6 x=5.8 x=6

Answered by Harley Weston.
Order of operations 2004-11-10
From Andrew:
I'm trying to solve this question, and I can't seem to remember the rules back from my high school days.

(40 x 8 ) / 2 + 55 - 15 =

Can you help me with the answer?

Answered by Harley Weston.
Bedmas 2004-11-09
From Fariha:
i have a single line question and am not sure of the method it would be appreciated if u could send me a method and a solution Q : 1 + 9 / 2 + 5 * 1000

would this be solved by BEDMAS? if not, why?

Answered by Penny Nom.
Factoring 2004-11-09
From Rise:
Q1: a2 + 5a + 6

I didn't really understand the purpose of the teacher's steps, but was able to get my daughter to find factors of 6 and then add them together to find which resulted in the sum of 5.

Unfortunately, this problem-solving technique did not help us to solve for the more complex problems as follows:

Q2: 9a3 - 24a2 + 12a

Answered by Penny Nom.
The points of intersection of two graphs 2004-11-05
From Benjamin:
How do I find the points of intersection of the two functions:
1) y =
2 - e-x 2) y = 1 + x2

Answered by Harley Weston.
The least common denominator 2004-11-03
From A student:
Write the LCD for each pair of fractions.

13. 1/3,1/5

14. 2/7,1/4

15. 3/4,3/5

Answered by Penny Nom.
Multiplying two algebraic fractions 2004-10-31
From A parent:
Multiply:

x+2 x²-4
----- x -------
x-2 x²+x-2


Answered by Harley Weston.
GCD and LCM 2004-10-31
From Cartalina:
how do you calculate the "positive difference between the GCF and LCM of two numbers"?
Answered by Penny Nom.
The combined force of two vectors 2004-10-30
From Brian:
Two teams are playing push ball with a large 8 foot diameter ball. One team exerts a force represented by the vector a = 2i + -5j, and the other team exerts a force represented by the vector b = -8i-3j.

1.Determine the direction of movement of the ball if the i axis is due east.

2.Determine the combined force magnitude.

Answered by Harley Weston.
Solving triangles 2004-10-30
From Allen:
Solve the following triangles.

Given

1. B = 20 Degrees, a = 25, b = 16
2. A = 35 Degrees, b = 2, c = 3
3. A = 32 Degrees, C = 44, c = 20

Answered by Harley Weston.
Bedmas 2004-10-30
From Gina:
I am just curious whether Bedmas would be used in the following question as it is not in the typical Bedmas format.

Multiply 12 x 24
Add 26
Divide by 2
Subtract 7


Would we go about doing it in the sequence it is given or in Bedmas ?

Answered by Penny Nom.
The force of a 5000 tonne lorry 2004-10-27
From Aaron:
A 50 tonne lorry is parked on a slope with a gradient of 4%. Due to the self weight of the lorry, what is the force

* acting down the slope
* acting normal (at right angles) to the slope

Answered by Penny Nom.
Ratios and rates 2004-10-27
From Kenneth:
What is the difference between a ratio and a rate?
Answered by Penny Nom.
20 Golfers 2004-10-26
From Ian:
I have 20 golfers playing over 6 days I would like to schedule all 20 golfers in 5 groups of 4 on each day here is the catch.... I would like each player to play with each other once during the 6 days.
Answered by Chris Fisher.
The point slope form of a line 2004-10-26
From Jack:
Given a set of ordered pairs, ie (1,1) (2,4) (3,7), how does one determine the rule f(n) other than by trial and error
Answered by Penny Nom.
Implicit differentiation 2004-10-24
From Emily:
If x^3+3xy+2y^3=17, then in terms of x and y, dy/dx =
Answered by Penny Nom.
Maximize income 2004-10-24
From Connie:
A company that sells x units of a product generates an income (I, in dollars) which is a function of x. The income generated is described by the equation

I = (-1/2)x^2 + 100x.

Discuss how to determine the number of units that must be sold so that the company can maximize its income. What is the maximum income?

Answered by Penny Nom.
Practical applications: parabolas and Pythagoras 2004-10-24
From Connie:
Provide two examples of real life objects that incorporate parabolic shapes. Explain the reason why the parabolic shape was used in each object.

I need at least one practical application of the Pythagorean Theorem.

Answered by Penny Nom.
Solve for a and t 2004-10-23
From Justin:
How do I solve for "a" and "t" in the equations:

1000t= -4000 + 2000t + (1/2)at^2
1000=2000 + at

Answered by Penny Nom.
An aircraft is flying directly from airport A to airport B 2004-10-22
From A student:
In this question, i is a unit vector due east and j is a unit vector due north. An aircraft is flying directly from airport A to airport B, which is 2000 km from A. The velocity in still air of the aircraft is (150i+50j)km/h and the ground speed is (200i-30j)km/h. Calculate

(i) the time of flight, to the nearest min
(ii) the direction of the wind.

Answered by Penny Nom.
sin(3A) 2004-10-20
From A student:
Express sin3A in terms of sinA and cosA.
Answered by Penny Nom.
A square footage problem 2004-10-19
From Jerome:
I have a peice of property that is almost a triangle but not exactly about 280 frontage with the side lines at 228 and 237

I was told I had about one acre plus a little, a developer says I have 33,000 sq ft. and I would like to know what the real result is

Answered by Penny Nom.
A rate of change problem 2004-10-15
From Frank:
Find the rate of change of the distance between the origin and a moving point on the graph of y = x(squared) + 1 if dx/dt = 2 centimeters per second.
Answered by Penny Nom.
Pythagoras in everyday life 2004-10-13
From Tiffany:
I was wondering if you have any real-life uses of the pythagorean theorem that you use in your everyday life.
Answered by Penny Nom.
A five digit number 2004-10-06
From A parent:
What is the 5 digit number in which the first 2 digits are the square of the 3rd digit and the last 2 digits are the square of the digit that is one less than the third digit? The sum of all the digits is 22.
Answered by Penny Nom.
Accelerating to the speed of light 2004-09-30
From Lars:
How long time would it take to accelerate up to the speed of light with an acceleration speed similar to 0-100 seconds in 5 seconds?
Answered by Penny Nom.
A theorem involving a trapezoid 2004-09-29
From Abraham:
Given:Trapezoid ROSE with diagonals RS and EO intersecting at point M
Prove:Diagonals RS and EO do not bisect each other.

Answered by Harley Weston.
The area of a lot 2004-09-29
From Deb:
I am trying to figure out how many square feet are in a piece of property. Start at Point A-then go 140 feet north-then 100 feet due east-then 300 feet at an angle southeast so that connecting to point A would be a straight line (right angle to first line north.)
Answered by Penny Nom.
The volume of a pile of mulch 2004-09-22
From Sam:
Is there a formula to determine the cubic feet of something in a pile. IE- I need to determine the cubic feet of a pile of mulch. The pile comes to a peak, so the length and width decrease as the pile increases.
Answered by Penny Nom.
f(x) = x-4 if x<2 2004-09-21
From Dani:
Hi, my name is Dani. I'm in tenth grade Algebra 2, and I'm learning about a lot of different kinds of functions, and I can't quite seem to understand how every part of the piecewise function works. I understand most of it, I think, but I've been looking at an example in my math book, and I'm not sure how they got the answer they did. The part I don't understand reads: "Graph f(x) = x-4 if x<2" and "Identify the domain and range." The picture in the book shows an open circle at (2,-2), with an arrow going down and to the left with the slope being one. How did they get the open circle to be at (2,-2), and why does the arrow point where it does?
Answered by Penny Nom.
The method of substitution 2004-09-20
From Kayla:
y=x-1
y=2x-3

Answered by Harley Weston.
Volume 2004-09-16
From Jonathan:
I have an area of 22ft x 43 ft, 6 inches high
Sq. ft..946
sq. yds...105.1

What is the cubic feet and cubic yards for this dimension.

Answered by Penny Nom.
The game of 24 2004-09-16
From Angelica:
how can you make 24 out of 7,7,6,5?
Answered by Claude Tardif.
Finding the height of a triangle 2004-09-14
From A student:
I have to calculate the height of a triangle. The base is 6 (units)say ab One of the top sides is 4 (units)say ca The other side is 2 (units) say cb I don't know the angles.

How to find the height?

Answered by Chris Fisher and Penny Nom.
The game of 24 2004-09-14
From Alan:
I was playing that math 24 game and i cant get 21,14,2,2 to equal 24
Answered by Claude Tardif.
Crossword math puzzle 2004-09-14
From Joyce and Tiffany:
The puzzle name was real number vocabulary and the question is (a number that divides evenly into another what is it?). (Space in between tic-marks on a number line).
Answered by Harley Weston.
differentiate Y=X^X^X 2004-09-13
From Kunle:
differentiate Y=X^X^X
Answered by Penny Nom.
Safe cracker 2004-09-09
From Adam:

Franklin's friend had taken an item from him, and put it in his family's safe, when franklin went to retrieve it he came to a combination lock on the safe, with the dial numbers going from 0 to 59. Unfortunately, he wasn't sure whether there were three or four numbers in the combination, or even which direction to turn the wheel

If it takes him 15 seconds to try a single combination, how many days will it take him to to try every possible combination? Please round to the nearest day.
Answered by Penny Nom.

The 24 game 2004-09-09
From Erik:
Ive spent around 40 hours on this one problem. I need help. 3,3,7,7
Answered by Claide Tardif.
An inverse 2004-09-09
From Hillary:
I cannot figure out the inverse of this function.
f(x)= 1/2x -1

Answered by Penny Nom.
Half way between 2004-09-08
From Ben:
Find the number halfway between the number shown

751,843

Answered by Penny Nom.
Constructing a cone 2004-09-07
From Steve:
I am trying to build crayfish traps; one of the components is a cone shaped entry section.

The cone I want to make would be 12" in diameter at the base and 12" in height, from base to peak.

I need a formula to calculate the dimensions and a method of transferring the shape onto a flat piece of material.
Answered by Harley Weston.

(2x+5y-6) + (3x-4y+12) 2004-09-04
From GG:
I am a grade 9 student and i do not understand a question. Plz help! OK here it is........ (2x+5y-6) + (3x-4y+12)
Answered by Penny Nom.
Order of operations 2004-09-04
From Leanne:
6 + 3 - 2 x 3 =
Answered by Claude Tardif.
The distance formula 2004-09-04
From Joe:
Why do we use square roots and squares in the distance formula Wholdnt they cancel each other out?
Answered by Claude Tardif.
GCD (a + b , a^2 - ab + b^2) 2004-08-27
From Carol:
Let a and b integer and relatively prime. Proof that:
GCD (a + b , a^2 - ab + b^2) = 1 or 3

Answered by Penny Nom.
Combined operations 2004-08-26
From Louise:
Question 2 1/4 x 1/8 / 1 3/4 x 12 4/9 x 3 =
Answered by Penny Nom.
50/5x - y 2004-08-22
From Rick:
Niece has a question that was marked wrong but we are unable to determine how the teacher calculated and arrived at the answer? The problem was as follows:
50 / 5x - y =

x=5
y=1

Answered by Harley Weston and Leeanne Boehm.
The area of a lot 2004-08-22
From Cassie:
I'm in the process of purchasing some land and I'm trying to figure out the square footage of the property. Unfortunately, it's be a while since I had to remember this type of formula. The property is 140 ft on left side, across the back it is 220.61 ft, the right side is 167.52 ft, the frontage is 105 ft.
Answered by Penny Nom.
Golfing with Norm 2004-08-19
From Norm:
I am organizing a golf tournament with 6 American and 6 Canadian players. We play 3 rounds of golf. Is there any way to arrange that everyone plays without playing with the same person twice. Each foursome must consist of 2 Americans and 2 Canadians with one of each in a cart. Your help would be much appreciated.
Answered by Claude Tardif.
Golfing with Sally 2004-08-19
From Sally:
We are a group of 8 golfers (2 foursomes) and want to golf 3 games together. How can we arrange it so that we all get to golf with each other at least once?

At another venue, we have 18 golfers (3 foursomes, 2 threesomes) and want to golf 6 games together. Same question, how can we arrange it so that we all get to golf with each other at least once?

Answered by Claude Tardif.
2.79 acres 2004-08-11
From Helen:
What would the square footage be of 2.79 acres?
Answered by Penny Nom.
Limits and composite functions 2004-08-07
From Sue:
I have two questions, one about a limit and the other about a composite function. If you could help me, I'd really appreciate it.

1. Find the limit:

lim[x->0] (x*csc(x))

I converted csc(x) to cos(x)/sin(x), but I didn't know what to do after that.

2. f(g(x)) = ln(x^2 + 4), f(x) = ln(x^2) and g(x) > 0 for all real x, find g(x):

I'm having trouble with this one because x^2 + 4 isn't a perfect square.

--Sue

Answered by Penny Nom.
The integrating factor method 2004-08-05
From A student:
Whilst using the integrating factor method, I am required to integrate a function multipled by another function.

say f(t) = exp(kt) and some other function g(t); where exp = exponential and k is some constant.

Integral f(t)*g(t) dt or Integral exp(kt)*g(t) dt

What would the result of this integral be? I have never met an integral like this before. Would it simply be exp(kt)*g(t)/k? More specifically, the problem and my attempted answer is in PDF format:

In my attempted solution, I am unsure about the last two lines I have written out, as it relates to integrating a function multipled by another function.

Answered by Harley Weston.
Differentiation 2004-08-04
From A parent:
I am a parent trying to understand higher level of maths and would be very grateful if you could help with differentiating the following functions, identifying general rules of calculus:

a) f(x)=e^2^xIn(cos(8x))

b) f(x)=secx/SQRTx^4+1

Answered by Penny Nom.
When is the limit of f(x) undefined? 2004-08-03
From Nicolasa:
When is the limit of f(x) undefined?
Answered by Penny Nom.
an integer with three factors 2004-08-03
From A student:
What is the probability that a randomly chosen 3 digit number has exactly 3 factors
Answered by Penny Nom.
sin^2x-5sinx-3=0 2004-08-02
From Kerri:
hey my name is kerri i'm a college students and i was wondering if you could help in with this problems

sin^2x-5sinx-3=0

Answered by Penny Nom.
The law of sines 2004-08-01
From Joy:
How do you solve this? Do you solve this triangle using the law of sines of the law of cosines? (ASA)

A=120DEG. B=40DEG c=35 cm I keep getting different answers.

Answered by Penny Nom.
Rate of change problems 2004-08-01
From Jim:
I just want to check a couple average rate of change problems because i just guessed on how to do them. Can you tell me how to do them?

the question says if f(x) = sqrt(x + 3), find f( x + rx). I got sqrt( x + rx + 3)

the other two are : f(x)= 3x-1 (f(x) - f(1)) / (x-1) ... I GOT 3 &

f(x)= x^3 - x (f(x) - f(1)) / (x-1) ... I GOT x^2 + x

Answered by Penny Nom.
Factoring 2004-07-28
From Nicole:
Can 4x2 + 19x + 12 be factored? i have tried but i cannot figure it out. Is there a way to find out if it cannot be factored before i try to work out the problem?
Answered by Penny Nom.
The intersection of two graphs 2004-07-28
From JJ:
Is there a way to find the intersections of these graphs algebraically?

x^2 + y = 4 & 2x - y = 1

I got (1.45, 1.9) and (-3.45, -7.9) with a graphing calculator.

AND THESE...

y = 3.29x & y = 5.5(x^0.5)+ 10000

I got x at 3133 with a graphing calculator.

Answered by Penny Nom.
x^2 = x + 2 2004-07-26
From A parent:
x2 = x+2

My daughter came home with this equation today and although I know the answer is x=2, i have no way of proving it by showing any working out.

Answered by Penny Nom.
Row echelon form 2004-07-24
From Michael:
My name is Michael and I'm in the 11th grade. I have a Math question that I can't solve. The problem is system of equations that I need to do in augmented matrix form, find the row echelon form, and solve it by using back substitution.

2x + 3y + 7z =13
3x + 2y - 5z = -22
5x+ 7y - 3z = -28

Answered by Penny Nom.
The inverse of a quintic function 2004-07-22
From A student:
I'm really having trouble finding out the inverse of this quintic equation. F(X)= X5+3X3+1.I know that this quintic has an inverse because it is one to one. But I can't find a method to solve this. Finding the inverse of a quadratic or a cubic equation is a lot easier, but with this quintc I am really lost. I even know how the graph of this equation and its inverse looks like, but I'm not sure if Iam getting the right equation. Please help me out.
Answered by Penny Nom.
Factoring 2004-07-19
From A student:
Factor completely:
3x3 - 24y3
54x6 + 16y3
16xy - 4x - 4y - 1
0.09x2 - 0.16y2

Answered by Penny Nom.
A proof in geometry 2004-07-16
From An:
Im taking a geometry course for the summer , and we just started to learn about proofs for about one week. Today in class, we started to do this one proof but didnt finish it because class ended. the problem is as follows.
Answered by Penny Nom.
Solving some quadratic equations 2004-07-15
From A student:
Could you please solve the following for x

x2+6=5x
x2+16=8x
2x+21=3x2

Answered by Penny Nom.
Factoring integers 2004-07-02
From A student:
After looking at all the info I could get about NFS, I still have some questions that are unsolved:

First of all: If someone found an algorithm that has a worst case running time of N*Log(N) to factor an integer n into his divisors, would it be quicker or slower then the number field sieve algorithm?

secondly, what exactly is the time complexity of the Number Field Sieve algorithm, if I would factor an integer n?

Answered by Claude Tardif.
Simplify 2004-06-27
From Tiff:
Simplify:

-3x (x2 + 2)

(x + 2) (x + 3)

Answered by Penny Nom.
The surface area of a pyramid 2004-06-19
From Jason:
I was wondering how to find the area of a pyramid when you donít know the height of the face but the height of the Pyramids highest point?
Answered by Penny Nom.
An equation with rational terms 2004-06-17
From Louise:
simplify the expression (a2+n2)/2n = a2 + (a2-n2)/2n
Answered by Penny Nom.
Area of a lot 2004-06-17
From Mike:
Could you please figure out the square footage and acreage of my lot? One side is 228 feet, another is 150 feet, another is 80 feet, and the last side is 164 feet. My house is in the middle and it is 62 feet by 37 feet. Is that subtracted from the total? I would like to know this when I have to buy fertilizer.
Answered by Penny Nom.
The Number of the Beast 2004-06-13
From A Heinlein reader:
Most people who have an interest in mathmatics are familiar with Robert Heinlein's novel "The Number of the Beast", where 666, or ((6 to the 6th power) to the 6th power) equals 1.0315 times 10 to the 28th power, which in the novel is the number of parallel universes in the cosmos. My question is what would the number of parallel universes be if you grouped 666 the other way: (6 to the power of (6 to 6th power)). I have tried this on my calculator, but it won"t register that high.
Answered by Penny Nom.
The game of 24 2004-06-04
From Samantha:
i have the game 24 (the one where you have to use multiplication, division, addition, and subtraction to get to the answer 24) I was playing when i came across the 4 numbers 20, 17, 7, 11 and i cant seem to get 24.
Answered by Claude Tardif.
112! 2004-05-28
From Beatriz:
1) A bus driver collects identical sets of 5 coins from each passenger. If the totoal colledted was $21.83, how many pennies did the driver get??

2) How many terminal zero in the base 10 expression of 112! (factional) N! means N(N-1)(N-2) .....(2) (1).

Answered by Penny Nom.
Different infinities 2004-05-27
From Plober:
How can I explain to a friend (in a bar, using as a pen and a paper napkin) that the integer's infinity is 'smaller' than the irrationals's one? The demo I tried was that you couldn't match the integers with the real numbers between 0 and 1 (that 0.xxxxx replacing the Nth number from a different one... that demo), but she used my argument >:| saying that you can add one to the integer's infinite, and the number I was creating was only one more...

I can't think of any other way, and I KNOW the real's cardinality is greater than the integer's one

Answered by Claude Tardif and Penny Nom.
The horizontal fraction bar 2004-05-27
From Kashia:
What is the name of the bar that separates a numerator and that denominator?
Answered by Penny Nom.
Finding bearings 2004-05-24
From James:
This question is about finding bearings. A boat race starts from point A, goes North to Point B, a distance of 1000 meters. The course is triangular. The bearing from point B to point C is South 70degrees West. The distance from Point B to point C is 1500 meters. Find the course bearing from C to A.
Answered by Penny Nom.
Some trig expressions 2004-05-23
From A student:
Prove:

sin A + sin B = 2sin(A+B/2)cos(A-B/2)

cos A - cos B = -2sin(A+B/2)sin(A-B/2)

cos A + cos B = 2cos(A+B/2)cos(A-B/2)

sin A - sin B = 2cos(A+B/2)sin(A+B/2)

Answered by Penny Nom.
33 3/4 doubled 2004-05-22
From A tailor:
need to know how to add 33 3/4 doubled. How do I come up with the solution? I need to know for measuring garments. Usually I measure on the half meaning on the front of the garment and then double it with the measuring tape but I always get stuck when I need to measure 3/4 such as 66 3/4 doubled and etc. Can you help me figure this out easily.
Answered by Penny Nom.
Frieze patterns 2004-05-19
From Christina:
How can I prove that there are only seven unique geometric frieze patterns?
Answered by Penny Nom.
Angles of elevation and depression 2004-05-18
From Anjum:
what is the difference between an angle of elevation and angle of depression?
Answered by Penny Nom.
Mrs. Smith's class 2004-05-12
From Amanda:
Mrs. Smith's class has 35 student. Today there are 2/3 of them absent. How many are absent? present?
Answered by Penny Nom.
Subtracting fractions 2004-05-11
From Filipe:

Question:
_5_ - __7__
6ab 8a


Answered by Penny Nom.
A pyramid 2004-05-11
From Quiana:
I am a student and I have the following questions I need to answer for homework:
How many sides, vertices, and edges does a pyramid have?

Answered by Penny Nom.
Three dice 2004-05-10
From A student:
If one has 3, 6 sided dice what is the probability of the numbers that are rolled to total 4 through 10 inclusively?

Subsequent to this, what is the probability to do this consecutively...say 3 times?

Answered by Peny Nom.
Pi 2004-05-10
From Kim:
I'm a 7th grader at Lakeside Middle School. And I was wondering if you can answer a question for me for my math project. I'm doing a project on pi, and my teacher said to ask an expert like you to ask about how you would use the subject, pi, in real life usage. How would you use pi in real life usage?
Answered by Penny Nom.
3x squared - 27 / x + 3 2004-05-04
From Stef:
3x squared - 27 / x + 3
Answered by Penny Nom.
Factoring 2004-04-27
From Bipin:
FACTORISE:

a to the power of 6 + b to the power of 6

Answered by Penny Nom.
An oil-drilling platform 2004-04-27
From Alie:
An oil-drilling platform is located in the Gulf of Mexico 3.25 miles from the nearest point on shore. From a point B on the shore due east of A the bearing of the platform is S51.2W. How far is it from B to the platform?
Answered by Penny Nom.
The problem of Apollonius 2004-04-25
From Mitja:
There are given 2 circles lying one out of another and one point out of both circles. How to construct a circle passing through a given point and internally tangent to one and externally tangent to the other cirlce?
Answered by Chris Fisher.
A Fibonacci triangle 2004-04-25
From Marcelle:
Is it possible to construct a triangle with sides that are three consecutive Fibonacci numbers?
Answered by Walter Whiteley.
square root 125 x 16 2004-04-23
From Mike:
how do you simplify a square root of "- square root 125 x16" when x is a variable... i don't really get how to do this....
Answered by Penny Nom.
Surface area of a cylinder 2004-04-17
From Kathryn:
How do you find the radius of a cylinder when you are given only the surface area and the height of the cylinder?
Answered by Penny Nom.
The line through (4,2) and (4,-8) 2004-04-17
From Kim:
Write an equation for the line.
Given points (4, -8) and (4,2) are on the line.
I believe m=undefined.

How do I solve for b when m is undefined, and then plug in the values for m, b, and y?

Answered by Penny Nom.
Some factoring problems 2004-04-15
From KJ:

Factor these:
x3+125 -----> (x+5)3
8x3-27 -----> (?)
x2+36 -----> (x+6)2
x4-5x2+4 --> (?)


Answered by Penny Nom.
A probability density function 2004-04-14
From A student:
The label on a bottle of liquid detergent shows contents to be 12 ounces per bottle. The production operation fills the bottle uniformly according to the following probability density function:

f(x) = 8 for 11.975 <= x <= 12.10
and
f(x) = 0 elsewhere

a. What is the probability that a bottle will be filled with 12.02 or more ounces? b. What is the probability that a bottle will be filled between 12 and 12.05 ounces? c. Quality control accepts production that is within .002 ounces of number of ounces shown on the container label. What is the probability that a bottle of this liquid detergent will fail to meet the quality control standard?

Answered by Penny Nom.
Percent difference 2004-04-10
From A parent:
For a school science project, my son Alex is taking measurements of plant growth at regular intervals. As part of the data, he must provide the maximum percent difference observed in the categories his team has identified.

So, for example he has six plants with four measurements each. (He has more, but I'll keep it simple) For the first plant he measured 2mm, 2.4mm, 2.9mm, and 3.2mm. For the 2nd, 3rd, and 4th plants, he has similar numbers. Is there a way to calculate the maximum percent difference between any two plants in his measurements during the project? Doing it for each combination would be tedious.

Answered by Penny Nom.
Differences in percentages 2004-04-06
From Jose:

Question:
You have two groups:

Group A has 10 people and 2 are Chinese
Group B has 12 people and 3 are Chinese

In terms of percentages what is the difference in Chinese between the two groups? In other words which group has the biggest percentage of Chinese and why?


Answered by Penny Nom.
x^2/3 - 7 x^1/3 + 12 = 0 2004-04-05
From Jackie:
I am having trouble solving this question for x:

1.) x^2/3 - 7 x^1/3 + 12 = 0

Answered by Penny Nom.
(3x50)+20/5=? 2004-04-03
From A student:
what is the answer to:

(3x50)+20/5=?

Answered by Penny Nom.
Some calculus problems 2004-04-01
From Weisu:

I have questions about three word problems and one
regular problem, all dealing with derivatives.

  1. Find all points on xy=exy where the tangent line
    is horizontal.
  2. The width x of a rectangle is decreasing at 3 cm/s,
    and its length y is increasing at 5 cm/s. At what rate
    is its area A changing when x=10 and y=15?
  3. A car and a truck leave the same intersection, the
    truck heading north at 60 mph and the car heading west
    at 55 mph. At what rate is the distance between the
    car and the truck changing when the car and the truck
    are 30 miles and 40 miles from the intersection,
    respectively?
  4. The production P of a company satisfies the
    equation P=x2 + 0.1xy + y2, where x and y are
    the inputs. At a certain period x=10 units and y=8
    units. Estimate the change in y that should be made to
    set up a decrease of 0.5 in the input x so that the
    production remains the same.

If you could just give me some hints on these
questions, I'd really appreciate it. Thanks!


Answered by Penny Nom.
Algebraic simplification 2004-03-31
From Chris:
Simplifying Exponents

Question

b^5d^2/b^3d^8

Answered by Penny Nom.
A derivative 2004-03-31
From A student:
What is the nth derivative of f(x) =(2x)/(1-(x2))?
Answered by Harley Weston.
n! > n^2 2004-03-30
From Jose:
How can you prove by mathematical induction that:

n! > n2.

Answered by Penny Nom.
A stained glass window 2004-03-29
From Kay:
I'm doing a stain glass project and it's on a 4 foot across octagonal window...and I'm trying to set up the pattern and I don't know how long the sides are!
Answered by Penny Nom.
Percentage difference 2004-03-28
From Someone:
what is the percentage difference between 157 to 251
Answered by Penny Nom.
BEDMAS 2004-03-20
From Brad:
I am in grade 7. My teacher tells me brackets always first, well i know that but, 5 (-4) x 2

Does the (-4) count as a bracket or is it just telling you not to minus 4 from 5 but to multiply 5 x -4 ? Am i correct?

Answered by Penny Nom.
A partial derivative 2004-03-19
From Penny Nom:
Is it possible to differentiate the following equation, if so could you please explain.

S=SQRT(T-(5/X^2))

I would like the derivative of S with respect to X.

Answered by Harley Weston.
A fraction of a fraction as a percent 2004-03-18
From Mary:

I would like to know if you could help me with this problem? Could you take me though the steps
to this math problems.

 3 =(?)% x 3
---       ---
16         4
	  

the question states: Finding what is the percentage of a fractional number and another fractional number.


Answered by Penny Nom.
Factoring x^2 + 5x + 4 2004-03-14
From Keenan:
how do I factor x^+5X+4?
Answered by Penny Nom.
Sum of the angles in a pentagon 2004-03-11
From Ashish:
What is the sum of all the measures of the angles of a Pentagon
Answered by Penny Nom.
The factor theorem 2004-03-08
From LisaMarie:

Hi, I have a career link question from gr.12 advanced functions and intro. calculus. The equations is:

R(t) = 5(t4 - 4t3 + 19/4 t2 - 3/2 t)

The question is to apply the factor theorem and quadratic formula to determine the times it crosses the t axis. The only method i know of for using the factor theorem is when the last number does not have a variable beside it so i'm not sure how you would even use the factor theorem for this? We haven't dealt with any that have 0 as the last number with no variable so i'm a little confused! please help!

Thank you

Answered by Penny Nom.

A metric prefix table 2004-03-07
From April:
Can you tell me what the scale is for nano, micro, mega, kilo, etc.... I know that mega is 10 to the sixth power but I can't remember the other ones.
Answered by Penny Nom.
The radius of a circle 2004-03-06
From A student:
what is the radius of a circle with the circumference of 12 inches?
Answered by Penny Nom.
A fifteen sided polygon 2004-03-05
From Joann:
WHAT IS THE NAME OF A FIFTEEN SIDED SHAPE?
Answered by Penny Nom.
Proof by induction 2004-03-02
From Chris:
I need some help of how to solve the problem

"use the principle of mathematical induction to prove that the following are true for all positive integers"

cos(n x pi + X) = (-1)^n cosX

any help would be appreciated

Answered by Penny Nom.
Volume of a fuel tank 2004-03-01
From Mike:
I have a fuel ank for my big truck that has a radius of 24inches and a length of 65 inches. I am trying to compute the volume of fuel in the tank. I tried pie x the radius squared times the length but no usable results. Can you help me please?
Answered by Penny Nom.
Rearranging an expression 2004-02-24
From Corey:

use the following formula to answer the question F=9c divided by 5 plus 32

rearange the formula to solve for c
how many degrees fahrenheit would it be it it was:
0 degrees celcius
-10 degrees celcius
-46 degrees celcius


Answered by Penny Nom.
Profit, markup and discount 2004-02-22
From Kick:
Using the calculation on your website, if I sell something for $885 and my cost is $296, I make $589 profit. How do I calculate the profit percent I made on that transaction?

Profit as a percentage of what it cost me?
Profit as a percentage of what I sold it for?

Answered by Penny Nom.
Selecting balls from a golf bag 2004-02-21
From A student:
There are 5 white and 5 yellow balls in a golf bag. Two are selected randomly, and simultaneously. What is the probability that the first one or the second one will be white?
Answered by Penny Nom.
Gallons in a cylinder 2004-02-20
From Jim:
Please advise how to use the formula I saw for determining the number of gallons in a cylinder? This is how it was written V = pi r2 h . does this mean pie, or 3.14 X radius squared, X the height? If so please use a standard 55 gal barrel (1' R & 3' H) for example?
Answered by Penny Nom.
The derivative of x to the x 2004-02-14
From Cher:
what about the derivative of x to the power x?
Answered by Penny Nom.
The substitution method 2004-02-14
From Keisha:
I need to solve this problem by using the substitution method.
3x+y=1
x=2y+5
Please help me understand the method to solve.

Answered by Penny Nom.
Fractions 2004-02-13
From A parent:
12/36=?/18=4/?=?/9
Answered by Penny Nom.
Common fractions to decimals 2004-02-12
From A parent:
I'm trying to help my son work with turning fractions into decimals and this seems foreigh to me after 20 years of not being in school. Can you give me a few illustrations and/or examples of how to do that??
Answered by Penny Nom.
Profit 2004-02-12
From Mohamed:
if i bought some thing with 100$ and the saller told me that his profit was 30% how can i know the actual price of the good
Answered by Penny Nom.
Sums and differences 2004-02-10
From David:
The sum of two whole numbers is 63. The difference between the numbers is 10. Find all the possible pairs.
Answered by Claude Tardif and Penny Nom.
A/30 + B/105 = (7A + 2B)/x 2004-02-05
From Jim:
If A/30 + B/105 = (7A + 2B)/x and A, B, and x are integers greater than 1, what must x equal?
Answered by Penny Nom.
Order of operations 2004-01-28
From John:

I am trying to find out any information concerning the development of the order of operations. for example, when (why/how) did it become the case that 2 + 3 x 5 =17, rather than 25.

any insight is appreciated. Thank you.


Answered by Penny Nom.
Lead weights 2004-01-26
From Jeff:
I am making a mold for my decoy anchors. I need 10 - 12 lead ounce weights. How would I determine the liquid measurement for that weight? For example if I melted down 10 ounces of lead how many cups would that be or quarter cups etc?

I would like the formula if I could and good instructions on how to use the formula.

Answered by Penny Nom.
Polynomials 2004-01-25
From Bruce:

A polynomial is defined as
Polynomial functions are functions that have this form:
f(x) = anxn + an-1xn-1 + ... + a1x + a0
The value of n must be an nonnegative integer. That is, it must be whole number; it is equal to zero or a positive integer.
The coefficients, as they are called, are an, an-1, ..., a1, a0. These are real numbers.

Questions:

  1. why must n be positive?
  2. what are some historical facts about the evolution of the definition?

Answered by Harley Weston.
Bundles of asphalt shingles 2004-01-24
From Larry:
According to my study material 4:12 multiplying factor for shingles is 1.054. The question reads as follows: A building with a floor plan of 3350 sq. ft. and a roof slope of 4:12 will require _______ bundles of standard asphalt shingles.
Answered by Harley Weston.
x^x^x^x^... 2004-01-23
From Ryan:
you have a number say x and it is to the power of x which is to the power of x and so on infinite times like x^x^x^x^x^x^x^... i have to figure out what x is so that the answer is always 2
Answered by Penny Nom.
A counterfeit coin 2004-01-23
From Kim:
You are considering buying 45 silver coins that look alike, but you have been told that one of the coins is a lightweight counterfeit. Find the least number of weighings on a balance scale that you can use to be certain you have found the counterfeit coin?
Answered by Penny Nom.
BEDMAS 2004-01-21
From Jessica and her mom:

my mom and I were wondering 2 things.

1. what is the reason for having bedmas.
2. 5+5-4+[6x3-(6+1-3x2) -5+9]


Answered by Penny Nom.
A problem with sets 2004-01-20
From Jason:

Given that the universal set S is the set of all sports fans, and

F={x|x is a football fan}
B={x|x is a basketball fan}
H={x|x is a hockey fan}
a)Describe (F^B)' (f intersect b)' in words
b)Draw a Venn Diagram and shade the region that represents the set of football fans or both basketball and hockey fans.


Answered by Penny Nom.
Cubic yards in a concrete wall 2004-01-18
From Danny:
I am studing for my contractors exam. I am haveing truble with a few questions. I have the answer to the problems but I dont know how to get them my self.
please help

How many cu. Yds. Of concrete are in a wall that is 150' long and 8'4" high an d 8' thick

Answered by Penny Nom.
3/8 as a decimal 2004-01-13
From A student:
My Math Midterm is tommorow and I am stumped... how do you turn 1 and 3/8 into a decimal... I know the answer is 1.375 but I just copied down what the teacher wrote on the board.
Oh, and what are the three cases of percents?

Answered by Penny Nom.
What is a Fact Family? 2004-01-13
From Keisha:
I am helping my child with his homework. The worksheet we are trying to do ask the child to choose a number from inside of a circle and then one from inside of a triangle and use the number to build a fact family. The title of the worksheet is "Building a Fact Family House". I noticed that Silver Burdett Ginn,Inc. was the publisher of the book the worksheet came from so I was on the website trying to find an answer to my question. My question is "What is a Fact Family?"
Answered by Judi McDonald.
Twelve acres 2004-01-13
From Heather:
I am looking at buying a property that includes 12 Acres of land and I am trying to visualize what this size is equivalent to. I have searched your site and figured out how many square feet it is but I was hoping that you could apply 12 Acres to something tangible, like a football field!
Answered by Penny Nom.
The square footage of an acre 2004-01-12
From Paul:
What is the square footage of an acre of land assuming all four sides are equal in length.
Answered by Penny Nom.
Cubic furlongs 2004-01-11
From A student:
1 mile=8 furlongs, how many cubic furlongs in a cubic mile?
Answered by Penny Nom.
Reflex angles 2004-01-09
From Sonya:
My daughter is searching for examples of reflex angles. We already have the hands of a clock but still need another example. Can you help us.
Answered by Chris Fisher.
Inequalities 2004-01-08
From Michael:
How do you use inequalities in your job and in your everyday life?
Answered by Penny Nom.
Dividing zero by infinity 2004-01-08
From Jason:
What do you get when dividing zero by infinity? Our Calculus teacher was pretty sure that the expression was indeterminate from. However, if this is so...Why? Zero divded by any number (except zero) is zero, true. Any number (except infinite) over infinite is zero. So, why isn't Zero divided by infinite zero. A simpler way if I had 4 potatoes and was to split them among 2 friends, each friend would get 2 potatoes. However, if I had 0 potatoes and split them a infinite number of ways, each person would still have 0. Explain please!
Answered by Penny Nom.
The sides of a circle 2004-01-07
From Helena:
My name is Helena and I am 10 years old. On a resent math exam I was asked the question" How many sides does a circle have?" and I wrote down none. The teacher said the answer was one side.
Answered by Chris Fisher.
Laying decorative bricks 2004-01-07
From Duk:
Loren is laying decorative brick along both edges of the 21-meter walkway up to his house. Each brick is 0.26 meters long. He is placing the bricks end to end. How many bricks does he need to do the job?
Answered by Penny Nom.
Factoring expressions 2004-01-03
From A student:

I'm having trouble factoring expressions that aren't monic. I can do things like x2-9x+8, but problems like 12x2+5x-3 have me stumped.

I also have a question about factoring out common factors. In a problem like x2-18x+81, wouldn't you divide by 9? But what happens to the x2, is it x2/9?
Answered by Penny Nom.

37 with four 4'2 2004-01-03
From Myra:
Using four fours and only four fours to get an answer of 37.
Answered by Claude Tardif.
Making a windmill 2004-01-02
From Matthew:
I am a farmer in Ontario. It has been almost 20 years since high school. I am toying with making a windmill. The output chart for the the old generator I have is shown below. Before I tear it appart I would like to develop a formula from the chart that can predict the output at various speeds.
Answered by Penny Nom.
Four digit combinations 2003-12-31
From A student:
i need all 4 digit combinations using 0-9 please
Answered by Penny Nom.
Fractions 2003-12-31
From Jae:
Replace the question marks with numbers that will make the sentence true.
1.1/3=?/9=?/6
2.?/18=8/12=4/?
3. 3/?=12/?=9/?
4.?/3=?/21=?/7
5. Which problems have more than one possible answer? Why do you think this is so?

Answered by Penny Nom.
A new way to measure randomness 2003-12-31
From Stephanie:

Last year, I did a science project in which I asked, "Which shuffles better, an automatic card shuffler or shuffling by hand?" To measure this I decided the "best" shuffler was the one to become random first. Last year, to measure randomness, I numbered cards 1-52 and had the subjects shuffle them until they broke up the rising sequences or reached 10 shuffles. (Usually 10 shuffles came first...) Anyway, I did the same thing with the automatic card shuffler, and, as hypothesized, the automatic card shuffler randomized the deck first.

This year, I have decided to continue the project. The problem is, I need a new way to measure randomness without the use of fancy computers or something. I have searched the Internet, I have posted my query on websites based on math, and I have searched the local library.

I have found many useful things on the Internet, but none of them can tell me a new way to measure randomness. I cannot do a perfect shuffle, and I am not terribly gifted in the art of using computers. If you have any information (anything will help) or advice, I would be greatly obliged.


Answered by Andrei Volodin.
Ratios and proportions 2003-12-31
From Frank:
I would like to know how you use 'ratios and proportions' in everyday life.
Answered by Penny Nom.
The area of a plot of land 2003-12-19
From David:
I have a plot of land that I cannot find the square footage of. Actually my home sits on a large lot. Local building codes require that a home is seated on no less than 7500 square feet of land and have no less than 50ft of road frontage (the rear of the lot has 65ft of frontage on another road). I need to know if I have enough square footage to build a second home on the rear lot. The lot has five sides. Only one angle is a 90 deg. angle. The others I cannot determine. Of the 4 remaining angles, all 4 angles are obtuse, or greater than 90 deg. The dementions are 89.19' x 130' x 118.52' x 65' x 170.94'. The two legs that make the 90 deg angle are 89.19' and 170.94'. The measurements fall in order using the 90 deg angle as the point of origin and proceeding clockwise, 89.19', 130', 118.52', 65', 170.94'. I have used all the skill I can muster, however, it has been a long time since school and regretably I cannot find an answer.
Answered by Penny Nom.
Multiplier and multiplicand 2003-12-17
From Karen:
The K-5 curriculum states that at a grade 5 level students should "understand and use the terms multiplier, multiplicand" The curriculum defines 'multiplicand' as " a number being multiplied by another number. A factor." There is no definition given for 'multiplier' Question: Define these words- are they just other words for 'factors'. If so, how important is it that students use these terms? It would seem like 'factors' would be the better choice of words to use. If multiplcand refers to eg. the first factor in the equation and the 'multiplier' the seond term, (if in fact it does)- is this not confusing for students when we teach them that multiplication is commutative?
Answered by Penny Nom.
Binomial distribution 2003-12-17
From Lesley:
my daughter is having difficulty with the following formula P(X=x) = ( n over x) px (1-p) n-x
The teacher has given them the formula but not taught them how to apply it or understand it.

Answered by Penny Nom.
Factoring in 2003-12-16
From Priscilla:
How can you use solving by factoring in real life applications?
Answered by Harley Weston.
How far can you see? 2003-12-15
From Judy:

How far apart, assuming no obstacles, can two people stand and still see each other?

i know this deals with the curvature of earth, but i can't figure out the formulas involved.


Answered by Chris Fisher.
Finding angles 2003-12-02
From Jason:
I AM TRYING TO SOLVE A TRIG PROBLEM AND HAVE FORGOT HOW TO DO IT. WHAT I HAVE IS A RIGHT TRIANGLE WITH SIDE A BEING 14 FEET AND SIDE B BEING 3 FEET, USING PYTHAGOREAMS THEOREM SIDE C SHOULD EQUAL 14.318 FEET ON A RIGHT TRIANGLE BUT I AM TRYING TO REMEMBER HOW TO FIND MY ANGLES OTHER THAN THE ONE THAT IS 90 DEGREES.
Answered by Penny Nom.
The area of a triangle 2003-12-01
From A student:
Find the Area of Triangle ABC
A(-3,2)
B(4,0)
C(0,8)

Answered by Penny Nom.
Room and board fees 2003-11-29
From Jennifer:
A certain university wishes to determine room and board fees for the next academic year. For the current year, the fee is $3600 per student and 1800 students are living in the residence halls. Past data suggests that for every $300 increase in the semester fee, 200 fewer students will choose to live in the dorms. There are also costs to the university associated with the residence halls. The fixed costs total $2,000,000 per semester. the variable costs are currently $1000 per student but will fall $100 per student for each decrease of 100 students. Your task is to help the university determine the optimal fee. Assume linear relationships between the number of students and the fee and between the number of students and the cost per student
Answered by Penny Nom.
A fish tank 2003-11-26
From Bill:
How do I work out the volume and weight of water in a fish tank that is 36in x 14in x 12in?
Answered by Penny Nom.
Mr.Carter is very cautious 2003-11-26
From Bob:
Mr.Carter is very cautious. He decides to invest in only three stocks: one low stock, one high stock, one medium stock. Given that the expected annual yields are 6% for low stock, 7% for medium stock, and 8% for high stock, he wants his investment in medium stock to be half of his total investment in low and high stock. How much should I invest in each type fo stocks to expect a total annual return fo $650 form my investments?
Answered by Harley Weston.
The volume of an irregular tetrahedron 2003-11-24
From Peter:
How do I calculate the volume of an irregular shaped tetrahedron where:-
side a = 1.4 m
side b = 1.4 m
side c = 1.2 m
and height = 0.75 m at the junction of sides a and b.

Answered by Penny Nom.
Markup, profit and margin 2003-11-24
From Ali:
I have question regarding Margins, Markups and Profit Purchase price = $ 10.00
Margin or Markup ?? = 40 %
Therefore Selling Price = 10/0.6 = $ 16.67
Profit ? = (16.67-10)/10 *100 = 66.67%
or is the profit = (16.67-10)/16.67*100 = 40 %
Can you please clarify What is Margin, Markup and Profit

Answered by Penny Nom.
Difference of squares 2003-11-24
From Susie:

Factor assuming that n is a positive #

Problem: (I will give it to you in words beacuse I don't know how to do exponents on the computer.) Forty-five r to the 2n power minus five s to the 4n power. I was hoping you could walk me through it not just give me the answer.


Answered by Penny Nom.
An octagon shaped bed frame 2003-11-23
From Trish:
My son and I are making an octagon shaped bed frame. We are going nuts trying to figure out what angle to cut the boards to make an outline of an octagon. It seems that the 8 inside angles of the 8 "corners" are 120*, but what is the angle that the 2x6 wood should be cut so that they will angle together to form the outline of the octagon?
Answered by Penny Nom.
Laws of sines and cosines 2003-11-23
From A parent:
On the one side of a stream lines PA= 586.3 feet, PB = 751.6 feet are measures, angle APB being 167 degrees and 36 min. Q is a point on the opposite side of the stream. Angle PAQ=63 degress and 18 min and PBQ=49 degrees and 24 min. Find PQ.
Answered by Penny Nom.
Four digit numbers 2003-11-20
From Rob:
Hello, I sure hope you can answer this question. I got into a discussion at work about how many possible combinations there are for a four digit number using the numbers 0 through 9. This was in reference to a discussion about the lottery. I said there was only 10,000 possible combinations. My friend said that there was many times more, but he could not remember how to figure it out. The answer and the formula would be greatly appreciated.
Answered by Penny Nom.
A riddle 2003-11-19
From Sarah:
Ok, our teacher gave us this riddle, and I cannot for the life of me figure it out. He said that there are three problems with the following proof:
Answered by Penny Nom.
Systems of equations 2003-11-19
From Scott:

I hope that u can help me....I am a college student taking a class in Pre Calculus.....I have homework due this Friday and it counts a BIG Percentage on my FINAL grade.....I am getting mixed up and can not figure out a few problems.....Please help me.....

Method Of Subsitution

Problem 1. y- 8x = -5
x(squared) + y(squared) = 25

Problem 2. y = x(squared) - 2x - 6
Y = x(squared) - 4

Answered by Penny Nom.

The 4th difference 2003-11-17
From Jack:
If i had a sequence which was the same at the 4th difference, aka difference to the 4th, like:

1 5 14 30 56 96 (sequence)
4 9 16 26 40
5 7 10 14
2 3 4
1 1

then what would be the set formulars to find out a, b, c, d, and e
as i know the equation has to be an4+bn3+cn2+dn+e

Answered by Penny Nom.
Adding fractions 2003-11-16
From Ken:
My name is Ken and I am taking my GED course for my High School and have not been in a class for 35 years. I am doing this for re-training. I am at the part about fractions. Here is an example that I am having trouble with.

1 3/7 + 4 2/3 + 11/21

They have no common denominators. Could you PLEASE help me. If you could send me a step by step explanation it would be greatly appreciated.

Answered by Penny Nom.
Divisibility by 7 2003-11-14
From A student:
how do you test a number to see if it is divisible by 7 or not?
Answered by Penny Nom.
What is a number that has exactly 13 factors? 2003-11-09
From Mary:
What is a number that has exactly 13 factors?
Answered by Penny Nom.
Greatest Common Factor 2003-11-08
From Beth:
How do you use the Greatest Common Factor in addtion and subtraction of fraction?
Answered by Penny Nom.
Chisenbop 2003-11-06
From Jaclyn:
Hi there i was wondering if you would have any information on "counting on your fingers" or " Chisabop".
Answered by Penny Nom.
Cubic yards of concrete 2003-11-02
From John:
The foundation is 20' X 20' and the depth is 6", there are also three footings that are 20' X 12" X 12" each. Can you show me the equation to figure out how many yards of concrete I will need to pour the footings and foundation.
Answered by Harley Weston.
Which one has the most factors? 2003-10-31
From Kristi:
Of all the whole numbers less than or equal to 5000, which one has the most factors?
Answered by Claude Tardif.
1/x + 1/y = 5/12 2003-10-30
From Ben:
1/x +1/y =5/12, what is the sum of x and y?
Answered by Penny Nom.
12 cookies 2003-10-29
From Joel:
there are 12 chocolate, sugar and cinnamon cookies. the probability of choosing a chocolate cookie is 1/3. the probability of choosing a sugar cookie is 1/4 what is the probability of choosing a cinnamon cookie
Answered by Penny Nom.
Solve for x 2003-10-27
From Lonnie:
i need help with solving equations and formulas such as -3x+b=6x, for x. I do not understand how you find x.
Answered by Penny Nom.
Knitting gauge and felting 2003-10-27
From Sara:

I wish to create a pattern to knit an item that will then be felted, thus, shrink as a result. In order to determine the proper gauge to use in creating the item, I produced a gauge swatch, noted the original properties and then the post-felting properties. They are as follow:

Before Felting:

20 stitches(width) x 20 rows(height)yielded:
Gauge: 3.2 stitches(width) and 4 rows(height) = 1"
Dimensions: 6.25"(width) x 5.25"(height)

After Felting:

20 stitches(width) x 20 rows(height)
Dimensions: 4"(width) x 3"(height)

How do I calculate the percentage change in size to accurately determine the number of stitches and rows needed to produce the desired dimensions for the finished (felted) item?


Answered by Penny Nom.
A parabola 2003-10-24
From Delores:
Given the vertex (4, -2) y intercept = -6 find if/where the parabola crosses the x axis?
Answered by Penny Nom.
0.810 and 0.801 2003-10-24
From Ahmed:

could you please tell me what is the greatest decimal fraction formed of the digits 0, 1 , 8?

Is it 0.810 or 0.801, please give me the reason of the answer in details and the references if it possible.


Answered by Penny Nom.
Arithmetic in bases other than 10 2003-10-22
From Kim:
how do you add, subtract, multiply and devide in base 3, base 5, etc?
Answered by Penny Nom.
Domain of a function 2003-10-14
From Karim:
Find the Domain of the Function
H(x) = Log3 (4x + 7)-10
H(x) = log base 3 times 4X plus 7 minus 10
i am having problem solving these kinds of problems.

Answered by Penny Nom.
Pairs of prime numbers 2003-10-13
From Nikolas:
Use pairs of prime numbers to find all the numbers less than 50 that have only two prime factors. Make an organized list.
Answered by Penny Nom.
Standard Deviation 2003-10-07
From Rebecca:

I have a task to complete, which is to calculate the mean and standard deviation of something. I have done this but am then asked to write a short explanation of my findings.

I know what the mean is about, and I thought I knew what the standard deviation meant too - shows the variation from the mean. However, on a task I completed earlier the feedback I got said 'you need to tell us that it is talking about the middle 66% of the data' - that has thrown me, I don't understand that. Can anyone help me get my head round this???


Answered by Penny Nom.
The sketch of a graph 2003-10-07
From A student:
I was wondering how do you figure out if a graph has a horizontal tangent line. One of my homework problem was to sketch the graph of the following function; (4/3)x3-2x2+x. I set f''(x) ( the second derivative) of the function equal to zero and got the inflection point:(1/2,1/6). Also i am having trouble finding the concavity for x>1/2 and x<1/2, i am getting a different answer from the back of the book, the graph i draw looks completely different from the correct answer.
Answered by Penny Nom.
Indeterminate forms 2003-10-06
From A teacher:
Is it possible for me to find any geometrical interpretation without using calculus to explain indeterminate forms?
Answered by Chris Fisher.
The slope of a tangent 2003-10-01
From A student:

find the slope of the tangent to each curve at the given point

f(x)=square root 16-x, where y=5


Answered by Penny Nom.
Relatively prime 2003-09-27
From James:
what is the definition a relative prime numbers?
Answered by Penny Nom.
0/0 2003-09-25
From Thomas:
How is 0/0 ever defined.
Answered by Penny Nom.
The Sieve of Eratosthenes 2003-09-18
From Lynn:
My daughter has been asked to find all the prime numbers by using the Sieve of Eratosthenes. I have no understanding what this means.
Answered by Penny Nom and Claude Tardif.
Applications of logarithms 2003-09-16
From A student:
I have a project due in my algebra two class dealing with logs and where they are used in life, but I am having trouble finding websites that relate. So I would really appreciate it if you couldhelp me.
Answered by Penny Nom.
Notation 2003-09-15
From Kim:

Question:
i have a couple of questions

1. write in standard form
a)4x2x8 b) 5 to the power of 4

.
.
.

Answered by Penny Nom.
Substitution method 2003-09-14
From John:

Hello, my name is john and while I was finishing up my work I stumbled into these problems. a/4-b=-1
a+b=11
and
6a-b=-5
4a-3b=-8

Thanks a lot. I really need some help :)
John


Answered by Penny Nom.
Converting from base 10 to base 5 2003-09-10
From Susy:

My son, who is 9 in grade 5 has been asked to convert base 10 numerals into base 5.

His first question of:

24(10) he has calculated to be 4x5 + 4x1 = 44 (5)

The next question however is the tricky one.Ý We know the answer is supposed to be 100 but we find it difficult to get this in the way he understands it.

25 (10) = _________________ 100 (5)

Can you help us figure out how we reach the answer.


Answered by Claude Tardif and Penny Nom.
Making a square 2003-09-07
From A student:
if I am given any number (say 80 for example), how may I determine the smallest whole number integer which when multipled by it will yield a square number ? In other words if I express this as: 80Ý*ÝnÝ=ÝsquareÝnumber, what is the least value of n which will yield a square.
Answered by Penny Nom.
Order of operations 2003-09-07
From Brian:

It has to do with the 4 rules of operations, Parentheses first before operations outside/evaluate all exponential expressions/all multiplication and divisions/then all additions and subtractions.

Who made these rules and when did they make them,,, I know its somewhat of an unorthadox question but I must know. I would really appreciate it.


Answered by Penny Nom.
7,473,000,000 divided by 52000 2003-09-06
From A student:
I am having trouble with a question. 7,473,000,000 divided by 52000. Our calculater gets 147311.5385. I come up with 143711.53846, and then the number starts to repeat itself
Answered by Penny Nom.
The game of 24 2003-09-03
From A student:
We have a game where four numbers must equal 24 using addition, subtraction, multiplication and/or division.Ý I am stuck on the following four numbers - 12, 24, 9 and 17.
Answered by Penny Nom.
Why percentage? 2003-09-03
From Vicky:
My cousin aske me a question about the use of percentages and I'm not sure how to answer it. He wants to know why when there is a sale in a shop we use percentages and not fractions or decimals.
Answered by Penny Nom.
Profit margin 2003-09-03
From Chet:

My salesman calculates a customers profit margin by dividing what the customer paid by what he sells for, then subtracting that answer from 1. For example a customer sells a product for $15. He bought it for $10. 15/10=.66 Subtract that from 1 and get a profit % of 33%

Here's my question: If I want to figure what a reseller will sell a product for if he wants 40% margin(by the technique above), what formula do I use? Let's say I want to sell a product for 8.50, and I want the reseller to make 40%


Answered by Penny Nom.
Water in a tank 2003-09-02
From A parent:
if a tank is 48 inches long 14 inches wide and 18 inches deep if a yardstick was inserted into the tank and measured 2 inches of water in the tank how many gallons of water would be in the tank.
Answered by Penny Nom.
The quadratic formula 2003-08-31
From Alex:
I am using Houghton Mifflin's Precalculus with Limits book, 2nd edition. However, the first chapter encompasses Algebra review, and I am stuck on a problem. All that's required is to solve the following and verify using a calculator:
3y2+6y+2=0
I have solved the problem using the quadratic formula, but from what I remember, the quadratic formula is used in the case of equations following the AX2+BX+C=0 pattern. As the problem I am attempting uses a y-variable, can I still use the quadratic formula? Since I am not sure what route to take in solving this problem, I am hoping you can assist me.

Answered by Harley Weston.
Newton's binomial theorem 2003-08-30
From William:
According to page 126 of Murtha & Willard's "Statistics and Calculus" (Prentice-Hall, 1973), Newton's binomial theorem can proved inductively. I suppose that was his method, which I would like to see.
Answered by Penny Nom.
Graphing a piecewise function 2003-08-24
From Amber:
How do i begin to graph a piecewise function, absolute function or step function?
Answered by Penny Nom.
Simplify 2003-08-24
From Jessica:
I have been asked to simplify the expression (5a2)(-2b2)(3b)+(7a2)(2b2)+(-3a)(-5a)(4b) and the answer is apparently 44(a2)(b2). For the life of me I can't figure out how they derive this.
Answered by Penny Nom.
A rule with a variable 2003-08-24
From Crystal:
what is a rule with a variable
Answered by Penny Nom.
X.9999... and X+1 2003-08-23
From David:
I have read your answers to the questions on rational numbers, esp. 6.9999... = ? and still have a question: The simple algebraic stunt of converting repeating decimals to rational numbers seems to work for all numbers except X.999999.... where X is any integer. The fact that the method yields the integer X+1 in each case seems to violate the completeness axiom of the real numbers, namely that there is no space on the number line which does not have an number and conversely that every geometric point on the number line is associated with a unique real number. In the case of 3.999... for example, it seems that both the number 4 and the number 3.9999.... occupy the same point on the number line. How is this possible???
Answered by Penny Nom.
A golf tournament 2003-08-23
From Dale:
I am running a golf tournament with twelve golfers. We will be playing four rounds of golf. I don't want to schedule the same two players together more than twice (if possible). How would I make a schedule that each player would play at least once with every other player? I would appreciate any help that you can give me.
Answered by Denis Hanson.
The volume of a fuel tank 2003-08-16
From Phil:
I hope you can help me out with a small problem. My boat has a fuel tank measuring 1680mm long x 630mm wide x 130mm high. Can you tell me how much fuel it will hold in both litres and imperial gallons.
Answered by Penny Nom.
Acres 2003-08-13
From A landowner:
Can you please tell me how many total acres the following measurements would equal? 280 feet by 80 feet
220 feet by 70 feet
200 feet by 75 feet

Answered by Penny Nom.
Yards and cubic feet 2003-08-11
From Sonya:
How many cubic feet are in a yard?
Answered by Penny Nom.
Two precalculus problems 2003-08-04
From Kate:

Please help me verify the identity:
cos2x(sec2x-1)=sin2x

Also I am having trouble withdetermining whether f(x) is odd, even, or neither
f(x)=x3-x


Answered by Penny Nom.
Surface area of a sphere 2003-07-23
From Rob:
I have a shere with a circumference of 12 feet. What is the total surface area of the shere in square feet and how did you find it?
Answered by Penny Nom.
A pyramid with its top cut off 2003-07-21
From David:
What is the name given to a 3D shape that looks like a pyramid with its top cut off?
Answered by Penny Nom.
Slope 2003-07-20
From Brian:

You are placing a pipe 60 feet long at a slope of 1.5%. How would you figure the amount of drop from the inlet of the pipe tot he outlet of the pipe? and what is the answer .

If you are trying to keep a shoulder slope of a road between 3/4'' per foot and 11/2'' per foot . If the shoulder is 10 feet wide, how would you figure how much lower should the outside of the shoulder be than the edge of pavement and what is the answer ?


Answered by Penny Nom.
"cubic feet" of a refrigerator 2003-07-16
From Ron:

Is the volume of a home appliance (e.g. fridge or freezer) calculated in a different way? I am planning to replace both my fridge and my freezer but I'm unable to determine what size replacement to get.

For example, one company descibes their fridge as being 32"(L) x 30"(W) x 66" (H). I converted those measurements to feet (2.67 x 2.5 x 5.5) and I calculate the fridge to be 36.7 cubic feet. However, the company says the fridge is 18 cf.


Answered by Chris Fisher.
Is 3/5 CLOSEST to 0, 1/2 or 1? 2003-07-09
From A student:
I would like to know is 3/5 CLOSEST to 0, 1/2 or 1. And can 3/10 go either way as to be closest to 0 or 1/2.
Answered by Penny Nom.
A geometry problem 2003-07-08
From Chris:
My name is Chris, I work for a custom fabricator company. I am needing a formula for the Height (H) shown in the attached picture. The picture shows dimensions for my current application. If you could please, assign variables to the dimensions.
Answered by Harley Weston.
The length of fabric in a roll 2003-07-03
From Paul:
We receive rolled fabric as part of our process and I would like to know is there a formula that could be used to estimate the overall length of the roll with out rolling the fabric out and measuring its length.
Answered by Harley Weston.
The domain of 1/g(x) - 5 2003-07-03
From Barbara:
If the range of g(x) is ( neg. infinity,4] and the domain of g(x) is ( neg. infinity, infinity), how do I find the domain of 1/g(x) - 5?
Answered by Penny Nom.
Polynomials degree 4 and bigger 2003-06-26
From Stephen:

How do I show kids how to find all the zeros for polynomials degree 4 and bigger.

For examples:
r(x) = x5-11x3-7x2+77 = (x2-11)(x3-7) and s(x) = x4-121 = (x2-11)(x2+11)

Also am I correct when I say that the following are irreducible?

f(x) = x2-11
g(x) = x3-7
h(x) = x4-5


Answered by Harley Weston.
4-3(m+1)=(-38) 2003-06-25
From Jamie:

I have a problem, like most of your mailers, I do remember BEDMAS but maybe I'm missing the finer points!

It's been a while, he goes

4-3(m+1)=(-38)

Answered by Penny Nom.
Flim 2003-06-24
From Dan:
I have a 4th grader and the word (Flims) came up in her summer text,
Answered by Penny Nom.
The sum of two numbers is 5 and their difference is 2003-06-16
From Akhil:
The sum of two numbers is 5 and their difference is 11. What is the product of the two numbers?
Answered by Penny Nom.
Two trig problems 2003-06-10
From Bett:

I have this ongoing trouble with trig and solving triangles with laws of cosines and sines!! For example if it asks to solve triangle FGH, given angle G=102.7 , side f=14.2, and h=18.6. Now do I use law of cosines because I don't have the measure of an angle and length of the opposite side??I don't know where to go from here,I am totally confused!!!

I also have a problem with this word problem I have been doing. It asks: An airplane flies 847.5 km at a bearing of 237.3 degrees. How far south and west fo its original position is it? Huh? Please help!


Answered by Penny Nom.
Definitions and descriptions 2003-06-08
From Tammy:
MY DAUGHTERS TEACHER ASKED HER TO GIVE BOTH A DESCRIPTION AND A DEFINITION OF THE FOLLOWING ... CIRCLE, SQUARE, TRIANGLE,HEXAGON...... THE LIST GOES ON. WHAT IS THE DIFFERENCE BETWEEN DEFINITION AND DESCRIPTION ? DO A CIRCLE FOR AN EXAMPLE PLEASE.
Answered by Penny Nom.
BEDMAS 2003-05-31
From Kristie:
(3x50)+20/5=?

I know bedmas but i forget how to do it.

Answered by Penny Nom.
5 to the power n 2003-05-28
From Sam:
What is the integer n for which

5 to the n power + 5 to the n power + 5 to the n power + 5 to the n power + 5 to the n power = 5 to the 25th power?

Answered by Penny Nom.
Two sequences that agree in the first 4 terms 2003-05-26
From A student:
Is it possible to have two formulas that define sequences that agree on the first four terms but not the rest?
Answered by Penny Nom.
The cross-section of a football field 2003-05-25
From Francis:
Have you ever walked on a football field covered with artificial turf? If so, you probably noticed that the field is not flat. The profile of the surface is arched and highest in the centre, permitting rainwater to drain away quickly.

height from base to highest point- 45.75 centimetres distance of the field- 50 metres

a) The diagram shows the profile of an actual field, viewed from the end of the field. Assuming that the cross-section is a parabola, find the algebraic model that describes this shape.

b) Use your equation to determine the distance from the sidelines where the field surface is 20 cm above the base line.


Answered by Harley Weston.
An array of pennies 2003-05-23
From Saundra:
If you have 100 pennies and you are suppose to make an array do you arrange the pennies by date?
Answered by Penny Nom.
The weight of a fish tank 2003-05-20
From A ponderer:
I had a question pertaining conversion units..I have a fish tank at home and the dimensions are 24, 44, and 48 inches, I want to find out how much it weighs in pounds when filled up with water...knowing water is 62.4 pounds per cubic feet...do I just divide the 62.4 pounds to 12 because I"m trying to find out through cubic inches.
Answered by Penny Nom.
The HCF and LCM of polynomials 2003-05-20
From Charanpal:

Question:
Find the HCF and LCM of the polynomials given below. Verify that he productof these HCF and LCM differs from the product of the polynomials, if at all, by a factor of -1

  1. 1-x2 and x3 -1
  2. 1 - x2 and x4 - 1

Answered by Penny Nom.
A trig identity 2003-05-20
From Patty:
Please help with the following

1/ tanx + cotx = sinxcosx

Answered by Penny Nom.
Subtracting rational expressions 2003-05-10
From Simone:
hi, i'm totally lost. i understand that you need to find a lowest common denominator to subtract two fractions (rational expressions) with different denominators. but what if the denominators are "x-1" and "x". is x the common denominator? if so what happens to the "-1"? do you know of any live online help i can get with the following:

3/(x-1) - (1-2x)/x

i've looked through my notes and have no examples that quite match that i can follow to get through it. please help!
Answered by Penny Nom.

x-6square root of x +8=0 2003-05-10
From Elizabeth:
x-6square root of x +8=0
Answered by Penny Nom.
Circumference 2003-05-09
From A parent:
Find the circumference use 3 1/7 for pi

1. r= 28 ft.
2. D=98 cm

Answered by Penny Nom.
2/sqrt(2) 2003-05-07
From Mike:
I was looking at an example of csc 45 deg where

csc = hyp / opp = 1 / (sqrt(2) / 2) = 2 / sqrt(2) = sqrt(2)

I just don't get this. If sqrt(2) = 1.4142145..... How can 2 / sqrt(2) = sqrt(2) as the example shows?


Answered by Penny Nom.
Rules of exponents 2003-05-05
From Carl:
Hi, I am a student who would like to recall how to multiply exponents. Here is such an equation:
6.02569 X 1025 X 5.254 =?

Also, adding exponents. Don't I just add subtract the exponents separately?
Such as 523 +15-12 =??

Answered by Penny Nom.
Write sin(3x) in terms of sin(x) 2003-05-05
From A student:
Write sin 2x in terms of sin x
Answered by Penny Nom.
Four digit palindromes 2003-05-05
From Heather:
How many palindromes were in between the numbers 1000 and 9999?
Answered by Penny Nom.
The volume of air flowing in windpipes 2003-05-02
From James:
The volume of air flowing in windpipes is given by V=kpR4, where k is a constant, p is the pressure difference at each end, R is the radius. The radius will decrease with increased pressure, according to the formula: Ro - R = cp, where Ro is the windpipe radius when p=0 & c is a positive constant. R is restricted such that:
0 < 0.5*Ro < R < Ro,
find the factor by which the radius of the windpipe contracts to give maximum flow?

Answered by Penny Nom.
positive multiples of 10 that are the sum of four consecutive integers 2003-05-01
From Taurus:
How many positive multiples of 10 that are less than 1000 are the sum of four consecutive integers?
Answered by Penny Nom.
Profit for a dance studio 2003-04-28
From Craig:
A dance studio charges $80 per student for a series of 2 hour lessons. The studio's costs are $30 per hour for the instructor, $15 per lesson for the room rental, and $3 per student for miscellaneous expenses. If x is the number of students enrolled in the class, express the studio's profit P(x) in terms of x. Find the profit if 10 students are enrolled in the class.
Answered by Penny Nom.
Gallons of rain 2003-04-25
From Jocelyn:
If you have a surface of 6500 square feet and 1 inch of rain falls on the surface, how many gallons of water does this equal?
Answered by Penny Nom.
8 faces, 12 vertices, and 18 edges 2003-04-22
From Thomas:
I would like to know the proper name for this geometrical solid. It has 8 faces, 12 vertices, and 18 edges?
Answered by Penny Nom.
Area of a trapezoid 2003-04-13
From A road builder:
My husband works with asphalt building roads. There are times when one end of the road will be (for example) 100ft wide the other end would be( for example) 200ft wide and he must figure the area in square feet. So far it has been a guessing game because he dosen't have the formula to figure the square feet.
Answered by Penny Nom.
A fractional inequality 2003-04-08
From Jessica:
Solve for x. Write in interval notation.

[(x2)-9]/[x-5] >= 0

Answered by Penny Nom.
The surface area of a cube 2003-04-07
From Sarah:
if the volume of a cube is 64 cm, then what's the total surface area? I also want to know how to show my work.
Answered by Penny Nom.
How many hits? 2003-04-06
From Jack:

My name is Jack. I'm a uncle. Student is in the 5th grade email is above.

If a baseball player at sping training had a good season with the following:

one seventh of his hits were doubles.
12.5% of his hits were home runs.
But didn't have any triples.
How many hits did he have?

Can you give me an explanation of you solved the problem.


Answered by Penny Nom.
Uses of conic sections 2003-04-01
From William:
My name is William and I am doing a research paper on conic sections for my 12th grade math class. Part of the project is to find two conic sections in our world today and explain what there purpose is. I really need help in this area because I've been searching the internet for where conic sections are used in our world today and I really can't find anything. If you can tell me specific building or a pyramid that contains conic sections that would be great. Or even something in the universe would be helpful.
Answered by Leeanne Boehm.
Doubling the size of an object 2003-04-01
From Dave:
If I have a known surface area and volume of an unknown object and I want to double the size of the object, how do I find the new area and volume?
Answered by Walter Whiteley.
The vertex of a cone 2003-03-27
From Holly:
I read your response to Callie about whether a cone has a vertex or not. Is it ONLY a vertex if both halves of the cone are together or can one half of the illustration have a vertex?
Answered by Walter Whiteley.
Can twice a square be a square? 2003-03-25
From Mike:
The other day it occurred to some students that they could think of no square number which is an integer, which can be divided into two equal square numbers which are intergers, Or put another way, no squared integer when doubled can equal another square integer. For example 5 squared plus 5 squared is 50, but 50 is not a square number.
Answered by Walter Whiteley and Claude Tardif.
A royal flush 2003-03-24
From Vikki:

A poker hand consists of 5 cards selected randomly from an ordinary deck of cards: find the probability of a ROYAL FLUSH : the 10 , jack, queen,king and ace of the same suit.

I was thinking somewhere along the lines of:

*the number of ways to get the suit is 4C1
*the number of ways to get a 10 out of the 13 cards etc....
...but Im not sure I am going about this the right way, could you help?


Answered by Andrei Volodin.
Friends and enemies 2003-03-24
From Becky:

Consider a room that contains six people. Any two people are either friends of each other, or they are enemies.

A. Argue why there are three people, all who are friends, or there are at least three people, all who are enemies

B. Rephrase the situation using graph terminology, using all of these terms correctly: vertex, edge, graph, complement, clique, independent set, and bipartite.


Answered by Penny Nom.
A triangle and a circle 2003-03-21
From Jynks:
We need a formula that we can use to figure this out for work. We aren't math wiz's or students. Basically we know 3 points in space of a triangle, we know the length of each side and the length of the line from apex to base line. Each point of the base line ends upon the circumference of a circle. IS three a way to work out the radius of that circle.
Answered by Penny Nom.
Scientific notation 2003-03-20
From A student:
what is one millionth times one thousand in scientific notation.
Answered by Penny Nom.
The origin of integers 2003-03-19
From Travis:
What are the real life usages of integers? Also what is the origin of integers?
Answered by Harley Weston.
Three proffs of a trig identity 2003-03-18
From Nadene:
Prove the identity. cos [x + (y-pi/2)] = sin (x+y)

A hint was also provided which is: "Apply cos (alpha + beta) first then within that apply cose (alpha-beta)"

Answered by Penny Nom.
Working backwards 2003-03-14
From Melody:
Triva went to the ardcade at the state fair to win some goldfish. She already had some goldfish at home, but wanted more. She won enough goldfish to double her stock. Her mom made her give four away. She put her new ones in the tank with the others, by the morning half of the goldfish had died. Triva's friend gave her six more. The next morning two-thirds of her goldfish died. She was left with two goldfish after giving one to a friend. How many goldfish did she start with?
Answered by Penny Nom.
1575 2003-03-13
From Sharyn:
find 6 integers that when multiplied with each other equal +1575 and when the same integers are added together equal zero. there are more positives than negatives and one of the numbers is a double digit between 10 and 20
Answered by Penny Nom.
Surface area of a sphere 2003-03-11
From Kim:
a sphere has a surface area of 128 pi sq. units. What is its exact radius?
formula is 4 pi r2 I believe but how do I get radius

Answered by Penny Nom.
Three digit number 2003-03-10
From Grace:
What three digit positive integer is exactly 32 times the sum of its digits?
Answered by Claude Tardif.
A question on combinations 2003-03-06
From Jose:

I'm an architect student and have a question on combinations. I have a grid of 3 x 3, hence a total of 9 spaces. I have 3 elements to place in this grid.

How many possible ways are there of arranging this elements on this grid ? (order, orientation not important)

First putting the elements each in its own space and secondly allowing the elements at a given moment to "share" one space.

Since I got kind of obsessed with this I went ahead and graphically did all the combinations allowing "sharing", a grand total of 729. How could I have known this before hand ?


Answered by Penny Nom and Claude Tardif.
Can a square be a rhombus? 2003-03-04
From Beth:
Can a square be a rhombus? Some sources say yes, some say no. Some sources define a rhombus as a quadrilateral and parallelogram with equal sides, but without right angles. Some sources say a square is a special case of a rhombus. Clarity, please!
Answered by Walter Whiteley.
A number line 2003-02-27
From Shery:

My seventh grader problem of the month

0__________1______________5____>

This is a arrow, the number should be below

A.Mrs Decker created an arrow representing a number line shown above. She wanted to find points and label them with a heart (G) for Valentine's Day so that the fraction 5/g is less than 1. (be sure to mark the G and not the fraction 5/g). She pondered, "Are there any other locations for G?" Is so help her description the location of all these points. If not why not?


Answered by Penny Nom.
X times Y equals ten million 2003-02-26
From Baneen:
Would you be able to tell me which two numbers would compute ten million without any number having zeros in it?
Answered by Penny Nom.
Excluded values 2003-02-22
From Josh:
Why do you think it is necessary to include the "excluded values" when you write your answers to rational expressions?
Answered by Penny Nom.
Density 2003-02-19
From Rob:
What is the formula for converting cubic feet to pounds? (with any substance)
Answered by Penny Nom.
Order of operations 2003-02-16
From A student:
How is the order of operation used in everyday life other than in a math class or at school? Also .... Can you give me a list of all the mathematician that are still living that uses the order of operation?
Answered by Claude Tardif.
Numbers in base 5 2003-02-16
From Lori:
I'm trying to help my 6th grader with converting numbers like 82 to base 5 and 182 to base 12. We saw your examples on 613, but still are confused.
Answered by Penny Nom.
Order of operations 2003-02-15
From Debbie:
Question: 20-(9+4)x7=?
Possible answer -71 or 49?

Answered by Penny Nom.
Storyteller figurines 2003-02-10
From A student:
It takes 3/4 of an hour to bake a storyteller figurine. If only one figurine can be baked at a time, how many can be baked in 6 hours?
Answered by Penny Nom.
Finite differences 2003-02-10
From Jenny:

I need to find a formula that will work with any number.
I am finding the volume of a 3d cross- shape. Here are my results so far:

Term Number      0   1    2    3     4      5
nth term         1    7   25   63   129    231
1rst diff           6   18   38   66     102
2nd diff              12    20   28   36
3rd diff                  8     8    8

I can't seem to find a formula that will work with any number. Any help would be much appreciated.

Answered by Penny Nom.

BEDMAS 2003-02-09
From Stefanie:
I do remember the rules of BEDMAS, but for some reason this question puzzles me.

6 X 9 - 3 + 44

I started with the Multiplication

54 - 3 + 44

But then I got stuck, do I proceed with adding the 44 and subtracting the 3
or figure out what -3 + 44 is, but then how would that work with 54?

Answered by Penny Nom.
Transformations of pattern blocks 2003-02-06
From Cheryl:
I am trying to help my 8th grade son with Transformations - use of patttern blocks. He has to trace a figure and reflect it across the x-axis, which I can do, but then it indicates include ordered pairs - that confuses me.
Answered by Penny Nom.
Factoring 2003-02-04
From Marcin:
I am having a really hard time with the factor theorem. Can you please help me with the following? (a'3 means a to the exponent 3)

a)8a'3+64b'3c'6
b)8x'4-12x'3-44x'2+24x
c)x'4-21x'2-100

Answered by Penny Nom.
Filling in the back yard with dirt 2003-02-04
From Joe:

We just bought a house and need to start filling in the back yard with dirt to level it out.

I measure 37'x 56' by 2' deep on an angle.

I will be building a retaining wall at the 2' deep portion. The slop will go from the 2' to nothing at the top or near the house. I can't remember how to get the Sq. foot or cubic yards.


Answered by Penny Nom.
Fibonacci exam 2003-01-28
From Mike:

My son will be participating in the national math contest. He is a grade 4 student but will be writing the Fibonacci exam for grade 5. He has been given last years exam for practice. Parents have been ask to assist for this part. I was therfore hoping for a little "inside" help on several questions with answers & short explanations. This with a would be greatly appreciated.(from last years exam)

  1. With 3 weights, Mathusalem can weigh any object that has a weight which is smaller or equal to 13 kg, but that weighs a whole number of kilograms. He wants to weigh an object of 7 kilograms. To do so, he must place two of the weights on the left side of the scale, and the third one on the right side. Which one will he have to place on the right side

    a)4 b)3 c)6 d)2 e)5
  2. Mathilda uses a rectangular prisms to build stairs. The cross section of each prism is a square which has a side of 1 dm. Mathilda has already put in place 4 steps of a staircase that must have a height of 10 dm. When finished, the staircase will have a volume of 550 dm3. What will the width of the staicase in dm be?

    a)10 b)55 c)11 d)1 e)100
  3. The year 1991 was the last palindrome year of the second millennium. The year 2002 is the first palindrome year of the third millennium. Including 2002, how many palindrome years will there be, altogether, in the third millennium ?
    a)8 b)2 c)10 d)182 e) 9


Greatly appreciated,
Mike


Answered by Penny Nom.
Factoring a trinomial 2003-01-26
From A student:
how do you factor: Xsquared + 8x + 12
Answered by Penny Nom.
Some 5th grade problems 2003-01-24
From Sabrina:

[What is the greatest common factor of 20,28, and 36?] next problem is

[5k+7(k+8)= ] next problem

[Curtis simplified 0.8 divided by 0.2 x 0.04 and got 10. Latoya Simplified and got 1.6. is either correct? Can you explaine how you got the answer please.]


Answered by Penny Nom.
Dorothy and the wizard 2003-01-24
From Ken:

dorothy was going to see the wizard of oz...she stopped at the bakery and bought a box of cookies.

she met the scarecrow and gave him half the cookies. she ate half of the remaining cookies and threw one half cookie away.

she met the tin man and gave him half the cookies. she ate half the remaining cookies and threw one half cookie away.

she met the lion. she gave him half of her cookies. she ate half the remaining cookies and threw the LAST half cookie away.

how many cookies did dorothy leave the bakery with?


Answered by Penny Nom.
Rational expressions 2003-01-22
From A student:

// = the main fraction line

1.) a+1/a-1 + a-1/a+1 // a+1/a-1 - a-1/a+1

2.) 2/a(squared)-3a+2 + 2/a(squared)-a-2 // 2/a(squared)-1 + 2/ a(squared)+4a+3


Answered by Penny Nom.
The least common denominator 2003-01-21
From Brittan:
Hi there I need help! My name is Brittany and i am in the 6th grade.

I need help finding the least common denominator(LCD), and the book says Find the LCM of the denominators and i've done that and then it says write equivalent fractions,using the LCM as the least commonn denominator.The directions say Use the LCD to write each pair as like fractions. and the problem is 1/8 and 5/40. Could u explain how in the word u do this? Thanks a lot

Brittany

Answered by Penny Nom.
0.027 acres 2003-01-16
From Cory Lee:
How many square feet are in How many square feet are in .027 acres??
Answered by Penny Nom.
What is larger than infinity? 2003-01-12
From Dana:
What is larger than infinity?
Answered by Claude Tardif and Harley Weston.
Cubic yards in a gravel pile 2003-01-10
From Ron:
I am looking to find a way to measure gravel piles to get yards. They are generaly not square.it would be like pouring sand out of a bag into a pile. they generaly are concave in dimension. can you help. I have attached a picture to show a small scale what I am working on is in much larger volume.
Answered by Harley Weston.
A rectangular prism 2003-01-09
From Julie:
How many faces on a rectangular prism and how many bases? Can the base also be a face and can a face also be a base?
Answered by Diane Hanson.
Repeating decimals 2003-01-08
From A student:
If k=.9repeating, and 10k=9.9repeating then 10k-k=9k, k=1 therefore .9repeating=1 and 1/3=.3repeating 3x1/3=.3repeatingx3, 3/3=.9repeating, therefore 1=.9repeating

It would seem to me that .9repeating approaches one but never quite makes it. Can you clarify?


Answered by Penny Nom.
The area of my lot 2003-01-07
From Linda:
I have a lot that is 210 feet in the front, 240 feet in the back and the sides each measure 150 feet. How many square feet is this all together and how close to an acre is it?
Answered by Penny Nom.
Augmented matrix 2002-12-31
From Michelle:
I am trying to augment a matrix so that i can find the values of the variables a, b, and c. For the life of me i can't find a solution to the matrix: 3 -5 2 ' 22 2 3 -1 ' -9 4 3 3 ' 1 I thank you for your help, Michelle
Answered by Penny Nom.
Successive coefficients in Pascal's Triangle 2002-12-27
From Quincy:
There is a formula connecting any (k+1) successive coefficients in the nth row of the Pascal Triangle with a coefficient in the (n+k)th row. Find this formula
Answered by Penny Nom and Walter Whiteley.
The intersection of conics 2002-12-19
From Glenda:
We are studying systems of equations where two conic sections are the two equations that we are solving simultaneously. We were studying the number of solutions that are possible if you have an ellipse and a parabola. We all agree that there can be none, one, two, three or four solutions. The question that the students had for me was whether or not a portion of an ellipse and a parabola can overlap and thereby allow an infinite number of solutions. What should I tell them?
Answered by Chris Fisher and Harley Weston.
A bouncing ball 2002-12-14
From Eman:

Q : When a childís ball is dropped from a height h metres on to a hard, flat floor, it rebounds to a height of 3/5h metres. The ball is dropped initially from a height of 1.2m.

  1. Find the maximum height to which the ball rises after two bounces.
  2. Find the total distance that the ball has traveled when it hits the floor for the tenth time.
  3. Assuming that the ball continues to bounce in the same way indefinitely, find the total distance that the ball travels.

Answered by Penny Nom.
Factoring 2002-12-11
From Larry:
Question:

how do u factor trinonmials

EX: X 3 + Y 3

X 3 - 8Y 3

8X 2 - 72

64A 3 - 125B 6

Answered by Penny Nom.
Fractions 2002-12-10
From Jean:
What are some activities to use with advanced students in grade 7 to apply their knowledge of fractions and decimals. I'm looking for a project type of activity.
Answered by Diane Hanson.
y = 1 - sin(x + 60) 2002-12-10
From Eman:
Sketch the graph of y = 1 - sin(x+60). for 0 <= x<= 360, giving the coordinates of the maximum and minimum points and the pints where the curves crosses the y axis.
Answered by Penny Nom.
A function that is onto but not one-to-one where f:N-->N 2002-12-06
From Lisa:
A function that is onto but not one-to-one where f:N-->N
Answered by Penny Nom.
Factoring a cubic 2002-12-05
From Dorota:
I got a problem with factorise the expression
2x3+11x2-5x-50.

Answered by Penny Nom.
65,125 2002-12-04
From Brittany:
I am a three-digit number.When you reverse my digits,a larger number is formed.the product of me and the new number is 65,125.What number am I.
Answered by Chris Fisher and Claude Tardif.
The dimensions of a box 2002-12-03
From Paula:
My son, who is in secondary school, needs to make a box with the inside dimensions of 1.25 cubic feet. It's supposed to be more in the shape of a rectangle than a square.

I would like to know the equation to use to find the inside dimensions of a box if I know how many cubic feet (or inches) I need the inside to be.


Answered by Penny Nom.
A two stage rocket 2002-11-26
From Hoda:
a two stage rocket accelerates in free space by ejecting fuel at a constant relative speed , v(ex). the full fuel load makes up 80% of the initial mass of the entire two stage rocket . the rocket accelerates from rest until at the end of the first stage when 75% of its fuel has been burnt. find an expression for the speed of the rocket at the end of the first stage in terms of v(ex).
Answered by Claude Tardif.
Decimals in everyday life 2002-11-24
From Fritz:
How do you use decimals in your every day life?
Answered by Penny Nom.
Factor completely 2002-11-21
From Shelley:
these two questions are to be factored completely but i have no idea how to factor them
  1. (x-4y) 2 - 3(x-4y) - 4
  2. x 6 + y 6

Answered by Penny Nom.
Performance reviews 2002-11-21
From Lara:
Perhaps you can help.

He's doing performance reviews at work.

The average performance review is 3.92 on a 5.0 scale. This person would receive a 3.5% raise for next year.

The highest performance review is 4.9 and this person would recieve a 6.0% raise for next year.

How do I solve for the other performance review numbers.


Answered by Penny Nom.
What percent is the same as three eighths? 2002-11-21
From Katelin:
What is the percent of three eight's?
Answered by Penny Nom.
Composites, primes, factors and common factors 2002-11-20
From Connie:
My son can't remember what a composite number is, what a prime number is and the explaination of factors and common factors. Help!
Answered by Penny Nom.
Differentiating inverses 2002-11-20
From Amy:
f(x)= x3+x+1, a=1 find g'(a) (g = f -1). I am having trouble finding g(a).
Answered by Penny Nom.
4 x abcd = dcba 2002-11-20
From A student:
Say you have a four digit number (e.g. abcd) and you multiply the number by 4. The answer you get will be the reverse order of the number you started with (dcba). What is the number? The four digits (a,b,c,d) cannot be the same number or cannot be repeated.
Answered by Penny Nom and Claude Tardif.
The 24 game 2002-11-16
From Michael:
Hello, my name is Michael and I am in elementary school .. We have a game we play in math class called 24 game. In it each card has 4 numbers on it. Each number can only be used once, in any order, using multiplication, subtraction, division, or addition. We are stuck on one and wondered if you could help. The four numbers are 10-9-16-4. And they must equal 24. Any clues?
Answered by Penny Nom.
Algebra 2002-11-13
From Carol:
I am a confused parent trying to help my dau hgter solve some math problems and I need help.

3[x-4]=27


Answered by Penny Nom.
Fuel consumption 2002-11-11
From Guy:
When a car is driven 'normally' the feul consumption is 7.5 per litres per 100km. On a particular journey the car was driven in 'hard' and the consumption changed to 9.1 litres per 100km. Fuel cost 80p per litre. Calculate:

a) The extra feul consumed.
b) The extra cost of feul, in pounds, for the journey when the car was driven in 'hard'.

Answered by Penny Nom.
Radicals with fractional radicands 2002-11-11
From A student:
how do I simplify radicals with fractional radicands?

ex. 7sqrt(2) + sqrt(50) - 2sqrt(18)

10sqrt(3/5) - 24sqrt(5/3)

3sqrt(2/9) + 1/2sqrt(32) + sqrt(9/8)


Answered by Penny Nom.
Cubic centimeters 2002-11-07
From Rita:
Was in emergency room yesterday-MD aspirated 40cc of fluid from my knee--what does a "cc" equate to?
Answered by Penny Nom.
Chisanbop 2002-11-07
From Paul:
I'm trying to find the book/books that explain this system? I have one small book that explains the very beginning of the system, but covers only about 10 to 20%. I'v tried several book sellers and have had no luck.
Answered by Diane Hanson.
The percentage grade of that hill 2002-11-05
From Cathy:
If there is an 80ft climb over a kilometer(about 3280ft) what is the percentage grade of that hill?
Answered by Penny Nom.
Rounding 27.27 2002-11-03
From A parent:
If you have a problem 27.27 and you need to round to the first 7 the answer would be 27

Would 27.0 be acceptable as well or is it completely wrong.


Answered by Penny Nom.
Simplifying Variable Expressions 2002-10-28
From Erika:
i would like to ask you a mathematical question on Simplifying Variable Expressions. The question is 2(n-4)+3.
Answered by Penny Nom.
The four colour theorem 2002-10-27
From Rhonda:
is there an actual equation for the four colour map theorem??? i cannot find it anywhere!! if so can you give a breif description on how it works!!
Answered by Penny Nom.
At the zoo 2002-10-20
From A student:
The number of adults at the zoo was 2/9 the number of children. there are 700 more children than adults. The number of men was four times the number of women. The number of girls was four times the number of boys. Find the total nymber of men and boys at the zoo.
Answered by Penny Nom.
Simplify 2002-10-19
From A student:
well i need to simplify this problems out and i cant do it so here is one
 3x^2+6x-45 ------------ = 3x^2+21x+30 

Answered by Penny Nom.
The repetition of doing many straight forward problems 2002-10-16
From Dan:
I am the father of a very bright 9 year old girl. She is very interested in math and she grasps concepts very quickly. I am concerned that she may become bored with the repetition of doing many straight forward problems. Her current math teacher has her working in a challenging math book and she is doing 3+ digit multiplication. She misses 2 or 3 out of ten of these problems due to simple addition errors. Should she continue to practice these problems until she can get them all correct? Or, should she move on?
Answered by Kathy Nolan and Claude Tardif.
Integers in real life 2002-10-15
From Rica:
What are the real life usages of integers?
Answered by Peny Nom.
A functional equation 2002-10-14
From Rob:
Let f be a function whose domain is a set of all positive integers and whose range is a subset of the set of all positive integers with these conditions: a) f(n+1)>f(n)

b) f(f(n))=3(n)

Answered by Claude Tardif.
Two problems 2002-10-14
From Eva:

a) How many different equivalence relations can be defined on the set X={a,b,c,d}?

b)Show that 6 divides the product of any 3 consecutive integers. I know it is true that 6 divides the product of any 3 consecutive integers. However, i have problem showing the proof.


Answered by Leeanne Boehm and Penny Nom.
Three algebra problems 2002-10-13
From Veronica:
Solve the following inequality:

5(xsqured-4)/(xto the 5th(2x-5)to the 3rd) < or equal 0

Solve the following equations for all roots

square root of x+2=-1+square root of 2x+3

what's ZERO FACTOR PROPERTY???

The hypotenuse of an isosceles right triangle is 7cm. long. Determine the lenghts of the other sides.


Answered by Leeanne Boehm.
Nixon, Jefferson, and Madison 2002-10-08
From Lisa:
The longest-lived US presidents are John Adams(age90), Herbert Hoover (also90), and Harry Truman (88). Behind them are James Madison, Thomas Jefferson, and Richard Nixon. The lattter three men lived a total of 249 years, and their ages at the time of death form consecutive odd integers. For how long did Nixon, Jefferson, and Madison live?
Answered by Penny Nom.
8 squares from 12 sticks 2002-10-08
From A student:
If you have 12 sticks the same size, how do you make them into 8 squares?
Answered by Claude Tardif.
A complex quadratic 2002-10-06
From Michael:
I would like to know, how to solve this Complex number: quadratic equation. ix 2 + x -i = 0
Answered by Harley Weston.
Rational expressions 2002-10-03
From Ashley:
1/x(squared) + 5/xy
Answered by Penny Nom.
Proof by induction 2002-09-26
From Pooh:
Use induction to show that

1 2 + 2 2 + .....+n 2 = (n 3)/3 + (n 2)/2 + n/6

Answered by Paul Betts.
Dividing fractions 2002-09-23
From Angie:
When I was taught to divide a fraction, I was told to multiply the reciprocal. Many times I find I can just divide the fraction, and it saves all the simplifying after.

For example, 4/6 x 9/14 divided by 2/4

I would say 4x9divided by 2=18 over 6x14divided by 4 =21, so the answer is 18/21 but if I multiply the reciprocal, I would end up with 4x9x4 over 6x14x2 which equals 144/168.

So, why are we taught to do it this way? Is it necessary?


Answered by Chris Fisher.
Fractional exponents 2002-09-20
From Jill:
The problem is with fractional exponants:
10 1/3 mult. by 10,000 The 1/3 is an exponant of 10.

Answered by Penny Nom.
Formulating equtions 2002-09-14
From Theresa:
Hello. In math class we have to formulate equations for certin questions for extra credit. We are allowed to ask anyone and anyone can do it for us and explain it to us. Could you please do that? Here are the questions
  1. Jordan has 3 times as much money as Reva. Together they have $44. How much money does each have?

  2. The Tigers played 48 games. They won twice as many as they lost. How many did they win?

  3. There are 900 students in Sewickley Academy. There are 20 more girls than boys. How many girls are there?

  4. A board 400 cm long is cut into 2 pieces. One piece is 66 cm longer than the other. Find the lenght of the shorter piece.


Answered by Penny Nom.
Base 5 numbers 2002-09-13
From Vera:
? how do you change a base 10 number to base 5
Answered by Penny Nom.
The number of decimal places in 1 over a power of 2 2002-09-12
From Allan:
Does anyone notice that the maximum number of decimal place of the number 2 dividing 1 and its increment (4, 8, 16...etc) is the same as the power of number 2? eg. 22=4, thus the max number of decimal of 1/4=0.25 which is 2 decimal place and 2 is the number of power of 2 take 64 as example: 26=64, and take 1/64=0.015625 which has 6 decimal place (and is the power 6)

Is there such a law in math? If yes, can you tell me what it is? Or is this my discovery?


Answered by Paul Betts.
Use a 4 four times 2002-09-12
From Amanda:
The question is what is if you can only use a 4 four times and get the answer 12, 10, and 6
Answered by Claude tardif.
The quadratic formula 2002-09-11
From Kathleen:
im having a problem trying to understand quadratic formulas can u please help me

i have problems like

3x2 - 4x - 11 = 0 how do i work out that problem? i have the formula written down but it doesnt seem to register for me.

Answered by Penny Nom.
A fish weighs 10 pounds more than half its weight. 2002-09-10
From Rebecca:
A fish weighs 10 pounds more than half its weight. How much does it weigh?
Answered by Penny Nom.
The factor theorem 2002-09-10
From Yael:
Prove (x - a - b) is a factor of x3 - a3 - b3 - 3ab (a + b)
Answered by Penny Nom.
A linear function 2002-09-09
From Chad:
If y = 9x - 13 does it represent a linear function? And if it does what is its slope and is it a direct variation?
Answered by Peny Nom.
The area of a triangle 2002-09-07
From Phill:
How do you find the area of a equilateral and other triangles?
Answered by Penny Nom.
Subsets of the Real Number System 2002-09-07
From Christine:
In looking for a French equivalent of the word "integer", I found that the word seems not to actually exist in French, and that Canadian schools use the term "natural number" to describe what we have been trained to call "whole numbers," while using the term "whole number" to describe what we have been trained to call "integers."
Answered by Claude Tardif.
What really is pi? 2002-09-05
From Rachel:
what really is pi? 3.1444
Answered by Penny Nom.
1/8 in decimal form 2002-09-04
From A student:
explain how you put 1/8 in a decimal form
Answered by Penny Nom.
A limit 2002-09-03
From Circe:

Lim 3x 3+ax+a+3 /(x 2+x-2), X->-2


Answered by Penny Nom.
A schedule for a fantasy football league 2002-09-01
From Steve:
i am in a ten(10) team league, split evenly into two five team divisions. we play a thirteen(13) game schedule and would like to play each team in your own division twice, and each team in the other division once. according to my rudimentary mathematics that equals 13 games. a good schedule should look somthing like this: team 1-5 = division 1 team 6-10 = division 2 team 1 should play teams 2-5 twice and teams 6-10 once.
Answered by Claude Tardif.
Proof by induction 2002-08-31
From Tabius:
Use mathematical induction to prove that the following formulae are true for all positive integers:

a) 1 + 3 + 5+...+(2n - 1) = n 2

b) 2 n > n.


Answered by Penny Nom.
How many cubic feet are in a cubic mile? 2002-08-30
From William:
How many cubic feet are in a cubic mile?
Answered by Penny Nom.
38-10+12divided by4multiplied by 16 2002-08-30
From Brenda:
my math question is as follows: 38-10+12divided by4multiplied by 16=?
Answered by Penny Nom.
A solution to an equation 2002-08-28
From A student:
How do you determine whether a pair of numbers is a solution for an equation?

If you could identify the specifics steps I need to take to determine whether it is a solution or not would be appreciated! Thanks

i.e. x-4=0 Pairs of numbers as possible solutions (4,7) (3,4) (0,-4).


Answered by Penny Nom.
Mixing gas and oil 2002-08-27
From Sherry:
how do I calculate the correct fuel ratio for a boat motor? 50:1 the oil containers only say how many ounces to add to 5 gallons of fuel to make a 50:1 ratio.(13 ounces to 5 gallons of fuel) I have a 20 gallon tank so what if I add 11.6 gallons of fuel, what formula would I apply to get the correct measurement of oil to add?
Answered by Penny Nom.
Common Denominator 2002-08-26
From Slobodanka:
What is a Common Denominator?
Answered by Penny Nom.
Cubic yards of concrete 2002-08-24
From Michael:
if you have a 10" dia pipe 8' long how many cubic yards would it take to fill with concrete. how how would you go about figuring it out.

and also if you would how many cubic feet, cubic inches in a cubic yard. i know you don't need to answer, seeing i am not in school, but would appreaciate it.


Answered by Penny Nom.
Gallons in a cylindrical tube 2002-08-21
From Heidi:
IF I HAVE A 12" DIA TUBE WITH 1/4" WALL AND 72" IN LENGTH HOW MANY GALLONS OF WATER WILL THIS HOLD
Answered by Penny Nom.
Sole Mates Shoes 2002-08-19
From Robert:
Sole Mates Shoes has expenses of $9,592 per month. What must the store's total income be if it is to make a profit of 12%?
Answered by Penny Nom.
When is 1! + 2! + 3! + ... + x! a square? 2002-08-19
From Sarathy:
Solve :

1! + 2! + 3! + ... + x! = y 2

How do i find the solutions ?


Answered by Claude tardif.
Volume of a sphere 2002-08-16
From Rahul:
How do i find the volume of a sphere without the knowledge of calculus?
Answered by Walter Whiteley.
Distance in 3-space 2002-08-16
From David:
The question is: how do I figure out the distance of one object in 3D space to another object in 3D space? I have an object at say x = 5.872, y = 2.876, and z = 7.290; and the other object is at x = 1.129, y = -8.213, and z = -11.127. I have been suggested to use the pythagorean theory on this, but since there are three variables, I don't understand how.
Answered by Penny Nom.
The circumference of a 72 2002-08-14
From Linda:
What is the circumference of a 72" diameter circle?
Answered by Penny Nom.
Two equations 2002-07-26
From Derek:
1. 3x + 2y = 4
2. -7x + 2y = 24

finding x and y.


Answered by Penny Nom.
Some inequalities 2002-07-25
From A student:

1)-5 <= -3X+1 < 1

2)4X-2 < 6 OR X+2 > 9


Answered by Penny Nom.
Domain of a function 2002-07-20
From Andy:
I'm having difficulty in finding the domain of

1/(x2 -2x + 4)

one over x squared minus 2 x plus 4.


Answered by Penny Nom.
Percentage 2002-07-20
From Sally:
If 20% of 100 is 80, then why is the percentage difference between 80 and 100 25%.
Answered by Penny Nom.
Order of operations 2002-07-18
From Danna:
I would like to know how to solve this type of problem; I already have the answer.

Problem: 2 [5 (4 + 6) - 2] = 96

Also, what do you call this type of problem? Thanks a lot.


Answered by Penny Nom.
Future value 2002-06-27
From Susan:
If one were to invest $115 a month for 20 years and expect a 4% annual return, what is the value of this money in 20 years?
Answered by Leeanne Boehm.
Successive coefficients in the nth row of Pascal's Triangle 2002-06-10
From Tim:
There is a formula connecting any (k+1) successive coefficients in the nth row of Pascal's Triangle with a coefficient in the (n+k)th row. find this formula.
Answered by Penny Nom.
0.63 of an acre 2002-06-08
From Sherry:
how many square feet are in 0.63 of an acre
Answered by Penny Nom.
Constructing the square root of 3 2002-06-07
From Allan:
I am a Math 7/8 teacher. I was wondering how you would show a student how to find the exact location of the square root of three on the number line using just a compass and a straight edge.
Answered by Penny Nom.
The area of a circle 2002-06-03
From Jessica:
I am doing a maths assigment for university, which is aimed towrds primary school students(k-6). I was wondering if you could give me some information as to how I could describe to students the rule for finding the area of circle, using a circle cut up into equal sectors (like a pizza). I know it has something to do with the fact that you can make these shapes into a parallelogram, but I am a bit uncertain as to how I can express this idea clearly and articulately to students.
Answered by Penny Nom.
Cubic feet and cubic yards 2002-05-27
From John:
how do you find the cubic feet/cubic yard of area 10 feet wide by 15 ft long by 2 inches high
Answered by Penny Nom.
One-fourth of a number is added to one-third of the same number 2002-05-26
From A student:
When one-fourth of a number is added to one-third of the same number, the result is 28. What is the number?
Answered by Penny Nom.
A spotlight shines on a wall 2002-05-25
From Barb:
A spotlight on the ground shines on a wall 12m away. If a man 2m tall walks from the spotlight toward the bldg at a speed of 1.6 m/s, how fast is his shadow on the bldg decreasing when he is 4m from the bldg?
Answered by Penny Nom.
A schedule for 24 golfers 2002-05-25
From John:
I am working on a schedule for 24 golfers. 6 groups of 4. I have 8 golf days (twice per week for a month).

Ideally, I would like to schedule all 24 golfers in 6 different groups of 4 on each day. Here is the catch.....no golfer in any group can be grouped togther more than once. Every group of 4 each day will have 4 new golfers that have never played together before. Is this possible?


Answered by Chris Fisher.
How far apart are the transmitters? 2002-05-18
From Jeff:
A ship at sea is 70 miles from one transmitter and 130 miles from another. The measurement of the angle between the signals is 130 degrees. How far apart are the transmitters?
Answered by Penny Nom.
Sequences that agree on their first four terms 2002-05-16
From Mike:
Are there two formulas that define sequences that agree on their first four terms, but differ on the fifth term and all succeeding terms?
Answered by Chris Fisher.
Thank you gifts 2002-05-15
From Navi:
Children usually attend birthday parties where a thank you gift is given to the children who attent the party. a family likes to hand out books at thank-you gifts.

(a) One child invited to the party is given 2 books as gift. if there was 15 different books to choose from, how many diffrent gifts were possible?

Answeer: I did this way 15C2=105

(b) if the 15 books consisted of 8 nonfiction and 7 fiction, how many diffrent gifts were possible if at least 1 of 2 books were fiction?


Answered by Penny Nom.
The law of cosines and obtuse angles 2002-05-09
From Bryant:
The question that I am pondering is that I need to derive the law of cosines for a case in which angle C is an obtuse angle.
Answered by Penny Nom.
8/13*26/27 2002-05-01
From Arias:
8/13*26/27=
Answered by Penny Nom.
Factor 12x^2 - 20x - 8 2002-04-30
From Crystal:
Factor Completely: 12x2 - 20x - 8
Answered by Penny Nom.
The Dugout, Don's Basement, Cd Corner,... 2002-04-30
From A student:
The Dugout, Don's Basement, Cd Corner, Harry's Deli, Bill's Software, Anne's Footwear, and Joanne's House cleaning.

The gift certificates are each in multiples of $5. There is a $100 range in the value of the gift certificates, which start at $25. The mean value of all seven gift certificates is $80, and the median and mode are both $70.

The certificate from The Dugout is worth the most and the one from Joanne House cleaning is worth the least.

The total value of the gift certificates from CD Corner, Harry's Deli, and Anne's Footwear is $270, but Anne's Footwear certificate is worth $50 more than the one from Harry's Deli. The Cd Corner gift certificate is equivalent to the mean for this group of three.

What is the value of the gift certificates from each store?


Answered by Penny Nom.
A 6,000 gallon tank 2002-04-28
From Someone with a big tank:
I was wondering if you can find out information about how much liquid fuel can fit in a 6,000 GAL. tank. Right now I do not have the exact dimensions of this tank but I will have it the 29th of April. Could you please E-mail me and let me know if you can answer this question after I have the correct specifications.
Answered by Penny Nom.
0/4 = ? 2002-04-28
From Danielle:
I am embarrased to be asking this question, but... is it possible to have a fraction with a zero? For example, 0/4. This does not make sense to me and I do not know what it would be representing, other than nothing! Is it proper to express such a fraction?
Answered by Peny Nom.
A triangular prism 2002-04-28
From A student:
Need the formula for the surface area of a triangular prisim?
Answered by Penny Nom.
Triangular numbers 2002-04-26
From Anika:
Can you please tell me what a triangular number is?
Answered by Penny Nom.
Functions and relations 2002-04-25
From Erin:
Here's a few math problems that might drive a person insane ;)
  1. If 2f(x) - 3f(1/x) = x^2 what is f(2)?

  2. What are the domain and range of the following relations?
    • x^2 - 4y^2 = 25
    • 4x^2 + 9y^2 = 36

  3. The port of Swan Harbor is 200km away from Merry Town Inlet on a bearing of N50E from Merry Town. A ship leaves Merry Town at 8am and sails N15W at 15 km/h. At the same time, a second ship leaves Swan Harbour on a course of S80W at 20km/hour. How close, to the nearest km, are the two ships at 13:00?

  4. Prove the identity. 1+sinx + cosx/ 1-sinx + cosx = 1+sinx / cosx

Answered by Penny Nom.
Solve for x 2002-04-20
From A student:
solve for x.
  c-x      2x+b ----- = --------   a        c 

Answered by Peny Nom.
An augmented matrix 2002-04-20
From A student:
Hi my math teacher asked us to solve an augmented matrix. I am in twelfth grade and need help. The book we are working on is college algebra. Here it is

a+2b+c=0
2a+5b+4c=-1
a-b-9c=-5


Answered by Penny Nom.
Moving a triangle 2002-04-18
From A student:
find the verticles of a triangle after it is translated 2 units to the left and then is reflected across the graph of y=x+2. The original verticles of the triangle are (2,0), (3,2), and (6,2).
Answered by Peny Nom.
A polynomial 2002-04-14
From A student:
A polynomial function is described by the following conditions: f(x) has two real zeros at x= -2 and x= 1, each of multiplicity 2 f(x) has two complex zeros as x goes to -infinity, f(x) goes to +infinity as x goes to +infinity, f(x) goes to +infinity f(x) has a y-intercept at (0,2)

Sketch a polynomial function that satisfies the above conditions.

Write an equation for function f(x)

Write another function g(x) that also satisfies the above parameters.


Answered by Penny Nom.
How many gallons of water in a cubic foot? 2002-04-14
From Richard:
I am tring to find out how many gallons of water in a cubic foot.
Answered by Penny Nom.
The square root of four 2002-04-13
From Frank:
In a recent quiz we where asked the square root of four. The quizmaster gave the answer as not only 2 but also -2. Being pedantic I queried this. Although -2 squared is 4, does it follow that the root of 4 is -2 Can you enlighten us?
Answered by Claude Tardif and Penny Nom.
Calculate profit 2002-04-13
From Allison:
If I sell something for $75; and my cost is $40, I make $35 profit. How do I calculate the profit percent I made on that transaction?
Answered by Penny Nom.
Sodding the yard 2002-04-08
From Craig:
my parents want to lay sod in our back yard.

the sod cost $2.00 per square foot.

The size of our yard is 20x20 and 26x41


Answered by Penny Nom.
Area of a triangle 2002-04-07
From Puzzled:
What is the surface area of a triangle 13 ft x 13 ft x 18 ft?
Answered by Penny Nom.
Composition of functions 2002-04-06
From Yvonne:
In our new text book, the following question occurs: State the domain and range of g(f(x))given that f(x) = -x2 - 4 and g(x) = sqrt(x)

The range of f(x), x<=-4, is the domain of g(x). BUT, there is no solution in the Real numbers for g(f(x))= sqrt(-x2 - 4).

In the solutions it says that this is not a function and therefore does not have a domain or range. Is it a relation? Is it anything?


Answered by Claude Tardif.
Pairs of equations 2002-04-04
From A student:

high school level
student is asking

y=4x x=-4y 
x+y=5 3x+2y=20   

y=x-1 3x-y=4 
x+y=3 2x-3y=-9   

x+5y=4 
3x+15y=-1 

. . . 

Answered by Penny Nom.
Common fractions to decimal fractions 2002-04-02
From Natalie:
My questions are: "how can I change 23/60 into a decimal. and give my answer to 2dp."??????????

2nd question:for each of these pairs, which is bigger?

4/7 or 36%____________
3/11or 0.29____________
14/24 or 0.57___________


Answered by Penny Nom.
Pascal's Triangle 2002-04-02
From Brian:
It's about (a+b)x. I remember there a triangle with numbers to remember for a faster solution. Can you please teach me?
Answered by Penny Nom.
Area of an octagon 2002-04-01
From Someone:
What is the actual square footage of a 12' octagon?
Answered by Penny Nom.
All four eights in three consecutive hands 2002-04-01
From Blair:
What are the odds against the same person holding all four eights in three consecutive hands.
Answered by Brian Alspach.
The reciprocal of a decimal mixed number 2002-03-29
From A student:
How do I find the reciprocal of a decimal mixed number?
Answered by Penny Nom.
Some 5 card hands 2002-03-28
From A student:
From a standard deck of cards how many 5 card hands are possible consisting of a. exactly 4 hearts

b. two cards of one kind and three of another(like a full house).

Answered by Penny Nom.
100 from four 9's 2002-03-27
From A student:
My teacher gave us a math problem to try and figure out and I am stumped. The problem was this.... by only using four nines, how can you get an answer of 100? You can add, subtract, multiply or divide...anything you need to do ,but you can only use four nines.
Answered by Leeanne Boehm.
Unit fractions 2002-03-23
From Laura:
Write 1/2 as the sum of ten different unit fractions.
Answered by Claude Tardif.
A hamburger and a soft drink 2002-03-23
From A student:
A GROUP OF FRIENDS WENT OUT TO LUNCH. EACH BROUGHT A HAMBURGER AND A SOFT DRINK. TOGETHER A HAMBURGER AND A SOFT DRINK COST MORE THAN A DOLLAR. THE TOTAL COST FOR THE GROUP WAS $17.81. THERE WAS NO TAX OR TIP INCLUDED.

IF A HAMBUGER COSTS 2 CENT MORE THAN TWICE A SODA, FIND THE COST OF THE HAMBURGER.


Answered by Leeanne Boehm.
What fraction of the world's motor vehicles are built in Canada? 2002-03-20
From A student:
About 1/4 of the world's motor vehicles is built in Canada or the United States. About 1/5 of the world's motor vehicles are built in the United States. What fraction of the world's motor vehicles are built in Canada?
Answered by Claude Tardif.
Adding algebraic fractions 2002-03-20
From Dolores:
I get totally confused with this problem. I get confused with the getting the lowest terms.

4/x-5 + -2/x -10/x^2-5x


Answered by Penny Nom.
24x^4 + 3x 2002-03-18
From gary:
24x 4 + 3x
Answered by Penny Nom.
How much olive oil? 2002-03-15
From Annie:
Joe used 1/3 cup of olive oil, which was 3/4 of the amount that he had. How much olive oil did he have!
Answered by Penny Nom.
Testing a hypothesis 2002-03-14
From A student:
A large distriutor of cosmetics has kept his outstanding accounts receivable to a mean age of 18 days over the past year. This average is considered a standard by which to measure the efficiency of the credit and collections department. Management wishes to check if receivables in the current month is over standard and will do this at a significance level of 0.50. A random sample of 100 accounts yields an average of 20 days with a standard deviation of 9 days. what should management conclude?
Answered by Andrei Volodin.
The square root of i 2002-03-14
From Arlene:
what is the square root of i, if i=x+yi?

what is the square root of 1-i? i'm getting problems like these in which I do not understand.


Answered by Harley Weston.
The isosceles triangle of smallest area 2002-03-08
From Lettie:
can you find the isosceles triangle of smallest area that circumscribes a circle of radius of one?
Answered by Walter Whiteley.
Percentage difference 2002-03-08
From Tedd:
Can you give me the formulas for calculating percentage differences between two numbers? For example: if the first number is 10 and the second number is 15, then the second number is 50% larger than the first number. However, the formula to calculate that percentage won't work if the first number is greater than the second number as in the following example: The first number is 15 and the second number is 10, with the second number being 33.3% less than the first number. Or maybe I'm mistaken and the same formula will work in both situations.
Answered by Penny Nom.
Factor completely 2002-03-07
From Taylor:
I'm supposed to completely factor this but I don't know what to do with all the variables.

a3b5 - a2b5 - 12ab5.

Does it have anything to do with b5 being a common factor or am I completely off?
Answered by Penny Nom.

The square root of 2 2002-03-05
From Roger:
Does two (2) have a square root or do the numbers just keep going? Are there any other numbers that behave like two when it comes to extracting the square root?
Answered by Penny Nom.
Mixed numbers, whole numbers and improper fractions which are less than 2. 2002-03-03
From Dawn:
Change each mixed number to a whole number and an improper fraction which is less than 2. 3 1/2 = 2 /
Answered by Penny Nom.
Simplify this expression 2002-02-26
From Francine:
2X power 5 Y power 3 times 18 X power -5 Y power -3 divided by X power 1/3 Y power 1/4.

I came up with
     36 ---------- X 1/3 Y 1/4 
The book says 36 x1/3 y1/4.
I don't get it.

Answered by Penny Nom.
sin 2x = cos 3x 2002-02-25
From Allan:
solve:

sin 2x = cos 3x

Primary question: how do you handle the cos 3x?


Answered by Paul Betts and Chris Fisher.
Alfredos house number 2002-02-21
From Aunt Patty:
Alfredos house number is between 20 and 35. The sum of the digits is less than 5. If you subtract 1 from it you would get a multiple of 3. If you add three, you get a multiple of 5. What is Alfredos house number.
Answered by Penny Nom.
Proof by induction 2002-02-20
From Tamaswati:
How do I prove the assertion that "the determinant of an upper triangular matrix is the product of the diagonal entries" by mathematical induction? (Before I check this assertion for a few values of n how do I rephrase the assertion slightly so that n appears explicitly in the assertion?)
Answered by Penny Nom.
A farmer takes 100 animals to the market to sell. 2002-02-19
From Ann:
A farmer takes 100 animals to the market to sell. All are sold and $100 is earned. The prices were $5.00 per cow, $1.00 per sheep and $.05 per pig. At least one of each kind of animal was sold. How many of each kind of animal did the farmer take to market?
Answered by Penny Nom.
Sampling distributions 2002-02-18
From A student:
  1. given: n = 40, standard deviation is not known, population of individual observations not normal. does the central limit theorem apply in this case? why or why not?

  2. for an estimation problem, list two ways of reducing the magnitude of sampling error?

  3. What will happen to the magnitude of sampling error if the confidence level is raised all other things remaining the same? justify your answer?

Answered by Harley Weston.
Double square roots 2002-02-17
From Ali:
i have a question about how to do double square roots with variables and powers.

example : v/"" v/"" 81y8


Answered by Harley Weston.
A degree 5 polynomial 2002-02-17
From Carla:
Write an equation for a polynomial of degree 5 given the following zeros:
-7+3i, -2+sqrt(13), 6

Answered by Penny Nom.
The perimeter of an ellipse 2002-02-14
From Harry:
I am planning to build a coffe table with an ellipse of 24x36 for the top. I wish to decorate the edge and need to know the lenght of the perimeter for lay out purposes. Is there an easy way to approximate this figure with out using intergal calculus?
Answered by Penny Nom.
Parabolas 2002-02-03
From Kuang:
-Who is credited for working with or studying the Parabola?

-What is a conic section?

-What does a parabola look like?

-How is a parabola formed?

-Where and how are parabolas used today in the real world?


Answered by Harley Weston.
Zeta[2]=(Pi2)/6 2002-02-02
From alex:
Can you please tell me a proof that Zeta[2]=(Pi2)/6
Answered by Chris Fisher.
I am a fraction 2002-02-01
From Anthony:
I am a fraction that is greater than 1 but less than 2. The sum of my numerator and denominator is 11. My denominator subtracted from my numerator is 1. What fraction am I?
Answered by Paul Betts.
The size of a lot 2002-01-26
From Claudia:
I own a piece of property that I need to know the square feet for assessment purposes. The figure they came up with is wrong. They measured from one point to another and halved the sums but that means I own the cul de sac and we don't. My lot is 55 feet wide and one side is 108.96 feet and the other side is 146.04 that extends all the way to a circle. The front of the lot on the cul de sac is stated on the survey like this. 78.21 feet where R=40 feet. This large arc is taken off the size of our land. How many square feet is our lot.
Answered by Harley Weston.
Adding algebraic fractions 2002-01-23
From Francine:

Hello, I've come up with an answer for the following problem but it's not the same as the book's answer. However, the book has a lot of mistakes in it and I don't know if I'm doing it properly or not. Your help would be soooo appreciated. It's driving me crazy.The problem is:

   
  2             + 3          + 4 
---------  +  ---------  +  ---------- 
(x-1)^3        (x-1)^2       (x-1) 

[(x-1)^3 is (x-1) to power of 3 etc.]


Answered by Penny Nom.
Successive differences 2002-01-20
From Linsey:
what is the rule for working out the formula for a sequence with three lines of difference? eg.
                      
                     1, 3, 7, 15, 29 
1st line differences:  2, 4, 8, 14 
2nd line differences:    2, 4, 6 
3rd line differences:     2, 2 

Answered by Penny Nom.
Rolling 5 sevens before rolling a six or an eight 2002-01-20
From Tony:
When rolling 2 dice, what is the probability of rolling 5 sevens before rolling a six or an eight?
Answered by Andrei Volodin and Penny Nom.
The distance across a circle 2002-01-18
From Douglas:
If you know how far around a circle is (say earth) 25000 miles how do you calculate the distance across?
Answered by Penny Nom.
Making 24 2002-01-17
From Renee:
My 4th grade daughter and I need to find a simple math sentence using 5, 5, 3, & 7 to equal 24. You can add, subtract, divide or multiple.
Answered by Penny Nom.
What is a group of three numbers called in a large number? 2002-01-15
From Laurie:
What is a group of three numbers called in a large number?

My son has this question on his 4th grade worksheet. I've taught middle school math for 7 years and nver heard of it.


Answered by Paul Betts.
Adding vectors 2002-01-12
From Lena:
how do you add vectors together?
If you are given the length and angles of both vectors and are asked to add/subtract them, how do you do it? I know you are supposed to do the head to tail method, but whenever i try it i get the wrong answer. I need help setting it up.

example: A is 2.7cm, and 60 degrees, B is 1.6cm and 135 degrees, find the magnitude and amplitude


Answered by Penny Nom.
A simplification problem 2002-01-10
From A Parent:
FIND

(A / B) . (C / D)
where
A = Y squared + 3 . Y cubed
B = Y squared + 4
C = 2 . Y + Y squared
D = Y + 4 Y squared + 3 Y cubed


Answered by Penny Nom.
Nickles, dimes, quarters and fifty cent pieces 2002-01-08
From A parent:
The total for all coins counted is $4,564.50 The last coin added to the pile is a 50 cent piece There are 8 times as many 50 cent pieces as there are quarters There are 6 times as many dimes as nickles How many of each are there?
Answered by Penny Nom.
All of the Christmas trees are fifty-five percent off. 2002-01-08
From A teacher:
All of the Christmas trees are fifty-five percent off. Madison bought a tree and paid $103.50. Assuming no tax, what was the original price of the tree?
Answered by Penny Nom.
Catenary 2002-01-02
From Jason:
I am a high school math teacher. I was asked by a friend who is in architectural design for a method for determining the volume of what he called a Catenary.

The Catenary curve is modeled by the equation y=a cosh(x/a). I ran into a mess when I tried to compute the volume of the solid formed by revolving that curve around the y-axis.

Any help you can provide would be greatly appreciated.


Answered by Harley Weston.
2/3 vs 3/4 2002-01-02
From Julia:
So, here's my question. Some of the problems they've had to do starts off with two fractions, and they have to tell whether the two fractions are:
  1. The first one is greater than the second one
  2. The first one is less than the second one, or
  3. They are equal.

One day my son had the problem of 2/3 vs 3/4. Without drawing out a pie chart, how in the world can a child "eyeball" a problem like this and tell what the correct answer is? I've also tried to teach about making common denominators, but that's going over his head at this point.


Answered by Denis Hanson and Harley Weston.
Fractions to decimals and percents 2001-12-31
From A student:
2/5 , 4/8 , 4/10 , 3/5 , 6/8 , 7/7 (what is the decimal and percent for each)

1/5 , 1/10 , 1/8 , 1/1 (what is the decimal for each)

100 PERCENT, WHAT IS THE DECIMAL?

The ratio of boys to girls in a class is 1:4. What percent of the class are girls?

Answered by Penny Nom.
Composition of functions 2001-12-16
From Paula:
  1. if f(x)= 3x-1 and g(x)= 1/2x + 3 find fog(2)

  2. find the values of x for which tanx=0

Answered by Penny Nom.
Algabra 2001-12-13
From Stephanie:
My name is Stephanie. My grade 7 class is doing some strange form of algebra which I've never done nor seen before. We are doing equations like these: 3y - 6 + 7 - 4y = 6y - 6. I don't really understand them and was wondering if you could help me do my homework and explain how to at the same time. It would be really great if you could!!!!! Thank you so much! -Stephanie
  1. -9x - 5 - 8 + x = ?

  2. 5 + 6x - 3y + x + 8y = ?

  3. -x - 3 + 5x + 6y + 8x - 9= ?

Answered by Penny Nom.
Reduce 79,537/3,623,420 2001-12-13
From Eliana:
How do you reduce 79,537/3,623,420.
Answered by Claude Tardif.
Recalling the basic facts 2001-12-12
From A parent:
My son is in 4th grade and is a very bright student. He is in the gifted program and makes straight A's on his report card and has through out school so far. My question is how can I teach him to memorize his basic math facts? He does well in math, but when he is placed in a times situation for completing math fact sheets he freezes up. He can not recall the basic facts when questioned at any other time either. He will calculate the answer in his head, like 6 + 3, but he can't just come out with the answer quickly. How can I help him?
Answered by Claude Tardif.
Exponential form 2001-12-11
From Crystal:
Hi, I am asking a question on behalf of a homeschooling grade 11 student. She is having trouble with changing something like (3-4)2 into a exponential form.

First of all she needs to know what the term means "exponential form" and then the process of it.


Answered by Penny Nom.
Piles of coins 2001-12-05
From A student:
Sharon has less than 20 coins. When she puts them in piles of 5, she has 1 left over. When she put them in piles of 3, she also has 1 left over. How many coins does Sharon have?
Answered by Penny Nom.
Fractions in lowest terms 2001-12-05
From Thomas:
how to change decimal to a fraction in the lowest terms .7 .65 .019 .0111 .225 .1225 .625 .25 .025 .26 .875 .5 .75 .750 .33 .09 .44 .15 .12 .90 .044 .2500 .95 .500 .02 .40 .125 .1276 .00009 .45 .125 .1276 .375 .975 .054 .0005 .20 .2 .0001 .85 .80 .9 .0007 9.8 10.17 25.75 48.4 80.35 19.64 276.20 31.042 18.1034
Answered by Penny Nom.
83.3% 2001-12-03
From Ellie:
Please explain how a percentage like 83.3% is changed into a fractions. The answer given is 5/6 and I changed 83.3% to a regular decimal of .833, which would be 833/1000. How do they get 5/6 from that? HELP
Answered by Chris Fisher.
Integers 2001-12-02
From Alison:
How do you use integers in your everday life?
Answered by Harley Weston.
Can a infinite set be smaller than another infinite set? 2001-11-29
From Carlos:
Can a infinite set be smaller than another infinite set? If so why?
Answered by Chris Fisher and Penny Nom.
Slope intercept form 2001-11-27
From A student:
what is slope intercept form?
Answered by Penny Nom.
Dual spaces 2001-11-26
From Hoda:
In the book "Linear Functional Analysis" from Epstein there is a proof (page 116) of the following theorem:

The dual space of Lp is equal to Lq.

Later, it is stated as a theorem without proof that

for 1 <= p < +infinite, The dual space of lp is equal to lq.


Answered by Doug Farenick.
Ratios and proportions 2001-11-26
From Adam:
I am doing a project where I come to this website and ask a math expert about a certain topic. My topic is ratios and proportions. Can you please tell me what your job is and how you use ratio's and proportions in your job. Can you also please explain what ratios and proportions are.
Answered by Harley Weston.
Undetermined coefficients 2001-11-22
From Hoda:
The equation is:

y" - 2y' + y = t et + 4

We need to use The method of Undetermined coefficients. I have tried assuming that the solution is Atet+Bet+C, but all I get is C=4 and I tried (At2+Bt+C)et+D, but again I get 0=0 when I calculate the first and second derivatives, so i get no information on the constants. Any suggestions?


Answered by Harley Weston.
A tangent line 2001-11-21
From A student:
write an equation of the line tangent to the graph of

ey + ln(xy) = 1 + e at (e,1)

Answered by Harley Weston.
Rewriting and evaluating formulas 2001-11-19
From A conserned mother:
Due to the wide range of temperatures experienced in Canada, engineers who construct roads must allow for expansion and contraction of the road surface. The following formula is used to calculate the amount of expansion E to allow for: E= kL(T-t), where k is the constant of expansion for the road surface. L is the length of the section of highway in metres. T is the temperature of the air in degrees Celsius. t is the temperature, in degress Celsius, at which the section of the highway was constructed.
Answered by Walter Whiteley.
Cardinality of sets 2001-11-19
From Tania:
  1. Show that the cardinality of P(X) (the power set of X) is equal to the cardinality of the set of all functions from X into {0,1}.

  2. Show that (the cardinality of the natural numbers set) |N| = |NxNxN|.

  3. Show that the cardinality of the set of prime numbers is the same as the cardinality of N+

Answered by Walter Whiteley.
Some functions without numbers 2001-11-16
From A student:
I have a worksheet that is about functions. It doesn't only use numbers. I need help to figure out the function and the solution to how the answer is solved.
Answered by Claude Tardif.
Subsets of a countably infinite set 2001-11-14
From Tania:
How could I show (and explain to my son) that any countably infinite set has uncontably many infinite subsets of which any two have only a finite number of elements in common?
Answered by Claude Tardif.
A cubic 2001-11-10
From Louise:
x3 + 9x2 - 7x - 63
Answered by Penny Nom.
Asymptotes 2001-11-09
From Frank:

given the function:

f(x) = (x2) / (x-1)

the correct answer to the limit of f(x) as x approaches infinity is:

y = x+1

all math references point to this answer and the method they all use is long division of x-1 into x2

however if one were to multiply both the numerator and denominator by 1/x and then take the limit, one gets:

y=x

how can the descrepency between the two answers be explained?


Answered by Chris Fisher and Penny Nom.
A 3 dimensional 5 pointed star 2001-11-08
From Kent:
I am looking for a formula that will give me a layout for a 3 dimensional 5 pointed star. I want to form it out of sheet metal, using 5 polygons and soldering them at the apex. Can you please help me with this? I would like to be able to give the formula the height of the star from the bottom two points to the top point and also how deep the star is. Thank you very much!
Answered by Judi McDonald.
Manufactures golf balls 2001-11-06
From Kevin:
A company manufactures golf balls. Golf balls are each numbered consecutively 0 to 4, and then the numbering starts over with zero again. Every 12th golf ball is yellow. Every 8th golf ball is tested for defects. The company produces 120,000 golf balls in an eight-hour shift.

A. How many yellow golf balls are tested for defects during the day? Explain your reasoning.

B. How many of these yellow balls that are tested are imprinted with the numeral 2? Explain your reasoning.


Answered by Penny Nom.
2=the square root of (2 + the square root of (2 + the square root of (2 +...))) 2001-11-05
From Cynthia:
justify algebreically, that:

2=the square root of 2 + the square root of 2 + the square root of 2 + the square root of 2 + the square root of 2 + and so on, .......


Answered by Penny Nom.
Squares of negative numbers 2001-11-03
From Susana:
I wanted to know if I can square a negative number..?
Answered by Leeanne Boehm.
3pi/2 - pi 2001-10-29
From Karen:
I need to simplify this

a) 3pi/2 - pi


Answered by Penny Nom.
A sample size problem 2001-10-28
From Charles:
The U.S Transportation Dept. will randomly sample traffic reports to estimate the proportion of accidents involving people over the age of 70. The Dept. has no advance estimate of this proportion. how many reports should the dept select to be atleast 97% confident that the estimate is within .01 of the true proportion?
Answered by Harley Weston.
Three problems 2001-10-28
From Brenda:
  1. Joseph is planting bushes around the perimeter of his lawn. If the bushes must be planted 4 feet apart and Joseph's lawn is 64 feet wide and 124 feet long, how many bushes will Joseph need to purchase?

  2. The cost of a long distance phone call is $1.50 for the first two minutes and $0.60 for each additional minute. How much will Maria pay for a 24 minute phone call?

  3. Find the next three numbers in the pattern. 1,3,7,15,31,___,____,___.

Answered by Penny Nom.
Why do you flip the inequality sign? 2001-10-26
From Sean:
I have a question about an inequality problem. Here it is: Solve and graph 5 - 3x => 17. (=> is greater than or equal). Please tell me why you flip the inequality sign when divinding by a negative number.
Answered by Penny Nom.
An algebraic fraction 2001-10-25
From brandi:
4x - 5
_____ =4
3-7x

Answered by Penny Nom.
Water in a conical tank 2001-10-20
From Sarah:
The problem: Water flows into a conical funnel at a continuous rate of one gallon per minute (One gallon = 231 Cu.In.). The height of the funnel is 5" and the diameter is 8".

The 1st formula: I need to develop a formula that will give the volume, in cubic inches, of the water in the funnel at any time t (in seconds). V = f(t).

The 2nd formula: I need to develop a formula that will give the height of the water in the funnel at any time t (in seconds). h = f(t).


Answered by Penny Nom.
Symmetry of f(x) = ax^n 2001-10-18
From Mohammed:
a function of the form f(x)=axn, where a doesn't equal 0 and n is a positive integer is called a power function . how is the exponent in the equation of a power function related to the symmetry of its graph?
Answered by Penny Nom.
A function or not a function? 2001-10-18
From Christian:
Which are the following are not functions?
  1. y = x2

  2. y = x3

  3. x = y2

  4. y = 3

Answered by Leeanne Boehm.
Dimensions of a frame 2001-10-16
From Rachel:
A rectangular picture frame has a perimiter of 44.2 cm.The width of the frame is seven tenths of its length what are the dimensions of the frame?
Answered by Penny Nom.
Proof by induction 2001-10-16
From John:
Can you help me with any of these?
  1. For any natural number n > 1, prove that

    (4n) / (n + 1) < [(2n)!] / [(n!)2].

  2. For any natural number n > 1, prove that

    1/sqrt(1) + 1/sqrt(2) + 1/sqrt(3) + ... + 1/sqrt(n) > sqrt(n).

  3. For any natural number n and any x > 0, prove that

    xn + xn - 2 + xn - 4 + ... + x-n >= n + 1.

Answered by Penny Nom.
Some algebra 2001-10-15
From James:
I cannot figure these out I was wondering if you could help me? I have no one to answer my questions.
  1. (7x2 – 3yz)2 – (7x2 + 3yz)2

  2. Use Pascal’s triangle to expand (2x – y)4

  3. 8x3 y - x3 y4

  4. (m + 3n)2 – 144

  5. 12x4 y – 16x3 y2 – 60x2 y3

  6. p3 q2 – 9p3 + 27q2 – 243

Answered by Peny Nom.
Rules of exponents 2001-10-14
From Carissa:
how do you work this out? Investigate the relationship between a,b,c and d if 2a*2b=4c/4d?
Answered by Leeanne Boehm.
4 sinx cosy = 1 2001-10-10
From A student:
How would i differentiate the following example in terms of t (x and y are functions of t)

4 sinx cosy = 1


Answered by Claude Tardif.
Acres 2001-10-10
From Allison:
how many feet are there in an acre?
Answered by Chris Fisher.
Ratio and proportion 2001-10-10
From Steve:
Where would you use a proportion and/or a ratio in a real life job or problem.
Answered by Leeanne Boehm and Walter Whiteley.
Lucas' theorem 2001-10-09
From Tania:
How could I demonstrate: nCp is congruent to floor(n/p) (modulo p)? where rCk is a binomial coefficient, rCk = r(r-1)...(r-k+1)/k(k-1)...1, and p is a prime number
Answered by Richard McIntosh.
Finite differences 2001-10-08
From Murray:
My name is Murray and I am a 10th grade student. Me and my friend have recently discovered and proved a theorem of a relitively advanced nature. It is that the the nth difference of an nth degree equation = n! times the coefficient of the highest power. One of my teachers said this theorem is part finite and that she thinks she has seen it before, but she does not remember what it is called, at what level it is taught, who discovered it or when it was invented. I would greatly appretiate answers to any of these questions.
Answered by Chris Fisher.
Simplify the expression 2001-10-08
From Natalia:
simplify expression

4(x-7)+(-5x)=-10


Answered by Leeanne Boehm.
Percentage in our daily lives 2001-10-08
From Natasha:
What are the uses of percentage in our daily lives?
Answered by Leeanne Boehm.
The amount of gravel needed to fill a hole 2001-10-06
From Rhett:
My name is Rhett and my problem is...I am a contractor and I am having problems determining the amount of gravel(in tonnage) needed to fill this hole,the measurements are 60' in length,22' in width,and 14' in height....the problem is that it is not a usual box shaped hole or in this case a rectangle shaped hole,but more of a triangular shaped hole.I refer to the triangular shape as the interior shape of the crater not the exterior,which is shaped like the above dimensions.If you could help it would be greatly appreciated.P.s.This was a retaining wall project,built by myself, in front of a sloped end of a yard,so if you could imagine a 14" high hill with a landslide looking face,then build in your mind a three sided rectangle in front of it using the hill side that slopes as the fourth side of the rectangle,then you may be able to imagine what I mean as a triangular shaped interior of the hole.
Answered by Penny Nom.
A polygon with five sides 2001-10-03
From A student:
I was asked a question in 5th grade today that goes: i am a polygon with five sides and 2 right angles what do i look like and what is my name
Answered by Penny Nom.
(x^2-5x-6)/(x-6) 2001-10-02
From Bill:
given f(x) = (x2-5x-6)/(x-6) find f'(6).
Answered by Harley Weston.
Proof by induction 2001-09-30
From Kyle:
I'm trying to learn induction and I need to see how this done please help with this problem...

20 + 21 + 22 +... + 2n = 2n+1 -1 is true whenever n is a positive integer.


Answered by Penny Nom.
Inverting a function 2001-09-30
From Brandie:
Could you please tell me what is the basic guideline for inverting a function

Example:
S(R)=2PiRal
V(R)=PiR(squared)bl
R(V)=?


Answered by Claude Tardif.
1:5 2001-09-27
From Sandy:
A colleague and I have a disagreement about dilutions. I seem to remember that I was taught (although it was a long time ago) that a RATIO was written thusly: 1:5 , and read " one part to five parts", for a total of 6 parts. On the other hand, a fraction is written: 1/5 and is read" one part in five" for a total of 5 parts. My colleague says that the ratio reads "one part in five" and is essentially the same as a fraction. Which is right? Have I been doing dilutions wrong all this time? I teach Animal Nursing and need to know where I can find a consise answer to such questions for making up medicated fluids, etc.
Answered by Penny Nom.
GCD and LCM 2001-09-24
From David:
Find the GCD adn LCM of the following numbers. Assume p 2 to the power p, 3 to power q, 5 to power r

and

2 to power q, 3 to power r, 7 to power p


Answered by Penny Nom.
Ciefficient of variation 2001-09-23
From Carmen:
I have a question from my OAC finite class. I've come across a problem with the coefficient of variation. I have taught my students that there are no units for coefficient of variation and it can be expressed as a percent. So, for example, a set of data with mean of 5 and standard deviation of 100 would have a CV of 5%. But what happens in this situation: the mean is 4meters and the standard deviation is 0.7mm. Is the CV 1.75% or 0.00175% or 0.0175%? I've had some students change leave the units as is, change them both to mm or change them both to meters...so which is it and why?
Answered by Penny Nom.
4 digit combinations 2001-09-22
From Laura:
I need all of the 4 digit combinations using the numbers 0-9.
Answered by Harley Weston.
The square root of 20 2001-09-18
From Dianna:
How do you simplify a square root? My daughter tells me that the square root of 20 simplified is 5root4
Answered by Penny Nom.
4x(2x-1)+(2x-1)2 2001-09-17
From Nick:
how do u solve 4x(2x-1)+(2x-1)2
Answered by Penny Nom.
A polygon 2001-09-11
From Sueling:
what is the smallest polygon. what is a polygon.
Answered by Penny Nom.
Three fractions 2001-09-10
From Kathleen:
a/bc + d/ef + g/hi = 1

Each of three fractions has a one-digit numerator and a two digit denominator. The three fractions together add up to one. Place the nine digits 1-9 into the fractions to make the equation correct.


Answered by Claude Tardif.
The square root of 21/168 2001-09-10
From A student:
can you show me how to do this square root of 21/168
Answered by Penny Nom.
A 91-gon 2001-09-10
From Ashley:
Hi! My name is Ashley and I am an 8th grader and I couldn't find the name of a polygon with 91 sides. i was a question from my math teacher.
Answered by Penny Nom.
The best of 7 2001-09-08
From Michelle:
Professional basketball, hockey, and baseball championships are decided on the best 4 out of 7 games. The first team to win four games wins the championship. In how many possible ways could the team win the championship series after winning the first game?

 Example:  
Game   1    2    3    4    5     6     7         
Result W    L    L    W    W     W 

Answered by Leeanne Boehm.
Don't worry too much about your difficulties in math... 2001-09-07
From Matt:
Do you know who said,"Don't worry too much about your difficulties in math, I assure you that mine are still greater"? Thanks for your help. this for extra credit and I can't find it ANYWHERE!
Answered by Penny Nom.
Sharing a donut 2001-09-06
From Amanda:
You have invited 11 people over to your house one day, and your friends are hungry. You go into the kitchen and find out you only have 1 donut (with a hole in the middle). In order to feed 12 people (including you), you must cut the donut into 12 pieces with only using a straight knife and cutting 3 times. This is NOT a trick question.
Answered by Claude Tardif.
Fourier transform 2001-08-07
From Adbul:
  1. Sir, we have the Dirichlet's condition for the Fourier transform : " The function should be integral over the real line " But why we are we neglecting this for example when we take the Fourier transform of an impulse train?

  2. Suppose we want to travel from one corner of a square of side 'a' to the diagonally opposite corner. We can travel along the sides which gives a pah length of '2a'. We can also do it in steps as shown below:

      _ | |_PATH |   |_ |_____| 


    Suppose The step size =DELTA x Then the path length will be again '2a'. Now in the limit DELTA x -->0 again we get '2a' But when we take the limit we get the straight line diagonal whose length is 'SQRT(2)X a' Where did I go wrong?

Answered by Chris Fisher.
How far does the fly fly? 2001-08-07
From Harold:
 
6 MPH                            4 MPH 
Rachel  ----------------------     Eli                        
                  10 Miles apart  

The fly is on Rachels handlebars. The fly is scared so it flys back and forth at 20 MP H. How far has the fly flown when Rachel and Eli meet?

f

Answered by Penny Nom.
Conformable matrices 2001-08-05
From Wayne:
Can someone explain the concept of conformable matrices in a way that is easy to understand ? One definition says to multiply matrices rows and columns must conform, ie, 5 x 3 matrix times a 3 x 5 matrix. In the next example, however a 8 x 1 matrix and a 8 x 3 matrix are said to be conformable!
Answered by Steve Kirkland.
Linear regression 2001-08-01
From A student and a mother:
My mom is taking a correspondence course in Simple Linear Regression and Correlation Analysis and we've been arguing about the relationship between the correlation -rxy and standard estimate of error-Sy.x. I took statistics last year in high school and I remember something about the Sy.x being proportionate to the r. Are they inversely related, directly related, not related, or can only range from 0 to 1.00? Her book doesn't say very much and I believe they are inversely related. She says they're directly related.
Answered by Chris Fisher and Penny Nom.
Intersection of perpendicular cylindrical surfaces 2001-07-31
From Charlie:
Please consider two right circular cylinders, perpendicular one to the other, and of unlike radii in a 3 dimensional Cartesian space with mutually perpendicular x,y,z axes. If one cylinder is centered on the y axis with radius ra, and the other on the z axis with radius rb, then the expression for the first surface would be x2 + z2 = ra2, y = any number. Likewise, the second cylinder's surface would be x2 + y2 = rb2, z = any number.

It is my intent to define the curve at the intersection of these two cylindrical surfaces. From sketching the conditions it appears that this intersection resembles an ellipse folded about its minor axis.


Answered by Chris Fisher.
e^pi > pi^e 2001-07-27
From Dusty:
What book(s) contain a proof that ePi > Pie? I think it might be in Problems in Analysis published by Springer-Verlag but I have not been able to check.
Answered by Chris Fisher.
Margin of error 2001-07-13
From Jim:
If a survey was completed by four different groups of people as indicated below what would the over-all margin of error be for this survey given the fact that the number of people in each group were different?
 
Group   # polled   Margin of error 
A.           779           +/- 4% 
B.           315           +/- 7% 
C.           323           +/- 6% 
D.           254           +/- 9% 

Answered by Andrei Volodin.
The speed of the boat 2001-07-12
From Sharon:
A motor boat is travelling in a southeasterly direction in water that is flowing from the south at 2km per hour. Show that the speed of the boat is (6 times the square root of 2) km per hour, given that it can travel at 10km per hour in still water.
Answered by Penny Nom.
Cakes and ribbons 2001-07-05
From Jennifer:
  1. A cake recipe calls for 3 1/4 cups of flour. You already put in 1 3/4 cups. How much more should you add?

  2. You have 3/4 yard of ribbon. How many 1/8-yard pieces can you cut the ribbon into?

Answered by Penny Nom.
Arclength of an ellipse 2001-07-03
From A hobbyist:
What is the equation (with the length of the arc as a variable) for one quadrant of the ellipse,...
Answered by Claude tardif.
A confidence interval 2001-06-28
From Murray:
An investigator wants to find out of there are any difference in "skills" between full and part time students. Records show the following:
Student            Mean Score         Std Dev        Number
----------      -----------------     ----------     -----------
Full time              83                   12          45
Part time               70                   15          55
Compute a 95% confidence interval for the difference in mean scores.

Answered by Andrei Volodin.
A factoring problem 2001-06-14
From A student:
What is the factoring of x squared -7+6 equal
Answered by Penny Nom.
Primes and square roots 2001-06-14
From Paul:
I have a bit of a math problem. It has to do with determining if a very large number is a prime. One method entails dividing the number by every smaller prime number. If any divide into it, it's not a prime. This would be a big job if the number was something like 400 digits long. Another way I read about was to take the square root of the number and test all the primes less than its square root. The explanation went like this: "When a number is divided by another number that is greater than its square root, the result is a number smaller than the square root. For example, the square root of 36 is 6. Dividing 36 by 2, a smaller number than 6, gives 18, a number that is larger than the square root. To prove that 37 is prime it is only necessary to divide it by primes less than 6, since if it had a prime factor greater than 6, it would have to have one less than 6 as well."

I understand the explanation, up to the last sentence. I fail to see the underlying logic. Why if a prime factor exists below the square does one have to exist above the square too? The number 40 can be divided by the prime 2, a number below its square root, but no other primes can do this above its square root. Have I missed something? What's the logic here?


Answered by Claude Tardif and Penny Nom.
Exponential form 2001-06-06
From A student:
Hello can u please answer my question what is the exponent form of 8x8x8x8
Answered by Penny Nom.
Securing a plank in four dimensions 2001-06-06
From Becca:
If it takes two nails to secure a plank from rotating on a wall in the third spatial dimension, how many nails would it take if you were attempting to secure the board from rotating in the fourth spatial dimension?
Answered by Walter Whiteley.
The shortest distance from a point to a line 2001-06-05
From Kat:
Find the shortest distance from the given line. Round to the nearest tenth. (-2,-1) and x+5y+20=0
Answered by Penny Nom.
Four digit numbers from 1,2,3,4,5,6,7,8 2001-05-31
From Katie:
Okay.......here's my question...how many different ways can you form a four digit number out of these digits..1,2,3,4,5,6,7,8? This is how my teacher said to do this.... 8*8, 7*7,6*6,5*5,4*4,3*3,2*2,1*1. then she said to add up the products and to multiply by 7. is this correct, and if not how can you figure this out.
Answered by Penny Nom and Claude Tardif.
Three goldfish 2001-05-30
From Nathan:
A man has three goldfish. When the youngest goldfish was born, the oldest fish was three times the middle fish's age. Nine years ago the oldest fish's age was the sum of the two other fish's ages. How old are the three goldfish?
Answered by Penny Nom.
New Quebec reform 2001-05-24
From Marie-Andrée:
I am a middle school teacher beginning to adapt to the new Quebec reform. I am regularely doing projects in écologie and géographie in French immersion based on the international schools way of thinking (What form does it have? How does it work? What is its history? How does it change? How is it linking to our lives? What is our responsibility about that fact or that thing?) and I have real difficulty to make it work for math. So now, I am looking for math concepts (8) which would link to the same way of thinking. What are your suggestions?
Answered by Claude Tardif.
The diameter of an oval 2001-05-23
From Tim:
Is there a such thing as a diameter of a oval? If not, is there a way to get the circumference?
Answered by Claude Tardif and Penny Nom.
Harmonic numbers 2001-05-23
From Leslie:
The harmonic numbers Hk, k = 1,2,3.....are defined by Hk = 1 + 1/2 + 1/3....1/k

I am trying to prove by mathematical induction:

H2n >= 1 + n/2 , whenever n is a nonnegative integer.

H8 = H23 >= 1 + 3/2

Can you help?


Answered by Harley Weston.
A 6 character field 2001-05-16
From A parent:
I have a 6 character field that can use any combination of alpha-numeric characters (A-Z, 0-9). How many total combinations are possible?
Answered by Penny Nom.
Quadratics 2001-05-16
From John:
I am in the final stages of a math project and I need to interview an expert for the last part. Please try to answer at least a few of these questions.
  1. How do you use quadratic equations in your everyday life?
  2. Do you find being a math expert very helpful in life?
  3. Is the quadratic equation useful to you?
  4. Why did you decide to become a math expert?
  5. What do you think is the most important function of the quadratic equation?

Answered by Harley Weston.
25 y^2 + 20 y z - 4 z^2 2001-05-15
From Kimmy:
i don't know how to factor the trinomial 25y2 + 20yz - 4z2. I can get it, but i get messed up on getting the minus 4 at the end. all i get is a plus 4. Can it even be factored.
Answered by Penny Nom.
A pile of sand 2001-05-14
From Gul:
  1. Sand for use on icy roads is stored in a conical pile 14.2 m high and with a base diameter of 34.4 m
    • calculate the volume of the pile

    • if one sander can take 6.9 m of sand, how many sanders can be filled from the pile?

Answered by Penny Nom.
Manifolds 2001-05-14
From Thierry:
Maybe you could help me, because I have serious problems for the translations of an English mathematic word which is "MANIFOLD". It's impossible to have a clear translation in French. Do you have a solution?
Answered by Claude Tardif.
Simplifying algebraic expressions 2001-05-12
From A student:
How do you simplify algebra expressions?
Answered by Penny Nom.
The angles in a triangle 2001-05-11
From Nikki:
Find the measure, to the nearest degree, of each angle of a triangle with sides of the given lengths.

26, 35, 40


Answered by Penny Nom.
A quadratic function 2001-05-10
From A parent:
A quadratic function is given by f:x------ax2+b

If the two points A(2, 2) and B (0, -8) are on the graph of the function, then find a and b.

Answered by Penny Nom.
Greatest Common Factor 2001-05-09
From Nicole:
What is the Greatest Common Facter for:

126,261
1008,1080
546,1995
128,256
and
255,256

Answered by Penny Nom.
National consumption function 2001-05-09
From Brian:
If consumption is $11 billion when disposable income is 0 and the marginal propensity to consume is dC/dy = 1/(2y+4)1/2+0.3(in billions of dollars), find the national consumption function.
Answered by Harley Weston.
Maximize profit 2001-05-09
From Brian:
The marginal cost for a certain product is given by MC = 6x+60 and the fixed costs are $100. The marginal revenue is given by MR = 180-2x. Find the level of production that will maximize profit and find the profit or loss at that level.
Answered by Harley Weston.
Dividing fractions 2001-05-09
From Rina:
I just wanted to ask if you could help me in math. See I'm having a test soon and its on Dividing Fractions and I just don't get it. My math teacher says that I'll be just fine but I failed my math quiz. I went to ask eric but they could help me so they told me to go to you. So here I am asking you if you could help me.
Answered by Penny Nom.
The average value of a continuous function 2001-05-08
From Esther:
The average value of a continuous function y = f(x) on the interval [a,b] is given by ________________?
Answered by Harley Weston.
Teaching square roots 2001-05-08
From Katie:
My friend and I are doing a project. We have to teach our class about square roots. What is the easiest way to teach square roots to a class?
Answered by Penny Nom.
Adding and subtracting rational expressions 2001-05-03
From Donna:
Adding and subtracting Rational expressions. I am in grade 10 and I am a student here is an example of the questions:

1/(x+1) - 1/(x-1) = ?


Answered by Penny Nom.
A sequence defined recursively 2001-05-01
From A student:
A sequence s is defined recursively as follows:

s0=1
s1=2
sk=2sk-2 for all integers

- Compute s2,s3,s4... to guess an explicit formula for the sequence sk.
Answered by Penny Nom.

A sequence of even terms 2001-04-29
From A student:
A sequence c is defined recursively as follows:

c0 = 2
c1 = 4
c2 = 6

ck= 5ck-3 for all integers

Prove that cn is even for all integers.


Answered by Leeanne Boehm and Penny Nom.
Choosing a car 2001-04-28
From Ashley:
THE CAR DEALERSHIP IN TOWN OFFERS 32 DIFFERENT MODELS OF VIHICLES.EACH MODEL HAS A CHOICE OF EIGHT INTERIOR COLORS,EIGHT EXTERIOR COLORS,AND ALSO THE OPTION OF AUTOMATIC OR MANUAL TRANSMISSION. HOW MANY COMBINATIONS ARE POSSIBLE?
Answered by Penny Nom and Andrei Volodin.
Triangles and fractions 2001-04-27
From Constance:
My name is Constance and I am thirteen years old (I am a student). The question that I am queering about I don't understand why you do ONE HALF x the base x the width WHEN YOU WANT TO FIND the area of a triangle? My second question is if you multiply one half and 10 together why does it come out as 5?
Answered by Penny Nom.
A confidence interval 2001-04-26
From Kim:
A poll asked 1528 adults if they were in favor of the death penalty, 1238 said yes, find 99% confidence level for percent of all adult who are in favor of the death penalty.
Answered by Andrei Volodin.
Squares of Fibonacci numbers 2001-04-24
From Vandan:
What discoveries can be made about the sum of squares of Fibonacci's Sequence?
Answered by Penny Nom.
Five 2001-04-21
From Billie:
If One is single, two is twin, three is triple, four is quadruple, five is ????? what is five and where would I find the rest of the numbers,six, seven, eight, nine, etc.
Answered by Penny Nom.
Repeating decimals 2001-04-21
From Sarah:
Hi, I'm working on a project for school. The theory I choose was "When turned into a fraction, a repeating decimal has a denometor that is a multiple of three." I have a couple of questions about this topic. My first question is, have you ever heard of this, and what can you tell me about it? My second question is, when I was testing this theory I came across .999... now, when this is a fraction it is 9/9 which is equal to one. The denometor is a multiple of 3, but it's a whole number. I don't understand how a decimal can be equal to a whole number since a decimal is a piece of a whole number. Please don't just show me a math problem, I don't want to see a math problem. I want to see an explanation of this theory and the decimal .999...
Answered by Penny Nom.
A geometry proof 2001-04-18
From Melissa:
Extend the bisectors of angle A, angle B, and angle C of triangle ABC to meet the circumcircle at points X, Y, and Z respectively. Show that I is the orthocenter of triangle XYZ.
Answered by Chris Fisher.
Differentiation 2001-04-17
From Esther:
Could you please tell me what the first derivative is of the following:

y = 2/(2x+e2x)

Is it (1+xe2x)/(2x+e2x)2 or perhaps -4(1+e2x)/(2x+e2x)2 ? I am a little confused between the two!


Answered by Harley Weston.
The domain of a function 2001-04-08
From Mina:
Let f(x) = (2x2+3x-17)/(x+5)
What is the domain of f? What are the values of x for which f'(x) does not = 0?

Answered by Harley Weston.
Five fourths 2001-04-08
From Linda:
Can you have 5/4 of something?
Answered by Penny Nom.
Equivalent fractions 2001-04-08
From A student:
which of the following pairs of fractions are equivalent?
questions a-f

(a) 1/5 and 6/30
(b) 4/9 and 16/27
(c) 6/10 and 30/50
(d) 15/20 and 48/64
(e) 2/3 and 33/100
(f) 12/32 and 3/16


please show how u got the answer thank you.

Answered by Penny Nom.
f(x), f(x) + 2, f(x +2) 2001-04-04
From Monica:
Could you explain to me how one should go about graphing functions such as f(x), f(x+2), and so on. Also, how should you explain things such as constants and relationships among functions?
Answered by Penny Nom.
Two integrals 2001-04-03
From Jim:
I'm having trouble with these integrals. Can you help me out?

1)the integral of:

x5 arctan x dx

2)the integral of:

2x5 + 9x4 + 19x3 + 13x2 - 5x - 25
----------------------------------------------- dx
x4 + 4x3 + 5x2


Answered by Claude Tardif.
Volume and surface area of a cylinder and a triangular prism 2001-03-31
From Kevin:
I was wondering how do you find the volume and surface area of a cylinder and a triangular prism.
Answered by Penny Nom.
Borrowing from zero 2001-03-30
From Nikki:
Here is the problem:

5,033.56
- 866.20

Now I know this equals 4,167.36 which I figured out on the calculator just to make sure. But when I did it by hand, I kept ending up with 5,167.36, which I knew was impossible. What am I doing wrong??


Answered by Leeanne Boehm and Claude Tardif.
Stem-and-leaf plot 2001-03-29
From Kelly:
I'm tutoring my 5th grade nephew, and he needs to correct test questions regarding a stem - and - leaf plot. He left his book at school and I'm not familiar with the term. Can you explain it to me so that I can help him?
Answered by Penny Nom.
A sugar cane crop 2001-03-27
From A student:
a sugar cane farmer harvested only 1/4 of his crop. he sold 2/3 of his harvest to the mill. what fraction of the total crop did he sell.

darens recipe for florida fudge calls for 5/8 cup of sugar. if he wants to make only 1/2 of the recipe how much sugar should he use.


Answered by Penny Nom.
Two ferry boats 2001-03-25
From Gil:
Two ferry boats leave from opposite shores. One is faster than the other. They meet 720 yards from the nearest shore. They proceed to destination and upon returning they meet 400 yards from the other shore. What is the exact width of the river.
Answered by Penny Nom.
Airflow in windpipes 2001-03-25
From Ena:
The volume of air flowing in windpipes is given by V=kpR4, where k is a constant, p is the pressure difference at each end, R is the radius. The radius will decrease with increased pressure, according to the formula: Ro - R = cp, where Ro is the windpipe radius when p=0 & c is a positive constant. R is restricted such that:
0 < 0.5*Ro < R < Ro,
find the factor by which the radius of the windpipe contracts to give maximum flow?

Answered by Harley Weston.
A famous landmark 2001-03-23
From Corinne:
A family is traveling due west on a road that passes a famous landmark. At a given time the bearing to the landmark is N 62 degrees W, and after the family travels 5 miles farther the bearing is N 38 degrees W. What is the closest the family will come to the landmark while on the road?
Answered by Harley Weston.
Factoring 2001-03-22
From Kaleena:
Factor this equation: 5x7-10x5+4x3-8x
Answered by Harley Weston.
Why exactly can't you divide fractions? 2001-03-22
From Dennis:
Why exactly can't you divide fractions?
Answered by Penny Nom.
Timing with hour glasses 2001-03-20
From Nathan:
How can a chef use an 11 minute hour glass and a seven minute hour glass to time a vegtable that needs to be steamed for 15 minutes.
Answered by Leeanne Boehm.
Systems of equations 2001-03-16
From joy:
How do u solve problems using systems of equations?
~ finding x and y~

ex:

26 = 3x - 2y
42 = 4x + y


Answered by Penny Nom.
How can you prove a quadrilateral to be a parallelogram? 2001-03-16
From Joy:
How can you prove a quadrilateral to be a parallelogram?
Answered by Walter Whiteley.
Names of the polygons 2001-03-15
From Sami:
I was asked to find out what the names for different polygons were. The only two I cannot find are the names for a polygon with 30 sides and a polygon with 40 sides. I would really appreciate it if you could help me. Thank you very much.
Answered by Harley Weston.
A jogger 2001-03-12
From Bill:
At time t=0 a jogger is running at a velocity of 300 meters per minute. The jogger is slowing down with a negative acceleration that is directly propotional to time t. This brings the jogger to a stop in 10 minutes.

a) write an expression for the velocity of the jogger at time t.

b) what is the total distance traveled by the jogger in that 10-minute interval.


Answered by Harley Weston.
The angle of elevation 2001-03-08
From Jeffrey:
At a Certain time, a vertical pole 3m tall cast a 4m shadow. What is the angle of elevation of the sun?
Answered by Harley Weston.
problem of the week 2001-03-06
From Peggy Allan:
My son has been challenged with "the problem of the week" and I am unable to assist him in finding a reasonable solution.

Problem 1,

Julio needs to draw a line segment 15cm long. He does not have a ruler. He does have some sheets of letter size paper 28 X 21.5 cm. Describe how Julio can use the paper to measur 15 cm.

Answered by Claude Tardif.
Powers 2001-03-04
From A student:
Hey, can you show me how you do ..

(2xy)to the 3rd power (x) to the 2nd power?


Answered by Penny Nom.
The price of a carpet 2001-03-01
From Amber:
i have to calculate the square footage of carpet for me room and im not quite sure how to do that. the carpet is $2.08 per sq. ft. and my room is 12x11ft.
Answered by Penny Nom.
Fibonacci 2001-02-28
From Shona:
We have been doing pattern finding with him, talking to a friend he mentioned the "Fibbinacci Series" ?, while I have tried to find a bit about it, how works etc.. what it is about, I have not really found out much, what I have I feel is way beyond him, but am still curious to know the basics of it myself. Would you be able to tell me in laymans language. Would be very much appreciated.
Answered by Claude Tardif.
Domain and Range and Zeros 2001-02-27
From Beth:
1. f(x)=15-2x-x2

2. g(t)=square root of 4-t2

3. y=t-3/t2-t-6


Answered by Penny Nom.
Solve for two variables 2001-02-25
From A student:
How do I solve for %1 and %2 in the following formula when T1, T2 and T3 are known? %1 and %2 are ratios of the same element, therefore %1 + %2 = 100%

(%1 x T1) + (%2 x T2) = T3

Answered by Penny Nom.
Difference of Squares 2001-02-22
From Bruno-Pierre:
I noticed the other day that if you substract two consecutive squared positive numbers, you end up with the same result as if you add up the two numbers.

Ex. 5 and 6 (2 consecutive positive numbers)
52 = 25
62 = 36
36 - 25 = 11 (Substraction of the squared numbers)
5 + 6 = 11 (Sum of the numbers)

A more algebric view:
a2 - b2 = a + b where a and b are consecutive positive positive numbers. (b = a + 1)

I wondered if this rule had a name, and who discovered it.


Answered by Penny Nom.
The domain of the derivative 2001-02-22
From Wayne:
I know that the domain of f'(x) is a subset of the domain of f(x). Is it necessarily true that the subset will have at most one less element than the domain of the original function?
Answered by Harley Wesston.
Mr. Moser's roof 2001-02-21
From Michelle:
Mr. Moser is planning to replace the roof of his home. He needs to order a pack of shingles. Each pack covers 100 sq. ft. of roof. Without a ladder, Mr. Moser can not climb to the roof to measure it. Instead, he measures his attic and finds it to be 40 ft. long, 24 ft. wide, and 5 ft. high at the peak of the roof which is in the center of the house. Although the roof is even with the side walls, he estimates the roof line continues 1.5 ft. beyond the front and back walls. How many full packs of shingles should Mr. Moser order to cover his roof?
Answered by Penny Nom.
Faces 2001-02-21
From Sandy:
How many faces are there on a sphere?

What are the faces of a cone?

What is the definition of a "face" of a 3D object?


Answered by Walter Whiteley.
Math game 24 2001-02-21
From Kristi:
We have a game we play in math class called 24 game. In it each card has 4 numbers on it. Each number can only be used once, in any order, using multiplication, subtraction, division, or addition. We are stuck on one and wondered if you could help. The four numbers are: 3, 3, 5, 7. And they must equal 24. Any clues?
Answered by Claude Tardif.
The law of cosines in the real world 2001-02-21
From Hope:
Do you have any examles and/or labs that show how the law of cosines is used in the real world?
Answered by Harley Weston.
Undecagon 2001-02-21
From Karen:
I need to know what kind of a polygon an undecagon is and how many sides it has. This is a problem in my math book.
Answered by Penny Nom and Michael Tsatsomeros.
A pole next to a building 2001-02-20
From Robert L. Haun:
A pole 100 feet tall is erected next to a building 10' X 10' X 10' . Object is to cut off the pole so the upper end remains touching the part cut off, while the cut-off portion touches the cutoff point, the edge of the building and the ground. Note assume the pole is touching the building and is "very thin" What is/are the cutoff points?
Answered by Penny Nom.
Cubic feet 2001-02-20
From Frances:
I'm trying to calculate cubic feet from the dimensions of a machine given in inches.

Width = 32"
Height = 79.5"
Length = 91.5"
Weight = 3920 lbs

Please help? Obviously I've been away from school too long!!


Answered by Penny Nom.
Law of cosines 2001-02-20
From Emily:
I missed a few days of class and I can't figure out how to solve Law of Cosines problems. I have the notes and formulas but I can't figure out how to do the math involved to answer the problems. I am also getting confused about how to use degrees and seconds in the Law of Sine and Cosine. I can't seem to get the correct answers and I don't really know how to solve them and enter them into my TI 86.
Answered by Harley Weston.
Difference of squares 2001-02-20
From Janna:
Hi! I was just wondering how you would factor x2 - 9y2.
Answered by Harley Weston.
Delivering firwood 2001-02-20
From Jackie:
Two college students earn extra money on weekends by delivering firwood in thier pickup truck. They have found that they can sell x cords per weekend at a price of p dollars per cord, where x=75-3/5 p. The students buy the firewood from a supplier who charges them C dollars for x cords to the equation C=500+15 x+1/5 x2.

a) find a function f such that P=f(p), where P dollars is the profit per weekend for the students if they charge p dollars per cord.

b)find the profit P dollars if p=$95

Answered by Penny Nom.
Adding functions 2001-02-19
From Jackie:
f(x) = x2 + 1; g(x) = x2 - 1

find
a) ((f+g) of h)) (x)
b) ((f-g) of h)) (x)
c) ((f of h) - (g of h)) (x)


Answered by Penny Nom.
Some fractions 2001-02-17
From Lady Bird:
Mr. Bob has 3/4 of a bottle of medicine left. He takes the medicine three times a day. Each dose is 1/16 of the bottle. How many days will it take for Mr. Bob to finish the bottle?

My friend gave me a problem to solve and it is so hard. Can you help me solve it? Here it is: If g- 4.8 = 9.66 and e-1/3 = 1/8, what is the sum of g and e?

Mariko's mother used ribbon to wrap 3 birthday gifts. She had 3 ribbons and each was 72 inches long. She used 8/9 of the first ribbon, 2/3 of the second ribbon, and 5/6 of the the third ribbon. How many feet did she use?


Answered by Leeanne Boehm and Penny Nom.
Differentiation of y = x n 2001-02-17
From Jashan:
i am studying differentation at the moment i have drawn some graphs such as y=x 2. i have found the formula for the gradient of this curve, this being 2x obtained by using differentation, but i need to know the general case for the formula where y=xn in order for me to understand this topic more throughly, i would also like to know how u derived this general formula
Answered by Harley Weston.
A four dimensional object 2001-02-14
From A student:
Can you give me some examples of a four dimensional object you can find around your house?
Answered by Harley Weston.
An order relation 2001-02-12
From Amy:
What is an order relation?
Answered by Claude Tardif.
Part of a part 2001-02-11
From Katie:
Hello! I was looking at your Fraction questions and since i am pursuing to be an elementry school teacher i feel i don't understand one of your responses. How would you explain this to an elem, midd, and high school student. Even me myself-in college really does not understand. The question was: When you multiply two positive fractions less than 1, how does the answer compare to the size of the fraction? Why?
Answered by Penny Nom.
What is wrong with these probelms? 2001-02-09
From Cindy:
Hi, I am a planning on becomming a teacher and i am asked to find out what is wrong with these probelms and how i would go about showing a student what is wrong with them!!

Error in patterns:
13/35=1/5; 27/73=2/3; 16/64=1/4

4/5+2/3=6/8; 2/5+3/4=5/9; 7/8+1/3=8/11

2/3*3=6/9; 1/4*6=6/24; 4/5*2=8/10


Answered by Leeanne Boehm and Penny Nom.
An irregular polygon 2001-02-09
From Jason:
I have a 5 sided irregular polygon I am trying to figure out the area of. There are no right angles in the polygon as far as I can tell. I do not know any angles.
Answered by Chris Fisher.
Buckets of golf balls 2001-02-08
From A student:
A bucket containing 40 golf balls weighs 135 ounces. The same bucket with 20 golf balls weighs 75 ounces. What is the weight of the bucket?
Answered by Claude Tardif.
Find an exprression for f(x) 2001-02-07
From A 12th grade AP Calc student:
Let f be the function defined for all x > -5 and having the following properties. Find an expression for f(x).

i) f ''(x) = 1/ (x+5)1/3 for all x in the domain of f

ii) the line tangent to the graph of f at (4,2) has an angle of inclination of 45 degress.


Answered by Harley Weston.
(-2b+3)(-b-1) 2001-02-06
From Melissa:
Alright, lets say you are multiplying (-2b+3)(-b-1) and you are using "FOIL" first you would get : 2b2+2b-3b-3 then you add like-terms. when multiplying,if the bases are the same, add the exponents...does the same thing apply when you are adding the results of the multiplication even though it's addition?

would the answer be 2b3-3b-3 or would it be 2b2-b-3?


Answered by Penny Nom.
Which fraction is greater? 2001-02-01
From Carol:
Hello My name is Carol I am a student teacher! I am asked to aproach these students on their problems. I am unsure of how to explain to them that these are wrong. Can you help? Thanks.

Iris claims that if we have two positive rational numbers, the one with the greatest numerator is the greatest.

Shirly claim that if we have two positive rational numbers, the one with the greatest denominator is the least. Thanks!


Answered by Claude Tardif.
Why is 0! = 1? 2001-01-30
From Diane:
Every math book always claims that 1!=1 and 0!=1 are givens, and that we should just memorize it. i understand the 1! part, but where is the basis for claiming that 0!=1????
Answered by Walter Whiteley.
Subsets of the natural numbers 2001-01-30
From Christina:
How do I explain why the set of natural numbers (N) cannot be equivalent to one of its finite subsets?
Answered by Penny Nom.
Fractions of fractions 2001-01-30
From Katie:
When you multiply two positive fractions less than 1, how does the answer compare to the size of the fraction? Why?
Answered by Penny Nom.
Factoring 2001-01-24
From Danielle:
4(x+1)2+y(x+1)
Answered by Penny Nom.
Factoring a trinomial 2001-01-24
From Kyle:
I am an eighth grade student.I can not figure out the following problem. 4x2 + 2x - 20
Answered by Leeanne Boehm.
1 + 1 = 1 2001-01-23
From Stephanie:
My friend has this as a bonus question the other day and I want to figure it out. I don't know how 1+1 in any form could equal 1. Please let me know how you come about geting that.
Answered by Claude Tardif.
What number from 1 to 25 has the most factors? 2001-01-23
From John:
What number from 1 to 25 has the most factors?
Answered by Penny Nom.
Square feet and cubic feet 2001-01-21
From Jenny:
How do I convert square feet to cubic feet? If I have 2000 square feet then how many cubic feet do I have?
Answered by Harley Weston.
A prism 2001-01-18
From Nigel Ayling:
What is the mathematical definition of a prism, I am confused by the following definitions as they appear to be contradicto...
Answered by Walter Whiteley.
Greatest common factor 2001-01-17
From A teacher:
I'm a teacher at a local Middle School and I would like the answers for the following questions.

What is the greatest common factor for..

546 and 780

156 and 732

.
.
.

Answered by Leeanne Boehm and Penny Nom.
A problem with distance 2001-01-16
From A student:
For what values of a is the distance between P(a, 3) and Q(5, 2a) greater than the square root of 26.
Answered by Penny Nom.
Polynomials and exponents 2001-01-15
From A student:
I am duing a project in math on polynomials and exponents. I need a real life usage of polynomials and exponents for my project.
Answered by Penny Nom.
Compression by a factor of ? 2001-01-12
From Wayne:
Given an equation like y = (1/2) (x-3)2 + 6, when describing the distortion from the base curve y = x2 is it correct to say there is a vertical compression by a factor of 2, or by a factor of 1/2 ? Which is more correct? I have seen both versions in different high school texts, and I am now unsure which is more correct.
Answered by Harley Weston.
64.5% profit 2001-01-12
From Ethel:
IF I PURCHASE AN ITEM FOR $2.00 AND I WANT TO RESELL IT WITH A MARGIN OF GROSS PROFIT OF 64.5% WHAT FORMULA DO I USE TO ARRIVE AT THAT RESELL FIGURE?
Answered by Penny Nom.
Blood flow and age 2001-01-11
From Gayle:
I don't understand the second part of this problem Here is the first part that was correct. A 35yr old has 100% blood flow. A 65yr old has 64% blood flow. Find the rate of change per yr in the percent of blood flow. (This part is correct) m=(64-100)/(35-65)= -36/30= -1.2 per year of blood flow.

Now, here is the second part I DON'T UNDERSTAND! .Let b represent the percent of blood flow and a represent a person's age. Write an equation that you could use to predict the percent of blood flow for a person of any age over 35.


Answered by Penny Nom.
8, 4, 2, 18, 1, 9 2001-01-11
From Neyra:
Place each number below in one of the blanks to create the most meaningful and realistic story possible.
Answered by Claude Tardif.
x^5 + y^5 + z^5 = w^5 2001-01-11
From Samuel Brown:
x5 + y5 + z5 = w5 I have no idea whether or not this is possible.
Answered by Claude Tardif.
Four crayons 2001-01-10
From Neyra Espinoza:
You have four crayons (red, blue, yellow, green). If you line them up, how many different combinations can you get?
Answered by Patrick Maidorn.
Multiplication of fractions 2001-01-08
From Angela:
Questions:

1. represent multiplication of fractions via an area model

2. describe why, not just how, to round decimals


Answered by Penny Nom.
The city transit system 2001-01-07
From Jacky:
The city transit system carries an average of 9450 bus riders a day, for a fare of $1.75 each. The city wants to reduce car pollution by increasing ridership and to maximize the transit system's revenue at the same time. A survey indicates that the number of riders will increase by 150 for every $0.05 decrease in fare.
Answered by Harley Weston.
Women mathematicians 2001-01-07
From Kisha:
Can you please tell me names of at least five female math teachers who changed history back then?
Answered by Penny Nom.
Derfs, Enajs and Sivads 2001-01-07
From John and Norman:
All Derfs are Enajs. One-third of all Enajs are Derfs. Half of all Sivads are Enajs. One Sivad is a Derf. Eight Sivads are Enajs. The number of Enajs is 90. How many Enajs are neither Derf nor Sivad?
Answered by Penny Nom.
A dollar, quarter, dime, nickle and penny 2001-01-07
From Sarah:
Arnold has a dollar coin, one dime, one quarter, one nickel, and a penny. The number of different sums of money that can be formed using three coins is...
Answered by Penny Nom.
The pythagorean theorem in everyday life 2001-01-06
From Josh:
What are some ways that we use the pythagorean theorem in jobs, or even in everyday life?
Answered by Claude tardif.
The laws of sines and cosines 2001-01-02
From Faydene:
Can the sine /cosine rule be applied to a right -angle triangle to find a particular solution or are these 'rules' applied only when the triangle is not right angled?
Answered by Penny Nom.
Bush fractals 2000-12-30
From Anita Wisecup:
My son has a report due on fractals. He needs information on bush fractals, but we cannot seem to find anything out about them.
Answered by Penny Nom.
A pair of numbers whose GCF is 28 2000-12-27
From John:
Name 2 different pairs of numbers whose GCF is 28.
Answered by Penny Nom.
How do you integrate secant(theta)? 2000-12-22
From Robert Williamson:
How do you integrate secant(theta)?

I know the answer is ln [sec(theta) + tan(theta)] but how do you get there?


Answered by Claude tardif.
An infinite series 2000-12-16
From John:
summation(n=1 to infinity)[n sin(1/(2n))]n
Answered by Harley Weston.
Factoring (u-v)3+v-u 2000-12-15
From A parent:
I am a middle school teacher and a parent. I am snowed in and trying to help my 9th grader get ready for 9 weeks exams. I have tried to factor this problem to no avail. (u-v)^3+v-u. I have the answer but I need to know how it is done.
Answered by Penny Nom.
Prime factorization 2000-12-13
From A student:
What is the prime factorization for 250 1296 and 2400
Answered by Penny Nom.
Scientific notation 2000-12-12
From Guy:
How do I express 1.000.000 in scientific notation?
Answered by Harley Weston.
A chemist had 8 flasks 2000-12-10
From Jimmy:
A chemist had 8 flasks capable of holding 12, 15, 27, 35, 37, 40, 53 and 69 fluid ounces respectively. He filled some with water and then filled all the rest except one with alcohol. He used exactly one and a half times as much alcohol as water. Which flask was left empty and which were left with water and which with alcohol?
Answered by Claude Tardif.
Mixed Fractions 2000-12-09
From Tim Finch:
My son is having problems with mixed decimals and mixed fractions.. I am unable to find any information or examples on this subject so I can show him how to do it.. for example how do you make a mixed decimal or fraction out of 7/16
Answered by Penny Nom.
Fibonacci 2000-12-07
From A student:
Can you give me any info about Fibonacci??
Answered by Penny Nom.
Kai's bike 2000-12-07
From Fumiko Muhammad:
The wheels on Kai's bike are 27 inches in diameter. His little sister, Masako, has a bike with wheels that are 20 inches in diameter.

a. How far will Kai go in one complete turn of his wheels?

b. How far will Masacko go in one complete turn of her wheels?

c. How far will Kai go in 500 turns of his wheels?

d. How far will Masako go in 500 turns of her wheels?


Answered by Penny Nom.
Scheduling a golf vacation 2000-12-05
From Michael:
I'm having a problem scheduling matches for a golf vacation. We have 12 people playing 7 rounds of golf in 7 days. We play 2 man teams vs. 2 man teams everyday.Is there a formula so that you play WITH a different partner everyday and AGAINST as many different people as possible? Thank You for any help you may be.
Answered by Penny Nom.
Equivalent fractions 2000-12-03
From A student:
please explain to us how to find the answer to this problem, we need three fractions that name the same amount as 2
Answered by Penny Nom.
A 16 slice pizza 2000-11-30
From A student:
What is 5/8 of a 16 slice pizza?
Answered by Penny Nom.
An integration problem 2000-11-30
From A student:
If a>0 and the integral from b to 0 of 1/(1+x) equals 1/2 the integral from a to 0 of 1/(1+x), express "b" in terms of "a"
Answered by Claude tardif.
Vector Problem 2000-11-27
From Ben:
An aircraft can fly 260km/h in still air and the wind is blowing at 70km/h towards the West. In what direction should the aircraft head so that its actual velocity is on a bearing of 030 degrees?
Answered by Harley Weston.
Order of operations 2000-11-26
From Margaret Pratt:
My daughter has a math question and I am afraid I am of no help. Can you help? 2x5/2+1-5= She arrives at 8 as the answer and has been told this is incorrect. Any help you can give would be appreciated.
Answered by Penny Nom.
Range of a function 2000-11-21
From David Bell:
Given a rational function such as f(x) = (8x-3)/(4x-1). How can the range be found.
Answered by Penny Nom.
Bridges and parabolas 2000-11-18
From Lauren:
My name is Lauren, and Im a secondary school student in Ontario. For my gr11 advanced math class I have to find out how and why parabolics are used in arch bridges and write 3 paragraphs on it. People who cohse satelites and whatnot are lucky - I've found a ton of info, but for arch bridges there seems to be nothing.
Answered by Harley Weston.
Domain of a function 2000-11-15
From Mickey:
state any restrictions on the domain of the function.

y = 5x - 12 over 27x + 6 x does not equal what________?


Answered by Penny Nom.
Five digit palindromes 2000-11-12
From Jacky:
Positive integers such as 1287821 and 4554, in which the number is unchanged when the digits are reversed, are called palindromes. The number of five-digit integers larger than or equal to 10,000 which are not palindromes is... a. 10 000 b. 81 000 c. 89100 d. 90 000 e. 99 100
Answered by Penny Mom.
Euler's Formula 2000-11-11
From Denise Roberts:
I'm trying to design a unit (possibly involving a Webquest) on Polyhedra and I cannot find a formula I once used. The formula involved the number of vertices (V), edges (E), and faces (F) of the polyhedra.
Answered by Walter Whiteley.
Infinite Geometric Series 2000-11-10
From Sam Carter:
I ran into a problem when studying how to find the sum of an infinite geometric series. My math book attempts to explain the concept by giving formulas involving sigma and |r|, but it does not really explain how to go about finding the sum of an infinite geometric series. If you could either help me with this or point me in the direction of an informative website that could help me, I'd appreciate it.
Answered by Harley Weston.
Graphing F(x) = 3x 2000-11-06
From Jose:
graph the exponential problem F(x)=3x
Answered by Harley Weston.
Perfect numbers 2000-10-31
From A student:
I was wondering if you could help me answer a question my pre-algebra teacher asked in class the other day. He asked if we knew what the perfect numbers where.

He told us the first number is 6 the second number is 28 but the third he did not tell us. Do you know what the third perfect number is?
Answered by Paul Betts and Chris Fisher.

Factoring a cubic 2000-10-31
From A student:
Factor Completely

18p3 - 51p2 - 135p


Answered by Paul Betts and Penny Nom.
False Positives 2000-10-29
From James:
A rare disease infected 1 in 1000 people in the population. A test for the disease is accurate 99% of the time when given to an infected person and also when given to a heathy person.
  1. Fill out a two-tier tree diagram and find the probability of the false positive(i.e the conditional probabily of being healty even when tested postive by the test) Comment on the result?

  2. ...

Answered by Penny Nom.
Square roots 2000-10-29
From Pamela:
HERE GOES(I WILL USE Q AS THE SYMBOL FOR SQUARE ROOT):

8(Q2) - 5(Q2) + Q2

SECOND PROBELM IS

(1 + Q2)2

LAST ONE

3/(2-Q5)


Answered by Claude Tardif.
Find the rule 2000-10-27
From Kelsa and Linda:
Find the rule: It had 13 in a circle then 2 blank circles then 31 in a circle then two more blank circles. Could you please help us.
Answered by Penny Nom.
A zip code problem 2000-10-26
From Rob Mathis:
Find the zip code of a place in a county so that the product of it and the zip code of another place in another county of the same name, but in a different state, is an exact multiple of the number 123456789
Answered by Claude Tardif.
Assessment and portfolios relating to math 2000-10-24
From Jackie:
I will be assisting in a presentation for grades 4-6 on assessment and portfolios relating to math. If anyone has some ideas will love to hear them. I have checked out a few sites on the net
Answered by Diane Hanson.
The perimeter of a football field 2000-10-18
From Paige:
The width of a football field is 67ft greater than the length. The perimeter is 480ft What is the width and length?
Answered by Penny Nom.
Dividing fractions 2000-10-18
From Paula:
Why do you have to change the division sign to a multiplication sign and invert the fraction that follows the division sign in order to get the answer to a division problem when you're working with fractions?
Answered by Penny Nom.
Where will we use this in the real world? 2000-10-11
From Jane Ann Musgrove:
As a teacher of mathematics, I am always asked "Where will we use this in the real world?". I am seeking ideas/sites via the internet where students can find answers to this type of question. Can you help me?

To be more specific, right now I am interested in finding careers where the employees would use the concepts of "Radicals", "Matrices", and "Logarithms". This information will be used by students to make presentations to the class on their findings from internet searches.


Answered by Harley Weston.
Sunflower seeds 2000-10-09
From A parent:
Find the number of 3/4 pound of sunflower seeds that can be filled with 225 pounds of sunflower seeds
Answered by Penny Nom.
Repeating decimals 2000-10-06
From Mary O'Sullivan:
I understand that with repeating decimals (those with a pattern), the number of digits repeated is put into fraction form with the same number of 9's

ex. 0.4444 = 4/9 0.145145145 = 145/999

How can I explain why this is so? Some inquisitive 8th graders are anxious to find out!
Answered by Penny Nom and Walter Whiteley.

Pillows and Cushions 2000-09-27
From Fiona:

The following problem was given to grade eleven algebra students as a homework assignment. To manufacture cushions and pillows, a firm uses two machines A and B. The time required on each machine is shown. Machine A is available for one full shift of 9.6 hours. Machine B is available for parts of two shifts for a total of 10.5 hours each day.
Answered by Harley Weston.

A cycloid in Cartesian form 2000-09-20
From Billy:
The parametric equation of cycloid is given:
x=r(t-sint)
y=r(1-cost)

How to eliminate t?


Answered by Harley Weston.
A proof that 1=2 2000-09-19
From sporky:
Why does the proof for 1=2 not work?

x = 1
x2 = 1
x = x2
1 = 2x (derivitive)
1 = 2(1)
1 = 2 ???

please tell me where the false logic is.


Answered by Walter Whiteley.
Order of operations 2000-09-19
From Nicole:
the problem is, 4x8-18 divided by 6= do you solve this by doing 4x8=32 then by dividing 18by 6= 3, then subtracting 3 from 32 = 29? If not what is the order of operation?
Answered by Penny Nom.
1234567890 2000-09-14
From Bradley Kloetzly:
Can you find two whole numbers, with the smallest possible difference between them, which when multipled together equal: 1234567890?
Answered by Harley Weston.
6-49 2000-09-14
From Steve:
In our state lottery you must choose 6 numbers (1-49). How many different combinations are there? They can be in any order.
Answered by Harley Weston.
Two linear equations 2000-09-14
From David Dean:
2a + 1b = 3.39 3a + 3b = 6.59
What formula do I use to find what a = ?

Answered by Harley Weston.
4-digit numbers using 7, 8, 9 and 0 2000-09-13
From Ryan:
Could you please help me with the following problem, How many different 4-digit numbers are there that use each of the digits 7,8,9,and 0? Please list them.
Answered by Penny Nom.
Manipulating powers 2000-09-13
From Phil:
21/5 x 42/5 = ?
Answered by Penny Nom.
Flim 2000-09-12
From Jodi and Ari:
Can you tell us the definition of the word FLIM? My daughter is in 4th grade, and this came up. Could not find it in the Websters Dictionary.
Answered by Chris Fisher.
Factoring 2000-09-12
From Melissa:
I would like you to help me factor (x 6-y 6) & (a+b)2+3(a+b)-4
Answered by Penny Nom.
Induction 2000-09-07
From Joe Peterson:
How do I prove by the principal of mathematical induction?
1.n+2.(n-1)+3.(n-2)+.....+(n-2).3+(n-1).2+n.1=(n(n+1)(n+2))/6

Answered by Paul Betts.
Roots of x^2 + 22 x + 112 2000-09-07
From Brian R.:
I need to find the root of the quadratic equation of a square. The problem is x2 + 22x + 112 = 0.
Answered by Penny Nom.
Derivatives, there must be an easier way 2000-09-06
From Brad Goorman:
The direction read: Take the derivative of each expression.

y = {1+[x+(x2 +x3)4]5}6


Answered by Harley Weston.
Velocity of a pendulum 2000-08-28
From Mekca:
A pendulum hangs from the ceiling. as the pendulum swings, its distance,d cm, form one wall of the room depends on the number of seconds,t, since it was set in motion. assume that the equation for d as a function of t is: d=80+30cos3.14/3t, t>0.

estimate the instantaneous rate of change of d at t=5 by finding the average rates for t=5 to 5.1, t=5 to 5.01, and t=5 to 5.001.


Answered by Harley Weston.
Cubic yards and cubic feet 2000-08-26
From Rhonda:
have 2.47 cubic yards. what could the dimensions, in feet, be to equal this?
Answered by Penny Nom.
Subtracting fractions 2000-08-26
From Sarah Stanczyk:
how is this problem solved??

3/ x+1 - 5/x
( 3 divided by x +1 subtracted by 5 divided by x)


Answered by Penny Nom.
The sum of the squares of 13 consecutive positive integers 2000-08-25
From Wallace:
Prove that it is not possible to have the sum of the squares of 13 consecutive positive integers be a square.
Answered by Harley Weston.
Covering 1, 1/2, 1/3,... 2000-08-22
From Wallace:
Consider the points 1, 1/2, 1/3, ... on the real number line. You are given five small bars, all of length p, which are to be placed on the number line such that all points will be covered. What is the minimum value of p that will allow you to do this?
Answered by Penny Nom.
The angle of rotation 2000-08-03
From Jay:
I have the following information Given.

(X1, Y1) Origional Point
(X2, Y2) Origional Point After a Rotation
(Xa, Xb) Center of Rotation

What formula would I use to figure out the angle the point was rotated?


Answered by Chris Fisher and Harley Weston.
Two linear systems 2000-08-01
From A student:
please help me with this homework question. 5/x + 6/y= 19/6 3/x + 4/y =2
hint: let a = 1/x and b = 1/y substitute these expressions into the system to find a and b. Then find x and y.

This is a question I answered wrong on a test: solve the given system for x, y, and z. Express your solutions in terms of a, b, and c.


Answered by Penny Nom.
The circumference of a circle 2000-07-30
From Not a student:
An equalateral triangle is enclosed in a circle. The three corners touch the edges of the circle. One side of the triangle is 12. What is the circumference of the circle?
Answered by Penny Nom.
An equation involving x to the x 2000-07-22
From Joy Peter:
I am joy, a teacher teaching Maths at the secondary level and while solving a sum came to this stage when i got xx (1 + log x) = 0, by which we can conclude that xx = 0 or 1 + log x = 0. If xx = 0, than what should be the value of x? I feel that the value of x should then be 0 (zero) but then how do I explain this to the students as we also know that anything to the power of 0 is 1 but here 0 raised to 0 is 1. If this is not defined then how do I explain this?
Answered by Penny Nom.
n! = 42(n-2)! 2000-07-21
From Damon Bailey:
Solve n! = 42(n-2)!
Answered by Paul Betts.
10,000! 2000-07-21
From Lauren:
Hi I was just wondering if you could tell me how many zeros are in 100,000! (factorial.)
Answered by Denis Hanson.
Simplify 21+2[3z+5(3z+8)] 2000-07-02
From Dennis Dyer:
My daughter has a math problem that I can't help her with. The problem is 21+2[3z+5(3z+8)] I can get the answer 101 +36z but I can't show her the correct way to write it out. Would you please show the correct way to show her work...
Answered by Penny Nom.
Parallel tangents 2000-06-30
From Ebony Indalecio:
I need to prove the theroem: Tangents to a circle at the end points of a diameter are parallel.
Answered by Walter Whiteley.
Tiling a floor 2000-06-27
From Carolyn Clarkston:
A square tile measures 6 inches by 6 inches. What is the least number of tiles needed to cover a rectangular floor area of 9 feet by 12 feet?
Answered by Walter Whiteley.
Central Limit Theorem and Law of Large Numbers 2000-06-26
From Jonathan Yam:
The Central limit Theorem states that when sample size tends to infinity, the sample mean will be normally distributed. The Law of Large Number states that when sample size tends to infinity, the sample mean equals to population mean. Is the two statements contradictory?
Answered by Paul Betts and Harley Weston.
The speed of light 2000-06-19
From Benny Lau:
I have a personal questions. In physics,if a person is running on a train, his relative speed on the ground will be the addition of the speed of train and the person. If u have a train inside another train and inside another train, so on...... can u reach the speed of light? It would need a very long distance. But if we make all the train travel in circles. Then we don't need that much distance. So can we achieve the speed of Light?
Answered by Robert Coleman and Harley Weston.
Numerator and denominator 2000-06-18
From Maureen Beard:
What is the origin of the terms numerator and denominater?
Answered by Penny Nom.
Simplifing 2000-06-15
From Angie Herbert:
My daughter is having problems with her algebra homework. She is a year 7 pupil at high school in the UK. She has been given homework on simplifying formulae and she doesn't understand how to do them. Here is an example of one of the questions :

9s+7t+5c+5s-3t

can you help her to understand how to do these and perhaps me too.
Answered by Penny Nom and Claude Tardif.

Surface area of a contact lens 2000-06-06
From Evie Contreras:
I would like to know how to calculate the surface area of a contact lens with a radius of 7mm? I know that the area of a circle is pi R squared, but a contact lens has a dome.
Answered by Harley Weston.
A derivative problem 2000-06-04
From Jeff Ellis:
If F(x)=(4+x)(3+2x2)2(2+3x3)3, find F'(0)
Answered by Harley Weston.
Percents with mixed numbers 2000-06-04
From Julie:
How would you find the percent of 33 1/3 out of 90 by using fraction settup!I can do whole numbers but not mixed. This is how to do whole numbers say it was 33 of 90

33 x 90 = 2970
100 x 1 = 100
2970 divided by 100
the answer is 29.7

so how do you do it with a mixed number


Answered by Penny Nom.
4 Color map Theorem 2000-06-02
From Rick Slen:
I am challenging my students to find a map that can not be coloured using only 4 colours. I know that it is not possible, but one boy drew a map with a "country" totally surrounding all the others so of course it touched all other colours. How do I explain that this type of map is not permissable?
Answered by Denis Hanson.
Powers of i 2000-05-24
From Paul Fieldhouse :
What is the result of raising i to the googol power? is there a rule or pattern to raising i by increasing powers of 10?
Answered by Penny Nom.
Angular velocity 2000-05-24
From Ashley Milliman:
The crankshaft of a particular automobile engine has an angular velocity of 1,500 rpm at 30 mph. The crankshaft pulley has a diameter of 10 cm, and it's attached to an air conditioner compressor pulley with a 7 cm diameter and an alternator pulley with a 5 cm diamter

At what angular velocities do the compressor and alternator turn?

.
.
.

Answered by Harley Weston.
The equation of a parabola 2000-05-22
From Ian Forsyth:
Given the points A(0,0) B(60,10) C(24,d) find the equation of the parabola. leave the equation in terms of x, y and d if the general form of a quadratic is y = ax2 + bx + c.
Answered by Penny Nom.
Calculus Research Questions 2000-05-22
From William Wright:
I am a Calculus Teacher, and me and my class ran into these two problems without solutions in my manual, we got answers, but are unable to check them. If anyone gets this email and can respond to this with the solutions it be greatly appreciated.
.
.
.

Answered by Harley Weston.
The square root of -1 2000-05-19
From Gary:
i am not a student i am just some one that heard something and i can't be sure on the answer...my ? is what is the square root of -1? i think it is -1 but not sure can you let me know please thank you
Answered by Harley Weston.
A conic in standard form 2000-05-18
From Tara McConkey:
Im havign trouble converting the following conic to standard form, i know that the conic is a hyperbola but that is all 16x2-9y2-160x-18y+247=0
Answered by Harley Weston.
Graphing a linear function 2000-05-17
From Chelsea:
I need help with grahing linear functions.If you could e-mail me back the basics and how tos I would be much appriciative.
Answered by Penny Nom.
Multiplying fractions 2000-05-14
From A student:
75/2 x 1/100 = 3/8

Please show how to came up with the answer?
Answered by Penny Nom.

A matrix equation 2000-05-14
From A student:
Right now, we are dealing with matrices and we are supposed to solve the following problem on our graphing-calculators: 2a+3b-4c+d=20
a-2b+3c-5d=-14
3a+4b-2c+3d=19
5a-b+6c+4d=-5

Answered by Penny Nom.
Reflection off a sphere 2000-05-12
From Mark Adami:
Given two points P,E and a sphere. Find a point on the sphere, T, so that a line from P that bounces off the sphere at T goes through point E.
Answered by Chris Fisher and Harley Weston.
The Village of Truth 2000-05-10
From Sean Pearson:
Our teacher gave us a logic problem involving two villages: the village of truth, and the village of lies. The problem is in asking two guards the right question to gain entrance to the fenced compound the villages are in without being shot. Have you heard of this problem ?
Answered by Harley Weston.
sin(7pi/12) 2000-05-04
From Kristel:
What is the exact value of sin 7pi/12?
Answered by Chris Fisher and Paul Betts.
Thearcius Functionius 2000-05-03
From Kevin Palmer:
With the Olympics fast approaching the networks are focusing in ona new and exciting runner from Greece. Thearcius Functionius has astounded the world with his speed. He has already established new world records in the 100 meter dash and looks to improve on those times at the 2000 Summer Olympics.

Thearcius Functionius stands a full 2 meters tall and the networks plan on placing a camera on the ground at some location after the finish line(in his lane) to film the history making run. The camera is set to film him from his knees(0.5 meters up from the ground) to 0.5 meters above his head at the instant he finishes the race. This is a total distance of two meters(the distance shown by the camera's lens).
Answered by Harley Weston.

An indefinite integral 2000-05-03
From Bonnie Null:
I am to find the indefinite integral of: (ex - e-x)2 dx
Answered by Claude Tardif.
Mary Fairfax Greig Somerville 2000-05-03
From Joshua Garrett:
I'm giving a report in class on her life. Could i recieve some sort of information about her. thank you
Answered by Claude Tardif.
Why is 1/4 larger than 1/5? 2000-05-02
From Sherry:
I have to do a math project for my teacher education class. The question that I am working on is why is 1/4 bigger than 1/5.
Answered by Paul Betts, Claude Tardif and Penny Nom.
Two calculus problems 2000-05-01
From Kaushal Shah:
How Do WE Integrate the following Functions,
  1. Integral xtanx dx
  2. How was natural base "e" discovered and why e=2.7.......

Answered by Claude Tardif.
Fractals 2000-04-29
From Rachel Maginn:
What are Fractals? I am doing a research project. Any information would be great. I need to know the history behind fractals, and how they are used. I would like some examples like the snowflake and a fractal tree. Any other examples would be appriciated greatly.
Answered by Harley Weston.
What percent is one fraction of another fraction? 2000-04-29
From A student in grade 4:
What is the method for finding out what percent one fraction is of another fraction. For instance 4/5 is what percent of 1/9?
Answered by Penny Nom.
Graphing cube roots 2000-04-27
From Heather Jones:
What do the graphs of cube roots look like?
Answered by Walter Whiteley.
Adding fractions 2000-04-23
From A grade 6 student:
In adding fractions, how do I rewrite the fractions so that the denominators are equal? The problem is 3/4 + 1/6 =
The other problem is 11/18 + 4/9 =

Answered by Penny Nom.
Circumference = Area 2000-04-19
From Scot George:
The area and circumference of a circle has the same measurement. Find the radius.
Answered by Chris Fisher.
Chisanbop 2000-04-19
From Steven D. Edmonds:
I once heard about a manual counting method which when learned at an early age allowed one to be able to perform large calculations manually. I believe that name of the method (phonetically) was "chisenbok".
Answered by Harley Weston.
Factoring trinomials that are squared 2000-04-17
From A student:
I have a question that I really need an answer to. I need help with factoring trinomials that are squared.
Answered by Penny Nom.
All the roots of x^6 - 64 2000-04-15
From Dakota:
Find ALL zeros of P(x) = x 6 - 64
Answered by Walter Whiteley.
Infinite sets 2000-04-12
From Brian Provost:
Here's the deal: There are an infinite amount of integers (1,2,3...). Agreed? There are an infinite amount of even integers, too (2,4,6...). Agreed? By convention, infinity equals infinity. Yet common sense tells us there are obviously more integers than there are even integers. Prove this to be true mathematically.
Answered by Harley Weston.
How many zeros? 2000-04-09
From Greg Potts:
The natural numbers 1 to 25 are multiplied together (1 x 2 x 3x..24 x 25). How many zeros are there in the product of this multiplication?
a)6 b)7 c)5 d)10 or e)4?

Answered by Harley Weston.
2^32 + 1 2000-04-08
From Un Etudiant:
232 +1 can be divided by 641 , why?
Answered by Claude Tradif.
Surface area of a cylinder 2000-04-07
From Jim Campbell:
I am going to cover the outside of a shaft with some material. How do I figure the square inches of the outside surface of the shaft? Shaft is 6" in diameter and 24" long
Answered by Harley Weston.
Lining a cone 2000-04-06
From Jim Campbell:
I am not a student, I am trying to solve a business problem. The question. If I want to put a lining in a chute that is cone shaped, how do I calculate the size steel plate I need to do that. The cone is 10' in diameter at the top and has a 20" hole at the bottom. The total height of the chute is 8'.
Answered by Harley Weston.
Why are equivalent fractions equal? 2000-04-05
From An elementary student:
What is the mathematical reason equivalent fractions are equal?
Answered by Penny Nom.
y = x^x^x^x... 2000-04-05
From Michael Hackman:
Find the derivative of: y = x^x^x^x... on to infinity.
Answered by Claude Tardif.
The square root of 3 2000-04-04
From Mr. William:
Prove that root 3 is irrational
Answered by Harley Weston.
Careers after a math degree 2000-03-31
From Jeanne Hyer:
What type of careers can a person have with a math degree, and what is the closest thing that you have to an undergraduate degree in financial mathematics? (Administration, math, actuarial science, etc.)
Answered by Harley Weston.
Making fruit salad 2000-03-29
From A grade 5 student:
Miss.Muffet males a fruit salad by adding 31/2 pounds of grapes and 4 1/4 pounds of apples to every 2 1/4 pounds of melon. How many pounds of grapes does she use in 50 pounds of fruit salad?
Answered by Claude Tardif.
Taxis in Chicago 2000-03-27
From A high school student high school student:
Suppose that taxis pick up and deliver passengers in Chicago, which is divided into three zones. Records kept by the drivers show that of the passengers picked up in Zone 1, 50% are taken to a destination in Zone 1, 40% to Zone 2, and 10% to Zone 3. OF the passengers picked up in Zone 2, 40% go to Zone 1, 30% to Zone 2, and 30% to Zone 3. Of the passengers picked up in Zone 3, 20% go to Zone 1, 60% to Zone 2 and 20% to Zone 3. Suppose that at the beginning of the day, 600 of the taxis are in Zone 1, 100 in Zone 2, and 300 in Zone 3. What is the distribution of taxis in the various zones after all have had two riders?
Answered by Harley Weston.
Reflex angles 2000-03-22
From D. Reed:
What is the name of an angle that exceeds 180 degrees?
Answered by Penny Nom.
Rule of 78 2000-03-22
From Dan Baumgarten:
Can you explain the rule of 78 and the reverse rule of 78? Thanks.
Answered by Claude Tardif.
Surface area of a sphere 2000-03-22
From Gina Wilkie:
How can I demonstrate to my middle school students the reason for the formula for the surface area of a sphere?
Answered by Walter Whiteley and Chris Fisher.
I am in lowest terms. 2000-03-21
From Travis Fox:
I am in lowest terms. When you double my numerator and my denominator the result is 10/15.
Answered by Penny Nom.
Binomial coefficients 2000-03-21
From Howard Lutz:
How do you find each successive numerical term in this equation y+dy=(x+dx)5 =x5+5*x4dx+10*x3(dx)2+10*x^2(dx)3+5*x(dx)4+(dx)5 I would appreciate an explanation of the method to find the numeric coefficient in a binomial expansion
Answered by Penny Nom.
A system of equations in five unknowns 2000-03-20
From Will:
I have been having some problem with the following question for some time. I would appreciate any help on solving the problem or a solution.

Q: Assume that a system of equations in the unknowns x1, x2, x3, x4 and x5 when converted to row echelon form gives

.
.
.

Answered by Penny Nom.
Six digit numbers using 1,2,5,6,7, and 9 2000-03-20
From Rachel:
How many different six-digit numbers can you make using the digits 1,2,5,6,7, and 9? How many of these six digit numbers are divisible by six?
Answered by Claude Tardif and Denis Hanson.
Division 2000-03-16
From Charday:
What is the easiest way to calculate the least number that can be divided evenly by all the numbers from 1 to 10.
Answered by Penny Nom.
Induction 2000-03-16
From William Tsang:
I am trying to prove a induction question

Sigam r=1 n (2r -1)cube = n square (2 n square - 1)


Answered by Harley Weston.
Functions that satisfy f' = f 2000-03-16
From Kevin Palmer :
Recently my calculus teacher asked his students to try and find any functions whose derivatives where the exact same as the original function.

The only function then I have determined that statement to be accurate in is all the natural exponential functions. Ex. f(x) = ex, f'(x) = ex

If possible could you please email me all the functions that you can find in which the original function and its derivative is identical.


Answered by Claude Tardif.
Eligibility in a contest 2000-03-14
From Ken Rabley:
Hi, hoping you can help. Dispute among co-workers. Tell me, what is the correct answer to the following question:
-1936 + (2406/4812) x (4756 - 3256) + 1250

Seems this is the question for eligibility in a contest...We have come up with various solutions, all which may be correct.


Answered by Penny Nom.
Simultaneous equations 2000-03-11
From Laura Molck:
My name is Laura Molck and I am in Year 11 in Australia. Please help me with the following. I know that they are all simultaneous equations which I can do but I have trouble with the formulae to work the equations. Can you please help!!

1. A tent manufacturer produces 2 models, Outback and Bushwalker. From earlier sales records it is known that 20% more of the Outback model is sold than the Bushwalker. A profit of $200 is made on each Outback sold, but $350 is made on each Bushwalker. If during the next year a profit of $177,000 is planned how many of each model must be sold?


Answered by Penny Nom.
Congruent parts of congruent figures 2000-03-10
From Erica:
Yesterday, I recieved a test problem asking to prove two line segments equal. Here is the problem as I was given it:

Given: paralleogram ABCD
AE is perpendicular to DB
CF is perpendicular to DB
Prove: AE is equal to CF

I answered the problem as follows:
.
.
.
Answered by Walter Whiteley.

Systems of linear equations 2000-03-10
From Ann Marie Devereux:
hi there!!, I guess I have a problem!!!
  • 3x+4y=10 (over)
    4x+y=9

  • 2x=5y+3 (over)
    x=3y+1


Answered by Penny Nom.
Making dolls 2000-03-07
From A 6th grader:
How many small dolls can be made from 3 lbs. of dough if each doll uses 7/8 lbs
same question if each doll takes 3/8 lbs.

Answered by Penny Nom.
A mixture problem 2000-03-06
From Rebecca Edwards:
A tank in which cholocate milk is being mixed contains a mixture of 460 liters of milk and 40 liters of chocolate syrup initially. Syrup and milk are then added to the tank at the rate of 2 liters per minute of syrup and 8 liters of milk per minute. Simultaneously the mixture is withdrawn at the rate of 10 liters per minute. Find the function giving the amount of syrup in the tank at time t.
Answered by Harley Weston.
Is a sheet of paper 2-dimensional? 2000-03-06
From Jaclyn Evans:
One day, we were discussing 3-dimensional items. Is a sheet of paper considered a 3-dimensional shape? Or is it 2-dimensional as described by 8.5 by 11 size? Also, could you give me an example of a 4-dimensional item or scenario?
Answered by Harley Weston.
10 000 000 = a x b 2000-03-06
From Spencer Jaffe:
What two numbers that don't end in zero, multiplied equal 10,000,000.
Answered by Claude tardif.
Angle of Intersection of Two Lines 2000-03-02
From Veronica Patterson:
I am having a real hard time trying to figure out this problem. Could you please help me! The homework question says to find the acute angle of intersection between the two lines y=3x+1 and y=(1/2)x-1. (It also says to use the results of a problem I had already figured out.) That problem was to use information from a picture shown that tan(theta sub1-theta sub2)= ((m sub2- m sub1)/(1+(m sub1 * m sub2))). I used the difference identity of tangent to figure out the answer. Any help on this problem would be greatly appreciated.
Answered by Harley Weston.
Folding a page 2000-03-01
From Krista Bischoff:
One corner of a page of width a is folded over and just reaches the opposite side. Express L, the length of the crease, in terms of x and a.

I can't get the picture to copy to this form so I guess I will have to try and describe the picture the best that I can. The top right hand corner is folded to the left side, almost half way down. The width of the paper is a ( the width of the bottom part which is not folded.) The creased side is L and the part shorter part of the folded area is x (the part that would have been the top right of the original piece.)


Answered by Chris Fisher.
Order of operations 2000-02-27
From Mildred Saulsbery:
Find the value of the expression.
 6(5+1) - 9(1+1) _____________    5(8 - 6) -23 

Answered by Harley Weston.
Fractions in algebra 2000-02-27
From Leslie:
Question:
add (4x+1)/(x-8) + (3x+2)/(x+4) + (49x+4)/(x^2-4x-32) and

solve x - 6/(x-3) = 2x(x-3)

Answered by Penny Nom.
Factoring, primes, GCF and LCM 2000-02-27
From Ruth Kroek:
My son is in grade seven, he has to do a Factoring Booklet the areas covered are:
  • Prime #'s
  • Composite #'s
  • Rainbow Factoring
  • Finding Multiples (consecutive multiples)
  • Finding GCF of 2-3 numbers uning Rainbow factoring
  • Finding LCM of 2 numbers using consecutive multiples
  • prime factor trees
  • prime factor ladders
  • finding GCF of 2 numbers using Prime number Method
Although his text 'Math Power' gives some information, we are at a loss ..

Answered by Penny Nom.
Polar coordinates 2000-02-24
From Tran Trung:
How do I convert r = 2/(2sin(theta) - 3cos(theta)) to rectangular and x2 + y2 -2y = 0 to polar form?
Answered by Harley Weston.
Triple angle formula 2000-02-23
From Sara:
Can one derive a triple angle formula for sine and cosine? If so, how?
Answered by Chris Fisher.
Party favors 2000-02-22
From Krystina Fernandez:
Luanne was making party favors for her little sister's birthday party. Each party favor was in the shape of a cube. Luanne had pink and purple paint to paint the cubes and she could paint each face only solid pink or solid purple (no dots,stripes,ect.).For example, one cube may be all purple, another may have two purple faces and four pink faces. Her little sister wanted to have a different cube for each guest.(A cube is not considered different if it can be turned so that all it's sides match the corresponding sides of another cube.)How many different cubes was it possible for Luanne to make?
Answered by Claude Tardif.
Pi 2000-02-22
From Shelley Collier:
Numbers that can be written in the form p/q where p and q are integers, (q not 0), are known as rational numbers. What you are missing is that p and q must be integers. The fact that Pi is irrational means that you can't have a circle with both the circumference and diameter being integers. In fact you can't even have the circumference and diameter both rational since the quotient of two rationals is again a rational.
Answered by Claude Tardif and Penny Nom.
The quotient rule 2000-02-21
From Charlene Anderson:
Question: I came across a question in our book that states: Let Q(x) = N(x) / D(x) Then re-write Q(x) in a form that can utilize the Power and Product Rules. Use this rearranged form to derive the Quotient Rule.

The Quotient Rule can be derived from the Power Rule and the Product Rule.

One must also use the chain rule too, right?


Answered by Harley Weston.
Irrational algebraic functions 2000-02-21
From Bucky Cadena:
Here is the multipart problem:

Given f(x) = x-3* squareroot of x + 4

What does the f(x) intercept equal Find the two values for which f(x) = -5 Find the one value for which f(x) = -3


Answered by Harley Weston.
Three factors 2000-02-21
From A parent:
Question from a parent helping a child, grade 4, with homework. Can a number have three factors? Name three numbers that have three factors.
Answered by Penny Nom.
Perimeter of an ellipse 2000-02-21
From Kurtis Proffit:
What is the formula for the perimeter or circumference of an ellipse?
Answered by Chris Fisher.
Number sense 2000-02-17
From A college student:
I am a college student and I am to answer the following question: Describe ways of telling by simple inspection when a fraction has a value close to 0; close to 1/2; and close to 1. I am to imagine that I am a 3rd grader answering this question. Any thoughts please?
Answered by Penny Nom.
Change for a dollar 2000-02-16
From Rachelle:
What is the least positive number of coins that is impossible to give out change for a dollar? It is higher than 50 i know and you can us dimes, nickels, pennies, quarters, 50 cent pieaces and silver dollars.
Answered by Claude Tardif.
2 = 1 2000-02-16
From Chuck Kennedy:
Question:
  1. Assume a=b
  2. Multiply both sides by a, a2=ab
  3. Subtract b2, a2-b2=ab-b2
  4. Factor (a-b)(a+b)=b(a-b)
  5. Cancel like factors a+b=b
  6. Substitue b for a b+b=b
  7. Then 2b=b
  8. Therefore 2=1
Question; Were is the mistake?

Answered by Claude Tardif.
Factors 2000-02-15
From Guy Rayfield:
Please can you tell me the factors of 6

I think the answer is 2 and 3 - am I right?


Answered by Penny Nom.
Is a square a rectangle? 2000-02-15
From Jaireh:
This is something that aroused a debate in class: A rectangle was defined as a parallelogram with 4 right angles. A square was defined as a parallelogram with 4 congruent sides and 4 right angles.

I need written and conclusive proof that some rectangles can or cannot be squares. I tried insisting that some of them can.. but without proof nobody will listen.


Answered by Walter Whiteley.
The sum of repeating decimals 2000-02-15
From Caitlin Harris:
Express 0.111... + 0.121212... + 0.123123123... as a repeating decimal and its equivalent fraction. Show work. Also, are there any extensions to this problem? In other words, are there any questions that we could ask that may be related to this problem?
Answered by Penny Nom.
Play ball 2000-02-03
From Jessie:
Here's a calc question that is probably a lot easier than I am making it. If you have a legendary "baseball problem" for the related rates section of Calc I, and you are given that the runner is running from 2nd to 3rd base at a given rate, and the umpire is standing at home plate, and you are given the distance between the bases on the field, how do you find the rate of change of the angle between the third base line (from the point of the umpire) and the runner? Here is a sample prob: Runner is moving from 2nd to 3rd base at a rate of 24 feet per second. Distance between the bases is 90 feet. What is the rate of change for the angle (theta, as described previously) when the runner is 30 feet from 3rd base?
Answered by Harley Weston.
Names of the polygons 2000-01-29
From Janemath:
I have been doing research on the names of polygons with 11 to 20 sides. I have found conflicting names for some of them . for example 18 sided-octadecagon and octokaidecagon. 11sided-undecagon and hendecagon. Why is this? Is one more correct than another? Or are they equally acceptable?
Answered by Harley Weston.
Factors 2000-01-27
From Pam:
My son [age 9, 4th grade] has a math problem with factors. I need to know how to do the problem that involves the factor 24. I'm not even sure where to start.
Answered by Harley Weston.
Functions 2000-01-23
From Tara:
Hi my name is Tara, I have two math problems that I need help with in my calculus math class.

  1. If f(x)= x - 2 show that (x+3)f(x)-(x+2)f(x+1)+4=0

  2. Graph this function and use the graph to determine the range y=2x2 - 8x - 3

Answered by Harley Weston.
Frogs and lily pads 2000-01-23
From Nick:
There are 5 lily pads and 4 frogs, 2 Red and 2 Green, how many moves does it take for them to swap sides?
Answer: 8

I have a formula which will tell you how many moves it will take with different total numbers of frogs, it is [(F*F)/4]+F Where F is the number of red frogs add the number of Green frogs. This formula only works with numbers of frogs that are the same on each side. What I would like to know is why this formula works and why you have to divide it by 4?


Answered by Penny Nom.
The limit of f(x)/x 2000-01-22
From Laurent Jullien:
I would appreciate help to prove that a twice continuously differentiable convex function from R+ to R has the property that f(x)/x has a limit when x tends to infinity.
Answered by Claude Tardif.
Domain of a function 2000-01-19
From Stacey Hutchison:

What is the domain of:

f(x) = -1/(x + 4)


Answered by Patrick Maidorn.
Pyramids and prisms 2000-01-18
From Tyler:
What's the definition of a Triangular Prism and a Triangular pyramid.
Answered by Penny Nom.
Algebra 2000-01-18
From Candis:
I want to know what exactly is Algebra?
Not how to do it, or methods. Just what is it?

Answered by Allen Herman.
Order of operations 2000-01-16
From Dorothy:
I was wondering if you could tell me where I can get some info. on how to solve the following problem. I was given the answer but I don't know how it is solved.

4+2x(6x2)-5=23


Answered by Penny Nom.
A roll of paper 2000-01-15
From Richard:
I have a roll of paper, wrapped around a corrugate core, whos diameter is 10.750 in. The outer diameter of the roll is approx. 60 in. The thickness of the paper is .014 in. I am trying to find out how much linear feet of paper is left on the roll, given only the diameter of paper remaining on the core.
Answered by Chris Fisher and Harley Weston.
Factoring by graphing 2000-01-11
From Dylan Kirkwood:
How can you factor a quadratic equation by graphing it?
Answered by Penny Nom.
Functions 2000-01-06
From Tori Morris:
Consider the function f(x)=x2-3. Which of the following are true?

  1. f(1)>f(0)
  2. f(2)>f(-3)
  3. f(2)=f(-2)
  4. f(-1)=f(3).
More than one answer can be true.

Answered by Penny Nom.
Ordering fractions 2000-01-05
From A seventh grader:
I am a middle school student (7th grade) and I was wondering if you could help me with ordering fractions from least to greatest the fractions are 31/12, 2 5/6 and 2 3/4.
Answered by Walter Whiteley.
Factoring ^6 2000-01-03
From Athena:

my name is Athena and I have a question on factoring: how would you figure this out:

(x6-y6) and (x6+y6)


Answered by Penny Nom.
zero 2000-01-01
From Jason:
What civilization first used zero?
Answered by Penny Nom.
Derive the quadratic equation 2000-01-01
From Thuy:
My teacher asked us this question in which i can't understand when she asked to derive the quadratic equation. What does this mean?
Answered by Penny Nom.
Solving a cubic equation 1999-12-30
From Natalia:
How can I solve:
2 x3 - x2 + x - 2 = 0 ???

Answered by Penny Nom.
Period 1999-12-28
From Mahdawi:
I have attached a diagram of the graph, and I need to find out its period. I really don't understand how to do so, please help!
Answered by Harley Weston.
Three algebra problems 1999-12-28
From Stephanie Branton:
  1. If P represents the product of all prime numbers less than 1000, what is the value of the unit's digit of P?

  2. Do any real numbers a and b exist such that: ln(a+b)=ln a + ln b? if so, what are they?

  3. Define a function by: f(x)=1/1-x where x is not equal to 0,1. what is f(f(f(a)))?

Answered by Harley Weston.
Graph of a sine function 1999-12-23
From Pierre:
Given;
amplitute:1
period: 540
Phase shift: 60 degree,right

I am ask to right the equation: sin 2/3 (value -60degree)

When I am asked to graph the equation, the period is mixing me up.
Answered by Harley Weston.

Specific heat 1999-12-22
From Irena:
In transferring 1260J of heat to water, the temperature of 0.20 kg of mercury falls 50 K. Calculate the specific heat capacity of mercury using the equation C= Q/(m)(delta t)

i know that Q= -1260J because it's loosing heat and m= 0.2kg and since i have to use 'C i have to convert Kelvins into Celsius (i also know that 'C has to be negative since it looses heat)
.
.
.
Answered by Harley Weston.

Two algebra problems 1999-12-17
From Michael Standfest:
If x+4 is a factor of 2x4+kx3-3kx2+6x-40, find k

and

Prove that n2-n is even for all n, using the proof of contradiction
Answered by Penny Nom.

Fibonacci Numbers 1999-12-15
From Gary Nelb:
I'm doing a project on fibonacci numbers and I'm using different starting values and finding out if different starting values to see whether or not the ratios still get closer to phi. I was wondering, what numbers should I use. Should I use two of the same # like 2 and 2, or numbers like 1 and 2, or even something totally different.
Answered by Denis Hanson.
Sines & cosine laws 1999-12-10
From Pierre Boivin:
Triangle LMN, angle L=71 degree , LM= 7.2 , MN=8.3 , ln= 5.9

The questiion was to find angle M. Using the cosine law I found the answer to be 44 degree. It is also the book answer. Using the sines law I found the answer to be 42.2 degree. why can't I use the sines law.
Answered by Chris Fisher.

A calculus problem 1999-12-08
From JT Wilkins:
These are the questions:

  1. Show that there exists a unique function that meets the following requirements:

    a) f is differentiable everywhere
    b) f(0)= f'(0)= 0
    c) f(x+y)= f(x)+ f(y), for all real values of x,y

  2. Consider the function F: R-->R (All Reals)

    F(x) = 0, for x irrational & 1/q, x=p/q gcd(p,q)=1 q > 0

    a)determine the values x where f is continuous, respectively discontinuous.
    b)determine the values x when f is differentiable and for each of these values compute f'(x).


Answered by Penny Nom.
Bases other than 10 1999-12-06
From Garret Magin:
We are doing a lesson on numbers of other bases than 10. We are working with binary, octal, and Hexadecimal. I was wondering what is used to represent number of different bases other then 16? Does it just continue on with the alphabet and if so what happens when you get to Z. It would be a help if you could answer this because it is really bugging me. And none of the math teachers at my school could let me know.
Answered by Claude Tardif and Patrick Maidorn.
Systems of equations 1999-12-06
From Roger Hornbaker:
I am having problem figuring out x and y solutions.
  1. 5x + y = 4
    3x - y = 4

  2. 3x + 2y = 6
    - 3x + y = 0

Answered by Penny Nom.
order 4+ determinants 1999-12-06
From Joe Kron:
Why is it never shown how to calculate the value of 4x4 (or larger size) deteminants by the diagonal multiply methods that are generally shown for 2x2 and 3x3 determinants? The method I'm talking about is called Cramer's Rule??? Is this method not extensible to order 4+ and if not why not? Anyway the method always shown for order 4+ is called "reduction by minors" which is not the answer to this question.
Answered by Walter Whiteley.
How many 3/5 are in 3/4? 1999-12-06
From Whiteny:
How many 3/5 are in 3/4?
Answered by Penny Nom.
Multinomial coefficients 1999-12-03
From Suraj Das:
Is there a formula for the expansion of (a+b+c) to the nth power? Does it have to do with Pascal's triangle?
Answered by Penny Nom.
The chain rule 1999-12-03
From Jennifer Stanley:
This problem is making me dizzy. I would greatly appreciate a little help!

Express the derivative dy/dx in terms of x.

y=u^2(u-u^4)^3 and u=1/x^2
Answered by Harley Weston.

Zero to the zero 1999-12-02
From A middle school teacher:
I am a middle school teacher who is looking for a precise explanation of why zero raised to the zero power is undefined. I am hoping to get an explanation using something other than the fact that diividing by zero is undefined.
Answered by Penny Nom.
Two calculus problems 1999-12-01
From O'Sullivan:
Question #1 Assume that a snowball melts so that its volume decreases at a rate proportional to its surface area. If it takes three hours for the snowball to decrease to half its original volume, how much longer will it take for the snowball to melt completely? It's under the chain rule section of differentiation if that any help.

I've set up a ratio and tried to find the constant but am stuck.

Question #2 The figure shows a lamp located three units to the right of the y-axis and a shadow created by the elliptical region x^2 + 4y^2 < or= 5. If the point (-5,0) is on the edge of the shadow, how far above the x axis is the lamp located?

The picture shows an x and y axis with only the points -5 and 3 written on the x axis. the lamp is on the upper right quadrant shining down diagonally to the left. There's an ellipse around the origin creating the shadow. It's formula is given as x^2 + 4y^2=5.


Answered by Harley Weston.
Repeating decimals 1999-11-22
From Andrew:
Is 1.9 repeating the same as 2?
Answered by Walter Whiteley.
Selecting some players 1999-11-20
From Korbin Brown:
Roger Craig, during his term as team manager of the San Francisco Giants, received a strange communication from the team general manager, Al Rosen. Mr. Rosen told him to select 25 players according to this formula: 1/2 of the team had to be outfielders and infielders 1/4 of the team had to be starting pitchers 1/6 of the team had to be relief pitchers 1/8 of the team had to be catchers Roger was a bit confused by Al's request, yet complied anyway. How did he do it?
Answered by Claude Tardif.
polynomial functions 1999-11-19
From Quinn:
  1. Without fully factoring the following show that they all have the same zeros:

    I: x4-x2+2x+6
    II: -x4+x2-2x-6
    III: -4x4+4x2-8x-24
    IV: 10x4-10x2+20x+60

  2. When P(x)=x3-3x2+5x+1 and G(x)=x3-2x2-x+10 are each divided by(x-a) the remainders are equal. At what coordinate point does the graph of P(x) intersect G(x)?

Answered by Walter Whiteley.
Two derivatives 1999-11-16
From Gina Renicker:
The derivative of:

y=e(xlnx) and y=x2arctan(x1/2)
Answered by Harley Weston.

Domain and Range of a Function 1999-11-14
From Michelle Mellott:
How do you find the range of the following function:

2y2/(y2+5y+6)??

I know the domain is x not equalled to -2, -3. How can one easily find the range?
Answered by Harley Weston.

Estimating the population mean 1999-11-13
From John Barekman:
Statitistics: Estimating the population mean when the standard deviation is known:

I am not sure which n to use in the formula for the confidence interval equation:

x +/- z*(standard deviation/sqrt(n))

If we have data of ten people, and if we have the data of ten sets of ten people each, what is the difference in the n that we use? What is the difference between the standard deviation and the standard error? Are we using the number of sampling means or just the number of samples?
Answered by Harley Weston.

Parabolic mirrors 1999-11-07
From Andy White:
I am working on a project concerning parabolic mirrors. I need to create a mirror to focus sunlight on a focal point, but I don't know how to do it. Is there some equation that tells where a focal point will be in relation to a parabola?

What is a directrix?
Answered by Penny Nom.

Division/Fractions 1999-11-06
From Azmat Hussain:
Is there a way to explain division by fractions using patterns? For example 3 divided by one half.
Answered by Chris Fisher and Penny Nom.
Inverses of functions 1999-11-01
From Leanne Hickey:
Let f(x) = 2x2 -3x + 2. Find f-1(4) given the fact that f(2) = 4. So the question is finding the inverse of 4, he said it's easier than it looks.
Answered by Penny Nom.
Powers of zero 1999-10-31
From Salina Young:
Hello! I have two questions to ask about the power of "0".

First, what is the value of "0' to the power of "0"? And why?

Second, what is the value of "0" to the power of "2" or '3" etc? And why?

Thank you for your help.
Answered by Walter Whiteley and Harley Weston.

Clockwise or Counterclockwise? 1999-10-27
From Tim:
A particle moves around the circle x2 + y2 = 1 with an x-velocity component dx/dt = y
  1. Find dy/dt

  2. Does the particle travel clockwise or counterclockwise around the circle? Why?

Answered by Harley Weston.
Derivatives with logs 1999-10-26
From Kate:
What is the derivative of 5 to the 5x-2 at x equals 0.8?
Answered by Harley Weston.
Order of operations 1999-10-25
From Garrett:
99*(57+76)*9-1085/9
Answered by Penny Nom.
An odd number of factors 1999-10-22
From Melissa:
What is the common name used for numbers that have an odd number of factors? What is the least positive integer that has exactly 13 factors?
Answered by Penny Nom.
0! + 1! + 2! + 3! + ... + 2000! 1999-10-21
From Melissa:
My name is melissa. I am a 9th grade student I am having trouble finding out how to do this: What is the tens digit of 0! + 1! + 2! + 3! + ... + 2000! I know how to find these, but my calculator cant go any higher than 69! Is there any way i can do this problem?
Answered by Penny Nom.
Piecewise functions. 1999-10-20
From Jenny:
How do you figure out a piecewise function by hand?

e.x.
( 2x (if x is not equal to 0)
f(x)=<
( 0 (if x is not equal to 0)
Answered by Harley Weston.

An equivalent decimal 1999-10-14
From Cheryl Wendlandt:
I have a question in my book that is one and five sixths and you are to put them in an equivalent decimal. I'm sick at home or I would be able to ask my instructor. I would appreciate any help.
Answered by Penny Nom.
(-5)^2, -5^2 and -(5)^2 1999-10-13
From Jennifer Brown:
What is the difference between the following problems:

(-5)2, -52 and -(5)2

Our text book (Beginning Algebra, fourth edition, published by McGraw Hill, by Streeter, Huthison and Hoetzle) says the second and third problem are exactly the same. I don't see how that can be. Is there a mathematical rule that explains this?
Answered by Penny Nom.

A famous mathematician 1999-10-12
From Yvette Perez:
Another way to write 3/15. Remove 0 add a line, unscramble, you have the name of a famous mathematician.
Answered by Claude Tardif.
Isosceles triangles 1999-10-12
From Amber:
In defining the types of triangles, our class was stumped by a question asked by one of the student. Maybe you could help. The definition of an equilateral triangle is a triangle with three congruent sides. The definiton of an isosceles triangle is a triangle with at LEAST two congruent sides. The question is, if an isosceles triangle only requires at Least two of the sides to be congruent, could an equilateral triangle be called an isosceles triangle?
Answered by Penny Nom, Walter Whiteley and Chris Fisher.
Equations with fractions 1999-10-10
From Lori:
I'm a secondary student. How would you figure out a problem such as t-2/t =14/3t - 1/3. The book gave one example but it is really easy. The work YOU have to do is totally different. You don't have to tell me the answer just how to do such problems like these.
Answered by Penny Nom.
Length of a line 1999-10-10
From Dagmara Sarudi:
My question has to do with the length of a diagonal. This problem came up when I thought about the shortest distance between two points, for example walking from one point to another in my neighborhood. I can choose a zig zag route and assuming the blocks I walk are exactly the same length, it shouldn't matter what route I took, the distance I travel should still be the same when I reached my goal. If, on the other hand I could travel in a diagonal line, the distance would be shorter. But what if, in my zig zag motion, the sections get so small the route approaches a diagonal. Shouldn't it be that each separate section added together equals the value of the two original sides? Or would it suddenly equal the value of the diagonal (which, of course was shorter than the two sides added together)?
What gives?


Answered by Chris Fisher and Harley Weston.

Common factors 1999-10-06
From Tommygirl98:
Ok I have a question about common factoring I'm not to sure if you could help me but it's worth a try here it goes. Ok I have a test on Friday and i'm having trouble understanding how to do it here's an example,

Factor,
-10 + 2b how do you do it and what do they mean by factor?
Answered by Penny Nom.

The circumference of a circle 1999-10-05
From Mara Frost:
what is the formula to find the circumference of a circle, or if there is no formula, how do you find the circumference of a circle?
Answered by Penny Nom.
Temperatures 1999-09-27
From Eula:
How do you cahnge farenheit degrees to celsius degrees?
Answered by Penny Nom.
Trigonometry history 1999-09-25
From Nikki:
What is trigonmetry ? Who invented it ? What is it's purpose ? And anything else that you can tell me that is related to Trigometry.
Answered by Chris Fisher.
A fraction problem 1999-09-23
From TruRed:
Seven ninths divided by a negative 3.Could you pleas answer my question and go step by step.
Answered by Penny Nom.
English to french maths words translators 1999-09-22
From Steph Keeley:
Can you foward be any information re English to french maths words translators, as i am having problems finding one.
Answered by Penny Nom.
-8*-(-5a)=15 1999-09-22
From Shelby:
-8*-(-5a)=15

i cannot get this question!! please help me!!
Answered by Harley Weston.

Fractions, percents and decimals 1999-09-19
From Sue Farr:
My son is in 5th grade, and I can't seem to find any papers that have a simple explanation of how to convert decimals into fractions, percents into decimals, and vice-versa.
Answered by Harley Weston.
Repeating decimals 1999-09-18
From Kavoos Bybordi:
I dont know how to change a reccuring decimal to a fraction please could you tell me the method.
Answered by Penny Nom.
Surface area of a cone 1999-09-18
From Frothy:
I don't understand how to find the surface area of a cone. The height is 12cm and the radius is 5cm.
Answered by Walter Whiteley.
The greatest possible difference 1999-09-17
From Denea Hugunin:
Use each of the digits 1 to 5 once to make a 2 digit number and a three digit number that the greatest possible difference.
Answered by Penny Nom.
Stem and Leaf Plot 1999-09-14
From Jeanette Sovick:
My 5th grade son brought home a math paper, the title of which reads, Reading Stem-and-Leaf Plots...can you explain this so I can explain it to him...There is no book, his teacher just sent this practice sheet home for him to complete and I have no clue!
Answered by Penny Nom.
Bales of hay 1999-09-13
From Ivy:
You are given 5 bales of hay. Two bales are weighed at a time, which equal the following weights:
110, 112, 113,114,115,116,117,118,120,121. What does each individual bale weigh?

Answered by Harley Weston.
2,5,8,1 yields 24 1999-09-12
From Terry Hill:
I was wondering how do I make the numbers 2,5,8,and 1 equal 24?
Answered by Penny Nom.
How many went to the game? 1999-09-09
From Lisa Schmidt:
If 3/4 more students went to the game than actually did go to the game, 25% of all students in the entire school would have gone. What fraction of students in the entire school went to the game?
Answered by Harley Weston.
Four fours 1999-09-09
From Roger:
I need help with a math problem my child asked me about I guess in her 7th grade math class they were told to come up with a answer from 1-10 only using four 4's and she got stuck on the problem that needs to be equal to ten, she asked me and I couldn't help her it's sad so if you could give me the problem and and answer so i could explain it to her I would really appreciate it.
Answered by Penny Nom.
Using Factoring in Problem Solving 1999-09-09
From Lori:
I have a problem with a problem. The Problem is....The sum of the squares of two consecutive positive even integers is 340. Find the integers.
Answered by Penny Nom.
Infinity 1999-09-08
From Richard Tracy:
In order to transverse from point A to point B one must first cross the halfway point (C). Additionally....One must also pass another halfway point labeled (D) in order to get to the halfway point of (C). There is also point (E) which is the halfway point between A and D. We have to assume that there are an infinite amount of halfway points points between (A) and (B). My understanding of infinity is something that goes on forever. But how can one expect to traverse over infinity in a finite amount of time? Will we never reach (B)?
Answered by Walter Whiteley.
Towers of Hanoi 1999-09-03
From Jeni:
In the puzzle called the Towers of Hanoi there are three peg and you are asked to move the rings from one peg and stack them in order on another peg. You can make as many moves as you want, but each move must consist of moving exactly one ring. Moreover, no ring may be placed on top of a smaller ring. The minimum number of moves required to move n rings is 1 for 1, 3 for 2 rings, 7 for 3 rings, 15 for 4 rings, and 31 for 5 rings. Find a formula for this sequence. What is the minimum number of moves required to move 6 rings?
Answered by Penny Nom.
The three jugs problem 1999-09-02
From Kent Lane:
I wonder if you could help me figure out this secondary Discrete Math Problem. I cannot figure out where this comes from.

Here's the question:

You have three containers. Container 1 is a three (3) liter container. Container 2 is a five (5) liter container. Container 3 is an eight (8) liter container that is full of liquid. The goal is to get 4 liters in one of the containers.

Stipulations: There are no marks on the containers to measure out the liquid.

All you know is that 1 is 3 l., 2 is 5 l., and 3 is 8 l. full of liquid.
Answered by Chris Fisher.

A Trigonometry Question 1999-08-28
From Diane Simms:
My question is can the following be factored. I am a teacher who needs the factors to this right away. 2 Sin2X + 2 SinX CosX - 1= 0
Answered by Harley Weston.
Invert and multiply 1999-08-28
From Debbie Walter:
I have another question please, WHY do we invert and multiply when dividing fractions? I know that's what we do but WHY? What is the reasoning behind it?
Answered by Penny Nom.
The sum of the cubes is the square of the sum 1999-08-25
From Bernard Yuen:
How to prove 13 + 23 + 33 + 43 + ... n3 is equal to (1+2+3+...n)2? (for n is positive integer)
Answered by Harley Weston.
Why is slope designated m? 1999-08-18
From Peter Komlos:
Why is the slope of a line is designated by the letter "m"?
Answered by Penny Nom.
Parametric Equations 1999-08-06
From Nicholas Lawton:
Show that an equation of the normal to the curve with parametric equations x=ct y=c/t t not equal to 0, at the point (cp, c/p) is :

y-c/p=xp^2-cp^3
Answered by Harley Weston.

From an airport control tower 1999-08-04
From Pammy:
Hi I am a 30 yo mature age student doing my HSC but am having difficulty understanding this, if you can help me.

From an airport control tower, a Cessna bears 023 degrees T and is 27km away. At the same time, a Boeing 767 bears 051 degrees T and is 61km from the tower. Both planes are at the same height.

i) What is the size of angle ATB?

ii) Using the cosine rule to calculate the distance the planes are apart, to nearest kilometre.

I figured out and drew the triangular diagram but can't figure out the rest and which formula to use. sorry about this, thankyou kindly
Answered by Harley Weston.

A difference of squares problem. 1999-07-24
From Michael and Stephanie Bixler:
If you have the equation x= n2 - m2 (ie 40= 72-32= 49-9) x must = a positive number

1) which squared numbers work as n and m
2) how does it work
3) if my teacher gave me the number for x; how could I figure out this problem
Answered by Harley Weston.

A calculus problem 1999-07-22
From Nicholas Lawton:
The curve y= e^x(px^2+qx+r) is such that the tangents at x=1 and x=3 are parallel to the x-axis. the point (0,9) is on the curve. Find the values of p,q and r.
Answered by Harley Weston.
Infinity Symbol 1999-07-13
From Mark E. Kelly:
There is a symbol that looks like a sideways 8 that is used to represent infinity. Does it have a name?
Answered by Doug Farenick and Penny Nom.
Even and Odd Function 1999-06-17
From Kent:
There is one function with the domain of all real numbers that is both even and odd. Please give me the answer to this question before I go insane.
Answered by Penny Nom.
Measuring the fourth dimension 1999-06-07
From Suzanne Hall:
Hi! The other day, two of my students asked a very interesting question: Is the fourth dimension measured with hypercubes? Their reasoning went like this: Lines are 1D and are measured with line segments, which are part of a line. Planes are 2D and are measured with squares, which are part of a plane. Space is 3D and is measured with cubes, which are part of space. So, logically, hypercubes would be used to measure the fourth dimension.
Answered by Chris Fisher.
An Invalid Argument 1999-05-31
From Rod Redding:
Can an invalid argument have a true conclusion? If yes then why?
Answered by Walter Whiteley.
Repeating decimals 1999-05-21
From Stan:
Hi, I am in Honors Math, and have confronted everyone, including teachers, about repeating decimals. What interests me is the number 0.9... and 1. Everyone says that since there is no number between 0.9...(repeating) and 1, that 0.9... = 1. However, isn't a repeating number a representation of a number, and not a real number? Let's look at it this way. 0.9 is close to 1. 0.99 is closer. 0.99999999999999 is even closer. so, 0.9... is a representation of it's closeness to 1. it's an active number... I don't understand how 0.9... is equal to 1. Please help me prove that 0.9... does NOT = 1.
Answered by Penny Nom.
Introductory Algebra 1999-05-19
From Pat:
(2 + sq. root of 3) x (2 - sq. root of 3) = 1

Please show me the work.
Answered by Harley Weston.

Circles, cirmcuference and area 1999-05-16
From Stephen Ehrler:
I would appreciate if you could please tell me if what I discovered here is something or my ignorance? I noticed that a circle with r radii has the folling characteristic.

r = [2 * ( pi * r2 / pi * 2r)]

The equation states that the ratio of a circles area over its circumfrence = 1/2 that of the circles radii. It works every time. Did you know this ? Is it some kind of therom and can it be used for any thing? I thought this was intresting and would appreciate any input you may have.
Thank you.
Answered by Chris Fisher.

Square feet and acres. 1999-05-15
From Helene Freeman:
How can I find out about acres in sq. ft. a house lot is 2360 sq. ft. and the house is on 1432 how many acres are left?
Answered by Penny Nom.
Re-Percentaging 1999-05-15
From Doug Guion:
My mind has gone blank. I have a series of percentages which have been establised by the same base number. I need to remove one of the percentages (13%) leaving me with 87%. I need to arifically inflate the remaining percentages to total 100%. Any help would be appreciated.
Answered by Penny Nom.
Related rates 1999-05-13
From Tammy:
The sides of a rectangle increase in such a way that dz/dt=1 and dx/dt=3*dy/dt. At the instant when x=4 and y=3, what is the value of dx/dt? (there is a picture of a rectangle with sides x and y, and they are connected by z, which cuts the rectangle in half)
Answered by Harley Weston.
Shape-preserving transformations 1999-05-04
From J McAndrew:
A shape preserves its shape if a rotation, translation or scaling is performed on it. Are these the only continuous transformations which have this property? These transformations if performed on the parts and then summed have the same effect as the transformation being applied to the whole; are these linear transformations? Who, and what area of mathematics has classified all transformations of this type completely?
Answered by Chris Fisher.
A 1999-05-02
From Leah:
a=b
a^2=ab
a^2+b^2=ab-b^2
(a-b)(a+b)=b(a-b)
a+b=b
b
2=1

why is this proof wrong?
Answered by Penny Nom.

Parallel and perpendicular lines 1999-04-23
From Crystal Pilling:
My name is Crystal Pilling and I am in 9th grade algebra. We are currently studying parallel and perpendicular lines. I am having trouble with this problem: 3/4x - 5y= 16, (5,-6) I have to find a line that is perpindicular to this line on a graph.
HELP ME PLEASE!!!!!

Answered by Penny Nom.
Radius of an arc 1999-04-22
From Rusty Riddleberger:
I need to find the equation for finding the radius of an arc; I know the length of the arc (i.e the distance of the line connecting the two ends of the arc) and the height; (i.e the rise of the arc at its apex,) I had the formula years ago but it has lost me; this would be invaluable for work in new homes i.e. where we need to build an "arch" with a rise of 21" between two columns 11 feet apart
Answered by Chris Fisher.
Radius of convergence 1999-04-21
From Nowl Stave:
Why is the radius of convergence of the first 6 terms of the power series expansion of x^(1/2) centered at 4 less than 6?
Answered by Harley Weston.
Circles 1999-04-21
From Alex Elkins:
How do you find the circumference of a circle if you only know the radius and the square feet or inches of the circle if the radius is 18 inches, If done in inches do you multiply by 12 to get the square feet?
Answered by Jack Lesage and Harley Weston.
The average rate of change of a function 1999-04-20
From Tammy:
Suppose that the average rate of change of a function f over the interval from x=3 to x=3+h is given by 5e^h-4cos(2h). what is f'(3)? I would appreciate any help with this question.
Answered by Harley Weston.
The halfway mark 1999-04-20
From Sherry & Tom McGee:
How do you figure the halfway mark between 1/3 and 2/3 as a fraction and a decimal point?
Answered by Penny Nom.
Volume of oil in a tank 1999-04-17
From Lars Waldemarsson:
My problem is to get an equation for the depth of the oil in a gastank formed like a cylinder. The cylinder is in a horizontal position and by a stick you will be able to get the depth of the oil in the tank. All I need is an exmaple which I can build on. By this equation you will be able to get the volume of the oil if you know the depth.
Answered by Harley Weston.
y=mx+b 1999-04-15
From Selena Royle:
When solving a problem on slopes,and in the formula y=mx+b what do the M and B stand for? Why M and B?
Answered by Penny Nom.
Volume of a cube. 1999-04-04
From Alex Elkins:
How do you find the volume cubic matter of a cube or space, example how many cubic feet in an example 6" high 4' X 6' rectangle

Thank You
Answered by Penny Nom.

Factoring 1999-03-30
From Maggie Stephens:
I don't know anything about factoring would you plese help me.

3x4 - 48

54x6 + 16y3

125-8x3

12x2 - 36x + 27

9 - 81x2

a3 + b3c3

I would greatly appreciate any help you can give me thanks.
Answered by Jack LeSage.

Standard Form 1999-03-30
From Pam:
How do you put this equation into standard form?

y=3.572x+5.409

Would you please put this equation in this form Ax+Dy+C=0
Answered by Penny Nom.

A Frustum 1999-03-29
From Monica Armour:
What do you call a square pyramid that has had the top chopped off?
Answered by Chris Fisher and Jack LeSage.
Factoring polynomials 1999-03-29
From Karen:
I have been unable to factorise a polynomial equation and was wondering if you could please help. It is level (10-12) maths. The polynomial is x3 + x2 - 24x + 36
I have tried a few factorisation methods such as foctorisation by grouping but it won't work this polynomial. Please help.

Answered by Jeff Walters and Jack LeSage.
Resources for real-world math activities 1999-03-26
From Kate O'Brien:
Where is there a collection of math acitivities or projects to use in high school Algebra I, Algebra II, or Trigonometry that tie concepts to real-world careers?
Answered by Jack LeSage.
Cubic feet and gallons 1999-03-26
From Karen Coheley:
How many gallons in a cubic ft. ??
Answered by Harley Weston.
0/11 1999-03-09
From Jacob:
What is the term applied to 0 divided by a number.

In calculating slope problem my answer was 0 divided by 11. I don't know what to do with this answer or how to interpret it.
Answered by Jack LeSage and Penny Nom.

Factoring 1999-03-08
From L. Sivad:
Question:
m2+6m+9-n2

Answered by Penny Nom.
Reversing the direction of an inequality. 1999-03-06
From Mallory White:
If the Problem was -4a plus -5 is less than or equal to 14, why would you change the sign to greater than or equal to?
Answered by Jack LeSage and Harley Weston.
The square root of two is never supposed to end 1999-03-06
From a wondering student:
i am algebra II and am in the 9th grade. today we were talking about rational and irrational numbers. When we were talking about square roots my friend and i were talking and we thought of something. if you have a square with sides of length one then the diagonal of the square is the square root of 2. Now the square root of two is never supposed to end. But the diagonal of the square ends so therefore doesn't the square root of 2 end. our math teacher did not really answer our question because it was not in the lesson plan and not to many people would see where we were coming from. the answer is really bugging me and i would like to have your input.
Answered by Jack LeSage and Penny Nom.
Finding a rule for a sequence 1999-02-17
From Lindsey Masters:
I'm doing a maths investigation and i have a sequence which goes:-

13,16,25,32,45,56,73.

Our teacher told us we have to find a rule by looking at the differences of the terms until we find a constant. The first differences are:-

3,9,7,13,11,17.

The differences of these are:- ......

Please could you tell me how to work it out so that I could work out the rules of similar sequences.
Answered by Penny Nom.

Four Corners Maths Problem 1999-02-16
From Helen Williams:
I am currently a student teacher in the UK and I have to write a 1000 word report on the following maths problem which I am completely stuck on! PLease HELP!! Choose and 3 by 3 section of the hundred square. Add the total of the four corners. How many different groups of four numbers can you find that add up to that number? eg,
123
111213
212223
Total of 4 corners add up to 48.

Adding 2, 13, 22, 11 also make 48 etc..

How many different groups of 4 numbers would add up to 48?

How would these results compare with thoses obtained from a 3 by 3 square in which the numbers are consective? eg,
123
456
789

PLEASE HELP AS I AM COMPLETELY STUCK? WHY DO ALL THESE DIFFERENT WAYS ADD UP TO THE SAME NUMBER??
Answered by Harley Weston.

Circumference and Area 1999-02-16
From Natalie:
finding the circumference of a circle? formula
finding the area of a parallelogram? formula

Answered by Penny Nom.
Non-Euclidean Geometry 1999-02-10
From Robert Smith:
Is non-euclidean geometry necessary for the college bound student? I have students that are inerested in teaching math one day. My school is restricted to Euclidean Geometry.
Answered by Walter Whiteley and Jack LeSage.
Grade 4 curriculum 1999-02-09
From John:
My daughter is in grade 4 and her math teacher has asked her to write out the number 1 to 10,000. She is currently at 4567 and is sick of it. When I complained that this was not a usefull exercise she said that the Ministry of Education curriculum requires this and gave a photocopy of the page. It states: "read and write whole number to 10,000 in standard, expanded, and written forms (eg., 9367=9000+300+60+7 = nine thousand three hundred sixty-seven). Has the teacher interpreted this the wrong way?

Thanks John
PS. Do you have a web address for Ontario Math and Science Teachers organization?
Answered by Jack LeSage.

Cannonballs 1999-01-27
From Roger King:
How many cannonballs can be stacked in a triangular pyramid?
Answered by Penny Nom.
Simplifying Radicals 1999-01-26
From Mary:
I would like to know how to simplify this question:
   
4 __________________ squareroot7 + squareroot3 
I know the answer is (sqrt7 - sqrt3) but i would really love to know how to get that answer!! Thanks.

Answered by Jack LeSage and Penny Nom.
Simplifying Radicals 1999-01-25
From Randy:
Find the perimeter of a rectangle in simplified radical form L=root80 W=root45. I would appreciate a detailed explination of the process for solving.
Answered by Jack LeSage and Penny Nom.
The Quadratic Formula 1999-01-22
From Eric Morgan:
Hi my name is Eric Morgan and I m in 8th Grade honors math (middle) and I'd like to know how to prove the quadratic formula -b±sqrt of b(a)(c)over 2(a)
Answered by Penny Nom.
Root 17 is Irrational 1999-01-21
From John Murdock:
If you could help me out with this I would appreciate it. Prove that the square root of 17 is irrational.
Answered by Harley Weston.
Fitting a Curve 1999-01-19
From Kirk:
Hello my name is Kirk from Scarborough, Ontario.

I have been out of a formal education system for thirty years. I program microcontrollers in my spare time. I have built a temperature sensing device ready to go but, thermistors are very non-linear. I do know that there is a way to calculate the input condition of the thermistor and display the correct temperature in degrees C. I am sending a file to show my progression so far.
Answered by Harley Weston.

Graphing the Derivative 1999-01-18
From Milena Ghebre:
This question has been nagging me for sometime now.

Is there a way of finding out the derivative of a function, just by looking at the graph of it?
Answered by Walter Whiteley.

Calculus 1999-01-16
From Kaylea Rankin:
Differentiate the following.

y = 1 /(2+3/x)
Answered by Jack LeSage and Penny Nom.

Turning Fractions into Decimals 1999-01-16
From Lynn:
How do you do it step by step
Answered by Jack LeSage.
Levers 1999-01-13
From Lane Campbell:
1. Solve for x or y.

(a)

<-----4m------><------4m-----><---------xm ---------> 
_____________________________________________________
^             ^               ^                     ^
25N          5N               ^                    15N 

Answered by Jack LeSage.
Pythagorean theorem research project 1998-12-31
From Mohammed Hasan:
Hi my name is Mohammed Hasan. I am a math honors student in 8th grade. I have to do a research project in math. The only problem is that I have to do the research project at a 10th grade level. I am having trouble raising the project at a tenth grade level. Would you please kindly take your time to give me some tips and web sites that will help me raise the Pythagorean theorem to a 10th grade level.
Answered by Jack LeSage and Walter Whiteley.
Complex numbers and the quadratic formula 1998-12-25
From Richard Peter:
My age is 16, and my name is Richard. My question relates to the topic complex numbers & the quadratic formula.

I would like to know how to solve quadratic equations in which the discriminant is less than 0 (i.e. we get two complex solutions to the quadratic)

3x2+2x+5 = 0

and how mathematicians like euler contributed to this field. If it would be possible I would also like to know how this type of quadratic equations can be graphed?
Answered by Harley Weston.

Indeterminate forms 1998-12-11
From R. Dixon:
What is the correct evaluation of infinity/0 ? I've checked three different math sites. One says definitively, that infinity/0 is "not" possible. Another states that infinity/0 is one of the indeterminate forms having a large range of different values. The last reasons that infinity/0 "is" equal to infinity.
Answered by Walter Whiteley and Harley Weston.
Platonic Solids Surface Area 1998-12-02
From Rachel Bidwell:
How do you find the surface area of the 5 platonic solids, when they each have a volume of one cubic inch
Answered by Harley Weston.
Duplation method of multiplication 1998-11-26
From Sara Whitford:
I am wondering why 16 and multiples of 16 do not work in the ancient duplation method of multiplication used by the Egyptians. I discovered the method in the Journeys math text gr 7 level. Just curious. Am I doing something wrong??
Answered by Jack LeSage and Harley Weston.
The bricklayers formula 1998-11-24
From Rachel Kaplan:
I have to do a report on the bricklayers formula N = 7LH. Can you give me any information on this.
Answered by Harley Weston.
Fraction division 1998-11-18
From Brett Darrough:
I am needing specific examples that illustrate division of fractions with In-depth understanding. I am enrolled in a CGI (cognitivly guided instruction) course which is trying to develop a method of teaching fraction division with understanding. Most people invert and multiply to obtain a correct answer. Why? I would like help in understanding this method that includes pictures and algorithms that illustrate reasoning and depend on understanding.
Answered by Jack LeSage and Harley Weston.
Linear systems of equations 1998-11-16
From Crystal Girvan:
My name is Crystal Girvan.Im in grade 11. I have a question.We are solving linear systems of equations: comparison method. My questions is. it says

a)use the method of comparison to solve this system a=b-1, 3a+ b=3

b) verify your solutin in a

I dont understand it. please help
Answered by Jack LeSage.

Golf Problem - The Sequel 1998-11-13
From Bob:
Sixteen golfers, seven rounds. What are optimum combinations for each golfer to play with as many different golfers as possible?

Many thanks,
Bob Payson
Answered by Denis Hanson.

What's the next term? 1998-11-12
From Ilia:
What the formula and explonation for formula for next patterns:

1) 1, 4, 10, 20, 35, 56, 84,...

2) 0, 6, 30, 90, 210, 420,...

Thanks!
Answered by Penny Nom.

Triangular Numbers 1998-10-30
From Matt:
i would like to know about triangular numbers and it history i would also like to know about the history of prime numbers thank you
Answered by Chris Fisher.
Fractions 1998-10-27
From Jessica Braimoh:
I have a problem that I just can't figure out!!

Put into a fraction a) -1.157... b) 0.046... c) 0.032... d) -0.3234... Please show how you got the answer and how you came to it(step by step)
Thanks Jessica Braimoh grade 9 Ontario
Answered by Harley Weston.

Adding Fractions 1998-10-15
From Lindsay:
please help me!!!!
Answered by Harley Weston.
Fractions 1998-10-07
From Nikki Lakevold:
-5 over 9 minus -2 over 3 plus -7 over 6 = ???

This is a fraction question and I really don't know why the back of my textbook says -19 over 18 when I got -23 over 18,, am I wrong???
Answered by Harley Weston.

Composite Numbers 1998-10-07
From Greg Murphy:
What type of composite number has an odd number of factors?
Answered by Harley Weston.
Scientific Notation 1998-10-07
From R. Pierce:
Help! How do you express the following numbers in scientific(exponential)notation:0.00036 and 2998000000. I understand how to solve them if it was an equation, but this is confusing me. Please help me answer my questions. Thank you.
Answered by Harley Weston.
A Kite 1998-10-07
From Paul Scott:
What is the mathematical term for the kite shape?
Answered by Walter Whiteley.
A Sum or Product of Fractions 1998-10-06
From Emily Robb:
Find a pair of fractions that when they are multiplyed and added the sum /product is the same
Answered by Penny Nom.
Points on a Graph 1998-10-03
From Nouver Cheung:
If the point P(-3,2) is on the graph of y=f(2x-1), what point must be on y=f(x)-3?
Answered by Harley Weston.
Adding Fractions 1998-10-03
From Pam Bailey:
Can you help me simplfy this?

(1/2a + 1/3b) - (1/4a - 1/5b) + (1/6a - 1/7)

thanx
Answered by Harley Weston.

Students and Lockers 1998-10-02
From Mike:
There is a row of 1000 lockers.
There is a line of 1000 students.

Student number 1 starts at the first locker and opens all 1000. Student number 2 starts at the second locker and closes every other one. Student number 3 starts at the third locker and goes to every third one, closing the open ones and opening the closed ones. Student number 4 does the same with every fourth locker and so on down the line... After all 1000 students have gone how many lockers are open and which ones are they?

Please help! There is proboly a simple solution but we couldnt figure it out for the life of us. Please let us know how you solve it.
Answered by Patrick Maidorn and Penny Nom.

French Francs 1998-10-01
From Richie Franklin:
1.what did one dollar equal in France francs ten years ago, twenty years ago, and fifty years ago?
Answered by Chris Fisher.
Five Factors 1998-09-19
From Derek Yau:
To whom it may concern,

I have difficulty in getting the solution to the following question:

Find 5 numbers that have exactly 5 factors.

I got 16, 81 but couldn't find the rest. I believe that in order to have 5 factors, it has to be a square number. Isn't it true? I guess there may be a pattern to this.

Thanks for your help.
Derek Yau.
Answered by Penny Nom.

Simplifying Roots 1998-09-18
From Lauren Lattimer:
1. 2root27 divided by root 12

2. 3 root 48-root 75
Answered by Penny Nom.

Unit Fractions 1998-09-16
From Murray MacNeil:
Sir, my nephew is having trouble with math,he is in grade 10. this is his question.

Find seven unit fractions whose whole sum is one.

thank you
murray macneil
Answered by Penny Nom.

Radicals 1998-09-15
From Lana Sabo:
Question:
fifteen times the square root of twenty, divided by the square root of 2.

nine subtract the square root of forty-five, divided by 3.

the square root of 18 plus the square root of 12, divided by the square root of 3.
Answered by Harley Weston.

3 to the power of 1994 1998-09-05
From Kim Tangney:
What are the last two digits of:
  1. 3 to the power of 1994

  2. 7 to the power of 1994

  3. 3 to the power of 1994 + 7 to the power of 1994

  4. 7 to the power of 1994 - 3 to the power of 1994

Answered by Penny Nom.
Rational Numbers 1998-09-05
From Kristin Mckenzie:
Iam a secondary student with a math question I don't know how to do.
This is the question:

Explain whether each decimal number represents a rational number.

(a) 0.16

(b) 0.12

(c) -3.125

(d) -0.27

(e) -0,212 112 111 2 ....

(f) 0. 457 92

If it wouldn't be a problem i would really appreciate it if you sent back the instructions on how to do the question.

My name is Kristin Mckenzie and my return e-mail is lexus1999@hotmail.com

Thank-you for you time
Answered by Jack LeSage and Harley Weston.

Volumes 1998-08-29
From Lorraine Wall:
Consider the region in the first quadrant bounded by the x and y axes, the vertical line x=3 and the curve y = 1 / (x squared + 3). Determine the volume of the solid by rotating this region about the x-axis. Now that is the first part.
I then have to find the coordinates of the centroid of the solid by rotating this region about the x-axis.

Thanks.
Lorraine
Answered by Harley Weston.

The area and the circumference of a circle. 1998-08-27
From Jason Wright:
I was looking at the relationship of the area of a circle and the circumference when I realized that 2*pi*r is the derivative of pi*r^2. I was wondering if there is any connective deep dark meaning as to why this appears to be related. Thanks for any help you can give me!
Answered by Walter Whiteley.
Golf problem 1998-08-25
From Bob:
Twelve golfers plan to play seven rounds of golf. Each would like to play as many rounds with different golfers as possible. What are the optimum combinations to accomplish that goal?
Answered by Denis Hanson.
Frieze Patterns 1998-08-19
From Brian Bairstow:
I am doing a research project on frieze patterns (also called band patterns or border patterns). I know that there are exactly seven different types of frieze patterns, but I have been unable to find a proof for this. If you could tell me this proof, or tell me some internet sites on which I can find material on this, I would be very grateful.
Answered by Chris Fisher.
Parabolas 1998-07-24
From Danica:
how do you find the focus, vertex, and directrix of 4x-y^2-2y-33=0
Answered by Penny Nom.
Volumes of Revolution 1998-07-24
From Lorraine Wall:
I'm on the section fpr The Computation of Volumes of Solids of Revolution and the following question is giving me problems:

-Consider the region in the first quadrant bounded by the x-and y-axes, the vertical line x=3, and the curve y=1/(xsquared + 3) I can determine the volume of the solid by rotating the region about the y-axis using the shell method but I can't seem to be able to get started with the volume when rotated about the x-axis.
Answered by Harley Weston.

Solving Quadratics 1998-07-16
From Fred Goodrich:
I need a step by step lesson on solving quadratic equations.

Thank you,
Answered by Penny Nom.

Calculus problems 1998-07-13
From Lorraine:
I'm stuck again. Can you help?

This involves integration using the method of partial fractions

the integral of:
7x(to the 5th) - 2x(cubed) + 3 dx
--------------------------------------
x(to the fourth) - 81

Do I have to do long division to reduce the numerator to the fourth power?

the integral of:
4- 16x +21x(squared) + 6x(cubed) - 3x(fourth) dx
----------------------------------------------------
x(cubed)(x - 2)(squared)

Lorraine
Answered by Harley Weston.

Percentages 1998-07-10
From Adam:
Hi,
I would like to know the basics of percentages. It is all rather confusing to me.I have excelled in math but want to be ready next year for more advanced math.

What is Percentages?
Answered by Penny Nom.

Difference of squares 1998-06-23
From Kristen Smelsky:
Solve the following using a difference of squares:

4x(squared) minus 4xy plus y(squared) minus m(squared) plus 2m minus 1
Answered by Penny Nom.

The Fourth Dimension 1998-05-24
From Whitney Page:
Okay, here goes all my effort to try to explain shat I'm trying to ask of you. It's about something I read in a book called A WRINKLE IN TIME, by Madeline L'Engle. It's called tesser, or tesseract.

It talks about first diminsion, a straight line, second diminsion, a flat square, and third diminsion, a square with sides, front and back, top and bottom.

I can picture all of that. Then it says that fourth diminsion is when you square the three diminsional square. It also described the fourth diminsion as time. I can't figure out how that can be. Then it says...
Answered by Chris Fisher.

Graph question 1998-05-12
From Rose Seminary:
Why is the point of intersection of two lines the solution to the corresponding system of equations?
Answered by Penny Nom.
Multiplying by Nine - Chismbop Style 1998-04-27
From Noria Jones:
About a year ago a grade 5 teacher at my son's school taught the children how to multiply the 9 times table on their fingers quickly.

It was part of a kind of finger math kind of thing...
Answered by Patrick Maidorn.

Using your fingers 1998-04-21
From Ian Sutherland:
My 7-year-old son, in Grade 2, constantly uses his fingers to count.  I know for now this is a useful crutch but at what point should he stop?
Answered by Diane Hanson.
A Car Wash 1998-04-04
From Lisa Gotimer-Strolla:
Al washes a car in six hours.
Fred washes a car in eight hours.
How long will it take them to wash a car together?

Answered by Harley Weston.
The day of the week, July 24, 1837 1998-03-21
From Colleen K.:
What day of the week (Mon. Tues. Wed. etc...) was July 24th, 1837?
Answered by Chris Fisher.
The Largest Factor 1998-03-15
From Senthuran Nadarajah:
When each expression is evaluated for different values of n, the answers will differ. For each expression, find largest natural number that will divide the result for all natural number values of n.

n^5 - 5n^3 + 4n
Answered by Harley Weston.

Pay Phone Problem 1998-02-26
From Shameq:
Hi, I've been given a problem that I'm having some trouble with. I'd really appreciate any help. Here's the question (it's called the Pay Phone Problem)

A pay phone will take only 10p, 20p, 50p, and £1 coins"(It's British).

A woman has plenty of 10p and 20p coins. She has no other coins. She can put the coins into the pay phone in any order.

INVESTIGATE the number of different ways, she could put the 10p and 20p coins into the pay phone.
Answered by Penny Nom.

(50^20)(20^50) 1998-02-24
From fion:
50 power of 20 X 20 power of 50?

How many zero can be found in the answer and why?
Answered by Penny Nom.

A Tightrope Walker. 1998-02-19
From Amy Zitron:
A tightrope is stretched 30 feet above the ground between the Jay and the Tee buildings, which are 50 feet apart. A tightrope walker, walking at a constant rate of 2 feet per second from point A to point B, is illuminated by a spotlight 70 feet above point A....
Answered by Harley Weston.
Intersecting Lines. 1998-02-10
From Erik Heppler:
My typewriter frequently makes mistakes. Suppose I gave you the following system of equations to solve:

35.26X + 14.95Y = 28.35
187.3X + 79.43Y = 83.29

When I looked back, I realized that I meant to use 14.96 as the "Y" coefficient in the first equation instead of 14.95. Solving the system using 14.95 and then solving another system using 14.96 with all other values the same both times results in the intersection points (1776, -4186) and (-770, 1816) respectively.
How can that be?
Answered by Penny nom.

Area of a triangle. 1998-02-01
From Jodi Blucher:
Is there a formula for the area of an equilateral triangle knowing the length of the sides?
Answered by Chris Fisher and Harley Weston.
Divisors of 6n 1998-01-26
From James:
If n is a positive integer such that 2n has 28 positive divisors and 3n has 30 positive divisors, then how many positive divisors does 6n have?
(a) 32 (b) 34 (c) 35 (d) 36 (e) 38

Answered by Penny Nom and Haragauri Gupta.
The Factor Theorem 1998-01-18
From Dhruv Bansal:
Hi, I'm a ninth grade student doing a math project for school. I'm trying to learn about various ways of solving 3rd and 4th degree polynomial equations using the remainder theorem, the factor theorem, and synthetic division. The books I have all mention x - r, which I know nothing about, not even how to get it.

I would really appreciate any information on this. Thanks.

Dhruv Bansal
Answered by Penny Nom.

Polynomials 1998-01-13
From Sarah Storkey:
Hi, My name is Sarah, My question is at the junior level.

My question is, what is a polynomial?
Answered by Harley Weston.

Greatest Common Factor. 1998-01-09
From Percy Stribling:
how do you figure out what the greater common factor is?
Answered by Penny Nom.
Limited area and unlimited perimeter. 1997-11-28
From Rosa:
There is a figure, it has unlimited perimeter but has limited area , what is the figure and how to draw it ?

Thank you very much!
Answered by Harley Weston.

Division by a fraction. 1997-11-21
From Roger Marchegiano:
I am interested in a solution to the following extra credit problem for my son in Geometry. We have been unable to produce a satisfactory response:

Show a geometric representation of the division of fractions algorithm, (When you divide fractions you multiply the numerator by the reciprocal of the denominator).
Answered by Penny Nom.

Fractions 1997-10-20
From Rebecca Henry:
When we add fractions, we find a common denominator and add the numerators When we multiply fractions, we simply multiply both numerators and denominators with no regard to commonality.
  1. Why do we not have to find a common denominator when multiplying?

  2. Why do we multiply both numerators and denominators?

Answered by Chris Fisher.
Cos(x) Cos(2x) Cos(4x)=1/8 1997-09-24
From Tan Wang:
How many distinct acute angles x are there for which cosx cos2x cos4x=1/8?
Answered by Chris Fisher Harley Weston and Haragauri Gupta.
Imperial System of Measurement. 1997-09-17
From Judy Pardilla:
Please also give the solution to the answers. Thank you.
  1. If a room is 12 feet by 15, how many square yards are there?

  2. If a single roll of wallpaper will cover 30 square feet, how many single rolls will it take to cover a wall 8' x 15'?

  3. If ceramic tiles are 8" x 8", how many tiles would it take to cover an area 4' x 6'?
If possible, can you e-mail me a table for imperial measurements. Thank you!

Answered by Penny Nom.
The Range of a Function. 1997-09-12
From Karel Marek:
Math Is Book 6 (Ebos/Tuck) question 5(b) gives a surprising answer at the back of the book..

The question is:
Indicate the domain and range for this: y = sqr(x-9) (Square root of)

The answer for the domain is x>=9 which is not surprisingly, but the range is y>=0, yER ... which I could understand if you were not allowed to transform the equation into x as the subject..

But all example on the facing page DO TRANSFORM the equation at will with x and/or y as the subject..

Can you explain this...
??
Answered by Chris Fisher.

A Finite Math Question. 1997-09-07
From Angela L.:
How many three-digit numbers can be formed using only the numbers 1 to 7 if the number 2 must be included?
Answered by Penny Nom.
A Question About Pi. 1997-07-08
From Mike Chan:
I have read the section Repeating Decimals in your data base. It mention that 1/17 has at most only 16 repeating digits. But, why does "pi" have an infinite number of digits (and not repeating ).
Answered by Penny Nom.
Surface Area 1997-04-30
From Amber Nobile:
The height of a cylinder is twice the diameter. Express the total surface area as a function of the height h.
Answered by Harley Weston.
A proof that e is Irrational. 1997-04-30
From Peter Hall and Jenny:
We have a little mathematical problem... we need some help proving e is an irrational number!

We don't feel very confident in our formulas, so if You have the time to give us a little explanation we would be very grateful!!!
Answered by Doug Farenick and Penny Nom.

Formulae for Surface Area. 1997-04-28
From Gary Millward:
I'm trying to help my son with his Math homework (Grade 10) and he has to find the surface area of a cone and rectangluar pyramid. We have the formulas for the volume of these solids, but can't seem to locate the formulas for surface area.
Answered by Walter Whiteley.
Square Roots and Functions. 1997-04-23
From Ed:
1. In most texts the solution to a question such as square root x = -6 is

x is undefined.

Yet when teaching to solve
xsquared = 36
x = +6 or -6

There appears to be a contradiction here. My question is when, where and why do we use the principle square root, not both + and -? This often occurs as the extraneous root in the solution of radical equations and in stating the domain and range of functions involving square roots.

2. Are there any simple rules for determining whether equations are functions without graphing them and doing a vertical line test?
Answered by Harley Weston.

Proofs 1997-04-13
From Daniel:
I'm having trouble understanding proofs. I don't know how to come up the answers on my own. I search through the book looking for the answer. I understand what they are doing, but I don't know how to do it.
Answered by Walter Whiteley.
The Division Bracket. 1997-04-09
From Judy Riley:
A fellow teacher recently asked if I remembered the exact word for a division bracket (not the symbol with dots, the horizontal line in a fraction, or a solidus). I couldn't. Can you help?
Answered by Walter Whiteley and Harley Weston.
Mathematical Induction and the Derivative 1997-03-18
From Shuling Chong:
"Obtain a formula for the nth derivative of the product of two functions, and prove the formula by induction on n."

Any educated tries are appreciated.
Answered by Penny Nom.

A Presidential Proof 1997-03-18
From Greg Smith:
Which US president developed a proof for the Pythagorean Theorem?

Where can a copy of the proof be located?
Answered by Chris Fisher and Harley Weston.

Ajax, Beverley, Canton and Dilltown 1997-03-14
From S. Johnson:
The following towns are placed on a coordinate system. Ajax at (-x,z), Dilltown at (-10,0), Canton at (0,0) and Beverly at (0,10). The roads from Beverly to Canton and from Canton to Dilltown are perpendiculat to each other and are each 10 miles in length. A car traveling at all times at a constant rate, would take 30 minutes to travel straight from Ajax to Canton, 35 minutes to travel from Ajax to Canton via Beverly, and 40 minutes to travel from Ajax to Canton via Dilltown. What is the constant rate of the car, to the nearest tenth of a mile per hour.
Answered by Chris Fisher and Harley Weston.
0! 1997-03-02
From Donna D. Hall:
I am looking for a quick and easy explanation as to why 0! is 1.
Answered by Walter Whiteley and Denis Hanson.
A Functional Equation. 1997-02-26
From Ronald Lui:
Find all functions f:S --> S statisfying the two conditions:

1)f(x+f(y)+xf(y))=y+f(x)+yf(x) for all x and y in S

2)f(x)/x is strictly increasing on each of the interval -1
Answered by Saroop Kaul and Chris Fisher.

Equivalent Expressions 1997-02-26
From Karyn Jones:
Josie says that s=n/(n+1) and s/(1-s)=n are two ways to write the same formula. Describe whether or not you agree with Joise. Explain how you made your decision.
Answered by Walter Whiteley and Penny Nom.
Division by zero 1997-02-14
From Linda Hood:
I am a college student and have been asked to explain and figure out why we can't divide by zero.
Answered by Chris Fisher.
Deriving the Quadratic Formula 1997-02-04
From James:
How do you derive the quadratic formula? I know what it is, but the textbook doesn't say how to derive it.
Answered by Penny Nom.
Parabolic Mirrors 1997-01-28
From Megan Wennberg:
Consider a ray of light that passes through a chord of a parabola (the chord is above the focus and parallel to the directrix), hits the parabola at a point (x,y) and is reflected through the focus. If d1 is the distance from the chord to the point of incidence (x,y) and d2 is the distance from (x,y) to the focus, can you prove that the sum of the distances d1+d2 is constant, independent of the particular point of incidence.
Answered by Penny Nom.
A donkey and his carrots 1997-01-28
From Emily Lind:
There is a donkey who carries carrots. A farmer has 3,000 carrots to carry to the market. The market is 1,000 miles away. The donkey can only carry 1,000 carrots at a time and he eats 1 carrot every mile that he walks but this is only when he is carrying carrots. How many carrots can the farmer get to market by having the donkey carrying them?
Answered by Penny Nom.
Repeating Decimals 1997-01-24
From Grant Reed:
Is there a way to tell that the repeating decimal for 1/17 has no more than 16 repeating digits?
Answered by Penny Nom.
Foci of an Ellipse 1997-01-22
From David Gilliam:
How do I find the focii of the following equation? 4x^2 + 9y^2 = 36
Answered by Harley Weston.
Could you tell me the name for the bar in a division problem? 1996-10-21
From Linda:
Could you tell me the name for the bar in a division problem. Not the line with dots on either side but the line that divides the two numbers? My name is Linda. I am asking for my niece who is in 8th grade.
Answered by Chris Fisher.
Why is lbs the abbreviation for pounds? 1996-10-03
From Nancy Mutdosch:
What does lbs stand for? Why is it the abbreviation for pounds?
Answered by Chris Fisher.
Divisibility of 2n choose n. 1996-09-24
From Kathy Doan:
Can you prove that "2n choose n" is not divisible by 3, 5, and 7 for infinitely many n?
Answered by Penny Nom.
Term definitions 1996-09-18
From M. Seltzer:
A 7th grade algebra student would like the definition of the terms proper factor and perfect number.
Answered by The Centralizer.
What are fractals and are they of any practical use? 1996-06-26
From Ron:
What are fractals and are they of any practical use?
Answered by Chris Fisher.
An application of Pythagoras' theorem 1996-04-09
From Mike:
We'd like to know what practical applications there may be for the Pythagorean theorem.
Answered by Penny Nom and Maxine Stinka.
Grade nine problem solving 1996-03-11
From Azmat Hussain:
Can math problems be actually fun and engaging?
Answered by Diane Hanson.
Stairs 1996-02-12
From Marianne and Carrie:
How many ways can you run up a flight of stairs?
Answered by Denis Hanson.
Number of factors 1996-01-30
From Chrissy:
What number has exactly 13 positive factors?
Answered by Penny Nom.
Magic Square 1995-10-20
From Marianne and Carrie:
How can an 8 by 8 square have the same area as a 5 by 13 rectangle?
Answered by Denis Hanson.
Deux nombres consécutifs 2010-01-18
From Janot:
Bonjour comment peut on faire pour démontrer que si deux nombres sont consécutifs, leur produit et leur somme sont premiers entre eux Merci de me mettre sur la voie , j'ai échoué avec Bezout.
Answered by Claude Tardif.
Fonctions dérivées 2008-11-28
From Genevois:
Bonjour ! Je dois trouvé la dérivée en 2 de la fonction 40-5x². Or, je sais comment faire et je connais la réponse mais j'ai beau refaire et refaire le calcul, mon résultat n'est jamais le meme que celui du corrigé, j'ai donc un probleme de calcul, pourriez vous donc me donner le calcul détaillé s'il vous plait ? Merci d'avance !
Answered by Pierre-Louis Gagnon.
Un bénéfice de 5% 2008-03-11
From mahiques:
Un propriétaire decide de vendre deux parcelles de terrains pour un montant total de 141750 F. Il fait un bénéfice de 15% sur la première et une perte de 10% sur la deuxième. L'ensemble de la transaction lui a rapporté un bénéfice de 5%.combien a-t-il vendu chacune des parcelles ?
Answered by Pierre-Louis Gagnon.
La somme de deux fonctions 2007-11-19
From maud:
Consigne : Ecrire la fonction f comme somme de deux fonctions u et v définies sur I. Donner le sens de variation de u et de v sur I. En déduire le sens de variation de la fonction f sur l'intervalle I indiqué. f(x)=-2x+(1sur x) I=]0;+infini[

Correction : Sens de variation de f sur I=]0;+infini[ On a f(x)=u(x) + v(x), avec {u(x) = -2x et v(x) = 1sur x La fonction u est strictement décroissante sur R, donc sur I ( droite avec coefficient directeur -2 négatif). La fonction v qui est la fonction inverse est stricyement décroissante sur [0;+infini[. Donc, la fonction f = u+v est strictement décroissante sur [0;+infini[.

Ma question : Pourquoi la fonction v et la fonction f ne sont pas définies sur le même intervalle que la fonction u c'est-à-dire sur l'intervalle I indiqué ?

Answered by Claude Tardif.
Quel est ce nombre 2007-09-30
From fred:
Un groupe de 7 enfants vient de recevoir une grosse quantité de billes. ils font le partage : toutes les billes sont distribuées et ils en ont chacun le même nombre. le nombre total de billes s'écrit avec les mêmes chiffres que 1991. Quel est ce nombre
Answered by Claude Tardif.
Je suis le plus petit entier de 3 chiffres. Qui suis-je? 2007-07-18
From Auerhaan:
De quel chiffre parle-t-on?
Je suis le plus petit entier de 3 chiffres Qui suis-je?
Le cahier de vacances donne comme solution : 120, ce qui me parrais très étrange !) Merci d'avance pour votre réponse..!

Answered by Claude Tardif.
proposition d'une fonction 2007-04-10
From Benoit:
Proposez une fonction f(x,y) dont l'ensemble de niveau c=0 est {(x,y) : x = 0} U {(x,y) : x*2+y*2=1} Je remarque l'équation d'un cercle centré à (0,0) et de rayon 1 ainsi que la droite x=0 mais ça ne clique pas. Merci de m'aider.
Answered by Claude Tardif.
algèbre et fractions 2007-02-05
From marion:
Une certaine somme était inscrite sur un livret de caisse d'épargne au 1er janvier. Le 1er mars, on retire le tiers de ce qui était placé au 1er janvier. Le 1er juin, on retire le quart de ce qui était placé au 1er janvier. Il reste sur ce livret une somme égale à 1/7 des sommes retirées plus 1 600 Quel était le montant du livret d'épargne au 1er janvier?
Answered by Claude Tardif.
Le produit des fractions 2007-01-23
From Jasmine:
Au mois de mars, le poids de la brebis augmente d'un huitième. Au mois d'avril, son poids augmente d'un neuvième, au mois de juin d'un onzième. Par quelle fraction faut-il multiplier son ancien poids pour obtenir son nouveau poids?
Answered by Claude Tardif.
boulier français 2007-01-19
From Stéphanie:
Je prépare une exposition sur le centième anniversaire du vieux couvent de Château-Richer dans la province du Québec qui s'ouvrira en mai 2007. Dans ce couvent les soeurs de la Congréagation du Perpétuel Secours ont enseigné pendant de nombreuses années. Pour mettre en valeur, le site et l'exposition, nous souhaitons mettre les visiteurs à l'épreuve dans l'ambiance d'une classe d'antan (début 20ème siècle) avec, entre autres, les bouliers français que nous possédons et leur faire faire des exercices de mathématiques avec. Malheureusement, nous ne savons pas comment faire les quatre opérations de base (addition, soustraction, multiplication et division) avec. Pouvez-vous nous l'expliquer de façon détaillé, s'il vous plaît?
Answered by Claude Tardif.
Multiplier et soustraire des nombres négatifs 2007-01-11
From Sylvie:
Voici trois nombres entiers: (-5), (-3) et (-4). On retranche le troisieme au produit des deux premiers, ecris la situation et donne le resultat.
Answered by Claude Tardif.
les fraction 2006-11-07
From Coullet:
béreger boit les 3/5 d'une bouteille de 3/4 de litre jennifer boit les 4/5 d'une bouteille de 1/2 litre diego boit les 2/5 d'une bouteille de 1 litre, puis le quart de ce qui lui reste
Answered by Claude Tardif.
Les cinq ligner de chiffre 2006-11-02
From Alex:

comment faire pour trouver les cinq ligner de chiffre apres les avoir observer les sept premiere

1
11
21
1112
3112
211213
312213
-
-
-
-
-


Answered by Claude Tardif.
Avec les chiffres 1,5,6,7, vous devez arriver a un total de 21 2006-09-28
From Danny:
J'ai un problème qui est le suivant: Avec les chiffres 1,5,6,7, vous devez arriver a un total de 21 en les utilisant qu'une seule fois et en utilisant la multiplication, l'addition, la division et la soustraction...
Answered by Claude Tardif.
Dernier théorème de Fermat 2006-06-06
From Jean-Claude:
Bonjour, je suis étudiant au baccalauréat en mathématique et j'aimerais savoir où et comment je peux me procurer la démonstration du dernier théorème de Fermat écrite par Wiles.
Answered by Claude Tardif.
Volume d'un cône 2006-04-24
From Christelle:
Caroline décide de se servir de ses connaissances mathématiques pour "rouler" son petit frère: "Arthur, dit-elle, je te propose que nous mettions chaucun un euro dans l'achat d'un cône glacé. Je mangerai la première, jusqu'à mi-hauteur, et toi, tu auras la seconde moitié." Combien la part de Caroline représenterai-elle par rapport à celle de son petit frère s'il acceptait ?
Answered by Claude Tardif.
exposant négatif 2006-01-23
From Jean:
je suis un grand-père qui veut aider sa petite fille :
niveau 4ème
3 exposant -2 =
-3 exposant -4 =
3 exposant -9 =
Je vous remercie de votre aide.
Corldialement.
Jean

Answered by Claude Tardif.
Une énigme échelle/échafaudage(ou boite) 2005-11-08
From Christian:
J'ai un problème & souhaite demander votre aide pour résoudre une énigme échelle/échafaudage(ou boite)
Answered by Claude Tardif.
problème de trigonométrie 2005-11-01
From Manon:
Soit un triangle ABC, de cotes a , b et c, avec A=45° et B=60°.
Apres avoir demontre que 3a²-2b²=0 (que jai fais en utilisant la regle du sinus) et a²-2c²+2ac=0 (que jai fait en utilisant la regle du cosinus), vous montrerez que laire du triangle ABC peut sexprimer: { [ 3 + racine carre 3 ] / 12 } b² unites.

Je ne sais pas comment arriver a cette expression de laire. Je narrive pas a exprimer c² et ac en fonction de b².. et je pense que ca pourrait vraiment aider..
puisque je suis arrive a letape:
ac[(racine carre 3) / 4]
avec ac = a² + c² - b²
et a² = 2/3 b²
ms c²= 1/3 b² + ac

Answered by Claude Tardif.
Calcul dun profil fictif 2005-09-29
From Mr madry loïc:
je suis actuellement en formation pour devenir dessinateur projeteur en BTP ; je rencontre un problème vis à vis d'une égalité qui normalement devrait mettre acquise ( elle doit être issue de thalès ou de l'égalité des proportion ?) cependant, je n'arrive pas trouver le fondement de cette égalité. pourriez vous m'aider à éclaircir cette relation fondamentale afin de poursuivre mon étude sur le calcul de profil en travers (pour les travaux publics) .
Answered by Claude Tardif.
combien de fruits de chaque sortes? 2005-06-13
From Halloy:
j'ai un panier de 32 fruits au total dans mon panier, j'ai 3 fois plus de bananes que de pommes et 4 fois plus de poires que de pomme. Combien il y a de bananes, de poires et de pommes dans mon panier?
Answered by Claude Tardif.
Avec les chiffres 1 3 4 6... 2005-04-14
From Ambre et Miel:
avec les chiffres 1 3 4 6 utilisé une seule fois chacun soit en div multipl addit ousoustr je dois avoir 24 comme résultat
Answered by Claude Tardif.
6 poules pondent 6 oeufs en 6 jours 2005-03-01
From Nathalie:
SACHANT QUE 6 POULES PONDENT 6 OEUFS EN 6 JOURS, COMBIEN 12 POULES PONDENT D'OEUFS EN 12 JOURS?
Answered by Claude Tardif.
Opérations de base 2003-07-05
From Anne-Marie:
Je m'appelle Anne-Marie et je suis enseignante de 5e année à l'école Domaine des copains de Balmoral au Nouveau-Brunswick. Récemment, j'ai participé à un concours intitulé: Budweiser King Pack Nascar Hat Promo. Afin de réclamer le prix, qui est une casquette, je devais répondre à une question réglementaire. Il s'agissait d'une phrase mathématique qui se présentait comme suit:

4 x 26 - 24 / 8 = qui se lit: 4 fois 26 moins 24 divisé
par 8

Answered by Claude Tardif.
Fractions 2003-05-10
From Jacques:
Voici la question qu'il pose: 2 x (1/2+1/4) + 3 + 2 x (1/3-1/4). La reponse est 4 2/3. Pourriez m'expliquer etapes par etapes ce que je dois faire.
Answered by Claude Tardif et Diane Hanson.
La somme de 51 nombres entiers consécutifs est 1785 2003-04-01
From Katrine et Armand:
La somme de 51 nombres entiers consécutifs est 1785, quels sont ces nombres? (indication : on rappelle que pour tout entier P, on a 1 + 2 + ....+ P = P (P + 1) / 2).
Answered by Claude Tardif et Penny Nom.
Nombre Premier 2003-02-24
From Un Etudiant:
Montrer que si p est facteur premier de 22n + 1 alors (p-1) est multiple de 2n+2
Answered by Claude Tardif.
Un exposant negatif 2002-11-25
From Vinny:
Moi je n'ai aucune idée de ce que vaut un exposant negatif com 3^(-5) ou encore 3^(-4/3)

de meme pour un exposant rationnel 3^(4/3)


Answered by Claude Tardif.
Comment fait on pour justifier que... 2002-09-30
From Lucie:
Comment fait on pour justifier que p au carré est égal à 2 fois q au carré?
Answered by Claude Tardif.
Un problème mathématique 2002-04-23
From Mylène:
En me servant de ces chiffres 1 2 3 4 5 6 je dois les utiliser seulement qu'une fois en faisant l'équation suivante.
? X ?   X  ? __    __    ?   ? X ?   = 2 

Answered by Claude Tardif.
Nombres algebriques sur Q 2002-04-11
From Un eleve:
On sait que le corps des nombres algebriques sur Q est algebriquement clos. A priori partant d'un polynome f a coefficients algebriques sur Q on devrait pouvoir un trouver un polynome g a coefficients rationnels dont une partie des racines est constituee de racines du premier. y-at-il un algorithme qui permet qui permet le passage de f a g ?
Answered by Claude Tardif.
Les quantités d'aliments 2002-03-09
From Benhacine:
soient les aliments A1, A2, A3.....An contenant respectivement le nutriment "a" aux quantités a1, a2, a3, et an sachant qu'un organisme a besoin d'une quantité connue de nutriment "a" égale à "Be",

Ayant: A1.a1 + A2.a2 +.....+ An.an = 100.Be

et

A1 + A2 + A3 +....+ An = 100

Quelles seront les quantités d'aliments A1, A2, et An qui satisferont le besoin défini "Be" de l'organisme?


Answered by Claude Tardif.
A/B = C/D 2002-03-06
From Un eleve:
Démontrer que si A sur B et = à C sur D, alors AxD et = à BxC.
Answered by Claude tardin.
Une autre bille de rayon différent 2002-02-27
From Sarah:
une bille de 6 cm repose au fond d'1 cylindre droit dont la base est un disque de rayon 10 cm. On verse de l'eau dans le cylindre de façon à recouvrir exactement la bille. Démontrez que l'on peut remplacer la bille par une autre bille de rayon différent (mais supérieur) de sorte que l'eau initialement versée recouvre exactement cette nouvelle bille
Answered by Claude Tardif.
Fraction rationnelle 2001-12-11
From Audrey:
réduire en une seule fraction et simplifier.

(4-2x+x2)/(2+x) - (x+2)

Answered by Claude Tardif.
Un nombre entier relatif ou reel 2001-06-13
From Omar:
J'aimerais savoir c'est quoi un nombre entier relatif ou réel?
Answered by Claude Tardif.
coercif 2001-04-17
From Sonia:
J'ai juste une petite question de vocabulaire à vous soumettre: connaitriez vous la traduction en anglais du mot : "coercif", que je n'arrive pas à trouver sur le net!
Answered by Claude Tardif.
preuve par 9 2001-04-04
From Michel:
Pourriez-vous m'expliquer en détail comment fonctionne la preuve par 9 (pour une division et une multiplication). Je sais l'appliquer mais je ne sais pas pourquoi ça marche. Je ne retrouve pas la démonstration. Merci de m'aider. Exemple . 17x2=34 ; preuve par neuf : 1+7=8 ; 8x2= 16 ; 1+6=7 et 3+4=7, on peut donc supposer (sans affirmer) que cette multiplication a un résultat juste car la preuve par 9 est bonne, 7=7. Comment fonctionne cette preuve par 9???
Answered by Claude Tardif.
Combien y aura t il de 9 dans la réponse? 2000-06-17
From Marie:
J'ai lu une énigme mais je n'ai pas la réponse."si l'on divise 123 456 789 par 999 999 999, combien y aura t il de 9 dans la réponse?"
Answered by Claude Tardif.
la dimension fractale 2000-05-01
From François RIVOIRE, Julien CHOLLET et Alexandre DECOSTER:
nous sommes étudiants en première année MPSI en école d'ingénieur ISEN ( Institut supérieur d'électronique du Nord ). Nous avons choisi les FRACTALES ( ou plus précisément sur la dimension fractale et topologique ainsi que sur les suites logistiques) comme sujet de TIPE; serait-il possible que vous nous envoyez des démonstrations mathématiques ou des conseils car jusqu'ici nous avons eût beaucoup de mal à trouver des démonstrations se rapportant à ce sujet,
Answered by Claude Tardif.
Resolution of the equation f(x)=0 2000-04-01
From Naoufal:
How can i evalf the equation f(x)=0 with f function . Can you ansear me in french thank you very much
Answered by Claude Tardif.
La longueur des segments formant un triangle 1999-10-05
From Michel Provencher:
Connaissant la longueur des segments formant un triangle,comment détermine-t-on chacun de ses angles si:
  1. il S'agit d'un triangle rectangle

    Sachant que la somme des angles d'un triangle est de 180 degrés et sachant par conséquant qu'un des angle est de 90 degrés (triangle rectangle) il reste donc, 90 degrés à partager entre les 2 angles restant. Si les 2 segments formant l'angle droit sont de même longueur on obtient un angle de 45 degrés pour les angles restant soit 1/2 angle droit ce qui ne me pose évidement aucun problème. Quel relation, S'il y en a une, y a t-il entre la longueur de ces 2 segments et les angles restants.

  2. il S'agit d'un triangle quelconque

Answered by Claude Tardif.
Application surjective 1999-09-30
From Charass:
je suis étudiant en premiere année de deug et je cherche apres une definition si vous pouviez m'aider, ça serait sympa. qu'est ce qu une application subjective?
Answered by Claude Tardif.
Traduction 1999-09-27
From Beatrice:
Pouvez-vous me traduire en anglais les terms ci-dessous?
  • Intrégration ensembliste
  • Espaces vectoriels normés
  • Traitement statistique du signal
  • Equations aux dérivées partielles
  • ...

Answered by Claude Tardif.
Le salinon d'Archimèdre 1999-03-11
From Don Craig:
I am trying to find the English translation of "Le salinon d'Archimèdre" and would appreciate any help. This is a figure, presumably studied by Archimedes, created from 4 semi-circles. Since I can't draw it for you, I will try to describe it with the help of the 5 collinear, horizontal points below.

. . . . . A B C D E 

A semi-circle is constructed on AE as diameter (let's say above AE).

Two more semi-circles are then constructed with diameters AB and DE on the same side of the line AE as the first semi-circle (above it). Finally, a fourth semi-circle is constructed on diameter BD, this time on the opposite side of the line AE from the others (i.e. below the line).

These semi-circles and the region enclosed by them constitute what is called in French "Le salinon d'Archimèdre".

If you know the English name of this curve I would appreciate it if you let me know.
Answered by Harley Weston.

les fractions 1998-02-24
From Colette Huguenin:
Bonjour je révise mes math de tout les secondaire et le livre louer la bibliothèque n'est pas entier voici le genre de problêmes qui me bloque

(5 4/5+1/2)divisé 1/3

je sais que je dois commencer par la parenthèse mais...je fait quoi comme opération avec le 5? dois-je le multiplier ou l'additionner ou.......????? si je pouvais seulement avoir la base des fractions je redébloquerais surement
Answered by Diane Hanson.

 
Page
1/1

 

 


Math Central is supported by the University of Regina and The Pacific Institute for the Mathematical Sciences.

CMS
.

 

Home Resource Room Home Resource Room Quandaries and Queries Mathematics with a Human Face About Math Central Problem of the Month Math Beyond School Outreach Activities Teacher's Bulletin Board Canadian Mathematical Society University of Regina PIMS